Sie sind auf Seite 1von 800

11

YEAR

MATHS
Quest

MATHS B
FOR QUEENSLAND
D EDITION
SECON
YEAR

MATHS
Quest 11
MATHS B
FOR QUEENSLAND

N D E DITION
S ECO

Nick Simpson
Robert Rowland
CONTRIBUTING AUTHOR
Catherine Smith
Second edition published 2009 by
John Wiley & Sons Australia, Ltd
42 McDougall Street, Milton, Qld 4064

First edition published 2001

Typeset in 10.5/12 pt Times

© John Wiley & Sons Australia, Ltd 2001, 2009

The moral rights of the authors have been asserted.

National Library of Australia


Cataloguing-in-Publication data

Author: Simpson, N. P. (Nicholas Patrick), 1957–

Title: Maths Quest : Maths B year 11 for


Queensland / Nick Simpson; Rob Rowland.

Edition: 2nd ed.

ISBN: 978 0 7314 0810 8 (pbk. : student edition)


978 0 7314 0864 1 (web : student edition)

Notes: Includes index.

Target
Audience: For Queensland Year 11 students.

Subjects: Mathematics — Textbooks.


Mathematics — Study and teaching
(Secondary) Queensland.
Mathematics — Problems, exercises, etc.

Other Authors/
Contributors: Rowland, Robert, 1963–

Dewey Number: 510.712

Reproduction and communication for educational purposes


The Australian Copyright Act 1968 (the Act) allows a
maximum of one chapter or 10% of the pages of this work,
whichever is the greater, to be reproduced and/or
communicated by any educational institution for its
educational purposes provided that the educational institution
(or the body that administers it) has given a remuneration
notice to Copyright Agency Limited (CAL).

Reproduction and communication for other purposes


Except as permitted under the Act (for example, a fair dealing
for the purposes of study, research, criticism or review), no part
of this book may be reproduced, stored in a retrieval system,
communicated or transmitted in any form or by any means
without prior written permission. All inquiries should be made
to the publisher.

Cover and internal design images: © Digital Vision

Illustrated by the Wiley Art Studio

Printed in China by
Printplus Limited
10 9 8 7 6 5 4 3 2 1
Contents
Introduction ix Functions and modelling 99
About eBookPLUS xi Exercise 2G 100
Summary 102
Acknowledgements xii
Chapter review 104

CHAPTER 1
Modelling using linear
CHAPTER 3
functions 1 Other graphs and
Introduction 2 modelling 111
Solving linear equations 3 Introduction 112
Exercise 1A 7 Transforming graphs 113
Rearrangement and substitution 8 Exercise 3A 116
Exercise 1B 12 Sketching graphs using transformations 117
Gradient of a straight line 15 Investigation — Investigating transformations
Exercise 1C 18 on the basic graphs of y = x2, y = x3 and
Career profile: Rick Morris — Vigneron 23 y = x4 118
Equations of the form y = mx + c 24 Exercise 3B 122
Exercise 1D 26 Sketching graphs using intercepts 123
Sketching linear graphs using Exercise 3C 131
intercepts 29 The hyperbola 133
Exercise 1E 31 Exercise 3D 136
Simultaneous equations 33 The square root function 138
Exercise 1F 40 Exercise 3E 140
Formula for finding the equation of a straight The absolute value function 142
line 41 Exercise 3F 146
Exercise 1G 45 Addition of ordinates 147
Linear modelling 47 Exercise 3G 152
Exercise 1H 50 Modelling 153
Summary 52 Exercise 3H 158
Chapter review 53 Modelling using a graphics calculator 161
Exercise 3I 165
Investigation — Goal accuracy 166
CHAPTER 2 Summary 167
Relations and functions 59 Chapter review 170
Introduction 60
Relations and graphs 62
Exercise 2A 67 CHAPTER 4
Domain and range 69 Triangle trigonometry 175
Exercise 2B 75 Introduction 176
Investigation — Interesting relations 77 Calculating trigonometric ratios 176
Types of relations (including functions) 77 Investigation — Looking at the tangent
Exercise 2C 80 ratio 176
Function notation 82 Investigation — Looking at the sine ratio 180
Special types of functions 84 Investigation — Looking at the cosine
Exercise 2D 88 ratio 181
Inverse relations and functions 90 Exercise 4A 184
Exercise 2E 91 Finding an unknown side 185
Investigation — A special relation 92 Exercise 4B 189
Circles 93 Finding angles 192
Exercise 2F 97 Exercise 4C 196
vi
Applications of right-angled triangles 199 Further trigonometric equations 286
Exercise 4D 203 Exercise 6C 288
Investigation — Fly like a bird 204 Investigation — Fishing 288
Using the sine rule to find side lengths 205 Identities 288
Investigation — Derivation of the sine Exercise 6D 292
rule 205 Investigation — Further trigonometric
Exercise 4E 209 identities 294
Investigation — Bearing east and west 211 Using the Pythagorean identity 294
Using the sine rule to find angle sizes 212 Exercise 6E 295
Exercise 4F 213 Summary 296
Using the cosine rule to find side lengths 215 Chapter review 297
Exercise 4G 218
Using the cosine rule to find angle sizes 220 CHAPTER 7
Exercise 4H 223
Summary 225
Exponential and logarithmic
Chapter review 226 functions 299
Introduction 300
Index laws 300
CHAPTER 5 Exercise 7A 305
Graphing periodic Negative and rational powers 307
functions 231 Exercise 7B 309
Introduction 232 Indicial equations 311
Period and amplitude of a periodic Exercise 7C 314
function 232 Investigation — Simulating radioactivity 315
Investigation — Temperature and tide 233 Graphs of exponential functions 317
Exercise 5A 236 Exercise 7D 320
Investigation — Rhythm of life 238 Investigation — A world population
Investigation — Ferris wheeling 238 model 321
Radian measure 238 Investigation — Bode’s Law 324
Investigation — Finding a radian 238 Logarithms 324
Exercise 5B 240 Exercise 7E 327
Exact values 241 Solving logarithmic equations 329
Exercise 5C 248 Exercise 7F 331
Symmetry 249 Investigation — Logarithmic graphs 332
Exercise 5D 253 Investigation — The slide rule 333
Trigonometric graphs 254 Applications of exponential and logarithmic
Investigation — The effect of 2 259 functions 335
Exercise 5E 264 Exercise 7G 337
Investigation — How high? 267 Investigation — The decibel 339
Applications 268 Investigation — The Richter scale 340
Exercise 5F 270 Summary 341
Investigation — Sunrise to sunset 271 Chapter review 343
Summary 272
Chapter review 274 CHAPTER 8
Applications of exponential
CHAPTER 6 and logarithmic functions in
Trigonometric equations 277 financial mathematics 347
Introduction 278 Introduction 348
Simple trigonometric equations 278 Geometric sequences 348
Exercise 6A 282 Exercise 8A 353
Equations using radians 283 Geometric series 354
Exercise 6B 285 Exercise 8B 360
vii
Investigation — Crossing the road 361 Cumulative data 464
Introduction to growth and decay 362 Investigation — A different display 467
Growth and decay functions 362 Exercise 9G 468
Exercise 8C 373 Summary 473
Compound interest formula 379 Chapter review 475
Exercise 8D 384
Reducing balance loans 389
Loan schedules 390
CHAPTER 10
Exercise 8E 395 Summary statistics 479
Investigation — Loan schedules using Introduction 480
spreadsheets 400 Measures of central tendency 481
Investigation — Spreadsheets and investing Exercise 10A 488
for the future 401 Investigation — Mean and median amount of
The annuities formula 402 soft drink 493
Exercise 8F 411 Range and interquartile range 494
Investigation — Buying a home 417 Exercise 10B 499
Summary 418 Investigation — Range of soft drink
Chapter review 420 amounts 501
The standard deviation 502
Exercise 10C 505
CHAPTER 9 Investigation — Standard deviation of soft
Presentation of data 425 drink amounts 507
Introduction 426 Boxplots 507
Types of variables and data 426 Exercise 10D 512
Investigation — Types of data 428 Back-to-back stem plots 515
Exercise 9A 429 Exercise 10E 518
Collection of data 431 Parallel boxplots 520
Investigation — Gallup poll 431 Exercise 10F 522
Investigation — Identifying the target Summary 524
population 431 Chapter review 526
Exercise 9B 436
Investigation — Census or sample? 438
Bias 439 CHAPTER 11
Investigation — Bias in statistics 440 Introduction to
Investigation — Biased sampling 440 probability 533
Investigation — Spreadsheets creating Introduction 534
misleading graphs 441 Informal description of chance 534
Investigation — Cost of a house 442 Exercise 11A 537
Exercise 9C 443 Investigation — What will the weather be? 539
Investigation — Bias 444 Single event probability 539
Displaying data 445 Exercise 11B 542
Stem plots 445 Investigation — Comparing theoretical
Exercise 9D 448 probabilities with experimental results 545
Frequency histograms and bar charts 451 Investigation — Experimental or
Exercise 9E 456 theoretical? 546
Investigation — Segmented bar chart 457 Relative frequency 546
Investigation — Looking at cost 458 Exercise 11C 548
Investigation — Using a database 458 Investigation — Researching relative
Describing the shape of stem plots and frequencies 551
histograms 459 Investigation — Applying relative
Exercise 9F 461 frequency 551
viii
Modelling probability 551
Exercise 11D 557
CHAPTER 13
Investigation — Random choice 558 Differentiation and
Long-run proportion 558 applications 623
Exercise 11E 562 Introduction 624
Investigation — Footy season 564 The limit 625
Summary 565 Exercise 13A 628
Chapter review 566 Investigation — Sneaking up on a limit 630
Limits of discontinuous, rational and hybrid
functions 630
CHAPTER 12 Investigation — Dirichlet’s function 634
Rates of change 569 Exercise 13B 634
Introduction 570 Differentiation using first principles 636
Investigation — Secants and tangents 637
Constant rates 570
Exercise 12A 572 Exercise 13C 640
Variable rates 575 Finding derivatives by rule 641
Exercise 12B 576 Exercise 13D 645
Investigation — Graphs of derivatives 647
Average rates of change 578
Exercise 12C 580 Rates of change 649
Instantaneous rates 584 Exercise 13E 652
Exercise 12D 587 Solving maximum and minimum
Motion graphs 590 problems 656
Exercise 12E 593 Exercise 13F 659
Investigation — When is a maximum not a
Relating the gradient function to the original
maximum? 661
function 598
Exercise 12F 603 Summary 662
Chapter review 663
Relating velocity–time graphs to
position–time graphs 604
Exercise 12G 606 Appendix 667
Rates of change of polynomials 609
Investigation — Investigating rates of Answers 729
change 611
Index 783
Exercise 12H 612
Career profile: Roger Farrer — Data
Manager 614
Summary 615
Chapter review 617
Introduction
Maths Quest Maths B Year 11 for Queensland 2nd edition is one of the
exciting Maths Quest resources specifically designed for the Queensland
senior mathematics syllabuses beginning in 2009. It has been written and
compiled by practising Queensland Maths B teachers. It breaks new ground
in mathematics textbook publishing.
This resource contains:
• a student textbook with accompanying student website (eBookPLUS)
• a teacher edition with accompanying teacher website (eGuidePLUS)
• a solutions manual containing fully worked solutions to all questions
contained in the student textbook.

Student textbook
Full colour is used throughout to produce clearer graphs and headings, to
provide bright, stimulating photos and to make navigation through the text
easier.
Clear, concise theory sections contain worked examples, graphics calculator
tips and highlighted important text and remember boxes.
Worked examples in a Think/Write format provide clear explanation of key
steps and suggest how solutions can be presented.
Exercises contain many carefully graded skills and application problems,
including multiple-choice questions. Cross-references to relevant worked
examples appear beside the first ‘matching’ question throughout the
exercises.
Investigations, often suggesting the use of technology, provide further dis-
covery learning opportunities.
Each chapter concludes with a summary and chapter review exercise contain-
ing questions that help consolidate students’ learning of new concepts.
As part of the chapter review, there is also a Modelling and problem solving
section. This provides students with further opportunities to practise their
skills.
Technology is fully integrated within the resource. To support the use of
graphics calculators, instructions for two models of calculator are presented
in worked examples and graphics calculator tips throughout the text. The
two models of graphics calculator featured are the Casio fx-9860G AU and
the TI-Nspire CAS. (Note that the screen shots shown in this text for the
TI-Nspire CAS calculator were produced using OS1.4. Screen displays may
vary depending on the operating system in use.)
For those students using the TI-89 model of graphics calculator, an appendix
containing matching instructions has been included at the back of the book.
The Maths Quest for Queensland series also features the use of spreadsheets
with supporting Excel files supplied on the student website. Demonstration
versions of several graphing packages and geometry software can also be
downloaded via the student website.
x
Student website — eBookPLUS
The accompanying student website contains an electronic version of the
entire student textbook plus the following additional learning resources:
WorkSHEETs — editable Word 97 documents that may be completed on
screen, or printed and completed later.
SkillSHEETs — printable pages that contain additional examples and prob-
lems designed to help students revise required concepts.
Test Yourself activities — multiple-choice quizzes for students to test their
skills after completing each chapter.

Programs included
Graphmatica: an excellent graphing utility
Equation grapher and regression analyser: like a graphics calculator for PCs
GrafEq: graphs any relation, including complicated inequalities
Poly: for visualising 3D polyhedra and their nets

Teacher edition
The teacher edition textbook contains everything in the student textbook and
more. To support teachers assisting students in the class, answers appear in
red next to most questions in the exercises and investigations. Each chapter is
annotated with relevant syllabus information.

Teacher website — eGuidePLUS


The accompanying teacher website contains everything in the student web-
site plus the following resources:
• two tests per chapter (with fully worked solutions)
• fully worked solutions to WorkSHEETs
• a syllabus planning document
• assessment tasks (and answers)
• fully worked solutions to all questions in the student textbook.

Solutions manual
Maths Quest Maths B Year 11 for Queensland Solutions Manual contains the
fully worked solutions to every question and investigation in the Maths Quest
Maths B Year 11 for Queensland 2nd edition student textbook.
Fully worked solutions are available for all titles in the Maths Quest for
Queensland senior series.

Maths Quest is a rich collection of teaching and learning resources within


one package.
Next generation teaching and learning

About eBookPLUS
Maths Quest Maths B Year 11 for Queensland 2nd edition features eBookPLUS: an electronic version of the
entire textbook and supporting multimedia resources. It is available for you online at the JacarandaPLUS
website (www.jacplus.com.au).
Using the JacarandaPLUS website Step 1. Create a user account
To access your eBookPLUS resources, simply log on The first time you use the JacarandaPLUS system,
to www.jacplus.com.au. There are three easy steps for you will need to create a user account. Go to the
using the JacarandaPLUS system. JacarandaPLUS home page (www.jacplus.com.au)
and follow the instructions on screen.

Step 2. Enter your registration code


Once you have created a new
account and logged in, you will
be prompted to enter your unique
registration code for this book,
which is printed on the inside
front cover of your textbook.

LOGIN
Once you have created your account,
you can use the same email address and
password in the future to register any
JacarandaPLUS books.

Step 3. View or download eBookPLUS resources


Your eBook and supporting resources are provided
in a chapter-by-chapter format. Simply select the
desired chapter from the drop-down list and navigate
through the tabs to locate the appropriate resource.

Minimum requirements Troubleshooting


• Internet Explorer 7, Mozilla Firefox 1.5 • Go to the JacarandaPLUS help page at
or Safari 1.3 www.jacplus.com.au
• Adobe Flash Player 9 • Contact John Wiley & Sons Australia, Ltd.
• Javascript must be enabled (most browsers Email: support@jacplus.com.au
are enabled by default). Phone: 1800 JAC PLUS (1800 522 7587)
Acknowledgements
The authors and publisher would like to thank the following copyright holders,
organisations and individuals for their assistance and for permission to reproduce
copyright material in this book.

Illustrative material
Screenshots
• TI–Nspire CAS and TI-89 screenshots reproduced with permission of Texas
Instruments; • Casio fx-9860G AU screenshots reproduced with permission of
Casio.

Images
• © Banana Stock, pp. 476, 521; • © Blend Images, p. 398; • © Brand X Pictures,
p. 438; • © Corbis Royalty Free, pp. 22 (bottom), 51 (right), 60 (top), 101
(bottom right), 175, 198 (top), 214, 228 (bottom), 276 (bottom, top), 300
(bottom), 321, 413, 439, 457 (top), 468, 530 (bottom), 532, 622 (top); •
© Creatas Images, p. 338 (bottom); • © Digital Stock, pp. 1, 15 (left), 28, 51
(left), 61 (top), 100, 202, 231, 300 (top), 447, 471, 566; • © Digital Vision, pp.
57, 60 (bottom), 101 (centre), 109 (top), 338 (top), 348 (bottom, top), 399, 413
(left), 417 (bottom), 477, 479, 480, 482, 514, 608, 623, 649, 653 (bottom); •
© EyeWire Images, pp. 198 (bottom), 235, 621; • © Fancy Images, p. 460; •
© Image Disk Photography, pp. 271 (bottom), 583; • © Image 100, pp. 219, 415 •
© John Wiley & Sons Australia / Belinda Rose, p. 237; / Kari-Ann Tapp, pp. 412,
445, 557; / Jo Patterson, p. 614; / Malcolm Cross, p. 654; • © Newspix / Michael
Perini, p. 40; / Wayne Ludbey, p. 437; / David Kapernick, p. 564; / Marc
Robertson, p. 625; • © Photodisc, pp. 14, 15 (right), 22 (centre), 50, 59, 61
(bottom), 62, 64, 98, 109 (bottom), 110, 174 (bottom right, centre), 175, 191,
210, 211, 224, 227, 228 (top), 229, 230, 232, 233, 238, 241, 254, 267, 271 (top),
277, 298 (bottom), 315, 316, 324, 333, 336, 337, 340, 346, 347, 367, 389, 396,
416, 417 (centre), 423, 429, 435, 444, 457 (bottom), 465, 470 (bottom, centre),
472, 478, 481, 489, 490, 492, 493, 501, 515, 519, 522, 523 (bottom, top), 529,
530 (top), 531, 533, 534, 542, 550, 551, 567, 568 (bottom, centre), 569, 572, 589
(bottom, top), 594, 598, 613 (bottom, top), 622 (bottom), 650, 651, 652, 653
(top), 659, 661; • © Photolibrary / Science Photo Library / Moredun Animal
Health Ltd., p. 299; • © PhotoAlto, p. 425; / Patrick Sheándell O’Carroll, p. 417
(top); • © PhotoEssentials, p. 269; • © Purestock, p. 111, 161, 166, 270, 649
(left); • © Rick Morris, p. 23; • © Shutterstock, p. 298 (top); • © Stockbyte,
pp. 361, 395, 496, 543; • © Viewfinder Australia Photo Library, pp. 204, 430.
xiii
Software
The authors and publisher would like to thank the following software providers
for their assistance and for permission to use their materials. However, the use of
such material does not imply that the providers endorse this product in any way.
Third party software — registered full version ordering information
Full versions of third party software may be obtained by contacting the
companies listed below.
GraphEq and Poly
Evaluation copies of GraphEq™ and Poly™ have been included with permission
from Pedagoguery Software, Inc.
email: peda@peda.com
Web: www.peda.com
Graphmatica
Reproduced with permission of kSoft, Inc.
345 Montecillo Dr., Walnut Creek, CA 94595-2654.
email: ksoft@graphmatica.com
Web: www.graphmatica.com
Software included is for evaluation purposes only. The user is expected to
register share-ware if use exceeds 30 days. Order forms are available at
www.graphmatica.com/register.txt
Equation Grapher with Regression Analyser
Reproduced with permission of MFSoft International.
email: info@mfsoft.com
Web: www.mfsoft.com
Microsoft® Excel
Screenshots reproduced by permission of Microsoft Corporation.
Note: Microsoft Software has been used only in screen dumps.
Microsoft Excel is a registered trademark of the Microsoft Corporation in the
United States and/or other countries.

Every effort has been made to trace the ownership of copyright material.
Information that will enable the publisher to trace the copyright holders or to
rectify any error or omission in subsequent reprints will be welcome. In such
cases, please contact the Permission Section of John Wiley & Sons Australia,
who will arrange for the payment of the usual fee.
Modelling
using linear
functions
1
syllabus reference
Topic:
• Introduction to functions

In this chapter
1A Solving linear equations
1B Rearrangement and
substitution
1C Gradient of a straight line
1D Equations of the form
y = mx + c
1E Sketching linear graphs
using intercepts
1F Simultaneous equations
1G Formula for finding the
equation of a straight line
1H Linear modelling
2 M a t h s Q u e s t M a t h s B Ye a r 1 1 f o r Q u e e n s l a n d

Introduction
The Garn Express crisis

Consider the following scenario: an emergency on the interstate rail system.


There has been a communications failure and the Garn Express, travelling south
from Cannes, is running on the wrong track. The train is being tracked by satellite as it
travels towards the main city of Brampton and its position at various times is known.
These positions are recorded in the table below.

Cannes
Time elapsed (minutes) Kilometres from Brampton

300 km 0 550
15 525
Alberton 34 497

250 km 40 488
45 480
Brampton

The Rail Authority needs to know when the train is likely to reach Alberton, 250 km
from Brampton. Once at Alberton the driver can be alerted and the train can be moved
to the correct track.
One approach to solving 600
the problem of when the 500
Brampton (km)
Distance from

train will reach Alberton is 400


to plot the data — position 300
versus time.
200
A linear model can then
100
be used to predict the
position of the Garn Express 0
0 50 100 150 200 250
in the future. A first Time (min)
approximation of this linear model is a straight line drawn through the data points.
Reading from the graph
we can predict that the Garn 600
Express will reach Alberton 500
Brampton (km)
Distance from

after about 190 minutes. 400


A linear model can also 300
provide a description of 200
the relationship between 100
variables in another way. 0
We can develop an equation 0 50 100 150 200 250
Time (min)
that relates distance and time.
In the following sections we will review and consolidate the concept of a linear
equation.
Chapter 1 Modelling using linear functions 3
Solving linear equations
Throughout the Maths B course, we will often need to call on our ability to rearrange
or solve equations.
A linear equation is one that involves a pronumeral raised to the power of 1 only.
Recall that x1 is the same as x, so a linear equation involving x would contain only x’s,
1---
and not x 2 , x2, x3, x−1, x−2 and so on.
For example, y = 7x − 3, ax + by = c and --83- x + 1 = 9 are all linear equations.
1 1
Note that y = --- is not a linear equation, as --- is really x−1. To isolate a particular
x x
pronumeral — known as ‘making x (or whatever the pronumeral is) the subject’ — we
focus on the pronumeral by ‘undoing’ other terms and operations. In doing so,
remember to do the same to both sides of an equation, in the reverse order to that
originally used to make up the equation.
Although the focus of this chapter is linear equations, in this section some other
types of equation will be included for skills practice.
When there is only one pronumeral involved in an equation, we may attempt to
solve and find a numerical value by rearranging to make the pronumeral the subject.

WORKED Example 1
Solve the following linear equations.
3x – 2 4x
a 7x − 4 = 17 b --------------- + 5 = 1 c 2  ------ – 1 = 6
4 5 
THINK WRITE
a 1 Write the equation. a 7x − 4 = 17
2 Add 4 to both sides. 7x = 21
21
3 Divide both sides by 7. x = ------
7
x=3
b 3x – 2
1 Write the equation. b --------------- + 5 = 1
4
2 Subtract 5 from both sides. 3x – 2
--------------- = −4
4
3 Multiply both sides by 4. 3x − 2 = −16
4 Add 2 to both sides. 3x = −14
Divide both sides by 3. 14
5 x = – ------
3
4x
c 1 Write the equation. c 2  ------ – 1 = 6
5 
Divide both sides by 2. 4x
2 ------ – 1 = 3
5
Add 1 to both sides. 4x
3 ------ = 4
5
4 Multiply both sides by 5. 4x = 20
Divide both sides by 4. 20
5 x = ------
4
x=5
4 M a t h s Q u e s t M a t h s B Ye a r 1 1 f o r Q u e e n s l a n d

When an equation has pronumerals (for example, x’s) on both sides, at some stage
they must be gathered together on the same side of the equation.

WORKED Example 2
Solve each of the following.
9x + 3 13 x + 7 3–x 9( x + 7)
a 4x − 3 = 3(6 − x) b --------------- = ------------------ c ------------ = -------------------- + 1
2 3 4 10
THINK WRITE
a 1 Write the equation. a 4x − 3 = 3(6 − x)
2 Expand the right-hand side (RHS). 4x − 3 = 18 − 3x
3 Collect x’s on one side, for example, the side which 7x − 3 = 18
results in a positive x term; in this case, the left-hand
side (LHS). (That is, add 3x to both sides.)
4 Add 3 to both sides. 7x = 21
21
5 Divide both sides by 7. x = ------
7
x=3
b 1 Write the equation. 9x + 3 13x + 7
b --------------- = ------------------
2 3
2 Find the lowest common denominator for all three
terms. Here, we use 6.
3 Write all terms with the common denominator,
adjusting numerators accordingly (so that 3 ( 9x + 3 ) 2 ( 13x + 7 )
----------------------- = --------------------------
numerator and denominator have been multiplied 6 6
by the same amount).
4 Now that all terms have the same denominator, the
3(9x + 3) = 2(13x + 7)
numerators must be equal. (Multiply each term by 6.)
5 Expand all brackets. 27x + 9 = 26x + 14
6 Collect x’s on the LHS and numbers on the RHS. 27x − 26x = 14 − 9
7 Simplify and solve. x=5
3 – x 9( x + 7)
c 1 Write the equation. c ----------- = -------------------- + 1
4 10
2 Find the lowest common denominator for all three
terms. Here, we use 20.
5 ( 3 – x ) 2 × 9 ( x + 7 ) 20
3 Write all terms with the common denominator, -------------------- = ----------------------------- + ------
adjusting numerators accordingly (so that 20 20 20
numerator and denominator have been multiplied
by the same amount).
4 Now that all terms have the same denominator, the 5(3 − x) = 18(x + 7) + 20
numerators must be equal. (Multiply each term by 20.)
5 Expand all brackets. 15 − 5x = 18x + 126 + 20
6 Collect x’s on the RHS and numbers on the LHS. 15 − 126 − 20 = 18x + 5x
−131 = 23x
131
Simplify and solve. − --------- = x
7 23
131
x = – ---------
23
Chapter 1 Modelling using linear functions 5

Graphics Calculator tip! Solving equations


A graphics calculator can be used to solve an equation. To find the solution to the linear
x
equation 3(x − 2) + 4 = --- + 1, follow the steps below.
3
For the Casio fx-9860G AU
1. Press MENU and then use the arrow keys to
highlight EQUA.

<Casio01.01>

2. Press EXEto select Equation from the MAIN


MENU. Three options are displayed.

<Casio01.02>

3. Press F3 (SOLV) to select Solver. There may be


an equation already shown in the first line from a
previous problem. You can delete this or type over
it with the new equation. <Casio01.03>
Enter the given equation. (Remember to press
X,q,T to enter x and press SHIFT then the .
key to enter =.) Press EXE . You will be given a
starting value for x and a lower and upper limit.

4. Press F6 (SOLV). The solution is shown as


X = 1.125. The third line shows Lft = 1.375. This
is the value when x is substituted into the left side
of the equation. Similarly, the fourth line shows <Casio01.04>
Rgt = 1.375. This is the value when x is
substituted into the right side of the equation. You
can see that Lft and Rgt are equal, so we have a
solution to the equation. The solution is x = 1.125.

For the TI-Nspire CAS


1. From the home screen (press c), select
1: Calculator. You can do this by pressing 1 or
by using the arrow keys on the NavPad to
highlight 1: Calculator and then pressing ·. <TI01.01>
6 M a t h s Q u e s t M a t h s B Ye a r 1 1 f o r Q u e e n s l a n d

2. Alternatively, open a new Calculator document.


Press /N and then follow the prompts as to
whether you wish to save the previous document
or not. Then press 1 to select 1: Add Calculator.

3. Press b and then select 3: Algebra. (To select


3: Algebra, press 3.)

4. Select 1: Solve. (Alternatively, you can enter the


solve function in the Calculator document by
pressing SOLVE and then (.)

5. Enter the equation as shown in the screen at right.


Press , and then X to indicate we are solving
with respect to the variable x. Press ) to close
the set of brackets.

6. Press · to display the solution. This gives the


solution as a common fraction.

7. To obtain the solution in decimal notation, press


/·. The solution is 9--- or 1.125.
8

remember
Linear equations can be solved by rearranging to make the pronumeral the subject.
Chapter 1 Modelling using linear functions 7

1A Solving linear equations


WORKED 1 Solve the following linear equations.
Example
a 3x − 19 = −13 b −6x + 11 = 59 c 8x − 23 = −47 eBook plus
1a
d 15 − 2x = 1 e 4x + 25 = −7 f 63 − 7x = 21
Digital doc:
g −9x + 21 = 3 h 45x + 72 = −423 i 9x + 19 = −2 EXCEL Spreadsheet
j 15 − 6x = 2 Equation solver

2 Solve:
3x – 1 4x + 2 2x + 8
a --------------- = 5 b --------------- = – 2 c --------------- = 6
4 11 3
5x + 20 12 – 3x 10 – x
d ------------------ = – 5 e ------------------ = 5 f -------------- = – 2
7 3 4
11 – 3x
g ------------------ = 6 6x + 13
h ------------------ = – 4
7 2
WORKED 3 Solve:
Example
1b 4x – 6 2x – 7 12 – 3x
a --------------- – 7 = 3 b --------------- + 11 = 8 c ------------------ – 5 = 6
3 5 3
12 – 9x 7x + 4 1–x
d ------------------ + 7 = – 5 e --------------- – 8 = – 9 f ----------- + 17 = 20
2 3 2
x + 16 19 – x
h -------------- + 3 = – 1
g --------------- – 4 = 0
5 –4
WORKED 4 Solve:
Example
1c 3x 5x 2x
a 2  ------ – 1 = 10 b 3  ------ + 4 = 27 c – 4  ------ + 5 = 4
5  6  3 
x 2x 20x
d 7  8 – --- = 77 e – 5  8 – ------ = – 20 f 6  --------- + 15 = – 150
 4  7  9 
WORKED 5 Solve:
Example
2a
a 2x − 9 = 3(2x − 11) b 5x + 6 = 2(3x + 4) c 7x − 1 = 17(3x − 13)
d 5x + 9 = −4(x + 9) e x + 11 = 2(x + 12) f 5 − 2x = 3(3 − x)
g 3x − 7 = 2(35 − 2x) h 16 − 4x = 7(1 − x)
WORKED 6 Solve:
Example
2b x+2 x–5 x + 11 2 ( x + 14 ) x–1 3x – 17
a ------------ = ----------- b --------------- = ----------------------- c ----------- = ------------------
6 3 3 9 12 8
2x – 4 11 – x 4x + 66 13 – 3x x + 10 2 ( 7 – 3x )
d --------------- = -------------- e ------------------ = ------------------ f --------------- = -----------------------
5 2 3 4 9 5
WORKED 7 Solve:
Example
2c 6x + 7 5x + 1 2x + 29 x + 44 7x – 9 5x + 3
a --------------- = --------------- + 1 b ------------------ = --------------- + 2 c --------------- = --------------- + 6
5 4 3 8 2 4
9x + 28 7x – 6 7x – 9 21 –x 17 – x 49 + 2x
d ------------------ = --------------- – 5 e --------------- = -------------- – 18 f -------------- = ------------------ + 5
5 2 9 3 2 5
8 Use a graphics calculator to solve:
a 3x + 4 = 11 4x – 1 c 2x − 4 = x + 5
b --------------- = 3
3
x+2 3x – 9 x 7 – 3x 6x + 1
d ------------ = --------------- e 7  --- – 2 = 5(3 − 2x)
 f --------------- = --------------- + 5
4 2 4 2 3
8 M a t h s Q u e s t M a t h s B Ye a r 1 1 f o r Q u e e n s l a n d

Rearrangement and substitution


When there is more than one pronumeral involved in an equation, we may rearrange to
make a particular pronumeral the subject using the same rules of equation solving described
in the previous section. In this section, we will also consider equations that are not linear.

WORKED Example 3
Rearrange each of the following to make the pronumeral in red the subject.
m
a 6x + 8y − 48 = 0 y b s = ut + 1--2- at2 u c T = 2 p ---- k
k
THINK WRITE
a 1 Write the equation. a 6x + 8y − 48 = 0
2 Add 48 to both sides. 6x + 8y = 48
3 Subtract 6x from both sides. 8y = 48 − 6x
48 – 6x
4 Divide both sides by 8. y = ------------------
8
24 – 3x
5 Cancel if possible. Here, divide the top and bottom of y = ------------------
the fraction by 2. 4
6 Other ways of representing the answer are shown 24 3
or y = ------ − --- x
opposite. 4 4
3
= 6 − --- x
4
3
= − --- x + 6
4
b 1 Write the equation. b s = ut + 1--- at2
2
2 Subtract 1--- at2 from both sides. s − 1--- at2 = ut
2 2
3 Multiply both sides by 2. 2s − at2 = 2ut
4 Divide both sides by 2t. 2s – at 2
------------------- = u
2t
Write the equation with the desired pronumeral on the 2s – at 2
5 u = -------------------
left. 2t

c m
1 Write the equation. c T = 2 π ----
k

Divide both sides by 2π. T m


2 ------ = ----
2π k

Square both sides. T 2 m


 -----
3 - = ----
 2 π k
2
4 Turn both sides upside down.  2-----π- = ---
k
-
Note: This can be done only if both sides are fractions. T m
2π 2
5 Multiply both sides by m. m  ------ = k
T
2π 2
6 Write the equation with the desired pronumeral on the left. k = m  ------
T
Chapter 1 Modelling using linear functions 9
Once a pronumeral is isolated, we may substitute values of other pronumerals to
calculate various values of the isolated pronumeral. The following worked example
illustrates some practical applications.

WORKED Example 4
a The formula for converting temperature in degrees Fahrenheit (F) (which is the system
5 ( F – 32 )
used in the USA) to degrees Celsius (C) is C = ------------------------ .
9
i Make F the subject.
ii What is the temperature in Fahrenheit when the temperature measured in degrees
Celsius is 21°C?
b The area (A) of a circle is given by A = π r2, where r is the radius. Find the value of r
when A = 20 cm2.

THINK WRITE

5 ( F – 32 )
ai 1 Write the equation. a i C = ------------------------
9
2 Multiply both sides by 9. 9C = 5(F − 32)
9C
3 Divide both sides by 5. ------- = F – 32
5
9C
4 Add 32 to both sides. ------- + 32 = F
5
9C
5 Write the equation with F first. F = ------- + 32
Sometimes it may be appropriate to 5
use a common denominator. 9C + 160
or F = ----------------------
5
9 × 21
a ii 1 Replace C with 21. Note that 9C a ii F = --------------- + 32
means 9 × C. 5
189
F = --------- + 32
2 Evaluate F. 5
F = 37.8 + 32
F = 69.8
So 21°C = 69.8°F

b 1 Write the equation. b A = πr2


2 Solve for r (make r the subject) as follows.
A
3 Divide both sides by π. --- = r 2
π
4 Take the square root of both sides, and A
write r first. As r is the radius, we take r = ---
π
the positive root only.
5 Substitute A = 20 into the new formula. If A = 20,
20
------
r= π
6 Evaluate r. = 2.523 cm
10 M a t h s Q u e s t M a t h s B Ye a r 1 1 f o r Q u e e n s l a n d

WORKED Example 5
Note: In this example, m is short for metres, m/s is short for metres per second (velocity),
and m/s2 is short for metres per second per second (acceleration).
The final velocity v m/s of an object that begins with velocity u m/s and accelerates at
a m/s2 over a distance of s m is given by the equation v2 = u2 + 2as.
a Find the value of v when u = 16, a = −2 and s = 60.
b Rearrange the given equation to make s the subject.
c Find the distance travelled by an object which begins with a velocity of 10 m/s, and has
a final velocity of 4 m/s while accelerating at −1 m/s2.
THINK WRITE
a 1 Write the given equation. a v2 = u2 + 2as
2 Substitute u = 16, a = −2 and s = 60. v2 = 162 + 2 × −2 × 60
3 Simplify. = 256 – 240
= 16
4 Take the square root of both sides to v = ± 16
evaluate v.
= ±4
Final velocity is 4 m/s.
b 1 Begin with the given equation. b v2 = u2 + 2as
2
2 Subtract u from both sides. v2 − u2 = 2as
v2 – u2
3 Divide both sides by 2a. ---------------- = s
2a
v2 – u2
4 Reverse so that s is given on the left. s = ----------------
2a
c 1 Match the pronumerals with the given c u = 10
information. v =4
a = −1
Write the formula that has s as the subject v2 – u2
2 s = ----------------
(see part b above). 2a

Substitute the values given in step 1. 4 2 – 10 2


3 = -------------------
2 × –1
16 – 100
4 Simplify and evaluate. = ---------------------
–2
– 84
= ---------
–2
= 42
5 Explain the answer in words. The object travels 42 m in the same
direction as its initial direction.

Some graphics calculators are able to isolate a pro-


numeral in an equation. The screen at right is from a
TI-Nspire CAS calculator. The solve function has been
used to rearrange the equation v2 = u2 + 2as to make s the
isolated pronumeral. (When entering 2as on the calcu-
lator, remember to enter a multiplication sign between
each pronumeral.) Notice the pronumeral shown after the
comma is s, the pronumeral being solved for.
Chapter 1 Modelling using linear functions 11

Graphics Calculator tip! Substituting


pronumerals
for

Consider the equation v2 = u2 + 2as given in Worked example 5. In part c, we were


asked to find the distance travelled by an object which begins with a velocity of 10 m/s
and has a final velocity of 4 m/s while accelerating at −1 m/s2. With a graphics calcu-
lator, we can substitute directly into the given equation and find the value for s.
For the Casio fx-9860G AU
1. Press MENU and then select EQUA. Press F3
(SOLV) to select Solver. Enter the equation
v2 = u2 + 2as. It is important to enter a
multiplication sign between variables. (So, to
enter 2as on the calculator, press 2 ×
ALPHA [A] × ALPHA [S].) Press EXE .
2. Enter the known values for v, u and a. Enter 4 for
v and then press EXE . Similarly, enter 10 for u
and −1 for a. Ensure the variable we want to solve
for is highlighted.

3. Press F6 (SOLV) to display the value for s. The


solution is s = 42.

For the TI-Nspire CAS


1. Open a new Calculator document. Enter the
equation v2 = u2 + 2as. It is important to enter a
multiplication sign between variables. (So, to
enter 2as on the calculator, press 2rArS.)
Move the cursor to the far right of the equation.

2. To enter the known values of the pronumerals v, u


and a, we first press *. This is the ‘given’ or
‘with’ symbol and is located on a grey button to
the right of the equals button (=). Enter u = 10.

3. Follow each piece of known information with the


word ‘and’. To enter the word ‘and’, press k to
access the catalog and then press 1 to display
the function menu. Use the NavPad to scroll down
to and.
12 M a t h s Q u e s t M a t h s B Ye a r 1 1 f o r Q u e e n s l a n d

4. Press · to insert and into the Calculator


document. Now enter v = 4 and a = − 1, with and
entered between each. Press · to display the
equation in terms of s.

5. To solve for s, press b and then select


3: Algebra followed by 1: Solve. Press /v to
enter the answer from the previous line followed
by , and S. Press ) to close the set of
brackets.

6. Press · to display the value for s. The solution is


s = 42.

remember
Equations may be rearranged by applying the same rules as those used to solve
equations.

1B Rearrangement and
substitution
WORKED 1 Each of the following is a real equation used in business, mathematics, physics or
Example
3
another area of science. Make the pronumeral shown in red the subject in each case.
a A=L+P P b A = lw l
d
c v = --- t d C = 2π r r
t
PRT f V = 1--- π r 3h
e A = ----------- R 3
r
100
g E = I4 R4 − I1 R1 + I2 R2 − I3 R3 I3 h R1 = R2(1 + αθ) α
kQq
i E = αθ + βθ 2 β j F = --------- - r
r2

k E = ------ φ l V1 N 2 = V2 N 1 V2
t
m pV = nRT n n s = ut + 1--- at2 a
2
Chapter 1 Modelling using linear functions 13
o Fd = 1--- mv2 − 1--- mu2 µI 2
2 2
v p F = --------- r
2πr
V f1
q f 2 = --------------
- U r v = γ rT γ
V +U
s S = 2w(l + h) + 2lh w t S = 2π r 2 + 2π rH H
WORKED 2 Find the value of the subject (the first mentioned pronumeral), given the values of the
Example
4
other pronumerals.
a W = Fd F = 10, d = 5.6
b P = 2l + 2w l = 6, w = 9
c A = 1--- bh b = 10, h = 16
2

d A = s(s – a)(s – b)(s – c) s = 12, a = 4, b = 9, c = 11


R1 R2
e R T = -----------------
- R1 = 50, R2 = 100
R1 + R2
k
f I = ----2- k = 60, d = 15
d
g E = K + mgh K = 250, m = 2, g = 10, h = 5
1
h D = (n – --- ) λ
2
n = 3, λ = 2.8
i E = hf0 − W h = 6.62, f0 = 5000, W = 20 000
j v = ω r 2 – y2 ω = 2, r = 1.6, y = 1
3 Make the pronumeral in red the subject, and find its value using the given information.
a A = l2 l A = 60
b V = 4--- π r 3 r V = 1000
3

c F = mg − kv2 v F = 250, m = 60, g = 10, k = 0.1


d v = u + at a v = 25, u = 0, t = 6
e S = π r(r + h) h S = 120, r = 2, π = 3.14
l
f T = 2π --- l T = 4, g = 9.8, π = 3.14
g
l2 – d 2
g f = --------------- d f = 2, l = 15
4l
h H = U + PV V H = 26, U = 4.5, P = 2
cα2
i K = ------------ c K = 6.9, α = 0.05
1–α
H v
j -------i = --- u Hi = 34, H0 = 4, v = 40
H0 u

WORKED 4 The perimeter, P, of a rectangle of length l and width w may be found using the
Example
5
equation P = 2(l + w).
a Find the perimeter of a rectangle of length 16 cm and width 5 cm.
b Rearrange the equation to make w the subject.
c Find the width of a rectangle that has perimeter 560 mm and length 240 mm.
14 M a t h s Q u e s t M a t h s B Ye a r 1 1 f o r Q u e e n s l a n d

5 The net force, F, measured in Newton (N) acting on a mass m kilograms (kg) is
found using the equation F = ma, where a is the acceleration of the mass measured
in m/s2.
a Find the net force required to accelerate a 2.5 kg rock at the rate of 4 m/s2.
b Make a the subject of the equation.
c Find the acceleration produced by a 700 N force acting on a 65 kg person.

a+b
6 The area of a trapezium (figure A below) is given by A =  ------------ h , where a and b
 2 
are the lengths of the parallel sides, and h is the height.
a Find the area of the trapezium shown in figure B below.
b Using figure A, find an equation for determining side a in terms of the area A and
side b.
c Find a in figure C below.
9m
a
50 cm
Area 16 m 62 cm a
h
A
Area = 2000 cm2
b 21 m
Figure A Figure B Figure C

7 The size of a 2-year investment account with a particular bank is given by


2
r
A = D  1 + --------- , where A is the amount ($) in the account after two years, D is the
 100
initial deposit ($) and r is the interest rate (%).
a Find the amount in such an account after two years if the initial deposit was $1000
and the interest rate was 6%.
b Make r the subject of the equation.
c Find the rate required for an initial deposit of
$1000 to grow to $2000 after 2 years. L

8 The length of a circular curve is given by the formula


r
πrθ
L = --------- where L = length of curve, π = 3.14,
180 θ r
r = radius of curve and θ is the angle of the curve.
a Determine, to the nearest degree, the angle required
for a curve of radius 15 m and length 40 m.
b Find the length of the curved edge of the fan at right.
123°
8 cm
9 The length, L, of a metal strip required to make the
w
bracket at right is given by L = a + b + ---- . a
2
w
a What length is required to make a bracket for
which a = 25 cm, b = 35 cm and w = 5 mm? b
Chapter 1 Modelling using linear functions 15
b Find the value of w (in cm) necessary 40 cm
to make the bracket at right from a
60 cm length of metal.

17 cm
10 The object and image positions for a
lens of focal length f are related by the
1 1 1
formula --- + --- = --- , where u is the distance of the object from the lens and v is the
u v f
distance of the image from the lens.
a Make f the subject of the equation.
b Make u the subject of the equation.
c How far from the lens is the image when an object is 30 cm in front of a lens of
focal length 25 cm?

Gradient of a straight line


The gradient of a line describes its slope or steepness. You may recall from previous
studies the following types of gradient for straight lines.

y y y y
t
ien
ad

Zero
gr

gradient
ive

Infinite
sit

Negative gradient
po

Positive gradient
ter

gradients
ea

x x x
Gr

The gradient may be calculated


using the formula gradient Scientists calculate the
required gradient of
rise y2 – y1 solar panels so that the
m = -------- or m = ---------------- . These
run x2 – x1 maximum amount of
terms are illustrated below. energy is absorbed.

y
(x2, y2)

Rise
(x1, y1) Run

Here are two examples of where


gradient can be very significant.
Can you think of others?
A cliff face with
a steeper gradient
provides a greater
challenge for climbers.
16 M a t h s Q u e s t M a t h s B Ye a r 1 1 f o r Q u e e n s l a n d

WORKED Example 6
Calculate the gradient of this linear graph using the intercepts shown. y

14
THINK WRITE
1 Identify the rise and run.
rise = 14, run = 2
rise 14 –2 x
2 Calculate m = -------- . m = ------
run 2
= 7

WORKED Example 7
Calculate the gradient of the line passing through the points (3, −6) and (−1, 8).

THINK WRITE
y2 – y1 y2 – y1
1 Use the formula m = ---------------
-. m = ---------------
-
x2 – x1 x2 – x1
2 Match up the terms in the formula with the
values given.
(x1, y1) (x2, y2)
(3, −6) (−1, 8)
Substitute the given values. 8 – –6
3 m = ---------------
−1 – 3
Simplify. 14
4 = ------
–4
Cancel if possible. 7
5 = – ---
2

Gradient and the angle made by the line with the x-axis
If the angle a line makes with the positive direction of the x-axis is known, the gradient
may be found using trigonometry applied to the triangle shown below.

rise

θ
run x

tanθ = rise = m
run
Chapter 1 Modelling using linear functions 17
WORKED Example 8
a Find the gradient (accurate to 3 decimal places) of a line
making an angle of 40o to the positive x-axis, as shown in
the figure at right. 40°
x
b Find the gradient of the line shown below.
Express your answer to 2 decimal places.
y

60°
x

THINK WRITE
a Since the angle the line makes with the a m = tan θ
positive x-axis is given, the formula = tan 40°
m = tan θ can be used. = 0.839
y
b 1 The angle given is not the one between b θ = 180° − 60°
the graph and the positive direction of the = 120° 60° θ
x-axis. Calculate the required angle θ. x
2 Use m = tan θ to calculate m to m = tan θ
2 decimal places. = tan 120°
= −1.73

remember
The gradient (m) of a straight line may be calculated using the following
formulas:
rise
m = -------- .
run
y

Rise

Run

y2 – y1
m = ---------------
- where (x1, y1) and (x2, y2) are points on the line.
x2 – x1
m = tan θ where θ is the angle the line makes with the positive direction of the
x-axis.
18 M a t h s Q u e s t M a t h s B Ye a r 1 1 f o r Q u e e n s l a n d

1C Gradient of a straight line

WORKED 1 Calculate the gradient of each of the following linear graphs using the intercepts
Example
6
shown.

a y b y c y

6
5

eBook plus 1

Digital doc: –3 x –1 x –4 x
EXCEL Spreadsheet
Gradient of a
straight line d y e y f y

2
12
7

6 x 3 x 7 x

g y h y
32 100

–10 x 45 x

WORKED 2 Without drawing a graph, calculate the gradient of the line passing through:
Example
7 a (2, 4) and (10, 20) b (4, 4) and (6, 14)
c (10, 4) and (3, 32) d (5, 31) and (−7, 25)
e (7, 2) and (12, −28) f (−3, 2) and (42, 17)
g (0, −30) and (−8, −31) h (−11, −25) and (0, −3)
i (217, 4) and (19, 4) j (3, −3) and (−45, 21)
k (1, 32) and (67, −100) l (−2, −5) and (0, 0).

3 Calculate the gradient of the line joining each pair of points.


a y b y
(1, 6)
c y
(5, 5)
(1, 3)
(6, 1)
x x x
(3, –2)
(–5, –4)
Chapter 1 Modelling using linear functions 19
d y e y f y

x (–3, 8)
(–2, –2) (2, 2)

x
(–2, –2)

x
(–7, –10)

(–2, –4)

g y h y
(6, 9)
(–5, 3) (4, 3)

(6, –3)

4 Find the gradient of the line joining each pair of points.


a y b y c y
5 5 5
4 4 4
3 3 3
2 2 2
1 1 1
–5 –4 –3 –2 –1 0 1 2 3 4 5 x –5 –4 –3 –2 –1 0 1 2 3 4 5 x –5 –4 –3 –2 –1 0 1 2 3 4 5 x
–1 –1 –1
–2 –2 –2
–3 –3 –3
–4 –4 –4
–5 –5 –5

d y e y f y
6
5 5 5
4 4 4
3 3 3
2 2 2
1 1 1
–5 –4 –3 –2 –1 0 1 2 3 4 5 x –5 –4 –3 –2 –1 0 1 2 3 4 5 x –5 –4 –3 –2 –1 0 1 2 3 4 5 x
–1 –1 –1
–2 –2 –2
–3 –3 –3
–4 –4 –4
–5 –5 –5

g y h y
5 5
4 4
3 3
2 2
1 1
–5 –4 –3 –2 –1 0 1 2 3 4 5 x –5 –4 –3 –2 –1 0 1 2 3 4 5 x
–1 –1
–2 –2
–3 –3
–4 –4
–5 –5
20 M a t h s Q u e s t M a t h s B Ye a r 1 1 f o r Q u e e n s l a n d

5 Find the gradient of each line below.


a y b y c y
6
5 5 5
4 4 4
3 3 3
2 2 2
1 1 1
–5 –4 –3 –2 –1 0 1 2 3 4 5 x –5 –4 –3 –2 –1 0 1 2 3 4 5 x –5 –4 –3 –2 –1 0 1 2 3 4 5 x
–1 –1 –1
–2 –2 –2
–3 –3 –3
–4 –4 –4
–5 –5 –5

d y
6
e y f y
5 5 5
4 4 4
3 3 3
2 2 2
1 1 1
–6 –5 –4 –3 –2 –1 0 1 2 3 4 5 6x –5 –4 –3 –2 –1 0 1 2 3 4 5 x –5 –4 –3 –2 –1 0 1 2 3 4 5 x
–1 –1 –1
–2 –2 –2
–3 –3 –3
–4 –4 –4
–5 –5 –5
–6

g y
6
h y
6
5 5
4 4
3 3
2 2
1 1
–6 –5 –4 –3 –2 –1 0 1 2 3 4 5 6 x –6 –5 –4 –3 –2 –1 0 1 2 3 4 5 6 x
–1 –1
–2 –2
–3 –3
–4 –4
–5 –5
–6 –6

WORKED 6 Find the gradient (accurate to 3 decimal places) of a line making the angle given with
Example
8a
the positive x-axis.
a 50° b 72° c 10° d −30°
e 150° f 0° g 45° h 89°
WORKED 7 Find the gradient of each line below. Give answers to 2 decimal places.
Example
8b a y b y c y d y

x
15°
43° 28°
x x
69°
x

8 Which of these lines has: y B D


a a non-zero positive gradient? 5
4
b a negative gradient? 3
c a zero gradient? 2
d an undefined gradient?
1 C
–5 –4 –3 –2 –1 0 1 2 3 4 5 x
–1
–2
A –3
–4
–5
Chapter 1 Modelling using linear functions 21
9 multiple choice
a Which of the following graphs has a b Which of the following lines has a
gradient of −2? gradient of 3?
y A y
B A C D B C
5 5
4 4
3 3
2
E 2
1 1
D
–5 –4 –3 –2 –1 0 1 2 3 4 5 x –5 –4 –3 –2 –1 0 1 2 3 4 5 x
–1 –1
–2 –2
–3 –3
–4 –4
–5 –5 E

c Which of the following lines has a d Which of the following lines has a
gradient of 1--- ? gradient of − 2--- ?
2 5
A y B y
B
5 5
4 4 A
3 3
C 2 D 2
1 E 1
–5 –4 –3 –2 –1 0 1 2 3 4 5 x –5 –4 –3 –2 –1 0 1 2 3 4 5 x
–1 –1
–2 –2 D
–3 –3
–4 –4
–5 –5
C

10 Arrange the following in order from smallest to largest gradient. The same scale has
been used to draw each graph.
a y b y c y d y e y

x x x x
x

11 Sketch two different graphs that have:


a the same gradient
b zero gradients
c different negative gradients.
12 Burghar plots the coordinates of a proposed driveway on a plan which is shown
below. What is the gradient of the proposed driveway?

Garage

way
Drive
2m

17 m
22 M a t h s Q u e s t M a t h s B Ye a r 1 1 f o r Q u e e n s l a n d

13 An assembly line is pictured below. What is the gradient of the sloping section? (Give
your answer as a fraction.)
BOFFO
Made in
Austra
lia
BOFF
Made inO
Austra
lia
BOFF
Made inO
0.85 m Austra
lia

15 m
14 A passenger jet takes off along the flight path shown below. What is the gradient of
the path?

150 m
110 m

Runway
500 m
15 Find the value of a in each case so the gradient joining the points is equal to the value
given.
a (3, 0) and (5, a) gradient: 2
b (2, 1) and (8, a) gradient: 5
c (0, 4) and (a, −11) gradient: 3
d (a, 5) and (5, 1) gradient: −2
16 For safety considerations, wheelchair ramps are constructed under regulated specifi-
eBook plus
cations. One regulation requires that the maximum gradient of a ramp exceeding
1
Digital doc: 1200 mm in length is to be -----
-.
14
SkillSHEET 1.1 a Does a ramp 25 cm high with a
Using gradient to find the
value of a parameter horizontal length of 210 cm
meet the requirements?
1
b Does a ramp with gradient ----- -
18
meet the specifications?
c A 16 cm high ramp needs to be
built. Find the horizontal length
of the ramp required to meet
the specifications.
Chapter 1 Modelling using linear functions 23
Career profile
R I C K M O R R I S — Vi g n e ro n

where X = litres of fortifying spirit required


V = litres of wine to be fortified
C = final alcohol strength of the wine
in % by volume
A = alcohol strength of the wine
before fortification
B = alcohol strength of fortifying
spirit
If we have 5000 litres of wine at 10%
Testing the sugar content of the
alcohol by volume and our fortifying spirit is
fermenting wine with an hydrometer 95% alcohol by volume then the litres of
to measure the specific gravity uses spirit that are required is:
a French graduation called
degrees Baum/e. 5000 ( 18.5 – 10 )
1 baum/e = 1.008 SG ---------------------------------------
95 – 18.5
Qualifications: Nil (No formal qualifications) 42 500
= ----------------
Employer: Self-employed 76.5
= 555.5.
I grew up on the family vineyard and was That is, 555.5 litres of spirit is required.
encouraged to develop an interest in wine- It is necessary to know the exact capacity
making at a very early age. Thus, I was of the mixing tank where the fortification is
familiar with the workings of a vineyard carried out. If the tank is cylinder shaped, then
from nearly day one. A usual day for me is the formula is a straightforward volume of a
to crush the previous day’s grape harvest, cylinder calculation. Then, once the number
add yeast to establish the correct of cubic centimetres is known, it can be
fermentation and then I am out to the converted into litres. A wooden dipstick is
vineyard for picking the next block of constructed with either centimetre marks and
grapevines. Part of my job is to look after the number of litres per centimetre calculated
customers by serving tastings of the wines. or marked in actual litres for that particular
I then fill, cork and label more wine for the tank. The exact amount of spirit to add to a
sales room. The wine fermentations are wine can be calculated using maths. This can
checked and any wine that has completed save a lot of money and reduce wastage.
its fermentation needs the skins to be
Questions
pressed off.
1. What are the two ways in which a wooden
Mathematics is very important in
dipstick can be marked?
winemaking as it is essential that quantities
2. Rick uses skills in substituting into an
are calculated accurately. For example, to equation to calculate the amount of pure
calculate the amount of pure alcohol to add alcohol needed to fortify a wine.What other
to a wine to fortify it to 18.5% by volume, areas of mathematics are useful to him?
the Pearson Square formula is used. 3. Although Rick has no formal qualifications,
V (C – A) are there any courses for vignerons? If so,
X = -----------------------
B–C what are they? Where else could you gain
skills for working at a vineyard?
24 M a t h s Q u e s t M a t h s B Ye a r 1 1 f o r Q u e e n s l a n d

Equations of the form y = mx + c


A common form for linear equations is y = mx + c, sometimes referred to as gradient-
intercept form.
y y
t m
d ien
a
Gr
y = mx + c
y-intercept

Gradient y-intercept x x

x-intercept

These lines have identical


gradients (equal m values).

The following exercise aims to demonstrate the effect of altering m and c. A graphics
calculator would be useful, but is not essential.

Graphics Calculator tip! Graphing


functions
linear

A graphics calculator can be used to draw the graph of a linear function. As an


example, let’s draw the graph of y = 2x − 7.
For the Casio fx-9860G AU
1. Press MENU and then select GRAPH.

2. The Graph Function screen shows you can enter


up to 6 functions. However, only a maximum of 3
functions can be drawn on the one set of axes.
Press F2 (DEL) to delete any equations shown.

3. Enter 2x − 7 for Y1 and then press EXE .

4. Press F6 (DRAW) to display the graph.


Chapter 1 Modelling using linear functions 25
5. To obtain a clearer view of the graph, the scale
settings can be changed. Press SHIFT F3 (V-WIN)
and then adjust the View Window settings so that
Xmin is −10, Xmax is 10, Ymin is −20 and Ymax is <Casio01.12>
15. Press EXE after each new entry or press the
down arrow to skip to the next line.
6. Press EXIT to return to the Graph Function
screen. Press F6 (DRAW) to display the graph
with the new settings.
<Casio01.13>

7. Press SHIFT F1 (TRCE) to display the equation


of the line and the coordinates of a point on the
line. Pressing the arrow keys allows you to move
the point (or trace) along the line. <Casio01.14>

For the TI-Nspire CAS


1. Open a new Graphs & Geometry document.
Press /N and follow the prompts as to whether
you wish to save the previous document. Then
press 2 to select 2: Add Graphs & Geometry. <TI01.15>
A set of axes for x and y (with default settings
for the axis scale) will be displayed. Along the
bottom of the screen, you will see a function entry
line.
2. Enter 2x − 7 in the function entry line for f1(x).

<TI01.15>

3. Press · to display the line with its rule.

<TI01.15>

4. If you wish to alter the scale on each axis, press


b and then select 4: Window followed by
1: Window Settings. Adjust the values so that
XMin is –10, XMax is 10, YMin is –20 and YMax is <TI01.15>
15. Press e to move to the next line. Continue to
press e until OK is highlighted.
26 M a t h s Q u e s t M a t h s B Ye a r 1 1 f o r Q u e e n s l a n d

5. Press · to accept the new Window Settings.


The graph will be redrawn with these settings.

<TI01.15>

6. To move the rule to a new position in the work


area, first press d to display a pointer (å), and
then use the NavPad to move the pointer so that it
hovers over the equation. The pointer will then <TI01.15>
appear as an open hand (÷). Press / followed
by the Click button (x) to close the hand ({). The
equation will start to flash. Use the NavPad to
move the equation to the desired location and then
press d to anchor it in position.

7. Press b and then select 5: Trace followed by


1: Graph Trace. A point on the line with its
coordinates will be displayed. Use the NavPad to
move the point (or trace) along the line. (The <TI01.15>
Trace Step can be altered. Press b and then
select 5: Trace followed by 2: Trace Settings.
Enter the number by which the x-value should
change in each step.)

remember
1. The general equation for a straight line of gradient m and y-intercept c is
y = mx + c.
2. Lines with the same gradient (m) are parallel.

1D Equations of the form y = mx + c

eBook plus 1 Use a graphics calculator or other method to sketch graphs of the following on the
same set of axes.
Digital doc: a y=x b y = 2x c y = 3x d y = −x e y = −2x
EXCEL Spreadsheet
Linear graphs
2 What is the effect on the graph of the number in front of the x (the ‘x-coefficient’ or
‘gradient’)?

3 Use a graphics calculator or other method to sketch graphs of the following on the
same set of axes.
a y=x+1 b y=x+2 c y=x+3 d y=x−4
Chapter 1 Modelling using linear functions 27
4 Use a graphics calculator or other method to sketch graphs of the following on the
same set of axes.
a y = 2x + 1 b y = 2x − 7 c y = −3x + 6 d y = 3x − 5

5 What is the effect on the graph of the number at the end of the equation (the
‘y-intercept’)?

6 State the gradient for each of the following equations.


a y = 5x + 7 b y = 6x − 4 c y = −9x + 1 d y = 2x − 13
e y = −8x − 5 f y=x+2 g y = −x − 10 h y = 5x + 0
i y = 3x j y = 0x + 17 k y=2 l y=0

7 State the y-intercept for each of the equations in the previous question.

8 Write the equation of a line having the following properties (where m = gradient and
c = y-intercept).
a m = 2, c = 7 b m = −3, c = 1 c m = 5, c = −2
d m = 0, c = 3 e m = 1, c = 0 f m = 1--- , c = −5
2 1 3 1 2
g m = --- , c = --- h m = − --- , c = − --- i y-intercept 12, gradient −2
3 3 4 2

9 Rearrange the following equations and hence state the gradient and y-intercept for each.
a y = 9 + 3x b y = −42 + 7x c y = 12 − 4x
d y = −35 − 5x e y − 3x = 10 f y + 6x = 24
g y + 16x = −15 h y + 9x + 1 = 0 i y − x + 23 = 0

10 Rearrange the following equations and hence state the gradient and y-intercept for each.
a 2y = 8x + 10 b 3y = 12x − 24 c −5y = −20x + 30
d −y = 3x − 1 e 16 − 4y = 8x f 22 − 2y = −6x
g 21x + 3y = −27 h −10x + 5y = 25 i 6y + 3x = −18
j −11y − 2x = 66 k 8x + 3y − 2 = 0 l −3x − 4y + 13 = 0
m 15 − 6y + x = 0 n 2y + 7 + 5x = 0

11 multiple choice
Which of the following lines is parallel to (that is, has the same gradient as)
y = −4x − 7?
A 5 − 4y = 13 B x = −4y − 7 C 4x + y + 8 = 0 D y = 4x − 7 E y = 5x − 8

12 multiple choice
Which of the following lines has the same y-intercept as y = 18x − 2?
A y = 2 − 18x B y − 18x − 2 = 0 C 3x + 7y = −2
D 14 − 7y − 2x = 0 E 2x = −14 − 7y

13 Write three equations starting with y =, that each have a gradient of −7.

14 Write three equations starting with y =, that each have a y-intercept of −6.

15 Write three equations that have the same gradient as 3y + 5x = 17. Write them in the
same form as the equation in this question.

16 Write three equations that have the same y-intercept as 3y + 5x = 17. Write them in
the same form as the equation in this question.
28 M a t h s Q u e s t M a t h s B Ye a r 1 1 f o r Q u e e n s l a n d

17 State the equation for each of the following graphs.


a y b y
6 3
5 2
4 1
3 –2 0 2 x
– –21 2 –1
1 –2
–3
–1 0 1 2 x –4
–1
–2 –5

c y d y
3 5
4
2
3
1 2
–2 –1 0 1 2 x 1
–1 –1 0
–1
1 2 3 4 5 6 x
–2
–3 –2
–3
–4
–4
–5 –5

e y f y
9
5 8
4 7
3 6
2 5
1 4
3
–1 0
–1
1 2 3 4 x 2
–2 1
–3
–3 –2 –1 –10 1 2 3x
–4
–5 –2
–3
–4
–5
–6
–7
–8
–9

y
18 a A set of axes is placed as shown
over the Leaning Tower of Pisa. 5.2 m
Find the equation of the line
representing the sloped left wall
of the Tower.
b What angle does the Tower make
with the ground?
19 State the gradient and y-intercept
eBook plus
(in that order) for the following.
a y = ax + b 55.8 m
Digital doc:
WorkSHEET 1.1 b ax + by = c
c ax + by + c = 0
d 2y = 4kx − 6h
Chapter 1 Modelling using linear functions 29
Sketching linear graphs using intercepts
To draw a graph, only two points are needed. A line may then be drawn y
through the two points, and will include all other points that follow the given
rule. Rather than construct a table of values and plot several coordinates, it
is quicker to mark only the points where a line cuts (or intersects) an axis.
These points are called x- and y-intercepts. The x-intercept occurs when
y = 0, while the y-intercept occurs when x = 0. This information is the basis
for the approach to sketching illustrated in the following worked examples. x

WORKED Example 9
Sketch the graph of y = −6x + 15, showing x- and y-intercepts.
THINK WRITE
1 Find the y-intercept (when x = 0). If x = 0, y = −6 × 0 + 15
Substitute x = 0 into the equation. y = 15
2 Find the x-intercept (when y = 0). If y = 0, 0 = −6x + 15
Substitute y = 0 into the equation. 6x = 15
15
x= ------
6
5
x= --- or 1 2---
3 3
3 Mark the intercepts on a set of axes. y
Note that 5--- = 1.66̇ (a little over 1 1--- ).
3 2
4 Rule a straight line through the two intercepts. 15

5– x
3

WORKED Example 10
Sketch the graph of 3x − 2y = 12.
THINK WRITE
1 Find the y-intercept (when x = 0). If x = 0, 3 × 0 − 2y = 12
Substitute x = 0 into the equation. −2y = 12
12
y = ------
–2
y = −6
2 Find the x-intercept (when y = 0). If y = 0, 3x − 2 × 0 = 12
Substitute y = 0 into the equation. 3x = 12
x=4
Continued over page
30 M a t h s Q u e s t M a t h s B Ye a r 1 1 f o r Q u e e n s l a n d

THINK WRITE
3 Mark the intercepts on a set of axes. y
4 Rule a straight line through the two
intercepts.
4 x

–6

The graphs of some equations do not have two intercepts because they pass through
the origin (0, 0). Such equations are of the form y = kx or ax + by = 0.
To sketch graphs of such equations, we use (0, 0) and any other point, for example,
the point where x = 1. (We could choose any other non-zero value.)

WORKED Example 11
Sketch the graph for the equation 4x − 3y = 0.
THINK WRITE
1 Try substituting x = 0 to find the If x = 0, 4 × 0 − 3y = 0
y-intercept. −3y = 0
y=0
2 Note that the graph passes through
(0, 0). There is no point substituting
y = 0, as we know we’ll get x = 0.
3 Substitute another x-value. In this If x = 1, 4 × 1 − 3y = 0
example we use x = 1. 4 − 3y = 0
4 = 3y
4
y = ---
3
4 Plot the points (0, 0) and (1, 4--- ) on a set y
3
of axes. Note that 4--- is 1 1--- , which is a
3 3
little less than 1 1--- . (1, 4–3 )
2

5 Rule a straight line through the two


x
points.

remember
To sketch a linear graph:
1. Let x = 0 and find the y-intercept.
2. Let y = 0 and find the x-intercept.
3. If (0, 0) is an intercept, find another point on the line by substituting x = 1 (or
any other convenient non-zero value).
4. Mark the intercepts and rule a straight line through the two points.
Chapter 1 Modelling using linear functions 31
Sketching linear graphs using
1E intercepts

WORKED 1 Sketch graphs of the following linear equations, showing x- and y-intercepts.
Example
9 a y = 6x + 18 b y = 3x − 21
c y = 5x + 12 d y = −2x − 3
e y = 10 − 5x f y=1−x
g y = −9x + 30 h y = 2(x − 8)

WORKED 2 Sketch graphs for each of the following.


Example
10 a 2x + 3y = 6 b 4x + 5y = 20
c −5x + 8y = 10 d 6x − 3y = −18
e 7x − 5y = 35 f 8y − x = 4
g x−y=2 h −2x + 11 = 6y

3 Sketch the graph for each equation.


a 6x + 7y + 42 = 0 b 5x − 2y + 20 = 0
c −3x + 4y − 16 = 0 d y − 3x + 6 = 0
e 9x + 18 − 2y = 0

WORKED
Example
4 Sketch the graph for each equation.
11 a x+y=0 b x−y=0
c 2x + y = 0

5 multiple choice
Which of the following is in the form ax + by = c? (One or more answers.)
A 2x − 3y − 1 = 0 B 2x + 3y + 1 = 0 C 2x + 3y = x
2
D 2x + 3y = 1 E y = --- x − 1
3

6 multiple choice
The x- and y-intercepts for the equation 2y = −3x + 12 are (respectively):
A 2 and 3 B −3 and 12 C −4 and 6
D −4 and −6 E 4 and 6

7 multiple choice
Which of the following has a y-intercept of −3?
A y = −3x − 3 B y = −3x + 3 C x + 3y = 9
D x − 3y + 9 = 0 E 3x + y + 9 = 0
32 M a t h s Q u e s t M a t h s B Ye a r 1 1 f o r Q u e e n s l a n d

Graphics Calculator tip! Finding x- and y-intercepts


To find the x- and y-intercepts from a graph displayed on a graphics calculator, follow
the steps below. As an example, we will consider the graph of y = 2x − 7 that we pro-
duced earlier.
For the Casio fx-9860G AU
1. Draw the graph of y = 2x − 7. (See instructions on
page 24.)

<Casio01.15>

2. To find the y-intercept, press SHIFT F5 (G-SLV)


and then F4 (Y-ICPT). The coordinates of the
point where the line crosses the y-axis will be
displayed. The y-intercept is −7. <Casio01.16>

3. To find the x-intercept (also known as the root of


the equation), press SHIFT F5 (G-SLV) and
then F1 (ROOT). The coordinates of the point
where the line crosses the x-axis will be displayed. <Casio01.16>
The x-intercept is 3.5.

For the TI-Nspire CAS


1. Draw the graph of y = 2x − 7. (See instructions on
page 25.)

<TI01.24>

2. To find the y-intercept, press b and then select


5: Trace followed by 1: Graph Trace. This will
automatically display the point on the line where
x is 0. The coordinates will also be shown. The <TI01.24>
y-intercept is –7.

3. To find the x-intercept, use the NavPad to move


the point along the line towards the x-axis. A box
containing the letter z will appear when the point
has reached the x-intercept. The coordinates will <TI01.24>
also be shown. The x-intercept is 3.5.
Chapter 1 Modelling using linear functions 33
Simultaneous equations
The Garn Express — the crisis worsens
Let us recall the problem that began our consideration of linear equations. A train is
travelling from Cannes to Brampton and is running on the wrong track. It can be con-
tacted when it reaches Alberton — a distance of 250 km from Brampton.
Cannes Now we are able to describe the train’s position more comprehen-
sively.
We have drawn a line of best fit on the graph of position
300 km
versus time. The gradient of the straight line is found by con-
sidering the following two points on the line: (0, 550) and
Alberton (225, 200).
550 – 200
Thus, the gradient is ------------------------ = −1.56. That is, the speed of the
250 km 0 – 225
train is 1.56 km/min with the negative sign indicating that the dis-
tance between it and Brampton is decreasing.
Brampton

600 Thus, the equation


500 relating the distance from
Brampton (km)
Distance from

400
Brampton, d, to the time
in minutes, t, is:
300
d = 550 − 1.56t
200
Using this model we
100 can predict the time the
0 train will reach Alberton:
0 50 100 150 200 250
Time (min) d = 250 km
250 = 550 − 1.56 × t
t = 192 min

News Flash! The Rail Authority has a crisis on its hands. With communications down,
they discover — too late — that a second train, the Moonlander, is on the line and
heading for a collision with the Garn Express. If the Express can reach Alberton first, a
disaster will be averted. Will this happen?
Again, the satellite data are used to develop a model and straight lines are drawn to
fit the data.
It appears, on the basis of
these data, that the Garn
600 Garn Express
Express will reach Alberton
500 Moonlander
Brampton (km)

before the Moonlander and


Distance from

be re-routed to safety. We 400


can see from the graph that 300
the Garn Express meets the 200
second train after passing 100
through Alberton, whose 0
position is marked by the 0 50 100 150 200 250
Time (min)
dotted line on the graph.
34 M a t h s Q u e s t M a t h s B Ye a r 1 1 f o r Q u e e n s l a n d

However, the technical officers report that the speed of both trains may vary by 8%.
As there is not a large margin for error, how can we determine what will happen if such
variation occurs?
First, let us examine the methods used to find the intersection of two straight lines.

Simultaneous equations — definition


Simultaneous equations are groups of
y
equations containing two or more pronumerals.
In this section, we look at pairs of linear
Both graphs have
equations involving the pronumerals x and y. the same x- and
Each equation, as we have learnt in previous x y-values here.
sections, may be represented by a linear graph
that is true for many x- and y-values. If the
graphs intersect (when wouldn’t they?), the
values of x and y at the intersection are those
that make both equations true.

Graphical solution
The following example shows how a graphics calculator may be used to solve
simultaneous equations graphically. Hand drawn sketch graphs may be used if graphics
calculators are not available. Refer to earlier sections if you need to revise how to draw
linear sketch graphs. When graphing equations, make sure you use a consistent and
accurate scale.

WORKED Example 12
Use a graphics calculator to solve the following simultaneous equations graphically and
sketch the screen from which the solution was obtained.
y = −3x + 5
4x − 7y + 8 = 0
THINK WRITE/DISPLAY
1 Convert the second equation into y = form so it 4x – 7y + 8 = 0
may be entered in a graphics calculator. 4x + 8 = 7y
7y = 4x + 8
4 8
y = --- x + ---
7 7
2 Enter the two equations in a graphics calculator
so that a graph can be produced showing both
lines on the same set of axes. Find the point of
intersection of the two lines.
For the Casio fx-9860G AU
1. Press MENU and then select GRAPH.
Enter −3x + 5 for Y1 and then press EXE .
4 8
Enter --- x + --- for Y2 and then press EXE . <Casio01.18>
7 7
(Use the ab/c key to enter the fractions.)
Chapter 1 Modelling using linear functions 35

THINK WRITE/DISPLAY
2. To make the second line appear different to the
first, highlight the Y2 line (press the up arrow)
and then press F4 (STYL). There are 3 other
line styles to choose from. <Casio01.19>

3. Press F2 to obtain a bold line. Press EXIT


to show the Graph Function options along the
bottom of the screen. Press F6 (DRAW) to
display the graph. Adjust the View Window if <Casio01.20>
necessary. (In this example, Xmin is −6, Xmax
is 6, Ymin is −6 and Ymax is 6.)

4. To find the point of intersection of the two lines,


press SHIFT F5 (G-SLV) and then F5
(ISCT). The point of intersection and its
coordinates are displayed. The coordinates are
<Casio01.25>
(1.08, 1.76).

For the TI-Nspire CAS


1. Open a new Graphs & Geometry document.
Enter −3x + 5 for f1(x) in the function entry line
and then press · to display the graph of the <TI01.25>
first equation.

4 8
2. Enter --- x + --- for f2(x) in the function entry
7 7
line. (Press /p to display the fraction
template.) Press · to display this graph on the
<TI01.26>
same set of axes.

3. Adjust the Window Settings if necessary. (In


this example, XMin is −6, XMax is 6, YMin is −6
and YMax is 6.) Press · to display the graph
with the new settings. <TI01.27>

Continued over page


36 M a t h s Q u e s t M a t h s B Ye a r 1 1 f o r Q u e e n s l a n d

THINK WRITE/DISPLAY
4. To find the point of intersection, press b and
then select 6: Points & Lines followed by
3: Intersection Point(s). Use the NavPad to
move the pointer to one of the lines. A graph <TI01.27>
label will appear and the line will begin
flashing.

5. Press Click (x) to select the line, then move the


pointer and select the second line. The
coordinates of the point of intersection will
appear. (If necessary, move any equation labels <TI01.27>
out of the way.) The coordinates are (1.08,
1.76).

3 Draw a sketch of the solution screen. y


5
4x – 7y + 8 = 0
<insert figure 1.26>
8
— (1.08, 1.76)
7

–2 0 5
x

3
y = –3x + 5

4 State the solution. The solution to the simultaneous


equations y = −3x + 5 and 4x − 7y + 8 = 0
is x = 1.08 and y = 1.76 or (1.08, 1.76).

Algebraic solution
It is possible to solve simultaneous equations without graphs, that is, algebraically. The
methods of substitution and elimination taught in earlier years may be used.

WORKED Example 13
Use the substitution method to solve the following simultaneously.
−2x + 7y = 8
y = 3x − 1
THINK WRITE
1 Write down and label the equations. −2x + 7y = 8 [1]
y = 3x − 1 [2]
2 Substitute [2] into [1] and label the Substituting [2] into [1]:
resulting equation. −2x + 7(3x − 1) = 8 [3]
Chapter 1 Modelling using linear functions 37
THINK WRITE
3 Solve [3] for x and label the solution as −2x + 21x − 7 = 8
equation [4]. 19x − 7 = 8
19x = 15
x = 15
------
19
[4]
4 Use the solution to solve for y. Substituting [4] into [2]:
y = 3  15
------ − 1
 19
45
= ------ −1
19
45 19
= ------ − ------
19 19
26 7
= ------ or 1 -----
- [5]
19 19

5 State the complete answer. Solution:  15 26


------ , ------ 
19 19 
6 Optional double-check: substitute Check: In [1],
equations [4] and [5] into [1] to check LHS = −2x + 7y
that these values for x and y make [1] = −2  15 26
------ + 7  ------
  
19 19
true.
= − 30 ------ + 182
---------
19 19
152
= ---------
19
=8
= RHS ✓

WORKED Example 14
Use the elimination method to solve these simultaneous equations.
2x + 9y = −5
5x − 2y − 12 = 0
THINK WRITE
1 Write down and label the equations. 2x + 9y = −5 [1]
5x − 2y − 12 = 0 [2]
2 Rearrange [2] so it is in a similar form to 5x − 2y = 12 [3]
[1]. Call this [3]. Write down [1] again. 2x + 9y = −5 [1]
3 Obtain 10x in both [1] and [3] as 2 × [3]: 10x − 4y = 24 [4]
explained opposite. 5 × [1]: 10x + 45y = −25 [5]
4 Eliminate x as shown. [5] − [4]: 49y = −49
49
5 Solve for y. y = − ------
49
y = −1 [6]
6 Substitute [6] into [1] to find x. Substituting [6] into [1]:
2x + 9 (−1) = −5
2x − 9 = −5
2x = −5 + 9
2x = 4
x=2 [7]

Continued over page


38 M a t h s Q u e s t M a t h s B Ye a r 1 1 f o r Q u e e n s l a n d

THINK WRITE
7 State the solution. Solution: (2, −1)
8 Again, [6] and [7] may be checked in Check: In [2],
[2] if desired. LHS = 5x − 2y − 12
LHS = 5(2) − 2(−1) − 12
LHS = 10 + 2 − 12
LHS = 0
LHS = RHS ✓

WORKED Example 15
Two shoppers buy the following at a fruit market, paying the amounts given. What was
the cost of each apple and each banana?
Shopper 1: 4 apples and 3 bananas for $5.18
Shopper 2: 6 apples and 5 bananas for $8.22
THINK WRITE
1 Decide on pronumeral names for the Let a = cost of an apple (in cents).
unknown quantities. Let b = cost of a banana (in cents).
2 Write equations involving these 4a + 3b = 518 [1]
pronumerals. Work in terms of cents. 6a + 5b = 822 [2]
3 Choose a pronumeral to eliminate. In 5 × [1]: 20a + 15b = 2590 [3]
this case b. 3 × [2]: 18a + 15b = 2466 [4]
4 Find [3] − [4] and solve for a. [3] − [4]: 2a = 124
a = 62 [5]
5 Solve for b. Substituting [5] into [1]:
4 × 31 + 3b = 518
124 + 3b = 518
3b = 270
b = 90 [6]
6 State the answer using [5] and [6] as a The cost of an apple is 62 cents, and the cost of
guide. a banana is 90 cents.

Graphics Calculator tip! Solving simultaneous


equations algebraically
Consider the following simultaneous equations solved graphically in Worked example 12:
y = −3x + 5 and 4x − 7y + 8 = 0
We can also solve these equations algebraically with a graphics calculator.
For the Casio fx-9860G AU
1. Press MENU and then select EQUA. Press F1
(SIML) to specify the type of equation as
Simultaneous.
<Casio01.22>
Chapter 1 Modelling using linear functions 39
2. Press F1 (2) for 2 unknowns. The resulting
screen will allow you to enter the two equations.

3. The equations need to be in the form ax + by = c. The


first equation can be rearranged to become 3x + y = 5
and the second to become 4x − 7y = −8. Enter the
numbers for a, b and c for each equation as shown in
the screen at right. Press EXE after each entry.

4. To find the simultaneous solution, press F1


(SOLV). You can use the arrow keys to move
between X and Y to see the corresponding value as
a fraction. The solution is (1.08, 1.76) or ( 27 44
------ , ------ ).
25 25

For the TI-Nspire CAS


1. Open a new Calculator document. Press b and
then select 3: Algebra followed by 1: Solve.
Enter the first equation y = −3x + 5.

2. Enter the word ‘and’. To enter the word ‘and’,


press k to access the catalog and then press 1
to display the function menu. Use the NavPad to
scroll down to and.

3. Press · to insert and into the Calculator


document. Now enter the equation 4x − 7y + 8 = 0.
Press , followed by X for the variable. Press )
to close the set of brackets and then press · to
display the solution.

4. Press /· to display the solution in decimal


notation. The solution is (1.08, 1.76) or ( 27
------ ,
44
------ ).
25 25
40 M a t h s Q u e s t M a t h s B Ye a r 1 1 f o r Q u e e n s l a n d

remember
1. If form is y = ax + b, y = cx + d consider using substitution.
2. If form is ax + by = c, dx + ey = f consider using elimination.

1F Simultaneous equations

WORKED 1 Use a graphics calculator to solve the following simultaneous equations graphically,
Example
12
and sketch the screen from which the solution was obtained.
a y = −2x, y = −4x − 6 b y = 4x, y = 3x − 5 eBook plus
c y = 3x − 5, y = 20 d y = −2x − 4, y = −5x + 5
e y = 3x + 5, y = 7x − 4 f y = −5x + 12, y = 2x − 1 Digital docs:
EXCEL Spreadsheet
g −3x + y = −4, y = 6x + 5 h 2x − y − 11 = 0, y = −4x + 8 Simultaneous
i y = 10x + 1, 2x + y = −6 j 9x + y = 17, x + y = 14 equations —
graphical method

WORKED 2 Use the method of substitution to solve the following simultaneously. EXCEL Spreadsheet
Example Simultaneous
a y = 3x + 1, y = 2x + 2 b y = −3x, y = 4x + 14 equations —
13
c y = 5x + 5, y = −x − 19 d y = −4x − 3, y = 3x − 24 algebraic method

e y = x + 2, 3x − 4y = −1 f y = 3x − 6, 2x + y = 9
g y = −2x + 3, −5x + 2y = 1 h 6x − y = 8, y = x + 4
i −4x − 3y = 2, y = −6x + 7 j y = 10 − x, 2x + 7y = 5
WORKED
Example 3 Use the elimination method to solve these simultaneous equations.
14 a 9x + 10y = 153, 3x − y = 12 b 2x + 4y = 8, 7x + y = −37
c 7x − 11y = −13, x + y = 11 d 6x − 9y = 51, −6x + 11y = −49
e 6x − 2y = 10, 2x + 5y = −8 f 2x + 7y = 16, 3x − 6y = 2
g −3x + y = 8, 4x + 2y = 21 h 8x + 3y = −9, 4x − y = −3
i 7y − x = 11, x + y = 10 j x − 11y = −15, y + 6x = 9
WORKED 4 At the conclusion of a tour of Wonky Willy’s confectionery factory, Nutrina buys
Example
15
10 choc balls and 8 fizz wizzers for $4.30, and her friend purchases 6 choc balls and
9 fizz wizzers for $4.05. Determine the cost of each type of lolly.
5 The sum of two whole numbers, x and y, is 41. The difference between them is 3.
Write two equations involving x and y and solve them to find the numbers.
6 A farmer counts emus and
cattle in a paddock, and notes
there are 57 animals and
196 feet. Assuming no animal
amputees, how many of each
animal are there?
7 A sports store supplies 24
basketballs and 16 cricket balls
to one school for $275.60, and
delivers 12 basketballs and 32
cricket balls to another school
for $211. If delivery is free,
how much did the supplier
charge for each type of ball?
Chapter 1 Modelling using linear functions 41
8 A businessperson hires a stretch limousine for 2 days and a sedan for 3 days while on
an interstate trip. If the total car hire cost of the trip was $675, and the limousine cost
triple the price of the sedan, find the cost per day of the limousine.

9 multiple choice
A manufacturing plant produces fixed size square and circular metal panels. If the
mass of a square panel is 13 kg and that of a circular panel is 22 kg, how many of
each panel are there in a truck loaded with 65 panels of total mass 1205 kg?
The equations to solve are:
A 13s + 22c = 1205, s + c = 65 B 22s + 13c = 1205, s + c = 65
C 13s + 22c = 65, s + c = 1205 D 22s + 13c = 65, s + c = 1205
E 13s + 22c = 1205, s + c = 35

10 multiple choice
Which of the following is a solution of 11x + 2y = −121 and 10x + 12y = −222?
A (11, 2) B (−121, −222) C (10, 12) D (−9, −11) E (6, 10)

Formula for finding the equation of a


straight line
Consider a general linear graph containing the particular y (x2, y2)
points (x1, y1), (x2, y2) and the general point (x, y) (which (x, y)
could be any point).
Using the first two of these points in the formula for (x1, y1)
gradient, we have
y2 – y1 x
m = ---------------
- [1]
x2 – x1
Using the first and last point in the same formula yields
y–y
m = -------------1- [2]
x – x1
y–y y2 – y1
Putting [2] = [1] gives -------------1- = ---------------
- which may be rearranged to
x – x1 x2 – x1

y 2 – y 1
y – y 1 =  ---------------
- ( x – x1 ) [3]
 x 2 – x 1
y2 – y1
Since m = ---------------
- , equation [3] may be written
x2 – x1
y – y1 = m ( x – x1 ) [4]
This last formula may be used to find the equation of a straight line when two points
are given or when the gradient and only one point are given. When two points are
y2 – y1
given, the gradient m may first be found using m = ---------------
- , and substituted into the
x2 – x1
formula y – y 1 = m ( x – x 1 ) along with one of the points.
42 M a t h s Q u e s t M a t h s B Ye a r 1 1 f o r Q u e e n s l a n d

WORKED Example 16
Find the equation of the line having gradient 3--4- , that passes through (7, 11).
Express your answer in the form i ax + by + c = 0 and ii y = mx + c.
THINK WRITE
1 As one point and the gradient are known, use y − y1 = m(x − x1)
the formula y − y1 = m(x − x1).
3
2 List the given information. m= --- (x1, y1)
4
(7, 11)
3 Substitute for all pronumerals except x and y. y − 11 = 3--- (x − 7)
4
4 Simplify, expressing in the form i 4y − 44 = 3(x − 7)
ax + by + c = 0. 4y − 44 = 3x − 21
3x − 4y + 23 = 0
5 Express your answer in the form ii 3x + 23 = 4y
y = mx + c. 3 23
y = --- x + ------
4 4

WORKED Example 17
Find the equation of the straight line containing the points (2, −5) and (−3, 1).
Express your answer in the form i ax + by + c = 0 and ii y = mx + c.
THINK WRITE
1 Write the points so they match the (x1, y1) (x2, y2)
pronumerals in the formula. (2, −5) (−3, 1)
y2 – y1
2 As two points are known, first use the m = ---------------
-
x2 – x1
y2 – y1
formula m = ---------------
- to find m. 1 – –5
x2 – x1 = ---------------
−3 – 2
6
= ------
–5
6
= – ---
5
3 Write the formula y − y1 = m(x − x1). y − y1 = m(x − x1)
6 6
4 Substitute the calculated gradient m = – --- y − −5 = – --- ( x – 2 )
5 5
and the first point (x1, y1) = (2, −5). Leave x
and y as they are.
6
5 Simplify and express in the two forms y + 5 = ------ (x − 2)
required. –5
−5y − 25 = 6(x − 2)
−5y − 25 = 6x − 12
−5y = 6x − +13
i So 6x + 5y + 13 = 0, or
6 13
ii y = – --- x – ------
5 5
Chapter 1 Modelling using linear functions 43

Graphics Calculator tip! Finding the equation of a line


through two given points
Consider the points (2, −5) and (−3, 1) given in Worked example 17. We can use a
graphics calculator to find the equation of the straight line containing these points.
For the Casio fx-9860G AU
1. Press MENU and then select STAT.

2. Enter the x-values in List 1 and the y-values in


List 2. Press EXE after each entry.

3. To draw a line through the points, press F1


(GRPH) and then F6 (SET). Highlight Graph
Type and then press F2 (xy). (If you wanted to
plot the points only, you would press F1 (Scat)
to produce a scatterplot.) Ensure that XList shows
List 1 and YList shows List 2.
4. Press EXIT to return to the List screen. Press
F4 (SEL) and ensure that StatGraph1 shows
DrawOn and the other graphs show DrawOff.

5. Press F6 (DRAW) to display the points


connected by a line.

6. To find the equation of the line, press F1


(CALC) and then F2 (X) for linear regression.
The equation is in the form y = ax + b where
a = −1.2 and b = −2.6 for this line. Therefore, the
equation of the line is y = −1.2x − 2.6.

For the TI-Nspire CAS


1. Open a new Lists & Spreadsheet document
(press /N and then select 3: Add Lists &
Spreadsheet). Enter a title for column A (type
xvals and then press ·). Similarly, enter a title
for column B. Enter the x-values of the two points
in column A and the corresponding y-values of the
two points in column B. Press · after each entry.
44 M a t h s Q u e s t M a t h s B Ye a r 1 1 f o r Q u e e n s l a n d

2. Insert a Graphs & Geometry screen (press /I


and then select 2: Add Graphs & Geometry).
Press b and then select 3: Graph Type.
Different graph options will be displayed.

3. Since we want to graph the points entered in the


Lists & Spreadsheet document, select 4: Scatter
Plot. The function entry line will now show one
field box for x and one for y. With the field box
highlighted for x, press · to display the options
you have to choose from. The two options are
xvals and yvals and refer to the titles of the
columns in the Lists & Spreadsheet document.

4. Press · to select xvals for the x field box. Press


e to move to the y field box and then press ·
to display the options. Use the NavPad to
highlight the second option and press · to select
yvals. Press · to display the two points on the
graph. (Adjust the Window Settings if necessary.
Here, YMin is –10 and YMax is 6.)

5. Press b and then select 6: Points & Lines


followed by 4: Line. Use the NavPad to move the
pencil symbol (✐) to one of the points. A label for
point will appear and the point will begin to flash.
Press ·. Move to the second point and then
press ·. A line will be drawn through the two
points. Press d.

6. To find the equation of the line, press b and then


select 1: Actions followed by 7: Coordinates
and Equations. Use the NavPad to move the
pointer to the line. A label for line will appear and
the line will begin to flash.

7. Press · and then x to display the equation of the


line. The equation of the line is y = –1.2x – 2.6.
Chapter 1 Modelling using linear functions 45
remember
1. If given a point and the gradient, use y − y1 = m(x − x1).
y 2 – y 1
2. If given two points, use y – y 1 =  ---------------- ( x – x1 )
 x 2 – x 1
or
y–y
first find m using m = -------------1- , then use y − y1 = m(x − x1).
x – x1

Formula for finding the


1G equation of a straight line
WORKED 1 Find the equation of a straight line passing through the point listed and having the
Example
16
given gradient. Express your answer in the form i ax + by + c = 0 and ii y = mx + c.
a (1, 2) gradient 3 b (5, 6) gradient 2
eBook plus
c (4, 1) gradient 5 d (−1, 7) gradient 4
e (3, −2) gradient −1 f (0, −5) gradient −3
Digital doc: g (−3, 2) gradient 1
--- h (9, 0) gradient − 4---
EXCEL Spreadsheet 2 3
Equation of a 4
straight line i (−6, −1) gradient --- j (12, 8) gradient − 1---
5 6
8 3
k (−4, 4) gradient --- l (0, 3) gradient − ------
7 11

WORKED 2 Determine the equation of the line containing each pair of points. Express your
Example
17
answers in the form i ax + by + c = 0 and ii y = mx + c.
a (5, 2) (3, 1) b (1, 1) (5, 5)
c (6, 3) (8, 2) d (2, −2) (0, 1)
e (−5, 8) (−1, −4) f (9, −5) (7, −3)
g (−1, −4) (2, 10) h (4, 0) (0, −3)
i (0, −6) (−14, 2) j (−7, 8) (8, −7)

3 multiple choice
A particular line has a gradient of 3--- and passes through the point (−1, 4). The
4
equation consistent with this information is:
A y = 4 + 3--- (x + 1) B y + 4 = 3--- (x − 1) C y − 4 = 3--- (x − 1)
4 4 4
D y = −4 + 4--- (x + 1) E y = 4 − 4--- (x − 1)
3 3

4 multiple choice
A line passes through the points (3, 4) and (−5, 6). An appropriate matching of pro-
numerals and values could be:
A x1 = 3, y1 = −5, x2 = 4, y2 = 6 B x1 = 4, y1 = 3, x2 = 6, y2 = −5
C x1 = 3, y1 = 4, x2 = −5, y2 = 6 D x1 = 3, y1 = 6, x2 = 4, y2 = −5
E x1 = −5, y1 = 4, x2 = 6, y2 = 3
5 Find the equation of the line passing through (3, −3) that makes an angle of 45° with
the positive x-axis.
46 M a t h s Q u e s t M a t h s B Ye a r 1 1 f o r Q u e e n s l a n d

6 Find the equation of the line containing (7, −2) that makes an angle of 71.565° with
the positive x-axis.

7 multiple choice
To determine the formula of a straight line, you need to know:
A one point only (that is, a particular x-value and y-value)
B the gradient only
C a point and the gradient
D an x-value and the gradient
E the x- and y-values for one point, and the x- or y-value for another point
8 Find the equation of the line (in y = mx + c form) that is:
a parallel to the line with equation y = 2--- x − 9, passing through (4, −7)
5
b parallel to the line with equation 3x + 6y = 8, passing through (2, 2)
c having gradient 2, passing through the intersection of the lines with equations
y = 3x − 5 and y = −2x + 5
d having gradient − 3--- , passing through the intersection of the lines with equations
4
x + 4y = −14 and −5x + 2y = 4.
9 Find the equation of the line which passes through the point of intersection of the
lines whose equations are 7x − 3y − 19 = 0 and 3x + 2y + 5 = 0, given that the required
line is parallel to the line with equation −5x − 2y = 3.
10 Find the equation of a line containing the intersection of the lines with equations
y = −3x + 4 and 5x − 3y + 40 = 0 which:
a has a gradient of 6---
7
b passes through the point (−1, 9)
c is parallel with the line joining (−8, 5) and (0, 4).
11 A line passes through the points (−8, −5), (4, −3) and (a, 12). Find the value of a.
12 A factory produces 25 components at a cost of $830, and on another run produces
35 components at a cost of $1050. Find an expression relating the cost, C, to the
number of components, n,
produced, assuming a
linear relationship.
13 The height of a par-
ticular young pine
tree is found to
increase in a linear
manner each week
in the first year
after planting.
Find an equation 52 cm
connecting
height with
time in 34 cm
months after
eBook plus planting,
using the
Digital doc: information
WorkSHEET 1.2
supplied.
After 2 months After 5 months
Chapter 1 Modelling using linear functions 47
Linear modelling
Many real life applications such as fees charged for services, cost of manufacturing or
running a business, patterns in nature, sporting records and so on follow linear relation-
ships. These relationships may take the form of a linear equation; for example,
F = 50 + 30t may be used by a tradesperson to calculate her fee for t hours of work.
Here, F is the fee in dollars, and t the time in hours. The 50 represents an initial
fee for simply turning up, while the 30t is the amount charged for the time spent on
the job.
For example, if t = 2 hours, 30t = 60, so the total charge for the work would be
$(50 + 60) = $110.
Equations like F = 50 + 30t are sometimes referred to as ‘linear models’, a common
form of which is
Total cost = Fixed cost + Cost per unit × Number of units.

This is, of course, similar to y = mx + c, or y = c + mx.

The Garn Express crisis — some answers

Now, let us return to the problem of the Garn Express and the Moonlander travelling
towards each other on the same line.
600 Garn Express
500 Moonlander
Brampton (km)
Distance from

400
300
200
100
0
0 50 100 150 200 250
Time (min)
Using the methods developed earlier, we find the equations which model the trains’
position:
dG = 550 − 1.56t
dM = 1.42t − 48.2
where dM and dG are the distance from Brampton of the Moonlander and the Garn
Express respectively.
The gradients of the lines represent the speed of the trains. If these can vary by 8%,
what can happen?
Recall that a disaster will be averted if the Garn Express can reach Alberton (the
dotted line) before meeting the second train.
If their speeds vary, the worst case scenario would be:
• The Garn Express decreases in speed by 8%.
• The Moonlander increases in speed by 8%.
48 M a t h s Q u e s t M a t h s B Ye a r 1 1 f o r Q u e e n s l a n d

Let us analyse this situation.


New speed of Garn Express = −1.56 × 0.92 = −1.44 km/min
New speed of Moonlander = 1.42 × 1.08 = 1.53 km/min
Thus the position of each of the trains is now modelled by the equations:
dG = 550 − 1.44t
dM = 1.53t − 48.2.
Using the techniques developed earlier, we find the simultaneous solution is:
t = 201 min, d = 260 km.
In this worst case situation the trains — on the same track — meet 260 km from
Brampton. That is, 10 km from Alberton and before the Garn Express can reach safety.
In this example you can see how the techniques we have developed to link graphs
and equations can provide powerful tools for investigating linear models.

WORKED Example 18
A generator company charges a $200 delivery fee, and a rental fee of $1500 per day.
a Find an expression relating total charge to the number of days for which the generator
is hired.
b Sketch a graph of the relationship.
c What would be the charge for 4 weeks of rental?

THINK WRITE
a 1 Define convenient pronumerals. a Let T = Total charge (in dollars) and
n = number of days the generator is hired.
2 The fixed cost is $200, and the cost T = 200 + 1500n
per unit is $1500.
(c = 200, m = 1500)

b 1 Find the y-intercept (when n = 0). b If n = 0, T = 200


2 The total cost rises $1500 each day, T ($)
so the graph must show this. 3200

1700

200

1 2 n (Days)

c 1 After 4 weeks, n = 28. Substitute this c If n = 28,


into the equation from part a. T = 200 + 1500 × 28
= 200 + 42 000
= 42 200
2 Write the answer in words. After 4 weeks, the total cost is $42 200.
Chapter 1 Modelling using linear functions 49
WORKED Example 19
‘Rent-a-Chef’ provides food cooked and served by a qualified chef at parties. The
company charges $120 as a booking fee, and an additional $30 per hour. Another
company, ‘Greased lightning’, provides fast food served by two students at a cost of
$65 per hour, with no booking fee. Under what conditions would it be cheaper to hire
‘Greased lightning’?
THINK WRITE
1 Define convenient pronumerals. Let C = Cost (total) in dollars and t = time
in hours.
2 Write an equation for the cost of hiring Rent-a-Chef C = 120 + 30t [1]
both organisations. Greased lightning C = 65t [2]
3 Use simultaneous equations to find when For [1] = [2], 120 + 30t = 65t
the cost is the same with each group. 120 = 35t
120
t = ---------
35
= 3.4 hours
4 At 3.4 hours, the cost is the same. Since It is cheaper to hire Greased lightning for
Greased lightning has the higher per hour food preparation and service of less than 3.4
cost, after 3.4 hours, they would be more hours (3 hours and 24 minutes) duration.
expensive.
Notes: 1. 0.4 hours = 0.4 × 60 minutes = 24 minutes.
2. An alternative approach would be to use a graphics calculator.
Graphical approach
Casio fx-9860G AU TI-Nspire CAS

<Casio01.32> <TI.01.41>

Algebraic approach
Casio fx-9860G AU TI-Nspire CAS

<TI01.42>
<Casio01.33>

remember
Though not all modelling questions involve costs, it is helpful to remember:

Total cost = Fixed cost + Cost per unit × Number of units.

y = c + mx
50 M a t h s Q u e s t M a t h s B Ye a r 1 1 f o r Q u e e n s l a n d

1H Linear modelling

WORKED 1 The cost of hiring a tennis


Hire Ch
Example
court consists of a booking
18
fee and an hourly rate. Bookin arges
Hourly g fee $5
rate
a Use the photo to write $10
eBook plus an equation for the total
hire cost in terms of the
Digital doc: hourly rate.
EXCEL Spreadsheet
Simultaneous linear b Sketch a graph of the
equations
relationship.
c What would be the
charge for 3 hours?

2 A singing telegram service charges a $60 appearance fee, and $8 per minute sung.
a Write an equation for the total cost of a singing telegram in terms of the number
of minutes sung.
b Sketch a graph of the relationship.
c What would be the charge for a 5-minute singing telegram?
3 Colleen delivers junk mail. She is paid $32 to traverse a particular route and a further
10 cents per leaflet delivered.
a Write an equation for the total payment she receives.
b Sketch a graph of the relationship expressed in a.
c What would be Colleen’s pay if she delivers 1650 leaflets along the route?
4 A pay-TV salesperson receives $300 per week plus $20 for every household he signs
up to have pay-TV connected. How much does the salesperson receive for a week in
which he signs up 33 households?
WORKED 5 A computer firm, SuperComputers Inc., offers a back-up plan covering the ongoing
Example
19
service and troubleshooting of its systems after sale. The cost of signing up for the
service plan is $215, and there is an hourly rate of $65 for the serviceperson’s time.
Purchasers not signing up for the plan are charged a flat rate of $150 per hour for
service. Would it be advisable to sign up for the service plan if you expected to need
3 hours of service assistance during the life of a computer purchased from
SuperComputers Inc?
6 A telephone company, Opus, offers calls to Biddelonia for a connection fee of $14,
and thereafter $1 per minute. Its rival, Belecom, offers calls for $2 per minute (no
connection fee) to the same country.
a Compare the cost of a 10 minute call to Biddelonia using each company.
b At what point would it be cheaper to use Opus?
7 It costs you $6 to get into a taxi (the ‘flagfall’), and $1.50 per kilometre if you use
‘PinkCabs’, while NoTop taxis charge $8 flagfall, and $1.20 per kilometre.
a How much would it cost with each company to travel 15 km in one of its cabs?
b When would it cost the same to use both companies?
Chapter 1 Modelling using linear functions 51
8 Two amusement parks show the following
information for school-age tourists in a
promotional brochure.
After how many rides does an excursion
to Fun World become the cheaper option
for the same number of rides?

rW
Wate orld
$8.00 entry
$2.50 per ride

$12 entry
$1.50 per ride

9 Medirank, a health insurance company, charges $860 per year (for a single person),
and requires customers to pay the first $100 of any hospital visit. HAB, on the other
hand, charges an annual fee of $560 and requires its members to pay the first $150 of
any hospital visit. Determine the number of hospital visits in a year for which the cost
of health services is the same whichever company insures you.

10 Nifty is a car hire firm that charges insurance of $135, and $50 per day car hire. A
competitor, Savus, simply charges $65 per day and offers ‘free’ insurance. You are
planning a holiday and would prefer to use Savus. Under what conditions (days hired)
could you justify this choice?
52 M a t h s Q u e s t M a t h s B Ye a r 1 1 f o r Q u e e n s l a n d

summary
Rearrangement and substitution
• Do the same to both sides and remember inverse operations including + and −,
× and ÷, and 2.
• ‘Make x the subject’ means manipulate into the form ‘x = . . .’.
• ‘Substitute’ means to replace a pronumeral with a value.
Solving linear equations
• Do the same to both sides and remember inverse operations including + and −,
× and ÷, and 2.
• Aim to get a single pronumeral by itself.
Gradient of a straight line
y2 – y1
• m = ---------------
- where (x1, y1) and (x2, y2) are points on the line.
x2 – x1
• m = tan θ where θ is the angle the line makes with the positive direction of the
x-axis.
Equations of the form y = mx + c
• The general equation for a straight line of gradient m and y-intercept c is
y = mx + c.
• Lines with the same gradient (m) are parallel.
Sketching linear graphs using intercepts
• To find the y-intercept, let x = 0, and find y.
• To find the x-intercept, let y = 0, and find x.
• If y = 0 when x = 0, the line passes through the origin (0, 0). Substitute another
x-value (for example, x = 1) to find another point on the line.
• Rule a straight line through the two points and/or intercepts.
Simultaneous equations
• y = ax + b, y = cx + d consider using substitution
• ax + by = c, dx + ey = f consider using elimination
Formulas for finding the equation of a straight line
y2 – y1 
• y = mx + c y – y 1 =  ----------------
- ( x – x1 ) y – y1 = m ( x – x1 )
 x 2 – x 1
Linear modelling
• Total cost = Fixed cost + Cost per unit × Number of units
• y = c + mx
Chapter 1 Modelling using linear functions 53

CHAPTER
review
1 multiple choice
1A
7x – 23
The first step in solving ------------------ = 99 would be to:
3
A add 23 to both sides B divide both sides by 3 C divide both sides by 7
D multiply both sides by 3 E multiply both sides by 7

2 multiple choice
x = −5 is a solution to the equation:
x + 25
1A
A 3x + 7 = – 8 B 2x – 17 = – 5 C --------------- = 5
6
– 5x
D 2 ( x + 3 ) = 10 E --------- = 45
9

3 ( 5x – 4 ) 6 ( 4x + 3 )
3 Solve the equation ----------------------- = ----------------------- .
7 5 1A
7x + 8
4 Find the value of x where 3  --------------- = 4x – 9 .
 10  1A
5 multiple choice
1B
When c2 = a2 + b2 is rearranged to make a the subject, the equation becomes:
A c = a2 + b2 B a2 = b2 + c2 C a2 = c2 – b2
D a = c2 – b2 E a = b+c

6 multiple choice
1B
1
Which values, when substituted into K = --- mv 2 , give a value for K of 4?
2
1
A m = 2, v = 4 B m = 4, v = 2 C m = 8, v = ---
2
D m = 8, v = 1 E m = 1, v = 16

7 multiple choice
Using the equation P = m1 v1 + m2 v2, if P = 10, m1 = 2, m2 = 6 and v1 = 4, v2 would equal:
A 1
--- B 1
--- C 1
1B
3 2
D 2 E 3
8 The following formula may be used to study planetary motion.
GmM m4 π 2 R
1B
-------------- = ------------------
R2 T2
Make T the subject of the equation.
54 M a t h s Q u e s t M a t h s B Ye a r 1 1 f o r Q u e e n s l a n d

9 multiple choice
1C y
The line shown has a gradient of:
6
A −6 B −3
C −2 D 2
6
E --
-
3
3 x
10 multiple choice
1C
The gradient of the line shown below is 3. The value of a must be:
A −2 y
B −1 (6, a)
C 5
D 7
x
E 11

11 multiple choice
1C (2, –5)
The gradient of the line joining (−1, 0) and (4, −10) is:
A −4 B −2 C 2 D 4 E 5

12 multiple choice
1C
Which of the graphs below has a gradient of 6--- ?
7
A y B y Cy D y E y
7 x x
6 –7 6
6

7 x –6 –7
–7 x
–6 x

13 Calculate the gradient of each of the following lines.


1C a y b y
6
6 5
4
3
2
1
–5 –4 –3 –2 –1 0 1 2 3 4 5 6x
x –1
–8 –2
–3

c y d y
1 grid
square = 1 unit x

(–12, –3)

(–4, –10)

1C 14 Find the gradient of the line joining (−7, 15) and (2, −6).
Chapter 1 Modelling using linear functions 55
15 Find the gradient of the line shown. y
1C

77°
x y

16 State the gradient of the line at right.


1C
5 x

17 multiple choice
1D
The gradient of the line with equation y = 6--- x − 1 is:
7
6 7
A −1 B --- C --- D 6 E 7
7 6

18 multiple choice
2
1D
The y-intercept of the line with equation y = 12x + --- is:
3
2 3
A --- B --- C 2 D 3 E 12
3 2

19 multiple choice
1D
The gradient and y-intercept (in that order) of the line with equation 2x − 3y = 7 are:
A 2 and −3 B 2 and 7 C − 2--- and 7 D 2
--- and − 7--- E −3 and 7
---
3 3 3 2

20 State the gradient and y-intercept (in that order) for each of the following.
a y = 3x − 7 b 5x + 3y = 30 c 2x − 4y − 8 = 0
1D
2
21 Find the equation for a linear graph having gradient and y-intercept −3.
---
5 1D
22 multiple choice
1E
Which of the following could be the graph of y = 2x + c?

A y B y C y

c
x c x

–c x

D y E y

c
–c x

x
56 M a t h s Q u e s t M a t h s B Ye a r 1 1 f o r Q u e e n s l a n d

23 multiple choice
1E y
The equation of this line is:
A 2x − 5y = 1
B 2x − y = 4 2 x
C 15x + 6y = −30
D 10y − x = −2
–5
x y
E --- – --- = 1
2 5
24 Sketch graphs of the following showing intercepts.
1E a y = −3x + 24 b −x + 8y = 40 c 9x − 7y − 63 = 0 d y + 6x = 0

25 multiple choice
1F
To solve the equations 2x + y = 5 and 3x − 6y = 12 graphically on a graphics calculator, you
would enter the following in the Y= menu:
A Y1=2X+Y and Y2=3X−6Y
B Y1=5 and Y2=12
C Y1=5−2X and Y2=12+6Y
D Y1=2X+5 and Y2=3X+12
E Y1=−2X+5 and Y2=1/2X−2

26 multiple choice
1F
Which of the following would be an effective way to solve the following equations
simultaneously?
y = 2x − 13 [1]
y = 7x + 2 [2]
A Multiply [1] by 2 and [2] by 13 and add the newly formed equations.
B Multiply [1] by 7, and put it equal to [2].
C Multiply [2] by 2 and put it equal to [1].
D Multiply [1] by 2 and [2] by 7 and subtract the newly formed equations.
E Put [1] equal to [2].
27 Solve graphically (using a graphics calculator if available): y = 3x + 10 and y = −2x − 15.
1F a Sketch the solution on a set of axes.
b State the solution (point of intersection).
28 Solve y = −3x, y = 6x − 15 using substitution.
1F
29 Use the method of elimination to solve 4x − 7y = 21, −2x + y = 6.
1F
30 A piggybank contains 67 coins. If there are only one and two dollar coins in the piggybank,
1F and there are 25 more one dollar coins than two dollar coins, how many of each type are
there?

31 multiple choice
1G
The equation of the line containing (1, −2) and (2, −3) could be expressed as:
A y−2=x−1 B y+2=1−x C y+3=x−1
D x − 2y = −3 E 3x − 5y = 1
Chapter 1 Modelling using linear functions 57
32 multiple choice
A line with equation y − 7 = 5(x − 1) has:
1G
A gradient 5 and contains the point (7, 1)
B gradient −7 and contains the point (−1, −7)
C gradient 5 and contains the point (1, 7)
D gradient −5 and contains the point (1, −7)
E gradient −1 and contains the point (5, 7)
33 Find the equation of the line containing (−4, 8) and (3, 1).
1G
34 Find the equation of the line having gradient − --6- that passes through (1, 4).
7 1G
35 multiple choice
Bote lives 5 kilometres from the nearest post office. At noon one day he begins cycling
1H
(from home) at 20 kilometres per hour in a constant direction away from the post office. The
distance, D km, Bote is from the post office, t hours after he begins cycling, is given by:
A D = 5t B D = 20t C D = 5t + 20 D D = 20t + 5 E D = 20t − 5
36 The washing machine repair company ‘Washed out’ charges $75 to come to your house, as
well as an hourly charge of $65, calculated to the nearest half hour. 1H
a Write an equation that may be used to calculate the cost of any service call if the
time taken by the repairer is known.
b Sketch a graph of the relationship between repair cost and time taken to do a
repair.
c How much would it cost to have a repair done that takes 3 1--- hours?
2

Modelling and problem solving y


1 The diagram at right is a rough sketch of three points
on a section of sheet metal that are to be drilled by a 16 C
programmed robotic drilling arm. Any deviation from 12 B
a straight path, no matter how slight, means the arm
must be programmed for more than one direction. The 6 A
coordinates marked are correct. Will the robotic arm
be able to move in one direction only to drill all three
8 14 20 x
holes?
2 Points A, B and C have the
coordinates (1, 6), (0, 0) and (−2, 2).
Find 3 possible coordinates for a
point D so that the four points form a
parallelogram. (Exclude the case
where all points lie in a straight line.)
3 The cost of a parachuting course
consists of a charge of $250 which
covers equipment hire and tuition,
and a further expense of $55 per
jump.
a Express the total cost, C, as a
function of j, the number of
jumps.
58 M a t h s Q u e s t M a t h s B Ye a r 1 1 f o r Q u e e n s l a n d

b How many jumps could a person doing the course afford if she was prepared to spend up
to $1000?

4 A physics student conducts an experiment to find out how much a spring stretches when
various weights are hung from it. Her results are shown in the table below.

Length of spring (cm) Force applied (N)


4 0
7 10
12 20
16.5 30
20.5 40
25 50

a What is the natural or ‘unstretched’ length of the spring?


b Plot a graph of the student’s results.
c Draw a straight line through the points that best describes the data.
d Select two points on the line and use them to fit a linear equation to the line.
A second student conducts the same experiment on a similar spring. His results are shown
below.

Length of spring (cm) Force applied (N)


5 0
10 10
16 20
21 30
24 40
28 50

e On the same set of axes you used in part b, plot the results of the second experiment and
join the points with a line of best fit.
f Write an equation that describes the relationship between the force applied and the length
of the second spring.
The gradients of graphs such as the ones you have drawn give an indication of the stiffness
eBook plus of a spring. The greater the gradient, the harder it is to stretch the spring. The lower the
gradient, the easier it is to stretch the spring.
Digital doc: g Comment on the stiffness of the two springs investigated by the students.
Test Yourself
Chapter 1 h Is it likely that these two springs will ever be the same length at a given force, that is, is it
likely the intersection of the two graphs could ever really happen? Explain your answer.
Relations and
functions
2
syllabus reference
Topic:
• Introduction to functions

In this chapter
2A Relations and graphs
2B Domain and range
2C Types of relations (including
functions)
2D Function notation and
special types of functions
2E Inverse relations and
functions
2F Circles
2G Functions and modelling
60 M a t h s Q u e s t M a t h s B Ye a r 1 1 f o r Q u e e n s l a n d

Introduction
Mathematical models are used in a wide variety of contexts.

The amount of erosion varies with wave energy.

The amount of money invested in the stock market varies with Reserve Bank interest rates.
Chapter 2 Relations and functions 61

Climate modellers investigate links between deforestation and greenhouse warming.

A medical scientist investigates whether there are links between the incidence of cancer
and the presence of microwave radiation.
As mathematicians develop models they use the concept of a function.
• Erosion is a function of wave energy.
• Money invested is a function of interest rates.
• Greenhouse warming as a function of deforestation.
• The incidence of cancer is a function of microwave radiation.
In this chapter we will learn the language of functions. The tools used in modelling
will be presented in an abstract setting and will be used later in practical applications.
62 M a t h s Q u e s t M a t h s B Ye a r 1 1 f o r Q u e e n s l a n d

Relations and graphs


A relation is a set of ‘ordered pairs’ of values or ‘variables’. Consider the following.
The cost of hiring a boat depends on the number of hours for which it is hired. We
can say that a relation exists between the number of hours and the cost. The table below
outlines the relation.

Number of hours of hire 3 4 5 6 7 8


Cost ($) 50 60 70 80 90 100

Since the cost depends upon the number of hours, the cost is said to be the
dependent variable, while the number of hours is called the independent variable. The
information in the table can be represented by a graph, which usually gives a better
indication of how two variables are related. When graphing a relation, the independent
variable is displayed on the horizontal (or x) axis and the dependent variable is
displayed on the vertical (or y) axis. Now we can plot the set of points {(3, 50), (4, 60),
(5, 70), (6, 80), (7, 90), (8, 100)}. The points are called (x, y) ordered pairs, where x is
the first element and y is the second element.

Cost of boat hire ($)


100
90
80
70
60
50
40
0 1 2 3 4 5 6 7 8 x
Number of hours

This graph clearly shows that the cost increases as the number of hours of hire
increases. The relation appears to be linear. That is, a straight line could be drawn that
passes through every point. However, the dots are not joined as the relation involves
‘integer-valued’ numbers of hours and not minutes or seconds. The number of hours
can be referred to as a discrete independent variable.
Discrete variables include names and numbers of things; that is, things that can be
counted (values are natural numbers or integers).
Some variables are referred to as continuous variables. Continuous variables include
height, weight and volume; that is, things that can be measured (values are real
numbers). If a relationship exists between the variables we may try to find a rule and
then write this rule in mathematical terms. In our example, the relationship appears to
be that for each extra hour of hire the cost increases by $10 after an initial cost of $20.
Cost = 10 × number of hours + 20
Using x and y terms, this is written as
y = 10x + 20
Sometimes the variable x may only take certain values. For example, if x is the number
on the face of a six-sided die, x may only take the values 1, 2, 3, 4, 5 or 6. We write this
as x ∈ {1, 2, 3, 4, 5, 6} and read it as x is an element of the set {1, 2, 3, 4, 5, 6}.
Chapter 2 Relations and functions 63
Sets of numbers
The letters N, J and R stand for sets of useful numbers.
N stands for the set of Natural numbers; that is, N = {1, 2, 3, …}.
J stands for the set of integers; that is, J = {…, −3, −2, −1, 0, 1, 2, 3, …}.
R stands for the set of Real numbers; that is, all numbers you can think of.
Some other sets commonly used are R+ (the set of positive Real numbers), R− (the set
of negative Real numbers) and sets where a number is excluded such as R\{0} (the set
of Real numbers excluding 0).
(Note: A list of commonly used sets and set notation is shown in the Summary on
page 102. For more on sets and set notation, log into www.jacplus.com.au and locate
the weblinks for this chapter.)

WORKED Example 1
Sketch the graph by plotting selected x-values for the following relations and state whether
each is discrete or continuous.
a y = x2, where x ∈ {1, 2, 3, 4} b y = 2 x + 1, where x ∈ R
THINK WRITE
a 1 Use the rule to calculate y and state a When x = 1, y = 12
the ordered pairs by letting x = 1, 2, 3 =1 (1, 1)
and 4. x = 2, y = 22
=4 (2, 4)
x = 3, y = 32
=9 (3, 9)
x = 4, y = 42
= 16 (4, 16)
2 Plot the points (1, 1), (2, 4), (3, 9) and y
(4, 16) on a set of axes. 16
12
8
4
1
0 1 2 3 4 x

3 Do not join the points as x is a discrete It is a discrete relation as x can be only


variable (whole numbers only). whole number values.

b 1 Use the rule to calculate y. Select b When x = 0, y = 2(0) + 1


values of x, say x = 0, 1 and 2 = 1 (0, 1)
(or find the intercepts). State the x = 1, y = 2(1) + 1
ordered pairs. = 3 (1, 3)
x = 2, y = 2(2) + 1
= 5 (2, 5)

Continued over page


64 M a t h s Q u e s t M a t h s B Ye a r 1 1 f o r Q u e e n s l a n d

THINK WRITE
2 Plot the points (0, 1), (1, 3) and (2, 5) y y = 2x + 1
on a set of axes. 5
4
3
2
1
x
–2 –1 –10 1 2
–2
–3
3 Join the points with a straight line, It is a continuous relation as x can be any
continuing in both directions as x is a real number.
continuous variable (any real number).

WORKED Example 2
The pulse rate of an athlete, R beats per minute,
t minutes after the athlete finishes a workout, is
shown in the table below.

t 0 2 4 6 8

R 180 150 100 80 70

a Plot the points on a graph.


b Estimate the athlete’s pulse rate after 3 minutes.

THINK WRITE
a 1 Draw a set of axes with t on the horizontal
axis and R on the vertical axis because
heart rate is dependent on the time.
2 Plot the points given in the table.
b 1 Join the points with a smooth curve since b
t (time) is a continuous variable. 180
2 Construct a vertical line up from t = 3
160
R (beats/min)

140
until it touches the curve. 120
3 From this point draw a horizontal line
100
80
back to the vertical axis. 60
40
20
0 1 2 3 4 5 6 7 8
t (min)

4 Estimate the value of R where this line When t = 3, the pulse rate is
touches the axis. approximately 125 beats per minute.
Chapter 2 Relations and functions 65

Graphics Calculator tip! Plotting points


A graphics calculator can be used to plot points rather than draw a continuous graph
based on a known formula or rule. To plot the points (1, 3), (2, 5), (6, 6), (9, 6), follow
the instructions below.
For the Casio fx-9860G AU
1. Press MENU and then select STAT. Enter the
x-values for each point in List 1 and the
corresponding y-values in List 2. Press EXE <Casio02.01>
after each entry.

2. If you wish, you can enter a title for each list.


Use the arrow keys to highlight the SUB cell
and then enter an appropriate title. Use the
ALPHA key to enter each letter. Press EXE <Casio02.02>
after completing each title.

3. Press F1 (GRPH) and then F6 (SET).


Highlight Graph Type and then press F1
(Scat) (if it doesn’t already show this setting).
This will specify that the points need to be
<Casio02.03>
displayed as a scatterplot. Ensure that XList
shows List 1 and YList shows List 2.

4. Press EXIT to return to the List screen. Press


F4 (SEL) and ensure that StatGraph1 shows
DrawOn and the other graphs show DrawOff.
<Casio02.04>

5. Press F6 (DRAW) to display the points.

<Casio02.01>

6. To view the coordinates of the points, press


SHIFT F1 (TRCE) and then use the arrow
keys to move between each point.
<Casio02.06>
66 M a t h s Q u e s t M a t h s B Ye a r 1 1 f o r Q u e e n s l a n d

For the TI-Nspire CAS


1. Open a new Lists & Spreadsheet document
(press /N and then select 3: Add Lists &
Spreadsheet). Enter a title for column A (type
xvals and then press ·). Similarly, enter a title <TI02.01>
for column B. Enter the x-values of the points in
column A and the corresponding y-values in
column B. Press · after each entry.
2. Insert a Graphs & Geometry screen (press /I
and then select 2: Add Graphs & Geometry).
Press b and then select 3: Graph Type.
Different graph options will be displayed. <TI02.02>

3. Since we want to plot the points entered in the


Lists & Spreadsheet document, select 4: Scatter
Plot. The function entry line will now show one
field box for x and one for y. With the field box for <TI02.03>
x highlighted, press · to display the available
options. The two options are xvals and yvals and
refer to the titles of the columns in the Lists &
Spreadsheet document.
4. Press · to select xvals for the x field box. Press
e to move to the y field box and then press ·
to display the options. Use the NavPad to
highlight yvals and then press ·. The points will <TI02.04>
be displayed.

5. To view the coordinates of the points, press b


and then select 5: Trace followed by 1: Graph
Trace. Use the NavPad to move between points.
<TI02.05>

remember
1. The independent variable (for example, x) is shown on the horizontal axis of a
graph.
2. The dependent variable (for example, y) is shown on the vertical axis of a
graph.
3. Discrete variables are things which can be counted. Graph points are not
joined.
4. Continuous variables are things which can be measured. Graph points may be
joined.
Chapter 2 Relations and functions 67

2A Relations and graphs


Questions 1, 2, and 3 refer to the following information.
A particular relation is described by the following ordered pairs:
{(0, 4), (1, 3), (2, 2), (3, 1)}.
1 multiple choice
The graph of this relation is represented by:
A y B y C y
4 4 4
3 3 3
2 2 2
1 1 1

0 1 2 3 4 x 0 1 2 3 4 x 0 1 2 3 4 x
D y E y
4 4
3 3
2 2
1 1

0 1 2 3 4 x 0 1 2 3 4 x

2 multiple choice
The elements of the dependent variable are:
A {1, 2, 3, 4} B {1, 2, 3} C {0, 1, 2, 3, 4}
D {0, 1, 2, 3} E {1, 2}

3 multiple choice
The rule for the relation is correctly described by:
A y = 4 − x, x ∈ R B y = x − 4, x ∈ N C y = 4 − x, x ∈ N
D y = x − 4, x ∈ J E y = 4 − x, x ∈ {0, 1, 2, 3}

4 multiple choice
During one week, the number of people travelling on a particular train at a certain
time progressively increases from Monday through to Friday. Which graph below best
represents this information?
A B C
Number of people

Number of people

Number of people

0 M T W T F 0 M T W T F 0 M T W T F
68 M a t h s Q u e s t M a t h s B Ye a r 1 1 f o r Q u e e n s l a n d

D E

Number of people

Number of people
0 M T W T F 0 M T W T F

5 State whether each of the following relations has discrete (D) or continuous (C)
variables.
a {(–4, 4), (–3, 2), (–2, 0), (–1, –2), (0, 0), (1, 2), (2, 4)}
b The relation which shows the air pressure at any time of the day.
c y d y

0 x 0 x

e The relation which shows the number of student absences per day during term 3 at
your school.
f The relation describing the weight of a child from age 3 months to one year.

6 Sketch the graph representing each of the following relations, and state whether each
eBook plus
is discrete or continuous.
Digital doc: a
EXCEL Spreadsheet Day Mon Tues Wed Thur Fri Sat Sun
Plotting relations
Cost of petrol (c/L) 168 167.1 166.5 164.9 167 168.5 170

b {(0, 0), (1, 1), (2, 4), (3, 9)}


WORKED c y = −x2, where x ∈ {−2, −1, 0, 1, 2}
Example
1 d y = x − 2, where x ∈ R
e y = 2x + 3, where x ∈ J
f y = x2 + 2, where −2 ≤ x ≤ 2 and x ∈ R

WORKED 7 The table below shows the temperature of a cup of coffee, T °C, t minutes after it is
Example
2
poured.

t (min) 0 2 4 6 8
T (°C) 80 64 54 48 44

a Plot the points on a graph.


b Join the points with a smooth curve.
c Explain why this can be done.
d Use the graph to determine how long it takes the coffee to reach half of its initial
temperature.
Chapter 2 Relations and functions 69
8 A salesperson in a computer store is paid a base salary of $300 per week plus $40
commission for each computer she sells. If n is the number of computers she sells per
week and P dollars is the total amount she earns per week, then:
a copy and complete the following table
n 0 1 2 3 4 5 6
P

b plot the information on a graph


c explain why the points cannot be joined together.
9 The speed of an aircraft, V km/h, t seconds after it starts to accelerate down the
runway, is shown in the table below.
t 0 1 2 3 4 5
V 0 30 80 150 240 350

a Plot a graph which represents the information shown in the table.


b Use the graph to estimate the speed after: i 2.5 s ii 4.8 s.
10 The cost, C dollars, of taking n students on an excursion to the zoo is $50 plus $6 per
student.
a Complete a table using 15 ≤ n ≤ 25.
b Plot these points on a graph.
c Explain why the dots can or cannot be joined.

Domain and range


Domain and range
A relation may be described by:
1. a listed set of ordered pairs
2. a graph or
3. a rule.
The set of all first elements of a set of ordered pairs is known as the domain and
the set of all second elements of a set of ordered pairs is known as the range. Alter-
natively, the domain is the set of independent values and the range is the set of
dependent values.
If a relation is described by a rule, it should also specify the domain. For example:
1. the relation {(x, y): y = 2x, x ∈ {1, 2, 3}} describes the set of ordered pairs {(1, 2),
(2, 4), (3, 6)}
2. the domain is the set X = {1, 2, 3}, which is given
3. the range is the set Y = {2, 4, 6}, and can be found by applying the rule y = 2x to the
domain values.
If the domain of a relation is not specifically stated, it is assumed to consist of all real
numbers for which the rule has meaning. This is referred to as the implied domain of a
relation. For example:
1. {(x, y): y = x3} has the implied domain R.
2. {(x, y): y = x } has the implied domain x ≥ 0, where x ∈ R, since the square root of
a negative number is an imaginary value.
70 M a t h s Q u e s t M a t h s B Ye a r 1 1 f o r Q u e e n s l a n d

Interval notation
If a and b are real numbers and a < b, then the following intervals are defined with an
accompanying number line:
(a, b) implies a < x < b or (a, b] implies a < x ≤ b or

a b x a b x
(a, ∞) implies x > a or [a, ∞) implies x ≥ a or

a x a x
(−∞, b) implies x < b or (−∞, b] implies x ≤ b or

b x b x

[a, b) implies a ≤ x < b or [a, b] implies a ≤ x ≤ b or

a b x a b x

The closed circle indicates that the number is included and the open circle indicates
that the number is not included.

WORKED Example 3
Describe each of the following subsets of the real numbers using interval notation.
a b c
–4 0 2 x –3 0 5 x 0 1 3 5 x

THINK WRITE
a The interval is x < 2 (2 is not included). a (−∞, 2)
b The interval is −3 ≤ x < 5 (3 is included). b [−3, 5)
c The interval is both 1 ≤ x < 3 and x ≥ 5 c [1, 3) ∪ [5, ∞)
(1 is included, 3 is not). The symbol ∪
indicates the combination of the two intervals.

WORKED Example 4
Illustrate the following number intervals on a number line.
a (−2, 10] b [1, ∞)
THINK WRITE
a The interval is −2 < x ≤ 10 (−2 is not a
included, 10 is). –2 0 10 x

b The interval is x ≥ 1 (1 is included). b x


0 1
Chapter 2 Relations and functions 71
WORKED Example 5
State the domain and range of each of the following relations.
a {(1, 2), (2, 5), (3, 8), (4, 11)}
b
Mass (kg) 10 15 20 25 30
Cost per kg ($) 3.5 3.2 3.0 2.8 2.7

c y d y
4

–4 0 4 x

0 x
–4

THINK WRITE
a 1 The domain is the set of first elements a Domain = {1, 2, 3, 4}
of the ordered pairs.
2 The range is the set of second elements Range = {2, 5, 8, 11}
of the ordered pairs.
b 1 The domain is the set of independent values b Domain = {10, 15, 20, 25, 30}
in the table, that is, the mass values.
2 The range is the set of dependent values Range = {2.7, 2.8, 3.0, 3.2, 3.5}
in the table, that is, the cost values.

c 1 The domain is the set of values that the c Domain = R


graph covers horizontally.
2 The range is the set of values that the Range = [0, ∞)
graph covers vertically.
d 1 The domain is the set of values that the d Domain = [−4, 4]
graph covers horizontally.
2 The range is the set of values that the Range = [−4, 4]
graph covers vertically.

WORKED Example 6
For each relation given, sketch its graph and state the domain and range using interval notation.
a {(x, y): y = x – 1 } b {(x, y): y = x2 − 4, x ∈ [0, 4]}

THINK WRITE
a 1 The rule has meaning for x ≥ 1 because a
if x < 1, y = negative number .

Continued over page


72 M a t h s Q u e s t M a t h s B Ye a r 1 1 f o r Q u e e n s l a n d

THINK WRITE
2 Therefore, calculate the value of y When x = 1, y = 0
when x = 1, 2, 3, 4 and 5, and state the =0 (1, 0)
coordinate points. x = 2, y = 1
=1 (2, 1)
x = 3, y = 2 (3, 2 )
x = 4, y = 3 (4, 3 )
x = 5, y = 4
=2 (5, 2)
3 Plot the points on a set of axes. y
y= x–1
4 Join the points with a smooth curve
2
starting from x = 1, extending it beyond
the last point. Since no domain is given 1
we can assume x ∈ R (continuous).
5 Place a closed circle on the point (1, 0) 0 x
1 2 3 4 5
and put an arrow on the other end of the
–1
curve.
6 The domain is the set of values covered Domain = [1, ∞)
horizontally by the graph, or implied by
the rule.
7 The range is the set of values covered Range = [0, ∞)
vertically by the graph.
b 1 Calculate the value of y when x = 0, 1, b When x = 0, y = 02 − 4
2, 3 and 4, as the domain is [0, 4]. State = −4 (0, −4)
the coordinate points. x = 1, y = 12 − 4
= −3 (1, −3)
x = 2, y = 22 − 4
=0 (2, 0)
x = 3, y = 32 − 4
=5 (3, 5)
x = 4, y = 42 − 4
= 12 (4, 12)
2 Plot these points on a set of axes. y
y = x2 – 4, x ∈ [0, 4]
12
3 Join the dots with a smooth curve from 10
x = 0 to x = 4. 8
6
4 Place a closed circle on the points 4
(0, −4) and (4, 12). 2
0 x
–2 1 2 3 4
–4
5 The domain is the set of values covered Domain = [0, 4]
by the graph horizontally.
6 The range is the set of values covered Range = [−4, 12]
by the graph vertically.
Verify that the graphs are correct using a graphics calculator.
Chapter 2 Relations and functions 73

Graphics Calculator tip! Graphing relations with


a restricted domain
We can specify the restricted domain of a relation when entering the rule on a graphics
calculator. Consider the graph of y = x2 – 4, x ∈ [0, 4].
For the Casio fx-9860G AU
1. Press MENU and then select GRAPH. Press
SHIFT F3 (V-WIN) and adjust the View
Window settings so that Xmin is −1, Xmax is 5,
Ymin is −10 and Ymax is 15. Press EXE after <Casio02.07>
each new entry or press the down arrow to skip to
the next line.
2. Press EXIT to return to the Graph Function
screen. Enter x2 − 4 for Y1. (Press X,q,T to enter
x.) To enter the restricted domain, first press the
comma key ( , ) and then enter [0, 4]. Press <Casio02.08>
EXE .

3. Press F6 (DRAW) to display the graph.

<Casio02.09>

4. To view the coordinates of points along the curve


and to investigate the range of the relation, press
SHIFT F1 (TRCE) and use the arrow keys.
<Casio02.10>

For the TI-Nspire CAS


1. Open a new Graphs & Geometry document
(press /N and then select 2: Add Graphs &
Geometry).
<TI02.06>

2. To enter a relation with a restricted domain, we


need to first define it. Press /I and then select
1: Add Calculator to insert a Calculator
document. Press b and then select 1: Actions <TI02.07>
followed by 1: Define.
74 M a t h s Q u e s t M a t h s B Ye a r 1 1 f o r Q u e e n s l a n d

3. Enter f1(x) = x2 − 4 (press F1 to enter f1)


and then press * for the ‘given’ symbol. Next
we need to enter the domain as 0 ≤ x ≤ 4. First
enter 0. <TI02.08>

4. To enter the ≤ symbol, press /k to access the


symbol palette. Use the NavPad to highlight the
≤ symbol.
<TI02.09>

5. Press · to display the chosen symbol in the


Calculator document. Continue to enter the rest
of the expression for the domain. Press · to
store the function as f1(x). <TI02.10>

6. Press / and then the left arrow of the NavPad


to return to the Graphs & Geometry document.
The rule and the restricted domain will now be
seen in the function entry line for f1(x). <TI02.11>

7. Press · to display the graph. Adjust the Window


Settings to see the graph clearly (press b and
then select 4: Window followed by 1: Window
Settings). In this example, YMin has been <TI02.12>
changed to –10 and YMax to 15.

8. To view the coordinates of points along the curve


and to investigate the range of the relation, press
b and then select 5: Trace followed by 1: Graph
<TI02.13>
Trace. Use the NavPad to move the point (or
trace) along the curve.
Chapter 2 Relations and functions 75
remember
1. The domain of a relation is the set of first elements of an ordered pair.
2. The range of a relation is the set of second elements of an ordered pair.
3. The implied domain of a relation is the set of first element values for which a
rule has meaning.
4. In interval notation a square bracket means the end point is included in a set of
values, whereas a curved bracket means the end point is not included.
(a, b]
a b

2B Domain and range

WORKED 1 Describe each of the following subsets of the real numbers using interval notation.
Example
3
a b
–2 0 1 0 5
c d
–3 0 4 –8 0 9
e f
–1 0 0 1
g h
–5 –2 0 3 –3 0 1 2 4

WORKED 2 Illustrate each of the following number intervals on a number line.


Example
4 a [−6, 2) b (−9, −3) c (−∞, 2]
d [5, ∞) e (1, 10] f (2, 7)
g (−∞, −2) ∪ [1, 3) h [−8, 0) ∪ (2, 6]
3 Describe each of the following sets using interval notation.
a {x: −4 ≤ x < 2} b {x: −3 < x ≤ 1} c {y: −1 < y < 3}
1 e {x: x > 3}
d {y: − 1--- < y ≤ ------- } f {x: x ≤ −3}
2 2
g R h R\{0}
y
4
4 multiple choice
eBook plus
The domain of the relation graphed at right is:
Digital doc: A [−4, 4] B (−4, 7) C [−1, 7]
SkillSHEET 2.1 –1 0 3 7 x
Domain and range D (−4, 4) E (−1, 7)

–4
5 multiple choice
The range of the relation {(x, y): y = 2x + 5, x ∈ [–1, 4]} is:
A [7, 13] B [3, 13] C [3, ∞) D R E [0, ∞)
76 M a t h s Q u e s t M a t h s B Ye a r 1 1 f o r Q u e e n s l a n d

WORKED 6 State i the domain and ii the range of each of the following relations.
Example
a {(3, 8), (4, 10), (5, 12), (6, 14), (7, 16)}
5a, b
b {(1.1, 2), (1.3, 1.8), (1.5, 1.6), (1.7, 1.4)}
c
Time (min) 3 4 5 6
Distance (m) 110 130 150 170

d
Day Monday Tuesday Wednesday Thursday Friday
Cost ($) 25 35 30 35 30

e y = 5x − 2, where x is an integer greater than 2 and less than 6.


f y = x2 − 1, x ∈ R

WORKED 7 State the domain and range of each of the following relations.
Example
5c, d a y b y c y
2
2 2

–3 0 x
0 x
–2 0 2x

d y e y f y
y= x–1
4 0 x

0 1 x –3

0 x

g y h y i y
1–
y= x 1
0 x

0 x 0 x –2

WORKED 8 For each relation given, sketch its graph and state the domain and range using interval
Example
6
notation.
a {(x, y): y = 2 − x2} b {(x, y): y = x3 + 1, x ∈ [−2, 2]}
c {(x, y): y = x2 + 3x + 2} d {(x, y): y = x2 − 4, x ∈ [−2, 1]}
e {(x, y): y = 2x − 5, x ∈ [−1, 4)} f {(x, y): y = 2x2 − x − 6}
Verify that the graphs are correct with a graphics calculator.

9 State the implied domain for each relation defined by the following rules:
eBook plus
a y = 10 − x b y=3 x c y = − 16 – x 2
Digital doc:
1
WorkSHEET 2.1 d y = x2 + 3 e y = --- f y = 10 − 7x2
x
Chapter 2 Relations and functions 77
Interesting relations
This investigation deals with graphs of different relations and will require the use
of graphing software such as GrafEq or Graphmatica to produce quick, accurate
eBook plus
graphs. For a demonstration version of GrafEq, log into www.jacplus.com.au and
Weblinks: locate the weblinks for this chapter.
GrafEq software Use a program such as GrafEq to produce a graph of each of the following.
Graphmatica software
Sketch each graph into your workbook, and label each with its equation.
1 x2 + 2y2 = 9
y
2 x3 + y3 = 1
2 2
3 sin (x + y ) = 1 1
4 x2 – y2 = 1
5 7x2 – 6 3 xy + 13y2 = 16
6 x4 = x2– y2 –1 1 x
7 x2 + y2 < 25
8 x2 + y2 > 25 –1
2 2
9 9 < x + y < 36
10 x sin x + y sin y < 1 The graph of y2(1 − x) = x2(x + 1)
produced by GrafEq.

Types of relations (including functions)


One-to-one relations
A one-to-one relation exists if for any x-value there is only one corresponding y-value
and vice versa. For example:
1. {(1, 1), (2, 2), (3, 3), (4, 4)} 2. y

0 x

One-to-many relations
A one-to-many relation exists if for any x-value there is more than one y-value, but for
any y-value there is only one x-value. For example:
1. {(1, 1), (1, 2), (2, 3), (3, 4)} 2. y

0 x

Many-to-one relations
A many-to-one relation exists if there is more than one x-value for any y-value but for
any x-value there is only one y-value. For example:
1. {(−1, 1), (0, 1), (1, 2)} 2. y

0 x
78 M a t h s Q u e s t M a t h s B Ye a r 1 1 f o r Q u e e n s l a n d

Many-to-many relations
A many-to-many relation exists if there is more than one x-value for any y-value and
vice versa. For example:
1. {(0, −1), (0, 1), (1, 0), (−1, 0)} 2. y y

0 x 0 x

WORKED Example 7
What type of relation does each graph represent?
a y b y c y

0 x
0 x 0 x

THINK WRITE
a 1 For some x-values there is more than one a One-to-many relation.
y-value. A line through some x-values
shows that 2 y-values are available:
y
F
x = –1
0 x

2 For any y-value there is only one x-value.


A line through any y-value shows that
only one x-value is available:
y

y=1
0 x

b 1 For any x-value there is only one y-value. b One-to-one relation.


2 For any y-value there is only one x-value.

c 1 For any x-value there is only one y-value. c Many-to-one relation.


2 For some y-values there is more than one
x-value.
Chapter 2 Relations and functions 79
Functions
Relations which are one-to-one or many-to-one are called functions. That is, a function
is a relation where for any x-value there is only one y-value. For example:
1. y 2. y

0 x 0 x

Vertical line test


A function is determined from a graph if a vertical line drawn anywhere on the graph
cannot intersect with the curve more than once.

WORKED Example 8
State whether or not each of the following relations are functions.
a {(−2, 1), (−1, 0), (0, −1), (1, −2)}
b y cy

0 x
0 x

THINK WRITE
a For each x-value there is only one y-value. a Function
(Or, a plot of the points would pass the
vertical line test.)
b It is possible for a vertical line to intersect b Not a function
with the curve more than once.
c It is not possible for any vertical line to c Function
intersect with the curve more than once.

remember
1. A function is a relation which does not repeat the first element in any of its
ordered pairs. That is, for any x-value there is only one y-value (one-to-one or
many-to-one relations.)
2. Vertical line test: The graph of a function cannot be crossed more than once by
any vertical line.
y y

0 x

0 x

Function Not a function


80 M a t h s Q u e s t M a t h s B Ye a r 1 1 f o r Q u e e n s l a n d

Types of relations
2C (including functions)
WORKED 1 What type of relation does each graph represent?
Example
7 a b c
y y y

0 x
0 x 0 x

d y e y f y

0 x 0 x
0 x

g y h y i y

0 x 0 x
0 x

j y k y l y

0 x
0 x 0 x

WORKED 2 Use the vertical line test to determine which of the relations in question 1 are functions.
Example
8

3 multiple choice
Which of the following relations is not a function?
A {(5, 8), (6, 9), (7, 9), (8, 10), (9, 12)}
B y C y2 = x D y = 8x − 3 E y

0 x 0 x
Chapter 2 Relations and functions 81
4 multiple choice
Consider the relation y ≥ x + 1.
a The graph which represents this relation is:
A y B y
1
1
–1 0 x
–1 0 x

C y D y
1
1 0 x
1
0 x
1

E y Note: The shaded side


indicates the region not
required.
1

–1 0 x

b This relation is:


A one-to-one
B one-to-many
C many-to-one
D many-to-many
E a function
c The domain and range are respectively:
A R and R+ (R+ stands for positive real numbers.)
B R and R
C R and R− (R− stands for negative real numbers.)
D R+ and R
E R− and R
5 Which of the following relations are functions? State the domain and range for each
function.
a {(0, 2), (0, 3), (1, 3), (2, 4), (3, 5)} b {(−3, −2), (−1, −1), (0, 1), (1, 3), (2, −2)}
c {(3, −1), (4, −1), (5, −1), (6, −1)} d {(1, 2), (1, 0), (2, 1), (3, 2), (4, 3)}
e {(x, y): y = 2, x ∈ R} f {(x, y): x = −3, y ∈ J}
g y = 1 − 2x h y>x+2
i x2 + y2 = 25 j y= x + 1 , x ≥ −1
3 2
k y=x +x l x=y +1
82 M a t h s Q u e s t M a t h s B Ye a r 1 1 f o r Q u e e n s l a n d

Function notation
Consider the relation y = 2x, which is a function.
The y-values are determined from the x-values, so we say ‘y is a function of x’,
which is abbreviated to y = f (x).
So, the rule y = 2x can also be written as f (x) = 2x.
If x = 1, then y = f (1) If x = 2, then y = f (2)
=2×1 =2×2
=2 = 4, and so on.

Evaluating functions
For a given function y = f (x), the value of y when x = 1 is written as f (1) or the value
of y when x = 5 is written as f (5) etc.

WORKED Example 9
If f (x) = x2 − 3, find:
a f (1) b f (−2) c f (a) d f (2a).

THINK WRITE
a 1 Write the rule. a f (x) = x2 − 3
2 Substitute x = 1 into the rule. f (1) = 12 − 3
3 Simplify. =1−3
= −2

b 1 Write the rule. b f (x) = x2 − 3


2 Substitute x = −2 into the rule. f (−2) = (−2)2 − 3
3 Simplify. =4−3
=1

c 1 Write the rule. c f (x) = x2 − 3


2 Substitute x = a into the rule. f (a) = a2 − 3

d 1 Write the rule. d f (x) = x2 − 3


2 Substitute x = 2a into the rule. f (2a) = (2a)2 − 3
3 Simplify the expression if possible. = 22a2 − 3
= 4a2 − 3
Chapter 2 Relations and functions 83

Graphics Calculator tip! Defining and


using functions
Consider the function f(x) = x2 − 3 shown in Worked example 9. We can define this
function and then use it to evaluate the function for different x-values.
For the Casio fx-9860G AU
1. Press MENU and then select TABLE. Enter x2 − 3
for Y1 and then press EXE .
<Casio02.11>
2. To set which x-values are to be used in the table,
press F5 (SET) and adjust the Start, End and
Step values as appropriate. Press EXE after
each new entry or press the down arrow to skip to
the next line. (Since we will be inputting specific
<Casio02.11>
values for x later, you may like to skip this step
and use the previous setting.)

3. Press EXIT to return to the Table Function


screen. To display the table of values, press F6
(TABL). <Casio02.11>

4. To enter your own value for x (say 5), highlight a


cell in the X column and enter 5. Press EXE to
display the corresponding function value in the Y1
column. <Casio02.11>

For the TI-Nspire CAS


1. Open a new Calculator document (press /N
and then select 1: Add Calculator). Press b and <TI02.14>
then select 1: Actions followed by 1: Define.

2. Enter f1(x) = x2 − 3 and then press ·. (To enter


f1, press F1.)
This defined function can be used in different
ways. It can be entered in an algebraic expression <TI02.15>
or used to graph the function.

3. We can now evaluate the function for different


values of x. To find the function value when x
is −2, enter f1(−2) and then press ·. Similarly, to
find the function value when x is 5, enter f1(5) and <TI02.16>
then press ·. To find the expression for the
function when x is 2a, enter f1(2a) and then
press ·.
84 M a t h s Q u e s t M a t h s B Ye a r 1 1 f o r Q u e e n s l a n d

Special types of functions


One-to-one functions
As we have already seen, one-to-one relations and many-to-one relations are functions. A
one-to-one function has, at most, one y-value for any x-value and vice versa. The graph of
a relation is a function if any vertical line crosses the curve at most once. Similarly, a one-
to-one function exists if any horizontal line crosses the curve at most once. For example:
y y

0 x 0 x

A function which is not one-to-one A one-to-one function

WORKED Example 10
Which of the following functions are one-to-one?
a {(0, 1), (1, 2), (2, 3), (3, 1)} b {(2, 3), (3, 5), (4, 7)} c f (x) = 3x
THINK WRITE
1 Check whether each function has, at most, a When x = 0 and x = 3, y = 1.
one y-value for any x-value and vice versa. It is not a one-to-one function.
b There is only one x-value for each y-value.
It is a one-to-one function.
2 Sketch the graph of f (x) = 3x. Check c y f (x)
whether both a vertical line and a
3
horizontal line crosses the graph only
once.
0 1 x

It is a one-to-one function.
3 Write a statement to answer the question. The functions are one-to-one for b and c.

WORKED Example 11
Which of the following graphs show a one-to-one function?
a y b y c y

0 x
0 x 0 x

THINK WRITE
If a function is one-to-one, any vertical or Only b shows a one-to-one function.
horizontal line crosses the graph only once.
Chapter 2 Relations and functions 85
Hybrid functions
A hybrid, mixed, or piecewise defined function is a function which has different rules
for different subsets of the domain. For example:

 x + 1, for x ≤ 0
f ( x) = 
 x2, for x > 0

is a hybrid function which obeys the rules y = x + 1 if x ∈ (−∞, 0] and y = x2


if x ∈ (0, ∞). The graph of f (x) is shown below.

y f (x)

–1 0 x

WORKED Example 12
 x, x<0

a Sketch the graph of f ( x ) =  x + 1, 0£x<2 b State the range of f.

 5 – x, x≥2

THINK WRITE
a (Calculate and plot points as shown or a If x = −1, y = x
use a graphics calculator.) = −1.
1 Sketch the graph of y = x for the If x = 0, y = x
domain (−∞, 0). = 0.
2 On the same axes sketch the graph of If x = 0, y = x + 1
y = x + 1 for the domain [0, 2). = 1.
If x = 2, y = x + 1
= 3.
3 On the same axes sketch the graph of If x = 2, y = 5 − x
y = 5 − x for the domain [2, ∞). Use = 3.
a graphics calculator to assist with If x = 5, y = 5 − x
the graphing if necessary. = 0.
y

3
2 f (x)
1

1 0 1 2 3 4 5 x
–1

b The range is made up of (or is the union b ran f = (−∞, 3]


of) two sections, (−∞, 0) with (−∞, 3].
86 M a t h s Q u e s t M a t h s B Ye a r 1 1 f o r Q u e e n s l a n d

Graphics Calculator tip! Graphing a


hybrid function
To graph the hybrid function shown in Worked example 12, follow the instructions
below.
For the Casio fx-9860G AU
With this calculator, we need to enter each rule with its subset of the domain as a
separate function and then graph all functions on the one set of axes.
1. Press MENU and then select GRAPH. Press
SHIFT F3 (V-WIN) and then adjust the View
Window settings as appropriate. For this example,
Xmin is −5, Xmax is 7, Ymin is −5 and Ymax is 5. <Casio02.15>
Press EXE after each new entry or press the
down arrow to skip to the next line.

2. Press EXIT to return to the Graph Function


screen. Enter x and a domain of (−∞, 0) for Y1 as
shown in the screen at right. Press EXE . Note
that we don’t enter −∞ and that square brackets <Casio02.15>
are used.

3. Enter x + 1 with a domain of [0, 2] for Y2 and then


press EXE . Similarly, enter 5 − x with a domain
of [2, ∞) for Y3 as shown and then press EXE .
<Casio02.15>

4. Press F6 (DRAW) to display the graph of the


hybrid function.

<Casio02.15>

5. To view the coordinates of points along the


function and to investigate the range, press
SHIFT F1 (TRCE) and use the arrow keys.
<Casio02.15>

For the TI-Nspire CAS


1. Open a new Graphs & Geometry document
(press /N and then select 2: Add Graphs &
Geometry). To enter a hybrid function in the <TI02.17>
function entry line, press /r to display the
maths expression palette and then use the NavPad
to highlight the 3-piece hybrid template.
Chapter 2 Relations and functions 87
2. Press ·. A piecewise function box appears.
Ensure the number of function pieces shows 3 and
then press e to highlight OK.
<TI02.18>

3. Press · to insert the template in the function


entry line. Complete the 6 boxes with the
appropriate rules and domains. Press e to move
to the next box. A shortcut to inserting the ≤ <TI02.19>
symbol is to press /<. To insert the ≥ symbol,
press />.

4. Press · to display the graph.

<TI02.20>

5. Move the equation label so the graph can be


viewed clearly. Press d to display a pointer (å).
Use the NavPad to move the pointer so that it
hovers over the equation. The pointer will then <TI02.21>
appear as an open hand (÷). Press / followed
by x to close the hand ({). The equation will
start to flash. Use the NavPad to move the
equation to the desired location and press d to
anchor it in position.

6. To view the coordinates of points along the


function and to investigate the range, press b
and then select 5: Trace followed by 1: Graph
Trace. Use the NavPad to move the point (or <TI02.22>
trace) along the function. (Note that the small
upper case M shown in a box indicates this is a
maximum point.)

remember
1. A function is one-to-one if for each x-value there is only one y-value and vice
versa.
2. A many-to-one function may be ‘converted to’ a one-to-one function by
restricting the domain.
3. A hybrid function obeys different rules for different subsets of the domain.
88 M a t h s Q u e s t M a t h s B Ye a r 1 1 f o r Q u e e n s l a n d

Function notation and special


2D types of functions
WORKED 1 a If f (x) = 3x + 1, find i f (0), ii f (2), iii f (−2) and iv f (5) respectively.
Example
9 b If g(x) = x + 4 , find i g(0), ii g(−3), iii g(5) and iv g(−4) respectively.
1
eBook plus c If g(x) = 4 − --- , find i g(1), ii g  1--- , iii g  – 1--- and iv g  – 1--- respectively.
x 2 2 5
d If f (x) = (x + 3)2, find i f (0), ii f (−2), iii f (1) and iv f (a) respectively.
Digital docs:
24
SkillSHEET 2.2
Substitution
e If h(x) = ------ , find i h(2), ii h(4), iii h(−6) and iv h(12) respectively.
x
SkillSHEET 2.3
Transposition of 2 Find the value (or values) of x for which each function has the value given.
equations
a f (x) = 3x − 4, f (x) = 5 b g(x) = x2 − 2, g(x) = 7
1
c f (x) = --- , f (x) = 3 d h(x) = x2 − 5x + 6, h(x) = 0
x
e g(x) = x2 + 3x, g(x) = 4 f f (x) = 8 – x , f (x) = 3
10
3 Given that f ( x ) = ------ – x, find:
x
a f (2) b f (−5) c f (2x)
d f (x2) e f (x + 3) f f (x − 1)
WORKED 4 Which of the following functions are one-to-one? Use a graphics calculator to obtain
Example
10
the graph of the function where appropriate.
a {(1, −1), (2, 1), (3, 3), (4, 5)} b {(−2, 1), (−1, 0), (0, 2), (1, 1)}
c {(x, y): y = x2 + 1, x ∈ [0, ∞)} d {(x, y): y = 3 − 4x}
e {(x, y): y = 3 − 2x2} f f (x) = x3 − 1
2
g y=x,x≤0 h g(x) = 1 – x 2

WORKED 5 Consider the relations below and state:


Example i which of them are functions ii which of them are one-to-one functions.
11
a y b y c y d y

0 x 0 x
0 x 0 x

e y f y g y h y

0 x x
0
0 x 0 x

i y j y k y l y

0 x
0 x 0 x
0 x
Chapter 2 Relations and functions 89
6 multiple choice
Consider the following hybrid function:
 – x, x<1
f ( x) = 
 x, x≥1
a The graph which correctly represents this function is:
A y B y C y

1 1
1
0 1 x 0 x
2 1 2
–1 –1 0 x
1
–1

D y E y

x 1
0 1
–1 x
0 1
–1

b The range of this hybrid function is:


A R B [−1, ∞) C (−1, ∞) D [0, ∞) E R+

1
 --- , x<0
WORKED 7 a Sketch the graph of the function f ( x ) =  x
Example
 x + 1, x≥0
12 
b State the range of f.
 x 2 + 1, x≥0
8 a Sketch the graph of the function g ( x ) = 
 2 – x, x<0
b State the range of g.
c Find i g(−1) ii g(0) iii g(1).
 x – 2, x < –2

9 a Sketch the graph of the function f ( x ) =  x – 4, – 2 ≤ x ≤ 2
2

 x + 2, x>2
b State the range of z.
c Find i f (−3) ii f (−2) iii f (1) iv f (2) v f (5). y
3 f (x)
2
10 Specify the rule for the function represented by the graph
1
at right.
–2 –1 0 x
–1 1
–2

y
11 The graph of the relation {(x, y): x2 + y2 = 1, x ≥ 0} is shown 1
eBook plus at right.
From this relation, form 2 one-to-one functions and state 0 1 x
Digital doc: the range of each.
WorkSHEET 2.2
–1
90 M a t h s Q u e s t M a t h s B Ye a r 1 1 f o r Q u e e n s l a n d

Inverse relations and functions


A relation is a set of ordered pairs that can be graphed or described by a rule. The
inverse of a set of ordered pairs is obtained simply by interchanging the x and y
elements. So, the inverse of {(1, 5), (2, 6), (3, 7)} is {(5, 1), (6, 2), (7, 3)}. If these
points are plotted on a set of axes, it can be seen that when each original point is
reflected across the line y = x, the inverse points are obtained.
Similarly, if the graph of a function is given, then its inverse function can be
sketched by reflecting the original function across the line y = x.

WORKED Example 13
Sketch the graph of the following and then sketch the inverse.
a {(3, −6), (4, −4), (5, −2), (6, 0)} b y c y = x2 for x ≥ 0

x
THINK WRITE/DRAW
a 1 Plot the points on a set of axes. a y y=x
2 Interchange the x and y values and 6
5
plot them. Alternatively, reflect the 4
original points across the line y = x. 3
2
1
0
–1 1 2 3 4 5 6 x
–6–5–4–3–2–1
–2
–3
–4
–5
–6
b 1 Re-draw the given graph. b y y=x
2 On the same set of axes, plot the line
y = x.
3 Sketch a reflection of the original
graph across the line y = x.

x
c 1 Sketch the original function. The c y y = x2 y=x
graph is a parabola with turning
point (0, 0). The domain is restricted.

0 x
1
Chapter 2 Relations and functions 91
remember
1. An inverse relation is obtained by interchanging the x and y values.
2. The graph of a function and its inverse are reflections of each other across
the line y = x.

Inverse relations and


2E functions
WORKED 1 Sketch the graph of the following and then sketch the inverse.
Example
13a a (1, 7), (2, 5), (3, 3)} b (1, 3), (2, 6), (3, 9)} c (−2, 11), (0, 6), (2, 1)}

WORKED 2 Sketch the graph of the following and then sketch the inverse. (Assume each set of
Example
13b
axes has the same scale for x and y.)
a y b yy

x xx
c y d y

x
e y
f y

WORKED 3 Sketch the graph of the following and then sketch the inverse
Example
13c a y = 4x
b y = x2 + 3, x ≥ 0
c y = 1--- x + 1
2
d y = x3 + 4
92 M a t h s Q u e s t M a t h s B Ye a r 1 1 f o r Q u e e n s l a n d

A special relation
You are familiar with the shape of the graph of y = x2 (a parabola), but what about
the relation x2 + y2 = 25?
If x = 0 and y = 5 are substituted into the rule, we get
x2 + y2 = 02 + 52 = 25 = RHS, so (0, 5) lies on the graph of x2 + y2 = 25.
eBook plus What other points lie on the graph?
Below is a table of coordinates. Twelve of the coordinate pairs listed lie on the
Digital doc:
EXCEL Spreadsheet
graph of x2 + y2 = 25.
Special relations 1 Copy and complete the table below.

Coordinate pairs
On the graph of
x y x2 y2 x 2 + y2 x2 + y2 = 25?
0 5 0 25 25 Yes
4 2 16 4 20 No
3 0
0 8
3 4
4 3
7 7
–4 −3
–5 0
–4 3
1 5
3 −4
9 0
4 −3
0 −5
6 −6
−3 4
−2 −5
5 0
−3 −4

2 Use the table to plot the graph of x2 + y2 = 25. (Use a smooth curve to join
points.)
3 Use graphing software (for example, Graphmatica) or the Maths Quest file on
eBook plus
www.jacplus.com.au to explore the effect of a on the graph of
Weblink: x2 + y2 = a2. Try values of a such as 1, 3, 9, 12, 36, 50 and 100.
Graphmatica software 4 Investigate graphs of relations of the form (x − h)2 + (y − k)2 = a2; for example,
(x − 1)2 + (y + 3)2 = 16. How is the equation related to features of the graph?
Chapter 2 Relations and functions 93
Circles
A circle is a many-to-many relation.
The rule that defines a circle with its centre at (0, 0) and of radius r is
x2 + y2 = r 2 y
r
The graph of this circle is shown at right.
The vertical-line test clearly verifies that the circle graph is not a
function. –r r x
Solving the equation for y we have y2 = r2 − x2, so y = r 2 – x 2
or y = – r 2 – x 2 . –r
These two relations represent two semicircles that together make a complete circle.
y = r 2 – x 2 is the ‘upper semicircle’ (above the x-axis).
y = – r 2 – x 2 is the ‘lower semicircle’ (below the x-axis).
y y
y= r2 – x2
r –r r x

–r y = – r 2 – x 2
–r r x

WORKED Example 14
Sketch the graphs of the following relations.
a x2 + y2 = 16 b x2 + y2 = 9, 0 ≤ x ≤ 3 c y= 8 – x2

THINK WRITE
a 1 This relation is a circle of centre a y
(0, 0) and radius = 16 = 4 . 4
2 On a set of axes mark x- and
y-intercepts of −4 and 4. –4 0 4 x
3 Draw the circle.
–4

b 1 This relation is part of a circle of b y


centre (0, 0) and radius = 9 = 3. 3
2 Since the domain is [0, 3], on a set of
axes mark y-intercepts −3 and 3 and
0 3 x
x-intercept 3.
3 Draw a semicircle on the right-hand –3
side of the y-axis.
c 1 This relation is an ‘upper semicircle’ c y
(as y > 0) of centre (0, 0) and
radius = 8 . 8

2 On a set of axes mark the x-intercepts


of − 8 and 8 and y-intercepts
of 8 . – 8 0 8x
3 Draw a semicircle above the x-axis.
94 M a t h s Q u e s t M a t h s B Ye a r 1 1 f o r Q u e e n s l a n d

General equation of a circle y


(x – h)2 + (y – k)2 = r 2
The general equation of a circle with centre (h, k) k+r
and radius r is (x − h)2 + (y − k)2 = r2.
The domain is [h − r, h + r]. (h, k)
Range k
The range is [k − r, k + r].
k–r
0 h–r h h+r x
Domain

WORKED Example 15
Sketch the graphs of the following circles. State the domain and range of each.
a x2 + ( y − 3)2 = 1 b (x + 3)2 + ( y + 2)2 = 9
THINK WRITE
a 1 This circle has centre (0, 3) and radius 1. a y
2 On a set of axes mark the centre and four points; 4 x 2 + (y – 3)2 = 1
1 unit (the radius) left and right of the centre, and 3
1 unit (the radius) above and below the centre.
2
3 Draw a circle which passes through these
four points. x –1 0 1

4 State the domain. Domain is [−1, 1].


5 State the range. Range is [2, 4].
b 1 This circle has centre (−3, −2) and radius 3. b y
(x + 3)2 + (y + 2)2 = 9
2 On a set of axes mark the centre and four 1
points; 3 units left and right of the centre,
–6 –3 0 x
and 3 units above and below the centre.
3 Draw a circle which passes through these –2
four points.

–5

4 State the domain. Domain is [−6, 0].


5 State the range. Range is [−5, 1].

Graphics Calculator tip! Graphing a circle


To graph the circle with the equation (x + 3)2 + (y + 2)2 = 9 shown in part b of Worked
example 15, we need to graph two separate functions.
To find the rules for the two functions, we can rearrange the equation to isolate y.
(x + 3)2 + (y + 2)2 = 9
(y + 2)2 = 9 − (x + 3)2
2
y + 2 = ± 9 – ( x + 3)
2
y = −2 ± 9 – ( x + 3 )
Chapter 2 Relations and functions 95
2
So the rule for the upper semicircle is given by y = −2 + 9 – ( x + 3 ) and the rule for
2
the lower semicircle is given by y = −2 − 9 – ( x + 3 ) . To obtain the graph of the
circle we will graph both functions on the one set of axes.
For the Casio fx-9860G AU
1. Press MENU and then select GRAPH. Press
SHIFT F3 (V-WIN) and adjust the View
Window settings as appropriate. For this example,
Xmin is −7, Xmax is 2, Ymin is −7 and Ymax is 2. <Casio02.20>
Press EXE after each new entry or press the
down arrow to skip to the next line.
2. Press EXIT to return to the Graph Function
2
screen. Enter −2 + 9 – ( x + 3 ) for Y1 as shown
in the screen at right. <Casio02.21>

2
3. Press EXE . Similarly, enter −2 − 9 – ( x + 3 )
for Y2 and press EXE .

<Casio02.21>

4. Press F6 (DRAW) to display the graph of the


circle.

<Casio02.21>

5. To show the graph in true proportion, press


SHIFT F2 (ZOOM) then F6 ( ) for more
options. Press F2 (SQR).
<Casio02.21>

6. To view the coordinates of points along the circle


and to investigate the domain and range of the
relation, press SHIFT F1 (TRCE) and use the
arrow keys. <Casio02.21>

For the TI-Nspire CAS


1. Open a new Graphs & Geometry document
(press /N and then select 2: Add Graphs &
2
<TI02.23>
Geometry). Enter −2 + 9 – ( x + 3) in the
function entry line for f1(x).
96 M a t h s Q u e s t M a t h s B Ye a r 1 1 f o r Q u e e n s l a n d

2. Press · to display the upper semicircle.

<TI02.24>

2
3. Enter −2 − 9 – ( x + 3 ) in the function entry line
for f2(x) and then press · to display the lower
semicircle on the same set of axes.
<TI02.25>

4. To see the circle more clearly, press /G to hide


the function entry line. The equation labels can
also be moved. Refer to page 87 for instructions
on how to do this. <TI02.26>
(To magnify the size of a circle (and keep the
true proportion), we can use the zoom function.
Press b and then select 4: Window followed by
3: Zoom - In. Use the NavPad to move the pointer
to the centre of the circle and then press · to
display the new graph.)

5. To view the coordinates of points along the circle


and to investigate the domain and range, press b
and then select 5: Trace followed by 1: Graph
Trace. Use the NavPad to move the point (or <TI02.27>
trace) along the curve. (Note that, in the second
screen at right, the small upper case M shown in a
box indicates that this is a maximum point.)

<TI02.28>

remember
1. The general equation of a circle with centre (h, k) and radius r is
(x − h)2 + (y − k)2 = r2.
2. An ‘upper semicircle’ with centre (0, 0) and radius r is y = r 2 – x 2 .
3. A ‘lower semicircle’ with centre (0, 0) and radius r is y = – r 2 – x 2 .
Chapter 2 Relations and functions 97

2F Circles

1 State the equation of each of the circles graphed below.


eBook plus a y b y c y d y
3 10
1 5
Digital doc:
EXCEL Spreadsheet
Circle graphs –3 0 3x –1 0 1 x –5 0 5 x –10 0 10 x
–1 –5
–3 –10

e y f y g y h y
6
2 2
3
–4 0 4 x
– 6 0 6x –2 2 0 2 2x
–3 0 3x –4
– 6 –2 2

2 State the domain and range of each circle in question 1.

WORKED 3 Sketch the graph of each of the following relations.


Example
14 a x2 + y2 = 4 b x2 + y2 = 16 c x2 + y2 = 49
d x2 + y2 = 7 e x2 + y2 = 12 f x2 + y2 = 1---
4

4 Sketch the graph of each of the following relations and state whether it is a function
or not.
a y = ± 81 – x 2 b y = 4 – x2 c y = – 1 – x2

1 1
d y = --- – x2 e y = – --- – x2 f y = 5 – x2
9 4

g y = ± 10 – x 2 h x 2 + y 2 = 3, – 3 ≤ x ≤ 0

5 multiple choice
Consider the circle below.
y
2

0 2 4 x
–2

a The equation of the circle is:


A x2 + (y − 2)2 = 4 B (x − 2)2 + y2 = 16 C (x + 2)2 + y2 = 16
D (x − 2)2 + y2 = 4 E (x + 2)2 + y2 = 4
b The range of the relation is:
A R B [−2, 2] C [0, 4] D [2, 4] E [−2, 1]
98 M a t h s Q u e s t M a t h s B Ye a r 1 1 f o r Q u e e n s l a n d

6 multiple choice
Consider the equation (x + 3)2 + (y − 1)2 = 1.
a The graph which represents this relation is:
A y B y C y
4
2 2
1 1
1
0 x 0 2 3 4 x 0 x
–6 –3 –4 –3 –2
–2
D y E y
2
0 2 3 4 x
–1 1
–2

–3.5 –3 –2.5 0 x

b The domain of the relation is:


A [−3.5, −2.5] B (−4, −2) C R
D [2, 4] E [−4, −2]
WORKED 7 Sketch the graph of the following circles. State the domain and range of each.
Example
15 a x2 + (y + 2)2 = 1 b x2 + (y − 2)2 = 4
2 2
c (x − 4) + y = 9 d (x − 2)2 + (y + 1)2 = 16
e (x + 3)2 + (y + 2)2 = 25 f (x − 3)2 + (y − 2)2 = 9
2 2
g (x + 5) + (y − 4) = 36 h (x − --1- )2 + (y + --3- )2 = --9-
2 2 4
8 Express the relation x2 + y2 = 36 as two functions and state the largest domain and
range of each.
9 Express the relation x2 + ( y − 2)2 = 9 as two functions stating the largest domain and
range of each.
10 Circular ripples are
formed when a water drop
hits the surface of a pond.
If one ripple is represented
by the equation x2 + y2 = 4
and then 3 seconds later
by x2 + y2 = 190, where
the length of measure-
ments are in centimetres,
a find the radius (in cm)
of the ripple in each
case
b calculate how fast the
ripple is moving out-
wards.
(State your answers to
1 decimal place.)
Chapter 2 Relations and functions 99
Functions and modelling
When using functions to model rules in real-life situations, the domain usually has
practical restrictions imposed on it. For example, the area of a circle is determined by
the function A(r) = π r 2.
For a circle to be drawn the radius needs to be a positive number. Hence, the domain
is (0, ∞) or R+.

WORKED Example 16
The table below describes hire rates for a removal van.
a Express the cost as a hybrid function.
b Sketch the graph of the function.

Hours of hire (h) Cost ($C)


Up to 3 200
Over 3 up to 5 300
Over 5 up to 8 450

THINK WRITE
a 1 The cost is $200 if 0 < h ≤ 3. a
2 The cost is $300 if 3 < h ≤ 5.
3 The cost is $450 if 5 < h ≤ 8.
 200, 0<h≤3

4 State the cost function C(h). C ( h ) =  300, 3<h≤5

 450, 5<h≤8
b Sketch a graph with 3 horizontal lines b C ($)
over the appropriate section of the 450
domain. 400
350
300
250
200
150
100
50
0 1 2 3 4 5 6 7 8 h (hours)

remember
When using functions to model situations:
1. form an equation involving one variable and sketch a graph
2. use the graph to determine domain and range etc.
100 M a t h s Q u e s t M a t h s B Ye a r 1 1 f o r Q u e e n s l a n d

2G Functions and modelling

WORKED 1 The cost of hiring a paper recycling Hours of hire Cost


Example
16
removalist is described in the table
at right. Up to 1 $40
a State the cost function, $C, Over 1 up to 2 $70
in terms of the time, t hours,
for hiring up to 6 hours. Over 2 up to 4 $110
b Sketch the graph of the Over 4 up to 6 $160
function.
2 A mobile phone plan charges a 30c connection fee and then 40c for each 30 seconds
or part thereof.
a State the cost function for the time of the call, t, from t = 0 s to t = 300 s.
b Sketch the graph of the function.

3 A car travels at a constant speed of 60 km/h for 1 1--2- hours, stops for half an hour
and then travels for another 2 hours at a constant speed of 80 km/h, reaching its
destination.
a Construct a function that describes the distance travelled by the car, d (km), at
time, t hours.
b State the domain and range of this function.
c Calculate the distance travelled after: i 1 hour ii 3 hours.

4 At a fun park, a motorised toy boat operates for 5 minutes for every dollar coin placed
in a meter. The meter will accept a maximum of 120 one-dollar coins.
a Write a rule which gives the time of boat operation, B hours, in terms of the
number of dollar coins, n.
b Sketch the graph of the function and state the domain and range.
c How much is in the meter when the boat has operated for 450 minutes?
5 Suppose the tax payable for those who earn a taxable income between $20 700 and
$38 000 is $3060 plus 34 cents for every dollar earned over $20 700.
a Write a rule for the tax payable, $T, for a taxable income, $x, where
20 701 ≤ x ≤ 38 000.
b Sketch a graph of this function.
c Calculate the tax paid on an income of $32 000.
Chapter 2 Relations and functions 101
6 The maximum side length of the rectangle
shown is 10 metres. (x + 4) m
a Write a function which gives the perimeter, (x – 1) m
P metres, of the rectangle.
b State the domain and range of this function.
7 A rectangular swimming pool is to have a length 4 metres greater than its width.
a Write a rule for the area of the pool, A m2, as a function of the width, x metres.
b State the domain and range if the maximum side length is 12 metres.
8 Timber increases in value (appreciates) by 2% each year. If a consignment of timber
is currently worth $100 000:
a Express the value of the timber, P dollars, as a function of time, t, where t is the
number of years from now.
b What will be the value of the timber in 10 years?

9 The number of koalas remaining in a parkland


t weeks after a virus strikes is given by the
96
function N ( t ) = 15 + ----------- koalas per hectare.
t+3
a How many koalas per hectare were there
before the virus struck?
b How many koalas per hectare are there
13 weeks after the virus strikes?
c How long after the virus strikes are there
23 koalas per hectare?
d Will the virus kill off all the koalas? Explain
why.

10 A school concert usually attracts 600 people at


a cost of $10 per person. On average, for every
$1 rise in admission price, 50 less people attend
the concert. If T is the total amount of takings
and n is the number of $1 increases:
a write the rule for the function which gives T in terms of n
b sketch the graph of T versus n
c find the admission price which will give the maximum takings.
102 M a t h s Q u e s t M a t h s B Ye a r 1 1 f o r Q u e e n s l a n d

summary
Set notation
• {. . .} refers to a set of something.
• ∈ means ‘is an element of’.
• ∉ means ‘is not an element of’.
• ⊂ means ‘is a subset of’.
• ⊄ means ‘is not a subset (or is not contained in)’.
• ∩ means ‘intersection with’.
• ∪ means ‘union with’.
• \ means ‘excluding’.
• ∅ refers to ‘the null, or empty set’.
• {(a, b), (c, d), . . .} is a set of ordered pairs.
• A relation is a set of ordered pairs.
• N refers to the set of Natural numbers.
• J refers to the set of integers.
• Q refers to the set of rational numbers.
• R refers to the set of Real numbers.
• R+ refers to the set of positive Real numbers.
• R− refers to the set of negative Real numbers.

Relations and graphs


• The independent variable (for example, x) is shown on the horizontal axis of a
graph.
• The dependent variable (for example, y) is shown on the vertical axis of a graph.
• Discrete variables are things which can be counted.
• Continuous variables are things which can be measured.

Domain and range


• The domain of a relation is the set of first elements of a set of ordered pairs.
• The range of a relation is the set of second elements of a set of ordered pairs.
• The implied domain of a relation is the set of first element values for which a rule
has meaning.
• In interval notation a square bracket means that the end point is included in a set of
values, whereas a curved bracket means that the end point is not included.
(a, b]
a b
Chapter 2 Relations and functions 103
Types of relations (including functions)
• A function is a relation which does not repeat the first element in any of its ordered
pairs. That is, for any x-value there is only one y-value.
• The graph of a function cannot be crossed more than once by any vertical line.

Function notation
• f (x) = . . . is used to describe ‘a function of x’. To evaluate the function, for example
when x = 2, find f (2) by replacing each occurrence of x on the RHS with 2.
• Functions are completely described if the domain and the rule are given.

Special types of function


• A function is one-to-one if for each x-value there is only one y-value and vice versa.
• A many-to-one function may be ‘converted to’ a one-to-one function by restricting
the domain.
• A hybrid function obeys different rules for different subsets of the domain.

Inverse relations and functions


• An inverse relation is obtained by interchanging the x and y values of the original
relation.
• The graph of a function and its inverse are reflections of each other across the line
y = x.

Circles
• The general equation of a circle with centre (h, k) and radius r is
(x − h)2 + (y − k)2 = r 2
• An ‘upper semicircle’ with centre (0, 0) and radius r is y = r 2 – x2 .
• A ‘lower semicircle’ with centre (0, 0) and radius r is y = − r 2 – x 2 .

Functions and modelling


• When using functions to model situations:
1. form an equation involving one variable and sketch a graph
2. use the graph to determine domain and range etc.
104 M a t h s Q u e s t M a t h s B Ye a r 1 1 f o r Q u e e n s l a n d

CHAPTER
review
1 multiple choice
2A
y The rule describing the relation shown is:
8 A y = 2x
6 B y = 2x, x ∈ {1, 2, 3, 4}
C y = 2x, x ∈ N
4
x
2 D y = ---
2
0 1 2 3 4 x E y = 2x, x ∈ R+

2 multiple choice
2A
Which one of the relations graphed below is continuous?
A y B y C y

0 x 0 x
0 x

D y E y

0 x
0 x

3 The total number of cars that have entered a car park during the first 5 hours after opening
2A is shown in the table below.

Time, t (hours) 1 2 3 4 5

Number of cars, n 30 75 180 330 500

a Plot these points on a graph.


b Explain why the dots cannot be joined.
c Estimate the number of cars in the park 2 1--- hours after the car park opens.
2

4 a Sketch the graph of the relation {(x, y): y = 1 − x2, x ∈ [−3, 3]}.
2A b State the domain and range of this relation.
Chapter 2 Relations and functions 105
5 multiple choice
2B
The interval shown below is:

–5 –1 0 1 4 x
A [−5, −1] ∪ [0, 4] B [−5, −1) ∪ [0, 4]
C (−5, −1) ∪ (1, 4] D (−5, −1) ∪ (1, 4)
E [−5, −1) ∪ (1, 4]

6 multiple choice
2B
The set (0, 2) ∪ (2, ∞) is correctly represented on which number line below?
A B
0 2 x 0 2 x

C D
0 2 x 0 2 x

E
0 2 x

7 multiple choice
y 2B
The domain of the relation shown at right is: 4
A (–∞, 0) ∪ (0, ∞) B (–∞, 1) ∪ (1, ∞)
2
C R D (–∞, 1] ∪ (1, ∞)
E (–∞, 0) ∪ [0, ∞)
0 1 x

8 multiple choice
2B
A relation has the rule y = x + 3, where x ∈ R+. The range of this relation is:
A R+ B [0, ∞)
C [3, ∞) D R
E (3, ∞)

9 multiple choice
1 2B
The implied domain of the relation described by the rule y = ---------------- is:
x–5
+
A (5, ∞) B R C [5, ∞) D (0, 5) E R−

10 multiple choice
2B
The range of the function, f (x) = 2 4 – x is:
A R B R+ C R− D [0, ∞) E (2, ∞)

11 multiple choice
y 2C
The relation shown is:
A one-to-one B one-to-many C many-to-many
0 x D many-to-one E none of the above
106 M a t h s Q u e s t M a t h s B Ye a r 1 1 f o r Q u e e n s l a n d

12 multiple choice
2C Which of the following is not a relation?
A y = x2 B x2 + y2 = 3 C {(1, 1), (2, 1), (3, 2), (4, 3)}
D y=5−x E {1, 3, 5, 7, 9}

13 multiple choice
2C Which one of the relations graphed below is not a function?
A y B y C y

0 x 0 x

0 x
D y E y

0 x
0 x

14 multiple choice
2C Which of the following rules does not describe a function?
x
A y = --- B y = 2 − 7x C x=5 D y = 10x2 + 3 E y = −8
5
15 Determine which of the following relations are functions.
2C a y = 2x2 − 1 b 3x + y = 2
2
c x=y +1 d x2 + y2 = 10
e y3 = x f y2 − x2 = 1

16 multiple choice
2D Which of the functions listed below is not one-to-one?
A {(10, 10), (11, 12), (12, 13)} B {(5, 8), (6, 10), (7, 8), (8, 9)}
C {(x, y): y = 4x} D {(x, y): y = 5 − 2x}
E f (x) = 2 − x3

17 multiple choice
2D Which of the graphs below represents a one-to-one function?
A y B y C y

0 x 0 x
0 x

D y E y

0 x
0 x
Chapter 2 Relations and functions 107
18 If g(x) = x + 2, where x ≥ 0, then find:
a g(x2) 2D
b the domain and range of g(x).

19 multiple choice
2D
The function f : {x: x = 0, 1, 2} → R, where f (x) = x − 4, may be expressed as:
A {(0, −4), (1, −3), (2, −2)} B {0, 1, 2} C {(0, 4), (1, 3), (2, 2)}
D {(−1, −5), (1, −3), (2, −2)} E {−4, −3, −2}

20 multiple choice
2D
If g(x) = 6 − x + x2, then g(−2) is equal to:
A 6 B 8 C 0 D 12 E 5

21 multiple choice
2D
If f (x) = 3x − 5, then f (2x + 1) is equal to:
A 6x − 8 B 6x − 5 C 3x − 5 D 3x − 4 E 6x − 2

22 multiple choice
2D
The hybrid function
 x + 1, x<0

f ( x ) =  x2, 0≤x≤2

 2 – x, x>2
is represented by which of the following graphs?
A y B y C y
4 4 4

1 1 1
–1 0 1 2 x –1 0 2 x 0 2 x

D y E y
4 4

1 1
–1 0 2 x –1 0 2 x

23 Express the following rules in full function notation.


1
2D
a y = ---
x
b y = (2 – x)
24 Sketch the graph of the function described below.
2D
 2 – x, x ≤ –1

f ( x ) =  3, –1 < x < 3

 2x – 5, x≥3
108 M a t h s Q u e s t M a t h s B Ye a r 1 1 f o r Q u e e n s l a n d

25 multiple choice
2E y
The inverse of the graph shown at right is:

A y B y C y

x x x

D y E y

x x

26 multiple choice
2F y
The equation of the circle shown is: 2
A (x + 3)2 + y2 = 4 B (x − 3)2 + y2 = 2
2 2
C (x + 3) + y = 2 D (x − 3)2 + y2 = 4 0 1 3 5 x
E x2 + (y − 3)2 = 4 –2

The circle with equation (x + 1)2 + (y − 4)2 = 9 applies to questions 28 and 29.
27 Sketch the graph of each of the following, stating the domain and range.
2F
a y = – 1 – x2
b (x − 2)2 + (y + 1)2 = 9

28 multiple choice
2F
The domain is:
A [−10, 8] B [−2, 4] C (−2, 4) D [−3, 3] E [−4, 2]

29 multiple choice
2F
The range is:
A [−7, −1] B [−5, 13] C [1, 7] D [−3, 3] E (1, 7)

30 multiple choice
2F
A circle has its centre at (4, −2) and a radius of 5 . The equation of the circle is:
A (x − 4)2 + (y + 2)2 = 25 B (x − 4)2 + (y + 2)2 = 5 C (x + 4)2 + (y − 2)2 = 5
D (x + 4)2 + (y − 2)2 = 25 E 4x2 − 2y2 = 5
Chapter 2 Relations and functions 109
31 a Sketch the graph of the relation x2 + y2 = 100.
b From this relation form two one-to-one functions and state the domain and range of each. 2F
32 multiple choice
The table of maths tutoring fees charged by
2G
a Year 11 student is as follows:
Hours (h) Charge (C$)
0<h≤2 50
2<h≤4 80
4<h≤6 100

Which of the following graphs best shows


the information in the preceding table?
A c B c C c
100 100 100

50 50 50

0 2 4 6 h 0 2 4 6 h 0 2 4 6 h

D c E c
100 100

50 50

0 2 4 6 h 0 2 4 6 h

33 A chicken farmer delivers chicken manure


according to the following fee schedule: 2G
Less than half a truckload: $50
Half to a full truckload: $75
FIG 2.202
More than 1 but less than 2 truckloads: $100
Sketch a graph showing this information.

Modelling and problem solving


1 Consider the diagram shown at right.
10 m
a Find an expression for the area, A, in terms of x and y.
b Find an expression for the perimeter, P. xm
c If the perimeter is 72 cm, express A as a function of x. ym
xm
d What is the domain of A(x)?
e Sketch the graph of this function.
f Hence find the maximum area.
110 M a t h s Q u e s t M a t h s B Ye a r 1 1 f o r Q u e e n s l a n d

2 For the graph shown: y (3, 18)


18

a state the domain


b state the range
c find the rule for x ∈ (−∞, −2)
4
d find the rule for x ∈ (−2, 0]
e find the rule for x ∈ [0, 3],
given it is of the form y = ax2 –2 0 4 x
f determine the rule when x ≥ 3  …, …

g describe the relation using hybrid function notation of the form f ( x ) =  …, …

 …, ….
3 A function f is defined as follows: f:[–2, a] → R, where f (x) = (x – 1)2 – 4.
a Find f (–2), f (–1), f (0), f (1), f (3).
b If f (a) = 12, find the value of a.
c Sketch the function f, labelling the graph appropriately.
d From the graph or otherwise, state the:
i domain of f (x) ii range of f (x).
4 The perimeter for a new rectangular
penguin enclosure is to have a
maximum side length of 8 m. The
width is to be twice the length (x).
a Draw a diagram of the enclosure
and label the sides.
b Define a rule which gives the
perimeter, P, of the new enclosure.
c What is the largest value that x
can be?
d State the domain and range.
e Write, in function notation, the
rule for the perimeter.
f Define the function for the area of the enclosure, A(x).
g If the maximum area allowed is 18 m2, find the dimensions of the enclosure.
5 Thomas is looking to connect to a mobile phone service. He has to decide on one of two plans
from Busytone Communications. The details are as follows.
Plan Flag fall (cents) Cost per minute (cents)
A 20 30
B 50 20
a Write a function A for the cost of making a call from plan A.
b Write a function B for the cost of making a call from plan B.
c What is the cost of a 2-minute call from:
i plan A? ii plan B?
eBook plus d What is the length of one call costing $5 from:
i plan A? ii plan B?
Digital doc: e How long would a call be for the call costs to be the same for both plans?
Test Yourself
Chapter 2
f If Thomas frequently makes calls lasting more than 4 minutes, which plan should he
connect with?
Other graphs
and modelling
3
syllabus reference
Topic:
• Introduction to functions

In this chapter
3A Transforming graphs
3B Sketching graphs using
transformations
3C Sketching graphs using
intercepts
3D The hyperbola
3E The square root function
3F The absolute value function
3G Addition of ordinates
3H Modelling
3I Modelling using a graphics
calculator
112 M a t h s Q u e s t M a t h s B Ye a r 1 1 f o r Q u e e n s l a n d

Introduction
In a science experiment, Katie drops a rock from a window 4 metres above the ground.
Using electronic equipment, she and her partner, Tegan, measure the height at various
times. Katie takes the data and produces the following table and graph.

4.5
Time Distance fallen
4.0
(s) (m)

Distance fallen (m)


3.5
0 0.0 3.0
2.5
0.25 0.2 2.0
1.5
0.5 0.6
1.0
0.75 1.4 0.5
0.0
1 2.5 0 0.2 0.4 0.6 0.8 1 1.2 1.4
Time (s)
1.25 3.9

Tegan takes the data and produces a different table and graph.

4.5
Distance above ground (m)

Time Distance above


4.0
(s) ground (m)
3.5
0 4.0 3.0
2.5
0.25 3.8 2.0
0.5 3.4 1.5
1.0
0.75 2.6 0.5
0.0
1 1.5 0 0.2 0.4 0.6 0.8 1 1.2 1.4
Time (s)
1.25 0.1

Although the graphs appear different they have the same underlying shape. How can
we transform one graph into the other?
In the discussion that follows, we could discuss particular functions like y = x2 or
y = x3. Instead, the discussion will refer to a general function and its graph y = f(x).
Recall that a function, f(x), takes a number, x, and assigns to it another number, f(x).
For example, if f(x) = x2 + 4 , then x = 3 is linked to f(3) where f(3) = 32 + 4 = 13.

y
13 f (x) = x2 + 4

0 3 x
Chapter 3 Other graphs and modelling 113
Transforming graphs
Dilation y
A dilation is a stretching or compressing of a graph.

Let the basic graph be y = f(x).


y = f (x)

0 x

If y = af(x) the graph is stretched in the y direction by y


a factor of a. Note that if 0 < a < 1 then the graph
y = af (x)
will actually shrink in the y direction.
y = f (x)

0 x

If y = f(ax), the graph is stretched in the x direction y


1
by a factor of --- .
a y = f (x)

0 x

y = f (ax)

Reflection
A graph can be transformed by reflecting it in either the x-axis or the y-axis.

If y = f(x) then y = f(−x) represents a reflection in the y


y-axis.

y = f(–x)

0 x

y = f(x)

If y = f(x) then y = −f(x) represents a reflection in the y


x-axis.
y = f(x)

0 x
y = –f(x)
114 M a t h s Q u e s t M a t h s B Ye a r 1 1 f o r Q u e e n s l a n d

Translation y
A translation is a shift of a graph.
y = f(x)
If y = f(x) then y = f(x − a) is a translation where a y = f(x – a)
each x-value is moved a units to the right.
0 x

If y = f(x) then y = f(x) + a is a translation where y


each y-value is moved a units upwards.
y = f(x)+a

In summary, the graph of y = af(b(x − h)) + k can a


be obtained from the graph of y = f(x) by performing y = f(x)
the following transformations (in order): 0 x
1. Stretch along the y-axis by a factor of a.
1
2. Shrink along the x-axis by a factor of --- .
b
3. Move the graph h units to the right.
4. Move the graph k units up.
Knowledge of transformations will help in drawing a rough sketch of a graph in
order to understand its general features. If an accurate drawing of a graph is required, a
graphics calculator can be used.

WORKED Example 1
Consider the graph of y = f(x) at far right. II y
y
From the graph near right, select the one
III y = f(x)
that best matches each of the following
functions. I
a y = f(−x) IV
0 x 0 x
b y = f(x) + 3
c y = f(x − 2)
d y = 2f(x)

THINK WRITE
a The graph of y = f(−x) represents a a Graph IV matches the equation y = f(−x) as it
reflection in the y-axis of the original is a reflection of the original function in the
graph. y-axis.
b The graph of y = f(x) + 3 represents a b Graph III matches the equation y = f(x) + 3
translation of 3 units upwards. (That is, as it has the same shape as the original
3 units in the positive y direction.) Note function and has been translated 3 units
that the same shape is maintained. upwards.
c The graph of y = f(x – 2) represents a c Graph I matches the equation y = f(x − 2) as
translation of 2 units to the right. (That it has the same shape as the original function
is, 2 units in the positive x direction). and has been translated 2 units to the right.
Note that the same shape is maintained.
d The graph of y = 2f(x) represents a d Graph II matches the equation y = 2f(x) as it
dilation in the y direction. Each y-value represents a stretching of the original
has been multiplied by 2. function in the y direction.
Chapter 3 Other graphs and modelling 115
WORKED Example 2
Consider the graph of y = f(x) shown at right. y
Use your knowledge of transformations to sketch
the graph of each of the following functions.
a y = 3f(x)
b y = f(x + 2) 0 x
c y = −f(2x)
d y = −f(x) + 3
y = f(x)

THINK WRITE

a The graph of y = 3f(x) will be the same as a y


y = 3f(x)
the graph of y = f(x) but stretched by a factor
of 3 along the y-axis.
y = f(x)
0 x

b The graph of y = f(x + 2) will be the same as b y


the graph of y = f(x) but moved 2 units to the
left.
Note that there is not enough information y = f(x + 2) y = f(x)
to know exactly how far ‘2 units’ is so a 0 x
number of answers are possible.

c The graph of y = −f(2x) will be the same c


as the graph of y = f(x) but shrunk along the y
1
x-axis by a factor of --- and reflected in the y = f(x)
2
x-axis.
0 x

y = –f(2x)

d The graph of y = −f(x) + 3 is the same as d


the graph of y = f(x) but it is reflected in the y
x-axis and lifted upwards by 3 units. y = f(x)

0 x

y = –f(x) + 3
116 M a t h s Q u e s t M a t h s B Ye a r 1 1 f o r Q u e e n s l a n d

remember
1. The graph of y = f(x) can be transformed through dilation, reflection or
translation.
2. y = af(x) represents a dilation of y = f(x) in the y direction by a factor of a.
1
y = f(ax) represents a dilation of y = f(x) in the x direction by a factor of --- .
y = f(−x) represents a reflection of y = f(x) in the y-axis. a
y = −f(x) represents a reflection of y = f(x) in the x-axis.
y = f(x − a) represents a translation of y = f(x) of a units to the right.
y = f(x) + a represents a translation of y = f(x) of a units upwards.
3. The graph of y = af(b(x − h)) + k can be obtained from the graph of y = f(x) by
performing the following transformations (in order):
(a) Stretch along the y-axis by a factor of a.
1
(b) Shrink along the x-axis by a factor of --- .
(c) Move the graph h units to the right. b
(d) Move the graph k units upwards.

3A Transforming graphs
WORKED 1 Consider the graph of y = f(x) shown at right. y
Example
1
From the graphs below, select the one that
best matches each of the following functions.
a y = f(−x) y III
b y = f(x) + 3
II
c y = f(x − 2)
0 x
d y = 2f(x) IV y = f(x)

0 x
I

2 Consider the graph of y = f(x) shown at right. y


From the graphs below, select the one that
best matches each of the following functions.
a y = −f(x) y
y = f(x)
b y = f(x + 1)
c y = f(x) − 2 IV
0 x
d y = 0.5f(x) III

0 x

II
Chapter 3 Other graphs and modelling 117
WORKED 3 Consider the graph of y = f(x) shown at right. y
Example
Use your knowledge of transformations to sketch y = f(x)
2
the graph of each of the following functions.
a y = 2f(x)
b y = f(x − 2) 0 x
c y = −f(0.5x)
d y = −f(x) + 2
4 Consider the graph of y = f(x) shown at right. y
Use your knowledge of transformations to sketch y = f(x)
the graph of each of the following functions.
a y = 3f(x)
b y = f(x + 3) 0 x
c y = −f(x)
d y = f(x) − 5
5 Consider the graph of y = f(x) shown at right. y
Use your knowledge of transformations to sketch
the graph of each of the following functions.
a y = 2f(x) + 1 y = f(x)
b y = f(x − 3) + 4
c y = f(x + 1) − 3 0 x
d y = −f(x + 4) − 2

Sketching graphs using transformations


So far we have considered a general function y = f(x) and y
applied different transformations. Now we will consider trans-
formations of the graphs of basic quadratic, cubic and quartic
functions. y = x2

The graph of the basic quadratic function is y = x2. This 0 x


graph is a parabola with a turning point at (0, 0). y
y = x3

The graph of the basic cubic function is y = x3. This graph


has a stationary point of inflection at (0, 0). 0 x

y
The graph of the basic quartic function is y = x4. This graph
has a turning point at (0, 0). Note that this graph is not a
parabola, although it looks similar. y = x4

0 x

All three of these basic graphs can undergo transformations of dilation, reflection
and translation. Complete the following activity to investigate the effect of these trans-
formations.
118 M a t h s Q u e s t M a t h s B Ye a r 1 1 f o r Q u e e n s l a n d

Investigating transformations on the


basic graphs of y = x2, y = x3 and y = x4
Use a graphics calculator or other graphing technology to investigate the effect
eBook plus
of different transformations on the basic graphs of y = x2, y = x3 and y = x4. The
Weblinks: following steps may guide you.
Graphmatica software 1 Draw the basic graph of y = x2. This is a graph of y = ax2 where a = 1.
Equation grapher software
Investigate the effect of changing the value of a on the shape of the graph.
Use both positive and negative values and make comparisons with the
basic graph of y = x2.
2 Consider the graph of y = (x − h)2. Draw a number of graphs to investigate
the effect on the basic graph of changing the value of h. Use both positive
and negative values. How does this affect the turning point of the parabola?
3 Consider the graph of y = x2 + k. Draw a number of graphs to investigate
the effect on the basic graph of changing the value of k. Use both positive
and negative values. How does this affect the turning point of the
parabola?
4 Repeat steps 1 to 3 for the basic cubic graph of y = x3. In this case, consider
the effect on the stationary point of inflection rather than a turning point.
5 Repeat steps 1 to 3 for the basic quartic graph of y = x4.
6 Use a graphics calculator or other graphing technology to investigate the
overall effect on the basic graphs when changing a, h and k to produce graphs
of y = a(x − h)2 + k, y = a(x − h)3 + k and y = a(x − h)4 + k.
7 Write a report on your findings.

Graphing the parabola using transformations


Throughout this chapter we refer to the graph of y = x2 as the basic parabola. From the
previous investigation, you were able to consider the effect of various transformations
on the graph of this basic parabola.
For y = ax2 a=2
y a=1 1
a is the dilation factor. a = –2
It dilates the graph in the y direction.
The larger a is, the thinner the graph of the parabola.
If a is a proper fraction, the graph is wider than the basic 0 x
parabola.
If a is negative, the graph is reflected in the x-axis.
a = –1
y = ax2
For y = (x – h)2 y
h translates the basic parabola h units along the x-axis. h = –3 h=2
If h > 0, the basic graph is moved to the right.
If h < 0, the basic graph is moved to the left.
0 x
–3 2
2
y = (x – h)
Chapter 3 Other graphs and modelling 119
For y = x2 + k y k=2
k translates the basic parabola k units along the y-axis. k = –1
If k > 0, the basic graph is moved up.
If k < 0, the basic graph is moved down. 2

0 x
–1 y = x + k
2

The graph of y = a(x − h)2 + k shows the combination of y y = a(x – h)2 + k


these transformations. The turning point of the graph has
coordinates (h, k). The equation is known as the turning point
form of the function.
(h, k)
0 x

Graphing the cubic function using transformations


Throughout this chapter we refer to the graph of y = x3 as the basic cubic graph. From
the previous investigation, you were able to consider the effect of various transform-
ations on the graph of this basic cubic.
For y = ax3 y a=2
a is the dilation factor. a = –1 a=1
a = 1–2
It dilates the graph in the y direction.
The larger a is, the thinner the graph of the cubic.
If a is a proper fraction, the graph is wider than the 0 x
basic cubic.
If a is negative, the graph is reflected in the x-axis.
y = ax3
3
For y = (x - h) y
h translates the basic cubic h units along the x-axis. h = –3 h=2
If h > 0, the basic graph is moved to the right.
If h < 0, the basic graph is moved to the left.
0 x
–3 2

y = (x – h)3

For y = x3 + k y k=1
k translates the basic cubic k units along the y-axis.
k = –2
If k > 0, the basic graph is moved up. 1
If k < 0, the basic graph is moved down. 0 x
–2

y = x3 + k
3
The graph of y = a(x − h) + k shows the combi- y = a(x – h)3 + k
y
nation of these transformations. The stationary point
of inflection has coordinates (h, k).
(h, k)
0 x
120 M a t h s Q u e s t M a t h s B Ye a r 1 1 f o r Q u e e n s l a n d

Graphing the quartic function using transformations


Throughout this chapter we refer to the graph of y = x4 y a=2
a = 1 1_
as the basic quartic graph. From the previous investi- a =2
gation, you were able to consider the effect of various
transformations on the graph of this basic quartic.
For y = ax4 0 x
a is the dilation factor.
It dilates the graph in the y direction.
The larger a is, the thinner the graph of the quartic. a = –1
If a is a proper fraction, the graph is wider than the y = ax4
basic quartic. y
h = –3 h=2
If a is negative, the graph is reflected in the x-axis.

–3 0 2 x
y = (x – h)4
For y = (x – h)4
y
h translates the basic quartic h units along the x-axis. k=2
If h > 0, the basic graph is moved to the right.
k = –1
If h < 0, the basic graph is moved to the left.
2
For y = x4 + k
k translates the basic quartic k units along the y-axis. 0 x
If k > 0, the basic graph is moved up. –1 y = x4+k
If k < 0, the basic graph is moved down.
y
4
The graph of y = a(x − h) + k shows the combi-
nation of these transformations. The turning point has
coordinates (h, k).
(h, k)
0 x
y = a(x – h)4+k

WORKED Example 3
State the changes required to transform the graph of y = x2 into the graph of y = 2(x + 3)2 − 4.
THINK WRITE
1 Write the general formula for the parabola. y = a(x − h)2 + k
2 Identify the value of a. a=2
3 State the effect of a on the graph. The graph of y = x2 is dilated by a factor of 2 in
the y direction.
4 Identify the value of h. h = −3
5 State the effect of h on the graph. The graph is translated 3 units to the left.
6 Identify the value of k. k = −4
7 State the effect of k on the graph. The graph is translated 4 units down.
Chapter 3 Other graphs and modelling 121
WORKED Example 4
Sketch the graph of y = 1--2- (x − 1)2 + 2, clearly showing the coordinates of the turning point
and the intercepts with the axes.
THINK WRITE
1 Write the general equation of the parabola. y = a(x − h)2 + k
2 Identify the values of the variables. a = 1--- , h = 1, k = 2
2
3 Write a brief statement on the transformation The graph of y = x2 is dilated in the y
of the basic parabola. direction by the factor of 1--- (that is, it is
2
wider than the basic curve); it is translated
1 unit to the right and 2 units up.
4 State the shape of the parabola (that is, positive Since a > 0, the parabola is positive.
or negative).
5 State the coordinates of the turning point (h, k). Turning point (1, 2)
6 As both a and k are positive, only the y-intercept: when x = 0,
y-intercept needs to be determined. Find the y = 1--- (0 − 1)2 + 2
2
y-intercept by making x = 0.
y = 1--- (−1)2 + 2
2
1
y= --- +2
2
y = 2 1---
2
7 Sketch the graph: y
Draw a set of axes and label them. Plot the
turning point and the y-intercept. Sketch the 1
2 –2
graph of the positive parabola, so that it 2
passes through the points previously marked.
0 x
1
y = 1–2 (x – 1)2 + 2

remember
1. The graph of y = x2 is called a basic parabola.
2. The graph of y = x3 is called a basic cubic graph.
3. The graph of y = x4 is called a basic quartic graph.
4. The graph of y = a(x − h)n + k where n = 2, 3 or 4 is the basic graph, dilated by
the factor a in the y direction, translated h units along the x-axis (to the right if
h > 0, or to the left if h < 0) and k units along the y-axis (up if k > 0, or down
if k < 0).
5. If a < 0, the graph is reflected in the x-axis.
y y = a(x – h)2 + k y y = a(x – h)3 + k y

(h, k)
0 x
(h, k) (h, k)
0 x
0 x y = a(x – h)4+k
122 M a t h s Q u e s t M a t h s B Ye a r 1 1 f o r Q u e e n s l a n d

Sketching graphs using


3B transformations
WORKED 1 State the changes required to transform the graph of y = x2 into the graph of each of
Example
3
the following.
a y = 2x2 b y= 1 2
--- x c y = −3x2
3
d y = x2 − 6 e y = 3x + 4 2
f y = 1 − 1--- x2
2
g y = (x − 2)2 h y = −(x + 3)2 i y = 2(3 − x)2
j y = (x + 2)2 − 1 k y = (x − 0.5)2 + 2 l y = 1 − 2(3 + x)2

2 Match the graphs of the parabolas with the following equations.


a y = x2 + 2 III II
y
b y = −2(x − 2)2 y = x2
2
c y = 2 − (x + 2)
d y = 1--- (2 − x)2 IV
2 2
e y = (2 + x)2 + 2

0 x
–2 2
I –2

WORKED 3 Sketch the graphs of the following, clearly showing the coordinates of the turning
Example
4
point and the intercepts with the axes.
a y = 2x2 + 3 b y = 1--- − 1--- x2 c y = −(x − 1)2
4 2

eBook plus d y = 3(2 − x)2 e y = 2(x + 3)2 + 2 f y = 1 − 4(2 − x)2

Digital docs: 4 State the changes necessary to transform the graph of y = x3 into the graph of each of
EXCEL Spreadsheet the following.
The parabola —
3
y = a (x − h )2 + k a y = 7x3 b y = − 2--3- x c y = x3 + 4
EXCEL Spreadsheet
Cubic graphs — d y = 6 − x3 e y = (x − 1)3 f y = −(x + 3)3
y = a (x − h )3 + k
EXCEL Spreadsheet g y = 4(2 − x)3 h y = (x − 2)3 + 1 i y = 3(x + 3)3 − 2
Function grapher

5 Which of these transformations were applied to the graph of y = x3 to obtain each of


the graphs below?
iii reflection in the x-axis iii translation to the left
iii translation to the right iv translation up
iv translation down
a y
b y
c y

0 x

0 x
0 x
Chapter 3 Other graphs and modelling 123
d y e y f y

0 x
0 x 0 x

6 Sketch the graph of each of the following, clearly showing the position of the
stationary point of inflection and the intercepts with the axes.
3 3
a y= --- x b y = 1 − 2x3 c y = 2--- x3 − 6
4 3

d y = 2(x − 4)3 e y = − 1--- (x − 2)3 f y = 4(1 − x)3


2
g y = (x − 1)3 + 2 h y = 3 − (x + 2)3 i y = 2(x + 1)3 − 6

7 State the changes necessary to transform the graph of y = x4 into the graph of each of
the following.
a y = −5x4 b y = x4 − 4 c y = (x − 1)4
4 4
d y = 2x + 3 e y = −(x − 2) f y = (x + 1)4 − 3

8 Sketch the graph of each of the following, clearly showing the position of the turning
point and the intercepts with the axes.
a y = 2x4 b y = x4 − 16 c y = (x − 3)4
d y = −x4 + 1 e y = (x + 2)4 − 1 f y = −3(x + 1)4 − 2

Sketching graphs using intercepts


In the previous section we looked at sketching graphs of quadratic, cubic and quartic
functions by transforming the basic graph of each. This is a suitable method if the
equations are expressed in the form y = a(x − h)n + k where n = 2, 3 or 4.
However, we often have the equations expressed in an expanded form. This form is
called the general form of the function. (Note the highest power of x in each case.)
General form of a quadratic function:
y = ax2 + bx + c (The highest power of x is 2.)
General form of a cubic function:
y = ax3 + bx2 + cx + d (The highest power of x is 3.)
General form of a quartic function:
y = ax4 + bx3 + cx2 + dx + e (The highest power of x is 4.)
To sketch these graphs, we draw a diagram showing the general shape of the func-
tion, the x- and y-intercepts and the general location of turning points.

Sketching a parabola using intercepts


To sketch a parabola we need to first recognise if it is a positive parabola or a negative
parabola. A positive parabola has a turning point that is a minimum. A negative
parabola has a turning point that is a maximum.
124 M a t h s Q u e s t M a t h s B Ye a r 1 1 f o r Q u e e n s l a n d

y y

x
x

Minimum parabola Maximum parabola


y = ax2 . . . (a is positive, not zero) y = ax2 . . . (a is negative)
To position the parabola on the axes, we need to know the values of the x-intercepts
(there could be 0, 1 or 2 x-intercepts) and the y-intercept. We can also find the coordi-
nates of the turning point.
To find the y-intercept, we substitute x = 0 into the equation and solve for the corre-
sponding y-value. That is, it is the value of c in the general form y = ax2 + bx + c.
Any x-intercepts are found by substituting y = 0 into the equation and solving for the
corresponding x-value. Let’s say the equation factorises to y = (x − d)(x − e); substituting
y = 0 into the equation gives 0 = (x − d)(x − e). For this to be true, x = d or x = e.

y
Axis of symmetry

y-intercept c x-intercepts

e x Turning point
d

If a quadratic function has two x-intercepts, the x-coordinate of the turning point may
be found by averaging them.
y

d+e
——
2
d e
x

If there is only one x-intercept, then it is the x-coordinate of the turning point.
y

d x

The y-coordinate of the turning point may be found by substituting the x-coordinate
into the original equation.
Alternatively, using xt and yt for the coordinates of the turning point, we have
–b b2
x t = ------ and y t = c – ------
2a 4a
Chapter 3 Other graphs and modelling 125
WORKED Example 5
Sketch graphs of the following, showing all intercepts and the turning point in each case.
a y = x2 − 4x − 32 b y = x2 + 10x + 25 c y = −2x2 + 11x − 15
THINK WRITE
a 1 Write the equation. Recognise that a y = x2 − 4x − 32
the graph will be a positive parabola.
2 Find the y-intercept (when x = 0). y-intercept: when x = 0, y = −32
Point: (0, –32)
3 Factorise before finding x-intercepts. y = (x + 4)(x − 8)
4 Find x-intercepts (when y = 0). x-intercepts: when y = 0,
0 = (x + 4)(x − 8)
x = −4 or x = 8
Points: (–4, 0), (8, 0)
Find the x-coordinate of the turning –4+8
5 Turning point: xt = ----------------
point (TP). 2
4
= ---
2
=2
6 Substitute into the original equation to
find the y-coordinate of the turning yt = 22 − 4(2) − 32
point. = 4 − 8 − 32
b2
Alternatively, use y t = c – ------ = −36
4a
7 Write the turning point coordinates. Turning point: (2, −36)
8 Combine the above information and y
sketch.

–4 8 x

(2, –36)

b 1 Write the equation. Recognise that b y = x2 + 10x + 25


the graph will be a positive parabola.
2 Find the y-intercept (when x = 0). y-intercept: when x = 0, y = 25
Point: (0, 25)
3 Factorise before finding x-intercepts. y = (x + 5)(x + 5)
= (x + 5)2
4 Find the x-intercept (when y = 0). x-intercept: when y = 0,
0 = (x + 5)2
x = −5
Point: (–5, 0)
–b
5 Find the x-coordinate of the turning Turning point: xt = ------ , b = 10, a = 1
2a
point.
– 10
Alternatively, since there is only = -----------
2(1)
one x-intercept (−5), it must be the
= −5
turning point x-coordinate.
Continued over page
126 M a t h s Q u e s t M a t h s B Ye a r 1 1 f o r Q u e e n s l a n d

THINK WRITE

6 Substitute into the original equation yt = (–5)2 + 10(–5) + 25


to find the y-coordinate of the turning = 25 – 50 + 25
point. =0
7 Write the turning point coordinates. Turning point: (−5, 0)
8 Combine the above information and y
sketch.
25
(Note: Compare this to the graph of
y = (x + 5)2 drawn by transforming the
graph of y = x2.) (–5, 0)
x

c 1 Write the equation. Recognise that the c y = −2x2 + 11x − 15


graph will be a negative parabola.
2 Find the y-intercept (when x = 0). y-intercept: when x = 0, y = −15
Point: (0, –15)
3 Factorise before finding x-intercepts. y = –(2x2 − 11x + 15)
= –(2x − 5)(x − 3)
4 Find x-intercepts (when y = 0). x-intercepts: when y = 0,
0 = −(2x − 5)(x − 3)
2x − 5 = 0 or x − 3 = 0
2x = 5 or x = 3
x = 5--- or x = 3
2
Points: ( 5--- , 0), (3, 0)
2
–b
5 Find the x-coordinate of the turning Turning point: xt = ------ , b = 11, a = −2
2a
point.
– 11
= --------------
2 ( –2 )
11
= ------ (or 2.75)
4
2
6 Substitute into the original equation to yt = −2  11 
11
------ + 11  ------ − 15
 
4 4
find the y-coordinate of the turning
– 242 121
point. = ------------ + --------- − 15
16 4
– 242 484 240
= ------------ + --------- − ---------
16 16 16
2
= ------
16
Use a calculator if you wish to avoid = 1
--- (or 0.125)
working with fractions. 8
11 1
7 Write the turning point coordinates. - , --- )
Turning point: ( -----
4 8

8 Sketch. y (11
—, 1– )
4 8

5–
2 3
x
–15
Chapter 3 Other graphs and modelling 127
Sketching a cubic using intercepts
To sketch a cubic function we need to first recognise if it is a positive cubic or a nega-
tive cubic. Shown below is the transformed basic graph of a positive cubic (a > 0). It
has a point of inflection.

Point of
inflection

However, there are other forms of cubic graphs where we see turning points. These
‘humps’ are not symmetrical as they are for parabolas but are skewed to one side.

y y

Turning
points
x x

A positive cubic A negative cubic

For positive cubic graphs, as positive values of x become larger, y-values also
become larger.
For negative cubic graphs, as positive values of x become larger, y-values become
smaller.
To position the cubic curve on the axes, we need to know the values of the
x-intercepts (there could be 1, 2 or 3 x-intercepts) and the y-intercept. We can also find
the coordinates of the turning points but this will not be covered in this chapter.
Consider the general factorised form of a cubic
function:
y = (x − a)(x − b)(x − c) y
To find the y-intercept, we substitute x = 0 into the
equation and solve for the corresponding y-value.
When x = 0, y = (0 − a)(0 − b)(0 − c)
= −abc
c b a x
The x-intercepts are found by substituting y = 0
into the equation and solving for the corresponding –abc
x-value.
When y = 0, 0 = (x − a)(x − b)(x − c)
x = a or x = b or x = c
128 M a t h s Q u e s t M a t h s B Ye a r 1 1 f o r Q u e e n s l a n d

WORKED Example 6
Sketch the following, showing all intercepts:
a y = (x − 2)(x − 3)(x + 5) b y = (x − 6)2 (4 − x)
THINK WRITE/DRAW
a 1 Note that the function is a y = (x − 2)(x − 3)(x + 5)
already factorised and that
the graph is a positive cubic.
2 The y-intercept occurs where x = 0. y-intercept: when x = 0,
Substitute x = 0 into the equation. y = (−2)(−3)(5)
y = 30
Point: (0, 30)
3 Find the x-intercepts. x-intercepts: when y = 0,
0 = (x – 2)(x – 3)(x + 5)
x − 2 = 0, x − 3 = 0 or x + 5 = 0
x = 2, x = 3 or x = −5
Points: (2, 0), (3, 0), (−5, 0)
4 Combine information from the y
above steps to sketch the graph.

30

–5 2 3 x

b 1 The graph is a negative cubic b y = (x − 6)2(4 − x)


(the −x in the last factor produces
a negative x3 coefficient if the
RHS is expanded).
2 Substitute x = 0 to find the y-intercept: when x = 0,
y-intercept. y = (−6)2(4)
y = 144
Point: (0, 144)
3 Find the x-intercepts. x-intercepts: when y = 0,
0 = (x – 6)2(4 – x)
x − 6 = 0 or 4 − x = 0
x = 6 or x=4
Points: (6, 0), (4, 0)
4 Combine all information and sketch y
the graph. Note the skimming of the
x-axis indicative of a repeated factor, 144
in this case the (x − 6)2 part of the
expression.

4 6 x
Chapter 3 Other graphs and modelling 129
Sketching a quartic using intercepts
The graphs shown below are the main types of positive quartic graphs.
y y y
y = x4

(2, 16) x

x
0 2 x
Negative quartics are reflected across the x-axis.
Consider the general factorised quartic, y = (x – a)(x – b)(x – c)(x – d).
The y-intercept occurs when x = 0, therefore the y-intercept is
y = (0 – a)(0 – b)(0 – c)(0 – d) y
= abcd abcd
The x-intercepts occur when y = 0, therefore
0 = (x – a)(x – b)(x – c)(x – d)
So x = a, x = b, x = c or x = d.
A quartic graph can have 0, 1, 2, 3 or 4 x-intercepts.
a b c d x

WORKED Example 7
Sketch the following graphs, showing all intercepts.
a y = (x – 2) (x – 1) (x + 1) (x + 3)
b y = (x – 3)2 (x + 1) (x + 5)
c y = (1 – 2x)(x + 1)3
THINK WRITE/DRAW
a 1 Find the y-intercept. a y-intercept: when x = 0,
y = (–2)(–1)(1)(3)
y=6
Point: (0, 6)
2 Find the x-intercepts. x-intercepts: when y = 0,
0 = (x – 2)(x – 1)(x + 1)(x + 3)
x – 2 = 0, x – 1 = 0, x + 1 = 0 or x + 3 = 0
x = 2, x = 1, x = –1 or x = –3
Points: (2, 0), (1, 0), (–1, 0), (–3, 0)
3 The graph has a positive x4 coefficient,
so large positive values for x result in
large positive values for y.
4 Combine information from the above y
steps to sketch the graph.

x
–3 –1 0 1 2

Continued over page


130 M a t h s Q u e s t M a t h s B Ye a r 1 1 f o r Q u e e n s l a n d

THINK WRITE/DRAW

b 1 Find the y-intercept. b y-intercept: when x = 0,


y = (–3)2(1)(5)
y = 45
Point: (0, 45)
2 Find the x-intercepts. x-intercepts: when y = 0,
0 = (x – 3)2(x + 1)(x + 5)
x – 3 = 0, x + 1 = 0 or x + 5 = 0
x = 3, x = –1 or x = –5
Points: (3, 0), (–1, 0), (–5, 0)
3 The graph has a positive x4 coefficient, so
large positive values for x result in large
positive values for y.
4 Combine all the information above to y
sketch the graph. Note that it touches the
45
x-axis where there are repeated squared
factors (x – 3)2. –5 –1 0 3 x

c 1 Find the y-intercept. c y-intercept: when x = 0,


y = (1)(1)3
y=1
Point: (0, 1)
2 Find the x-intercepts. x-intercepts: when y = 0,
0 = (1 – 2x)(x + 1)3
1 – 2x = 0 or x + 1 = 0
1
x= --- or x = –1
2
Points: ( 1--- , 0) and (–1, 0)
2

3 The graph has a negative x4 coefficient,


so large positive values for x result in
large negative values for y.
4 Combine all the information from above y
to sketch the graph. Note that the graph
has a point of inflection where it crosses
the x-axis with repeated cubic factors 1
(x + 1)3.
–1 0 1_
2
x
Chapter 3 Other graphs and modelling 131
remember
To sketch graphs using intercepts:
1. Identify the shape of the graph and whether it is a positive or negative curve.
2. Find the y-intercept by substituting x = 0 into the equation and solving for the
corresponding y-value.
3. Find any x-intercepts by substituting y = 0 into the equation and solving for the
corresponding x-values.
4. Indicate the general position of any turning points. For a parabola with general
equation y = ax2 + bx + c, the x-coordinate of the turning point is midway
b
between the x-intercepts or – ------ .
2a

Sketching graphs
3C using intercepts
In this exercise, check your answers using a graphics calculator.
eBook plus 1 State the x-intercepts and the y-intercept for each of the following.
a y = (x − 1)(x − 6) b y = (x − 3)(x + 2) c y = (x + 5)(x + 1)
Digital doc: d y = (x − 4)(x − 5) e y = (x + 7)2 f y = x(x − 2)
EXCEL Spreadsheet
Quadratic graphs —
g y = (3 − x)(4 − x) h y = (x + 5)(5 − x) i y = −x(x + 8)
2
factored form j y = −(x + 9) k y = 2(x + 6)(x − 6) l y = −3(2 − x)(x + 10)
WORKED 2 Sketch graphs of the following, showing all intercepts and the turning point in each
Example
5
case.
a y = x2 − 4x + 3 b y = x2 + 2x + 1 c y = x2 + 6x + 8
2
d y = x + 12x + 35 e y = x2 − 8x + 12 f y = x2 + 2x − 63
eBook plus 2
g y = x + 3x + 2 h y = x2 − 5x + 6 i y = x2 −11x − 12
Digital doc:
EXCEL Spreadsheet
3 Sketch the following graphs, showing all intercepts and the turning point in each case.
Quadratic graphs —
general form
a y = 3x2 + 2x − 8 b y = 5x2 + 18x − 8 c y = 3x2 − 4x − 15
2 2
d y = 4x − 8x + 3 e y = 8x − 10x + 3 f y = 7x2 + 18x − 9

4 Sketch the following graphs, showing all intercepts and the turning point in each case.
a y = −x2 − 8x + 33 b y = −x2 + 2x + 3 c y = −x2 − 18x − 45
2 2
d y = −x + 18x − 81 e y = −4x + 12x − 5 f y = −8x2 − 6x + 5

5 Sketch each of the following graphs, showing all intercepts and the turning point in
each case. (Hint: Factorise using the difference of perfect squares or by taking out a
common factor.)
a y = x2 − 25 b y = −x2 + 1 c y = 3 − x2
2 2
d y = 2x − 18 e y = −3x + 12 f y = x2 + 5x
2 2
g y = 5x − 10x h y = −4x − 24x i y = 21x − 3x2
WORKED
Example
6 Sketch the following, showing all intercepts.
6 a y = (x − 1)(x − 2)(x − 3) b y = (x + 6)(x + 1)(x − 7)
c y = (x + 8)(x − 11)(x + 1) d y = (2x − 5)(x + 4)(x − 3)
e y = (4x − 3)(2x + 1)(x − 4) f y = (x − 3)2(x − 6)
132 M a t h s Q u e s t M a t h s B Ye a r 1 1 f o r Q u e e n s l a n d

7 Sketch the following mixture of positive and negative cubics, showing all intercepts.
eBook plus a y = (2 − x)(x + 5)(x + 3) b y = (x + 8)(x − 8)(2x + 3)
Digital doc:
c y = x(x + 1)(x − 2) d y = 3(x + 1)(x + 10)(x + 5)
EXCEL Spreadsheet e y = 4x2(x + 8) f y = (6x − 1)2(x + 7)
Cubic graphs —
factored form
8 multiple choice
Which of the following is a reasonable sketch of y = (x + 2)(x − 3)(2x + 1)?
A y C y E y
3
1– 2 3 x
B 2
D
y x y
– 1–2
–3 1– 2 x
2 –2 –2 – 1–2 3 x

–2 – 1–2 3 x
y
9 multiple choice
The graph shown could be that of:
A y = x2(x + 2) B y = (x + 2)3 –2 2 x
2
C y = (x − 2)(x + 2) D y = (x − 2)2(x + 2)
3
E y = (x − 2)

10 multiple choice –8
y
The graph at right has the equation:
A y = (x + 1)(x + 2)(x + 3) B y = (x + 1)(x − 2)(x + 3)
C y = (x − 1)(x + 2)(x + 3) D y = (x + 1)(x + 2)(x − 3)
E y = (x − 1)(x − 2)(x − 3) –3 –1 2 x

11 Sketch the following graphs, showing all intercepts.


WORKED
–6
Example
7
a y = (x – 3)(x – 2)(x + 1)(x + 2) b y = (x – 2)2(x + 1)(x + 2)
3
c y = (x + 5)(x – 1) d y = (x – 1)4
eBook plus
e y = –x(2x – 1)(x – 3)(x + 3) f y = (x – 2)2(x + 1)2
4
g y = (1 – 3x) h y = (x + 5)3(1 – x)
Digital doc:
EXCEL Spreadsheet 12 multiple choice
Quartic graphs —
factored form Which of the following has two distinct x-intercepts when graphed?
A y = x (x + 7)(x – 7)(x + 2) B y = x2(x + 3)(x – 3) C y = x3(x + 27)
D y = x (x + 7)2(x – 7) E y = x4

13 multiple choice
If a, b and c are positive numbers, the equation of y
the graph shown is:
A y = (x + a)2(x + b)(x + c)
B y = (x – a)2(x – b)(x – c) –b a c x
C y = (x + a)2(x – b)(x + c) –abc
D y = (x – a)(x + b)(x – c)
E y = (x – a)2(x + b)(x – c)
Chapter 3 Other graphs and modelling 133
14 a If your graphics calculator has a factor facility, use it to show that the expression
eBook plus 3x3 + 17x2 + 28x + 12 can be written as (3x + 2)(x + 2)(x + 3).
b Hence, sketch the graph of y = 3x3 + 17x2 + 28x + 12.
Digital docs:
EXCEL Spreadsheet
Cubic graphs —
15 a If your graphics calculator has a factor facility, use it to show that the expression
general form −x3 − 8x2 – 5x + 14 can be written as (1 − x)(x + 2)(x + 7).
EXCEL Spreadsheet b Hence, sketch the graph of y = −x3 − 8x2 – 5x + 14.
Function grapher
16 a If your graphics calculator has a factor facility, use it to show that the expression
x4 + 3x3 − 69x2 – 127x + 840 can be written as (x − 7)(x − 3)(x + 5)(x + 8).
b Hence, sketch the graph of y = x4 + 3x3 − 69x2 – 127x + 840.
17 a If your graphics calculator has a factor facility, use it to show that the expression
−x4 − 9x3 − 21x2 + x + 30 can be written as −(x − 1)(x + 2)(x + 3)(x + 5).
b Hence, sketch the graph of y = −x4 − 9x3 − 21x2 + x + 30.

The hyperbola y

The graph shown at right is called a hyperbola and is


1
given by y = --- . This is one of the reciprocal functions. 0 x
x
The graph exhibits asymptotic behaviour. That is, as x
becomes very large (approaches ∞), the graph approaches the
x-axis but never touches it, and as x becomes very small (approaches 0),
the graph approaches the y-axis but never touches it. So the line x = 0 (the y-axis) is a
vertical asymptote and the line y = 0 (the x-axis) is the horizontal asymptote. Both the
domain and the range of the function are all real numbers, except 0; that is, R\{0}.
1
The graph of y = --- can be subject to a number of transformations.
x
y
a
Consider y = ----------- + k.
x–h a=2
The value a is a dilation factor. It dilates the graph in a=1
the y direction. If a is negative, the graph is reflected in a = 1–2
0 x
the x-axis.
a = –1
The value h translates the graph h units horizontally
y = a–x
(to the right if h > 0, or to the left if h < 0). Hence,
the equation of the vertical asymptote is x = h and the domain is R\{h}.
The value k translates the graph k units along the y-axis (up if k > 0, or down if
k < 0). Hence, the horizontal asymptote is y = k and the range of the function is R\{k}.
a y
The graph of y = ----------- + k shows the combination of these a
eBook plus x–h y = x—
–h + k
transformations.
Digital doc: Finally, if the coefficient of x is a number other than 1, k
EXCEL Spreadsheet
The hyperbola
the equation must be rearranged before the value of h can be x
0 h
obtained. For example:
4 4
y = --------------- = --------------------
3x + 6 3 ( x + 2 )
Therefore, h = −2 (not −6 as it may seem at first); that is, the graph is translated 2 units
to the left.
134 M a t h s Q u e s t M a t h s B Ye a r 1 1 f o r Q u e e n s l a n d

WORKED Example 8
1
State the changes that should be made to the graph of y = --- to obtain the graph of
–4 x
y = ------------ − 1.
x+2
THINK WRITE
a
1 Write the general equation of the y = ----------- + k
hyperbola. x–h
2 Identify the value of a. a = −4
1 1
3 State the changes to y = --- , caused by a. The graph of y = --- is dilated by the factor of 4
x x
in the y direction and reflected in the x-axis.
4 Identify the value of h. h = −2
5 State the effect of h on the graph. The graph is translated 2 units to the left.
6 Identify the value of k. k = −1
7 State the changes to the graph caused The graph is translated 1 unit down.
by k.

Sketching the graph of the hyperbola by hand can be easily done by following these steps:
Step 1 Find the position of the asymptotes.
Step 2 Find the values of the intercepts with the axes.
Step 3 Decide whether the hyperbola is positive or negative.
Step 4 On the set of axes draw the asymptotes (using dotted lines) and mark the
intercepts with the axes.
Step 5 Treating the asymptotes as the new set of axes, sketch either the positive or
negative hyperbola, making sure it passes through the intercepts that have been
previously marked.

WORKED Example 9
2
Sketch the graph of y = ------------ − 4, clearly showing the intercepts with the axes and the
x+2
position of the asymptotes.
THINK WRITE

1 Compare the given equation with a = 2, h = −2, k = −4


a
y = ----------- + k and state the values of a,
x–h
h and k.
1
2 Write a short statement about the The graph of y = --- is dilated by the factor of 2
effects of a, h and k on the graph of x
in the y direction and translated 2 units to the
1 left and 4 units down.
y = --- .
x
3 Write the equations of the asymptotes. Asymptotes: x = −2; y = −4
Chapter 3 Other graphs and modelling 135

THINK WRITE

4 Find the value of the y-intercept by letting y-intercept: when x = 0,


x = 0. 2
y = ------------ − 4
0+2
=1−4
= −3
5 Find the value of the x-intercept by making x-intercept: when y = 0,
y = 0. 2
0 = ------------ − 4
x+2
2
------------ = 4
x+2
2 = 4(x + 2)
= 4x + 8
4x = 2 − 8
= −6
x = − 6---
4

= − 3---
2

6 To sketch the graph: y


(a) Draw the set of axes and label them.
– 3–2
(b) Use dotted lines to draw the asymptotes. x
–2 0
(c) Mark the intercepts with the axes.
(d) Treating the asymptotes as your new set –3
of axes, sketch the graph of the hyperbola –4
(as a is positive, the graph is not
reflected); make sure the upper branch
passes through the x- and y-intercepts
previously marked.

remember
1
1. The graph of y = --- is called a hyperbola. y
x a
y = x—
–h + k
a
2. The graph of y = ----------- + k is the graph of the basic
x–h
hyperbola, dilated by the factor of a in the y direction, k
translated h units horizontally (to the right if h > 0, or 0 h x
to the left if h < 0) and k units vertically (up if k > 0,
or down if k < 0). If a < 0, the graph is reflected in the
x-axis. The equations of the asymptotes are: x = h and
y = k. The domain of the function is R\{h} and its range is R\{k}.
136 M a t h s Q u e s t M a t h s B Ye a r 1 1 f o r Q u e e n s l a n d

3D The hyperbola
1
WORKED 1 State the changes that should be made to the graph of y = --- to obtain the graph of
Example
each of the following. x
8
2 3 1
a y = --- b y = − --- c y = -----------
x x x–6
2 1 2
d y = ------------ e y = --- + 7 f y = --- − 5
x+4 x x
1 2 4
g y = ------------ − 3 h y = ----------- + 6 i y = − ----------- − 4
4+x x–3 x–1
1
2 Which of the following transformations were applied to the graph of y = --- to obtain
each of the graphs shown below? x
iii translation to the right iii translation to the left
iii translation up iv translation down
iv reflection in the x-axis
a y b y

0 x
0 x

c y d y

0 x
0 x

e y f y

0 x 0 x

g y h y

0 x
0 x
Chapter 3 Other graphs and modelling 137
3 For each of the following graphs, state:
i the equations of the asymptotes ii the domain iii the range.
a y b y c y

2
0 x 2
4 x
0 x 0 3

d y e y f y

–1
0 x n
–1 a
0 m x 0 x
b

WORKED 4 Sketch each of the following, clearly showing the position of the asymptotes and the
Example
9
intercepts with the axes. Check your answers using a graphics calculator.
1 1 3 3
a y = ------------ b y = ------------ − 1 c y = ----------- − ---
x+3 x+2 x–1 4
eBook plus
2 6 3
d y = − ------------ e y = ----------- − 3 f y = − ----------- + 6
Digital docs: x+5 1–x x–2
EXCEL Spreadsheet
1 2 4 1
The hyperbola
g y = 1 − ----------- h y = --- + ------------ i y = --------------- + 4
EXCEL Spreadsheet 2–x 5 1+x 2x + 3
Function grapher

5 multiple choice
The equation of the graph shown is likely to be:
y

3
0 4 x

1 1
A y = 3 + ----------- B y = ----------- + 4
x–4 x–3
1 1
C y = 3 − ----------- D y = ----------- − 3
4–x 4–x
1
E y = 3 − -----------
x–4
1
eBook plus 6 If a function is given by f(x) = --- , sketch each of the following, labelling the
x
asymptotes and the intercepts with the axes.
Digital doc:
WorkSHEET 3.1 a f(x + 2) b f(x) − 1 c −f(x) − 2
d f(1 − x) + 2 e −f(x − 1) − 1
138 M a t h s Q u e s t M a t h s B Ye a r 1 1 f o r Q u e e n s l a n d

The square root function 1


---
2
The square root function is given by y = x (or y = x ).
The function is defined for x ≥ 0; that is, the domain is y
[0, ∞). As can be seen from the graph, the range of the square
root function is also [0, ∞). Throughout this section we will
refer to the graph of y = x as ‘the basic square root curve’. 0 x
Let us now investigate the effects of various transformations
on the basic square root curve.
y a=3
Consider the function y = a x – h + k. a=2
The value a is a dilation factor; it dilates the graph
a=1
parallel to the y-axis. If a < 0, the graph is reflected in
a = 1–2
the x-axis. x
0
a = –1
y=a x

y
h = –2
The value h translates the graph along the x-axis by
h units (to the right if h > 0 and to the left if h < 0). h=3
The domain of the square root function is given by x ≥ h. –2 0 x
3
y= x–h

y
k=2
The value k translates the graph k units along the y-axis 2
(up if k > 0 or down if k < 0).
0 x
The range of the square root function is given by y ≥ k for k = –4
a > 0, or y ≤ k for a < 0.
–4 y= x+k

The graph of y = a x – h + k shows the combination of y


y=a x–h+k
eBook plus these transformations.
Digital doc:
The point (h, k) is the end point of the square root curve.
EXCEL Spreadsheet (h, k)
Square root function x
0

Consider the function y = a h – x + k.


The effects of a, h and k on the graph of y = x discussed above still apply. The end
point of the curve and the range of the function do not change either. The domain, how-
ever, will differ. Since we require the expression under the root sign to be larger than or
equal to 0, we must have h − x ≥ 0, so h ≥ x.
y

The graph of the function y = a h – x + k y=a h–x+k y=a x–h+k


can be thought of as the graph of h
y = a x – h + k, reflected in the line x = h. x
0
(h, k)
Chapter 3 Other graphs and modelling 139
WORKED Example 10
State the transformations required to change y = x to y = −3 x + 5 + 3.
THINK WRITE
1 Write the general formula for the square y=a x–h +k
root curve.
2 Identify the value of a. a = −3
3 State the effect of a on the graph. The graph is dilated by a factor of 3 in the
y direction and reflected in the x-axis.
4 Identify the value of h. h = −5
5 State the effect of h on the graph. The graph is translated 5 units to the left.
6 Identify the value of k. k=3
7 State the effect of k on the graph. The graph is translated 3 units up.

To sketch the graph of the square root function, a > 0 a>0


we need to compare the given formula with
y = a x – h + k. This will give us an idea of the y=a h–x+k y=a x–h+k
changes required to transform the basic square a<0 (h, k) a<0
root curve into the one we want. It will also let us
know the way the curve will look. The diagram above illustrates the idea.
Once the coordinates of the end point and the direction of the curve are known the
intercepts with the axes (if any) should be found before sketching.

WORKED Example 11
Sketch the graph of y = −2 x – 3 + 1.

THINK WRITE
1 Write the general formula for the y=a x–h +k
square root function.
2 Identify the values of the variables. a = −2, h = 3, k = 1
3 Write a short statement of the changes The graph of y = x is dilated by the factor of
necessary to transform the basic curve 2 in the y direction, reflected in the x-axis and
into the given one. translated 3 units to the right and 1 unit up.
4 State the coordinates of the end point End point: (3, 1)
(h, k).
5 State the shape of the graph Shape:
(for y = a x – h + k, a > 0).
6 Find the x-intercept by letting y = 0. x-intercept: when y = 0 ,
0 = −2 x – 3 + 1
2 x–3 =1
Continued over page

1
140 M a t h s Q u e s t M a t h s B Ye a r 1 1 f o r Q u e e n s l a n d

THINK WRITE
1
x–3 = ---
2

x − 3 = ( --1- )2
2
1
= ---
4
1
x= --- +3
4

x = 3 1---
4
7 The y-intercept occurs when x = 0. There is no y-intercept.
However, the domain of the function is
x ≥ h, or in this case, x ≥ 3, so x ≠ 0;
hence, there is no y-intercept.
8 To sketch the graph: draw a set of axes y
and label them; plot the end point; mark (3, 1)
1
the x-intercept; draw the curve so that it 3 1–4
starts at the end point, goes in the direc- 0 3 x
tion you’ve established in step 5 and y = –2 x –3 +1
passes through the marked x-intercept.

remember
1. The graph of the function y = a x – h + k is the y
y=a x–h+k
graph of y = x , dilated by the factor of a in the
y direction and translated h units along the x-axis
and k units in the y-axis. (h, k)
2. If a < 0, the graph is reflected in the x-axis. x
0
3. The end point of the graph is (h, k).
4. The domain is x ≥ h.
5. The range is y ≥ k for a > 0, or y ≤ k for a < 0.
6. If y = a h – x + k, the domain is x ≤ h; the graph is reflected in the y-axis.

3E The square root function

WORKED 1 State the transformations required to change y = x to each of the following.


Example
10 a y=2 x b y= − 1--- x c y=3 x–1
3
d y = −2 4 + x e y= x −1 f y=2−3 x
g y= x–4 +3 h y=6−2 3+x i y = − 1--- 2 – x + 2
---
2 3

2 For each of the functions in question 1 write the coordinates of the end point.
Chapter 3 Other graphs and modelling 141
3 multiple choice
The graph shown below was obtained by translating the graph of y = − x :
A 3 units up and 9 units to the right y
B 3 units down and 9 units to the right
C 9 units up and 3 units to the right 3
D 9 units down and 3 units to the right
0 9 x
E none of the above

4 multiple choice
To obtain the graph in the diagram, the graph of y = x was: y
A translated 2 units to the right and 4 units up
B translated 4 units to the right and 2 units up (2, 4)
C translated 2 units to the right, 4 units up and reflected in the x-axis
D reflected in the y-axis, translated 4 units up and 2 units to the right 0 x
E reflected in the x-axis, translated 4 units up and 2 units to the left

5 Find the domain and range for each of the following functions.
a y= x+1 b y= x −3 c y=5− x
d y= 2+x −1 e y = − 3--5- 3x – 4 + 2 f y=6+ 4 – 2x

WORKED 6 Sketch the graph of each of the following. Check your answers with a graphics
Example
11
calculator.
a y= x+2 b y = 1--- x + 3 c y=2− x
3
eBook plus d y= x–6 +1 e y= 3+x +2 f y = 1--- − 4 + x
2
g y = 2x – 3 h y = 6 + 3x + 2 i y= 2–x −1
Digital docs:
EXCEL Spreadsheet
Square root function 7 multiple choice y
EXCEL Spreadsheet The equation of the graph shown at right is: (1, 2)
Function grapher
A y=2− x–1 B y=2−2 1–x
0 x
2
C y= x–1 −2 D y= x–2 +1
E y=2−2 x–1

8 The end point of the square root curve is at (4, 3) and its y-intercept is 9. Sketch the
graph of the curve and hence establish its equation.

9 The graph of y = x was dilated by the factor of 4 in the y direction, reflected in the
x-axis, translated 1 unit to the left and p units up.
a Find:
i the value of p, if the graph cuts the y-axis at y = 4
ii the equation of the curve
iii the x-intercept
iv the domain
v the range.
b Hence, sketch the graph, showing the coordinates of the end points and the
intercepts with the axes.
142 M a t h s Q u e s t M a t h s B Ye a r 1 1 f o r Q u e e n s l a n d

The absolute value function


The function f(x) = |x| is called an absolute value function. The domain of the function
is R and its range is [0, ∞).
The graph is symmetrical in the y-axis and has a cusp (a sharp point) at the origin.
y

0 x
y = |x|

The symbol |x| represents the magnitude of x, (that is, the size of x), regardless of its
sign.
 x, if x ≥ 0
Therefore, |x| = 
 −x, if x < 0
Compare the graphs of y = x and y = |x|.
y y

0 x
0 x
y=x y = |x|

For x ≥ 0 the graphs of the 2 functions are identical, while for x < 0 the graph of
y = |x| is the reflection of y = x in the x-axis.
In general, to sketch the graph of y = | f(x)|, we need to sketch the graph of y = f(x)
first and then reflect in the x-axis the portion of the graph which is below the x-axis.
Similar to the graphs discussed in the previous sections, the graph of the absolute
value function can be transformed through dilations, translations and reflections.
y a=2

a=1

0 a = –1 x
y = a |x|

If y = a| f(x)| + k, a is the dilation factor. It dilates the graph in the y direction. The
larger a is, the thinner the graph.
eBook plus
If a < 0, the graph is reflected in the x-axis. The value k translates the graph along the
y-axis. If k > 0, the graph is moved k units up and if k < 0, it is shifted down.
Digital doc:
y k=1
EXCEL Spreadsheet
Absolute value function

k = –2
1
0 x
–2
y = |x| + k
Chapter 3 Other graphs and modelling 143
WORKED Example 12
Sketch the graph of y = |x2 − 1|.

THINK WRITE

1 We first need to sketch the graph of Let y = x2 − 1


y = x2 − 1. State the shape of this graph. Shape: positive parabola, translated 1 unit down
2 State the coordinates of the turning point Turning point: (0, −1)
(note that it is also the y-intercept).
3 Find the x-intercepts. x-intercepts: when y = 0,
x2 − 1 = 0
x2 = 1
x= ± 1
x = +1 or −1
4 Sketch the graph of the parabola. y

x
–1 0 1
–1
y = x2 – 1
5 Reflect the portion of the parabola for y
−1 < x < 1 in the x-axis; mark the new
y-intercept. 1

x
–1 0 1
y = |x2 – 1|

WORKED Example 13
Sketch the graph of y = |x − 2| + 1.

THINK WRITE

1 Compare the given function with The graph of y = |x − 2| is shifted 1 unit up.
y = a| f(x)| + k, and write a short comment.
2 To sketch the required shape, we first Let y = x − 2.
need to sketch y = x − 2 (a straight line). y-intercept: when x = 0,
Find the y-intercept. y=0−2
y = −2
3 Find the x-intercept. x-intercept: when y = 0,
x−2=0
2−x=2

Continued over page


144 M a t h s Q u e s t M a t h s B Ye a r 1 1 f o r Q u e e n s l a n d

THINK WRITE
4 Sketch the line. y

0 x
2
–2
y = x–2

5 Reflect the portion below the x-axis in y


the x-axis; mark the new y-intercept.
2

0 x
2
y = |x–2|
6 Move the graph 1 unit up; mark the y
new y-intercept and the coordinates of
the cusp. 3 y = |x–2| + 1
(2, 1)
0 x
2

Graphics Calculator tip! Graphing the absolute


value function
To graph the function y = x – 2 + 1, follow the instructions below.
For the Casio fx-9860G AU
1. Press MENU and then select GRAPH. To enter the
absolute value function for Y1, press OPTN and
then F5 (NUM) followed by F1 (Abs).
<Casio03.125>

2. Enter (x − 2) + 1 and then press EXE .

<Casio03.125>

3. Press F6 (DRAW) to display the graph. If


necessary, press SHIFT F3 (V-WIN) to access
and adjust the View Window. (In this example,
Xmin is −3, Xmax is 6, Ymin is −5 and Ymax is 5.) <Casio03.125>

4. To explore points of interest on the graph, press


SHIFT F1 (TRCE).

<Casio03.125>
Chapter 3 Other graphs and modelling 145
For the TI-Nspire CAS
1. Open a new Graphs & Geometry document
(press /N and then select 2: Add Graphs &
Geometry). To obtain the absolute value function, <TI04.01>
press k to access the catalog and then press 1
to select the function menu. Use the NavPad to
highlight abs(.
2. Press · to display the absolute value function in
the function entry line. (Alternatively, you can
press ABS( to enter abs( directly into the
function entry line.) <TI04.01>

3. Complete the function entry line as shown in the


screen at right.

<TI04.01>

4. Press · to display the graph.

<TI04.01>

5. If you wish to adjust the view of the graph,


press b and then select 4: Window followed by
1: Window Settings. (In this example, XMin is set
as −3, XMax as 6, YMin as −5 and YMax as 5.) To <TI04.01>
explore points of interest on the graph, press b
and then select 5: Trace followed by 1: Graph
Trace.

remember y
1. The symbol |x| denotes the magnitude of x.
2. |x| = x, if x ≥ 0
|x| = −x, if x < 0
3. To sketch the graph of y = | f(x)|:
(a) sketch the graph of y = f(x) 0 x
(b) reflect the portion of the graph which is y = |x|
below the x-axis in the x-axis.
146 M a t h s Q u e s t M a t h s B Ye a r 1 1 f o r Q u e e n s l a n d

3F The absolute value function

WORKED 1 Sketch the graph of each of the following. Use a graphics calculator to check your
Example
12
answers.
a y = |2x| b y = |x − 1| c y = |3 − 6x|
2 2
eBook plus d y = |x − 6| e y = |4 − x | f y = |(x − 3)2 − 4|

Digital doc: g y = |3x3| h y = |(x + 2)3 − 1| i 2


y = -----------
EXCEL Spreadsheet x–1
Absolute value
function

2 multiple choice y

Which of the following functions best describes this graph?


A y = |(x − 1)3| B y = |(x + 1)3| 1

C y = |x3 + 1| D y = |x3 − 1|
x
–1 0
E y = |(x + 1)3 + 1|

3 State the domain and range for each of the following functions.
a y = 2|x| b y = |x| + 1 c y = 4 − 3|x|

d y = |x2 − 3| − 2 1
e y = -----------
1 f
- +1
y = 2 – -----
2
x+1 x
WORKED 4 Sketch the graphs of each of the following. Check your answers with a graphics
Example
13
calculator.
a y = −2|x| b y = |x + 5| − 6 c y = 2|3 − x| + 1
2 2
d y = |x − 1| + 1 e y = 2 − |x − 2| f y = |(x + 1)2 − 1| − 2
3 1 1
g y=1
--- − ---
2 +3
h y = ----------- i y = --- ----2- − 4
x 4 6–x 4 x

1
j y = − ----- – 1 k y = | 2 – x − 2| + 3 l y = 2 − | x + 1 − 8|
2
x

5 The design shown at right is to be embroidered on the outer y


side of children’s socks. 6
The total length of the design is 12 cm and its width is 8 cm.
If we draw the set of axes through the centre of the design,
the red section can be thought of as the absolute value
function on a restricted domain. 4 4x
a Find the rule for the red section and specify the domain.
b Using your knowledge of the transformations, and the rule
eBook plus for the red section, find the rules for the blue, green and
–6
yellow sections of the design.
Digital doc:
c Using a graphics calculator, sketch the 4 functions that were obtained in a and b.
WorkSHEET 3.2
Have you obtained the right design?
Chapter 3 Other graphs and modelling 147
Addition of ordinates
Sometimes we need to sketch the graph of a function that can be thought of as the sum
of two functions. For example, the function y = x2 + x3 can be thought of as the sum of
the functions y = x2 and y = x3.
Such a graph can be drawn by sketching the 2 individual functions on the same set of
axes and then adding the y-values (ordinates) for each x-value.
This method is convenient to use when we cannot recognise the basic shape of the
given function, but can state the basic shape of the individual functions.
For example, the graph of y = x2 + x can be sketched using the addition-of-
ordinates technique. Although the basic shape of the function is not known, the two
individual functions are a basic positive parabola and a basic square root curve respec-
tively. The graph of y = x2 + 1 can also be thought of as the sum of two functions, but
since we can recognise its shape as the basic parabola, translated 1 unit up, there is no
need to use the addition-of-ordinates method.
Note that the ‘sum’ function can only be defined for the domain over which both of
the individual functions are defined. So the domain of the ‘sum’ function is obtained by
finding the intersection of the domains of the individual functions.
That is, if h(x) = f(x) + g(x), domain h(x) = domain f(x) ∩ domain g(x).
Sometimes a function is defined as the difference between two individual functions.
We can still use the method of addition of ordinates to graph such a function, because
the second function can be expressed as the addition of a negative function. That is,
h(x) = f(x) − g(x) can be written as h(x) = f(x) + (−g(x)).
So if, for example, we need to graph y = x2 − x3, we could think of it as y = x2 + (−x3).
We then would sketch the basic parabola and basic negative cubic curves on the same
set of axes and use the addition-of-ordinates technique to obtain the graph of the original
function.

WORKED Example 14
State the domain of f(x) = 3x2 + x.

THINK WRITE
1 f(x) can be considered as the sum of the two Let f(x) = g(x) + p(x), where g(x) = 3x2 and
individual functions. Write them down. p(x) = x .
2 State the domain of g(x) (a quadratic Domain g(x) = R
function).
3 State the domain of p(x) (a square root Domain p(x) = [0, ∞)
function).
4 State the domain of f(x) by finding the Domain f(x) = domain g(x) ∩ domain p(x)
intersection of the domains of individual Domain f(x) = R ∩ [0, ∞)
functions. Domain f(x) = [0, ∞)

When sketching the graph by hand, it is important to wisely select the x-values for
which the ordinates should be added. As a guide, the good points to select are:
1. the end points of the graph
2. the points of intersection of the 2 graphs
3. the x-intercepts of either of the 2 graphs.
148 M a t h s Q u e s t M a t h s B Ye a r 1 1 f o r Q u e e n s l a n d

WORKED Example 15 y
Given the graphs of f(x) and g(x), sketch the graph of f(x)
h(x) = f(x) + g(x).
g(x)

0 x

THINK WRITE/DRAW
1 Add the ordinates at the LHS end ➀ y ➁ y
points of the graph: a large positive
value plus a smaller negative value f(x) f(x)
should give a smaller positive value.
Mark this point on the axes. g(x) g(x)
2 Add the ordinates at the point where
0 x 0 x
g(x) has the x-intercept and mark the
resulting point on the set of axes. Note
that g(x) = 0 at this point; therefore
f(x) + 0 = f(x) (that is, if one of the
functions cuts the x-axis, the sum is
equal to the y-value of the other
function).
3 Add the ordinates at the first point of ➂ ➃
y y
intersection of the 2 functions. Note
that at the point of intersection the f(x) f(x)
value of y is the same for both
functions, so the resulting point is g(x) g(x)
double the y-value. Mark the point on
the set of axes. 0 x 0 x
4 Add the ordinates at the point where
f(x) has the x-intercept and mark the
result on the set of axes.
5 Add the ordinates at the second point of ➄ y ➅ y
intersection of the 2 graphs and mark
the resulting point on the set of axes. f(x) f(x)
6 Add the ordinates at the RHS end
points of the graph: 2 positive values g(x) g(x)
together give an even larger positive
0 x 0 x
value.
7 Join the points with a smooth curve to y h(x)
produce the graph of h(x).
f(x)

g(x)

0 x
Chapter 3 Other graphs and modelling 149
WORKED Example 16
Sketch the graph of f(x) = x + x using addition of ordinates.

THINK WRITE/DRAW
1 State the two individual functions. Let f(x) = g(x) + p(x), where g(x) = x and
p(x) = x .
2 State the domain of f(x) by finding the Domain g(x) = R
intersection of the domains of Domain p(x) = [0, ∞)
individual functions. Domain f(x) = domain g(x) ∩ domain p(x)
Domain f(x) = R ∩ [0, ∞)
Domain f(x) = [0, ∞)
3 On the set of axes sketch the graph of y
g(x) (a straight line, passing through the ➀ g(x)
origin and bisecting the first quadrant)
and p(x) (a basic square root curve).
p(x)
Note that only the first quadrant is
needed, since the domain is [0, ∞).
0 x
4 The first point of intersection of g(x) y
y
and p(x) is at the origin, f(x) = 0 + 0 = 0 ➁ g(x) ➂
(that is, f(x) will also start at the g(x)
origin). Mark this point on the set of
p(x)
axes. p(x)
5 Add the ordinates at the second point of
intersection of the two graphs and mark 0 x
0 x
the resulting point on the set of axes.
6 Add the ordinates at the RHS end
points of the graph and mark the
resulting point on the set of axes.
7 Sketch the graph of f(x) by joining the f(x)
points. y
g(x)

p(x)

0 x

Note: When sketching, we judge the distances by eye, so the values of y obtained by
adding ordinates are approximate, rather than exact.
If you feel that some exact values would help produce a more accurate graph, they
can be easily found by substitution. For instance, for the function f(x) = x + x in the
above example, we can obtain a couple of y-values, say at x = 1 and at x = 4, as follows:
f(1) = 1 + 1 = 1 and f(4) = 4 + 4 = 6. So the graph of the function passes through
the points (1, 1) and (4, 6).
150 M a t h s Q u e s t M a t h s B Ye a r 1 1 f o r Q u e e n s l a n d

Graphics Calculator tip! Addition of ordinates


To explore addition of ordinates on a graphics calculator, let’s consider the two
functions f(x) = x2 and g(x) = 3x − 4. We can graph both of these functions on the one
set of axes and then graph the resulting function f(x) + g(x). The steps are shown below.
You may like to then investigate the addition of ordinates for the graphs of
f(x) = 0.25x3 and g(x) = (x − 1)2 to obtain the resulting function f(x) + g(x).
For the Casio fx-9860G AU
1. Press MENU and then select GRAPH. Press
SHIFT F3 (V-WIN) to access and adjust the
View Window. (In this example, Xmin is −7,
Xmax is 7, Ymin is −10 and Ymax is 15.) Enter x2 <Casio04.01>
for Y1 and then press EXE . Enter 3x − 4 for Y2
and then press EXE .

2. Press F6 (DRAW) to display the graphs of Y1


and Y2 on the same axes.
<Casio04.01>

3. Press EXIT to return to the Graph Function


screen. For Y3, we wish to enter Y1 + Y2. Press
VARS and then F4 (GRPH) to indicate we
want to enter a graphing variable. To enter Y1, <Casio04.01>
press F1 (Y) and then 1 followed by + .
To enter Y2, press F1 (Y) and then 2 . Press
EXE .
4. To make the third curve appear different from the
other two, highlight the Y3 line and then press
F4 (STYL). Select F2 for the darker line.
Press EXE to obtain the graph. <Casio04.01>

5. To explore points of interest on the graph, press


SHIFT F1 (TRCE).

<Casio04.01>

For the TI-Nspire CAS


1. Open a new Graphs & Geometry document
(press /N and then select 2: Add Graphs & <TI04.01>
Geometry). Enter x2 for f1(x) and then press ·.
Enter 3x − 4 for f2(x) and then press ·.
Chapter 3 Other graphs and modelling 151
2. For f3(x), we wish to enter f1(x) + f2(x) in the
function entry line. To enter f1(x), press h to
indicate we want to enter a variable, highlight f1 and
then press ·. Press X) and then +. Similarly, <TI04.01>
to enter f2(x), press h, highlight f2 and then press
·. Complete the entry line by pressing X).
3. Press · to display the graph of f3(x) on the same
axes.
4. To view the graphs more clearly, press b and then
select 4: Window followed by 1: Window Settings.
(In this example, XMin is set as −7, XMax as 7, YMin <TI04.01>
as −10 and YMax as 15.) To move the position of the
rules for each graph, first press d to obtain a pointer
(å) in the work area, then use the NavPad to move the
pointer so that it hovers over a rule. The pointer will
then appear as an open hand (÷) with the word ‘label’
shown. Press /x. The hand will close ({) and the
equation will flash. Use the NavPad to move the rule
to the desired location and then press d. <TI04.01>
To hide the function entry line, press /G. (Press
/G again to bring back the function entry line.)
5. To highlight the graph of f3(x), we can change the
appearance of the curve. Move the pointer towards the
curve until it starts to flash. Press b and then select
1: Actions followed by 4: Attributes. The pointer will
change to a horizontal arrow (ë). Press ·. The first <TI04.01>
option in Attributes is for the thickness of the line.
Using the right arrow on the NavPad, select (2/3) Line
weight is medium (or press the arrow again for (3/3)
Line weight is thick), then press ·.
To explore points of interest on the graph, press b
and then select 5: Trace followed by 1: Graph Trace.

remember
1. A graph of the sum of 2 functions can be drawn by sketching the 2 functions on
the same set of axes and then adding the y-values for each value of x.
2. The domain of the ‘sum’ function is obtained by finding the intersection of the
domains of individual functions: if h(x) = f(x) + g(h),
domain h(x) = domain f(x) ∩ g(x).
3. Suitable points at which to add ordinates are:
(a) LHS and RHS end points
(b) the points of intersection of the 2 graphs
(c) the x-intercepts of either of the 2 functions.
4. Exact values of y can be obtained by substituting the required x-values into the
given function.
5. The ‘difference’ function can be rewritten as the ‘sum’ function as follows:
h(x) = f(x) − g(x)
= f(x) + [−g(x)],
so that addition of ordinates can be used.
152 M a t h s Q u e s t M a t h s B Ye a r 1 1 f o r Q u e e n s l a n d

3G Addition of ordinates

WORKED 1 State the domain of each of the following functions:


Example
14 1
a y = x + --- b y=3 x −x c y = x2 + 1
--- x −1
x 2
1
d y= x+2 −x e y = x2 − 3x3 f y = 2x3 − -----------
x–3
2 2
g y = ------------------- − x2 h y = 3x − 2 1 – x i y = 3(2 − x)2 + ------
2 3x
( x + 1)
j y=2 3–x −3 x+1

2 multiple choice
If f(x) = g(x) + p(x), and if f(x) is defined over the domain [−2, 7] and g(x) is defined
for x ≤ 7, the domain of p(x) could not be:
A [−2, 7] B [−2, ∞) C [−2, 7)
D [−2, 7.5) E [−2, 15]

WORKED 3 Given the graph of f(x) and g(x), sketch the graph of h(x) = f(x) + g(x).
Example
15 a y b y c y
f(x)

x
g(x) 0 x 0
f(x)
f(x)
0 x
g(x) g(x)

d y e y f y
f(x)
g(x)
f(x)

0 x
g(x)
0 x f(x)

0 x
g(x)

4 On the same set of axes sketch the graph of f(x) = x2 and g(x) = 5x + 6. Use the
addition-of-ordinates method to sketch the graph of y = x2 + 5x + 6.

5 On the same set of axes sketch the graph of f(x) = x3 and g(x) = x2 − 1 and hence
sketch the graph of y = x3 + x2 − 1, using the addition-of-ordinates technique.

WORKED 6 Sketch the graph of each of the following functions, using addition of ordinates.
Example
16 Check your answers with a graphics calculator.
a y= x + 2–x b y = 2x − x
2
c y= x–3 −x d y= x+5 + 5–x
Chapter 3 Other graphs and modelling 153
7 multiple choice y
The graph at right is likely to represent the sum of which
two functions?
A y B y x
0

0 x
0 x

C y D y E y

0 x

0 x 0 x

Modelling
A mathematical model is an attempt to describe some aspect of the real world in terms
of one or more mathematical relationships or rules. The process of finding such a rule
is called modelling and the rule itself is known as the mathematical model.
People such as scientists, financial advisers, business analysts, economists and statis-
ticians often have to deal with large and small sets of data.
The following examples demonstrate how modelling is used.
An engineer creates a mathematical model of a bridge. Using this model, the bridge
can be tested under different loads and in different weather conditions to see whether it
falls down or not. It is much cheaper and safer to do this using a model than a real
bridge.
One-day cricket matches are often affected by rain. Two mathematicians, Frank
Duckworth and Tony Lewis, developed a mathematical model to help decide the winner
of a rain-affected match.
Climatologists are interested to know the likely impact of rising CO2 levels in the
atmosphere on temperature. They have developed mathematical models to assist in this
prediction. A number of different models may be developed. While each of these
models should account for past links between CO2 levels and temperature, they can
possibly differ in their predictions of the future.
Economists develop models in an attempt to predict a variety of phenomena
including stockmarket prices, demand for commodities and inflation rates.
A meteorologist develops a mathematical model to predict the tide times and heights
for the coming year.
In its simplest form, a mathematical model attempts to relate two variables using a
mathematical relationship or rule. For example, the cost of a phone call, C, is related to
the length of time of the call, t, by C = 0.45 + 0.015t. A model such as this one can be
developed, by logical argument, from information supplied by the phone company.
Many models are developed from numerical data.
When finding the model, the best way to start is to plot the data, as the shape of the
graph might suggest the type of relationship between the variables.
154 M a t h s Q u e s t M a t h s B Ye a r 1 1 f o r Q u e e n s l a n d

WORKED Example 17
x 0 1.5 2.55 3.33 4 5.55
y 0 2.5 3.54 4.33 5 5.59

The data in the above table exactly fit one of these rules: y = ax2, y = ax3, y = --a- or y = a x .
x
a Plot the values of y against x. b Select the appropriate rule and state the value of a.

THINK WRITE
a Plot the values of y against x. a y
5
4
3
2
1
0 x
1 2 3 4 5

b 1 Study the graph. It appears to be a b Assume that y = a x .


square root curve. Write the
appropriate rule.
2 To find the value of a, select any Using (1, 2.5):
pair of corresponding values of x
and y. (Since we need to take a
square root, the best choice is one
where x is a perfect square.)
3 Substitute selected values into the 2.5 = a 1
rule and solve for a. 2.5 = a × 1
a = 2.5
4 We need to make sure that the Verifying:
selected rule is the right one. y = 2.5 x
Replace a with 2.5 in the rule.
5 Substitute the values of x from (0, 0): y = 2.5 0
the table into the formula and =0
check if you will obtain the correct (2, 3.54): y = 2.5 2
values of y. = 3.54
(3, 4.33): y = 2.5 3
= 4.33
(4, 5): y = 2.5 4
=5
(5, 5.59): y = 2.5 5
= 5.59
6 As the values of y obtained by using The rule that fits the data is y = a x , where
the rule match those in the table, the a = 2.5.
choice of model is correct.
Chapter 3 Other graphs and modelling 155
Sometimes non-linear relationships between the 2 variables can be reduced to
m
linear relationships. For instance, y = ---- + c is a hyperbola. However, if we substi-
x
1
tute p for --- , the rule becomes linear: y = mp + c. The graph of y versus p will be
x
the straight line with the gradient of m and the y-intercept of c. These values (m
and c) can then be established from the graph and thus the hyperbolic model can
be determined.

WORKED Example 18
x 0 1.3 2.5 13.8 44 45.4
y 4 5.3 8.6 14.8 23 34.4
It is believed that, for the data in the table above, the relationship between x and y can be
modelled by y = ax2 + b.
a Plot the values of y against x.
b Plot the values of y against x2 and draw the line of best fit.
c Find the values of a and b.

THINK WRITE
a Plot the values of y against x. The graph a y
does appear to be a quadratic. 35
30
25
20
15
10
5
0 x
1 2 3 4 5

b 1 Make a new table of values: square b


all values of x and leave the values
of y unchanged. x2 0 1.3 4.6 19.8 16 25.4

y 4 5.3 8.6 14.8 23 34.4

2 Plot the values of y against x2 and y


draw the line of best fit. 35
30
25
20
15
10
5
2
0 5 10 15 20 25 x

Continued over page


156 M a t h s Q u e s t M a t h s B Ye a r 1 1 f o r Q u e e n s l a n d

THINK WRITE
y2 – y1
c 1 If we replace x2 with X, the rule will c a = m = ---------------
-
become y = aX + b. This is the equation x2 – x1
of the straight line, where a is the
gradient and b is the y-intercept. To find
a, write the formula for the gradient of
the line passing through 2 given points.
2 Select any 2 points on the line (note that Use (0, 4) and (25, 34.4).
they should not necessarily be from the
actual data).
34.4 – 4
3 Substitute the coordinates of the chosen a = -------------------
points into the formula and evaluate. 25 – 0
30.4
a = ----------
25
a = 1.216
4 To find b, simply read the value of the b=4
y-intercept from the graph.

If the relationship between the variables is not given, we have to make an assumption
of a model from the graph of the data. We then have to transform the data according to
our assumption. If the assumption was correct, the transformed data, when plotted, will
produce a perfectly straight, or nearly straight, line.
Note: In this section we will consider only the rules of the type y = ax2 + b, y = ax3 + b,
a
y = --- + b and so on (we will not allow for a horizontal translation), so that the
x
appropriate substitution can be made.

WORKED Example 19
The table below gives the values of 2 variables, x and y.
x 01 02 03 04 05 06
y 35 21 16 12 11 10
Establish the rule, connecting x and y, that fits these data.
THINK WRITE/DRAW
1 Plot y against x. y
35
30
25
20
15
10
5
0 x
1 2 3 4 5 6
Chapter 3 Other graphs and modelling 157
THINK WRITE/DRAW
a
2 Study the scatterplot. It appears to be a Assumption: y = --- + b
hyperbola. Write the appropriate formula x
(remember that we do not consider
horizontal translations in this section).
3 Check your assumption: prepare a new
1 1 1 0.5 0.33 0.25 0.2 0.17
table by replacing values of x with --- ---
x x
(leave the values of y unchanged). y 35 21 16 12 11 10
1
4 Plot y against --- . y
x
35
30
25
20
15
10
5
0 0.2 0.4 0.6 0.8 1 1
x–
5 Comment on the shape of the graph. The graph is very close to a straight line,
therefore the assumption of a hyperbolic
model is correct.
1 y
6 If we replace --- with X, the rule becomes
x 35
30
y = aX + b, which is the equation of the 25
straight line where a is the gradient and b 20
15
is the y-intercept. These (a and b) can be 10
found from the graph as follows. First, draw 5
in the line of best fit. 0 X
0.2 0.4 0.6 0.8 1
y2 – y1
7 Write the formula for the gradient. m = ---------------
-
x2 – x1
8 Select any 2 points on the line. Using (0.17, 10) and (1, 35):
35 – 10
9 Substitute the coordinates of the points m = -------------------
into the formula and evaluate. 1 – 0.17
25
m = ----------
0.83
m = 30.12
10 Write the value of a. Since a is the gradient, a = m
= 30.12.
11 Write the general equation of the y = mx + c
straight line.
12 Substitute the value of m and the 35 = 30.12 × 1 + c
coordinates of any of the 2 points, say
(1, 35) into the equation.
Continued over page
158 M a t h s Q u e s t M a t h s B Ye a r 1 1 f o r Q u e e n s l a n d

THINK WRITE/DRAW

13 Solve for c. 35 = 30.12 + c


c = 35 − 30.12
c = 4.88
14 State the value of b. Since b is the y-intercept, b = c = 4.88.
15 Substitute the values of a and b into The rule for the given data is:
a 30.12
y = --- + b to obtain the rule that fits the y = ------------- + 4.88.
x x
given data.

You may like to verify the relationships found in the previous worked examples by
eBook plus
using the Excel spreadsheet file found at www.jacplus.com.au.
Digital doc:
EXCEL Spreadsheet
Modelling
remember
1. Modelling is the process of finding the rule that fits the given data.
2. The rule itself is called a mathematical model.
3. The best way to start modelling is to produce a scatterplot of the original
data.
4. Use the scatterplot of the data to make an assumption of the model of the
a
relationship. It should be of the type y = ax2 + b, y = ax3 + b, y = --- + b and so
x
on. To test the assumption, transform the data accordingly. If the assumption is
correct, the transformed data when plotted will produce a straight, or nearly
straight line.
5. To find the values of a and b in the model, draw a line of best fit; a is the
gradient of the line and b is the y-intercept.

3H Modelling

WORKED 1 The data in each of the tables below exactly fit one of these rules: y = ax2, y = ax3,
Example
17 a
y = --- or y = a x . For each set of data, plot the values of y against x and hence select
x
the most appropriate rule and state the value of a.
eBook plus
a x −3 −2 −1 0 1 2 3
Digital docs:
SkillSHEET 3.1
y −8.1 −2.4 −0.3 0 0.3 2.4 8.1
Gradient of a straight line
EXCEL Spreadsheet
b x 0−2 −1 0 −1 −02 −03
Modelling
y −24 −6 0 −6 −24 −54
Chapter 3 Other graphs and modelling 159
c x 0 0.56 1.6 1.56 2.00
y 0 1.13 1.6 1.96 2.26
d x 1 2.5 4.25 5 10.5
y 5 2.5 1.25 1 00.5
e x 0−3.5 0−2 −1.5 0 −1.5 −02
y −40.5 −12 −1.5 0 −1.5 −12

2 Match each of the graphs with the appropriate model.


a
i y = ax2 + b ii y = ax3 + b iii y = --- + b iv y = a x + b
x
a y b y c y d y

x x x

3 multiple choice
a
Which of the graphs below could be modelled by y = --- + b?
x
y y
i y ii y
iii iv v y

x x

x x x

A i only B i, ii and iii C iv and v D i, ii and iv E i, iv, and v

WORKED 4 It is believed that for the data in the table below, the relationship between x and y can
Example
18
a
be modelled by y = ----2- + b.
x
x −0.2 −1 2.2 3.2 04 05.2
y −3.2 −1 4.9 14.5 29 46.8

a Plot the values of y against x.


b Plot the values of y against x2 and draw the line of best fit.
c Find the values of a and b.
5 The table below shows the values of 2 variables, x and y.

x 0−4 0−2.5 −10.5 −22 4.3 06


y −28 −13.5 −12.5 −10 4.3 41

Establish the mathematical model of the relationship between the variables, if it is


known that it is of the form y = ax3 + b.
160 M a t h s Q u e s t M a t h s B Ye a r 1 1 f o r Q u e e n s l a n d

6 The table below shows the results obtained from an experiment investigating the fre-
quency of a sound, f, and the length of the sound wave, λ.

λ 0000.3 000.5 001 003 05 08 10

f 1130.3 680.5 340 110 70 40 35

a Plot f against λ.
b From the following relationships, select the one which you think is suggested by
a
the plot: f = aλ2, f = --- , f = a λ .
λ
1
c Based on your choice in part b, plot f against either λ2, --- or λ , draw in the line
λ
of best fit and use it to find the rule that connects the 2 variables.

7 For her science assignment, Rachel has to find the relationship between the intensity
of the light, I, and the distance between the observer and the source of light, d. From
the experiments she obtains the following results.

d 001 001.5 02 02.5 03 3.5 4

I 270 120.5 68 43.5 30 22 17

a Plot the values of I against d. What form of relationship does the graph suggest?
b Nathan, Rachel’s older brother, is a physics student. He tells Rachel that from his
a
studies he is certain that the relationship is of the type I = ----2- . Use this information
d
to help Rachel to find the model for the required relationship.

WORKED 8 The table below gives the values of 2 variables, x and y. Establish the rule, connecting
Example
19
x and y, that fits these data.

x 0 1 3 05 07 09

y 4 7 9 11 12 13

9 Joseph is a financial adviser. He is studying the prices of shares of a particular com-


pany over the last 10 months.

Months 1.00 2.00 3.00 4.00 5.00 6.00 7.00 8.00 09.00 10.00

Price, $ 6.00 6.80 7.45 8.00 8.50 8.90 9.30 9.65 10.00 10.30

a Represent the information graphically.


b Establish a suitable mathematical model, which relates the share price, P, and the
number of the month, m.
c Use your model to help Joseph predict the share price for the next 2 months.
Chapter 3 Other graphs and modelling 161
Modelling using a graphics calculator
How does the accuracy of basketball shooters vary as their distance from the hoop post
changes? The following results were collected from a number of basketballers at a
training session. Each basketballer took 10 shots from various distances.

Distance from
hoop post (m) Accuracy

1 9

1.5 5.5

2 4.3

2.5 3.8

3 3.4

3.5 3.3

4 3.1

4.5 3.1

5 3

Note: At distances of greater than 5 metres from the hoop post, results became very
inconsistent.
How do we model these data? How can we find a model that will allow us to predict
the likely accuracy of a shot from a given distance? Plotting the data will reveal the general
relationship between accuracy of shooting and the shooter’s distance from the hoop post.

10
9
8
7
Accuracy out of 10

6
5
4
3
2
1
0
0 1 2 3 4 5 6
Distance (m)

As expected, the general trend shows that the further the shooter is from the hoop
post, d, the less accurate, a, the shooting becomes.
Although a linear model might provide a rough approximation for these data, we still
hope to find a model that fits better.
162 M a t h s Q u e s t M a t h s B Ye a r 1 1 f o r Q u e e n s l a n d

For a set of data in Exercise 3H, we first made an assumption of the model of the
relationship and then transformed the data accordingly. If the assumption was correct,
the transformed data when plotted produced a straight line. We then could use a line of
best fit to find the rule or model for the data.
Instead, let’s investigate the model for this data using a graphics calculator. We can
not only draw appropriate graphs but also use the statistical calculation function of the
calculator to inform us on how close the model fits the data.
A measure of how closely the data fit can be obtained by calculating the correlation
coefficient, r. The value of r lies between –1 and 1 and the closer it is to 1, the stronger
the association is between the model and the data. More commonly, r2, the coefficient
of determination, is used to measure the degree of correspondence between a model
and the data. The formula for r2 is quite involved and so a graphics calculator is useful
here.
For the Casio fx-9860G AU
1. Press MENU and then select STAT. Enter the data
for the distance from the hoop post in List 1 and
then enter the accuracy of the basketballers in
List 2. Press EXE after each entry. Label List 1 <Casio04.01>
as DIST and List 2 as ACC.

2. Press F1 (GRPH) and then F6 (SET) to


display the screen for StatGraph1. Ensure the
Graph Type is set as Scatter and that the XList is
shown as List 1 and the YList as List 2. <Casio04.01>

3. Press EXIT (or EXE ) to return to the List


screen. To see the plot of the data, press F1
(GPH1).

4. To try a linear model for the data, press F1 <Casio04.01>


(CALC) and then F2 (X). The Linear
Regression screen will be displayed. This shows
the model would have an equation of y = ax + b
where a = −1.14 and b = 7.69. (Remember that y
represents accuracy and x represents distance.)
However, the r2 value is 0.64, which indicates a <Casio04.01>
poor fit to the data.

5. Press F6 (DRAW) to see the model line drawn


with the data.

6. Press EXIT twice to return to the List screen. <Casio04.01>


Press F1 (GPH1) to show the plot of the original
data again and then press F1 (CALC). To try a
quadratic model for the data, press F4 (X^2). The
Quadratic Regression screen will be displayed.
This shows the model would have an equation of
y = ax2 + bx + c where a = 0.64, b = −4.99 and <Casio04.01>
c = 12.40. The value of 0.91 for r2 indicates that the
Chapter 3 Other graphs and modelling 163
model is a reasonable match for the data. (If you
wish, press F6 (DRAW) to see the model curve
drawn with the data.)
7. To see if a better match can be found, we will
perform a cubic regression on the original data. As
before, press EXIT until you return to the List
screen. Press F6 (GPH1) to show the plot of the <Casio04.01>
original data again and then press F1 (CALC). To
try a cubic model for the data, press F5 (X^3).
The Cubic Regression screen will be displayed.
This shows the model would have an equation of
y = ax3 + bx2 + cx + d where a = −0.32, b = 3.50,
c = −12.62 and d = 18.15. The value of 0.98 for
r2 indicates that the model is a good fit for the
data.
8. Press F6 (DRAW) to see how the model fits the
data. This model has the equation
y = −0.32x3 + 3.5x2 – 12.62x + 18.15
where y represents the accuracy and x represents <Casio04.01>
the distance.

We could continue modelling the data with other relationships. Try a quartic function
or some other relationship to see whether you can find a better match.

For the TI-Nspire CAS


1. Open a new Lists & Spreadsheet document
(press /N and then select 3: Lists &
Spreadsheet). Enter the data for distance in
column A and accuracy in column B. Enter a title <TI04.01>
of dist in column A and a title of acc in column B.

2. Press b and then select 4: Statistics followed


by 1: Stat Calculations.

<TI04.01>

3. To try a linear model for the data, select 3: Linear


Regression (mx + b). Press x to display the
options for X List. Use the NavPad to highlight
dist and then press ·. Press e to move to the
next line. Similarly, highlight acc for Y List and <TI04.01>
then press · followed by e to move to the next
line. (Note that the regression equation will be
saved as f1. This means you can easily graph the
equation if you wish. It will be stored as the
function f1(x).)
164 M a t h s Q u e s t M a t h s B Ye a r 1 1 f o r Q u e e n s l a n d

4. Press e until you reach the 1st column result


line. Ensure that c[] is entered to indicate the
statistics should be shown starting from column C.
Press e to highlight OK. <TI04.01>
5. Press · to display the Linear Regression
statistics in columns C and D. This shows the
model would have an equation of y = mx + b where
m = −1.14 and b = 7.69. (Remember that y
represents accuracy and x represents distance.)
However, the r2 value is 0.64, which indicates a
poor fit to the data.
6. As before, press b and then select 4: Statistics <TI04.01>
followed by 1: Stat Calculations. To try a quadratic
model for the data, select 6: Quadratic Regression.
Select dist for X List, acc for Y List and ensure e[] is
shown for 1st Result Column. (Note that the
regression equation will be saved as f2.)
Press · to display the Quadratic Regression
statistics in columns E and F. Use the NavPad to
move down the column to display the r2 value. This
<TI04.01>
shows the model would have an equation of
y = ax2 + bx + c where a = 0.64, b = –4.99 and
c = 12.40. The value of 0.91 for r2 indicates that the
model is a reasonable match for the data.
7. To see if a better match can be found, we will
perform a cubic regression on the original data.
Repeat step 6 but instead select 7: Cubic
Regression and ensure that g[] is shown for the 1st
Column Result. (Note that the regression equation <TI04.01>
will be saved as f3.) The Cubic Regression
statistics will be displayed in columns G and H.
This shows the model would have an equation of
y = ax3 + bx2 + cx + d where a = −0.32, b = 3.50,
c = −12.62 and d = 18.15. The value of 0.98 for r2
indicates that the model is a good fit for the data.
This model has the equation
y = −0.32x3 + 3.5x2 – 12.62x + 18.15
where y represents the accuracy and x represents
the distance.
8. To see how the model fits the data we can draw the
data points and the model curve on the same axes.
Press /I to insert a new document and then select
2: Add Graphs & Geometry. Adjust the Window <TI04.01>
Settings (press b and then select 4: Window
followed by 1: Window Settings) so that XMin is set
as 0, XMax as 6, YMin as 0 and YMax as 10.
To have the data points plotted as a scatterplot,
press b and then select 3: Graph Type followed
by 4: Scatter Plot. Press x to display the options
available for x. Highlight dist and then press ·.
Chapter 3 Other graphs and modelling 165
Press e to move to the field box for y. Press x to
display the options available for y. Highlight acc
and then press ·.
9. Press b and then select 3: Graph Type followed
by 1: Function so that the equation of the model
can be drawn. Press the up arrow on the NavPad
until the equation for f3(x) is shown in the function <TI04.01>
entry line. This is the equation for the cubic
regression model. Press · to display the curve
with the original data points.

We could continue modelling the data with other relationships. Try a quartic function
or some other relationship to see whether you can find a better match.

Assumptions and limitations of the model


When using this model to predict the accuracy of a shooter from a given distance, any
assumptions or limitations of the model must be recognised.
Assumptions
1. No specific mention in the data was made of the gender of the shooter. They may
have been all male or all female or a mixture. The gender of the shooter may affect
the model.
2. No knowledge of the location where the data were collected was given. Whether it
was an outdoor location or an indoor location may affect the results.
3. Were the data collected from a game or a specific test for shooting?
4. The type of ball was not indicated.
5. The extent of tiredness of the players involved in the trials may also affect the
results.
Limitations
1. The model should only be considered valid for distances between 1 and 5 metres.
2. The model may not be valid for young children.

Modelling using a graphics


3I calculator
1 Find a model using a graphics calculator to fit the data presented in Question 5 of
Exercise 3H. Compare this model to the one you found originally.
2 Find a model using a graphics calculator to fit the data presented in Question 7 of
Exercise 3H.
3 Find a model using a graphics calculator to fit the data presented in Question 9 of
Exercise 3H.
4 Data about wildlife can often be compiled using aerial photographs. As an example, the
lengths of crocodiles in an area can be accurately estimated using such a technique.
However it is not possible to estimate the weight of the crocodile from a photograph. It
is necessary to develop a model to predict the weight of a crocodile given its length.
166 M a t h s Q u e s t M a t h s B Ye a r 1 1 f o r Q u e e n s l a n d

Use the following data to develop a model. State the assumptions and limitations of
this model.

Length (cm) Weight (kg) Length (cm) Weight (kg)


325 59 208 38
128 23 175 32
1600 291 153 28
70 13 135 25
200 36 110 20
275 50 265 48
83 15 210 38
225 41 98 18
90 16 105 19
95 17 493 90
915 166 255 46
210 38 143 26
200 36

Goal accuracy
The results below were obtained when a basketball player at a training session had
10 shots for goal from various distances.

Horizontal distance from


2 3 4 5 6 7 8 9 10
hoop post (m)
Number of goals (from 10
8 6 3 4 3 1 2 2 1
shots)
a
1 Find the model of the form y = --- + b for
these data. x

2 Perform a goal shooting experiment


of your own for a particular
sport (e.g. netball, football, soccer) and
investigate a suitable model for your results.
3 Use your model to predict the number of
goals for a distance not included in your
results table, then carry out a goal shooting
trial from that distance to test the model.
Chapter 3 Other graphs and modelling 167

summary
Transforming graphs
• The graph of y = f(x) can be transformed through dilation, reflection or translation.
y = af(x) represents a dilation of y = f(x) in the y direction by a factor of a.
1
y = f(ax) represents a dilation of y = f(x) in the x direction by a factor of --- .
y = f(−x) represents a reflection of y = f(x) in the y-axis. a
y = −f(x) represents a reflection of y = f(x) in the x-axis.
y = f(x − a) represents a translation of y = f(x) of a units to the right.
y = f(x) + a represents a translation of y = f(x) of a units upwards.
• The graph of y = af(b(x − h)) + k can be obtained from the graph of y = f(x) by
performing the following transformations (in order):
1. Stretch along the y-axis by a factor of a.
1
2. Shrink along the x-axis by a factor of --- .
3. Move the graph h units to the right. b
4. Move the graph k units upwards.
Graphs of the parabolic, cubic, quartic, hyperbolic and square root
functions
Special
Name Equation Basic shape Domain Range feature
Parabola y = a(x − h)2 + k y R If a > 0 Turning
y≥k point at (h, k)
If a < 0
(h, k)
x y≤k
0
3 y
Cubic y = a(x − h) + k R R Stationary
point of
(h, k)
0 x inflection at
(h, k)
Quartic y = a(x − h)4 + k y R If a > 0 Turning
y≥k point at (h, k)
If a < 0
y≤k
(h, k)
0 x
y = a(x – h)4+k

Hyper- a y R\{h} R\{k} Horizontal


y = ----------- + k
bola x–h asymptote
k
y = k, vertical
0 x
asymptote
h x=h
Square y=a x–h +k
y x≥h If a > 0 End point at
root y≥k (h, k)
If a < 0
(h, k)
0 x y≤k
168 M a t h s Q u e s t M a t h s B Ye a r 1 1 f o r Q u e e n s l a n d

• For all of the functions in the previous table:


1. a is the dilation factor: it dilates the graph in the y direction. If a < 0, the graph
is reflected in the x-axis.
2. h translates the graph h units along the x-axis (to the right if h > 0, or to the left
if h < 0).
3. k translates the graph k units along the y-axis (up if k > 0, or down if k < 0).
4. If the equation contains (h − x) rather than (x − h), the graph is reflected in the
y-axis.
• To sketch graphs using intercepts:
1. Identify the shape of the graph and whether it is a positive or negative curve.
2. Find the y-intercept by substituting x = 0 into the equation and solving for the
corresponding y-value.
3. Find any x-intercepts by substituting y = 0 into the equation and solving for the
corresponding x-values.
4. Indicate the general position of any turning points. For a parabola with general
equation y = ax2 + bx + c, the x-coordinate of the turning point is midway
–b
between the x-intercepts or ------ .
2a
The absolute value function
• y = |x| means y = x, if x ≥ 0
and y = −x, if x < 0
y

• Domain: R
• Range: [0, ∞)
• To sketch the graph of y = | f(x)|:
1. Sketch the graph of y = f(x).
2. Reflect the portion of the graph that is below the x-axis in the x-axis.
• For functions of the form y = a| f(x)| + k, a and k have the same impact on the graph
of the absolute value function as on the graphs of all other functions discussed in
this section.

Addition of ordinates
• A graph of the sum of 2 functions can be drawn by sketching the 2 functions on the
same set of axes and then adding the y-values for each value of x.
• If h(x) = f(x) + g(x), domain h(x) = domain f(x) ∩ domain g(x).
• Suitable points at which to add ordinates are:
1. the end points of the graph
2. the points of intersection of the 2 graphs
3. the x-intercepts of the 2 graphs.
• The technique can be used for the difference of the 2 functions if it is rewritten as a
sum: h(x) = f(x) − g(x) = f(x) + [−g(x)].
Chapter 3 Other graphs and modelling 169
Modelling
• Modelling is the process of finding the rule (mathematical model) that fits the given
data.
• To model:
1. Plot the original data.
2. Make the assumption of the model.
3. Transform the data in accordance with your assumption.
4. Check the assumption by plotting the transformed data (if correct, the graph
will be a straight, or nearly straight, line).
5. Draw in a line of best fit.
6. Find the equation of the line (y = mx + c).
1
7. Replace x in the equation with the transformed variable (for example, x2, --- ).
x
170 M a t h s Q u e s t M a t h s B Ye a r 1 1 f o r Q u e e n s l a n d

CHAPTER
review y

1 Consider the graph of y = f(x) shown at right.


3A Use your knowledge of transformations to sketch the
–3 0 4 x
graph of each of the following functions.
a y = 2f(x) b y = f(x − 3)
c y = −f(x) d y = f(x) + 4

2 State the changes required to transform the graph of y = x2 into the graph of each of the
3B following.
a y = x2 + 7 b y = (x + 1)2 − 3 c y = −2(x − 3)2 + 1

3 For the function y = 2(x − 3)2 − 4:


3B a state the coordinates of the turning point
b state the domain and range
c sketch the graph.

4 Sketch the following graphs using transformations of the basic graphs of y = x2, y = x3 or
y = x4.
a y = −4(x + 1)2 + 2 b y = (x − 5)3 − 3 c y = (x + 2)4 − 5

5 multiple choice
3B Which of the following shows the graph of y = −2(x + 5)3 − 12?
A B C D E
y y y y y
(5, 12)
x x
(–5, 12)
(–5, –12) (5, –12) x
x
x (–5, –12)

6 multiple choice
3B The graph of y = 2--- (bx − 3)3 + 1 is dilated in the y direction by a factor of:
3
3
2b 2
A 2
--- B 2
--- b C -------- D b3 E -------3-
3 3 3 3b
7 Sketch the following graphs, showing all intercepts.
3C a y = (x − 6)(x − 14) b y = x2 + 2x − 80
c y = −2x2 − 5x − 3 d y = x(x − 2)(x + 11)
e y = −(x − 6)(x − 1)(x + 3) f y = (x − 1)2(x + 8)
g y = (3 − x)(x − 1)(x + 2)(x + 5) h y = x(x − 7)(x − 2)(x + 4)
2
i y = (2x − 1)(x + 1)(x + 4)
Chapter 3 Other graphs and modelling 171
8 multiple choice y
3C
The equation for this graph could be:
A y = (x − 5)(x + 1)(x + 3)
B y = (x − 3)(x − 1)(x + 5)
C y = (x − 3)(x + 1)(x + 5)
D y = (3x − 1)(x + 1)(x − 5)
E y = (5 − x)(1 + x)(3 + x) –3 –1 5 x

9 multiple choice
3C
The equation for the graph shown could be: y
A y = (x – 3)2 (x + 3)2
B y = (x – 3) (x + 3)3
0 x
C y = (x – 3)3 (x + 3) –3 3
D y = (x – 3)2 (x + 3)2 –81
E y = (x + 3)4

10 multiple choice
2
3D
If f(x) = --- + 1, then f(x) + 2 will have:
x
A the horizontal asymptote y = 2
B the horizontal asymptote y = 1
C the horizontal asymptote y = 3
D the vertical asymptote x = 2
E the vertical asymptote x = 1

11 multiple choice y 3D
The equation of the graph shown is likely to be:
2 2 2
A – ----------- – 1 B 1 − ------------ C – ------------ – 2
x–2 x+2 x+1 –2 x
–1
2 –2
D ------------ − 1 E ------------ − 1
x+2 x+2

1
12 The graph of y = --- was dilated by the factor of 4 in the y direction, reflected in the x-axis
x 3D
and then translated 2 units to the left and 1 unit down.
a State the equation of the asymptotes.
b State the domain and range.
c State the equation of the new graph.
d Sketch the graph.

13 Sketch the graph of each of the following, clearly showing the position of the asymptotes
and the intercepts with the axes. 3D
2 4 2
a y = ----------- b y = – --- – 1 c y = ----------- + 2
x–2 x x–4
14 Sketch the graph of each of the following.
a y=2 x +6 b y= x+3 −2 c y=5– x–1
3E
172 M a t h s Q u e s t M a t h s B Ye a r 1 1 f o r Q u e e n s l a n d

15 multiple choice
3E The equation of this graph is of the form: y
A y = a x – m + n, a > 0 B y = a m – x + n, a > 0 (m, n)

C y = a x – m + n, a < 0 D y = a m + x + n, a < 0 x
E y = a m – x + n, a < 0

16 multiple choice
3E The equation of this graph could be: y
A y=d− a–x B y= x–a +d d

C y=d− x–a D y=c− a–x


b a x
E y= a–x −c c
17 multiple choice
3F The equation of the graph shown in the diagram at right y
is best described by: 2
A y = |x + 2| + 2 B y = 2 − |x + 2|
C y = |2 − x| + 2 D y = 2 − |2 − x| –2 x
E y = |x + 2| − 2

18 Sketch the graph of each of the following.


3F a y = |x − 1| − 7 b y = |x2 − 4| + 3 c y = 5 – |x|

19 multiple choice
3G Which of the following shows the graph of y = x +4 + 4– x?
A y B y C y D y E y
4 4 4
2
4
–4 4 x –4 4 x
–4 4 x –4 4 x –2 2 x

20 The graphs of f(x) and g(x) are shown below. Sketch the graph of ( f + g)(x), using the
3G addition-of-ordinates technique.
a y b g(x)
y
g(x)
f(x) f(x)

x x

21 multiple choice
3H The data in the following table exactly fit one of these models: y = ax2, y = ax3 or y = a x .
x 1.3 2.3 3.3 04.3
y 0.3 2.4 8.1 19.2
The value of a is:
A 2.4 B 1.2 C 2.7 D 0.9 E 0.3
Chapter 3 Other graphs and modelling 173
22 multiple choice
1 3H
For certain data, the values of y are plotted against --- and the line y
x (1, 19)
of best fit is drawn as seen on the diagram at right. The model that
relates the variables x and y is:
A y = 20x − 1 B y = 19x + 1
x 19
C y = ------ − 1 D y = ------ − 1 (0.1, 1)
20 x 1
x–
20
E y = ------ − 1
x
a
23 The data in the table below exactly fit one of these models: y = ax3, y = ----2- or y = a x .
x 3H
x 02 4.25 5 10 20.25 25.25
y 25 6.25 4 01 00.25 00.16
a Plot the values of y against x and use the scatterplot to choose a suitable model.
1
b Plot the values of y against either x3, ----2- or x (depending on your choice in part a).
x
Did you choose the right model? Explain your answer.

Modelling and problem solving


1 The graph of y = f(x) is shown at right. y
D(4, 6)
a Sketch the graph of each of the following functions on the same
set of axes with the original graph and give the coordinates of
the points A, B, C and D. B C(2, 3)
A 2
iii y = −f(x) iii y = f(−x) iii y = f(x − 2) –3 –2 7 x
iv y = f(x) + 3 iv y = 2f(x) vi y = 1 − f(x + 1)
b Maya, a fabric designer, wishes to use the curve of
y = f(x) (red) to create a ‘wavy’ pattern as shown in 2 units
the diagram at right. apart
If she wants the waves to be 2 units apart vertically,
suggest the best way she could alter the equation of
y = f(x). (Remember a fabric has a fixed width!)

2 Lena and Alex are planning to buy a plot of land to build a new house. They watch the prices
of vacant lots in a specific area for the whole year. During each month they collect the data
and then, at the end of the month, calculate the average price for that month. The results of
their calculations are shown in the table below. (The prices given are in thousands of dollars.)

Month 1 2 3 4 5 6 7 8 9 10 11 12
Price 240 248 255 261 266 271 273 274 275 274 272 270
a Plot the prices against the months. What model does the graph suggest?
b If the model of the form y = a(x − h)2 + k is to be used for these data, what is (judging from
the graph) the most suitable value for h?
c Plot the values of y (the prices) against (x − h)2, where h is the value you’ve selected in
part b. Comment on the shape of the graph.
174 M a t h s Q u e s t M a t h s B Ye a r 1 1 f o r Q u e e n s l a n d

d Draw a line of best fit and find its equation. Hence, state the values of a and k in the model.
e Write the equation of the model.
f Find the equation of the model using a graphics calculator.
g According to the Real Estate Institute, the property market is on a steady rise (that is, the
prices are going up and are likely to rise further). Do the data collected by Lena and Alex
support this theory?
h Use the model to predict the average price for the next 2 months.
i Lena and Alex were planning to spend no more than $250 000 for their new plot. Several
months ago the prices were in their range, but they could not find what they wanted. If the
prices are going to behave according to our model, how long do they have to wait until the
prices fall back into their range?

NEW IMAGE TO COME 3E

3 An eagle soars from the top of a cliff that is 48.4 metres


above the ground and then descends towards
unsuspecting prey below.
The eagle’s height, h metres above the ground,
at time t seconds can be modelled by the equation
a
h = 50 + ------------- , where 0 ≤ t < 25 and a is a constant.
t – 25
a Find the value of a.
b Find the eagle’s height above the ground after
i 5 seconds ii 20 seconds.
c After how many seconds will the eagle reach the
ground?
d Comment on the changes in speed during the eagle’s
descent.
e Sketch the graph of the equation.
After 24 seconds, the eagle becomes distracted by another bird and reaches the ground
eBook plus exactly 2 seconds later. For this second part of the journey, the relationship between h and
t can be modelled by the equation h = a(t – 24)2 + c.
Digital doc: f Find the values of a and c.
Test Yourself
Chapter 3 g Fully define the hybrid function that describes the descent of the eagle from the top of the
cliff to the ground below.
Triangle
trigonometry
4
syllabus reference
Topic:
• Periodic functions and
applications

In this chapter
4A Calculating trigonometric
ratios
4B Finding an unknown side
4C Finding angles
4D Applications of right-angled
triangles
4E Using the sine rule to find
side lengths
4F Using the sine rule to find
angle sizes
4G Using the cosine rule to find
side lengths
4H Using the cosine rule to find
angle sizes
176 M a t h s Q u e s t M a t h s B Ye a r 1 1 f o r Q u e e n s l a n d

Introduction
Rick is standing on the southern side of the Grand
Canyon, wondering how wide it is. Through his binocu-
lars, Rick sights a lookout on the northern side of the
canyon, bearing N5°E. He then heads 8.1 km east along
the canyon’s southern rim and notes that the bearing of
the lookout is now N12°W. Using this information,
Rick is able to calculate the width of the Grand Canyon.
After studying this chapter you will be able to make
this calculation for yourself, as well as being able to
solve many other practical problems about sides and
angles within a triangle.

Calculating trigonometric ratios


From previous years work you should be familiar with Pythagoras’ theorem. This
enabled us to find the length of one side of a right-angled triangle given the length of
the other two. To use Pythagoras’ theorem, we had to recognise the hypotenuse in a
right-angled triangle. In trigonometry, we need to be able to name the two shorter sides
as well. We do this with reference to a given angle, and
label them opposite and adjacent. They are the sides hypotenuse
opposite
opposite and adjacent to the given angle. The diagram θ
shows the sides labelled with respect to the angle, θ. adjacent

Looking at the tangent ratio


The tangent ratio is a ratio of sides in similar I
right-angled triangles, such as those in the
G
diagram. ∠BAC is common to each triangle
E
eBook plus and is equal to 30°. We are going to look at the
ratio of the opposite side to the adjacent side in C
Digital doc: each triangle. You can do this either on your
EXCEL Spreadsheet
calculator or by logging into www.jacplus.com.au
Tangent A
and locating the weblinks for this chapter. H F D B
Complete each of the following measurements and calculations.
BC
1 a BC = mm b AB = mm c -------- =
AB
DE
2 a DE = mm b AD = mm c --------- =
AD
FG
3 a FG = mm b AF = mm c -------- =
AF
HI
4 a HI = mm b AH = mm c --------- =
AH
Remember that ∠BAC is common to each triangle. In each of the above, part c
is the ratio of the opposite side to the adjacent side of ∠BAC. What do you notice
about each of these answers?
C h a p t e r 4 Tr i a n g l e t r i g o n o m e t r y 177
Trigonometry uses the ratio of side lengths to calculate the lengths of sides and the
size of angles. The ratio of the opposite side to the adjacent side is called the tangent
ratio. This ratio is fixed for any particular angle.
The tangent ratio for any angle, θ, can be found using the result:
opposite side
tan θ = -------------------------------
adjacent side
In the investigation on the previous page, we found that for a 30° angle the ratio was
0.58. We can find a more accurate value for the tangent ratio on a calculator by pressing
tan and entering 30.
For all calculations in trigonometry you will need to make sure that your calculator
is in DEGREES MODE. For most calculators you can check this by looking for a DEG
in the display.

Graphics Calculator tip! Degrees mode


The following steps show how to set your calculator in degrees mode.
For the Casio fx-9860G AU
1. Press MENU to display the MAIN MENU. Use
the cursor keys to highlight RUN-MAT. Select it
by pressing EXE .

2. Press SHIFT [SET UP] and use the down


arrow to select Angle. Press F1 (Deg) to set
degrees.

For the TI-Nspire CAS


1. Go to the home screen (press c) and then
select 8: System Info.

2. Select 2: System Settings.


178 M a t h s Q u e s t M a t h s B Ye a r 1 1 f o r Q u e e n s l a n d

3. Press the e key until Angle is highlighted. Press


the down arrow on the NavPad to display the
Angle menu and then use the NavPad to select
Degree. Press · to accept this.
The calculator also needs to be set to
approximate values so that it will give decimal
answers to trigonometric expressions.

4. Continue to use the e key to highlight Auto


or Approx and then use the NavPad to select
Approximate. Press · to accept this.

5. Continue pressing the e key until OK is


highlighted and then press ·.

When measuring angles:


1 degree = 60 minutes
1 minute = 60 seconds
You need to be able to enter angles using both degrees and minutes into your calcu-
lator. Most scientific calculators use a DMS (Degrees, Minutes, Seconds) button or a
° ’ ” button.

WORKED Example 1
Using your calculator, find the following, correct to 3 decimal places.
8
a tan 60° b 15 tan 75° c ------------------ d tan 49°32′
tan 69∞
THINK WRITE/DISPLAY
a With a scientific calculator, press a tan 60° = 1.732
tan and enter 60.

b Enter 15, press × and tan , b 15 tan 75° = 55.981


enter 75.

8
c Enter 8, press ÷ and tan , c ----------------- = 3.071
enter 69. tan 69°

d Press tan , enter 49, press DMS , d tan 49°32′ = 1.172


enter 32, press DMS .
C h a p t e r 4 Tr i a n g l e t r i g o n o m e t r y 179

Graphics Calculator tip! Calculating trigonometric ratios


involving degrees and minutes
The following steps show how to calculate the value of both tan 60° and tan 49º32′.
Remember to check that your calculator is in degrees mode.
For the Casio fx-9860G AU
If the angle is in degrees only, the tangent of the angle is found in exactly the same way
as if using a scientific calculator. To calculate tan 60°, press tan , enter 60 and then
press EXE . Check you can do this by finding the value of tan 60° as shown above in
Worked example 1. If you get a different answer, it may be because the calculator is
not set to degrees. Check the bottom of your screen to see if it is set correctly.

To calculate the tangent of an angle in degrees and


minutes (for example, tan 49º32′), follow these steps.
1. Press MENU and then select RUN-MAT.
2. To set the calculator up to accept degrees and
minutes (and seconds), press OPTN and then
F6 ( ) for more options, followed by F5
(ANGL). The screen should appear as shown.
3. To calculate tan 49º32′, press tan , enter 49
and then press F4 (° ¢ ¢¢) . Enter 32 and again
press F4 (° ¢ ¢¢) . Press EXE to calculate its
value.

For the TI-Nspire CAS


If the angle is in degrees only, the tangent of the angle is found in exactly the same way
as if using a scientific calculator. Check you can do this by finding the value of tan 60°
as shown previously in Worked example 1. If you get a different answer, it may be
because the calculator is not set to degrees or approximation. Check the top of your
screen to see if it is set correctly.

To calculate the tangent of an angle in degrees and minutes (for example, tan 49º32),
follow these steps.
1. Open a new Calculator document (press /N
and then select 1: Add Calculator). Press o,
enter 49, then press / k to bring up the
symbol palette. Use the arrow keys on the NavPad
to highlight the degrees symbol.

2. Press · to accept this. Enter 32 and again access


the symbol palette (press / k). This time use
the arrow keys to highlight the minute symbol.
180 M a t h s Q u e s t M a t h s B Ye a r 1 1 f o r Q u e e n s l a n d

3. Press · and then press ) to close the set of


brackets. Press · to display the answer.

<TI04.03>

The tangent ratio is used to solve problems involving the opposite side and the
adjacent side of a right-angled triangle. The tangent ratio does not allow us to solve
problems that involve the hypotenuse.
The sine ratio (abbreviated to sin) is the name given to the ratio of the opposite side
and the hypotenuse.

Looking at the sine ratio


The tangent ratio is the ratio of the opposite side and the adjacent side in a right-
angled triangle. The sine ratio is the ratio of the opposite side and the hypotenuse.
eBook plus
Look back to the right-angled triangles used in the tangent investigation on page 176.
Digital doc: Complete each of the following measurements and calculations by using your
EXCEL Spreadsheet calculator or by logging into www.jacplus.com.au and locating the weblinks for
Sine
this chapter.
As we saw earlier, ∠BAC is common to all of these similar triangles; so in this
investigation, we look at the ratio of the side opposite ∠BAC to the hypotenuse of
each triangle.
BC
1 a BC = mm b AC = mm c -------- =
AC
DE
2 a DE = mm b AE = mm c -------- =
AE
FG
3 a FG = mm b AG = mm c --------- =
AG
HI
4 a HI = mm b AI = mm c ------ =
AI
In this investigation, part c is the ratio of the opposite side to ∠BAC to the
hypotenuse. You should again notice that the answers are the same (or very close,
allowing for measurement error).

In any right-angled triangle with equal angles, the ratio of the opposite side to the
hypotenuse will remain the same, regardless of the size of the triangle. The formula for
the sine ratio is:
opposite side
sin θ = -------------------------------
hypotenuse
The value of the sine ratio for any angle is found using the sin function on the calculator.
sin 30° = 0.5
Check this on your calculator.
C h a p t e r 4 Tr i a n g l e t r i g o n o m e t r y 181
WORKED Example 2
Find, correct to 3 decimal places:
18
a sin 57° b 9 sin 45° c ----------------- d 9.6 sin 26°12′.
sin 44∞
THINK WRITE/DISPLAY
a With a scientific calculator, press sin a sin 57° = 0.839
and enter 57.

b Enter 9, press × and sin , enter 45. b 9 sin 45° = 6.364

18
c Enter 18, press ÷ and sin , enter 44. c ----------------- = 25.912
sin 44°
d Enter 9.6, press × and sin , enter d 9.6 sin 26°12′ = 4.238
26, press DMS , enter 12, press DMS .

To calculate the sine of an angle using a graphics calculator, refer to the previous
Graphics calculator tip on page 179 and enter sin rather than tan.
A third trigonometric ratio is the cosine ratio. This ratio compares the length of the
adjacent side and the hypotenuse.

Looking at the cosine ratio


Look back to the right-angled triangles used in the tangent investigation on
eBook plus
page 176.
Digital doc: Complete each of the following measurements and calculations. You may do so
EXCEL Spreadsheet on the ‘Cosine’ spreadsheet found by logging into www.jacplus.com.au and
Cosine
locating the weblinks for this chapter.
AB
1 a AB = mm b AC = mm c -------- =
AC
AD
2 a AD = mm b AE = mm c --------- =
AE
AF
3 a AF = mm b AG = mm c --------- =
AG
AH
4 a AH = mm b AI = mm c --------- =
AI
Again for part c, you should get the same answer for each triangle. In each case,
this is the cosine ratio of the common angle BAC.

The cosine ratio is found using the formula:


adjacent side
cos θ = -------------------------------
hypotenuse
To calculate the cosine ratio for a given angle on your calculator, use the cos func-
tion. On your calculator check the calculation:
cos 30° = 0.866
182 M a t h s Q u e s t M a t h s B Ye a r 1 1 f o r Q u e e n s l a n d

WORKED Example 3
Find, correct to 3 decimal places:
21.3 4.5
a cos 27° b 6 cos 55° c ------------------ d -------------------------- .
cos 74∞ cos 82∞46¢
THINK WRITE/DISPLAY
a With a scientific calculator, press cos and enter 27. a cos 27° = 0.891

b Enter 6, press × and cos , enter 55. b 6 cos 55° = 3.441

21.3
c Enter 21.3, press ÷ and cos , enter 74. c ------------------ = 77.275
cos 74°
4.5
d Enter 4.5, press ÷ and cos , d -------------------------- = 35.740
cos 82°46′
enter 82, press DMS , enter 46, press DMS .

To calculate the cosine of an angle using a graphics calculator, refer to the previous
graphics calculator tip on page 179 and enter cos rather than tan.
Similarly, if we are given the sin, cos or tan of an angle, we are able to calculate the
size of that angle using the calculator. We do this using the inverse functions. On most
calculators these are the 2nd function of the sin, cos and tan functions and are denoted
sin−1, cos−1 and tan−1. On most calculators the shift key is used, as shown in the
following worked examples.

WORKED Example 4
Find θ, correct to the nearest degree, given that sin θ = 0.738.
THINK WRITE/DISPLAY
1 With a scientific calculator, press SHIFT [sin–1], θ = 47.561°
enter .738 and press = .
2 Round your answer to the nearest degree. θ = 48°

So far, we have dealt only with angles that are whole degrees. You need to be able to
make calculations using minutes as well. On most calculators, you will use the DMS
(Degrees, Minutes, Seconds) function or the ° ’ ” function.

WORKED Example 5
Given that tan θ = 1.647, calculate θ to the nearest minute.
THINK WRITE/DISPLAY
–1
1 With a scientific calculator, press SHIFT [tan ], θ = 58.735°
enter 1.647 and press = .
2 Convert your answer to degrees and minutes by θ = 58°44′
pressing DMS .
C h a p t e r 4 Tr i a n g l e t r i g o n o m e t r y 183

Graphics Calculator tip! Displaying an answer in degrees,


minutes and seconds
The following steps show how to convert an angle in decimal degrees to degrees,
minutes and seconds. Consider Worked example 5. If the tan of an angle is equal to
1.647, we can find the value of the angle to the nearest minute.
For the Casio fx-9860G AU
The ANGL function of the calculator can be used to convert angles in decimal degrees
to degrees, minutes and seconds.
1. First check that your calculator is in degrees
mode and that it is set to receive degrees,
minutes and seconds.
2. To calculate the angle, press SHIFT [tan–1],
enter 1.647 and then press EXE . The answer
is an angle in degrees as a decimal.

3. Press F5 (°¢¢¢) to convert the angle to degrees,


minutes and seconds. The answer required (to
the nearest minute) is 58º44′. (Note that when
rounding to the nearest minute, any value equal
to or greater than 30 seconds will result in the
angle being rounded up to the next minute.)

For the TI-Nspire CAS


1. Open a new Calculator document. Press /
then o to obtain tan−1, enter 1.647, press )
to close the set of brackets, and then press ·.
This displays the angle in degrees as a decimal.

2. To convert to degrees, minutes and seconds, press


k to access the catalog and then press 1 for
the list of functions. Scroll down until you find
DMS. (This can be done more quickly by first

pressing D.)

3. Press · to return to the calculator screen and


then press · again to display the answer. The
answer required (to the nearest minute) is
58º44. (Note that when rounding to the nearest
minute, any value equal to or greater than
30 seconds will result in the angle being
rounded up to the next minute.)
184 M a t h s Q u e s t M a t h s B Ye a r 1 1 f o r Q u e e n s l a n d

remember
1. The tangent ratio is the ratio of the opposite side
opposite side and the adjacent side. tan θ = -------------------------------
adjacent side
2. The sine ratio is the ratio of the opposite side
sin θ = -------------------------------
opposite side and the hypotenuse. hypotenuse
3. The cosine ratio is the ratio of the adjacent side
cos θ = -------------------------------
adjacent side and the hypotenuse. hypotenuse
4. The value of the trigonometric ratios can be found using the sin, cos and tan
functions on your calculator.
5. The angle can be found when given the trigonometric ratio using the sin−1,
cos−1 and tan−1 functions on your calculator.

Calculating trigonometric
4A ratios eBook plus
WORKED 1 Calculate the value of each of the following, correct to 3 decimal places. Digital docs:
Example
1 8.6 SkillSHEET 4.1
a tan 57° b 9 tan 63° c ----------------- d tan 33°19′ Rounding
tan 12°
SkillSHEET 4.2
Calculating
WORKED 2 Calculate the value of each of the following, correct to 3 decimal places. trigonometric
Example
ratios
2 14.5 48
a sin 37° b 9.3 sin 13° c ----------------- d -------------------------
sin 72° sin 67°40′
WORKED 3 Calculate the value of each of the following, correct to 3 decimal places.
Example
3 6
a cos 45° b 0.25 cos 9° c ------------------ d 5.9 cos 2°3′
cos 24°
4 Calculate the value of each of the following, correct to 4 significant figures.
a sin 30° b cos 15° c tan 45°
d 48 tan 85° e 128 cos 60° f 9.35 sin 8°
4.5 0.5 15
g ------------------ h ----------------- i -----------------
cos 32° tan 20° sin 72°
5 Calculate the value of each of the following, correct to 2 decimal places.
a sin 24°38′ b tan 57°21′ c cos 84°40′
d 9 cos 55°30′ e 4.9 sin 35°50′ f 2.39 tan 8°59′
19 49.6 0.84
g ------------------------- h -------------------------- i ----------------------
tan 67°45′ cos 47°25′ sin 75°5′
WORKED
Example 6 Find θ, correct to the nearest degree, given that sin θ = 0.167.
4
7 Find θ, correct to the nearest degree, given that:
a sin θ = 0.698 b cos θ = 0.173 c tan θ = 1.517.
WORKED
Example 8 Find θ, correct to the nearest minute, given that cos θ = 0.058.
5
9 Find θ, correct to the nearest minute, given that:
a tan θ = 0.931 b cos θ = 0.854 c sin θ = 0.277.
C h a p t e r 4 Tr i a n g l e t r i g o n o m e t r y 185
Finding an unknown side
We can use the trigonometric ratios to find the length of one hyp
side of a right-angled triangle if we know the length of another x opp
side and an angle. Consider the triangle at right. 30°
14 cm
In this triangle we are asked to find the length of the opposite adj
side and have been given the length of the adjacent side.
opposite x
We know from the formula that: tan θ = -------------------- . In this example, tan 30° = ------ .
adjacent 14
From our calculator we know that tan 30° = 0.577. We can set up an equation that will
allow us to find the value of x.
x
tan 30° = ------
14
x = 14 tan 30°
≈ 8.083 cm

WORKED Example 6
Use the tangent ratio to find the value of x in the triangle at right, h
correct to 2 decimal places.
55°
THINK WRITE 17 m

1 Label the sides of the triangle opp, adj


and hyp. hyp
h opp
55°
17 cm
adj
opp
2 Write the tangent formula. tan θ = ---------
adj
x
3 Substitute for θ (55°) and the adjacent tan 55° = ------
side (17 m). 17
4 Make x the subject of the equation. x = 17 tan 55°
5 Calculate and round to 2 decimal places. ≈ 24.28 cm

In the example above, we were told to use the tangent ratio. In practice, we need to
be able to look at a problem and then decide if the solution is found using the sin, cos
or tan ratio. To do this we need to examine the three formulas.
opposite side We use this formula when we are finding either the opposite or
tan θ = -------------------------------
adjacent side adjacent side and are given the length of the other.
opposite side The sin ratio is used when finding the opposite side or the
sin θ = -------------------------------
hypotenuse hypotenuse and we are given the length of the other.
adjacent side The cos ratio is for problems where we are finding the adjacent
cos θ = -------------------------------
hypotenuse side or the hypotenuse and are given the length of the other.
To make the decision we need to label the sides of the triangle and make a
decision based on these labels.
186 M a t h s Q u e s t M a t h s B Ye a r 1 1 f o r Q u e e n s l a n d

WORKED Example 7
Find the length of the side marked x, correct to 2 decimal places.
24 m
x
THINK WRITE
50°
1 Label the sides of the triangle.
hyp
24 m x opp

50°
adj
opp
2 Since x is the opposite side and 24 m is the sin θ = ---------
hypotenuse, use the sin formula. hyp
x
3 Substitute for θ and the hypotenuse. sin 50° = ------
24
4 Make x the subject of the equation. x = 24 sin 50°
5 Calculate and round to 2 decimal places. ≈ 18.39 m

Graphics Calculator tip! Using the equation solver


When finding the length of a side, you will be left with an equation to solve to obtain
your final answer. The solution steps can be cut out by using the equation solver.
x
Consider Worked example 7. At step 3 of the solution we are left with sin 50° = ------ ,
which can be solved as shown below. 24

For the Casio fx-9860G AU


1. Press MENU and select EQUA.

2. Press F3 (SOLV) and then select F3: Solver.

3. Delete any existing equation by pressing


F2 (DEL) and following the prompt. To enter
x
sin 50° = ------ , press sin and enter 50, and
24
then press SHIFT [=] followed by X,q,T (to
enter X), ÷ and 2 4 . Press EXE .
Ignore the value of X in the display at this
stage as it is a starting value for X.

4. Press F6 (SOLV) to solve this equation.


C h a p t e r 4 Tr i a n g l e t r i g o n o m e t r y 187
For the TI-Nspire CAS
1. Open a new Calculator document. Press k to
access the catalog and then press 1 for the list
of functions. Use the NavPad to scroll down to
solve(. You can do this more quickly by first
pressing S.

2. Press · to return to the calculator screen and


x
then enter the expression sin 50° = ------ . (Press
24
m50)=Xp24. Notice that ) was
used to close the set of brackets around 50.)

3. We then need to tell the calculator to solve the


equation for x. First press , to enter the
comma symbol and then press X to indicate
the variable. Close the set of brackets by
pressing ). Finally, press · to solve the
equation.

To remember each of the formulas more easily, we can use this acronym:
SOHCAHTOA
We may pronounce this acronym as ‘Sock ca toe her’. The initials of the acronym
represent the three trigonometric formulas.
opp opp
adj opp
sin q = cos q = tan q =
hyp hyp adj
Care needs to be taken at the substitution stage. In the above examples, the
unknown side was the numerator in the fraction; hence we multiplied to find the
answer. If after substitution, the unknown side is in the denominator, the final step is
done by division.

WORKED Example 8
Find the length of the side marked z in the triangle at right. z
23°15'
THINK WRITE
12.5 m
1 Label the sides opp, adj and hyp. hyp
z
23°15' opp

12.5 m
adj

Continued over page


188 M a t h s Q u e s t M a t h s B Ye a r 1 1 f o r Q u e e n s l a n d

THINK WRITE

2 Choose the cosine ratio because we are adj


cos θ = ---------
finding the hypotenuse and have been hyp
given the adjacent side. Write the formula.
12.5
3 Substitute for θ and the adjacent side. cos 23°15′ = ----------
z
4 Make z the subject of the equation. z cos 23°15′ = 12.5
12.5
z = --------------------------
cos 23°15′

5 Calculate and round appropriately. ≈ 13.60 m

Trigonometry is used to solve many practical problems. In these cases, it is necessary


to draw a diagram to represent the problem and then use trigonometry to solve the
problem. With written problems that require you to draw the diagram, it is necessary to
give the answer in words.

WORKED Example 9
A flying fox is used in an army training camp. The flying fox is supported by a cable that
runs from the top of a cliff face to a point 100 m from the base of the cliff. The cable
makes a 15° angle with the horizontal. Find the length of the cable used to support the
flying fox.

THINK WRITE

1 Draw a diagram and show information. hyp


Label the sides of the triangle opp, adj f
opp
and hyp.
15°
100 m
adj
adj
2 Choose the cosine ratio because we are cos θ = ---------
finding the hypotenuse and have been hyp
given the adjacent side. Write the
formula.
100
3 Substitute for θ and the adjacent side. cos 15° = ---------
f
4 Make f the subject of the equation. f cos 15° = 100
100
f = ------------------
cos 15°
5 Calculate and round appropriately. ≈ 103.5 m
6 Give a written answer. The cable is approximately 103.5 m long.
C h a p t e r 4 Tr i a n g l e t r i g o n o m e t r y 189
remember
1. Trigonometry can be used to find a side in a right-angled triangle when we are
given the length of one side and the size of an angle.
2. The trig formulas are:
opp adj opp
sin θ = --------- cos θ = --------- tan θ = ---------
hyp hyp adj
3. Take care to choose the correct trigonometric ratio for each question.
4. Substitute carefully and note the change in the calculation, depending upon
whether the unknown side is in the numerator or denominator.
5. Before using your calculator, check that it is in degrees mode.
6. Be sure that you know how to enter degrees and minutes into your calculator.
7. Problem questions will require you to draw a diagram and give a written
answer.

4B Finding an unknown side


1 Label the sides of each of the following triangles, with respect to the angle marked
eBook plus
with the pronumeral.
Digital doc: a b c
SkillSHEET 4.3 α
Labelling right-angled
triangles θ γ

WORKED 2 Use the tangent ratio to find the length of the side marked x
Example
6
(correct to 1 decimal place).
x

71°
51 mm

3 Use the sine ratio to find the length of the side marked a
(correct to 2 decimal places). 13 m
a
23°

4 Use the cosine ratio to find the length of the side marked d
(correct to 3 significant figures).
35 cm
31°
d
190 M a t h s Q u e s t M a t h s B Ye a r 1 1 f o r Q u e e n s l a n d

WORKED 5 The following questions use the tan, sin or cos ratios in their solution. Find the size of
Example
the side marked with the pronumeral, correct to 3 significant figures.
7
a b c
49°
12.5 km
x 13 cm 48 m
41°
68° y z

WORKED 6 Find the length of the side marked with the pronumeral in each of the following
Example
8
(correct to 1 decimal place).
a b c
p 77° q

21° 36°
t 87 mm 8.2 m

4.8 m

7 Find the length of the side marked with the pronumeral in each of the following
eBook plus
(correct to 3 significant figures).
Digital doc:
a 23°
b c x
SkillSHEET 4.4
Finding an
76°
unknown side b
a 0.85 km
8.5 km
39°
2.3 m

d 9° e d f
64.75 m
m
13°
116 mm x
16.75 cm
11°

g h i m
g
44.3 m 20°
15.75 km 84°9'
2.34 m
83°
x

j q k l
60°32' t 75°19' r
21.4 m
26.8 cm
84.6 km
29°32'
C h a p t e r 4 Tr i a n g l e t r i g o n o m e t r y 191
8 multiple choice
Look at the diagram at right and state which of the following is correct.
9.2
A x = 9.2 sin 69° B x = -----------------
sin 69° x
9.2
C x = 9.2 cos 69° D x = ------------------ 69°
cos 69°
9.2
E To find x you will need the length of the opposite side.

9 multiple choice
Study the triangle at right and state which of the following
φ
is correct.
8 15 15 17
A tan φ = ------ B tan φ = ------ C sin φ = ------ 15
15 8 17
8 8
D sin φ = ------ E cos φ = ------
15 17 8

WORKED 10 A tree casts a 3.6 m shadow when the sun’s angle of elevation is 59°. Calculate the
Example
9
height of the tree, correct to the nearest metre.

11 A 10 m ladder just reaches to the top of a wall when it is leaning at 65° to the ground.
How far from the foot of the wall is the ladder (correct to 1 decimal place)?

12 The diagram at right shows the paths of two ships, A and B, Port A
after they have left port. 60°
If ship B sends a distress signal, how far must ship A sail to 23 km
give assistance (to the nearest kilometre)?
B
13 A rectangle 13.5 cm wide has a diagonal that makes a 24° angle with the horizontal.
a Draw a diagram of this situation.
b Calculate the length of the rectangle, correct to 1 decimal place.

14 A wooden gate has a diagonal brace built in for support. The gate stands 1.4 m high
and the diagonal makes a 60° angle with the horizontal.
a Draw a diagram of the gate.
b Calculate the length that the diagonal brace needs to be.

15 The wire support for a flagpole makes a 70° angle with the ground. If the support is
3.3 m from the base of the flagpole, calculate the length of the wire support (correct
to 2 decimal places).

16 A ship drops anchor vertically with


an anchor line 60 m long. After one
hour the anchor line makes a 15° angle
with the vertical.
a Draw a diagram of this situation.
b Calculate the depth of water,
correct to the nearest metre.
c Calculate the distance that the
ship has drifted, correct to
1 decimal place.
192 M a t h s Q u e s t M a t h s B Ye a r 1 1 f o r Q u e e n s l a n d

Finding angles
In this chapter so far, we have concerned ourselves with finding side lengths. We are
also able to use trigonometry to find the sizes of angles when we have been given side
lengths. We need to reverse our previous processes.
Consider the triangle at right. We want to find the size of the angle marked θ. Using
the formula sin θ = opp
--------- we know that in this triangle: 10 cm
hyp
5 cm
θ
5
sin θ = ------
10
1
= ---
2
= 0.5
We then calculate sin−1 (0.5) to find that θ = 30°.
As with all trigonometry it is important that you have your calculator set to degrees
mode for this work.

WORKED Example 10
Find the size of angle θ, correct to the nearest degree, in the 4.3 m
triangle at right. θ
6.5 m
THINK WRITE
1 Label the sides of the triangle and hyp 4.3
opp
choose the tan ratio.
θ
6.5
adj
opp
tan θ = ---------
adj
4.3
2 Substitute for the opposite and adjacent = -------
sides in the triangle and simplify. 6.5
≈ 0.6615
3 Make θ the subject of the equation. θ = tan−1(0.6615)
4 Calculate and round to the nearest degree. ≈ 33°

Graphics Calculator tip! Using the equation solver to


find an angle
The equation solver can be used to find an angle in the same way that it can be used to
find the length of a side. Consider Worked example 10.
For the Casio fx-9860G AU
1. Press MENU and select EQUA.

2. Press F3 (SOLV) and then select F3: Solver.


C h a p t e r 4 Tr i a n g l e t r i g o n o m e t r y 193
3. Delete any existing equation by pressing
F2 (DEL) and following the prompt. Enter
tan x = 4.3 ÷ 6.5 and press EXE . Ignore the
value of X in the display at this stage as it is a
starting value.

4. Press F6 (SOLV) to solve this equation.

For the TI-Nspire CAS


1. Open a new Calculator document. Press k to
access the catalog and then press 1 for the
list of functions. Scroll down to solve(. You
can do this more quickly by first pressing S.

2. Press · to return to the calculator screen and


then enter the equation tan x = 4.3 ÷ 6.5.
(Remember to include brackets around x.)

3. The equation must be solved with respect to x


so, after the equation, press , to insert a
comma followed by X for the variable. Press
) to close the set of brackets.

4. On this calculator, we must specify the domain


over which we are to solve the equation; in this
case, between 0o and 90o. To do this, first press
* to enter the | symbol and then enter
0 ≤ x ≤ 90 (the ≤ symbol is found in the
symbol palette; press / k to access it).
Finally, press · to solve the equation.

In many cases we will need to calculate the size of an angle, correct to the nearest
minute. The same method for finding the solution is used; however, you will need to
use your calculator to convert to degrees and minutes.
194 M a t h s Q u e s t M a t h s B Ye a r 1 1 f o r Q u e e n s l a n d

WORKED Example 11 4.6 cm


Find the size of the angle θ on the right, correct to the nearest minute.

THINK WRITE 7.1 cm


θ
1 Label the sides of the triangle and opp
choose the sin ratio. 4.6 cm

adj hyp
7.1 cm
θ

opp
sin θ = ---------
hyp
4.6
2 Substitute for the opposite side and = -------
adjacent in the triangle and simplify. 7.1
≈ 0.6479
3 Make θ the subject of the equation. θ = sin−1(0.6479)
4 Calculate and convert your answer to ≈ 43°23′
degrees and minutes.

Graphics Calculator tip! Finding a solution in degrees


and minutes
If using the equation solver, the solution can only be displayed in degrees as a decimal.
Consider Worked example 11. To obtain the solution in degrees and minutes, use the
following steps.
For the Casio fx-9860G AU
1. Solve the equation as shown previously, which
gives an answer in degrees as a decimal.

2. Press MENU and then select RUN-MAT.

3. Press X,q,T and then EXE to recall the value


of X from the equation.

4. To access the angle functions, press OPTN


and then F6 ( ) for more options, followed
by F5 (ANGL). Press F5 (°¢¢¢) to convert to
degrees, minutes and seconds. The required
answer (to the nearest minute) is 40°23′.
C h a p t e r 4 Tr i a n g l e t r i g o n o m e t r y 195
For the TI-Nspire CAS
The solve function will display angles in degrees
as a decimal. The DMS function must be used to


convert the answer into degrees, minutes and sec-
onds. As shown earlier, it is best to use the inverse
sin function rather than trying to use the solve
function.
1. Enter the expression and display the answer.
2. Press k to access the catalog and then select
the DMS function.

3. Press · to return to the calculator screen and


then press · again to display the answer in
degrees, minutes and seconds. The required
answer (to the nearest minute) is 40°23′.

The same methods can be used to solve problems. As with finding sides, we set the
question up by drawing a diagram of the situation.

WORKED Example 12
A ladder is leaned against a wall. The foot of the ladder is 4 m from the base of the wall
and the ladder reaches 10 m up the wall. Calculate the angle that the ladder makes with
the ground.
THINK WRITE
1 Draw a diagram and label the sides.

opp
10 m hyp

θ
4m
adj
opp
2 Choose the tangent ratio and write the tan θ = ---------
formula. adj
10
3 Substitute for the opposite and adjacent = ------
side, and then simplify. 4
= 2.5
4 Make θ the subject of the equation. θ = tan−1(2.5)
5 Calculate and round to the nearest minute. = 68°12′
6 Give a written answer. The ladder makes an angle of 68°12′ with the
ground.
196 M a t h s Q u e s t M a t h s B Ye a r 1 1 f o r Q u e e n s l a n d

remember
1. Make sure that the calculator is in degrees mode.
2. To find an angle given the trig ratio, press SHIFT and then the appropriate
ratio button.
3. Be sure to know how to get your calculator to display an answer in degrees and
minutes. When rounding to the nearest minute, check if the number of seconds
is equal to or greater than 30.
4. When solving triangles remember the SOHCAHTOA rule to choose the correct
formula
5. In written problems, draw a diagram and give an answer in words.

4C Finding angles

1 Use the tangent ratio to find the size of the angle marked with the pronumeral in each
of the following, correct to the nearest degree.
a b c 25 mm
γ
162 mm
7m
θ 11 m
12 m
φ
3m

2 Use the sine ratio to find the size of the angle marked with the pronumeral in each of
the following, correct to the nearest minute.
a b 4.6 m c
24 m α 9.7 km
13 m
θ
6.5 m
5.6 km
θ

3 Use the cosine ratio to find the size of the angle marked with the pronumeral in each
of the following, correct to the nearest minute.
a b c
27.8 cm
15 cm 4.6 m 2.6 m
α β
θ 19.5 cm
9 cm
C h a p t e r 4 Tr i a n g l e t r i g o n o m e t r y 197
WORKED 4 In the following triangles, you will need to use all three trig ratios. Find the size of the
Example
angle marked θ, correct to the nearest degree.
10
a b c 14 cm
eBook plus θ θ
11 cm 15 cm 9 cm
Digital doc:
SkillSHEET 4.5
Finding an θ
unknown angle
7 cm 8 cm

d 3.6 m e f θ
32 mm
196 mm θ 26.8 m 14.9 m
9.2 m

WORKED 5 In each of the following find the size of the angle marked θ, correct to the nearest
Example
11
minute.

a b c 2.5 m
θ 0.6 m
θ
30 m
63 cm

θ
19.2 m 10 cm

d 3.5 m e f 6.3 m
16.3 m
θ 8.3 m
θ 18.9 m
18.5 m

6 multiple choice
Look at the triangle drawn at right. A
Which of the statements below is correct? 10 cm
5 cm
A ∠ABC = 30° B ∠ABC = 60° θ
C B
C ∠CAB = 30° D ∠ABC = 45°
E ∠CAB = 45°

7 multiple choice

3
The exact value of sin θ = ------- . The angle θ =
2
A 15° B 30° C 45° D 60° E 90°
WORKED
Example 8 A 10 m ladder leans against a 6 m high wall. Find the angle that the ladder makes
12 with the horizontal, correct to the nearest degree.
198 M a t h s Q u e s t M a t h s B Ye a r 1 1 f o r Q u e e n s l a n d

kite
9 A kite is flying on a 40 m string. The kite is flying 10 m away
from the vertical as shown in the figure at right.
Find the angle the string makes with the horizontal, correct
40 m
to the nearest minute.

10 m

10 A ship’s compass shows a course


due east of the port from which it
sails. After sailing 10 nautical
miles, it is found that the ship is
1.5 nautical miles off course as
shown in the figure below.

1.5 nm
10 nm

Find the error in the compass


reading, correct to the nearest
minute.

7m
11 The diagram at right shows a footballer’s shot at goal.
By dividing the isosceles triangle in half calculate, to the
nearest degree, the angle within which the footballer must
kick to get the ball to go between the posts. 30 m

12 A golfer hits the ball 250 m, but


20 m off centre. Calculate the
angle at which the ball deviated
from a straight line, correct to the
nearest minute.
C h a p t e r 4 Tr i a n g l e t r i g o n o m e t r y 199
Applications of right-angled triangles
The principles of trigonometry have been used throughout the ages, from the construction
of ancient Egyptian pyramids through to modern-day architecture, as well as for
measuring distances and heights which are either inaccessible or impractical. Two
important applications of right-angled triangles involve:
1. angles of elevation and depression, and
2. bearings.

Angles of elevation and depression


Angles of elevation and depression are employed when

ht
sig
dealing with directions which require us to look up and

of
ne
down respectively.

Li
An angle of elevation is the angle between the horizontal
and an object which is higher than the observer (for example, θ
the top of a mountain or flagpole). Angle of elevation

An angle of depression is the angle between the Angle of depression


horizontal and an object which is lower than the observer θ
(for example, a boat at sea when the observer is on a cliff). ht
sig
of
Unless otherwise stated, the angle of elevation or ne
Li
depression is measured and drawn from the horizontal.

Angles of elevation and depression are each measured D


from the horizontal.
When solving problems involving angles of elevation and
depression, it is best always to draw a diagram. E

The angle of elevation is equal to the angle of depression D and E are alternate angles
∴∠D=∠E
since they are alternate ‘Z’ angles.

WORKED Example 13
From a cliff 50 metres high, the angle of depression of a boat at sea is 12°. How far is the
boat from the base of the cliff?
THINK WRITE
1 Draw a diagram and label all the given 12°
information. Also, label the sides of the hyp 50 m opp
triangle.
12°
adj x
opp
2 Choose the tangent ratio because we are tan θ = ---------
finding the adjacent side and have been adj
given the opposite side.
50
3 Substitute for θ and the opposite side. tan 12° = ------
x
Continued over page
200 M a t h s Q u e s t M a t h s B Ye a r 1 1 f o r Q u e e n s l a n d

THINK WRITE

4 Make x the subject of the equation. x tan 12° = 50


50
x = -----------------
tan 12°
5 Calculate and round appropriately. ≈ 235.23
6 Give a written answer. The boat is 235.23 m away from the base of
the cliff.

WORKED Example 14
From a rescue helicopter 1800 m above the ocean, the angles of depression of two
shipwreck survivors are 60° (survivor 1) and 40° (survivor 2).
a Draw a labelled diagram which represents the situation.
b Calculate how far apart the two survivors are.
THINK WRITE
a Draw a diagram and label all the given a
information. Also, label the sides of the Helicopter
triangles.
opp
hyp
hyp 1800
40° 60°
S2 adj S1

b b Let x represent the horizontal distance


from the helicopter to a survivor.
1 For survivor 1, set up an equation to For survivor number 1:
solve to find the horizontal distance 1800
from the helicopter. tan 60° = ------------
x
2 Make x the subject of the equation. x tan 60° = 1800
1800
x = -----------------
tan 60°
3 Calculate and round appropriately. ≈ 1039.23 m
4 For survivor 2, set up an equation to For survivor number 2:
solve to find the horizontal distance 1800
from the helicopter. tan 40° = ------------
x
5 Make x the subject of the equation. x tan 40° = 1800
1800
x = -----------------
tan 40°
6 Calculate and round appropriately. ≈ 2145.16 m
7 Determine the distance between the two Distance apart = 2145.16 − 1039.23
survivors. = 1105.93
8 Give a written answer. The two survivors are 1105.93 m apart.
C h a p t e r 4 Tr i a n g l e t r i g o n o m e t r y 201
Bearings N
Bearings measure the direction of one object from another.
There are two systems used for describing bearings. 150° T
True bearings are measured in a clockwise direction,
starting from north (0° T).
Compass bearing
equivalent is S30°E

N N

Conventional or compass bearings are 20°


20°
measured first, relative to north or south; W E W E
then, relative to east or west.
S S
N20°W S70°E
True bearing equivalent True bearing equivalent
is 340° T is 110° T

N N
The two systems are interchangeable.
For example, a bearing of 240° T is the same
as S60°W. W E W E
When solving questions involving direction, 240° T 60°
always start with a diagram showing the basic
S S
compass points: north, south, east and west. S60°W

WORKED Example 15
A ship sails 40 km in a direction of N52°W. How far west of the starting point is it?

THINK WRITE

1 Draw a diagram of the situation, labelling N


each of the compass points and the given opp x
information. Also, label the sides of the hyp 40 hyp
adj
triangle. km 52°
W E

S
opp
2 Choose the sine ratio as we are finding the sin θ = ---------
opposite side and have been given the hyp
hypotenuse.
x
3 Substitute for θ and the hypotenuse. sin 52° = ------
40
4 Make x the subject of the equation. 40 sin 52° = x
x = 40 sin 52°
5 Calculate and round appropriately. ≈ 31.52
6 Give a written answer. The ship is 31.52 km west of the starting point.
202 M a t h s Q u e s t M a t h s B Ye a r 1 1 f o r Q u e e n s l a n d

WORKED Example 16
A ship sails 10 km east, then 4 km south.
What is its bearing from its starting point?

THINK WRITE

1 Draw a diagram of the N


adj
situation, labelling each of 10 km
the compass points and the θ opp
given information. Also, hyp 4 km
label the sides of the
S
triangle.
opp
2 Choose the tangent ratio, as tan θ = ---------
we have been given the adj
opposite and adjacent sides.
4
3 Substitute for opposite and tan θ = ------
adjacent. 10
4
4 Make θ the subject of the θ = tan−1  ------
equation. 10

5 Calculate and round to the = 21°48′


nearest minute.
6 Express the angle in Bearing = 90° + 21°48′
bearings form. The bearing Bearing = 110°48′ T
of the ship was initially 0° T;
it has since rotated through
an angle of 90° and an
additional angle of 21°48′.
To obtain the final bearing
these values are added.
7 Give a written answer. The bearing of the ship from its starting
point is 110°48′ T.

remember
1. Angles of elevation and depression are each measured from the horizontal.
2. The angle of elevation is equal to the angle of depression since they are
alternate ‘Z’ angles.
3. True bearings are measured in a clockwise direction, starting from north (000° T).
4. Conventional or compass bearings are first measured relative to north or south;
then, relative to east or west.
5. Whenever solving problems involving either angles or elevation and depression
bearings, it is best always to draw a diagram and to label all the given information.
6. Set up a compass as the basis of your diagram for bearings questions.
C h a p t e r 4 Tr i a n g l e t r i g o n o m e t r y 203
Applications of
4D right-angled triangles
WORKED 1 From a vertical fire tower 60 m high, the angle of depression to a fire is 6°. How far
Example
13
away, to the nearest metre, is the fire?
2 A person stands 20 m from the base of a building, and measures the angle of elevation
to the top of the building as 55°. If the person is 1.7 m tall, how high, to the nearest
metre, is the building?
3 An observer on a cliff top 57 m high observes a ship at sea. The angle of depression
eBook plus
to the ship is 15°. The ship sails towards the cliff, and the angle of depression is then
Digital docs: 25°. How far, to the nearest metre, did the ship sail between sightings?
SkillSHEET 4.6
Angles of elevation and
4 Two vertical buildings, 40 m and 62 m high, are directly opposite each other across a
depression river. The angle of elevation of the top of the taller building from the top of the
SkillSHEET 4.7 smaller building is 27°. How wide is the river? (Give the answer to 2 decimal places.)
Using trigonometric ratios
5 To calculate the height of a crane which is on top of a building, Dennis measures the
angle of elevation to the bottom and top of the crane. These were 62° and 68° respec-
tively. If the building is 42 m high, find, to 2 decimal places:
a how far Dennis is from the building
b the height of the crane.
6 A new skyscraper is proposed for the Brisbane city region. It is to be 200 m tall. What
would be the angle of depression, in degrees and minutes, from the top of the building
to the base of the Kangaroo Point cliffs, which is 4.2 km away?
WORKED 7 From a rescue helicopter 2500 m above the ocean, the angles of depression of two
Example
14
shipwreck survivors are 48° (survivor 1) and 35° (survivor 2).
a Draw a labelled diagram which represents the situation.
b Calculate how far apart the two survivors are.
8 A lookout tower has been erected on top of a mountain. At a distance of 5.8 km, the
angle of elevation from the ground to the base of the tower is 15.7° and the angle of
elevation to the observation deck (on the top of the tower) is 15.9°. How high, to the
nearest metre, is the observation deck above the top of the mountain?
9 From point A on level ground, the angle of elevation of the top of a building 50 m
high is 45°. From point B on the ground and in line with A and the foot of the
building, the angle of elevation of the top of the building is 60°. Find, in simplest surd
form, the distance from A to B.
10 Express the following conventional bearings as true bearings.
eBook plus
a N35°W b S47°W c N58°E d S17°E
Digital doc: 11 Express the following true bearings in conventional form.
SkillSHEET 4.8
Bearings
a 246° T b 107° T c 321° T d 074° T

12 multiple choice
a A bearing of S30°E is the same as:
A 030° T B 120° T C 150° T D 210° T E 240° T
b A bearing of 280° T is the same as:
A N10°W B S10°W C S80°W D N80°W E N10°E
204 M a t h s Q u e s t M a t h s B Ye a r 1 1 f o r Q u e e n s l a n d

WORKED 13 A canoeist paddles 1800 m on a bearing of N20°E. How far north of her starting point
Example
is she, to the nearest metre?
15
14 A yacht race consists of four legs. The first three legs are 4 km due east, then 5 km
south, followed by 2 km due west.
a How long is the final leg, if the race finishes at the starting point?
b On what bearing must the final leg be sailed?
WORKED
Example 15 A ship sails 20 km south, then 8 km west. What is its bearing from the starting point?
16
16 A cross-country competitor runs on a bearing of N60°W for 2 km, then due north for
3 km.
a How far is he from the starting point?
b What is the true bearing of the starting point from the runner?
17 Two hikers set out from the same camp site. One walks 7 km in the direction 043° T
and the other walks 10 km in the direction 133° T.
a What is the distance between the two hikers?
b What is the bearing of the first hiker from the second?
18 A ship sails 30 km on a bearing of 220°, then 20 km on a bearing of 250°. Find:
a how far south of the original position it is
b how far west of the original position it is
c the true bearing of the ship from its original position, to the nearest degree.
19 The town of Bracknaw is due west of Arley. Chris, in an ultralight plane, starts at a
third town, Champton, which is due north of Bracknaw, and flies directly towards Arley
at a speed of 40 km/h in a direction of 110° T. She reaches Arley in 3 hours. Find:
a the distance between Arley and Bracknaw
b the time to complete the journey from Champton to Bracknaw, via Arley, if she
increases her speed to 45 km/h between Arley and Bracknaw.
20 From point A on the ground, the angle of elevation of the top of a vertical tower due
north of A is 46°. From point B, due east of A, the angle of elevation of the top of the
eBook plus tower is 32°. If the tower is 85 m high, find:
a the distance from A to the foot of the tower
Digital doc: b the distance from B to the foot of the tower
WorkSHEET 4.1
c the true bearing of the tower from B.

Fly like a bird


A bird flying at 50 m above the ground was observed at noon from my front door
at an angle of elevation of 5°. Two minutes later its angle of elevation was 4°.
a If the bird was flying straight
and level, find the horizontal
distance of the bird:
i from my doorway at noon
ii from my doorway at 12.02 pm.
b Hence, find:
i the distance travelled by the bird in the
two minutes
ii its speed of flight in km/h.
C h a p t e r 4 Tr i a n g l e t r i g o n o m e t r y 205
Using the sine rule to find
side lengths
The trigonometry we have studied so far has been appli- C
cable to only right-angled triangles. The sine rule allows C
b a
us to calculate the lengths of sides and the size of angles
in non-right-angled triangles. Consider the triangle drawn A B
on the right. A c B

Derivation of the sine rule


A, B and C represent the three angles in the triangle ABC and a, b and c represent
the three sides, remembering that each side is named with the lower-case letter of
the opposite vertex.
Construct a line from C to a point, D, perpendicular to AB. CD is the
perpendicular height of the triangle, h.
C

b a
h

A B
A D B
c
Now consider ∆ACD and ∆BCD separately.
C C

b a
h h

A D D B
Using the formula for the sine ratio:
opp opp
sin θ = --------- sin θ = ---------
hyp hyp
h h
sin A = --- sin B = ---
b a
h = b sin A h = a sin B
We are now able to equate these two expressions for h.
a sin B = b sin A Dividing both sides by sin A sin B we get:
a sin B b sin A
--------------------------- = ---------------------------
sin A sin B sin A sin B
a b
------------ = ------------
sin A sin B
c
Similarly, we are able to show that each of these is also equal to ------------- . Try it!
sin C
206 M a t h s Q u e s t M a t h s B Ye a r 1 1 f o r Q u e e n s l a n d

The sine rule states that in any triangle, ABC, the ratio of each side to the sine of its
opposite angle will be equal.
a b c
------------- = ------------- = -------------
sin A sin B sin C
This formula allows us to calculate the length of a side in any triangle if we are given
the length of one other side and two angles. When using the formula we need to use
only two parts of it.

WORKED Example 17 A
Calculate the length of the side marked x in the 80°
triangle on the right, correct to 1 decimal place. 16 cm

40°
B x C
THINK WRITE

a b
1 Write the formula. ------------ = ------------
sin A sin B
x 16
2 Substitute a = x, b = 16, A = 80° and ----------------- = -----------------
sin 80° sin 40°
B = 40°.
16 sin 80°
3 Make x the subject of the equation by x = -------------------------
sin 40°
multiplying by sin 80°.
4 Calculate and round to 1 decimal place. x ≈ 24.5 cm

You can use the equation solver on a graphics calculator (as shown earlier) to solve
sine rule questions as well.
To use the sine rule we need to know the angle opposite the side we are finding and
the angle opposite the side we are given. In some cases these are not the angles we are
given. In such cases we need to use the fact that the angles in a triangle add to 180° to
calculate the required angle.

WORKED Example 18 A
Calculate the length of the side labelled m in the figure on the
65°
right, correct to 4 significant figures.
m

75°
B 16 m C
THINK WRITE
1 Calculate the size of angle C. C = 180° − 65° − 75°
= 40°
C h a p t e r 4 Tr i a n g l e t r i g o n o m e t r y 207

THINK WRITE

a c
2 Write the formula. ------------ = -------------
sin A sin C
16 m
3 Substitute a = 16, c = m, A = 65° and C = 40°. ----------------- = -----------------
sin 65° sin 40°
16 sin 40°
4 Make m the subject of the equation. m = -------------------------
sin 65°
5 Calculate and round to 4 significant figures. = 11.35 m

Using the sine rule allows us to solve a number of more complex problems. As with
our earlier trigonometry problems, we begin each with a diagram and give a written
answer to each.

WORKED Example 19 A
George looks south and observes an aeroplane at an angle of elevation
of 60°. Henrietta is 20 km south of where George is and she faces
x
north to see the aeroplane at an angle of elevation of 75°.
Calculate the distance of the aeroplane from Henrietta’s
observation point, to the nearest metre. 60° 75°
G 20 km H
THINK WRITE

1 Calculate the size of ∠GAH. A = 180° − 60° − 75°


= 45°
g a
2 Write the formula. ------------- = ------------
sin G sin A
x 20
3 Substitute g = x, a = 20, G = 60° and H = 75°. ----------------- = -----------------
sin 60° sin 45°
20 sin 60°
4 Make x the subject. x = -------------------------
sin 45°
5 Calculate and round to 3 decimal places x = 24.495 km
(nearest metre).
6 Give a written answer. The distance of the aeroplane from
Henrietta’s observation point is 24.495 km.

The sine rule can be used to calculate the height of objects that it would otherwise
be difficult to measure. Problems such as this can be solved by combining the use of
the sine rule with the trigonometry of right-angled triangles covered earlier in this
chapter.
208 M a t h s Q u e s t M a t h s B Ye a r 1 1 f o r Q u e e n s l a n d

WORKED Example 20
To calculate the height of a building, Kevin measures the angle of elevation to the top as
52°. He then walks 20 m closer to the building and measures the angle of elevation as 60°.
How high is the building?
THINK WRITE
1 Draw a labelled diagram of the situation C
and fill in the given information.

h
120°
52° 60°
A B D
20 x – 20
x

2 Check that one of the criteria for the sine The sine rule can be used for triangle ABC since
rule has been satisfied for triangle ABC. two angles and one side length have been given.
3 Determine the value of angle ACB, using ∠ACB = 180° − (52° + 120°)
the fact that the angle sum of any triangle = 8°
is 180°.
4 Write down the sine rule to find b. To find side length b of triangle ABC:
b c
------------ = -------------
sin B sin C
Substitute the known values into the rule. b 20
5 -------------------- = --------------
sin 120° sin 8°
20 × sin 120°
6 Make b the subject of the equation. b = --------------------------------
sin 8°
7 Calculate and round the answer to 2 decimal
places and include the appropriate unit. = 124.45 m
8 Draw a diagram of the situation, that is, C
triangle ADC, labelling the required
information. Also label the sides of the
124.45 m
triangle. h

52°
A D

opp
9 Choose the sine ratio as we are finding the sin θ = ---------
opposite side and have been given the hyp
hypotenuse.
h
10 Substitute for θ and the hypotenuse. sin 52° = ----------------
124.45
11 Make h the subject of the equation. 124.45 sin 52° = h
h = 124.45 sin 52°
12 Calculate and round appropriately. = 98.07
13 Give a written answer. The height of the building is 98.07 m.
C h a p t e r 4 Tr i a n g l e t r i g o n o m e t r y 209
remember
a b c
1. The sine rule formula is ------------ = ------------ = ------------- .
sin A sin B sin C
2. The sine rule is used to find a side in any triangle when we are given the length
of one other side and two angles.
3. We need to use only two parts of the sine rule formula.
4. For written problems, begin by drawing a diagram and finish by giving a
written answer.

Using the sine rule to find


4E side lengths
1 Write down the sine rule formula as it applies to each of the triangles below.
a A b X c P

c b

B a C Z Y
R Q
WORKED 2 Use the sine rule to calculate the length of the side marked with the pronumeral in
Example
17
each of the following, correct to 3 significant figures.
a A b L c R

x 63° 52°
16 cm 1.9 km t

50° 45°
B C 59° 84°
M q N T 89 mm S

WORKED 3 In each of the following, use the sine rule to calculate the length of the side marked with
Example
18
the pronumeral, correct to 1 decimal place, by first finding the size of the third angle.
a G x H b N c B
74° 74° 85° y
19.4 km

80° m
18.2 mm 62° P 27°
C
A
35.3 cm
I M

4 ABC is a triangle in which BC = 9 cm, ∠BAC = 54° and ∠ACB = 62°. Calculate the
length of side AB, correct to 1 decimal place.
5 XYZ is a triangle in which y = 19.2 m, ∠XYZ = 42° and ∠XZY = 28°. Calculate x,
correct to 3 significant figures.
210 M a t h s Q u e s t M a t h s B Ye a r 1 1 f o r Q u e e n s l a n d

WORKED 6 X and Y are two trees, 30 m apart on one side of a river. Z is a tree on the opposite
Example
side of the river, as shown in the diagram below.
19
Z

59° 72°
X 30 m Y

It is found that ∠XYZ = 72° and ∠YXZ = 59°. Calculate the distance XZ, correct to
1 decimal place.
7 From a point, M, the angle of elevation to the top of a building, B, is 34°. From a
point, N, 20 m closer to the building, the angle of elevation is 49°.
a Draw a diagram of this situation.
b Calculate the distance NB, correct to 1 decimal place.
c Calculate the height of the building, correct to the nearest metre.
WORKED 8 To calculate the height of a building, Kevin measures the angle of elevation to the top
Example
20
as 48°. He then walks 18 m closer to the building and measures the angle of elevation
as 64°. How high is the building?
9 A river has parallel banks which run directly east–west. Kylie takes a bearing to a tree
on the opposite side. The bearing is 047° T. She then walks 10 m due east, and takes
a second bearing to the tree. This is 305° T. Find:
a her distance from the second measuring point to the tree
b the width of the river, to the nearest metre.

10 A cross-country runner runs at 8 km/h


on a bearing of 150° T for 45 mins, then
changes direction to a bearing of 053° T
and runs for 80 mins until she is due
east of the starting point.
a How far was the second part of the
run?
b What was her speed for this section?
c How far does she need to run to get
back to the starting point?
11 From a fire tower, A, a fire is spotted on
a bearing of N42°E. From a second
tower, B, the fire is on a bearing of
N12°W. The two fire towers are 23 km
apart, and A is N63°W of B. How far is
the fire from each tower?
C h a p t e r 4 Tr i a n g l e t r i g o n o m e t r y 211
12 multiple choice
A boat sails on a bearing of N15°E for 10 km, then on a bearing of S85°E until it is
due east of the starting point. The distance from the starting point to the nearest
kilometre is:
A 10 km B 38 km C 110 km D 113 km E 114 km

13 multiple choice
A hill slopes at an angle of 30° to the horizontal. A tree which is 8 m tall is growing
at an angle of 10° to the vertical and is part-way up the slope. The vertical height of
the top of the tree above the slope is:
A 7.37 m B 8.68 m C 10.84 m D 15.04 m E 39.89 m

14 A cliff is 37 m high. The rock slopes outward at an angle


of 50° to the horizontal, then cuts back at an angle of 25°
50°
to the vertical, meeting the ground directly below the top
of the cliff.
25° rock
Carol wishes to abseil from the top of the cliff to the
rope 37 m
ground as shown in the diagram. Her climbing rope is
45 m long, and she needs 2 m to secure it to a tree at the
top of the cliff. Will the rope be long enough to allow her
to reach the ground?

Bearing east and west


At the beginning of this
chapter we looked at Rick,
who wanted to calculate the
width of the Grand Canyon.
From a point on the
southern side of the canyon
Rick sighted a lookout on a
bearing of N5°E. Rick then
headed east along the
southern rim for 8.1 km,
where the bearing of the
lookout was N12°W.
1 Draw a diagram of this situation and label the lookout on the northern rim A,
the easternmost point B and the westernmost point C.
2 Use the sine rule to calculate the distance AB.
3 Draw a perpendicular line from A to the side AB. This will represent the width
of the canyon.
4 Use right-angled triangle trigonometry to find the width of the canyon in
kilometres, correct to 1 decimal place.
212 M a t h s Q u e s t M a t h s B Ye a r 1 1 f o r Q u e e n s l a n d

Using the sine rule to find angle sizes


Using the sine rule result we are able to calculate angle sizes as well. To do this, we
need to be given the length of two sides and the angle opposite one of them. For
simplicity, in solving the triangle we invert the sine rule formula when we are using it
to find an angle.
The formula is written:
sin A sin B sin C
------------ = ------------ = -------------
a b c
As with finding side lengths, we use only two parts of the formula.

WORKED Example 21
Find the size of the angle, θ, in the figure on the right, A
correct to the nearest degree. 110°
6 cm
θ C
20 cm
B
THINK WRITE
sin A sin C
1 Write the formula. ------------ = -------------
a c
sin 110° sin θ
2 Substitute A = 110°, C = θ, a = 20 and -------------------- = ------------
20 6
c = 6.
6 sin 110°
3 Make sin θ the subject of the equation. sin θ = -------------------------
20
4 Calculate a value for sin θ. sin θ = 0.2819
5 Calculate sin−1(0.2819) to find θ. θ = 16°

As with finding side lengths, some questions will be problems that require you to
draw a diagram to extract the required information and then give the answer in written
form.

WORKED Example 22
From point P, a ship (S) is sighted 12.4 km away on a bearing of 137°. Point Q is due south
of P and is a distance of 31.2 km from the ship. Calculate the bearing of the ship from Q,
correct to the nearest degree.
THINK WRITE
1 Draw a diagram. P 137°
12.4 km
43° S

31.2 km

Q
C h a p t e r 4 Tr i a n g l e t r i g o n o m e t r y 213

THINK WRITE

sin Q sin P
2 Write the formula. ------------- = ------------
q p
sin Q sin 43°
3 Substitute for p, q and P. ------------- = -----------------
12.4 31.2
12.4 sin 43°
4 Make sin Q the subject. sin Q = ------------------------------
31.2
5 Calculate a value for sin Q. sin Q = 0.271
6 Calculate sin−1(0.271) to find Q. Q = 16°
7 Give a written answer. The bearing of the ship from Q is 016° T.

Remember that you can use the equation solver on a graphics calculator to solve
these questions.

remember
sin A sin B sin C
1. The sine rule formula for finding an angle is ------------ = ------------ = ------------- .
a b c
2. We can use this formula when we are given two sides and the angle opposite
one of them.
3. Problem questions should begin with a diagram and finish with a written
answer.

Using the sine rule to find


4F angle sizes
WORKED 1 Find the size of the angle marked with a pronumeral in each of the following, correct to
Example
21 the nearest degree.
a A b P c L
φ
100° 32 cm
29.5 m 153 mm
θ 79 mm
B C
46 cm
Q 60° R 117° α
18.9 m M N

d V e X f 27 mm
170°
16.5 cm

23.6 km 23.6 km 27.6 cm θ 156 mm

75° θ 86° β
U W Y Z
214 M a t h s Q u e s t M a t h s B Ye a r 1 1 f o r Q u e e n s l a n d

2 multiple choice
7 13
Which of the statements below give the
correct value for sin θ ?
36° θ

13 sin 36° 7 sin 36°


A sin θ = ------------------------- B sin θ = ----------------------
7 13
36 sin 13° 7 sin 13°
C sin θ = ------------------------- D sin θ = ----------------------
7 36
13 sin 7°
E sin θ = ----------------------
36

3 multiple choice
In which of the triangles below is the information insufficient to use the sine rule?
A B θ
θ
14.8 m 12.7 m

57° 45°
12.6 m 16.2 m

C D 8.7 m
115° 6.2 m 12.7 m
θ 9°
θ
12.9 m
E 28.1 m
θ
15.3 m 122°

4 In ∆PQR, q = 12 cm, r = 16 cm and ∠PRQ = 56°. Find the size of ∠PQR, correct to
the nearest degree.
5 In ∆KLM, LM = 4.2 m, KL = 5.6 m and ∠KML = 27°. Find the size of ∠LKM, correct
to the nearest degree.
WORKED 6 A, B and C are three towns marked on a map. Judy calculates that the distance between
Example
22
A and B is 45 km and the distance between B and C is 32 km. ∠CAB is 45°. Calculate
∠ACB, correct to the nearest degree.
7 A surveyor marks three points X, Y and Z in the ground. The surveyor measures XY to
be 13.7 m and XZ to be 14.2 m. ∠XYZ is 60°.
a Calculate ∠XZY to the nearest degree.
b Calculate ∠YXZ to the nearest degree.
8 Two wires support a flagpole. The first wire
is 8 m long and makes a 65° angle with the
ground. The second wire is 9 m long. Find
the angle that the second wire makes with
the ground.
9 A ship sails on a bearing of S20°W for 14 km, then changes direction and sails for
20 km and drops anchor. Its bearing from the starting point is now N65°W.
a How far is it from the starting point?
b On what bearing did it sail the 20 km leg?
C h a p t e r 4 Tr i a n g l e t r i g o n o m e t r y 215
Using the cosine rule to find
side lengths
The sine rule will not allow us to solve all triangles. Depending on the information pro-
vided about the triangle we may need to use the cosine rule.
In any non-right-angled triangle, ABC, a perpendicular line can be drawn from angle
B to side b. Let D be the point where the perpendicular line meets side b, and the length
of the perpendicular line be h. Let the length AD = x units. The perpendicular line
creates two right-angled triangles, ADB and CDB. B
Using triangle ADB and Pythagoras’ theorem, we obtain:
c2 = h2 + x2 [1] c a
h
Using triangle CDB and Pythagoras’ theorem, we obtain:
a2 = h2 + (b − x)2 [2]
A D C
Expanding the brackets in equation [2]: x b–x
2 2 2 2 b
a = h + b − 2bx + x
Rearranging equation [2] and using c2 = h2 + x2 from equation [1]:
a2 = h2 + x2 + b2 − 2bx
= c2 + b2 − 2bx
= b2 + c2 − 2bx
From triangle ABD, x = c cos A, therefore a2 = b2 + c2 − 2bx becomes
a2 = b2 + c2 − 2bc cos A
This is called the cosine rule and is a generalisation of Pythagoras’ theorem.
In a similar way, if the perpendicular line was drawn from angle A to side a or from
angle C to side c, the two right-angled triangles would give c2 = a2 + b2 − 2ab cos C
and b2 = a2 + c2 − 2ac cos B respectively. From this, the cosine rule can be stated:
In any triangle ABC B
2 2 2
a = b + c − 2bc cos A c a
b2 = a2 + c2 − 2ac cos B
c2 = a2 + b2 − 2ab cos C A b C
The cosine rule can be used to solve non-right-angled triangles if we are given:
1. three sides of the triangle
2. two sides of the triangle and the included angle (the angle between the given sides).

WORKED Example 23
Find the third side of triangle ABC given a = 6, c = 10 and B = 76°.
THINK WRITE
1 Draw a labelled diagram of the triangle B
ABC and fill in the given information. c = 10 76° a=6

A b C
2 2 2
2 Write the appropriate cosine rule to find b. b = a + c − 2ac cos B
3 Substitute the given values into the rule. = 62 + 102 − 2 × 6 × 10 × cos 76°
4 Evaluate. ≈ 106.969 372 5
b = 106.969 372 5
5 Round the answer to 2 decimal places. ≈ 10.34
216 M a t h s Q u e s t M a t h s B Ye a r 1 1 f o r Q u e e n s l a n d

Graphics Calculator tip! Using equation solver to solve cosine


rule problems (sides)
Using the equation solver method for the cosine rule is very useful as many students
forget the final step of the solution, which is to take the square root of a2, b2 or c2.
In the same way as with earlier questions, we write the formula and then substitute
the appropriate values, leaving one unknown. Hence we have an equation, which can be
typed into the equation solver of a graphics calculator. Consider Worked example 23 on
the previous page.
For the Casio fx-9860G AU
1. Press MENU and select EQUA.
2. Press F3 (SOLV) and then select F3: Solver.

3. Delete any existing equation.


Enter B2 = 62 + 102 − 2 × 6 × 10 × cos 76° and
then press EXE . (Ignore the value of B in the
display as this is a starting value.)

4. Press F6 (SOLV) to solve this equation.

For the TI-Nspire CAS


1. Open a new Calculator document. Press k to
access the catalog and then press 1 for the list
of functions. Scroll down to solve(. You can do
this more quickly by first pressing S.

2. Press · to return to the calculator screen and


then enter b2 = 62 + 102 − 2 × 6 × 10 × cos 76°.
Remember to solve the equation for b at the end
of the expression. Press · to display the
solution.
Notice that because the formula uses b2 there
can be two solutions; hence the calculator shows
both solutions –10.3426 and 10.3426. In this
situation we should ignore the negative solution.
C h a p t e r 4 Tr i a n g l e t r i g o n o m e t r y 217
The cosine rule also allows us to solve a wider range of practical problems. The
important part of solving such problems is marking the correct information on your
diagram. If you can identify two side lengths and the included angle, you can use the
cosine rule.

WORKED Example 24
A surveyor standing at point X sights point M, 50 m away and point N, 80 m away.
If the angle between the lines XM and XN is 45°, calculate the distance between the points
M and N, correct to 1 decimal place.

THINK WRITE

1 Draw a diagram and mark all given X


information on it.
45° 80 m
50 m
N
M
2 Write the formula with x2 as the subject. x2 = m2 + n2 − 2mn cos X
3 Substitute m = 80, n = 50 and X = 45°. = 802 + 502 − 2 × 80 × 50 × cos 45°
4 Calculate the value of x2. = 3243.15
5 Calculate x by taking the square root of x2. x = 3243.15
6 Give a written answer. = 56.9 m

WORKED Example 25
Two rowers set out from the same point. One rows N70°E for 2000 m and the other rows
S15°W for 1800 m. How far apart are the two rowers?
THINK WRITE
1 Draw a labelled diagram of the triangle, N A
2000 m
call it ABC and fill in the given
C 70°
information.
15°

1800 m

B
2 Write down the appropriate cosine rule to c = a2 + b2 − 2ab cos C
2
find side c.
3 Substitute the given values into the rule. = 20002 + 18002 − 2 × 2000 ×
1800 cos 125°
4 Evaluate. ≈ 11 369 750.342
c = 11 369 750.342
5 Round the answer to 2 decimal places. ≈ 3371.91
6 Give a written answer. The rowers are 3371.91 m apart.
218 M a t h s Q u e s t M a t h s B Ye a r 1 1 f o r Q u e e n s l a n d

remember
1. To use the cosine rule to find a side length, you need to be given the length of
two sides and the included angle.
2. The cosine rule formulas are:
a2 = b2 + c2 − 2bc cos A
b2 = a2 + c2 − 2ac cos B
c2 = a2 + b2 − 2ab cos C.
3. In the solution to cosine rule questions, your final answer is found by taking the
square root of the calculation.
4. Begin worded questions by drawing a diagram and finish them by giving a
written answer.

Using the cosine rule to find


4G side lengths
1 Write down the cosine rule formula as it applies to each of the triangles below. In
each case, make the boldfaced pronumeral the subject.
a A b P c n
L M
r q m
c b
l
Q R
B C p
a
N

WORKED 2 Find the third side of triangle ABC given a = 3.4, b = 7.8 and C = 80°.
Example
23 3 Find the length of the side marked with a pronumeral in each of the following, correct
to 3 significant figures.
a A b P c X
x
12 m r 13 cm 60°
C 12 m 12 m
35° 42°
B 14 m Q 21 cm R

Y x Z

4 In each of the following obtuse-angled triangles, find the length of the side marked
with the pronumeral, correct to 1 decimal place.
a X bA c R
114 cm b q
112 cm 110°
6.1 m 63 mm
Z 130° 160°
B C P
x 9.7 m 43 mm Q
Y

5 In triangle ABC, b = 64.5, c = 38.1 and A = 58°34′. Find a.


C h a p t e r 4 Tr i a n g l e t r i g o n o m e t r y 219
6 In triangle ABC, a = 17, c = 10 and B = 115°. Find b, and hence find A and C.
WORKED 7 Len and Morag walk separate paths that diverge from one another at an angle of 48°.
Example
24
After three hours Len has walked 7.9 km and Morag 8.6 km. Find the distance
between the two walkers at this time, correct to the nearest metre.
8 A cricketer is fielding 20 m from the batsman and at an angle of 35° to the pitch. The
batsman hits a ball 55 m and straight behind the bowler. How far must the fieldsman
run to field the ball? (Give your answer to the nearest metre.)

WORKED 9 Two rowers set out from the same point. One rows N30°E for 1500 m and the other
Example
25
rows S40°E for 1200 m. How far apart are the two rowers?
10 Ship A is 16.2 km from port on a bearing of 053° T and ship B is 31.6 km from the
same port on a bearing of 117° T. Calculate the distance between the two ships.
11 From the top of a vertical cliff 68 m high, an observer notices a yacht at sea. The
angle of depression to the yacht is 47°. The yacht sails directly away from the cliff,
and after 10 minutes the angle of depression is 15°. How fast does the yacht sail?

12 multiple choice
In a triangle ABC, a = 5, b = 6 and C = 105°. The length of c would be:
A 3.04 B 5.15 C 6.74 D 7.83 E 8.75

13 multiple choice
A plane flies on a bearing of 200° T for 58 km then on a bearing of 150° T for 42 km.
Its distance from the start is then:
A 44.68 km B 90.88 km C 96.68 km D 98.52 km E 100 km
14 The cord supporting a picture frame is 58 cm
long. It is hung over a single hook in the 58 cm
centre of the cord and the cord then makes an 145°
angle of 145° as shown in the figure on the
right. Calculate the length of the backing of
the picture frame, to the nearest centimetre. ?
220 M a t h s Q u e s t M a t h s B Ye a r 1 1 f o r Q u e e n s l a n d

Using the cosine rule to find angle sizes


We can use the cosine rule to find the size of the angles within a triangle. Consider the
cosine rule formula.
a2 = b2 + c2 − 2bc cos A
We now make cos A the subject of this formula.
a2 = b2 + c2 − 2bc cos A
a2 + 2bc cos A = b2 + c2
2bc cos A = b2 + c2 − a2
b2 + c2 – a2
cos A = ----------------------------
2bc
In this form, we can use the cosine rule to find the size of an angle if we are given all
three side lengths. We should be able to write the cosine rule in three forms depending
upon which angle we wish to find.

b2 + c2 – a2
cos A = ----------------------------
2bc
a2 + c2 – b2
cos B = ----------------------------
2ac
a2 + b2 – c2
cos C = ----------------------------
2ab

WORKED Example 26 A
Find the size of angle B in the triangle
on the right, correct to the nearest degree. 7 cm 5 cm

B C
9 cm

THINK WRITE
1 Write the formula with cos B as the a2 + c2 – b2
cos B = ----------------------------
subject. 2ac
9 + 72 – 52
2
2 Substitute a = 9, b = 5 and c = 7. cos B = ----------------------------
2×9×7
105
3 Calculate the value of cos B. cos B = ---------
126
= 0.8333
4 Make B the subject of the equation. B = cos−1(0.8333)
5 Calculate B. B = 34°

As we found earlier, the cosine ratio for an obtuse angle will be negative. So, when
we get a negative result to the calculation for the cosine ratio, this means that the angle
we are finding is obtuse. Your calculator will give the obtuse angle when we take the
inverse.
C h a p t e r 4 Tr i a n g l e t r i g o n o m e t r y 221
WORKED Example 27
Find the size of angle Q in the triangle Q
on the right, correct to the nearest 4 cm 3 cm
degree.
P R
6 cm
THINK WRITE
p2 + r 2 – q2
1 Write the formula with cos Q as the cos Q = ----------------------------
subject. 2 pr
32 + 42 – 62
2 Substitute p = 3, q = 6 and r = 4. cos Q = ----------------------------
2×4×3
– 11
3 Calculate the value of cos Q. cos Q = ---------
24
= −0.4583
4 Make Q the subject of the equation. Q = cos−1(−0.4583)
5 Calculate Q. Q = 117°

In some cosine rule questions, you need to work out which angle you need to find.
For example, you could be asked to calculate the size of the largest angle in a triangle.
To do this you do not need to calculate all three angles. The largest angle in any
triangle will be the one opposite the longest side. Similarly, the smallest angle will lie
opposite the shortest side.

WORKED Example 28 R
Find the size of the largest angle in
the triangle drawn on the right. 3.4 m 4.9 m

S
5.7 m T
THINK WRITE
1 ST is the longest side, therefore angle R
is the largest angle.
s2 + t 2 – r 2
2 Write the formula with cos R as the cos R = --------------------------
subject. 2st
4.9 2 + 3.4 2 – 5.7 2
3 Substitute r = 5.7, s = 4.9 and t = 3.4. cos R = ------------------------------------------
2 × 4.9 × 3.4
3.08
4 Calculate the value of cos R. cos R = -------------
33.32
= 0.0924
5 Make R the subject of the equation. R = cos−1(0.0924)
6 Calculate R. R = 85°
7 Give a written answer. The largest angle in the triangle is 85°.
222 M a t h s Q u e s t M a t h s B Ye a r 1 1 f o r Q u e e n s l a n d

As shown with angle questions earlier, these can be solved using your graphics
calculator.
Many problems that require you to find an angle are solved using the cosine rule. As
always, these begin with a diagram and are finished off by giving a written answer.

WORKED Example 29
Two paths diverge from point A. The first path goes for 1.25 km to point B. The second
path goes for 1.4 km to point C. B and C are exactly 2 km apart. Find the angle at which
the two paths diverge.
THINK WRITE
B
1 Draw a diagram.

1.25 km 2 km

A
1.4 km C
b2
+ – c2 a2
2 Write the formula with cos A as the cos A = ----------------------------
subject. 2bc

1.4 2 + 1.25 2 – 2 2
3 Substitute a = 2, b = 1.4 and c = 1.25. cos A = ----------------------------------------
2 × 1.4 × 1.25
– 0.4775
4 Calculate the value of cos A. cos A = -------------------
3.5
= −0.1364
5 Make A the subject of the equation. A = cos−1(−0.1364)
6 Calculate the value of A. = 98°
7 Give a written answer. The roads diverge at an angle of 98°.

remember
1. The cosine rule formulas are:
b2 + c2 – a2
cos A = ----------------------------
2bc
a2 + c2 – b2
cos B = ----------------------------
2ac
a2 + b2 – c2
cos C = ----------------------------
2ab
2. If the value of the cosine ratio is negative, the angle is obtuse.
3. In any triangle, the largest angle lies opposite the largest side and the smallest
angle lies opposite the smallest side.
4. Worded problems begin with a diagram and end with a written answer.
C h a p t e r 4 Tr i a n g l e t r i g o n o m e t r y 223
Using the cosine rule to find
4H angle sizes
1 For each of the following, write the cosine rule formula as it applies to the triangle
drawn with the boldfaced angle as the subject.

a A b P c
A

P M

B C R Q

WORKED 2 Find the size of the angle marked with the pronumeral in each of the following
Example
26
triangles, correct to the nearest degree.

a A b B c M
θ θ
8 cm 11 cm 3.2 m 2.8 m 4.5 m 5.4 m

C
B C N
θ O
13 cm A 4.0 m 6.2 m

WORKED 3 In each of the obtuse-angled triangles below find the size of the angle marked with the
Example
27
pronumeral, to the nearest degree.

a b c
θ
6m 8m 9.6 m θ
4.2 m 12.9 m
α 9.2 m
11 m 6.1 m

4.2 m

WORKED 4 In triangle ABC, a = 356, b = 207 and c = 296. Find the largest angle.
Example
28
5 Find the smallest angle in the triangle with sides 6 cm, 4 cm and 8 cm.

6 In triangle ABC, a = 23.6, b = 17.3 and c = 26.4. Find the size of all the angles.

7 Calculate the size of all three angles (correct to the nearest degree) in a triangle with
side lengths 12 cm, 14 cm and 17 cm.

WORKED 8 Two roads diverge from point P. The first road is 5 km long and leads to point Q. The
Example
29
second road is 8 km long and leads to point R. The distance between Q and R is
4.6 km. Calculate the angle at which the two roads diverge.
224 M a t h s Q u e s t M a t h s B Ye a r 1 1 f o r Q u e e n s l a n d

9 Mario cycles 12 km in a direction N68°W, then 7 km in a direction of N34°E.


a How far is he from his starting point?
b What is the bearing of the starting point from his finishing point?

10 A hockey goal is 3 m wide. When Sophie is 7 m from one post and 5.2 m from the
other, she shoots for goal. Within what angle, to the nearest degree, must the shot be
made if it is to score a goal?

11 An advertising balloon is attached to two ropes 120 m and 100 m long. The ropes are
anchored to level ground 35 m apart. How high can the balloon fly?

12 A plane flies in a direction of N70°E for 80 km, then on a bearing of S10°W for 150 km.
a How far is the plane from its starting point?
b What direction is the plane from its starting point?

13 A plane takes off at 10.00 am from an airfield, and flies at 120 km/h on a bearing of
N35°W. A second plane takes off at 10.05 am from the same airfield, and flies on a
bearing of S80°E at a speed of 90 km/h. How far apart are the planes at 10.25 am?
eBook plus
14 Three circles of radii 5 cm, 6 cm and 8 cm are pos- 5 cm
Digital doc: itioned so that they just touch one another. Their 6 cm
WorkSHEET 4.2
centres form the vertices of a triangle. Find the largest
angle in the triangle.

8 cm
C h a p t e r 4 Tr i a n g l e t r i g o n o m e t r y 225

summary
Trigonometry formulas for right-angled triangles
opp
• sin θ = ---------
hyp
adj
• cos θ = ---------
hyp
opp
• tan θ = ---------
adj
• SOHCAHTOA — this acronym will help you remember trig formulas.

Steps to find a side of a right-angled triangle


• Label the sides of the triangle opposite, adjacent and hypotenuse.
• Choose the correct ratio.
• Substitute given information.
• Make the unknown side the subject of the equation.
• Calculate.

Steps to find an angle in a right-angled triangle


• Label the sides of the triangle opposite, adjacent and hypotenuse.
• Choose the correct ratio.
• Substitute given information.
• Make the unknown angle the subject of the equation.
• Calculate by using the inverse trig functions.

Applications of right-angled triangles


• Angles of elevation and depression are each measured from the horizontal.
• The angle of elevation is equal to the angle of depression since they are alternate
‘Z’ angles.
• True bearings are measured in a clockwise direction, starting from north (000° T).

Sine rule
• The sine rule allows us to calculate sides and angles in non-right-angled triangles.
• When finding a side length you need to be given the length of one other side and
two angles.
• The sine rule formula when finding a side length in any triangle ABC is
a b c
------------ = ------------ = ------------- .
sin A sin B sin C
• When finding an angle you need to be given two side lengths and one angle.
sin A sin B sin C
• The sine rule formula when finding an angle is ------------ = ------------ = ------------- .
a b c

Cosine rule
• In any triangle ABC:
a2 = b2 + c2 − 2bc cos A
b2 = a2 + c2 − 2ac cos B
c2 = a2 + b2 − 2ab cos C
226 M a t h s Q u e s t M a t h s B Ye a r 1 1 f o r Q u e e n s l a n d

CHAPTER
review
1 Calculate each of the following, correct to 4 decimal places.
4A
a sin 46° b tan 76°42′ c 4.9 cos 56°
5.69 2.5
d 8.9 sin 67°3′ e ------------------ f ----------------------
cos 75° tan 9°55′

2 Calculate θ, correct to the nearest degree, given that:


4A
a cos θ = 0.5874 b tan θ = 1.23 c sin θ = 0.8.

3 Calculate θ, correct to the nearest minute, given that:


4A
a cos θ = 0.199 b tan θ = 0.5 c sin θ = 0.257.

4 Find the length of each side marked with a pronumeral, correct to 1 decimal place.
4B
a 6 cm b c
3.9 m m

22°
78° x
9° q 12.6 cm

d e f 6.8 m
12.6 cm 65°
32° q
22°
n t

7.8 cm

g g h i
138 mm
z 83°30'
h
2.9 m 4.8 cm
26°42'
77°18'

j 4.32 m k l
38.5 m
k 63 km
j 16°8'
85°12'
29°51'
m
C h a p t e r 4 Tr i a n g l e t r i g o n o m e t r y 227
5 A rope that is used to support a
flagpole makes an angle of 70° 4B
with the ground. If the rope is
tied down 3.1 m from the foot of
the flagpole, find the height of
the flagpole, correct to
1 decimal place.

6 A dirt track runs off a road at an


angle of 34° to the road. If I 4B
travel for 4.5 km along the dirt
track, what is the shortest
distance back to the road
(correct to 1 decimal place)?

7 A fire is burning in a building


and people need to be rescued. 4B
The fire brigade’s ladder must
reach a height of 60 m and must
be angled at 70° to the
horizontal. How long must the
ladder be to complete the
rescue?

8 Find the size of the angle


marked θ in each of the 4C
following, giving your answer
correct to the nearest degree.
a b 2.3 m c θ 116 cm
43 cm

16 m 19 m
4.6 m

θ θ

9 Find the size of the angle marked θ in each of the following, giving your answer correct to
the nearest minute. 4C
a 10.8 m b 2.9 m c
θ θ 11.9 cm
4.6 m θ
6.1 m 13.8 cm

10 A kite on an 80 m string reaches a height of 50 m in a strong wind. Calculate the angle the
string makes with the horizontal. 4C
228 M a t h s Q u e s t M a t h s B Ye a r 1 1 f o r Q u e e n s l a n d

11 There is 50 m of line on a fishing reel. When all the line is out, the bait sits on the bed of a
4C lake and has drifted 20 m from the boat. Calculate the angle that the fishing line makes with
the vertical.

12 Two buildings, 15 m and 27 m high, are directly opposite each other across a river. The
4D angle of depression of the top of the smaller building from the top of the taller one is 52°.
How wide is the river?
13 A four-wheel drive vehicle leaves a camp site
4D and travels across a flat, sandy plain in a
direction of S65°E, for a distance of 8.2 km.
It then heads due south for 6.7 km to reach a
waterhole.
a How far is the waterhole from the camp
site?
b What is the bearing of the waterhole from
the camp site?
14 From point A on level ground, the angle of
4D elevation to a plane is 72°. From point B on
the ground, due west of A, the angle of
elevation is 47°. If A and B are 3500 m apart,
find the height of the plane off the ground.
15 Use the sine rule to calculate each of the sides
4E marked with a pronumeral, correct to
3 significant figures.
a a
b c e
d
117° 19°
70° 31° 28° 4.6 km
4.6 cm
136 mm
20°
C h a p t e r 4 Tr i a n g l e t r i g o n o m e t r y 229
16 In ∆XYZ: x = 9.2 cm, ∠XYZ = 56° and ∠YXZ = 38°. Find y, correct to 1 decimal place.
4E
17 Use the sine rule to calculate the size of the angle marked with a pronumeral, correct to the
nearest degree. 4F
a b c
α
9.7 cm 7.1 m 9°
8 cm 9 cm

123° φ
63° θ 4.1 cm 1.2 m

18 Find all unknown sides and angles of triangle ABC, given a = 25 m, A = 120° and B = 50°.
4F
19 Use the cosine rule to find each of the following unknown sides, correct to 3 significant
figures. 4G
a b c
9m 6.9 cm
b 6.2 cm 128°
a 5.7 m
50° c
117°
11 m 4.6 m

20 In ∆LMN: LM = 63 cm, MN = 84 cm and ∠LMN = 68°. Find the length of LN, correct to
1 decimal place. 4G
21 During a stunt show two
aeroplanes fly side by side until 4G
they suddenly diverge at an
angle of 160°. After both planes
have flown 500 m what is the
distance between the planes,
correct to the nearest metre?

22 Use the cosine rule to find the size


of the angle in each of the following, 4H
correct to the nearest degree.

a b c 9 cm 7 cm
θ θ θ
4.2 m 5.3 m
6 cm
6 cm 15 cm
7.9 m

6 cm

23 In ∆XYZ: x = 8.3 m, y = 12.45 m and z = 7.2 m. Find ∠YZX, to the nearest degree.
4H
24 Two wooden fences are 50 m and 80 m long respectively. Their ends are connected by a
barbed wire fence 44 m long. Find the angle at which the two wooden fences meet. 4H
230 M a t h s Q u e s t M a t h s B Ye a r 1 1 f o r Q u e e n s l a n d

Modelling and problem solving


1 A soccer goal is 8 m wide.
a A player is directly in front of the goal
such that he is 12 m from each post.
Within what angle must he kick the
ball to score a goal?
b A second player takes an angled shot.
This player is 12 m from the nearest
post and 17 m from the far post. Within
what angle must this player kick to
score a goal?
2 An observer sights the top of a building at
an angle of elevation of 20°. From a point 30 m T
closer to the building, the angle of elevation is h
35° as shown in the figure on the right. 20° 35°
a Calculate the size of ∠ATB. A 30 m B C
b Show that the distance BT can be given by the
expression
30 sin 20°
BT = ------------------------- .
sin 15°
c Show that the height of the building can be given by the expression
30 sin 20° sin 35°
h = --------------------------------------------
sin 15°
d Calculate the height of the building correct to 1 decimal place.
3 Every car should carry a jack. One type of C
jack used to raise a car is a scissor-jack. A
simple diagram of a scissor-jack is given D
B M
at right. 120 mm
The threaded rod is rotated to increase or
decrease the length of the line segment BD.
a iii In ∆BCD, M is the midpoint of BD. A
What is the length of CM? AB = BC = CD = AD = 200 mm
ii If ∠BCD = 160°, what is the length
of BD, correct to the nearest millimetre?
iii What is the size of ∠MBC?
The jack is raised by reducing the length of the line segment BD.
b iii If the height of the jack, AC, is raised to 250 mm, what is the length of BD, correct to
the nearest millimetre?
ii If ∠MBC is 70°, what is the length of BD and what is the height of the jack?
4 Two fire-spotting towers are 17 kilometres apart on an east-west line. From one tower, fire
is seen on a bearing of 130° T. From the other tower, the same fire is spotted on a bearing of
eBook plus S20°W. Which tower is closest to the fire and how far is that tower from the fire?
Digital doc: 5 Two lighthouses are 20 km apart on a north–south line. The northern lighthouse spots a ship
Test Yourself
Chapter 4
on a bearing of S80°E. The southern lighthouse spots the same ship on a bearing of 040° T.
Find the distance from each lighthouse to the ship.
Graphing
periodic
functions 5
syllabus reference
Topic:
• Periodic functions and
applications

In this chapter
5A Period and amplitude of a
periodic function
5B Radian measure
5C Exact values
5D Symmetry
5E Trigonometric graphs
5F Applications
232 M a t h s Q u e s t M a t h s B Ye a r 1 1 f o r Q u e e n s l a n d

Introduction
Andrew is riding a ferris wheel
20 metres in diameter. It takes
2 minutes to complete one revol-
ution. Andrew wants to graph his
height above ground against time
and find a rule from which he
can calculate his height at any
time. The graph of height against
time will be an example of a
periodic function because it
repeats itself every 2 minutes.
In earlier chapters we have
looked at several different types
of functions. In this chapter we
will look at periodic functions —
functions that repeat themselves
over a period of time. Periodic
functions usually involve the
trigonometric ratios that we
began looking at in chapter 4.

Period and
amplitude of a periodic function
A function that repeats itself over a period of time is called a periodic function. The
figure below, showing the amount of rainfall at a certain weather station each month, is
an example of such a function, because the pattern of rainfall tends to repeat each year.
Note that all the previous function properties have been satisfied. The only new
aspect is the periodicity.
Rainfall (mm)

150
100
50

0 J F M A M J J A S O N D J F M A
Month

The highlighted region on the graph shows the section of curve that repeats itself.
There are many everyday occurrences — including the tides and daily temperatures —
that can be closely represented by a periodic function.
Chapter 5 Graphing periodic functions 233
Temperature and tide
Tides
1 Use the newspaper or other sources to find the time and depth of each high and
low tide in Moreton Bay over a period of 3 days.
2 Plot the points generated, showing time on the horizontal axis and depth on the
vertical axis. Join the points with a smooth curve.
3 Over what period of time does there appear to be a repetition of the graph?

Temperature
4 Use the newspaper or the Weather Bureau’s website to find the temperature in
Brisbane at 2-hourly intervals over a period of 3 days.
5 Plot the points generated, showing time on the horizontal axis and temperature
on the vertical axis. Join the points with a smooth curve.
6 Over what period of time does there appear to be a repetition of the graph?

The length of time that it takes for a


periodic function to begin repeating itself
is called the period, T, of the function.
Amy is a keen snorkeller who wants to
know the best time to go snorkelling. She
measures the depth of water in the inlet
every 4 hours for several days and plots
the results on the graph below.

10
Depth (m)

8
6
4
2

Time
0
on

on
am
am

pm
pm

id
am
am

pm
pm

id
am
m

m
no

no
4
8

4
8

4
4
8

4
8
12

12
12

12

Every 12 hours the graph repeats itself


and so we can say that 12 hours is the
period of this function. That is, T = 12 h.
From the graph it can be seen that
the maximum depth of water in the
inlet is 10 metres and the minimum
depth is 4 metres.
The equilibrium position is the centre
of the oscillation and so in this case
the equilibrium position is a depth of
7 metres. The maximum depth of water
is 3 metres above this equilibrium and the minimum depth of water is 3 metres
below equilibrium.
The amplitude is the maximum variation in the function from its equilibrium. We can
therefore say that this periodic function has an amplitude, A, of 3 metres. That is,
A = 3 m.
234 M a t h s Q u e s t M a t h s B Ye a r 1 1 f o r Q u e e n s l a n d

Generally the easiest way to calculate the amplitude is to halve the difference
between the maximum and minimum values.
By finding the point that lies halfway between the maximum and minimum values,
we are able to find this equilibrium position. In the preceding case, we can see that
halfway between the maximum value of 10 and the minimum value of 4 is the
equilibrium position of 7 metres.

WORKED Example 1 y
4
For the function drawn at right state: 3
a the period 2
b the amplitude. 1

0 1 2 3 4 5 6 7 8 9 10 11 12
x

THINK WRITE
a The function repeats itself every 4 units on a T = 4
the x-axis.
b 1 The maximum value on the y-axis is 4. b
2 The minimum value on the y-axis is 2.
4–2
3 Halve the difference between 2 and 4. Amplitude = ------------
2
Amplitude = 1

Once we have calculated the period of a function we can calculate the frequency, f,
1
using the formula f = --- . The frequency tells us how many repetitions of the function
T
occur in one unit of time.
For the case of a tidal function, where the period of the function is 12 hours ( 1--- day),
2
1
the frequency would be f = ---
T
1
= ------ 1
---
2
= 2.
This means that the function repeats twice every day.

WORKED Example 2
A periodic function has a period of 6.25 units. Calculate the frequency of the function.
THINK WRITE
1
1 Write the formula. f = ---
T
1
2 Substitute the value of T. f = ----------
6.25
3 Calculate. f = 0.16

In many practical examples the function may not be exactly the same each period
and so we make an estimate of both the period and amplitude.
Chapter 5 Graphing periodic functions 235
WORKED Example 3
The graph below shows the number of customers that pass through the checkouts of a
24-hour supermarket each day. State the period and amplitude of the function.

800
Number of customers

700
600
500
400
300
200
100
00
00
00
00
00
00
00
00
00
00
00
00
00
00
04
08
12
16
20
00
04
08
12
16
20
00

Time of day

THINK WRITE
1 The pattern appears to repeat every T = 24 hours
24 hours.

2 The maximum value is 600.

3 The minimum value is 40.


600 – 40
4 Halve the difference between 40 and 600. Amplitude = ---------------------
2
Amplitude = 280

remember
1. The period, T, of a function is the length of time (as shown on the horizontal
axis) that a function takes to begin to repeat itself.
2. The amplitude, A, of a periodic function is the maximum distance (as shown on
the vertical axis) that the function differs from its equilibrium position.
3. The amplitude is most easily found by halving the difference between the
maximum and minimum values of a periodic function.
4. The equilibrium position can be determined by finding the average of the
maximum and minimum values.
5. The frequency, f, tells us how many repetitions of the function occur in one unit
of time.
1
6. The frequency can be found using the formula f = --- .
T
7. The greater the period of the function, the smaller the frequency, and vice
versa.
236 M a t h s Q u e s t M a t h s B Ye a r 1 1 f o r Q u e e n s l a n d

Period and amplitude of a


5A periodic function

WORKED 1 For the function below, state:


Example
1
6
5
4
3
2
1

0 1 2 3 4 5 6 7 8 9 10

a the period
b the amplitude.

2 For each of the functions drawn below, state the period and the amplitude.
a y b y c y
2 1 1.5

0 x 0 x
2π 4π –π π
5— 0 3π x
4 4
–2 –1 –1.5

d y e y f y
4 2 3

0 π x x
3—
2 –π 0 π – –3π 0 π x
–4
–2 –3
g y h y
2.5 0.5

– –2π 0 π x 0 –π –π
x
6 2

–2.5 –0.5

WORKED 3 A periodic function has a period of 4 units. Calculate the frequency of the function.
Example
2

WORKED 4 The graph at right shows the number of people


Example 6
3
crossing an intersection during a 12-minute
5
period. For the function at right, state the 4
period and amplitude. 3
2
1

0 2 4 6 8 10 12
t
Chapter 5 Graphing periodic functions 237
5 The graph below shows the average humidity in
Brisbane each month of the year.

90

Humidity 80

70

60

50

40

0 J F MAM J J A S O ND J F MAM J J A S O NDJ

Months of the year


a Does the graph appear to be a periodic function?
b If so, what appears to be the period of the function?
c Which month appears to be the hottest month of
the year?
d Which month appears to be the coldest month of
the year?
e What is the amplitude of the function?

6 multiple choice
Which of the following graphs does NOT appear to represent a periodic function?
A B

C D

E All are periodic functions.


7 The graph at right shows the levels
Progesterone
of oestrogen and progesterone in
Oestrogen
a woman’s body over the 28 days
of her menstrual cycle.
a Which graph has the greater
amplitude?
b What is the frequency for the
functions? 1 2 3 4 5 6 7 8 9 10 11 12 13 14 15 16 17 18 19 20 21 22 23 24 25 26 27 28
238 M a t h s Q u e s t M a t h s B Ye a r 1 1 f o r Q u e e n s l a n d

Rhythm of life
Examine your pulse rate 4 times a day for a
week as you go about your regular routine.
Try to get someone else to take your pulse
at least once during the week while you
sleep.
1 Graph your pulse rate against the time of
day.
a Does the graph appear to be a
periodic function?
b What is the period of the function?
2 Find the maximum and minimum values
and, hence, the amplitude of the function.

Ferris wheeling
At the beginning of this chapter we looked at Andrew, who was riding the ferris wheel.
1 Given that the ferris wheel takes 2 minutes to complete one revolution, state the
period of the function.
2 What is the frequency of the function?

Radian measure
In earlier chapters we saw that both the linear and quadratic functions could be defined
algebraically. A linear function can be written in the form y = mx + c, while a quadratic
function takes the form y = ax2 + bx + c. A periodic function can also be related to an
algebraic expression. y
Before we investigate this expression, we need to explore
other areas. One such area is the concept of a radian.
1r
adi

You should be familiar with measuring angles in


us

degrees (°), and will recall that there are 360° in a full
1 radian
circle. An alternative unit for angle measurement is the x
radian.
Consider the circle drawn at right, which shows a
sector that cuts off an arc on the circle equal to the radius
of the circle.

Finding a radian
1 Draw a circle of radius 5 cm.
2 Cut off a piece of string 5 cm long and place it on the circumference of the circle.
3 Mark the end points of the arc formed by the strings and draw radii at these points.
4 Use a protractor to measure the angle between these radii.
Chapter 5 Graphing periodic functions 239
This angle is defined to be equal to 1 radian, which is written 1c. In many examples
no unit is given, in which case radians are assumed.
The circumference of a circle = 2π r units.
So the angle swept in one revolution is equivalent to 2π radians or 2π c.
Therefore, 2π c = 360°, so:
π c = 180°
180°
1c = ----------
c
-
π
c
π
1° = ----------- .
180°
These relationships are used to convert degrees to radians and vice versa. In most
cases an angle measured in radians will be given in terms of π. This is an exact radian
measure.

WORKED Example 4
Convert the following angles into exact radians.
a 120∞ b 0∞
THINK WRITE
c c
π π
a 1 Multiply 120 by ----------- . a 120° = 120° × -----------
180° 180°
Simplify by dividing through by the c
2 2π
highest common factor, 60. 120° = --------
3
c c
π π
b 1 Multiply 70 by ----------- . b 70° = 70° × -----------
180° 180°
c

2 Simplify by dividing through by the 70° = --------
highest common factor, 10. 18

WORKED Example 5
Convert the following angles to degrees.
c

a -------- b 0.8π c
6
THINK WRITE
c c c
5π 180° 5π 5π 180°
a 1 Multiply -------- by ----------
c
-. a -------- = -------- × -----------
c
6 π 6 6 π
5
2 Cancel out the π c. = --- × 180°
6
3 Simplify. = 150°
180° 180°
b 1 Multiply 0.8π c by ----------
c
-. b 0.8π c = 0.8πc × ---------- c
-
π π
2 Cancel out the π c. = 0.8 × 180°
3 Simplify. = 144°
240 M a t h s Q u e s t M a t h s B Ye a r 1 1 f o r Q u e e n s l a n d

When angles expressed in radians are not given in terms of π, they do not give a
whole number when converted to degrees. In such cases the angle will need to be
approximated.

WORKED Example 6
Use a calculator to make the following conversions.
a 57.2∞ to radians, correct to 3 decimal places
b 2.75c to degrees, correct to the nearest degree
THINK WRITE
c c
π π
a 1 Multiply 57.2 by ----------- . a 57.2° = 57.2° × -----------
180° 180°
2 Evaluate and round off to 3 decimal = 0.998c
places.
180° 180°
b 1 Multiply 2.75 by ----------
c
-. b 2.75c = 2.75c × ----------
c
-
π π
2 Evaluate and round off to the nearest = 158°
degree.

remember
1. An angular measure of 1 radian will cut off an arc equal to one radius on a
circle.
2. π c = 180° c
π
3. To convert an angle in degrees to radians, we multiply by ----------- .
180°
180°
4. To convert an angle in radians to degrees, we multiply by ----------
c
-.
π
5. If no unit is given for the angle then the angle is assumed to be in radians.

5B Radian measure
eBook plus

Digital docs:
WORKED 1 Convert the following angles into exact radians. SkillSHEET 5.1
Example
4
a 30° b 45° c 60° d 20° e 50° Converting degrees
to radians
f 90° g 270° h 360° i 150° j −225°
SkillSHEET 5.2
k −420° l 300° m 48° n 72° Converting radians
to degrees
WORKED 2 Convert the following angles to degrees.
Example
5 π 2π 2π 11 π e 3.6π
a --- b ------ c ------ d ---------
5 3 9 9
π
f – ---
π
g – --- h 0.375π
6 4
Chapter 5 Graphing periodic functions 241
3 multiple choice
The smallest angle measurement listed below is:
A 85° B π 2π 2π
C 1.92c D ------ E ------
3 5

4 multiple choice
The largest angle measurement listed below is:
A 3π 4π C 250°
B ------
3

D ------ E 3.84c
4

WORKED 5 Use a calculator to convert the following angles to


Example
6a
radians, correct to 3 decimal places.
a 49° b 78° c −125°
d 191° e 234° f 327°
g 170° h 217°

WORKED 6 Use a calculator to convert each of the following


Example
6b
radian measures to degrees. Give each answer cor-
rect to the nearest degree.
a 0.35 c b 1.47 c c 3.142 c
c c
d 0.958 e 15.6 f −2.71
g 4.06 c h 8c

Exact values
Calculated trigonometric values (sin, cos and tan) of most
angles are rational approximations, correct to several decimal
places.
However, for a few particular angles, exact trigonometric 1 2
values can be determined. Consider the isosceles triangle
drawn at right.
This triangle with equal sides of 1 unit has a hypotenuse 45°
of 2 units. We can therefore calculate the exact trigonometric 1
values for 45°.
So using SOH CAH TOA
1 1 1
sin 45° = ------- cos 45° = ------- tan 45° = ---
2 2 1
1 2 1 2
= ------- × ------- = ------- × ------- =1
2 2 2 2
2 2
= ------- = -------
2 2
242 M a t h s Q u e s t M a t h s B Ye a r 1 1 f o r Q u e e n s l a n d

To find the exact trigonometric values for 30° and 60° consider the equilateral tri-
angle with each side length 2 units. The triangle has been bisected down the middle
giving a perpendicular height of 3 units.

30°

2 3

60°
1
So using SOH CAH TOA
1 3 1
sin 30° = --- cos 30° = ------- tan 30° = -------
2 2 3
1 3
= ------- × -------
3 3
3
= -------
3
3 1 3
sin 60° = ------- cos 60° = --- tan 60° = -------
2 2 1
= 3
These results should be memorised for both degrees and radians. The triangles using
radian measures are shown in the figures below.

π

6
1 2 2
3
π
— π

4 3
1 1

The results are summarised in this table.

Angle Angle
(degrees) (radians) sin q cos q tan q

π
---
1
--- 3 3
30° ------- -------
6 2 2 3

π
--- 2 2
45° ------- ------- 1
4 2 2

π
--- 3 1
---
60° ------- 3
3 2 2
Chapter 5 Graphing periodic functions 243
WORKED Example 7
Without using a calculator, state the exact value of:
a sin 45° b cos 60° c tan 30°.

THINK WRITE
1
a Read the value from the table, or a sin 45° = -------
2
determine using the special 45°
2 = 1
------- × 2
-------
opp
triangle, sin θ = --------- . 1 2 2
hyp 45° 2
= -------
1 2
1
b Read the value from the table, or b cos 60° = ---
2
determine using the special
adj 2 30°
triangle, cos θ = --------- . 3
hyp
60°
1
1
c Read the value from the table, or c tan 30° = -------
3
determine using the special
1 3
opp = ------- × -------
triangle, tan θ = --------- . 3 3
adj 2 30° 3
3 = -------
3

60°
1

WORKED Example 8
Find the exact value of:
p p
a sin --- b tan --- .
3 6

THINK WRITE

π 3
a Read the exact value from a sin --- = -------
the table or use a special 3 2
triangle, π

6
opp
sin θ = --------- .
hyp 2
3

π–
3

1
Continued over page
244 M a t h s Q u e s t M a t h s B Ye a r 1 1 f o r Q u e e n s l a n d

THINK WRITE

π 1 3
b Read the exact value from b tan --- = ------- or -------
the table or use a special 6 3 3
triangle, π–
6
opp
tan θ = --------- .
adj 2
3

π

3
1

y
The unit circle
Trigonometric ratios can be considered using a unit circle. 2nd 1st
The coordinate axes divide the unit circle into four quadrant quadrant

quadrants as shown in the diagram at right. 3rd 4th x


quadrant quadrant

y
1
The position of point P can be described using the
P(θ )
coordinates x and y and the angle, θ. The point, P,
1
which is on the circle, is an angle of θ from the positive y
x-axis and can be expressed as P(θ ). θ
The x-coordinate of P is x = cosine θ. –1 Ox 1 x
The y-coordinate of P is y = sine θ.
These may be abbreviated: x = cos θ
y = sin θ –1
The point, P, can also be denoted as P (cos θ, sin θ). Note: −1 ≤ sin θ ≤ 1, for
all θ and −1 ≤ cos θ ≤ 1,
for all θ.
The ‘tangent line’ on a y y
unit circle is the line that is a
tangent to the circle at the
point (1, 0). Tangent θ (or
tan θ
tan θ for short) is the height θ
at which a line along an θ
x x
angle hits the tangent line.
tan θ
The diagrams at right show
tan θ for angles in the first
and second quadrants.

The tangent line


y sin θ
Note: Tangent θ (or tan θ) may also be defined as the ratio -- . That is, tan θ = ------------ .
x cos θ
Chapter 5 Graphing periodic functions 245
Therefore:
quadrant 1 is where 0° < θ < 90°
quadrant 2 is where 90° < θ < 180°
quadrant 3 is where 180° < θ < 270°
quadrant 4 is where 270° < θ < 360°

Since y = sin θ, sine is positive in quadrants 1 and 2 and


y
sine is negative in quadrants 3 and 4.
Since x = cos θ, cosine is positive in quadrants 1 and 4 and
cosine is negative in quadrants 2 and 3.
y Sine All
Since tan = -- , tangent is positive in quadrants 1 and 3
x and tangent is negative in quadrants 2
positive positive

and 4. x
Tangent Cosine
positive positive
The diagram summarises which of the trigonometric
functions is positive in each quadrant.
The word ‘CAST’ will assist in recalling this.
You may also use a short phrase, such as ‘All Stations To Central’.

WORKED Example 9
If a is any angle, find the quadrant where:
a sin a > 0 and tan a < 0 b sin a < 0 and cos a > 0.

THINK WRITE
a Sine is positive and tangent is negative in a Quadrant 2
quadrant 2.
b Sine is negative and cosine is positive in b Quadrant 4
quadrant 4.

Note that for angles which are greater than 360° we can find the quadrant by making
more than one revolution. The equivalent angle in the first revolution can be found by
repeatedly subtracting 360° until an angle between 0 and 360° is found. For example,
an angle of 560° is in quadrant 3 (560° − 360° = 200°).
For angles that are negative, we move in the
opposite (clockwise) direction. To find the
equivalent positive angle, add 360° to the negative
angle. For example, an angle of −210° will lie in
quadrant 2 (−210° + 360° = 150°).
Before we can consider angles within quadrants 90°
(0,1)
2, 3 and 4 we need to be able to calculate the
trigonometric ratios for the angles on the boundary
of each quadrant. Exact values of the boundary
angles can be found using the following figure and 180° (–1,0) (0,1) 0°/360°
the fact that:
y
sin θ = y, cos θ = x and tan θ = -- . (0,–1)
x 270°
246 M a t h s Q u e s t M a t h s B Ye a r 1 1 f o r Q u e e n s l a n d

As with the exact values studied earlier, we should know these results in both
degrees and radians. This figure can also be drawn in radians.

π

2
(0,1)

π 0/2π
(–1,0) (0,1)

(0,–1)
π
3—
2

These results are summarised in the table below.

Angle Angle
(degrees) (radians) sin q cos q tan q

0° 0 0 1 0

π
---
90° 1 0 undefined
2

180° π 0 −1 0


270° ------ −1 0 undefined
2

360° 2π 0 1 0

WORKED Example 10
Without using a calculator, find:
a sin 90° b cos 180° c tan 270° d sin 360°.

THINK WRITE

a 1 Sine corresponds to the y-coordinate on a


the unit circle.
2 The value of y at the point where sin 90° = 1
θ = 90° is 1.

b 1 Cosine corresponds to the x-coordinate b


on the unit circle.
2 The value of x at the point where cos 180° = −1
θ = 180° is −1.
Chapter 5 Graphing periodic functions 247

THINK WRITE
y
c 1 Tangent corresponds to the value of -- c
on the unit circle. x

–1
2 The value of y is −1 and the value of x tan 270° = ------
0
is 0 at the point where θ = 270°.
–1
3 The value of ------ is not defined. tan 270° is undefined
0

d 1 Sine corresponds to the y-coordinate on d


the unit circle.

2 The value of y where θ = 360° is 0. sin 360° = 0

remember
On a unit circle:
y
1. sin θ = y
2. cos θ = x tan θ
3. tan θ = height of point
where angle line meets
tangent line at RHS of θ
unit circle x
y
4. tan θ = -- .
x

Quadrants and angles


–π
2
90° Special triangles
y
1st quadrant
2nd quadrant P(θ )
2 π–
S A 1 6
sin θ
θ π– 45°
π , 180° 4
1
30°
x
cos θ 0, 360°, 2π
T C 2
3
3rd quadrant 4th quadrant

270°
3π π– 60°
— 3
2 1
248 M a t h s Q u e s t M a t h s B Ye a r 1 1 f o r Q u e e n s l a n d

5C Exact values
WORKED 1 Without using a calculator, state the exact value of:
Example
7
a sin 60° b cos 45° c tan 30° d cos 60° e sin 30°
f tan 45° g cos 30° h tan 60°.
WORKED 2 Without the use of a calculator, find the exact value of each of the following.
Example
8 π π π π π π π π
a sin --- b cos --- c tan --- d sin --- e tan --- f cos --- g cos --- h tan ---
4 3 3 6 4 6 4 6
eBook plus 3 Without using a calculator, state whether the following values are positive (P) or
negative (N).
Digital doc:
a sin 40° b sin 65° c sin 110° d sin 160° e sin 230°
EXCEL Spreadsheet
The unit circle f sin 260° g sin 215° h sin 321° i sin 425° j sin (−36°)
Verify your answers using a calculator.
4 Without using a calculator, state whether the following values are positive (P) or
negative (N).
a cos 27° b cos 68° c cos 115° d cos 200° e cos 250°
f cos 295° g cos 402° h cos (−83°) i cos (−240°) j cos 157°
Verify your answers using a calculator.
5 Without using a calculator, state whether the following values are positive (P) or
negative (N).
a tan 12° b tan 75° c tan 118° d tan 166° e tan 199°
f tan 255° g tan 308° h tan 500° i tan (−45°) j tan (−137°)
Verify your answers using a calculator.
WORKED 6 If a is any angle, find the quadrant where:
Example
9 a sin a < 0 and tan a > 0 b sin a > 0 and cos a > 0.

7 multiple choice
a If 0° < a < 360°, sin a < 0 and cos a < 0, which one of the following is true?
A 0° < a < 90° B 90° < a < 180° C 180° < a < 270°
D 270° < a < 360° E 0° < a < 180°
b If 0° < a < 360°, sin a > 0 and tan a < 0, which one of the following is true?
A 0° < a < 90° B cos a < 0 C 180° < a < 270°
D cos a > 0 E 270° < a < 360°
c Given that 0° < a < 360°, the equation cos a = −1 has:
A one solution B no solutions C two solutions
D three solutions E four solutions
d If 0° < a < 360°, the equation sin a = cos a has:
eBook plus A no solutions B two solutions C three solutions
D one solution E four solutions
Digital doc:
e If 0° ≤ a ≤ 180°, and sin a = cos a, then a is equal to:
WorkSHEET 5.1
A 60° B 150° C 45° D 0° E 90°
WORKED 8 Without using a calculator, find:
Example
10
a cos 180° b sin 270° c tan 360° d sin 180° e cos 270°
f cos 360° g tan 270° h tan 180° i sin 630° j cos 720°.
Chapter 5 Graphing periodic functions 249
Symmetry π

2
c

The unit circle can be divided into 1


symmetrical sections, as shown in the P(π – θ ) y P(θ ) = (cos θ , sin θ )
eBook plus = (x, y)
diagram on the right.
Digital doc: Relationships between the circular θ 0 or 2π c
EXCEL Spreadsheet π c –x
functions — sine, cosine and tangent — –1 x 1
The unit circle
can be established based on these
symmetrical properties. P(π + θ ) P(2π – θ )
–y
For simplicity, assume θ is an acute angle,
–1
although the following properties hold for any θ. c
π
3—
2

Quadrant 1
As already seen: sin θ = y
cos θ = x
y
tan θ = ---
x

Quadrant 2
By symmetry: sin (π − θ) = y = sin θ
cos (π − θ) = −x = −cos θ
y
tan (π − θ) = ------ = −tan θ
–x

Quadrant 3
By symmetry: sin (π + θ) = −y = −sin θ
cos (π + θ) = −x = −cos θ
–y
tan (π + θ) = − ------ = tan θ
x

Quadrant 4
By symmetry: sin (2π − θ) = −y = −sin θ
cos (2π − θ) = x = cos θ
–y
tan (2π − θ) = ------ = −tan θ
x
Notes
1. These relationships also apply if degrees are used in
place of radians — that is, if π is replaced by 180°
or 2π is replaced by 360°.
2. An angle measurement is assumed to be in radians
unless the degree symbol is given. S A

Provided an angle is expressed as π ± θ or 2π ± θ , T C


the trig function (sin or cos) remains the same,
only the sign (+ or −) may change.
Use this diagram to determine the sign.
250 M a t h s Q u e s t M a t h s B Ye a r 1 1 f o r Q u e e n s l a n d

WORKED Example 11
a If sin θ ° = 0.93, find sin (180 + θ )°. b If cos θ ° = 0.44, find cos (360 − θ )°.
c If tan θ = 1.72, find tan (π − θ ). d If cos θ = 0.83, find cos (π + θ ).
THINK WRITE
a 1 Sketch the angle on a unit circle and a y
relate it to the first quadrant.
S A
θ θ sinθ
180°
sin (180 + θ )° x
+ θ )° T C
(180

2 By symmetry sin (180 + θ)° = −sin θ. sin (180 + θ )° = −sin θ °


3 Replace sin θ ° with 0.93. = −0.93
b 1 Sketch the angle on a unit circle and b y
relate it to the first quadrant.

S A cosθ
θ
θ x
T C cos (360 –θ )°
(36
0–
θ )°

2 By symmetry cos (360 − θ)° = cos θ. cos (360 − θ )° = cos θ °


3 Replace cos θ ° with 0.44. = 0.44
c 1 Sketch the angle on a unit circle and c y
relate it to the first quadrant.
π–
θ S A
tan θ
π θ θ
x
T C tan (π – θ )

2 By symmetry tan (π − θ) = −tan θ. tan (π − θ ) = −tan θ


3 Replace tan θ with 1.72. = −1.72
d 1 Sketch the angle on a unit circle and d y
relate it to the first quadrant.
cos (π + θ ) S A
θ
θ cos θ x
π +θ
T C

2 By symmetry cos (π + θ) = −cos θ. cos (π + θ ) = −cos θ


3 Replace cos θ with 0.83. cos (π + θ ) = −0.83
Chapter 5 Graphing periodic functions 251
WORKED Example 12
Find, without using a calculator, the exact value of each of the following.
3p 7p
a tan 150° b sin 330° c cos ------ d tan ------
4 6

THINK WRITE
a 1 Express tan 150° as tan (180 − 30)°. a tan 150° = tan (180 − 30)°
2 Sketch the angle on a unit circle and relate y
it to the first quadrant. (18
0 –3
0)°
S A tan 30°
30°
x
T C
tan (180 – 30)°

3 By symmetry tan (180 − 30)° = −tan 30°. tan 150° = −tan 30°
1 3
1 3 = − ------
- or − -------
4 Replace tan 30° with its exact value, ------- or ------- . 3 3
3 3

b 1 Express sin 330° as sin (360 − 30)°. b sin 330° = sin (360 − 30)°
2 Sketch the angle on a unit circle and relate y
it to the first quadrant.

S A sin 30°
30°
x
sin (360 – 30)°
T C (36
0–
30

3 By symmetry sin (360 − 30)° = −sin 30°. sin 330° = −sin 30°
4 Replace sin 30° with its exact value, --1- . = − 1---
2 2

3π π 3π π
c 1 Express cos ------ as cos  π – --- . c cos ------ = cos  π – ---
4  4 4  4
2 Sketch the angle on a unit circle and relate y
it to the first quadrant.
π

π–
4

S A
π

4
cos π–4 x
T C
cos (π – π–4 )

Continued over page


252 M a t h s Q u e s t M a t h s B Ye a r 1 1 f o r Q u e e n s l a n d

THINK WRITE
π π 3π π
3 By symmetry cos  π – --- = −cos --- . cos ------ = −cos ---
 4 4 4 4
π 1 1 2
4 Replace cos --- with its exact value of ------- or = − ------
- or − -------
4 2 2 2
2
------- .
2

7π π 7π π
d 1 Express tan ------ as tan  π + --- . d tan ------ = tan  π + ---
6  6 6  6
2 Sketch the angle on a unit circle and relate it y
to the first quadrant.
π
S A tan –6 =
π–
6
tan (π + π–6 )
x
π– T C
π+
6

π π 7π π
3 By symmetry tan  π + --- = tan --- . tan ------ = tan ---
 6 6 6 6
π 1
------- 3 = 1
------- or 3
4 Replace tan --- with its exact value or ------- . -------
6 3 3 3 3

WORKED Example 13
3p
If sin ------ = 0.924, evaluate each of the following (without using a calculator).
8
5p 11 p
a sin ------ b sin ---------
8 8

THINK WRITE
5π  8-----π- – 3-----π- = sin  π – 3-----π- . 5π 3π
a 1 Express sin ------ as sin a sin ------ = sin  π – ------
8 8 8  8 8  8
y
π – 3—8π π
3—
8

S A

x
T C
Chapter 5 Graphing periodic functions 253
THINK WRITE

3π 3π 3π
2 Using symmetry express sin  π – ------ as sin ------ . = sin ------
 8 8 8

3 Replace sin ------ with 0.924. = 0.924
8

11 π  8-----π- + 3-----π- = sin  π + 3-----π- . 11 π 3π


b 1 Express sin --------- as sin b sin --------- = sin  π + ------
8 8 8  8 8  8
y
π
3—
8
S A

x
T C
π + 3—8π

3π 3π 3π
2 By symmetry express sin  π + ------ as −sin ------ . = −sin ------
 8 8 8

3 Replace sin ------ with 0.924. = −0.924
8

remember
1. A unit circle sketch of the given angle related back
to the first quadrant is often helpful.
2. Provided an angle is expressed as π ± θ or 2π ± θ,
S A
the trig function (sin or cos) remains the same,
only the sign (+ or −) may change.
3. Use this diagram to determine the sign. T C
3π 4π 6π 8π
4. π may be written as ------ , ------ , ------ , ------ and so on.
3 4 6 8

5D Symmetry
WORKED 1 If sin θ ° = 0.63, find:
Example
11
a sin (180 − θ )° b sin (180 + θ )° c sin (−θ )° d sin (360 − θ )°.

2 If cos θ = 0.25, find:


eBook plus
a cos (π − θ ) b cos (π + θ ) c cos (2π − θ ) d cos (−θ ).
Digital doc:
EXCEL Spreadsheet 3 If tan θ = 2.1, find:
The unit circle a tan (2π − θ ) b tan (−θ ) c tan (π + θ ) d tan (π − θ ).
254 M a t h s Q u e s t M a t h s B Ye a r 1 1 f o r Q u e e n s l a n d

4 Given that sin a° = 0.3, cos b° = 0.7 and tan c° = 0.9, write down the value of each of
the following.
a sin (180 + a)° b cos (180 − b)° c tan (360 − c)°
d sin (−a)° e sin (180 − a)° f cos (−b)°
g cos (360 − b)° h tan (180 − c)° i tan (180 + c)°
WORKED 5 Find, without using a calculator, the exact value of each of the following.
Example
12a, b
a sin 150° b cos 135° c tan 240° d tan 330°
e sin 240° f cos 210° g tan 120° h sin 300°
i cos (−60)° j sin (−135)° k tan 180° l sin 270°
WORKED 6 Find, without using a calculator, the exact value of each of the following.
Example
12c, d 5π 5π 7π 2π
a cos ------ b sin ------ c tan ------ d sin ------
4 6 4 3
7π π 4 π 7 π
e cos ------ f tan  – --- g sin ------ h cos ------
3  6 3 6
11 π 5π π
i tan --------- j sin  – ------ k cos π l sin  – ---
6  4  2

π π π
WORKED 7 If sin --- = 0.383, cos --- = 0.924 and tan --- = 0.414, evaluate each of the following
Example 8 8 8
13 (without using a calculator).
9π 7π 17 π
a sin ------ b cos ------ c tan ---------
8 8 8
π 7 π 15 π
d cos  – --- e sin ------ f tan ---------
 8 8 8
8 Given that sin 75° = 0.966, cos 75° = 0.259 and tan 75° = 3.732, find the value of each
of the following (without using a calculator).
a sin 105° b cos 255° c tan 285°
d sin 255° e cos 435° f tan (−75)°
9 If sin 0.7 = 0.644, cos 0.7 = 0.765 and tan 0.7 = 0.842, find the value of each of the
following, without using a calculator. (Hint: π = 3.142, approximately.)
a sin 2.442 b cos 3.842 c tan 5.584 d sin (−0.7)

Trigonometric graphs
The trigonometric functions are the most basic of all
periodic functions. Now that we can find the exact values
of the most important angles we are able to graph the
trigonometric functions.
Convention has it that trigonometric functions are usually
graphed with radians on the x-axis. Because we are now
working with the standard coordinate axes and the standard
form of a function is y = f(x) we use x rather than θ as our
variable.
Chapter 5 Graphing periodic functions 255
The graph of y = sin x
To get an idea of what the graph of y = sin x looks like, we may first construct a table
of values.

x −π − 3-----π- −π
--- −π
--- 0 π
---
π
---

------ π 5π
------

------

------ 2π
4 2 4 4 2 4 4 2 4
y = sin x 0 −0.71 −1 −0.71 0 0.71 1 0.71 0 −0.71 −1 −0.71 0

Next, these values are plotted on a set of coordinate axes and a smooth curve is
drawn to join the points. The range (the set of possible y-values for the functions) of
y = sin x is {y: −1 ≤ y ≤ 1}.

( –2π, 1)
1 (3—4π , 0.71) y = sin x
1–
2 ( π–4 , 0.71)
(–π , 0) (0, 0) (π , 0) (2π , 0)
0 π π 2π x
–π – –2π – –4π –π
4
–π
2
3—
2
– 1–2 ( 5—4π , ( 7—4π , –0.71)
( – 3—4π , –0.71) –0.71)
(– –2π , –1) –1
( 3—2π , –1)

It can be observed that the curve repeats itself in cycles after an interval of 2π units.
Due to this repetition it is called a periodic function and the period is the interval
between repetitions. The period of y = sin x is 2π radians (or 360°).
When dealing with graphs of periodic functions
such as sin x, we assume that the units for x are
radians unless otherwise stated. This is because on
the unit circle from which all trigonometric ratios
are developed, the arc length corresponds to the
angle at the centre of the circle in radians. 1c
The mean position of the curve is y = 0 and the
maximum and minimum values are 1 and −1
1 unit
respectively. The distance from the mean position
to the maximum (or minimum) position is called
the amplitude of the periodic function. The ampli-
tude of y = sin x is 1 unit.

Period

Amplitude

x
Mean
position

Period
256 M a t h s Q u e s t M a t h s B Ye a r 1 1 f o r Q u e e n s l a n d

Graphics Calculator tip! The graph of y = sin x


The following steps can be used to draw the graph of y = sin x and explore the key
points on the graph.
For the Casio fx-9860G AU
1. Press MENU and then select GRAPH .

2. Before continuing you will need to check that the


calculator is in radian mode. To do this, press
SHIFT [SET UP] and then arrow down to Angle.
Press F2 (Rad) to change to radian mode.

3. Enter sin x for Y1 and then press EXE .

4. We need to set up the View Window, so press


SHIFT F3 (V-WIN) and then adjust the settings
so that Xmin is 0, Xmax is 2π, Ymin is −1.2 and
Ymax is 1.2. In both cases, use a scale of 1.

5. Press EXIT to return to the previous screen and


then press F6 (DRAW) to draw the graph.

6. To find the amplitude, we need to find the


maximum distance of the graph from its mean
position of y = 0. To do this, press SHIFT F5
(G-SLV) and then press F2 (MAX). The y-value
shown is the amplitude of the graph.

7. The period can be explored by finding the


x-intercepts. To do this, press SHIFT F5 (G-SLV)
and then press F1 (Root). Press the right arrow
key to move to the third x-intercept shown, which is
where the graph has completed one cycle. Note that
the x-value is actually equal to 2π.
Chapter 5 Graphing periodic functions 257
For the TI-Nspire CAS
1. Before beginning you will need to check that the
calculator is in radian mode. To do this, go to the
home screen (press c) and then select
8: System Info followed by 2: System Settings.
Press the e key until Angle is highlighted. Press
the down arrow on the NavPad to display the
Angle menu and then use the NavPad to select
Radian.

2. Press · to accept this. Continue to press the e


key until OK is highlighted and then press ·.
Open a new Graphs & Geometry document
(press /N and then select 2: Add Graphs &
Geometry). Adjust the Window Settings (press
b and then select 4: Window followed by
1: Window Settings) so that XMin is 0, XMax is
10, YMin is −1.2 and YMax is 1.2.

3. Press e until OK is highlighted and then press


·. Complete the function entry line for f1(x)
with sin (x) and then press ·. To see more of the
curve, press / G to hide the function entry line.
(Press / G again to bring the function entry
line back.)

4. The Trace tool can be used to identify key points


on the graph. Press b and then select 5: Trace.

5. Select 1: Graph Trace. To find the amplitude,


press the right arrow on the NavPad to move the
trace point to the maximum point on the graph.
An ‘M’ will appear in a box on the screen when
the maximum point is reached. The y-value gives
the amplitude, which we can see is 1.

6. The period can be explored by looking at the


x-intercepts. Continue to trace to a point where the
graph has completed one cycle. From the origin,
this is at the second x-intercept shown. A ‘z’
appears in a box on the screen when an x-intercept
is reached. Note that the x-value is actually equal
to 2π.
258 M a t h s Q u e s t M a t h s B Ye a r 1 1 f o r Q u e e n s l a n d

The graph of y = cos x


As for the graph of y = sin x, the graph of y = cos x can be established by first
completing a table of values.

x −π − 3-----π- −π
--- −π
--- 0 π
---
π
---

------ π 5π
------

------

------ 2π
4 2 4 4 2 4 4 2 4

y = cos x −1 −0.71 0 0.71 1 0.71 0 −0.71 −1 −0.71 0 0.71 0

When these points are plotted y


on a set of axes and joined with a y = cos x
smooth curve, the graph looks like 1 (0, 1) (2π, 1)
this: (– π–4 , 0.71) 1– ( π–4 , 0.71) ( 7—4π , 0.71)
2
Verify the shape of this curve (– π–2 , 0) ( –2π, 0) ( 3—2π , 0)
using a graphics calculator as 0 x
–π –π–2 – –4π –π –π π 3—π 2π
shown with y = sin x. 4 2 2
(– 3—4π , –0.71) 1
– –2 ( 5—4π , –0.71)
It can be seen that the basic –1 ( 3—4π , –0.71)
shape of the curve y = cos x is the (–π, –1) (π, –1)
same as that of y = sin x. It is also
a periodic function with:
period = 2π
amplitude = 1.
π
Note: The graph of y = cos x is exactly the same as that of y = sin x translated ---
radians or 90° to the left. 2

The graph of y = tan x


Consider the unit circle with an angle of θ at the centre. 1
From this circle we can see that sin θ = y, cos θ = x and y

y sin θ
tan θ = -- . From this we are able to conclude that tan θ = ------------ . θ
x cos θ –1 x 1
We can use this information to generate the table of values
below, or we can use our knowledge of special angles.
–1

x −π − 3-----π- −π
--- −π
--- 0 π
---
π
---

------ π 5π
------

------

------ 2π
4 2 4 4 2 4 4 2 4

y = tan x 0 1 undefined −1 0 1 undefined −1 0 1 undefined −1 0

π
(Note: Multiples of --- could be used to give more points to plot and a clearer indication
8
of the shape of the graph.)
sin x
Note the presence of some undefined y-values. This is because tan x = ------------ and
cos x = 0 at these values. cos x
These undefined values are shown as vertical asymptotes through the given value of
x for which they occur. (An asymptote is a line that a graph approaches, but never quite
reaches. In the case of y = tan x, y approaches, but never actually reaches, −∞ and +∞
for particular x-values.)
The graph of y = tan x is shown in the following figure.
Verify this graph using a graphics calculator as shown with y = sin x.
Chapter 5 Graphing periodic functions 259
The features of the graph of Vertical asymptotes
y = tan x are:
1. It has vertical asymptotes at y
π π 3π y = tan x
x = . . . − --- , --- , ------ , . . .
2 2 2
3π 5π
2. It has no amplitude. (– —4
, 1) 1 ( π–4 , 1) ( —4 , 1)
3. It has a period of π. (–π , 0) (0, 0) (π , 0) (2 π , 0)
–π π π 0 π π π π
3— 2π x
4. It has a range of R (the set of all Real – –2 – –4 – –
4 2 2
numbers). –1 (3—4π, –1) (7—4π, –1)

Further trigonometric graphs


From the basic sketch of y = sin x and y = cos x we need to be able to sketch a graph
that is in the form y = A sin Bx.

The effect of 2
The following investigation is best completed using a graphics calculator but can
also be done on paper.
1 On the same set of axes sketch the graphs of y = sin x and y = 2 sin x.
For the Casio fx-9860G AU
1. Press MENU and then select GRAPH .
2. Enter sin x for Y1 and 2 sin x for Y2.

3. You can change the line style of either


function by pressing F4 (STYL) and
then choosing a different line style. In
this case we have chosen the thick black
line for Y2 by pressing F2 .

4. Press EXIT and then reset the View


Window by pressing SHIFT F3
(V-WIN). Enter a minimum value of –2π
and a maximum of 2π for x and a
minimum value of –2 and a maximum
value of 2 for y.
5. Again press EXIT to return to the
previous screen and then press
F6 (DRAW) to display the graphs.
260 M a t h s Q u e s t M a t h s B Ye a r 1 1 f o r Q u e e n s l a n d

For the TI-Nspire CAS


1. Open a new Graphs & Geometry
document. Adjust the Window Settings
(press b and then select 4: Window
followed by 1: Window Settings) so that
XMin is −2π, XMax is 2π, YMin is −2 and
YMax is 2.

2. Press e until OK is highlighted and


then press ·. Complete the function
entry line for f1(x) with sin (x) and then
press ·.

3. Complete the function entry line for f2(x)


with 2 sin (x) and then press ·. To
see more of the graph, press / G to
hide the function entry line. (To return
to the function entry line, press / G
again.)

2 What has been the effect of the 2 on the graph of y = 2 sin x?


3 Now, on the same axes draw the graphs of y = sin x and y = sin 2x.
4 What has been the effect of the 2 on the graph of y = sin 2x?

From the above investigation we can see that the effect of A on the graph of
y = A sin Bx is to multiply the amplitude of y = sin x by A. The period of the function is
not affected by the value of A; that is, the period of y = A sin x remains 2π.
The amplitude of any periodic function is taken to be positive. We can say that the
amplitude of y = A sin Bx is equal to A .
The effect of B is to divide the period of y = sin x by B. The amplitude of the func-
tion is not affected by the value of B; that is, the amplitude of y = sin Bx remains 1.
We can therefore sketch the graph of y = A sin Bx by using y = sin x as our basic

model, but with an amplitude of A and a period of ------ .
B

360°
When x is given in degrees, the period is ----------- but the amplitude is unchanged.
B
The same results apply for graphs of the form y = A cos Bx.
Next year we will extend this further to look at graphs of the form y = A sin B(x + C) + D.
Chapter 5 Graphing periodic functions 261
WORKED Example 14
State i the period and ii the amplitude of each of the following functions.
x
a y b y = −1.5 sin 4x c y = 6 cos ---
4 3

0 π
3— 3π x
2

–4

THINK WRITE
a 1 From the graph the cycle repeats after 3π units. a ii Period = 3π
2 From the graph, the distance from the mean ii Amplitude = 4
position to the maximum position is 4 units.

b 1 State the function being considered. b y = −1.5 sin 4x



2 Write the formula for the period of ii Period = ------ where B = 4
B
y = A sin Bx.

3 Substitute B = 4. ii Period = ------
4
π
4 Simplify. ii Period = ---
2
5 By rule, the amplitude is | A | or ii Amplitude = | −1.5 |
| −1.5 |. ii Amplitude = 1.5

x
c 1 State the function being considered. c y = 4 cos ---
3
2π 1
2 Write the formula for the period. ii Period = ------ where B = ---
B 3


3 Substitute B = 1--- . ii Period = -----
1
-
3 ---
3

4 Simplify. ii Period = 6π
5 By rule, the amplitude is 6. ii Amplitude = 6

eBook plus Once we have determined both the period and amplitude of the periodic function, we
are able to sketch the function using either y = sin x or y = cos x as a model.
Digital docs: The shape of each function remains unchanged with only the amplitude and period
EXCEL Spreadsheet
Sine graphs being adjusted. The pattern of the periodic function must be continued over the domain
EXCEL Spreadsheet for which we are asked to sketch the function.
Cosine graphs Note that if A is negative, the shape of the graph is reflected in the x-axis.
Log into www.jacplus.com.au and locate the weblinks for this chapter to access tech-
nology files which can be used to show the graphs of sine and cosine functions.
262 M a t h s Q u e s t M a t h s B Ye a r 1 1 f o r Q u e e n s l a n d

WORKED Example 15
Sketch the graphs of the following functions and state i the period and ii the amplitude of
each.
a y = 4 cos 3x for –360∞ ≤ x ≤ 360∞ b y = − 2--3- sin 4x for –2π ≤ x ≤ 2π

THINK WRITE

a 1 State the function being considered. a y = 4 cos 3x


360°
2 Write the formula for the period. ii Period = -----------
B

Substitute B = 1--- . 360°


3
3
= -----------
3
4 Simplify the value of the period. = 120°
5 The amplitude is the value in front ii Amplitude = 4
of cos, written as a positive value.
6 Draw a set of axes.
7 Since the amplitude is 4, mark or
y
imagine horizontal guidelines at y = 4 cos 3x
y = −4 and 4. 4
8 Sketch one cycle of the graph every
period (every 120°) along the x-axis,
for –360° ≤ x ≤ 360°, showing key –360° –240° –120° 0 120° 240° 360°
x
x-values.
–4

b 1 State the function being considered. b y = − 2--- sin 4x


3

2 Write the formula for the period. ii Period = ------
B

3 Substitute B = 4. ii Period = ------
4
π
4 Simplify the value of the period. ii Period = ---
2
2
5 The amplitude is the value in front of ii Amplitude = ---
3
sin, written as a positive value.
6 Draw a set of axes.
7 Since the amplitude is 2--- , mark or y
3 y = – –32 sin 4x
imagine horizontal guidelines at 2–
y = − 2--- and 2--- . 3
3 3
8 Sketch one cycle of the graph every
π –2π –— –π – π– 0 2π x
period (every --- ) along the x-axis, 3π π
– π 3π

2 2 2 2 2

for –2π ≤ x ≤ 2π. – 2–3


9 Determine the x-intercepts and mark
these on the graph.
Chapter 5 Graphing periodic functions 263
WORKED Example 16
1
Sketch the graph of f(x) = 2 cos --- x in the domain -2π ≤ x ≤ 2π.
2
THINK WRITE

1
1 State the function being considered. f(x) = 2 cos --- x
2
2π 2π
2 The period of the function is ------ , Period = ------
1 B 1
where B = --- . ---
2 2

3 Simplify the value of the period. = 4π


4 The amplitude is 2. Amplitude = 2
5 Draw a set of axes. y
y = 2 cos 1–2 x
6 Mark the x-axis from −2π to 2π (the
specified domain). 2
7 Show −2 and 2 on the y-axis, since the
amplitude is 2.
–2π –π 0 π 2π x

–2

8 Visualise, or check, the general shape


of the graph of y = A cos Bx.
9 Starting from the point (0, 2) complete a
half cycle of the cosine function forward
to (2π, –2).
10 Complete half of a cycle back from
(0, 2) to (−2π, −2).
11 The x-intercepts are halfway between
the maximum and minimum points.

Verify that this graph is correct by using a graphics calculator.

The graph of y = A tan Bx can be drawn using a similar technique. The period will be
eBook plus
π
--- and although it has no amplitude, the value of A will cause the graph to increase
Digital doc: B
EXCEL Spreadsheet more rapidly. Experiment with the graph of y = A tan Bx on a graphics calculator,
Tangent graphs
although at this stage you do not need to sketch graphs of this type yourself. Notice
what happens to the x-and y-intercepts and the position of the asymptotes as A and B
are varied.
Alternatively, you may like to investigate the graph of y = A tan Bx online by logging
into www.jacplus.com.au and locating the weblinks for this chapter.
264 M a t h s Q u e s t M a t h s B Ye a r 1 1 f o r Q u e e n s l a n d

remember
1. Basic graph types
(a) y = A sin Bx y = A cos Bx

(b) Period = ------ , amplitude = | A |
y B y
y = A sin Bx y = A cos Bx
A A

0 –π
x 0 π x
–π π π
π
2— 3— 2—
B B = period —
2B B 2B B

–A –A
2. Sketching
(a) Recall basic graph type.
(b) Find period and amplitude.
(c) Sketch in sections of one period.
(d) Find x-intercepts between other known intercepts or minimum and
maximum points.
(e) If A < 0 the shape of the graph is reflected in the x-axis.

5E Trigonometric graphs
WORKED 1 State i the period and ii the amplitude for each of the following functions:
Example
a y b y c y
14a
2 1 1.5

0 x x
2π 4π 0
–π π
5— 0 3π x
4 4
–2 –1 –1.5
d y e y f y
4 2 3
eBook plus

Digital docs: 0 π x x
3—
2 –π 0 π – –3π 0 π x
EXCEL Spreadsheet
Sine graphs –4
EXCEL Spreadsheet
–2 –3
Cosine graphs
g y h y
2.5 0.5

– –2π 0 π x 0 –π –π
x
6 2

–2.5 –0.5
WORKED 2 State i the period and ii the amplitude of each of the following functions.
Example
14b, c a y = sin x b y = 3 sin x c y = 2 sin 2x
1 x
d y = −4 sin 3x e y = --- sin --- f y = 2 cos 4x
2 3
Chapter 5 Graphing periodic functions 265
x πx
g y = 0.4 cos --- h y = −3 cos 5x i
y = 2.5 cos ------
3 4
π x
cos ------
1 2
j y = −sin 6x k y = --- sin π x l y = ----------------
5 4
WORKED 3 Sketch the graph of the following functions for –2π ≤ x ≤ 2π and state i the period and
Example
15
ii the amplitude of each. Check your answers using a graphics calculator.
x
a y = sin 2x b y = 2 cos x c y = 3 sin ---
2
x
d y = 4 cos 2x e y = --1- sin 3x f y = 2--- cos ---
2 3 2
x x
g y = 5 sin --- h y = −4 cos --- i y = 2 cos 4x
3 2

4 multiple choice
Parts a to c refer to the graph. y
a The amplitude of the function is:
π 3
A --- B −3 C π
2
D 3 E 6
0 –π
x
b The period of the function is: 2

π –3
A 2π B --- C π
2
D 6 E 3
c The equation of the function could be:
A y = 3 sin 2x B y = 3 sin x C y = 3 cos 2x
x
D y = −3 sin 2x E y = 3 sin ---
2

5 multiple choice y
The equation of this curve could be: 2
x
A y = 2 cos x B y = 2 sin ---
2
x
C y = −2 cos x D y = cos --- 0 π 2π 3π x
2
x
E y = −2 cos --
- –2
2
6 State the equation of each of the functions graphed below.
a y b y
1.5
2

0 3π x
–1.5 0 x
– π–4 π

4


4

–2
266 M a t h s Q u e s t M a t h s B Ye a r 1 1 f o r Q u e e n s l a n d

c y d y

5 4

–2π 0 2π 4π x 0 3π

x
2

–5 –4

e y f y
1
3

0 2π



x
3 3
– π– 0 π– x
–1 6 6

–3

7 Sketch the graph of each of the following functions for –360° ≤ x ≤ 360°.
x
a f(x) = cos 2x b f(x) = 3 cos --- c f(x) = 2 sin 3x
2
8 Sketch the graph of each of the following functions for –2π ≤ x ≤ 2π.
a f(x) = −3 cos 2x b f(x) = 1.5 sin --x- c f(x) = −4 sin 4x
3
Check your graphs using a graphics calculator.

WORKED 9 Sketch the graphs of each of the following functions for –2π ≤ x ≤ 2π.
Example
16
Check your graphs using a graphics calculator.
2x x
a f(x) = 3
--- sin ------ b f(x) = 1.8 cos ---
2 3 3
πx 3x
d f(x) = −3 cos ------
c f(x) = −1.4 sin ------
2 4
10 For each of the functions graphed below, state the rule using full function notation.
a y b y c y
f(x) 2
3 5–
2
f(x) f(x)
0 π x 0 5π x
8— 0 x
6 – 5–2 –1 1
–3
–2
d y e y f y
1.8 3 2.4
f(x)
f(x) f(x)
0 x 0 x – 1–3 0 x
–1 3 3 1
eBook plus
–1.8 –3 –2.4
Digital doc:
WorkSHEET 5.2
11 Sketch the graph of y = tan x for the domain –2π ≤ x ≤ 2π.
Chapter 5 Graphing periodic functions 267
How high?
We are now ready to try and draw the
graph of Andrew’s height above ground
level as he rides the ferris wheel.
1 What is the maximum and minimum
height of the ferris wheel?
2 Draw a set of axes allowing us to
follow the path of the ferris wheel for
3 revolutions (6 minutes) on the x-axis
and allowing for the minimum and
maximum height on the y-axis.
3 Assuming that the ferris wheel revolves
at a constant rate, complete the table of
values below, then plot the points
generated on your axes.

Time (min) 0 1 2 3 4 5 6
Height (m)

4 If the wheel is revolving at a constant


rate, what will be the height of the wheel
after 1--2- minute. Plot the point generated
on your graph. Similarly, plot the points
1 1
generated after 1 --2- minutes, 2 --2- minutes,
3 1--2- minutes, 4 1--2- minutes and 5 1--2- minutes.
5 We now need to consider some of the
intermediate points. After 15 seconds
the wheel will have passed through an
π
angle of --- radians. The height of the
4
ferris wheel can be found by finding
h in the figure below. (Note: h = 10 − x.) Plot this point on your axes.

x 10 m

10 – x h

6 Draw a diagram and use the same method to find the height of the wheel after
10 seconds and after 20 seconds. Plot the points generated on your graph.
7 You should now be able to plot the height of the wheel after 40, 45, 50, 70, 75,
80, 100, 105 and 110 seconds to complete the first revolution.
8 Join the points plotted with a smooth curve and repeat the function for the
second and third revolutions of the ferris wheel.
9 Use your knowledge of trigonometric graphs to state the function of height
against time.
268 M a t h s Q u e s t M a t h s B Ye a r 1 1 f o r Q u e e n s l a n d

Applications
Many situations arise in science and nature where relationships between two variables
exhibit periodic behaviour. Tide heights, sound waves, biorhythms and ovulation cycles
are examples.
In these situations trigonometric functions can be used to model the behaviour of the
variables. The independent variable, x, is often a measurement such as time. When
modelling with trigonometric functions you should work in radians unless otherwise
instructed.

WORKED Example 17
A river flows through Tony’s property. d
Tony measures the water level by 5
measuring the distance above or below 4
a marker in the water. Tony lets d be 3
the level of water above or below the 2
marker and t is the number of hours
1
since midnight. Tony’s results are
shown in the graph at right. 0 2 4 6 8 10 12 14 16 t
–1
Express d as a function of t.
–2

THINK WRITE

1 The graph is in the basic shape of a sine The graph is of the form y = A sin Bx.
graph.
2 The amplitude of the graph is 2. Amp = 2: y = 2 sin Bx

3 The period of the function is 4. Put this ------ = 4
equal to the period and solve for B. B

2π = 4B

B = ------
4
π
= ---
2
π
4 Write the equation. y = 2 sin --- x
2

Having found the equation, we are able to then find other information about the
particular function.
Chapter 5 Graphing periodic functions 269
WORKED Example 18
E. coli is a type of bacterium. Its concentration,
P parts per million (ppm), at a particular
beach over a 12-hour period t hours after 6 am,
is described by the function:
pt
P = 0.05 sin ------ + 0.1.
12
a Find the i maximum and ii minimum E. coli
levels at this beach.
b What is the level at 3 pm?

THINK WRITE
πt
a Write the function. a P = 0.05 sin ------ + 0.1
12
πt
ii The maximum value of the sine
1 ii The maximum P occurs when sin ------ = 1.
function is 1. 12
πt
2 Substitute sin ------ = 1 into the Max. P = 0.05(1) + 0.1
12 = 0.15
equation for P and evaluate.
3 State the solution. The maximum E. coli level is 0.15 ppm.
ii 1 The minimum value of the sine πt
ii The minimum P occurs when sin ------ = −1.
function is −1. 12
πt
2 Substitute sin ------ = −1 into the Min. P = 0.05(−1) + 0.1
12 Min. P = 0.05
equation for P and evaluate.
3 State the solution. The minimum E. coli level is 0.05 ppm.
b 1 At 3 pm it is 9 hours since 6 am. b At 3 pm, t = 9.

2 Substitute t = 9 into the equation When t = 9, P = 0.05 sin ------ + 0.1
for P, and evaluate. 12

= 0.05 sin ------ + 0.1
4
1
= 0.05 ------- + 0.1
2
= 0.035 + 0.1
= 0.135
3 State the solution. The E. coli level at 3 pm is approximately
0.135 ppm.

remember
1. General equations: y = A sin Bx + C, y = A cos Bx + C

2. Period = ------ , amplitude = | A |.
B
3. To find maximum value of a function, replace sin x or cos x with +1.
4. To find minimum value of a function, replace sin x or cos x with −1.
5. Initial values occur at t = 0.
6. Sketch the graph for greater understanding.
270 M a t h s Q u e s t M a t h s B Ye a r 1 1 f o r Q u e e n s l a n d

5F Applications
W (kg)
WORKED 1 The weight of a rabbit over a period of 4
Example
17
time is modelled by the graph. 3
a State i the amplitude and ii the period. 2
b Express W as a function of t. 1
eBook plus
0 t (days)
3 6
Digital docs:
SkillSHEET 5.3 2 The diagram shows the heart rate of an athlete H (beats/min)
Period and amplitude of
sine and cosine graphs during a particular hour of a workout.
160
EXCEL Spreadsheet a Find the initial heart rate.
Trigonometric equations b State i the amplitude and ii the period. 110
c Express H as a function of t. 60

0 t (min)
15 30 45 60
WORKED 3 The height above the ground, h metres, of a
Example
18
child on a swing at any time t seconds after
being released is:
πt
h = 1 + 0.6 cos ----- .
Find: 2
a the maximum height of the swing
b the height after
i 3 seconds and ii 4--- seconds.
3
4 The temperature, T (°C), inside a building on a
given day is given by the function:
πt
T = 8 sin ------ + 18
12
where t is the number of hours after 8 am.
a What is the maximum temperature in the
building and the time at which it first occurs?
b Find the temperature at
i 8 pm, ii 6 pm, iii 12 am (midnight).
5 The displacement, x (in mm), of a harp string t seconds after it is initially plucked is
modelled by the function:
x(t) = 12 sin 20π t.
a What is the i amplitude and ii period of this function?
b How many vibrations (that is, cycles) will it complete in one second?
c Find the displacement after 0.08 seconds.
6 The height of a bungee jumper, h metres, above a pool of water at any time t seconds
after jumping is described by the function:
h(t) = 20 cos 0.8t + 20.
a What is the initial height of the bungee jumper?
b When, if at all, does the bungee jumper first touch the water?
c Assuming the cord is perfectly elastic, how long is it until the bungee jumper
returns to the lowest position?
Chapter 5 Graphing periodic functions 271
7 A cyclist rides one lap of a circular track at
a constant speed so that her distance, d metres,
from her starting point at any time, t seconds,
after starting is:
πt
d = 50 − 50 cos ------
30
What is:
a the time taken to complete one lap?
b the radius of the track?
c the maximum distance from the start?
d the length of the track?
e the distance from her starting point after:
ii 15 seconds and
ii 40 seconds?
8 The depth of water, d metres, at a port
entrance is given by the function
πt
d(t) = 4.5 + 1.5 sin ------
12
where t is in hours.
a Find i the maximum and ii the
minimum depth at the port entrance.
b A certain ship needs the depth at the
port entrance to be more than 5 metres.
The ship can be loaded and unloaded,
and in and out of the port, in 9 hours.
Assuming that the ship enters the port
just as the depth at the entrance passes
5 metres, will the ship be able to exit
9 hours later? How long will it have to
spare, or by how many minutes will it
miss out?

Sunrise to sunset
As you will know, throughout the year the hours of daylight per day vary according
to season. Days are longer in summer and shorter in winter. In this investigation we
are going to explore how this may be related to periodic functions.
1 What is the longest day of the year? (That is, the day with the most hours of
daylight.)
2 What is the shortest day of the year?
3 What is the equinox and when does the equinox occur?
4 Numbering the days of the year from 1 to 365, draw a graph to show the hours
of daylight each day.
5 What is the period of your function?
6 Try to write an equation for the function.
272 M a t h s Q u e s t M a t h s B Ye a r 1 1 f o r Q u e e n s l a n d

summary
Definition of a periodic function
• A periodic function is one that repeats itself over a period of time.
• The period of the function is the amount of time taken for the function to repeat
itself.
• The amplitude of the function is the maximum displacement of the function from
its equilibrium.
Radian measure
• An angular measure of 1 radian will cut off an arc of one unit on the unit circle.
• π c = 180°.
c
π
• To convert an angle in degrees to radians we multiply by ----------- .
180°
180°
• To convert an angle in radians to degrees we multiply by ----------
c
-.
π
Exact values
π–
• For exact values — use special triangles: 3

30°

2
2 3
1
π 45° π 60°
– –
4 1 6 1

Angle (θ ) sin θ cos θ tan θ


0 0 1 0
π
--- or 30° 1 3 1 3
--- ------- ------- = -------
6 2 2 3 3

π
--- or 45° 1 2 1 2
------- = ------- ------- = ------- 1
4 2 2 2 2

π
--- or 60° 3 1
------- --- 3
3 2 2

π
--- or 90° 1 0 undefined
2

Symmetry
A unit circle sketch of the given angle related back to the first quadrant is often
helpful. Provided an angle is expressed as π ± θ or 2π ± θ, the trig function (sin or
cos) remains the same; only the sign (+ or −) may change. So π may be written as
3π 4π 6π 8π
------ , ------ , ------ , ------ etc.
3 4 6 8
Chapter 5 Graphing periodic functions 273
• sin (π − θ) = sin θ • sin (π + θ) = −sin θ • sin (2π − θ) = −sin θ
• cos (π − θ) = −cos θ • cos (π + θ) = −cos θ • cos (2π − θ) = cos θ
• tan (π − θ) = −tan θ • tan (π + θ) = tan θ • tan (2π − θ) = −tan θ

Trigonometric graphs
• Basic graph types:
1. y = A sin Bx y = A cos Bx

2. Period = ------ , amplitude = | A |
B
y y
y = A sin Bx y = A cos Bx
A A

0 –π π
2— x 0 —π –π π
3— π
2— x
B B = period 2B B 2B B

–A –A

• Sketching:
1. Recall basic graph type.
2. Find period and amplitude.
3. Sketch in sections of one period.
4. Find x-intercepts between other known intercepts or minimum and maximum
points.
• Tangent graphs:
1. y = A tan Bx
π
2. Period = --- , no amplitude
B
π
3. Asymptotes at x = ± ------- , and every period to the left and right of these.
2B
4. The formula for asymptotes applies:
( 2n + 1 ) π
xasymptote = ± ------------------------ , where n = 0, 1, 2, . . .
2B
y = A tan Bx
y

3π 0 x
– 2B
— – Bπ– π
– 2B
— —π
2B
π–
B
3—π
2B

Applications of sine and cosine functions


• To find the maximum value of a function, replace sin x or cos x with +1.
• To find the minimum value of a function, replace sin x or cos x with −1.
• Initial values occur at t = 0.
• A sketch graph may provide greater understanding.
274 M a t h s Q u e s t M a t h s B Ye a r 1 1 f o r Q u e e n s l a n d

CHAPTER
review
1 State the period and the amplitude of the function drawn below.
5A y
2
1
0 1 2 3 4 5 6 7
x
–1
–2

2 A periodic function has a period of 6 units. Find the frequency of the function.
5A
3 Describe the features of a periodic function.
5A
4 Convert the following angles to radians.
5B a 45° b 150° c 50° d 260°
5 Convert the following angles to degrees.
5B c c
π 5π
a ----- b 1.2π c c -------- d 3c
6 3

6 multiple choice
5C In which quadrants is tan x positive?
A 1 and 2 B 1 and 4 C 2 and 4 D 2 and 3 E 1 and 3

7 multiple choice
5C
The value of cos 320° is:
A positive, as 320° is in the 1st quadrant B negative, as 320° is in the 2nd quadrant
C negative, as 320° is in the 3rd quadrant D negative, as 320° is in the 4th quadrant
E positive, as 320° is in the 4th quadrant

8 multiple choice
5C
If tan a < 0, sin a < 0 and 0° < a < 360°, then which one of the following is correct?
A 0° < a < 90° B 180° < a < 270° C 90° < a < 180°
D 0° < a < 180° E 270° < a < 360°

9 multiple choice
5C
If tan θ = 3 , then sin θ could be equal to:
1 1
A --- B 3
------- C −1 D ------- E 0
2 2 2

10 multiple choice
5C
The angle 150° is equivalent to:
2π 7π 5π 5π 4π
A ------ B ------ C ------ D ------ E ------
3 6 9 6 3
Chapter 5 Graphing periodic functions 275
11 multiple choice

5C
The angle which is equivalent to ------ is:
4
A 405° B 315° C 540° D 270° E 300°

12 multiple choice
4π 5C
The value of cos ------ is:
3 1
A ------3- B 1--- C − ------3- D ------- E − 1---
2 2 2 2 2

13 Without using a calculator, state the exact value of:


a sin 30° b cos 60° c tan 45°. 5C
14 Without the use of a calculator, state the exact value of:
π π π
5C
a sin --- b cos --- c tan --- .
4 6 3

15 If cos θ = 0.69 find:


π 5D
a sin ( --- − θ ) b cos (2π − θ ) c cos (π + θ ).
2
Questions 16 to 18 refer to the function: f(x) = −2 sin 3x
16 multiple choice
5E
The amplitude of f(x) is equal to:
A 3 B −2 C −3 D 2 E 1

17 multiple choice
5E
The period of f(x) is equal to:

A 6π B 4π C ------ D 2π E 3
3

18 The range of f(x) is:


A [0, 2] B [0, 4] C [−2, 0] D [−1, 1] E [−2, 2] 5E
19 multiple choice
5E
The rule for this graph is: y

–2π –π 0 π 2π x

–3
x
A y = 3 cos --- B y = 3 sin 2x
2
x
C y = 2 cos --- D y = 3 cos 2x
3
x
E y = −3 cos ---
2
276 M a t h s Q u e s t M a t h s B Ye a r 1 1 f o r Q u e e n s l a n d

20 Sketch the graphs of the following functions.


5E x
a y = −4 sin --- for the domain −2π ≤ x ≤ 4π
2
b y = 1.5 cos 2x for the domain −π ≤ x ≤ π
21 The number of rabbits in a national park is
5F observed for one year. At any time, t months,
after observation begins, the number of rabbits
is modelled by the function
πt
P = 2 − 0.8 sin ----- where P is in thousands.
6
a Find:
i the minimum number of rabbits
ii the maximum number of rabbits
iii the equilibrium number of rabbits.
b Find:
i the period of the function
ii the amplitude of the function
iii the frequency of the function.
c Sketch the graph of the function.
d Find the population after 5 months.

Modelling and problem solving


1 The temperature in an office is controlled by a
thermostat. The preferred temperature P can be
set to values between 18 and 25 degrees Celsius.
The temperature (T ºC) in the office at time
t hours after 9 am is given by the rule
T = P + 2.4 sin (π t).
If the preferred temperature on the thermostat
has been set to 23 °C:
a find the maximum and minimum temperatures
b find the temperature at i noon and ii 3.30 pm
c sketch the graph of the function between 9 am
and 5 pm.
Freddy feels thirsty if the temperature is above 24.2 degrees.
d Find the amount of time between 9 am and 5 pm that Freddy feels thirsty.
2 A ‘standing wave’ on a guitar string may be y
π
approximated by the function y = 0.3 sin ------ x , L
20
where x cm and y cm are defined on the diagram
x
at right.
eBook plus a Find the period of the standing wave.
b If the frets coincide with the mean positions
Digital doc: of the wave, find the value of L.
Test Yourself
Chapter 5 c If the frets were to be spaced at 16 cm, what would be the equation of a similar standing
wave of amplitude 0.3 cm such that a fret is at each mean position?
Trigonometric
equations
6
syllabus reference
Topic:
• Periodic functions and
applications

In this chapter
6A Simple trigonometric
equations
6B Equations using radians
6C Further trigonometric
equations
6D Identities
6E Using the Pythagorean
identity
278 M a t h s Q u e s t M a t h s B Ye a r 1 1 f o r Q u e e n s l a n d

Introduction
Sudhira is a keen fisherman. The ideal depth for fishing in Sudhira’s favourite tidal lake
is 3 metres. The depth of water in the lake can be found using the equation
π
D = 5 − 4 sin  --- t
 6
where t is the time in hours after midnight. What is the best time of day for Sudhira to
fish?
To solve this problem we need to solve a trigonometric equation

Simple trigonometric equations


From your earlier work on trigonometry, you will be familiar
with problems of the type: ‘Find the size of the angle marked
θ in the figure at right’. 13 cm
The solution to this problem is set out as:
adj
cos θ = ---------
hyp θ
9 9 cm
cos θ = ------
13
θ = 46°.
9
The equation cos θ = ------ is an example of a trigonometric equation. This trigono-
13
metric equation had to be solved in order to find the size of the angle in the triangle. In
this particular case we knew that the angle θ was acute from the triangle that was
drawn.
In the earlier chapter on graphing periodic functions we saw that the cos function
was periodic. This means that there are values of θ, other than the one already found
9
for which cos θ = ------ . There will, in fact, be an infinite number of solutions to this
13
trigonometric equation, so for practical reasons we are usually given a domain within
which to solve the equation. This domain will often be in the form 0° ≤ θ ≤ 360°,
meaning that we want solutions within the first positive revolution.
If the trigonometric ratio is positive the calculator will y
give a first quadrant answer. To complete the solution we
need to consider all quadrants for which the trigonometric
ratio is positive.
Sine All
9 positive positive
In the case of cos θ = ------ the cosine ratio is positive in
13 x
the first and fourth quadrants. We found earlier that the Tangent Cosine
first quadrant solution to this equation was 46°. The fourth positive positive
quadrant solution will therefore be 360° − 46° = 314°.
For a negative trigonometric ratio we solve the
corresponding positive equation to find a first quadrant angle to use, then find the
corresponding angles in the negative quadrants.
C h a p t e r 6 Tr i g o n o m e t r i c e q u a t i o n s 279
WORKED Example 1
Solve the following trigonometric equations over the domain 0∞ ≤ θ ≤ 360∞, correct to the
nearest degree.
a sin θ = 0.412
4
b tan θ = − ------
11

THINK WRITE
a 1 Write the equation. a sin θ = 0.412
2 Use your calculator to find the first First quadrant angle = 24°
quadrant angle. y

180
°−
24°
24°
x

3 The sine ratio is positive in the first and


second quadrants.
4 Find the second quadrant angle by 180° − 24° = 156°
subtracting 24° from 180°.
5 Write the answer. θ = 24° or 156°

4
b 1 Write the equation. b tan θ = − ------
11
2 Use your calculator to find the first First quadrant angle = 20°
quadrant angle. y

180
°−2

20°
x
360
°−2

3 The tangent ratio is negative in the


second and fourth quadrants.
4 Find the second quadrant angle by 180° − 20° = 160°
subtracting 20° from 180° and the 360° − 20° = 340°
fourth quadrant angle by subtracting
20° from 360°.
5 Write the answer. θ = 160° or 340°
280 M a t h s Q u e s t M a t h s B Ye a r 1 1 f o r Q u e e n s l a n d

Graphics Calculator tip! Solving a simple


trigonometric equation
As we saw in chapter 4, you can solve a trigonometric equation using the solver or
solve function. Consider Worked example 1a on the previous page.
For the Casio fx-9860G AU
1. Press MENU and then select EQUA.
Press F3 (SOLV) and select F3: Solver.

2. Check that your calculator is in degrees mode and


then enter the equation sin x = 0.412.

3. Press F6 (SOLV) to solve this equation.

The calculator will only give you the first quadrant solution. You will need to find
further solutions using the method described in Worked example 1.

For the TI-Nspire CAS


1. Open a new Calculator document. Check that
your calculator is in degrees mode. Press k to
access the catalog and then press 1 for the list
of functions. Scroll down to select solve(. You can
do this more quickly by first pressing S.
(Alternatively, you can use the letter keys to type
in solve and then press (.)

2. Press · to return to the calculator screen and


then enter the equation sin x = 0.412. (Remember
to include brackets where appropriate.) Include
the variable and the domain over which we are to
solve the equation. (Press * for the | symbol. The
≤ symbol is found in the symbol palette. Press
/ k to access the symbol palette.) Finally,
press · to solve the equation.
eBook plus

Digital doc: Simple trigonometric equations can also be solved using the Excel spreadsheet titled
EXCEL Spreadsheet ‘Trigonometric equations’, which can be accessed by logging into www.jacplus.com.au
Trigonometric equations
and locating the weblinks for this chapter.
C h a p t e r 6 Tr i g o n o m e t r i c e q u a t i o n s 281
In the earlier chapter we also found that we were able to find exact values of special
angles using the triangles below.

45°
30° Line of bisection
2
2
1

45° 60°
1 1

These special angles should be used where possible in the solution to a trigonometric
equation. They are used when we recognise any of the values produced by the triangles.
1 3 3
sin 30° = --- cos 30° = ------- tan 30° = -------
2 2 3
2 2
sin 45° = ------- cos 45° = ------- tan 45° = 1
2 2
3 1
sin 60° = ------- cos 60° = --- tan 60° = 3
2 2

WORKED Example 2
3
Solve the equation cos θ = – ------- over the domain 0∞ ≤ θ ≤ 360∞.
2

THINK WRITE

3
1 Write the equation. cos θ = – -------
2
2 Use the special triangles to find the first First quadrant angle = 30°
quadrant angle.
3 The cosine ratio is negative in the second
and third quadrants.
4 Find the second quadrant angle by 180° − 30° = 150°
subtracting 30° from 180° and find the third 180° + 30° = 210°
quadrant angle by adding 30° to 180°.
5 Write the answer. θ = 150° or 210°

Similarly, we must be aware of when the y


boundary angles should be used in the solution 1 90°
of the equation. Remember from the work on
the unit circle that y = sin θ, x = cos θ and 0° or 360°
y 180° x
–1 0 1
tan θ = -- .
x

–1 270°
282 M a t h s Q u e s t M a t h s B Ye a r 1 1 f o r Q u e e n s l a n d

WORKED Example 3
Solve the equation sin θ = −1 in the domain 0∞ ≤ θ ≤ 360∞.
THINK WRITE
1 Write the equation. sin θ = −1
2 y = sin θ so find the angle with a θ = 270°
y-value of −1.

remember
1. Trigonometric equations are equations that use the trigonometric ratios.
2. The trigonometric functions are periodic and so they have an infinite number of
solutions. The equation is usually written with a restricted domain to limit the
number of answers.
3. There are two solutions to most trigonometric equations with a domain
0° ≤ θ ≤ 360°.
4. Remember the special triangles as they are used in many solutions.
5. Boundary angles may also provide the solution to an equation.

6A Simple trigonometric equations

WORKED 1 Solve each of the following trigonometric equations over the domain 0° ≤ θ ≤ 360°,
Example
1 correct to the nearest degree.
a sin θ = 0.6 b cos θ = −0.25 c tan θ = 5.72 d sin θ = −0.85
e cos θ = 0.195 f tan θ = −0.837 g sin θ = −0.333 h cos θ = 0.757
WORKED 2 Find exact solutions to each of the following trigonometric equations over the domain
Example
2 0° ≤ θ ≤ 360°.
3 2 1
a sin θ = ------- b cos θ = ------- c tan θ = 3 d sin θ = – ---
2 2 2
1 3 2 3
e cos θ = – --- f tan θ = − ------- g sin θ = − ------- h cos θ = -------
2 3 2 2

3 multiple choice
eBook plus
2
Digital docs:
If sin x = cos x = − ------- and 0° ≤ x ≤ 360°, then x is:
2
SkillSHEET 6.1
Finding trigonometric A 150° or 210° B 135° or 225° C 225°
values and angles
D 135° or 315° E 120°
SkillSHEET 6.2
Exact values of
trigonometric ratios
4 It is known that sin θ < 0 and that tan θ > 0. Which quadrant does the angle θ lie in?
Explain your answer.
C h a p t e r 6 Tr i g o n o m e t r i c e q u a t i o n s 283
5 Yvonne is doing a trigonometric problem that has reduced to the equation sin θ = 1.5.
a When Yvonne tries to solve this equation her calculator returns an error message.
Why?
b When checking her working Yvonne realises that she should have used the tangent
ratio. Why is it now possible to achieve a solution to the equation tan θ = 1.5?
WORKED 6 Solve each of the following equations over the domain 0° ≤ θ ≤ 360°.
Example
3
a sin θ = 1 b cos θ = 0 c tan θ = 0
d sin θ = 0 e cos θ = −1 f sin θ = −1
7 Solve the following trigonometric equations over the domain 0° ≤ θ ≤ 360°.
a sin θ = 0.5 b cos θ = 0.35 c tan θ = −1
d sin θ = −0.87 e cos θ = −0.87 f tan θ = 1.4

Equations using radians


We have seen that a radian is an alternative method of measuring an angle. A trigon-
ometric equation can be solved using radians as well as degrees. Usually the domain
given will indicate whether it is expected that you will solve the equation in degrees or
in radians.
For example, if you are asked to solve an equation over the domain 0° ≤ θ ≤ 360°
then degrees are expected for the answer. However, if the given domain is 0 ≤ θ ≤ 2π
then it is expected that the answer will be given in radians.
The method of solving the equations is the same, but be sure that your calculator is
in radian mode before attempting to solve the problem to give an answer in radians.

WORKED Example 4
Solve the equation tan θ = 0.8 over the domain 0 ≤ θ ≤ 2π. Give the answer correct to
2 decimal places.
THINK WRITE
1 Write the equation. tan θ = 0.8
2 Use your calculator to find the first First quadrant angle = 0.67
quadrant angle.
y
3 The tangent ratio is positive in the first
and third quadrants.

0.67
x
π + 0.67

4 Find the third quadrant angle by adding π + 0.67 = 3.81


0.67 to π.
5 Write the answer. θ = 0.67 or 3.81
284 M a t h s Q u e s t M a t h s B Ye a r 1 1 f o r Q u e e n s l a n d

When the special angles are used, it is still important to recognise them and recog-
nise their radian equivalents in terms of π.

π 1 π 3 π 3
sin --- = --- cos --- = ------- tan --- = -------
6 2 6 2 6 3

π 2 π 2 π
sin --- = ------- cos --- = ------- tan --- = 1
4 2 4 2 4

π 3 π 1 π
sin --- = ------- cos --- = --- tan --- = 3
3 2 3 2 3

WORKED Example 5
3
Solve the equation sin θ = – ------- over the domain 0 ≤ θ ≤ 2π.
2

THINK WRITE

3
1 Write the equation. sin θ = – -------
2
π
2 Use the special triangles to find the first First quadrant angle = ---
quadrant angle. 3
3 The sine ratio is negative in the third y
and fourth quadrants.

π

3
x
π
2π – —
3
π
π+—
3

π π
4 Find the third quadrant angle by adding θ = π + --- or θ = 2π − ---
3 3
π
--- to π and the fourth quadrant angle by 4π 5π
3 θ = ------ or θ = ------
3 3
π
subtracting --- from 2π.
3
4π 5π
5 Write the answer. θ = ------ or ------
3 3

All of the equations that we have dealt with so far have been one-step solutions. In
many examples we may need to rearrange the equation before we are able to use the
calculator to solve it.
When rearranging the equation, we attempt to place the trigonometric ratio alone on
one side of the equation, as in the example above.
C h a p t e r 6 Tr i g o n o m e t r i c e q u a t i o n s 285
WORKED Example 6
Find x if 2 sin x = 0.984 over the domain 0 ≤ x ≤ 2π.
THINK WRITE
1 Write the equation. 2 sin x = 0.984
2 Divide both sides by 2 to get sin x by sin x = 0.492
itself.
3 Use your calculator to find the first First quadrant angle = 0.514
quadrant angle.
4 The sine ratio is positive in the first and y
second quadrants. π
–0
.51
4

0.514
x

5 Find the second quadrant angle by θ = π − 0.514


subtracting 0.514 from π. θ = 2.628
6 Write the answer. θ = 0.514 or 2.628

remember
1. Many trigonometric equations will need to be solved using radians.
2. The domain within which you are asked to solve the equation will tell you
whether to use degrees or radians.
3. You will need to know the special angle results as they apply to radians.
4. You must isolate the trigonometric ratio before you can solve any equation
using either your calculator or the special angles.

6B Equations using radians


WORKED 1 Solve each of the following equations over the domain 0 ≤ x ≤ 2π. Give your answers
Example
4
correct to 2 decimal places.
a sin x = 0.8 b cos x = −0.5 c tan x = 1.5
d sin x = −0.327 e cos x = 0.707 f tan x = −0.39
WORKED 2 Solve each of the following over the domain 0 ≤ x ≤ 2π.
Example
5 3 1
a sin x = ------- b cos x = − --- c tan x = 1
2 2
2 1
d cos x = ------- e tan x = − 3 f sin x = − ---
2 2
286 M a t h s Q u e s t M a t h s B Ye a r 1 1 f o r Q u e e n s l a n d

3 Solve each of the following over the domain 0 ≤ x ≤ 2π.


a sin x = 0 b tan x = 0 c cos x = 0 d sin x = 1
e cos x = 1 f cos x = −1 g sin x = −1
WORKED 4 Find exact solutions to each of the following equations over the domain 0 ≤ x ≤ 2π.
Example
6 a 2 sin x = 1 b 2 cos x = 3 c 2 tan x = 2
d 2 sin x + 3 = 0 e 2 cos x + 2 = 0 f 3 tan x + 3 = 0

5 multiple choice
The solution to the equation 2 cos x + 1 = 0 over the domain 0 ≤ x ≤ 2π is:
π 2π 5π 7π π 5π
A ---, ------ B ------, ------ C ---, ------
3 3 6 6 6 6
2π 4π 4 π 5π
D ------, ------ E ------, ------
3 3 3 3
6 Solve each of the following over the domain 0 ≤ x ≤ 2π.
a 4 sin x = 1 b 3 cos x = −2 c 2 tan x − 7 = 0
d 4 + sin x = 3 e 1 + 2 cos x = 2 f 3 tan x + 9 = 0

Further trigonometric equations


In many cases the equation that we have to solve may not be in the domain
0 ≤ x ≤ 2π. We may be asked to solve the equation in the domain 0 ≤ x ≤ 4π (2 revol-
utions) or −2π ≤ x ≤ 2π (also 2 revolutions, but one in the negative sense).
To find the solutions to a trigonometric equation beyond the first revolution we
simply add or subtract 2π to the first revolution solutions.

WORKED Example 7
Find α if sin α = 0.7 in the domain 0 ≤ α ≤ 4π.
THINK WRITE
1 Write the equation. sin α = 0.7
2 Use your calculator to find the first First quadrant angle = 0.7754
quadrant angle. y
The sine ratio is positive in the first and π–
3 0.7
second quadrants. 75
4

0.7754
x
C h a p t e r 6 Tr i g o n o m e t r i c e q u a t i o n s 287

THINK WRITE
4 Find the second quadrant angle by α = π − 0.7754
subtracting 0.7754 from π. α = 2.3662
5 Find the solutions between 2π and 4π α = 0.7754 + 2π α = 2.3662 + 2π
adding 2π to each of the first revolution α = 7.0586 α = 8.6494
solutions.
6 Write the answer. α = 0.7754, 2.3662, 7.0586, 8.6494

In many equations you will first need to make the trigonometric ratio the subject of
the equation.

WORKED Example 8
Find x if 2 cos x + 1 = 0 over the domain −2π ≤ x ≤ 2π.
THINK WRITE
1 Write the equation. 2 cos x + 1 = 0
2 Make cos x the subject of the equation. 2 cos x = −1
–1
cos x = -------
2
π
3 Use the special triangles to find the first First quadrant angle = ---
quadrant angle. 4
4 The cosine ratio is negative in the
second and third quadrants.
π π
5 Find the second quadrant angle by x = π − --- x = π + ---
4 4
π 3π 5π
subtracting --- from π. Find the third x = ------ x = ------
4 4 4
π
quadrant angle by adding --- to π.
4
3π 5π
6 To find the solutions between −2π and x = ------ − 2π x = ------ − 2π
0, subtract 2π from each of the first 4 4
revolution solutions. 5π 3π
x = − ------ x = − ------
4 4
5π 3π 3π 5π
7 Write the answer. x = − ------ , − ------ , ------ , ------
4 4 4 4

remember
1. To find solutions to trigonometric equations between 2π and 4π we add 2π to
any solutions in the first revolution.
2. To find solutions to trigonometric equations between −2π and 0 we subtract 2π
from any solutions in the first revolution.
3. In many cases it may be necessary to rearrange an equation to make the
trigonometric ratio the subject.
288 M a t h s Q u e s t M a t h s B Ye a r 1 1 f o r Q u e e n s l a n d

Further trigonometric
6C equations
WORKED 1 Solve each of the following trigonometric equations over the domain 0 ≤ x ≤ 4π.
Example
7 a cos x = −0.6591 b sin x = 0.9104 c cos x = 0.48
d sin x = −0.371 e tan x = 0.58 f tan x = −2.1

2 Solve each of the following trigonometric equations over the domain −2π ≤ x ≤ 2π.
a sin x = 0.2686 b cos x = −0.7421 c tan x = −0.4776
d sin x = −0.5432 e cos x = 0.1937 f tan x = 3

WORKED 3 Find the solutions to the following trigonometric equations over the domain −2π ≤ x ≤ 2π.
Example
8 a 2 sin x − 1 = 0 b 3 cos x = 0 c 2 sin x + 3 = 0
d tan x + 3 = 0 e 2 cos x = 1 f 3 tan x − 1 = 0

4 Find all the solutions to the following equations over the domain −2π ≤ x ≤ 2π. Give
each answer correct to 2 decimal places.
1
a 4 sin x + 2 = 6 b 3 cos x − 3 = 0 c --- cos x + 4 = 4.21
2

d 2 sin x − 5 = −4 e 2 cos x + 2 = 3 f 2 cos x + 3 =0

5 A particle moves in a straight line so that its distance, x metres, from point O is given
by the equation x = 3 + 4 sin t, where t is the time in seconds after the particle begins
eBook plus to move.
a Find the distance from O when the particle begins to move.
Digital doc:
WorkSHEET 6.1
b Find the time when the particle first reaches O. Give your answer correct to
2 decimal places.

Fishing
You should now be able to solve the fishing problem given at the start of this
chapter. The depth of water in the lake was given by

π
D = 5 − 4 sin  --- t
 6

Substitute D = 3 and solve to find the best time for Sudhira to fish. The solutions
should be found in the domain 0 < t < 24.

Identities
An identity is a relationship that holds true for all legitimate values of a pronumeral or
pronumerals. For example, a simple identity is x + x = 2x. The identities described in
this section are far more interesting and useful than this, as you will see.
C h a p t e r 6 Tr i g o n o m e t r i c e q u a t i o n s 289
The Pythagorean identity y
Consider the right-angled triangle in the unit
P(θ )
circle shown.
1
Applying Pythagoras’ theorem to this triangle gives
sin θ
the identity: θ
O cos θ D
2 2
sin θ + cos θ = 1

The tangent y B
1
Consider the unit circle on the right. P(θ ) C
A tangent is drawn at A and extended to the point 1 sinθ tan θ
C, so that OC is an extension of OP. This tangent is
called tangent θ, which is abbreviated to tan θ. θ
O cos θ D 1A x
Triangles ODP and OAC are similar, because they
have their three corresponding angles equal.

tan θ sin θ
It follows that: ------------ = ------------ (corresponding sides)
1 cos θ

sin θ
or tan θ = ------------ (as mentioned in an earlier section).
cos θ

Another relationship between sine and cosine —


complementary functions
Consider the unit circle shown on the right: y
The triangles OAB and ODC are congruent because 1 y
D C(90 – θ )
they have all corresponding angles equal and the
hypotenuse equal (radius = 1). x 1 B(θ )
θ 1
Therefore all corresponding sides are equal and it y
θ
follows that: –1 O x A1 x

sin (90 − θ )° = cos θ = x


and cos (90 − θ )° = sin θ = y –1
OR
π
sin ( --- − θ ) = cos θ
2
π
and cos ( --- − θ ) = sin θ
2

We say that sine and cosine are complementary functions.


Although the complementary function for tangent is not required for this course, you
may like to try to find it; that is,
tan (90 − θ )° = ?
290 M a t h s Q u e s t M a t h s B Ye a r 1 1 f o r Q u e e n s l a n d

WORKED Example 9
If sin θ = 0.4 and 0° < θ < 90°, find, correct to 3 decimal places: a cos θ b tan θ.
THINK WRITE
a 1 Use the identity sin2 θ + cos2 θ = 1. a sin2 θ + cos2 θ = 1
2 Substitute 0.4 for sin θ. (0.4)2 + cos2 θ = 1
3 Solve the equation for cos θ correct cos2 θ = 1 − 0.16
to 3 decimal places. = 0.84
cos θ = ± 0.84
= 0.917 or −0.917
4 Retain the positive answer only as For 0° < θ < 90°, cos is positive
cosine is positive in the first quadrant. so cos θ = 0.917.
sin θ sin θ
b 1 Use the identity tan θ = ------------ . b tan θ = ------------
cos θ cos θ
2 Substitute 0.4 for sin θ and 0.917 for
0.4
= -------------
cos θ. 0.917
3 Calculate the solution correct to = 0.436
3 decimal places.

WORKED Example 10
Find all possible values of sin θ if cos θ = 0.75.
THINK WRITE
2 2
1 Use the identity sin θ + cos θ = 1. sin2 θ + cos2 θ = 1
2 Substitute 0.75 for cos θ. sin2 θ + (0.75)2 = 1
3 Solve the equation for sin θ correct to sin2 θ = 1 − 0.5625
3 decimal places. = 0.4375
sin θ = ± 0.4375
4 Retain both the positive and negative = 0.661 or −0.661
solutions, since the angle could be in
either the first or fourth quadrants.

WORKED Example 11
Find a if 0° < a < 90° and a sin a = cos 42° b cos a = sin 73°.
THINK WRITE
a 1 Write the equation. a sin a = cos 42°
2 Replace cos 42° with sin (90 − 42)° sin a = sin (90 − 42)°
(complementary functions). sin a = sin 48°
a = 48°
b 1 Write the equation. b cos a = sin 73°
2 Replace sin 73° with cos (90 − 73)°. cos a = cos (90 − 73)°
cos a = cos 17°
a = 17°
C h a p t e r 6 Tr i g o n o m e t r i c e q u a t i o n s 291
WORKED Example 12
If 0° < a < 90° and cos a = 2--3- , find the exact values of:
a sin a b tan a c cos (90 − a)° d sin (180 + a)°.

THINK WRITE
1 Draw a right-angled triangle.
Mark in angle a, its adjacent side (adj) 2 and a
2
the hypotenuse (hyp) 3. hyp = 3
adj = 2

opp = 5

3 Use Pythagoras to calculate the opposite side O 2 = 3 2 − 22


(opp) to a. =5
O = 5

opp opp
a 1 Use the right-angled triangle to find --------- . a sin a = ---------
hyp hyp
5
2 Substitute opp = 5 and hyp = 3. = -------
3

opp opp
b 1 Use the right-angled triangle to find --------- . b tan a = ---------
adj adj
5
2 Substitute opp = 5 and adj = 2. = -------
2

c 1 Use the identity cos (90 − a)° = sin a. c cos (90 − a)° = sin a
5 5
2 Substitute sin a = ------- . cos (90 − a) = -------
3 3

d 1 Use the symmetry property d sin (180 + a)° = −sin a


sin (180 + a)° = −sin a.
5
2 Substitute sin a = ------- . sin (180 + a) = − ------5-
3 3

(Note: The above results could have been obtained using the identities directly.)

remember
sin θ
1. sin2 θ + cos2 θ = 1 2. tan θ = ------------ 3. sin (90 − θ )° = cos θ
cos θ

4. cos (90 − θ )° = sin θ


π
5. sin  --- – θ = cos θ
π
6. sin  --- – θ = cos θ
2  2 
292 M a t h s Q u e s t M a t h s B Ye a r 1 1 f o r Q u e e n s l a n d

6D Identities
1 Copy and complete the table below, correct to 3 decimal places:

θ 30° 81° 129° 193° 260° 350° −47°

sin2 θ

cos2 θ

sin2 θ + cos2 θ

WORKED 2 If sin θ = 0.8 and 0° < θ < 90°, find, correct to 3 decimal places:
Example
9a a cos θ b tan θ.

WORKED 3 If cos θ = 0.3 and 0° < θ < 90°, find, correct to 3 decimal places:
Example
9b a sin θ b tan θ.

WORKED 4 Find all possible values of the following.


Example
10 a cos x if sin x = 0.4 b cos x if sin x = −0.7
c sin x if cos x = 0.24 d sin x if cos x = −0.9

5 Use the diagram at left to find the exact values of:


a c
3
5 b sin x
c cos x. b
x
c

6 Use the diagram at right to find the exact values of: 8 2 7


x
a b b cos x c tan x.

7 Find the exact values of:


12
a cos x if sin x = ------ and 90° < x < 180°
13
b sin x if cos x = − 3--- and x is in the third quadrant
5
7
c cos x if sin x = − ------ and x is in the fourth quadrant
25
3 3π
d sin x if cos x = ------- and ------ < x < 2π
2 2
1
8 Given that cos θ = --- and θ is in the first quadrant, find:
9
a the exact value of sin θ
b the exact value of tan θ.

3
9 Given that sin θ = ------- and 90° <θ <180°, find:
2
a the exact value of cos θ
b the exact value of tan θ.
C h a p t e r 6 Tr i g o n o m e t r i c e q u a t i o n s 293

10 Given that sin θ = – ------5- and ------ <θ < 2π, find:
4 2
a the exact value of cos θ
b the exact value of tan θ.

11 Given that cos θ = – 2--- and sin θ < 0, find the exact value of tan θ.
3

36° c
12 multiple choice a

Examine the diagram at right and answer the following questions. 54°
b
a sin 54° is equal to:
A cos 54° B cos 36° C tan 36°
D sin 36° E tan 54°
b cos 54° is equal to:
A tan 36° B cos 36° C tan 54°
D sin 36° E sin 54°
c tan 36° is equal to:
cos 36° sin 36°
A ------------------ B sin 36° cos 36° C ------------------
sin 36° cos 36°
D sin 54° cos 54° E sin 36° + cos 36°
d tan 54° is equal to:
cos 36° cos 54°
A ------------------ B sin 54° − cos 54° C ------------------
sin 36° sin 54°
D sin 54° cos 54° E sin 36° cos 36°

WORKED 13 Find a if 0° ≤ a ≤ 90° and:


Example
11 a sin a = cos 20° b sin a = cos 58° c cos a = sin 39°
d cos a = sin 82° e sin 8° = cos a f cos 44° = sin a
g sin 89° = cos a h cos 17° = sin a.

14 Copy and complete the following table.

sin θ 0.8 0.28 0.77 0.573


cos θ 0.6 0.96 0.3 0.447
tan θ 3.18 1.2 2 0.7

2
WORKED 15 If 0° < a, b, c < 90° and sin a = ------- , cos b = 3--- , tan c = 11
---------- , find:
Example 5 5 5
12
a sin b b tan b c cos a
d tan a e sin c f cos c
g sin (90 − a)° h cos (90 − b)° i sin (90 − c)°
j sin (180 − a)° k cos (180 + b)° l tan (180 + c)°.
294 M a t h s Q u e s t M a t h s B Ye a r 1 1 f o r Q u e e n s l a n d

Further trigonometric identities


sin θ
The equations tan θ = ------------ and sin2 θ + cos2 θ = 1 are not the only non-trivial
cos θ
trigonometric identities.
Prove (or at least verify) that the equations below are also identities using one of
the following methods:
i Use the identities above, and algebraic manipulation.
ii Complete a table of values for several values of x and show that the left side
of the equation equals the right side.
iii Plot the left-hand side as Y1 and the right-hand side as Y2 using a graphics
calculator (or using graphing software) to show both sides’ graphs are
identical.
1 sin 2x = 2 sin x cos x 2 sin 3x + sin x = 2 sin 2x cos x
1
3 1 + tan2 x = --------------- 4 sin (x + y) = sin x cos y + cos x sin y
2
cos x

Using the Pythagorean identity


Consider the quadratic equation x2 − x = 0. This equation is solved by first factorising
the expression, then solving each factor equal to zero. Hence, there are two solutions to
the equation, as shown.
x2 − x = 0
x(x − 1) = 0
so x = 0 or x − 1 = 0.
That is, x = 0 or x = 1.
A similar equation involves the use of the trigonometric ratios. Consider the equation
2 sin2 θ = sin θ.
This equation is solved in the same way as a normal quadratic equation; however, the
two answers are in terms of the trigonometric ratio and then have to be solved.

WORKED Example 13
Solve the equation 2 sin2 θ = sin θ over the domain 0 ≤ θ ≤ 2π.
THINK WRITE
1 Write the equation. 2 sin2 θ = sin θ
2 Move sin θ to the left of the equation. 2 sin2 θ − sin θ = 0
3 Factorise the expression. sin θ (2 sin θ − 1) = 0
4 Set each factor equal to zero and solve. sin θ = 0 or 2 sin θ − 1 = 0
1
sin θ = ---
2
Solve sin θ = 0 and 2 sin θ − 1 = 0. π 5 π
5 θ = 0, π, 2π θ = ---, ------
6 6
π 5π
6 Combine all five solutions to the equation. θ = 0, ---, π, ------, 2 π
6 6

Some equations of this type will involve both the sin and cos ratios, but to solve the
equation there must be only one ratio. We use the identity sin2θ + cos2 θ = 1.
C h a p t e r 6 Tr i g o n o m e t r i c e q u a t i o n s 295
WORKED Example 14
Solve the equation 2 sin2 x = cos x + 1 over the domain 0 ≤ θ ≤ 2π.
THINK WRITE
1 Write the equation. 2 sin2 x = cos x + 1
2 2
2 Make the substitution sin x = 1 − cos x. 2(1 − cos2 x) = cos x + 1
3 Form a quadratic equation by bringing all 2 − 2 cos2 x = cos x + 1
terms to one side of the equation. 1 − 2 cos2 x − cos x = 0
2 cos2 x + cos x − 1 = 0
4 Factorise the quadratic. (2 cos x − 1)(cos x + 1) = 0
5 Solve each factor equal to 0. 2 cos x − 1 = 0 or cos x − 1 = 0
1
cos x = --- cos x = −1
2
1 π 5π
6 Solve cos x = --- and cos x = −1. x = ---, ------ x=π
2 3 3
π 5π
7 Combine all solutions. x = ---, π, ------
3 3

remember
1. Some trigonometric equations are solved as quadratic equations.
2. sin2 x + cos2 x = 1
3. Equations should have only one trigonometric ratio. The identities
cos2 x = 1 − sin2 x and sin2 x = 1 − cos2 x can be used to reduce an
equation to one trigonometric ratio.

6E Using the Pythagorean identity


WORKED
Example 1 Solve the trigonometric equation 2 cos2 θ = cos θ over the domain 0 ≤ θ ≤ 2π.
13
2 Solve the trigonometric equation tan2 x = 3 tan x over the domain 0 ≤ x ≤ 360°.
3 Solve each of the following equations over the domain 0 ≤ x ≤ 2π.
a sin2 x − sin x = 0 b cos2 x + cos x = 0 c 2 sin2 x + 3 sin x = 0
d 2 cos2 x + cos x − 1 = 0 e sin2 x + 3 sin x − 4 = 0 f 2 sin2 x − sin x − 1 = 0
4 Solve the trigonometric equation 2 sin2 x = 1 − sin x over the domain 0 ≤ x ≤ 2π.
5 Solve the trigonometric equation tan2 x − 1 = 0 over the domain 0 ≤ x ≤ 360°.
WORKED 6 Solve each of the following equations over the domain 0 ≤ θ ≤ 2π.
Example
14 a 2 cos2 θ = 1 + sin θ b 2 sin2 θ + sin θ − 1 = 0 c 2 sin2 θ − 1 = 0
d 1 + cos θ = 2 sin2 θ e sin2 θ = 1 + cos θ f 2 cos2 θ = 5 + 5 sin θ
eBook plus
7 Solve the following over the domain 0 ≤ x ≤ 2π, giving your answers correct to two
Digital doc: decimal places.
WorkSHEET 6.2 a 3 tan2 x − tan x − 2 = 0 b 2 sin2 x − 7sin x + 1 = 0 c 15 cos2 x − 8cos x + 1 = 0
296 M a t h s Q u e s t M a t h s B Ye a r 1 1 f o r Q u e e n s l a n d

summary
Simple trigonometric equations
• Trigonometric equations are equations that use trigonometric ratios.
• Trigonometric equations are periodic and so may have an infinite number of
solutions unless the domain is restricted.
• In a domain of one revolution most trigonometric equations will have two
solutions.
• Be aware of the special triangles as they may provide the solution to many
equations.
Radians
• A trigonometric equation may need to be solved using radians.
• The domain within which you are asked to solve the equation will tell you whether
to use degrees or radians.
Further trigonometric equations
• To find solutions to a trigonometric equation between 2π and 4π, add 2π to any
solutions in the first revolution.
• To find solutions to trigonometric equations between −2π and 0, subtract 2π from
any solutions in the first revolution.
Identities
• sin2 θ + cos2 θ = 1
sin θ
• tan θ = ------------
cos θ
π
• sin (90 − θ )° = cos θ or sin  --- – θ = cos θ
2 

π
• cos (90 − θ )° = sin θ or sin  --- – θ = cos θ
2 

Using the Pythagorean identity


• The Pythagorean identity can be used to simplify a quadratic equation using two
trigonometric ratios when one of them is squared.
• cos2 x = 1 − sin2 x and sin2 x = 1 − cos2 x
C h a p t e r 6 Tr i g o n o m e t r i c e q u a t i o n s 297

CHAPTER
review
1 Solve the following trigonometric equations over the domain 0° ≤ θ ≤ 360°, correct to the
nearest degree. 6A
a sin θ = 0.9 b cos θ = −0.4 c tan θ = 1.6
2 Find exact solutions to the following trigonometric equations over the domain 0° ≤ θ ≤ 360°.
3 1
6A
a sin θ = – ------- b cos θ = --- c tan θ = 1
2 2
3 Solve each of the following equations over the domain 0° ≤ θ ≤ 360°.
a sin θ = −1 b cos θ = −1 c tan θ = −1 6A
4 Solve the following trigonometric equations over the domain −360° ≤ θ ≤ 360°.
a cos θ = −0.5 b tan θ = −2.25 c sin θ = 0.95 6A
5 Solve each of the following equations over the domain 0 ≤ x ≤ 2π. Give your answers
correct to 2 decimal places. 6B
a sin x = 0.7 b cos x = −0.85 c tan x = 0.2
6 Solve each of the following over the domain 0 ≤ x ≤ 2π.
3 2
6B
a cos x = – ------- b tan x = – 3 c sin x = – -------
2 2
7 Find exact solutions to each of the following equations over the domain 0 ≤ x ≤ 2π.
a 2 sin x = 3 b 2 cos x = −1 c 4 tan x = −4 6B
8 Solve each of the following, to the nearest degree, over the domain 0° ≤ x ≤ 360°.
a 4 sin x = 3 b 4 cos x = −3 c 2 tan x − 6 = 0 6B
9 Solve each of the trigonometric equations below over the domain 0 ≤ x ≤ 4π.
a cos x = −0.458 b sin x = −0.504 c tan x = −0.84 6C
10 Solve the trigonometric equations below over the domain −2π ≤ x ≤ 2π.
a sin x = −0.816 b cos x = 0.427 c tan x = −1.6774 6C
11 Find solutions to the following trigonometric equations over the domain −2π ≤ x ≤ 2π.
1 6C
a 3 cos x + 1 = 0 b 4 sin x = 0 c ------- tan x – 1 = 0
3
12 Find:
a cos θ if sin θ = 0.5 and θ lies in the second quadrant 6D
5
b sin x if cos x = – ------ and x is in the third quadrant.
12
13 Given that a lies in the first quadrant, find a if:
a sin a = cos 30° b cos a = cos 28°. 6D
14 Solve the trigonometric equation 2 sin2 θ = sin θ for the domain 0 ≤ θ ≤ 2π.
6E
15 Solve each of the following equations over the domain 0 ≤ x ≤ 2π.
a 3 cos2 x − 2 cos x = 0 b sin2 x + 3 sin x − 4 = 0 6E
c 2 cos2 x = 1 + sin x d 2 − sin x − 3 cos2 x = 0
298 M a t h s Q u e s t M a t h s B Ye a r 1 1 f o r Q u e e n s l a n d

Modelling and problem solving


1 The sound level of a siren follows the rule
L(t) = 6 sin π t + 80, where L is the sound
level measured in decibels (dB) and t is the
time in seconds.
a What is the amplitude and period of L(t)?
b What are the minimum and maximum
sound levels of the siren?
c How long does it take for the sound to
reach its maximum level the first time?
d Find the first time the sound level reaches
83 dB.
e Sketch the graph of L(t) for 0 ≤ t ≤ 2.

2 The height (in centimetres) that a clock’s pendulum swings above its base can be

approximated by the function H = 14 + 5.9 cos  ------t at any time t seconds after being
4 
released.
Give all answers correct to 3 decimal places.
a Find i the maximum and ii the minimum
heights that the pendulum reaches.
b Find the height after
ii 1.5 seconds and
ii 1 minute.
c Sketch the graph of the function for the first
2 seconds.
d On the same set of axes sketch the median
position.
e Find the length of time that the pendulum is
below 14 centimetres travelling from one side
to the other.
f Find the number of times the pendulum swings
eBook plus in 1 minute.
The pendulum is found to be losing time and needs
Digital doc: its swing adjusted to 75 swings per minute.
Test Yourself
Chapter 6 g Find the new function H(t) that approximates
the height of the pendulum.
Exponential
and
logarithmic
functions
7
syllabus reference
Topic:
• Exponential and
logarithmic functions and
applications

In this chapter
7A Index laws
7B Negative and rational
powers
7C Indicial equations
7D Graphs of exponential
functions
7E Logarithms
7F Solving logarithmic
equations
7G Applications of exponential
and logarithmic functions
300 M a t h s Q u e s t M a t h s B Ye a r 1 1 f o r Q u e e n s l a n d

Introduction
There are one million millimetres in a kilometre.
It would take a person almost two weeks of non-stop
counting, day and night, to count to one million.
Although it is difficult to comprehend numbers of
this size, at times we must do so. Consider, for example,
measuring the intensity of sound ranging from a quiet
room to a jet engine.

How can we express the loudness of


these sounds without using numbers so
large or so small that the mind cannot
comprehend them?
The answer involves the use of a scale
based on exponents or powers of a
number. Exponents or, more specifically,
exponential functions provide a powerful
means of describing phenomena such as
radioactive decay, population growth or
the value of investments.

Index laws
Recall that a number, a, which is multiplied by itself n times can be represented in
index notation.
Index (or power or exponent)
a × a × a × … × a = an
Base






n lots of a
where a is the base number and n is the index (or power or exponent).
a n is read as ‘a to the power of n’ or ‘a to the n’.
Multiplication
When multiplying two numbers in index form
with the same base, add the indices. am × an = am + n
For example, 23 × 24 = 2 × 2 × 2 × 2 × 2 × 2 × 2 = 27
Division
When dividing two numbers in index form
with the same base, subtract the indices. am ÷ an = am − n
2×2×2×2×2×2
For example, 2 6 ÷ 2 2 = ------------------------------------------------- = 2 4
2×2
Raising to a power
To raise an indicial expression to a power,
multiply the indices. (a m)n = a m × n = a mn
For example, (24)3 = 24 × 24 × 24 = 24 + 4 + 4 = 212
Chapter 7 Exponential and logarithmic functions 301
Raising to the power of zero
Any number raised to the power of zero is equal to one. a0 = 1, a ≠ 0
For example 2 3 ÷ 2 3 = 2 3 − 3 = 20 [1]
3 3
or 2 ÷ 2 = (2 × 2 × 2) ÷ (2 × 2 × 2)
=8÷8
=1
So 23 ÷ 23 = 1 [2]
Using [1] and [2] we have 20 = 1.

Products and quotients


Note the following. (ab)n = anbn
For example, (2 × 3)4 = (2 × 3) × (2 × 3) × (2 × 3) × (2 × 3) n
 --a- an
=2×3 × 2×3 × 2×3 × 2×3 = -----n
 b b
=2×2×2×2 × 3×3×3×3
= 24 × 34

WORKED Example 1
Simplify each of the following.
8 p6 m2 × ( 3 p )3 m5
a 2x3y2 × 4x2y b (2x2y3)2 × xy4 c (3a)5b6 ÷ 9a4b3 d ---------------------------------------------
6 p4 m
THINK WRITE
a 1 Collect ‘plain’ numbers (2 a 2x 3y 2 × 4x 2y
and 4) and terms with the = 2 × 4 × x3 × x2 × y2 × y
same base.
2 Simplify by multiplying plain = 8x5y3
numbers and adding powers
with the same base.

b 1 Remove the bracket by b (2x2y3)2 × xy4


multiplying the powers. (The = 22 × x4 × y6 × xy4
power of the 2 inside the
bracket is 1.)
2 Convert 22 to a plain number = 4 × x4 × x × y6 × y4
(4) first and collect terms with
the same base.
3 Simplify by adding powers = 4x5y10
with the same base.

( 3a ) 5 b 6
c 1 Write the quotient as a fraction. c (3a)5b6 ÷ 9a4b3 = ------------------
-
9a 4 b 3
243a 5 b 6
2 Remove the bracket by = -------------------
-
multiplying the powers. 9a 4 b 3
Continued over page
302 M a t h s Q u e s t M a t h s B Ye a r 1 1 f o r Q u e e n s l a n d

THINK WRITE
27a 5 b 6
3 Simplify by first cancelling = ----------------
-
plain numbers. a4b3
4 Complete simplification by = 27ab3
subtracting powers with the
same base. (Note: a1 = a.)

8 p6m2 × ( 3 p )3m5 8 p6m2 × 33 p3m5


d 1 Expand the brackets by raising d ------------------------------------------- = ----------------------------------------
-
each term to the power of 3. 6 p4m 6 p4m

8 × 27 × p 6 × p 3 × m 2 × m 5
2 Convert 33 to 27 and collect = -----------------------------------------------------------------
‘like’ pronumerals. 6 p4m

3 Simplify by first reducing the = 36p6 + 3 − 4m2 + 5 − 1


plain numbers, then simplify the
pronumerals by adding the indices
for multiplication and subtracting
the indices for division.
4 Simplify the indices of each base. = 36p5m6

WORKED Example 2
6a 4 b 3  3a 2 b  3
Simplify ----------------- ÷ -------------- .
16a 7 b 6  2a 3 b 2

THINK WRITE
6a 4 b 3 3a 2 b 3
1 Write the expression. ----------------- ÷  --------------
16a b 7 6  2a 3 b 2
6a 4 b 3  2a 3 b 2 3
2 Change the division sign to multiplication and = ----------------
- × --------------
replace the second term with its reciprocal 16a 7 b 6  3a 2 b 
(turn the second term upside down).
6a 4 b 3 2 3 a 9 b 6
3 Remove the brackets by multiplying the = ----------------
- × -----------------
powers. 16a 7 b 6 3 3 a 6 b 3
6 × 8a 4 + 9 – 7 – 6 b 3 + 6 – 6 – 3
4 Collect plain numbers and terms with the = --------------------------------------------------------------
same base. 16 × 27

5 Cancel plain numbers and apply index laws. a0b0


= -----------
9
6 Simplify. = 1---
9

Expressions involving just numbers and numerical indices can be simplified using
index laws and then evaluated.
Chapter 7 Exponential and logarithmic functions 303
WORKED Example 3
Write in simplest index notation and evaluate.
95 × 34
a 23 × 162 b ----------------
-
27 3
THINK WRITE
a 1 Rewrite the bases in terms of their a 23 × 162 = 23 × (2 × 2 × 2 × 2)2
prime factors.
2 Simplify the brackets using index = 23 × (24)2
notation.
3 Remove the brackets by multiplying the = 23 × 28
powers.
4 Simplify by adding the powers. = 211
5 Evaluate as a basic number. = 2048
95 × 34 ( 3 × 3 )5 × 34
b 1 Rewrite the bases in terms of their b ----------------
- = -------------------------------
prime factors. 27 3 ( 3 × 3 × 3 )3
( 32 )5 × 34
2 Simplify the brackets using index = -----------------------
-
notation. ( 33 )3
3 10 × 3 4
3 Remove the brackets by multiplying the = ------------------
-
powers. 39
4 Write in simplest index form. = 35
5 Evaluate as a basic number. = 243

Complex expressions involving terms with different bases have to be simplified by


replacing each base with its prime factors.

WORKED Example 4
3 4n × 18 n + 1
Simplify ----------------------------- .
6 3n – 2
THINK WRITE
3 4n × 18 n + 1 3 4n × ( 3 × 3 × 2 ) n + 1
1 Rewrite the bases in terms of their ----------------------------- = -------------------------------------------------
prime factors. 6 3n – 2 ( 2 × 3 ) 3n – 2
3 4n × ( 3 2 × 2 1 ) n + 1
2 Simplify the brackets using index = --------------------------------------------
-
notation. ( 2 × 3 ) 3n – 2
3 4n × 3 2n + 2 × 2 n + 1
3 Remove the brackets by multiplying = ----------------------------------------------
-
powers. 2 3n – 2 × 3 3n – 2
4 Collect terms with the same base by = 3 4n + 2n + 2 – ( 3n – 2 ) × 2 n + 1 – ( 3n – 2 )
adding the powers in the products and
subtracting the powers in the quotients.
5 Simplify. = 3 6n + 2 – 3n + 2 × 2 n + 1 – 3n + 2
= 3 3n + 4 × 2 – 2n + 3
= 3 3n + 4 × 2 3 – 2n
304 M a t h s Q u e s t M a t h s B Ye a r 1 1 f o r Q u e e n s l a n d

Graphics Calculator tip! Simplifying expressions


Some graphics calculators have the facility to
simplify expressions. Consider simplifying the
4n n+1
3 × 18
expression ----------------------------
3n – 2
- shown in Worked example 4.
6
For the TI-Nspire CAS
1. Open a new Calculator document. Ensure that the
calculator is set to Auto. (To change the system
settings, press c and select 8: System Info <TI04.01>
followed by 2: System Settings.) Press /p to
insert a fraction template.

2. Enter the expression, using the l key for the


indices. Move the cursor to the end of the
expression and then press · to display the
simplified expression. <TI04.01>

3. To check that this is the same as the answer given


in Worked example 4, enter the expression
33n+4 × 23−2n and then press ·.
<TI04.01>

remember
1. Index laws:
(a) a m × a n = a m + n
(b) a m ÷ a n = a m − n
(c) a0 = 1
(d) (a m)n = a mn
(e) (ab)n = a nbn
a n an
(f)  --- = ----n-
 b b
2. To simplify indicial expressions:
(a) when dealing with questions in the form (expression 1) ÷ (expression 2),
replace expression 2 with its reciprocal and change ÷ to ×
(b) remove brackets using laws (d), (e) and (f)
(c) collect plain numbers and terms of the same base
(d) simplify using laws (a), (b) and (c).
Chapter 7 Exponential and logarithmic functions 305

7A Index laws

WORKED 1 Simplify each of the following.


Example
1a, b a x2 × x5 × x3 b m3 × m2p × p4
c 52 × 57 × (53)3 d 4y3 × 2y × y7
e (xy)3 × x4y5 f (2x4)2 × (4x2)5
g 3m2p5 × (mp2)3 × 2m4p6 h 5x2y3 × (5xy2)4 × (5x2y)2
WORKED 2 Simplify each of the following.
Example
1c a a7b8 ÷ a2b5 b 2a12b9 ÷ (2a)3b4
c (3x5)y11 ÷ 6x2y2 d p13q10 ÷ (pq4)2
a3b4
e (4mn4)2 ÷ 14n3 f ----------
-
ab 2
15a 6 b 7
g 25r15s10t4 ÷ r5(s5)2(5t)3 h -----------------
3a 3 b 4
24x 4 y 3
i -----------------
20x 2 y 3
WORKED 3 Simplify each of the following.
Example
1d 6 p8m4 × 2 p7m6 ( 3x ) 2 y 2 × 5x 6 y 3
a --------------------------------------
- b --------------------------------------
-
9 p5m2 10x 7 y
14u 11 v 9 × ( 3u 2 ) 3 v ( 5e 3 ) 2 f 4 × 8e 4 f 3
c -------------------------------------------- d ------------------------------------------
-
21u 6 v 5 20e f 5
6w 2 t 7 × 9w 4 t 12 ( 2x ) 4 y × ( 3x 7 y ) 2
e ------------------------------------- f ------------------------------------------
( 3w ) 5 t 13 18x 5 ( 2y ) 3
( – 3x 3 y 2 ) 3 6x 7 y 5 ( – 3mp ) 2 × 4m 4 p
g -----------------------
- × --------------- h -----------------------------------------
2x 3 y 6 ( x2 y )2 12 ( mp ) 2
m3 p4 × ( m p3 )2 4( u7v6 )3
i ------------------------------------- j ----------------------------------------------------
( –m p 2 )4 ( – 2u 3 v 2 ) 2 × u 4 ( 3v 5 ) 2

WORKED 4 Simplify each of the following.


Example
2 15a 8 b 3  2a 3 b 2 5k 12 d 6kd 4
b ---------------2 ÷ ------------------------3
a ----------------
- ÷ ------------
9a 4 b 5  3ab 2 ( 2k ) 25 ( k 2 d 3 )
3

4g 4 ( 2 p 11 ) 2 8g 4 p 3 jn 2 3 ( 4 j 2 n ) 2
c --------------------------- ÷ ----------------- d  ----------
- ÷ ---------------------
g3 p7 ( 2gp ) 3  n5  n 13 ( 2 j ) 4
x4 y7 x3 y2 6x 3 y 8 ( 2xy 3 ) 2
e ---------- ÷ ---------- f ------------------ ÷ -------------------
x3 y2 x5 y ( x 2 y 3 ) 3 8x 5 y 7

5 multiple choice
3 p3m4
a The fraction ---------------
- can be simplified to:
p1m2
A 3p2m2 B 3p4m6 C 3p3m8 D 3p3m2 E 3
306 M a t h s Q u e s t M a t h s B Ye a r 1 1 f o r Q u e e n s l a n d

6x 6 y 5 x4
b The product -------------
- × ------------- can be simplified to:
x 5 y 3 ( 2y ) 2

A 2x5y4 B 3x5y4 C x30y16 3x 9 y 10 3x 5


D ---------------- E --------
2 2
3ab 3 a 2 b 2
c The quotient ------------ ÷  -------- is equal to:
– ab  a 5 

A −a18 B −3a6 C −3a6b – 3a 15 E 3a6


D -------------
-
b6
6 Simplify each of the following.
n m+3 2 2
xn + 1 × y5 × z4 – n (x y ) x y
a ----------------------------------------------
- b ------------------------
- × --------------------------------
xn – 2 × y4 – n × z3 – n x
n+2 3–m
y x
n–5
×y
5 – 3m

WORKED 7 Write in simplest index notation.


Example
3 a 24 × 42 × 8 b 37 × 92 × 273 × 81 c 53 × 152 × 32
3 4 × 27 2 8 × 52
d 205 × 84 × 125 e -------------------- f -----------------
64 × 35 2 3 × 10

8 Write in simplest index notation and evaluate.


45 ( 16 2 ) 3 27 2
a ----7- b 9 4 × 3 5 ÷ 27 c --------------- d -----------3-
2 ( 25 )4 ( 32 )

( 625 ) 4 ( 25 ) 4 4 11 ÷ 8 2 27 2 × 81
e ---------------- f ---------------- g ------------------
- h --------------------
( 53 )5 ( 125 ) 3 16 3 93 × 35

WORKED 9 Simplify each of the following.


Example
4 2 n × 9 2n + 1 25 3n × 5 n – 3
a -------------------------
- b ----------------------------
-
6n – 2 5 4n + 3
12 x – 2 × 4 x 12 n – 3 × 27 1 – n
c -------------------------- d ----------------------------------------
6x – 2 9 2n × 8 n – 1 × 16 n
4 n × 7 n – 3 × 49 3n + 1 35 2 × 5 5 × 7 6
e ----------------------------------------------- f -------------------------------
14 n + 2 25 4 × 49 3
3 5n – 4 × 16 n × 9 3 3n + 3n + 1
g -----------------------------------------------------
+
h* -----------------------
4 1 × 18 1 – n × 6 3 – 2n
n 3n + 3n – 1
5n – 5n + 1
i * -----------------------
5n + 1 + 5n
*Hint: Factorise the numerator and denominator first.

10 multiple choice
36 2n × 6 n + 3
In simplest index notation, ----------------------------
- is equal to:
216 n – 2
A 216n + 5 B 65n + 1 C 62n + 5 D 69 E 62n + 9
Chapter 7 Exponential and logarithmic functions 307
Negative and rational powers
Negative powers
Wherever possible, negative index numbers should be expressed with positive index
numbers using the simple rule:

When an index number is moved from the numerator to denominator or vice


versa, the sign of the power changes.
1
a – n = -----n , a ≠ 0
a
This is easily verified as follows:
1 a0
----- = ----- since a0 = 1
a n an
= a0 − n using division rule for indices
= a−n simplifying the index.
n
a –n 1 1 a
In other words, ------- = ----n- and ------- = -----
1 a a –n 1
Note: Change the level, change the sign.

WORKED Example 5
Express each of the following with positive index numbers.
–4 x 4 y –2 × ( x 2 y ) –5
a  5--- b ---------------------------------------
 8 x –3 y 3

THINK WRITE

 5--- – 4 5 –4
a 1 Remove the brackets by raising the a  8
= ------
-
denominator and numerator to the 8 –4
power of −4.
84
2 Interchange the numerator and denominator, = ----4-
changing the signs of the powers. 5
4
8
3 Simplify by expressing as a fraction to =  ---
 5
the power of 4.
x 4 y –2 × ( x 2 y ) –5 x 4 y –2 × x –10 y –5
b 1 Remove the brackets by multiplying b ------------------------------------
- = -------------------------------------
powers. x –3 y 3 x –3 y 3
x –6 y –7
2 Collect terms with the same base by adding = --------------
-
the powers on the numerator and x –3 y 3
–6– ( – 3 ) y – 7 – 3
subtracting the powers on the denominator. = x
= x –3 y –10
1
3 Rewrite the answer with positive = ------------
-
powers. x 3 y 10
308 M a t h s Q u e s t M a t h s B Ye a r 1 1 f o r Q u e e n s l a n d

Rational powers
Until now, the indices have all been integers. In theory, an index can be any number.
We will confine ourselves to the case of indices which are rational numbers (fractions).
1
---
a n , where n is a positive integer, is defined as the nth root of a.
1
---
an = n a

For example, we know that a × a =a


1 1 1
--- --- --- + 1---
but a × a2 = a2
2 2

= a1
=a
1
---
Therefore, a = a2 .
1 1
--- ---
Similarly, 3 a = a3 , 4 a = a 4 . . . etc.
1
---
a n is defined for all a ≥ 0 if n is a positive integer.
In general, for any rational number,
m 1 m
----
an = (a ) ---
n = (n a)m = n
am

WORKED Example 6
Evaluate each of the following without a calculator.
3 3
--- 9  – --2-
 -----
a 16 2 b -
 25

THINK WRITE
a 1 Rewrite the base number in terms of its 3
---
3
---
a 16 2 = ( 2 4 ) 2
prime factors.
2 Remove the brackets by multiplying the = 26
powers.
3 Evaluate as a basic number. = 64
3 3
9 – --- 3 2 – ---
b 1 Rewrite the base numbers of the b  ------ 2 =  ----2- 2
 25 5 
fraction in terms of their prime factors.
3 –3
2 Remove the brackets by multiplying the = ------
-
powers. 5 –3

3 Rewrite with positive powers by 53


= ----3-
interchanging the numerator and 3
denominator.
4 Evaluate the numerator and 125
= ---------
denominator as basic numbers. 27
Chapter 7 Exponential and logarithmic functions 309
WORKED Example 7
Simplify the following, expressing your answer with positive indices.
3
a 3 64 × 4 512 b x2 y6 ÷ x3 y5

THINK WRITE

a 1 Write the expression. a 3 64 × 4 512


1 1
--- ---
2 Write using fractional indices. = 64 3 × 512 4
1 1
6 --- 9 ---
3 Write 64 and 512 in index form. = (2 )3 × (2 )4
9
2 ---
4 Multiply the powers. = 2 × 24
17
------
5 Simplify the powers. = 24

3
b 1 Write the expression. b x2 y6 ÷ x3 y5
1 1
--- ---
2 Express the roots in index notation. = ( x2 y6 )3 ÷ ( x3 y5 )2
2 3 5
--- --- ---
3 Remove the brackets by multiplying the powers. = x3 y2 ÷ x2 y2
2
--- – 3--- 2 – 5---
4 Collect terms with the same base by subtracting the powers. = x3 2 y 2

– 5--- – 1---
5 Simplify the powers. = x y 6 2

1
6 Rewrite with positive powers. = ---------
5 1
-
--- ---
x6 y2

remember
1
1. a−n = ----n- , a ≠ 0
1
a
---
n n
2. a = a
m 1
---- ---
n n n m
3. a = ( a )m = ( n a )m = a

7B Negative and rational powers

WORKED 1 Express each of the following with positive index numbers.


Example
5a a 6−3 b 5−4 c ( 3--- )−2 d ( 7--- )−5
5 4

e ( 1--- )−2 3 4 –4
9 f ( 64 –2 ) 3 g (−3)−1 h  ----3-
2 
310 M a t h s Q u e s t M a t h s B Ye a r 1 1 f o r Q u e e n s l a n d

WORKED 2 Simplify each of the following, expressing your answer with positive index numbers.
Example
5b ( – 2 ) 3 × 2 –4 ( x –2 ) 3 × ( y 4 ) –2 ( – m ) 2 × m –3
a --------------------------
- b -----------------------------------
- c --------------------------------
2 –3 x –5 × ( y –2 ) 3 ( p –2 ) –1 × p –4
x 5 ( x 4 ) –2 ( 3 –2 ) 2 × ( 2 –5 ) –1 x 3 y –2 × ( xy 2 ) –3
d ------- ÷ -------------- e -------------------------------------- f -------------------------------------
x –3 ( x 2 ) –3 ( 2 4 ) –2 × ( 3 4 ) –3 ( 2x 3 ) 2 × ( y –3 ) 2

WORKED 3 Evaluate the following without a calculator.


Example
6 1
---
1
---
1
---
1
---
a 92 b 27 3 c 625 4 d 256 8
2 3 4 1
eBook plus e 83
---
f 81 4
---
g 125 3
---
h 8  --3-
 --------
-
 125

Digital doc:
 16 ---
1
 25 ---
3
 27
2
--- – 2---
SkillSHEET 7.1
Negative and rational
i ------ 4
 81
j ------ 2
 16
k ------ 3
 64 l 32 5

powers

– 3--- 8  – --3-
2
16  – --2-
1
 125 – --- 1
m 81 4 n  -----
 27
- o  --------
 121
- p --------- 3
 216

4 multiple choice
a The exact value of 6−2 is:

A −12 B −36 C − 1--- D 1


------
1
E − -----
-
6 36 36

1
– ---
b The exact value of  27
------ 3 is:
 8

A − --2- B 6 C 2
--- D − --3- E 3
---
3 3 2 2

c 3 25 × 125 simplifies to:


5 7 3 11 13
--- --- --- ------ ------
A 25 6 B 56 C 52 D 56 E 56

WORKED 5 Simplify each of the following, expressing your answer with positive indices.
Example
7 2
---
1
--- – 3--- 9
---
a 9 × 3 81 b x3 × x6 c x 4 × x8
5 1 4
d x 2 ÷  x 3
--- ---
e 3
( xy 3 ) ÷ ( x 2 y ) f 5 32 × 4 8
 
1
---
5
– 1--- – 2--- – 1--- 2
– 5--- 18 2
g 2 ×4 2×8
---
4 3 h 27 4 × 9 × 3
---
3 4 i ---------------
4
-3
--- ---
9 × 44
3

4
---
2 3
( 64m 6 ) 3 x3
j ( 4 x3 ) × ( 3 x4 )
---
3
---
8 k -------------------- l ---------
4m –2 x
1
m ----------- ( x + 1 )2 o
1
n ------------------- x – -------
x –4 x+1 x
x – --2-
p x + 2 + ---------------- q ( y – 4) y – 4 r ( p + 3)( p + 3) 5
x+2
Chapter 7 Exponential and logarithmic functions 311
Indicial equations
1
---
We can solve equations of the form: x 3 = 2 as follows:
1 3
Take the cube of both sides:  x --3- = 23
 
The left-hand side becomes x, so x = 8.
However, when the unknown (or variable) is not a base number but is an index
number, a different approach is required.

Method 1: Exact solutions without a calculator


To attempt to solve index equations exactly, express both sides of the equation to the
same base and equate the powers.

If a m = a n, then m = n (unless a = -1, 0 or 1).

WORKED Example 8
Find the value of x in each of the following equations.
a 3x = 81 b 4x − 1 = 256 c 63x − 1 = 362x − 3

THINK WRITE

a 1 Write the equation. a 3x = 81


2 Express both sides to the same base. 3x = 34
3 Equate the powers. Therefore, x = 4.
b 1 Write the equation. b 4x − 1 = 256
2 Express both sides to the same base. 4x − 1 = 44
3 Equate the powers. Therefore, x − 1 = 4.
4 Solve the linear equation for x by x=5
adding one to both sides.
c 1 Write the equation. c 63x − 1 = 362x − 3
2 Express both sides to the same base. 63x − 1 = (62)2x − 3
3 Remove the brackets by multiplying the 63x − 1 = 64x − 6
powers.
4 Equate the powers. Therefore, 3x − 1 = 4x − 6
5 Subtract 3x from both sides to make x −1 = x − 6
the subject.
6 Add 6 to both sides to solve the x=5
equation.

More complicated equations can be solved using the same technique.


312 M a t h s Q u e s t M a t h s B Ye a r 1 1 f o r Q u e e n s l a n d

WORKED Example 9
Solve for n in the following equation:
23n × 16n + 1 = 32

THINK WRITE

1 Write the equation. 23n × 16n + 1 = 32


2 Express both sides using the same base, 23n × (24)n + 1 = 25
2.
3 Remove the brackets by multiplying the 23n × 24n + 4 = 25
powers.
4 Multiply the terms on the left-hand side 27n + 4 = 25
by adding the powers.
5 Equate the powers. Therefore, 7n + 4 = 5
6 Solve the linear equation for n. 7n = 1
n = 1---
7

In some cases indicial equations can be expressed in a quadratic form and solved
using the Null Factor Law. Look for numbers in index form similar to a2x and ax
appearing in different terms.

WORKED Example 10
Solve for x if 52x − 4(5x) − 5 = 0.

THINK WRITE

1 Write the equation. ()52x − 4(5x) − 5 = 0


2 Rewrite the equation in quadratic form. (5x)2 − 4(5x) − 5 = 0
Note that 52x = (5x)2.
3 Substitute y for 5x. Let y = 5x.
4 Rewrite the equation in terms of y. Therefore, y2 − 4y − 5 = 0.
5 Factorise the left-hand side. (y − 5)(y + 1) = 0
6 Solve for y using the Null Factor Law. Therefore, y = 5 or y = −1.
7 Substitute 5x for y. 5x = 5 or 5x = −1
8 Equate the powers. ⇒ 5x = 51 and 5x = −1 has no solution.
9 State the solution(s). ⇒x=1

Note that in step 8, the possible solution 5x = −1 was rejected because there is no
value of x for which it will be satisfied. Recall that exponential functions such as 5x are
always positive.
Chapter 7 Exponential and logarithmic functions 313
Method 2: Using a scientific calculator and
trial and error
Indicial equations which cannot have both sides expressed to the same base number do
not generally have exact, rational solutions. A trial and error method using a scientific
calculator can find solutions to a desired degree of accuracy.

WORKED Example 11
Solve 2x = 5 to 2 decimal places.

THINK WRITE
1 Write the equation. 2x = 5
2 Get a rough estimate of the solution. Since 22 = 4 and 23 = 8 then x is between 2 and 3.
3 Try x = 2.5 and evaluate 22.5. 22.5 = 5.657 — too big, so try 2.3
4 Repeat step 3 until an estimate of 22.3 = 4.925 — too small, so try x = 2.4
desired accuracy is found. 22.4 = 5.278 — too big, so try x = 2.35
22.35 = 5.098 — too big, so try x = 2.32
22.32 = 4.993 — too small, so try x = 2.33
22.33 = 5.028 — too big, so try x = 2.325
22.325 = 5.011 — too big
5 Select the value of x closest to 5. Since x = 2.320 is too small, and x = 2.325 is too
big, x = 2.32, to 2 decimal places.

Method 3: Solving indicial equations using


a graphics calculator
Consider the equation 2x = 5 shown in Worked example 11. We can solve this equation
using the Solver function on the Casio fx-9860G AU graphics calculator or the solve(
function on the TI-Nspire CAS.
For the Casio fx-9860G AU
1. Press MENU and then select EQUA.

2. Press F3 (SOLV) and then select F3: Solver. A


previous equation and its solution will be
displayed. Enter the required equation in the first
line and then press EXE .

3. Press F6 (SOLV) to display the solution. (If you


wish to use the Solver again, press F1 (REPT).)
314 M a t h s Q u e s t M a t h s B Ye a r 1 1 f o r Q u e e n s l a n d

For the TI-Nspire CAS


1. Open a new Calculator document. Press b and
select 3: Algebra followed by 1: Solve.
Alternatively, type the word solve and then press
( or press k and locate the solve( function.

2. Enter the equation to be solved and then the


variable, separated by a comma. Press ) to close
the set of brackets and then press ·. This
displays the solution as an exact value. The
function ln(x) is a logarithmic function that uses a
base other than 10.

3. Press /· to display the approximate value of


the solution.

remember
1. If a m = a n, then m = n (unless a = −1, 0 or 1).
2. Inexact solutions require the use of a calculator.

7C Indicial equations

WORKED 1 Solve for x in each of the following equations.


Example
8a
a 2x = 32 b 5x = 625 c 3x = 243
−x 1
d 10 = --------- e 4−x = 16 f 6x = --------
1
-
100 216
g 3−x = -----
1
- h 2 =1 −x
i x
8 =2 6
81
x
j 25 = 5 k 81x = 33 l 125x = 54

WORKED 2 Solve for n in each of the following equations.


Example
a 23n + 1 = 64 b 52n + 3 = 25 c 32 − n = 27
8b n+3
d 16 =2 3
e 495 − 3n = 1--- f 364n − 3 = 216
7

WORKED 3 Find x in each of the following.


Example
a 42x = 8x − 1 b 274 − x = 92x + 1 c 163x + 1 = 128x − 2
8c
d 252x − 3 = --------
1
- e 325 − x = 43x + 2 f 642 − 3x = 16x + 1
125
g 93x + 5 = --------
1
- h 164 − 3x = -------------
1
x+3
243 8
Chapter 7 Exponential and logarithmic functions 315
WORKED 4 Solve for x in each of the following equations.
Example
a 2x × 83x − 1 = 64 b 52x × 1253 − x = 25
9 4x
c 3 × 27 x+3
= 81 d 16x + 4 × 23 + 2x = 45x
81 2 – x
e 3125 × 252x + 1 = 53x + 4 f -------------- = 92x
27 x + 3
g 493 − 4x × 72x + 3 = 3432 − x h 57x + 5 ÷ 6252x + 1 = 253 − x
i 2562 − x × 43x + 1 = 64x − 1 36 2 x + 1 1
- = ---
j ----------------
216 x – 2 6
WORKED 5 Solve for x in each of the following.
Example
a 32x − 4(3x) + 3 = 0 b 22x − 6(2x) + 8 = 0
10 2x x
c 3(2 ) − 36(2 ) + 96 = 0 d 2(52x) − 12(5x) + 10 = 0
2x x
e 3(4 ) = 15(4 ) − 12 f 25x − 30(5x) + 125 = 0
x x
g 4 − 16(2 ) = −64 h 2(25x) + 10 = 12(5x)

6 multiple choice
Consider the indicial equation 32x − 12(3x) + 27 = 0. The equation can be solved by
making the substitution:
A y = 3x B y = 2x C y = 32x D y = 2x E y = 3x

7 multiple choice
The quadratic equation formed by the appropriate substitution in question 6 is:
A y2 − 3y + 27 = 0 B y2 − 11y + 27 = 0 C y2 + 12y + 27 = 0
2 2
D y − 12y + 27 = 0 E y − 9y + 3 = 0

8 multiple choice
The solutions to the equation in question 7 are x equals:
A 2 or 3 B 1 or 2 C 1 or 3 D 0 or 1 E 0 or 2
WORKED 9 Solve each of the following to 2 decimal places.
Example
a 2x = 3 b 2x = 12 c 10x = 45
11 x
d 3 =8 e 10x = 19 f 4x = 10

10 multiple choice
The nearest solution to the equation 3x = 10 is:
A x = 2.5 B x = 2.3 C x = 1.9 D x=2 E x = 2.1

Simulating radioactivity
Consider the imaginary element Braggium. It is an
unstable element and every hour there is a 1 in 6
chance that an atom will decay. We will use random
numbers to simulate the decay of Braggium.
Suppose that initially there are 100 atoms of
Braggium.
Generate a random number from 1 to 6 using a die,
a calculator or a spreadsheet. If the number is 6 this
means that the atom decays.
Repeat this process 100 times.
316 M a t h s Q u e s t M a t h s B Ye a r 1 1 f o r Q u e e n s l a n d

1 After one hour, how many atoms have decayed?

2 After one hour, how many atoms of Braggium remain?

What happens to this sample of 100 atoms by the end of the second hour? To
simulate the decay of atoms in the second hour, count the number of atoms not
yet decayed at the end of the first hour, n say. Then generate n random numbers
between 1 and 6. The number of 6s generated will give the number of atoms
that have decayed during the second hour.

3 Copy the following table and complete it by generating random numbers


between 1 and 6 as described above.

Time (t) Number of atoms Number of atoms


in hours remaining (N) decayed this hour

0 100

10

4 Use the data obtained in the table for t = 0 and t = 4 to devise a model for the
simulation. That is, find values of a and b in N = a bt.

5 Devise a theoretical model for the simulation. That is, find values of a and b in
N = a bt given that, in theory, 1 in 6 of the Braggium atoms decays each hour.

6 Compare the theoretical model devised in question 5 with the data. Which
value of N in the table differs most from the theoretical prediction obtained in
question 5?
Chapter 7 Exponential and logarithmic functions 317
Graphs of exponential functions
Functions of the form f (x) = a x, where a is a positive real number other than 1 and x is
a real number, are called exponential functions.
In general, there are two basic shapes for exponential graphs:
y = ax, a > 1 or y = ax, 0 < a < 1
y y
y = a x, a > 1

y = a x, 0 < a < 1
1 1

0 x 0 x
Increasing exponential Decreasing exponential
However, in both cases:
• the y-intercept is (0, 1) • the asymptote is y = 0 (x-axis)
• the domain is R • the range is R+.
x
Verify the shapes of these graphs by graphing, say y = 2x, y = 3x, y =  1---
 2
and
x
y=  1--- on a graphics calculator.
 3

The following sections on graphing exponential functions show the range of


variation of exponential graphs.

Reflections of exponential functions


The graph of y = a−x is obtained by The graph of y = −a x is obtained by
reflecting y = a x through the y-axis. reflecting y = a x through the x-axis.

y y
y= a x, a>1 y = a x, a > 1
1
y= a–x, a>1 0 x
1 –1
x y= –a x, a>1
0

Horizontal translations of exponential functions


The graph of y = a x + b is obtained by translating y = a x: y
1. b units to the right if b < 0 y = 2x y = 2x – 3

2. b units to the left if b > 0.


For example, the graph of y = 2x − 3 is obtained by 3 units
translating y = 2x to the right 3 units. 2
1
Check this graph using a graphics calculator. Note also –1 0 1 2 3 4 x
that 2x − 3 = (2x)(2−3) = ( 1--- )2x so that the effect is identical
8
to that of multiplying by a constant.
318 M a t h s Q u e s t M a t h s B Ye a r 1 1 f o r Q u e e n s l a n d

Vertical translations of exponential y y = 10 x


functions 10
The graph of y = a x + c is obtained by translating y = a x:
1. up by c units if c > 0 5
2. down by c units if c < 0. y = 10 x – 5
Furthermore the equation of the asymptote becomes y = c. 1
For example, the graph of y = 10 x − 5 is obtained –1 1
x
by translating y = 10 x down by 5 units. –5 units
–4
The equation of the asymptote is y = −5. (Asymptote)
–5
Check this graph using a graphics calculator.

WORKED Example 12
Find the equation of the asymptote and the y-intercept. Hence, sketch the graph of
y = 2x + 3 − 5 and state its domain and range.
THINK WRITE
1 Write the rule. y = 2x + 3 − 5
x
2 The graph is the same as y = 2
translated 3 units left and 5 units down.
3 State the asymptote. Asymptote is y = −5.
4 Evaluate y when x = 0 to find the When x = 0, y = 23 − 5
y-intercept. =3
Therefore, the y-intercept is (0, 3).
5 Locate the y-intercept and asymptote on y
y = 2x +3 – 5
a set of axes.
3
6 Sketch the graph of the exponential
function using the y-intercept and 0 x

asymptote as a guide.
–5

7 Use the graph to state the domain and Domain is R.


range. Range is (−5, ∞).

WORKED Example 13
Use a graphics calculator to solve 2x = 15 using the intersection of two graphs. Give the
answer rounded to 2 decimal places.
THINK DISPLAY/WRITE
x
1 To solve 2 = 15, we can draw the graphs of Let f1(x) = 2x and f2(x) = 15
two functions f1(x) and f2(x) and then find
where they intersect. Write the equations of
the two functions to be graphed.
2 Use a graphics calculator to draw the two
functions on the one set of axes and then
locate the coordinates of the intersection
point.
Chapter 7 Exponential and logarithmic functions 319
THINK DISPLAY/WRITE
For the Casio fx-9860G AU
1. Press MENU and then select GRAPH. Complete the
entry line for Y1 with 2x (press 2 ^ X,q,T ) and
then press EXE . Similarly, complete the entry line
for Y2 with 15 and then press EXE . To adjust the
style of the second line to look different to the first,
press F4 (STYL) and select one of the options.

2. Press F6 (DRAW) to display the graph. To obtain a


clearer view of the graph, you can adjust the View
Window settings. Press SHIFT F3 (V-WIN) and
adjust the values for Xmin, Xmax, Ymin and Ymax as
shown in the second screen.
3. Press F6 (DRAW) to display the graph with the new
settings.
4. To display the coordinates of the intersection point,
press SHIFT F5 (G-SLV) followed by F5 (ISCT).

For the TI-Nspire CAS


1. Open a new Graphs & Geometry document.
Complete the function entry line for f1(x) with 2x
(press 2lX) and then press ·. Similarly,
complete the function entry line for f2(x) with 15 and
then press ·. With the current Window Settings,
the graph for f2(x) cannot be seen.

2. To adjust the viewing window, press b, then select


4: Window followed by 1: Window Settings. Enter
the settings as shown. Use the e key to move to the
next line.

3. With OK highlighted, press · to display the two


graphs. To see more of the screen, press /G to
hide the function entry panel. (Press /G again to
bring the entry panel back.)

4. To display the coordinates of the intersection point,


press b and then select 6: Points & Lines followed
by 3: Intersection Point(s). Navigate the pointer to
each line and then press ·. (Press d to fix any
errors.)

3 Write the solution to 2 decimal places. Solution: x = 3.91


320 M a t h s Q u e s t M a t h s B Ye a r 1 1 f o r Q u e e n s l a n d

remember
General shapes of graphs of exponential functions:
If f (x) = a x, a > 1 If f (x) = a x, 0 < a < 1
y y

f(x) = ax, a > 1


f(x) = ax, 0 < a < 1
1 1

0 x 0 x

In both cases, the y-intercept is (0, 1)


the asymptote is y = 0
the domain = R
the range = R+.

Graphs of exponential
7D functions
1 Sketch the graph of each of the following on separate axes. (Use a table of values or
eBook plus
copy a graphics calculator screen).
Digital docs: a y = 3x b y = 5x c y = 6x
−x
SkillSHEET 7.2
Substitution in
d y = 10 x
e y=2 f y = 4−x
exponential functions g y = −3 x
h y = −2 x
i y = −3−x
EXCEL Spreadsheet j y = 0.5x k y = 2.7x l y = ( 2--- )x
Exponential functions 3

2 Sketch the following graphs, using a table of values or by copying a graphics calculator
screen. State the equation of the asymptote and the y-intercept for each.
a y = 2(3x) b y = 3(2x) c y = 0.5(4x)
x 1 x
d y = 4(5 ) e y = --- (2 ) f y = 4( 1--- )x
4 3

WORKED 3 Find the equation of the asymptote and the y-intercept for each of the following. Hence,
Example
sketch the graph of each and state its domain and range.
12
a y = 2x − 1 b y = 3x + 2 c y = 51 − x
d y=2 +3 x
e y=3 −3 x
f y = 2x + 3 − 1
g y = 6−x + 3 h y = 102 − x + 5 i y = 3x − 4 − 2
x+2
j y = −2 +1

4 multiple choice
a The rule for the graph at right is: y
A y = 3x − 2
3
B y = 3x 2
C y = 2x − 3 1
D y = 3x + 2 0 1 2 3 x
E y = 3x − 1
Chapter 7 Exponential and logarithmic functions 321
b The rule for the graph at right is: y
A y = 2x − 3
B y = 3x − 2
0 x
C y = 2x + 1 − 3
D y = 2x − 1 + 3 –2 (1, –2)
E y = 2x − 1 − 3 –3
–4

WORKED 5 Use a graphics calculator to solve the following indicial equations using the intersection
Example
13
of two graphs. Give answers rounded to 2 decimal places.
a 2x = 10 b 2x = 21 c 2x = 0.7
eBook plus d 10 x = 20 e 10 x = 8 f 10 x = 45
g 5x = 9 h 3x = 12 i 2x = x + 3
Digital doc:
WorkSHEET 7.1 j 3x = x + 4

A world population model

The statistics below describe P, the estimated world population (in billions) at
various times, t.
t 0 1000 1250 1500 1750 1800 1850 1900 1910
P 0.30 0.31 0.40 0.50 0.79 0.98 1.26 1.65 1.75

t 1920 1930 1940 1950 1960 1970 1980 1990 2000


P 1.86 2.07 2.30 2.52 3.02 3.70 4.45 5.30 6.23
(continued)
322 M a t h s Q u e s t M a t h s B Ye a r 1 1 f o r Q u e e n s l a n d

1 Use a graphics calculator or the Excel file ‘Exponential model for world
eBook plus
population’ located at www.jacplus.com.au to plot the data and fit an exponential
Digital doc: curve.
EXCEL Spreadsheet
Exponential model for If using a graphics calculator:
world population
For the Casio fx-9860G AU
1. Press MENU and then select STAT. Label List 1
as YR and List 2 as POP. Enter the data provided
in the table on page 321.

2. To graph these points as a scatterplot, press


F1 (GRPH), then F6 (SET) and ensure that
Graph Type is shown as Scatter (press
F1 (Scat)). Press EXE . Press F4 (SEL)
and ensure that StatGraph1 shows DrawOn
(with the other graphs Off). Press F6 (DRAW)
to display the scatterplot.

3. To find the exponential regression, press


F1 (CALC) and then F6 ( ) for more
options, followed by F3 (Exp). The r2 value
indicates that the model does not fit the data very
well. Note that the equation of the exponential
curve can be obtained from this screen.

4. Press F6 (DRAW) to display the exponential


curve that models the data.

5. Now consider modelling the data from 1750


onwards. Repeat the steps above to plot the new
set of data with its regression function. You
should see that the r2 value of 0.89 indicates a
much better fit.

For the TI-Nspire CAS


1. Open a new Lists & Spreadsheet document. Label
column A as yr and column B as pop. Enter the
data provided in the table on page 321.
Chapter 7 Exponential and logarithmic functions 323
2. To display these data points as a scatterplot, press
/I to insert a new screen. Select 2: Add
Graphs & Geometry. Press b and then select
4: Window followed by 1: Window Settings to
adjust the axes scales. (In this example, we have
XMin as −1, XMax as 2100, YMin as −2 and YMax
as 10.) Press /b and then select 4: Scatterplot.
Assign yr to x and pop to y. A scatterplot of the
data will be displayed.
3. Press / and then the left arrow on the NavPad to
return to the Lists & Spreadsheet document. Press
b and then select 4: Statistics, then 1: Stat
Calculations followed by A: Exponential
Regression. Adjust the Exponential Regression
screen options using the e key to move to the
next line. Set X List as yr, Y List as pop and 1st
Result Column as c[]. Note that the regression
equation will be saved as f1. Continue to press the
e key until OK is highlighted.

4. Press · to display the exponential regression


statistics. The r2 value indicates the model does not
fit the data very well. (The closer the value of r2 is
to 1, the better the fit.)

5. To draw the regression function with the original


data, return to the Graphs & Geometry document
(press / and then the right arrow on the NavPad).
Press /b and then select 1: Function. Press the
up arrow on the Navpad to display the function
entry line for f1(x). The equation for the regression
function will be shown. Press · to display the
regression curve. To see more of the work area,
press /G to hide the function entry line. (To
bring the function entry line back, press /G
again.)
6. Now consider modelling the data from 1750
onwards. Press /I to insert a new spreadsheet.
Repeat the steps above to draw the new regression
function. You should see that the r2 value of 0.89
indicates a much better fit.

2 Write the equation of the exponential curve which best models the data.
3 Use the equation for the curve to predict the world population in 2050.
4 What limitations are there on the use of the equation to predict future populations?
5 If using the Excel spreadsheet, comment on the effect of each part of the
equation on the shape of the graph.
324 M a t h s Q u e s t M a t h s B Ye a r 1 1 f o r Q u e e n s l a n d

Bode’s Law
In 1772, Johann Bode discovered a curious
relationship between pure numbers and the distance
of planets from the Sun.
His law consisted of a simple formula relating the
number of the planet to its distance from the Sun.
The actual distances of the planets from the Sun
are given in the table below. By graphing the
distance against 2 raised to the power of the planet
number, discover the relationship that Bode found.
(Hint: Use either a spreadsheet or a graphics
calculator to graph the data and then find the
regression line.)

Planet Distance in AU (1 AU = distance


number Planet from the Earth to the Sun)
0 Mercury 0.39
1 Venus 0.72
2 Earth 1
3 Mars 1.52
4 Ceres (dwarf planet) 2.77
5 Jupiter 5.2
6 Saturn 9.54
7 Uranus 19.18
8 Pluto (dwarf planet) 39.4

(Note: The discovery of Neptune and large bodies in the Kuiper Belt such as Eris
discredited Bode’s Law in the eyes of many astronomers. If Neptune’s orbit
actually falls between Uranus and Pluto at a distance of 30.1 AU, what do you
notice about its relationship with Pluto and Bode’s Law?)

Logarithms x
Logarithm
The index, power or exponent (x) in the indicial y=a
equation y = a x is also known as a logarithm. Base numeral
Base
x
This means that y = a can be written in an alternative form: log a y = x
which is read as ‘the logarithm of y to the base a is equal to x’.
For example, 32 = 9 can be written as log 3 9 = 2.
105 = 100 000 can be written as log 10 100 000 = 5.
In general, for a > 0 and a ≠ 1: a x = y is equivalent to x = log a y.
Using the indicial equivalent, it is possible to find the exact value of some logarithms.
Chapter 7 Exponential and logarithmic functions 325
WORKED Example 14
Evaluate the following without a calculator.
a log 6 216 b log 2 ( 1--8- )

THINK WRITE

a 1 Let x equal the quantity we wish to find. a Let x = log 6 216


2 Express the logarithmic equation as an 6x = 216
indicial equation.
3 Express both sides of the equation to the same 6x = 63
base.
4 Equate the powers. x=3
b 1 Write the logarithm as a logarithmic equation. b Let x = log2 ( 1--- )
8
2 Express the logarithmic equation as an 2x = 1---
8
indicial equation. =  1--- 3
 2
= ( 2–1 ) 3
3 Express both sides of the equation to the same 2x = 2−3
base.
4 Equate the powers. x = −3

Logarithm laws
The index laws can be used to establish corresponding rules for calculations involving
logarithms. These rules are summarised in the following table.

Name Rule Restrictions

Logarithm of a product log a (mn) = log a m + log a n m, n > 0


a > 0, a ≠ 1

Logarithm of a quotient log a  m


---- = log a m – log a n m, n > 0
n
a > 0 and a ≠ 1

Logarithm of a power log a mn = n log a m m>0


a > 0 and a ≠ 1

Logarithm of the base log a a = 1 a > 0 and a ≠ 1


Logarithm of one log a 1 = 0 a > 0 and a ≠ 1

It is important to remember that each rule works only if the base, a, is the same for
each term. Note that it is the ‘logarithm of a product’ and ‘logarithm of a quotient’
rules that formed the basis for the pre-1970s calculation device for multiplication and
division — the slide rule.
326 M a t h s Q u e s t M a t h s B Ye a r 1 1 f o r Q u e e n s l a n d

WORKED Example 15
Simplify, and evaluate where possible, each of the following without a calculator.
a log 10 5 + log 10 4 b log 2 12 + log 2 8 − log 2 3
THINK WRITE
a 1 Apply the ‘logarithm of a product’ rule. a log10 5 + log 10 4
= log 10 (5 × 4)
2 Simplify. = log 10 20
b 1 Multiply the base numerals of the logs being b log 2 12 + log 2 8 − log 2 3
added since their bases are the same. = log 2 (12 × 8) − log 2 3
2 Apply the ‘logarithm of a quotient’ law. = log 2 (96 ÷ 3)
3 Simplify, noting that 32 is a power of 2. = log 2 32
= log 2 25
4 Evaluate using the ‘logarithm of a power’ and = 5 log 2 2
‘logarithm of the base’ laws. =5

WORKED Example 16
Simplify 3 log 2 5 − 2 log 2 10.
THINK WRITE
1 Express both terms as logarithms of 3 log 2 5 − 2 log 2 10 = log 2 53 − log 2 102
index numbers.
2 Simplify each logarithm. = log 2 125 − log 2 100
3 Apply the ‘logarithm of a quotient’ law. = log 2 (125 ÷ 100)
4 Simplify. = log 2( 5--4-) or log 2 1.25

WORKED Example 17
Simplify each of the following.
log 8 49
a --------------------- b 2 log 10 x + 1
log 8 343
THINK WRITE
2
log 8 49 log 8 7
a 1 Express each base numeral as powers a --------------------
- = ---------------- -
to the same base, 7. log 8 343 log 7 3
8
2 log 8 7
2 Apply the ‘logarithm of a power’ law. = -------------------
3 log 8 7
3 Simplify by cancelling out the common = 2---
3
factor of log 8 7.
2
b 1 Express 2 log 10 x as log 10 x and 1 as a b 2 log 10 x + 1 = log 10 x2 + log 10 10
logarithm to base 10 also.
2 Simplify using the ‘logarithm of a = log 10 10x2
product’ law.
Chapter 7 Exponential and logarithmic functions 327
remember
1. If y = a x then log a y = x where a = the base, x = the power, index or logarithm
and y = the base numeral. Note that a > 0, a ≠ 1, and therefore y > 0.
2. Log laws:
m
(a) log a m + log a n = log a (mn) (b) log a m − log a n = log a  ----
 n
(c) log a mn = n log a m (d) log a a = 1
(e) log a 1 = 0

7E Logarithms

1 Express the following indicial equations in logarithmic form.


a 23 = 8 b 35 = 243 c 50 = 1
−2
d 0.01 = 10 n
e b =a f 2−4 = -----
1
-
16
2 Express the following logarithmic equations in indicial form.
a log 4 16 = 2 b log 10 1 000 000 = 6 c log 2 1--- = −1
2
d log 3 27 = 3 e log 5 625 = 4 f log 2 128 = 7
g log 3 1--- = −2 h log b a = x
9

3 multiple choice
The value of log 5 25 is:
A −2 B 5 C 1 D 2 E 4

4 multiple choice
When expressed in logarithmic form, 83 = 512 is:
A log 3 8 = 512 B log 3 512 = 8 C log 8 512 = 3
D log 512 3 = 8 E log 8 3 = 512

5 multiple choice
When expressed in indicial form, log 10 10 000 = 4 is:
A 104 = 10 000 B 10 0004 = 10 C 10 00010 = 4
10 000
D 10 =4 E 410 = 10 000

WORKED 6 Evaluate each of the following without a calculator.


Example
14 a log 2 16 b log 3 81 c log 5 125 d log 2 1
---
4
1
e log 10 1000 f log 10 (0.000 01) g log 2 0.25 h log 3 --------
-
243
i log 2 32 j log 2 1
------ k log 3 (−3) l log n n5
64

WORKED 7 Simplify, and evaluate where possible, each of the following without a calculator.
Example
15
a log 2 8 + log 2 10 b log 3 7 + log 3 15 c log 10 20 + log 10 5
d log 6 8 + log 6 7 e log 2 20 − log 2 5 f log 3 36 − log 3 12
g log 5 100 − log 5 8 h log 2 1--- + log 2 9 i log 4 25 + log 4 1---
3 5
328 M a t h s Q u e s t M a t h s B Ye a r 1 1 f o r Q u e e n s l a n d

j log 10 5 − log 10 20 k log 3 4--- − log 3 1--- l log 2 9 + log 2 4 − log 2 12


5 5
m log 3 8 − log 3 2 + log 2 5 n log 4 24 − log 4 2 − log 4 6

WORKED 8 Simplify each of the following.


Example
16 a 3 log 10 5 + log 10 2 b 2 log 2 8 + 3 log 2 3
c 2 log 3 2 + 3 log 3 1 d log 5 12 − 2 log 5 2
e 4 log 10 2 + 2 log 10 8 f log 3 42 + 3 log 3 2
1 1
g --- log 2 27 − --- log 2 36 h log 2 (x − 4) + 3 log 2 x
3 2
1
i --- log 3 16 + 2 log 3 4 j 2 log 10 (x + 3) − log 10 (x − 2)
2

WORKED 9 Simplify the following.


Example
17a log 3 25 log 2 81 log 4 36
a --------------------
- b -----------------
- c ------------------
log 3 125 log 2 9 log 4 6
2 log 10 8 3 log 5 27 4 log 3 32
d --------------------
- e ---------------------- f ----------------------
log 10 16 2 log 5 9 5 log 3 4
3
6 3 ---
log 3 x log 10 x log 5 x
2
g ----------------
-
2
h --------------------
- i ------------------
-
log 3 x log 10 x log 5 x
2 log 2 ( x + 1 ) 3
j -----------------------------------
log 2 ( x + 1 )

10 multiple choice
The expression log 10 xy is equal to:
log 10 x
A log 10 x × log 10 y B log 10 x − log 10 y C ----------------
-
log 10 y
D y log 10 x E log 10 x + log 10 y

11 multiple choice
The expression log 5 xy is equal to:
A x log 5 y B y log 5 x C 5 log x y
D log 5 x + log 5 y E 5y

12 multiple choice
1
The expression --- log 2 64 + log 2 5 can be simplified to:
3

A log 2 40 B 1 C log 2 64
------ D log 2 20 E log 2 320
---------
15 3

13 multiple choice
5
log 4 x
The expression ----------------
- can be simplified to:
2
log 4 x
5
---
A log 4 x3 B log4 x 2 C 5
--- D log 4 (x5 − x2) E log 4 x7
2
Chapter 7 Exponential and logarithmic functions 329
WORKED 14 Express each of the following in simplest form:
Example
17b a log 3 27 + 1 b log 4 16 + 3 c 3 log 5 2 − 2
d 2 + 3 log 10 x e 2 log 2 5 − 3 f 4 log 3 2 − 2 log 3 6 + 2
g 2 log 6 6 − log 6 4 h 1
--- + 3 log 10 x2
2

Solving logarithmic equations


Logarithms to the base 10
Logarithms to the base 10 are called common logarithms and can be evaluated using
the LOG function on a calculator.
For example, to evaluate log 10 8, correct to 3 decimal places:
1. Press 8 LOG on the calculator and then press ENTER . (On some calculators,
press LOG 8 .)
2. The display shows 0.903 089 887.
This means log10 8 = 0.903 to 3 decimal places, or 100.903 ≈ 8.
When solving logarithmic equations involving bases other than 10 consider the
following steps:
Step 1 Simplify the equation using logarithm laws.
Step 2 Express the equation in index form if required.
Step 3 Solve by:
(a) evaluating if the base numeral is unknown
(b) equating the powers if possible
(c) equating the bases if possible.
Note: The logarithm of a negative number or zero is not defined. Therefore:

log a x is defined for x > 0, if a > 0

This can be seen more clearly using index notation as follows:


Let n = log a x.
Therefore, a n = x (indicial equivalent of logarithmic expression).
However, a n > 0 for all values of n if a > 0 (positive based exponentials are always
positive). Therefore, x > 0.

WORKED Example 18
Find x if log 3 9 = x − 2.
THINK WRITE
1 Write the equation. log 3 9 = x − 2
2 Simplify the logarithm using the log 3 32 = x – 2
‘logarithm of a power’ law and the fact 2 log 3 3 = x – 2
that log 3 3 = 1. 2=x–2
3 Solve for x by adding 2 to both sides. x=4
330 M a t h s Q u e s t M a t h s B Ye a r 1 1 f o r Q u e e n s l a n d

WORKED Example 19
Solve for x if log 6 x = −2.
THINK WRITE
1 Write the equation. log 6 x = −2
2 Express in index form. Therefore, x = 6−2
1
3 Evaluate the index number.
x = -----
2
6
1
= ------
36

WORKED Example 20
Find x if 2 logx 25 = 4, x > 0.
THINK WRITE
1 Write the equation. 2 log x 25 = 4
2 Divide both sides by 2. log x 25 = 2
3 Write as an index equation. Therefore, x2 = 25
4 Express both sides of the equation to the same base, 5. x2 = 52
5 Equate the bases.
Note that x = −5 is rejected as a solution, because x > 0. x=5

Solving exponential equations using log 10 on the


calculator
We have already seen three methods for solving exponential equations:
1. equating the bases, which is not always an option, for example, 2x = 7
2. using a calculator and trial and error, which can be time consuming
3. using a graphical technique and a graphics calculator.
An efficient method for solving equations involves the use of logarithms and the
log 10 function on the calculator. This is outlined in the following example.

WORKED Example 21
Solve for x, correct to 3 decimal places, if 2x = 7.
THINK WRITE

1 Write the equation. 2x = 7


2 Take log 10 of both sides. log 10 2x = log 10 7
3 Use the ‘logarithm of a power’ law to bring the x log 10 2 = log 10 7
power, x, to the front of the logarithmic equation.
log 10 7
4 Divide both sides by log 10 2 to get x by itself. Therefore, x = ---------------
log 10 2
0.8451
5 Evaluate the logarithms correct to 4 decimal places, at = ----------------
0.3010
least one more than the answer requires.
6 Solve for x. x = 2.808
Chapter 7 Exponential and logarithmic functions 331
Therefore, we can state the following rule:
log 10 b
If a x = b, then x = -----------------
log 10 a
This rule applies to any base, but since most calculators use base 10, this is the most
commonly used base for this solution technique.

remember
1. Logarithmic equations are solved more easily by:
(a) simplifying using log laws
(b) expressing in index form
(c) solving as required.
log 10 b
2. If a x = b, then x = ----------------
-.
log 10 a

7F Solving logarithmic equations

WORKED 1 Find x in each of the following.


Example
18 a log 2 4 = x b log 9 1 = x c log 3 27 = x d log 4 256 = x
1
e log 10 -----
- = x f log 3 1--- = x g 2 log 2 8 = x h log 3 81 = 2x
10 9
i log 10 1000 = 2x − 1 j 2 log 2 32 = 3x + 1
WORKED 2 Solve for x.
Example
19 a log 2 x = 3 b log 3 x = 2 c log 5 x = 4 d log 10 x = 1
e log 8 x = −1 f log 3 x = −3 g log 2 x = 6 h log 10 x = 4
i log 3 (x − 3) = 3 j log 2 (3x + 1) = 4 k log 10 (2x) = 1 l 2 log 6 (3x) = 1
m log 5 x = log 5 4 + log 5 6 n log 3 5 − log 3 4 = log 3 x – log 3 8
WORKED 3 Solve for x given that:
Example
20 a log x 36 = 2 b log x 125 = 3 c 3 log x 16 = 6
1
d −2 log x --------
- = 4 e 1
--- log x 64 = 3 f 5 log x 625 = 10
100 2
g log x + 1 27 = 3 1
h −log 3x − 1 -----
- = 5
32

4 multiple choice
a The solution to the equation log 7 343 = x is:
A x=2 B x=3 C x=1 D x=0 E x = −2
b If log 8 x = 4, then x is equal to:
1
A 4096 B 512 C 64 D 2 E ---
2
c Given that log x 3 = 1--- , x must be equal to:
2
A 3 B 6 C 81 D 1 E 9
d The solution to the equation log 3 x − 2 = log 3 (x − 8) is:
A x=8 B x=6 C x=9 D x = −4 E x=2
332 M a t h s Q u e s t M a t h s B Ye a r 1 1 f o r Q u e e n s l a n d

WORKED 5 Solve the following equations correct to 3 decimal places.


Example
21 a 2x = 11 b 2x = 0.6 c 3x = 20
d 3x = 1.7 e 5x = 8 f 0.7x = 3
g 10 x − 1 = 18 h 3x + 2 = 12 i 22x + 1 = 5
j 43x + 1 = 24 k 10−2x = 7 l 82 − x = 0.75

6 multiple choice
The nearest solution to the equation 4x = 5 is:
A x = 0.86 B x = 1.2 C x = 1.25 D x=1 E x = 0.5

eBook plus 7 multiple choice

Digital doc:
The nearest solution to the equation 0.62x − 1 = 2 is:
WorkSHEET 7.2 A x = 0.18 B x = 0.13 C x = −0.18 D x = −0.71 E x = −0.13

Logarithmic graphs
1 Using a graphics calculator or graphing software, produce graphs of the
following on the same set of axes. Ensure equal axis scales if possible.
If using a Casio fx-9860G AU graphics calculator, press SHIFT F6
(ZOOM), then press F6 ( ) for more options followed by F2 (SQR).
If using a TI-Nspire CAS graphics calculator, press b and then select
4: Window followed by 5: Zoom-Standard. (This is the default setting.)
Copy the screen view into your workbook.
a y = log10x b y = 10x c y=x

2 Copy and complete:


a The graph of y = log10 x is the r of the graph of y = 10x in the line
y = x. Such functions are called inverses of each other.
b An asymptote is a line that a graph never quite intersects. The line x =
is an asymptote for the graph of y = log10 x.

3 Use technology to investigate the shape of the graph of


y = Aloga (x + b) + B
for various values of the pronumerals A, a, b and B.
Sketch several examples into your workbook, showing asymptotes. The
eBook plus
Excel spreadsheet ‘Logarithmic graphs’ is ideal for this.
Digital doc: a What is the effect of A on the graph?
EXCEL Spreadsheet
Logarithmic graphs
b What is the effect of a on the graph?
c What is the effect of b on the graph?
d What is the effect of B on the graph?
Chapter 7 Exponential and logarithmic functions 333
4 Try sketching graphs of the following without using technology.
(Hint: Find x- and y-intercepts by substituting y = 0 and x = 0 respectively.)
a y = 2 log10 (x – 3) + 1
b y = 3log10 (x + 2) – 5
c y = –log10 x – 2

eBook plus Further work on logarithmic graphs is available by logging into www.jacplus.com.au
and locating the ‘Extension — Logarithmic graphs’ weblink.
Digital doc:
Logarithmic graphs

The slide rule


The logarithmic slide rule is a compact
device for rapidly performing calculations
with limited accuracy. The invention of
logarithms in 1614 by John Napier made it
possible to multiply and divide numbers
by the more simple operations of addition
and subtraction.
In this investigation we will construct
a primitive slide rule.
334 M a t h s Q u e s t M a t h s B Ye a r 1 1 f o r Q u e e n s l a n d

Number Power of 2
0.25 −2
0.5 −1
1 0
2 1
4 2
8 3
16 4
32 5
64 6

1 Take two strips of card about


3 cm by 20 cm. Mark both cards
as shown, using the numbers 0.25 0.5 1 2 4 8 16 32 64
from the table above.
You will notice that the scale
0.25 0.5 1 2 4 8 16 32 64
used is a logarithmic scale using
2 as a base. That is, the distance
from 1 to 8 is 3 units (log2 (8) = 3).
Also, the distance from 1 to 0.25 is −2 (log2 (0.25) = −2).
To multiply two numbers we need only to add the powers so that
8 × 4 ➞ 23 × 22 ➞ 25 ➞ 32.
Thus, multiplying 8 by 4 is equivalent to adding 2 and 3. The operation of
multiplication is converted to addition. Your ‘slide rule can be used to perform
this addition.

0.25 0.5 1 2 4 8 16 32 64

0.25 0.5 1 2 4 8 16 32 64

Line 1 up with Read the result by reading


the first factor. the number corresponding

8 × 4 = 32
to the second factor.

This slide rule is quite primitive and in its present form you would not use it to
multiply 5 by 10. However, this principle provided the basis for scientific
calculations before the advent of the electronic calculator in the 1960s and 1970s.
2 Use your slide rule to calculate 0.25 × 32.
3 Use your slide rule to calculate 32 ÷ 4. (Remember, division corresponds to a
subtraction of exponents.)
4 Construct a base 10 slide rule.
Chapter 7 Exponential and logarithmic functions 335
Applications of exponential and
logarithmic functions
Exponential and logarithmic functions can be used to model many practical situations
in science, medicine, engineering and economics.

WORKED Example 22
A square sheet of paper which is 0.1 mm thick is repeatedly folded in half.
a Find a rule which gives the thickness, T mm, as a function of the number of folds, n.
b What is the thickness after 10 folds?
c How many folds are required for the thickness to reach 6 cm?
THINK WRITE
a 1 T = 0.1 when n = 0 and doubles with a When n = 0, T = 0.1 and as n increases by 1,
each fold. This doubling implies that T doubles.
the base should be 2.
2 Complete a table of values showing
n 0 1 2 3 4 5
the thickness, T, for values of n from
0 to 5. T 0.1 0.2 0.4 0.8 1.6 3.2
n
3 Determine the rule for T(n). There is a T(n) = 0.1(2 )
doubling term (2n) and a multiplying
constant for the starting thickness (0.1).
4 Compare the rule for T(n) against the
table of values in step 2.

b 1 Substitute n = 10 into the formula b When n = 10,


for T. T(10) = 0.1(210)
2 Calculate T. T = 102.4 mm

c 1 Change 6 cm to millimetres. c 6 cm = 60 mm
2 Substitute T = 60 into the formula. When T = 60,
60 = 0.1 (2n)
3 Divide both sides by 0.1. 600 = 2n
4 Take log 10 of both sides. log 10 600 = log 10 2n
5 Use the ‘logarithm of a power’ law log 10 600 = n log 10 2
to bring the power n to the front of
the logarithm.
log 10 600
6 Divide both sides by log 10 2. n = ----------------------
-
log 10 2
7 Evaluate. n = 9.23
8 Round the answer up to the nearest Therefore, n = 10 folds.
whole number since the number of
folds are positive integers and if you
round down, the thickness will not
have reached 60 mm.
336 M a t h s Q u e s t M a t h s B Ye a r 1 1 f o r Q u e e n s l a n d

WORKED Example 23
An investment broker develops a model
for the share price of a mining company
that is based on the price of gold.
S = 47.2 log (G) − 93.23
where S is the share price of the stock
and G is the price of gold.
According to this model,
a what is the share price when the
price of gold is $900?
b when will the share price exceed $50?

THINK WRITE

a 1 State the modelling function. a S = 47.2 log (G) – 93.23

2 Substitute 900 for G into the modelling S = 47.2 log (900) – 93.23
function. = 46.21
3 Write the answer using a sentence. When the price of gold is $900, the model
predicts that the share price of the stock
will be $46.21.

b 1 Substitute 50 for S in the equation and b 50 = 47.2 log (G) – 93.23


solve to find the value of log (G).
2 Simplify by isolating the logarithm part 47.2 log(G) = 50 + 93.23
of the equation. 143.23
log (G) = ----------------
47.2
log (G) = 3.0345
3 Express this equation in its equivalent G = 103.0345
indicial form and calculate G. = 1082.76
4 Write the answer. The model predicts that the share price of
the stock will exceed $50 when the price
of gold exceeds $1082.76.

remember
1. Read the question carefully.
2. Use the skills developed in the previous sections to answer the question being
asked.
Chapter 7 Exponential and logarithmic functions 337
Applications of exponential
7G and logarithmic functions
WORKED 1 Prior to a mice plague which lasts 6 months, the population of mice in a country
Example
22 region is estimated to be 10 000. The mice population doubles every month during the
plague. If P represents the mice population and t is the number of months after the
plague starts:
a express P as a function of t
b find the population after: i 3 months ii 6 months
c calculate how long it takes the population to reach 100 000 during the plague.
WORKED 2 The population of a town, N, is modelled by the function N = 15 000(20.01t ) where t is
Example
23
the number of years since 1990.
a Find the population in 1990.
b Find the population in: i 1995 ii 2000.
c What is the predicted population in 2015?
d In what year will the population reach 20 000?
3 The weight of a baby, W kg, t weeks after birth can be modelled by W = 3 log 10 (8t + 10).
a Find the initial weight.
b Find the weight after: i 1 week ii 5 weeks iii 10 weeks.
c Sketch the graph.
d When will the baby reach a weight of 7 kg?
4 If $A is the amount an investment of $P grows to after n years at 5% p.a.:
a write A as a function of P
b use the function from a to find the value of $10 000 after 10 years
c calculate how many years it will be until an investment of $10 000 reaches $26 500.
5 The value of a car, $V, decreases according to the function V = 25 000  2--- 0.1t.
5
a Find the value of the car when new.
b Find the value of the car after 6 years.
c In how many years will the car be worth $10 000?
6 The temperature, T (°C), of a cooling cup of
coffee in a room of temperature 20°C can be
modelled by T = 90(3−0.05t ), where t is the
number of minutes after it is poured.
a Find the initial temperature.
b Find the temperature: i 3 minutes after
pouring ii 6 minutes after pouring.
c How long is it until the temperature
reaches half its initial value?
7 A number of deer, N, are introduced to a reserve and its population can be predicted
by the model N = 120(1.1t ), where t is the number of years since introduction.
a Find the initial number of deer in the reserve.
b Find the number of deer after: i 2 years ii 4 years iii 6 years.
c How long does it take the population to treble?
d Sketch the graph of N versus t.
e Explain why the model is not reliable for an indefinite time period.
338 M a t h s Q u e s t M a t h s B Ye a r 1 1 f o r Q u e e n s l a n d

8 After a recycling program is


introduced the weight of rubbish
disposed of by a household each
week is given by W = 80(2−0.015t ),
where W is the weight in kg and t is
the number of weeks since
recycling was introduced.
a Find the weight of rubbish
disposed of before recycling
starts.
b Find the weight of rubbish
disposed of after recycling
has been introduced for:
i 10 weeks ii 40 weeks.
c How long is it after recycling starts until the weight of rubbish disposed of is half
its initial value?
d ii Sketch the graph of W versus t.
ii Will the model be realistic in 10 years’ time? Explain.
9 The number of hectares (N) of forest land destroyed by fire t hours after it started is
given by N = 40 log 10 (500t + 1).
a Find the amount of land destroyed after:
iii 1 hour ii 2 hours iii 10 hours.
b How long does the fire take to burn out 155 hectares?
10 A discus thrower competes at several competitions during the
year. The best distance, d metres, that he achieves at each
consecutive competition is modelled by d = 50 + log 10 (15n),
where n is the competition number.
a Find the distance thrown at the:
i 1st ii 3rd iii 6th iv 10th competition.
b Sketch the graph of d versus n.
c How many competitions does it take for the thrower to reach
a distance of 53 metres?
11 The population, P, of a certain fish t months after being introduced to a reservoir is
P = 400(100.08t ), 0 ≤ t ≤ 20. After 20 months, fishing is allowed and the population is
then modelled by P = 15 000 + 924 log 10 [10(t − 19)], t ≥ 20.
a Find the initial population.
b Find the population after:
i 5 months ii 15 months iii 25 months iv 40 months.
c How long does it take the population to pass 10 000?
7
12 A ball is dropped from a height of 5 metres and rebounds to -----
- of its previous height.
10
a Find the rule that describes the height of the ball (h metres) after n bounces.
b Find the height after: i 4 bounces ii 8 bounces.
c Sketch the graph of the height of the ball after n bounces.
13 A painting appreciates in value by 10% per year. If the painting costs $5000 when
new, find:
a the rule describing the value, V, of the painting at any time, t years, after purchase
b the value of the painting after 6 years
c the number of years it takes to reach double its original value.
Chapter 7 Exponential and logarithmic functions 339
At the beginning of this chapter we discussed the problem of dealing, in a mean-
ingful way, with the large numbers that arose in the measurement of sound. We are now
in a position to propose a solution to this problem. The solution involves the use of a
logarithmic scale.

The decibel
The loudness of a sound, L, is measured in decibels (dB) and is defined as follows:
I
L = 10 log10 ----
I0

where I is the sound intensity measured in watts per square metre (W/m2) and I0 is
the threshold of hearing and has a value of 1 × 10−12 W/m2.
Intensity is not widely used to measure sound because of the difficulty identified
in the introduction to this chapter; that is, the range in magnitude from the
threshold of human hearing (1 × 10−12 W/m2) to the sound of a jackhammer at a
distance of 10 m (1.5 × 10−3 W/m2).
The jackhammer noise is 1.5 thousand million times as intense as the softest
sound. Such numbers are difficult to comprehend.
The human brain can deal with numbers that range from 0 to 150, whereas it
would struggle with 1.5 thousand million. It is the logarithmic scale that converts
numbers that have a large range to those that are meaningful to us.
In practical situations, we are usually interested in the effect of sound intensity
on people. Clearly, a sound level drops as we move away from its source. Measures
of loudness may therefore need to show the distance between the source of sound
and the observer. Consider the following data.

Sound Loudness in dB

Jet engine 150

Jackhammer 90

Heavy traffic 75

Conversational speech 60

Quiet living room 20

1 The threshold of pain for hearing is 135 dB. How many times as loud as a
jackhammer is the pain threshold?
2 Compare the intensity of the sound of a conversation with that of heavy traffic.
3 How many times is the sound of a quiet living room as loud as that of the
threshold of human hearing?
340 M a t h s Q u e s t M a t h s B Ye a r 1 1 f o r Q u e e n s l a n d

The Richter scale


The Richter scale is used to describe the ‘strength’ of earthquakes. A formula for
the Richter scale is:
R = 2--3- log K – 0.9, where R is the Richter scale value for an earthquake that
releases K kilojoules (kJ) of energy.

1 Find the Richter scale value for an earthquake that releases the following
amounts of energy:
a 1000 kJ b 2000 kJ c 3000 kJ
d 10 000 kJ e 100 000 kJ f 1 000 000 kJ.
2 Does doubling the energy released double the Richter scale value?
3 Find the energy released by an earthquake of:
a magnitude 4 on the Richter scale
b magnitude 5 on the Richter scale
c magnitude 6 on the Richter scale.
4 What is the effect (on the amount of energy released) of increasing the Richter
scale value by 1?
5 Why is an earthquake measuring 8 on the Richter scale so much more
devastating than one that measures 5?
Chapter 7 Exponential and logarithmic functions 341

summary
Index laws
• am × an = am + n
• am ÷ an = am − n
• (a m)n = a mn
• a0 = 1
• (ab)n = a nbn
a n an
•  --- = ----n-
 b b

Negative and rational powers


1
• a –n = ----n- , a ≠ 0
a
1
---
• an = n a
m m
--1-
• a =
----
n (a )n = (n a)m = n
am

Indicial equations
• If a m = a n, then m = n (unless a = −1, 0 or 1).
• A graphics calculator may be used to solve indicial equations, using the solve(
function.

Graphs of exponential functions


• If f (x) = a x, a > 1
y

f(x) = ax, a > 1


1

0 x

• If f (x) = a x, 0 < a < 1


y

f(x) = ax, 0 < a < 1


1
0 x

• y-intercept is (0, 1)
• Asymptote is y = 0 (x-axis)
• Domain = R
• Range = R+
342 M a t h s Q u e s t M a t h s B Ye a r 1 1 f o r Q u e e n s l a n d

Logarithms
• If y = a x then log a y = x where a = the base, x = the power, index or logarithm and
y = the base numeral.
Log laws:
• log a m + log a n = log a (mn) m, n > 0
m
• log a m − log a n = log a  ---- m, n > 0
 n
• log a mn = n log a m m > 0
• log a a = 1
• log a 1 = 0

Solving logarithmic equations


• Logarithmic equations are solved more easily by:
1. simplifying using log laws
2. expressing in index form
3. solving as required.
log 10 b
• If a x = b, then x = ----------------
-.
log 10 a

Logarithmic graphs
• The logarithmic function f (x) = log a x is the inverse function of the exponential
function g(x) = a x.
y
g(x) = a x , a > 1

y=x

1 f(x) = log ax, a > 1

0 1 x

• x-intercept is (1, 0)
• Asymptote is y = 0
• Domain = R+
• Range = R
Chapter 7 Exponential and logarithmic functions 343

CHAPTER
review
1 multiple choice
7A
( 2xy 3 ) 2 3x 5 y 2
- × -------------- is equal to:
When simplified, ------------------
7x 3 4y
x4 y7 3x 4 y 7 3y 7 3x 4 x
E --
A ---------- B -------------- C -------
- D -------
-
7 7 x2 y6 y

2 multiple choice
7A
5m 4 p 2 ( 5m 2 p 6 ) 3
---------------- ÷ ----------------------- may be simplified to:
2m 3 p 3m 7 p
m2 3m 10 3m 2 m 15 m 20 p 24
A -------------
- B -----------
- C -------------
- D -------- E -----------------
47 p 16 2 p 32 50 p 16 p 29 25

3 Simplify the following expression with positive indices.


1
---
7A,B
( 16x –6 y 10 ) 2 ÷ 3 ( 27x 3 y 9 )

4 multiple choice
7B
1
64  – --3-
The value of 5−2  --------
 125
- is:
1 4 5
A ------ B 5 C --- D --- E −5
20 5 4

5 multiple choice
7C
If 252 − x = 125, then x is equal to:
1
A 1 B --- C −1 D 2 E 5
2

6 multiple choice
7C
If 42x − 17(4x) + 16 = 0, then x is equal to:
A 1 or 16 B 0 or 1 C 2 or 8 D 1 or 4 E 0 or 2

7 Solve the following equations.


a 2x5 = 100 7C
b 8x + 1 × 22x = 43x − 1

8 Find the solution to 9x − 5(3x) + 6 = 0 correct to 2 decimal places. 7C


344 M a t h s Q u e s t M a t h s B Ye a r 1 1 f o r Q u e e n s l a n d

7D 9 multiple choice y
The rule for the graph at right could be:
A y = 3x − 2
B y = 3x + 2 0 x
C y = 3x − 2 –1
D y = −2x –2
E y = 2x + 2

Questions 10 to 12 refer to the function defined by the rule y = 2x + 3 − 1.

7D 10 multiple choice
The graph which best represents this function is:
A y B y C y
7

–3 0 x
1
0 –1 x –1
–3 0 x
D y E y
1

–3 0 x
0 3 x
–1

7D 11 multiple choice
The domain is:
A (3, ∞) B [−1, ∞) C R+ D R E (−3, ∞)

7D 12 multiple choice
The range is:
A [−1, ∞) B R C R+ D (1, ∞) E (−1, ∞)
13 For the function with the rule f (x) = 3x − 2 + 1:
7D a find the y-intercept
b state the equation of the asymptote
c sketch the graph of f (x)
d state the domain and range.

7E 14 multiple choice
When expressed in log form, 5x = 250 becomes:
A log x 5 = 250 B log 5 x = 250 C log 5 250 = x D log x 250 = 5 E log 250 x = 5

7E 15 multiple choice
The value of log 7 49 + 3 log 2 8 − 4 is:
A 3 B 7 C 0 D 69 E 1

7E
1
16 a Evaluate log 3 -----
-.
27
b Express y in terms of x if log 10 x + log 10 y = 2 log 10 (x + 1).
Chapter 7 Exponential and logarithmic functions 345
17 Simplify the following.
a 3 log 4 5 − 2 log 4 6 7E
2
2 log 5 x
b ---------------------
1
--- log 5 x
3

18 multiple choice
7F
log 3 25
The value of -----------------
- is nearest to:
log 3 5
A 3 B 5 C 2 D 9 E 20

19 multiple choice
7F
5
---
log 7 x 4
The expression ------------------
- simplifies to:
log 7 x
5 3
--- ---
5 3 5
A --- B log7 x 2 C log7 x 4 D --- E ---
2 4 8

20 multiple choice
7F
The solution to log 5 x = 4 is:
A 25 B 125 C 1 D 625 E 20

21 multiple choice
7F
The value of x if 2 log x 343 = 6 is:
A 3 B 7 C 5 D 14 E 7

22 multiple choice
7F
If log 3 (2x − 1) + log 3 2 = 2, then x is equal to:
5 11
A 2 B 1 C 3 D --- E ------
2 4

23 multiple choice
7F
The solution to the equation 43 − 2x = 12 is nearest to:
A x = −1 B x = 0.35 C x = 0.604 D x = 0.2 E x=0

24 Solve each of the following.


a log 6 x = 3 7F
b 2 log x 125 = 6
c log 2 (3x + 6) − log 2 5 = 2

25 If 5x = 4, find x correct to 3 decimal places. 7F


26 Solve for x where 102 − 3x = 8. 7F
346 M a t h s Q u e s t M a t h s B Ye a r 1 1 f o r Q u e e n s l a n d

27 The number of bacteria in a culture, N, is given


7G by the exponential function
N = 1500(20.18t ), where t is the number of days.
a Find the initial number of bacteria in the
culture.
b Find the number of bacteria (to the nearest
100) after:
i 5 days ii 10 days.
c How many days does it take for the number
of bacteria to reach 9000?

Modelling and problem solving


1 The number of lions, L, in a wildlife park is given
by L = 20(100.1t ), where t is the number of years
since counting started. At the same time the
number of cheetahs, C, is given by C = 25(100.05t ).
a Find the number of:
i lions ii cheetahs
when counting began.
b Find the numbers of each after
i 1 year ii 18 months.
c Which of the animals is the first to reach a population of 40 and by how long?
d After how many months are the populations equal and what is this population?
2 The temperature T °C of a coffee in a ceramic mug at time t minutes after it is poured is given
by T = 60(4–0.05t) + 20.
a Find the initial temperature of the coffee.
b Find the temperature of the coffee, correct to one decimal place, at:
i 2 minutes after it is poured ii 25 minutes after it is poured.
c Sketch the graph of the equation for 0 ≤ t ≤ 50.
If the coffee can be comfortably drunk when it is between temperatures of 30°C and
45°C, find:
d the time available to drink the coffee
e the final temperature the coffee will settle to.
3 The number of bacteria (N) in a culture is given by the exponential function
N = 12 000(20.125t), where t is the number of days.
a Find the initial number of bacteria in the culture.
b Find the number of bacteria in the culture after:
i 4 days ii 2 weeks.
c Find the time taken for the bacteria to reach 32 000.
When the bacteria reach a certain number, they are treated with an anti-bacterial serum.
The serum destroys bacteria according to the exponential function D = N0 × 3–0.789t, where
D is the number of bacteria remaining after time t and N0 is the number of bacteria present
at the time the serum is added. The culture is considered cured when the number of
eBook plus bacteria drops below 1000.
d If the bacteria are treated with the serum when their numbers reach 32 000, find the
Digital doc: number of days it takes for the culture to be classed as cured.
Test Yourself
Chapter 7
e How much longer would it take the culture to be cured if the serum is applied after
6 weeks?
Applications
of exponential
and logarithmic
functions in
financial
mathematics
8
syllabus reference
Topic:
• Exponential and
logarithmic functions and
applications

In this chapter
8A Geometric sequences
8B Geometric series
8C Growth and decay
functions
8D Compound interest
formula
8E Loan schedules
8F The annuities formula
348 M a t h s Q u e s t M a t h s B Ye a r 1 1 f o r Q u e e n s l a n d

Introduction
Harry and Maude are planning to borrow
money to buy a home. Harry says that
one of the people he works with
encouraged him to make fortnightly
repayments on the loan. This person
claimed that, on a loan of $100 000 at
8% per annum, making fortnightly repay-
ments of $400 instead of monthly repay-
ments of $800 saves you $25 000 over the
course of the loan.
The change is small. Instead of 12 monthly payments of $800, the borrowers make
26 fortnightly payments of $400. This amounts to an extra $800 per year, or $66.67 a
month. How can you save $25 000 by making such a small change?

Geometric sequences
A farmer is breeding worms which he hopes to sell to
local shire councils for use in the decomposition of
waste at rubbish dumps. Worms reproduce readily and
the farmer expects a 10% increase per week in the
mass of worms that he is farming. A 10% increase per
week would mean that the mass of worms would
10
increase by a constant factor of (1 + --------
- ) or 1.1.
100
He starts off with 10 kg of worms. By the beginning of the second week he will
expect 10 × 1.1 = 11 kg of worms, by the start of the third week he would expect
11 × 1.1 = 10 × (1.1)2 = 12.1 kg of worms, and so on. This is an example of a
geometric sequence.
A geometric sequence is the sequence where each term is obtained by multiplying
the preceding term by a certain constant factor.
The first term is 10 and the common factor here is 1.10 which represents a 10%
increase on the previous term. We can put the results of the above example into a table:

n tn tn
1 10 × (1.1)0 10
1
2 10 × (1.1) 11
2
3 10 × (1.1) 12.1
4 10 × (1.1)3 13.31
n 10 × (1.1)n − 1 10 × (1.1)n − 1
From this table we can see that t2 = 1.1 × t1, t3 = 1.1 × t2 and so on. In general:
tn + 1 = 1.1 × tn
The common factor or common ratio whose value is 1.1 for this example can be
tn + 1
found by dividing any two successive terms: ----------.
tn
Chapter 8 Applications of exponential and logarithmic functions in financial mathematics 349
A geometric sequence, t, can be written in terms of the first term, a, and the common
ratio, r. Thus:
t: {a, ar, ar 2, ar 3, . . . , ar n − 1, . . .}
The first term t1 = a, the second term t2 = ar, the third term t3 = ar2 and consequently
the nth term, tn is ar n − 1.
For a geometric sequence:
tn = ar n − 1
where a is the first term and r the common ratio, given by
tn + 1
r = -----------
tn

If we consider three consecutive terms in a geometric sequence, x, y and z, then


y z
-- = r = --
x y
where r is the common factor.
Thus the middle term, y, called the geometric mean, can be calculated in terms of the
outer two terms, x and z.
For a geometric sequence . . ., x, y, z, . . . :
y2 = xz

WORKED Example 1
State whether the sequence is geometric by finding the ratio of successive terms:
tn: {2, 6, 18, . . .}. If it is geometric, find the next term in the sequence, t4, and the nth term
for the sequence, tn.
THINK WRITE
t t2 6
1 Find the ratio ---2- . ---- = ---
t1 t1 2
=3
t t 3 18
2 Find the ratio ---3- . ---- = ------
t2 t2 6
=3
t t
3 Compare the ratios and make your Since ---2- = ---3- = 3, the sequence is geometric
conclusion. t1 t2
with the common ratio r = 3.
4 Since the sequence is geometric, to find t4 = t3 × r
the fourth term, multiply the preceding t4 = 18 × 3
(third) term by the common ratio. t4 = 54
5 Write the general formula for the nth tn = ar n − 1
term of the geometric sequence.
6 Identify the values of a and r. a = 2; r = 3
7 Substitute the values of a and r into the tn = 2 × 3n − 1
general formula.
350 M a t h s Q u e s t M a t h s B Ye a r 1 1 f o r Q u e e n s l a n d

WORKED Example 2
Find the nth term and the 10th term in the geometric sequence where the first term is 3
and the third term is 12.
THINK WRITE
1 Write the general formula for the nth tn = ar n − 1
term in the geometric sequence.
2 State the value of a (the first term in the a = 3; t3 = 12
sequence) and the value of the third
term.
3 Substitute all known values into the 12 = 3 × r3 − 1
general formula. 12 = 3 × r 2
4 Solve for r (note that there are r2 = 12
------
3
2 possible solutions).
r2 = 4
2
r= ± 4
r 2 = ±2
5 Substitute the values of a and r into the So tn = 3 × 2n − 1, or tn = 3 × (−2)n − 1.
general equation. Since there are
2 possible values for r, you must show
both expressions for the nth term of the
sequence.
6 Find the 10th term by substituting When n = 10, t10 = 3 × 210 − 1 (using r = 2)
n = 10 into each of the two expressions = 3 × 29
for the nth term. = 1536
or t10 = 3 × (−2)10 − 1 (using r = −2)
= 3 × (−2)9
= −1536

WORKED Example 3
The fifth term in a geometric sequence is 14 and the seventh term is 0.56. Find the
common ratio, r, the first term, a, and the nth term for the sequence.
THINK WRITE
1 Write the general rule for the nth term tn = ar n − 1
of the geometric sequence.
2 Use the information about the 5th term When n = 5, tn = 14
to form an equation. Label it [1]. 14 = a × r 5 − 1
14 = a × r4 [1]
3 Similarly, use information about the 7th When n = 7, tn = 0.56
term to form an equation. Label it [2]. 0.56 = a × r7 − 1
0.56 = a × r 6 [2]
Chapter 8 Applications of exponential and logarithmic functions in financial mathematics 351

THINK WRITE

4 Solve equations simultaneously: Dividing [2] by [1]:


Divide equation [2] by equation [1] to
ar 6 0.56
eliminate a. -------- = ----------
ar 4 14
5 Solve for r. r 2 = 0.04
2
r = ± 0.04
2
r = ±0.2
6 Since there are two solutions, we have to If r = 0.2
perform two sets of computations. Substituting r into [1]:
Consider the positive value of r first. a × ( 0.2 ) 4 = 14
Substitute the value of r into either of the
0.0016a = 14
two equations, say equation [1], and solve
for a. a = 14 ÷ 0.0016
= 8750
7 Substitute the values of r and a into the The nth term is:
general equation to find the expression for tn = 8750 × (0.2)n − 1
the nth term.
8 Now consider the negative value of r. If r = −0.2
9 Substitute the value of r into either of the Substituting r into [1]:
two equations, say equation [1], and solve a × ( – 0.2 ) 4 = 14
for a. (Note that the value of a is the same
0.0016a = 14
for both values of r.)
a = 14 ÷ 0.0016
= 8750
10 Substitute the values of r and a into the The nth term is:
general formula to find the second tn = 8750 × (−0.2)n − 1
expression for the nth term of the sequence.

Graphics Calculator tip! Listing the terms of a


geometric sequence
If you know the rule for a geometric sequence, successive terms can be listed using a
graphics calculator. The steps for generating these terms are shown below. Consider
listing the first 5 terms of a sequence where the first term is 3 and the ratio is 2, so the
rule is tn = 3 × 2n − 1 .
For the Casio fx-9860G AU
1. Press MENU and then select RUN-MAT. Press
OPTN and then F1 (LIST) followed by F5
(Seq).
<Casio04.01>
352 M a t h s Q u e s t M a t h s B Ye a r 1 1 f o r Q u e e n s l a n d

2. Enter the rule 3 × 2^ (n − 1) followed by the


variable name (n), the start value for n (1), the end
value of n (5) and the increment (1), each
separated by a comma ( , ). Press ) to close
the set of brackets. (To enter the variable n, press
ALPHA [N].)
3. Press EXE to display the list of the first 5 terms.
Use the down arrow key to scroll down the list.

For the TI-Nspire CAS


1. Open a new Lists & Spreadsheet document
(press /N and select 3: Add Lists &
Spreadsheet). Press b and then select 3: Data.

2. Press 1: Generate Sequence and complete at


least the first two entry boxes, pressing e to
move to the next line. Enter 3 × 2^ (n − 1) for the
Formula and 3 for the Initial Terms. In this case
we want 5 for the Max No. Terms. Note that
completing the boxes for Max No. Terms (the
default value is 255) and Ceiling Value is
optional. Press e until OK is highlighted.
3. Press · to display the list of the first 5 terms.
Use the NavPad to scroll down the list.

remember
1. A geometric sequence is one where each successive term is obtained by
multiplying the preceding term by the constant number. This number is called
tn + 1
- = r for all values of n.
the common ratio and is given the symbol r. Thus ---------
tn
The first term in the sequence is given the symbol a.
2. If x, y, z are successive terms in the geometric sequence then y is called a
geometric mean and is given by y2 = xz.
3. A geometric sequence can be written as a, ar, ar 2, . . . and so the nth term tn is:
tn = ar n − 1 using the function notation, or
tn + 1 = rtn, t1 = a using the iterative notation.
Chapter 8 Applications of exponential and logarithmic functions in financial mathematics 353

8A Geometric sequences

WORKED 1 State which of the following are geometric sequences by finding the ratio of suc-
Example
1
cessive terms. For those which are geometric, find the next term in the sequence, t4
and the nth term for the sequence, tn.
a tn: {3, 6, 9, . . .} b tn: {4, 12, 36, . . .} c tn: {3, 6, 12, . . .}
d tn: {4, 6, 9, . . .} e tn: {−3, 1, − 1--- , . . .} f tn: {2, −6, 18, . . .}
3
g tn: { 2--- , 6 9
------ , ------ , . . .} h tn: { 3--- , 3--- , 3--- , . . .} i tn: { 3--- , 3--- , 9--- , . . .}
7 14 14 4 2 1 4 2 4

eBook plus j tn: { 1--- , − 3--- , 9, . . .} 2


k tn: {2π, 4π , 8π , . . .} 3
4 2

Digital doc: 2 For each of the following:


EXCEL Spreadsheet i show that the sequence is geometric
Geometric sequences
and series ii find the nth term and consequently the 6th and the 10th terms.
a t: {5, 10, 20, . . .} b t: {2, 5, 12.5, . . .}
c t: {1, −3, 9, . . .} d t: {2, −4, 8, . . .}
e t: {2.3, 3.45, 5.175, . . .} f t: { 1--- , 1, 2, . . .}
2

g t: { 1--- , -----
1 1
- , ------ , . . .} h t: { 3--- , − 1--- , 1
------ , . . .}
3 12 48 5 5 15
1 2 4
i t: {x, 3x4, 9x7, . . .} j t: { --- , ----2- , ----3- , . . .}
x x x
WORKED 3 Find the nth term and the 10th term in the geometric sequence where:
Example
2
a the first term is 2 and the third term is 18 (Why are there two possible answers?)
b the first term is 1 and the third term is 4 (Why are there two possible answers?)
c the first term is 5 and the fourth term is 40
d the first term is −1 and the second term is 2
1
e the first term is 9 and the third term is -----
- . (Why are there two possible answers?)
81
4 Find the 4th term in the geometric sequence where the first term is 6 and the 7th term
3
is -----
-.
32
5 Find the nth term in the geometric sequence where the first term is 3 and the fourth
term is 6 2 .
6 For the geometric sequence 3, m, n, 192, . . . , find the values for m and n.
7 Consider the geometric sequence t: {16, m, 81, n, . . .}. Find the values of m and n, if
it is known that both are positive numbers.
8 For the geometric sequence a, 15, b, 0.0375, . . . , find the values of a and b, given that
they are positive numbers.
WORKED 9 The third term in a geometric sequence is 100 and the fifth term is 400. Find the
Example
3
common ratio, r, the first term, a, and the nth term for the sequence.
10 If t2 = 1--- and t5 = 27
------ , find the first term, a, the common factor, r, and hence the nth
2 16
term for the geometric sequence.
11 Find the value of x such that the following sequence forms a geometric progression:
x − 1, 3x + 4, 6x + 8.
354 M a t h s Q u e s t M a t h s B Ye a r 1 1 f o r Q u e e n s l a n d

1 1
12 Insert three terms in between 8 and ------ such that the sequence of numbers, 8, _, _, _, ------ ,
32 32
is geometric.
13 The difference between the first term and the second term in a geometric sequence
is 6. The difference between the second term and the third term is 3.
a Calculate the difference between the third term and the fourth term.
b Find the nth term in the sequence.
14 The first two terms in a geometric sequence are 120, 24, and the kth term is 0.0384.
Find the value for k.

Geometric series
When we add up or sum the terms in a sequence we get the series for that sequence. If
we look at the geometric sequence {2, 6, 18, 54, . . .} where the first term t1 = a = 2 and
the common ratio is 3 we can quickly calculate the first few terms in the series of this
sequence.
S1 = t1 = 2
S2 = t1 + t2 = 2 + 6 = 8
S3 = t1 + t2 + t3 = 2 + 6 + 18 = 26
S4 = t1 + t2 + t3 + t4 = 2 + 6 + 18 + 54 = 80
In general the sum of the first n terms is:
Sn = t1 + t2 + t3 + . . . + tn − 2 + tn − 1 + tn.
For a geometric sequence the first term is a, the second term is ar, the third term is
ar and so on up to the nth term which is ar n − 1. Thus:
2

Sn = a + ar + ar 2 + . . . + ar n − 3 + ar n − 2 + ar n − 1 [1]
If we multiply equation [1] by r we get:
rSn = ar + ar 2 + ar 3 + . . . ar n − 2 + ar n − 1 + ar n [2]

Note that on the right-hand side of equations [1] and [2] all but two terms are
common, namely the first term in equation [1], a, and the last term in equation [2], ar n.
If we take the difference between equation [2] and equation [1] we get:
rSn − Sn = ar n − a [2] − [1]
∴ (r − 1)Sn = a(r n − 1)
a(rn – 1)
∴ Sn = ---------------------- ; r ≠ 1 (r cannot equal 1)
r–1

We now have an equation which allows us to calculate the sum of the first n terms of a
geometric sequence.

The sum of the first n terms of a geometric sequence is given by:


a( rn – 1 )
Sn = ---------------------- ; r ≠ 1
r–1
where a is the first term of the sequence and r is the common ratio.
Chapter 8 Applications of exponential and logarithmic functions in financial mathematics 355
WORKED Example 4
Find the sum of the first 5 terms (S5) of these geometric sequences.
a tn: {1, 4, 16, . . .}
b tn = 2 (2)n - 1, n ∈ {1, 2, 3, . . .}
c tn + 1 = 1--4- tn, t1 = − 1--2-

THINK WRITE
a(rn – 1)
a 1 Write the general formula for the sum a S n = ----------------------
of the first n terms of the geometric r–1
sequence.
2 Write the question. tn: {1, 4, 16, . . .}
3 Identify the variables: a is the first a = 1; r = 4
--- = 4; n = 5
1
term; r can be established by finding
the ratio; n is known from the question.
Substitute the values of a, r and n into 5
4 1(4 – 1)
the formula and evaluate. S5 = ----------------------
4–1
1024 – 1
S5 = ---------------------
3
S5 = 341

b 1 Write the question. b tn = 2 (2)n − 1, n ∈ {1, 2, 3, . . .}


2 Compare the given rule with the a = 2; r = 2; n = 5
general formula for the nth term of the
geometric sequence tn = ar n − 1 and
identify values of a and r; the value of n
is known from the question.
3 Substitute values of a, r and n into the 5
2(2 – 1)
general formula for the sum and S5 = ----------------------
2–1
evaluate. 2 ( 32 – 1 )
S5 = -----------------------
1
S5 = 62

c 1 Write the question. c tn + 1 = 1--- tn, t1 = − 1---


4 2
2 This is an iterative formula, so the r= 1
--- ; a= − 1--- ; n=5
coefficient of tn is our r; a = t1; n is 4 2
known from the question.
Substitute values of a, r and n into the 5
3 – --12- [ ( --14- ) – 1 ]
general formula for the sum and S5 = ---------------------------
1
-
--- – 1
evaluate. 4

– 1--2- × ( -----------
1
1024
- – 1)
S5 = -------------------------------
3
-
–4 --
-

S5 = − 341
---------
512
356 M a t h s Q u e s t M a t h s B Ye a r 1 1 f o r Q u e e n s l a n d

Graphics Calculator tip! Finding the sum of a


geometric sequence
Consider the sequence tn = 3 × 2n−1 used in the previous Graphics calculator tip on
page 351. Let’s find the sum of the first 5 terms of this sequence. There are a number of
ways to find the sum using a graphics calculator. Two methods are shown below.
Method 1: Using the SUM function
For the Casio fx-9860G AU
1. As before, produce the first 5 terms of the sequence.
Press MENU and then select RUN-MAT. Press OPTN
and then F1 (LIST) followed by F5 (Seq). Enter
the rule 3 × 2^ (n − 1) followed by the variable name
(n), the start value for n (1), the end value of n (5) and
the increment (1), each separated by a comma ( , ).
Press ) to close the set of brackets.
2. To store this sequence of 5 terms as List 1, press
→ then F1 (List) and 1 . Press EXE .

3. Press F6 ( ) twice to display the Sum option and


then press F1 (Sum).

4. Press F6 ( ) again for more options followed by


F1 (List) and then 1 to indicate List 1. Press
EXE to display the sum of the first 5 terms. The
sum of the first 5 terms is 93.

For the TI-Nspire CAS


1. As before, produce the first 5 terms of the sequence.
Open a Lists & Spreadsheet document, press b,
then 3: Data followed by 1: Generate Sequence.
Enter the formula 3 × 2^ (n − 1) with an initial term
of 3 and a maximum number of terms of 5. Press e
until OK is highlighted.

2. Press · to display the list of terms. Use the NavPad


to move across to cell B1.
Chapter 8 Applications of exponential and logarithmic functions in financial mathematics 357
3. To calculate the sum of the first 5 terms, add the
numbers in cells A1 to A5. Enter the formula
= sum(A1:A5) in cell B1.
(Press =SUM(A1:A5).)

4. Press · to display the sum in cell B1. The sum


of the first 5 terms is 93.

Method 2: Finding S
This method involves the use of the Greek letter Σ, pronounced sigma. In mathematics,
this letter stands for ‘sum of’.
For the Casio fx-9860G AU
(Note that the first two steps are the same as Method 1.)
1. As before, produce the first 5 terms of the
sequence. Press MENU and then select RUN-MAT.
Press OPTN and then F1 (LIST) followed by
F5 (Seq). Enter the rule 3 × 2^ (n − 1) followed
by the variable name (n), the start value for n (1),
the end value of n (5) and the increment (1), each
separated by a comma ( , ). Press ) to close
the set of brackets.
2. To store this sequence of 5 terms as List 1, press
→ then F1 (List) and 1 . Press EXE .

3. Press MENU and then select STAT. Press


F2 (CALC) followed by F1 (1VAR) and you
will see that Σx = 93. The sum of the first 5 terms
is 93.

For the TI-Nspire CAS


1. Open a new Calculator document. Press /k
to access the symbol palette and highlight Σ.
358 M a t h s Q u e s t M a t h s B Ye a r 1 1 f o r Q u e e n s l a n d

2. Press · to display the symbol Σ in the calculator


screen. Press ( and then enter the formula
3 × 2^ (n − 1) followed by the variable (n), the
first term (1) and the end term (5), each separated
by a comma (,). Press ) to close the set of
brackets.

3. Press · to display the sum of the first 5 terms.


The sum of the first 5 terms is 93. Note the use of
5
the sigma notation ∑ . This should be read and
n=1
understood as ‘the sum of the expression in
brackets for the value of n = 1 and all values of n
up to and including 5’.

The infinite sum of a geometric sequence where r < 1


When the constant ratio, r, is less than 1 or greater than −1, that is, {r: −1 < r < 1}, each
successive term in the sequence gets closer to zero. This can readily be shown with the
following two examples.
g: {2, −1, 1--- , − 1--- , . . .} where a = 2 and r = − 1---
2 4 2
1 1 1
h: {40, --- , --------- , . . .} where a = 40 and r = ------
2 160 80
In both the examples, successive terms approach zero as n increases. In the second
case the approach is more rapid than in the first and the first sequence alternates posi-
tive and negative. A simple investigation with a spreadsheet will quickly reveal that for
geometric sequences with the size or magnitude of r < 1 the series eventually settles
down to a near constant value. We say that the series converges to a value S∞ which is
the sum to infinity of all terms in the geometric sequence. We can find the value S∞ by
recognising that as n → ∞ the term r n → 0, provided r is between −1 and 1. We write
this technically as −1 < r < 1 or |r| < 1. The symbol |r| means the magnitude or size of
r. Using our equation for the sum of the first n terms:
a(rn – 1)
S n = ---------------------- ; r ≠ 1
r–1
Taking −1 as a common factor from the numerator and denominator:
a(1 – rn)
S n = ----------------------
1–r
n n
As n → ∞, r → 0 and hence 1 − r → 1. Thus the top line or numerator will equal
a when n → ∞:
a
S ∞ = ----------- ; r < 1
1–r
We now have an equation which allows us to calculate the sum to infinity, S∞ , of a
geometric sequence.
The sum to infinity, S∞ , of the geometric sequence is given by:
a
S • = ----------- ; r < 1
1–r
where a is the first term of the sequence and r is the common ratio whose
magnitude is less than one.
Chapter 8 Applications of exponential and logarithmic functions in financial mathematics 359
WORKED Example 5
a Find the sum to infinity for the sequence tn: {10, 1, 0.1, . . .}.
b Find the fourth term in the geometric sequence whose first term is 6 and whose sum to
infinity is 10.
THINK WRITE
a 1 Write the formula for the nth term of the a tn = ar n − 1
geometric sequence.
2 From the question we know that the first a = 10, r = 0.1
term, a, is 10 and r = 0.1.
3 Write the formula for the sum to infinity. a
S ∞ = ----------- ; r < 1
1–r
4 Substitute a = 10 and r = 0.10 into the 10
formula and evaluate. S ∞ = ----------------
1 – 0.1
10 100
S∞ = -------
0.9
= ---------
9
= 11 1--9-

b a
b S ∞ = ----------
1 Write the formula for the sum to -; r <1
infinity. 1–r
2 From the question it is known that the
a = 6; S∞ = 10
infinite sum is equal to 10 and that the first
term a is 6. Write down this information.
3 Substitute known values into the 6
10 = -----------
formula. 1–r
4 Solve for r. 10(1 − r) = 6
10 − 10r = 6
10r = 4
r = 0.4
5 Write the general formula for the tn = ar n − 1
nth term of the geometric sequence.
6 To find the 4th term substitute a = 6, n = 4 t4 = 6 × (0.4)3
and r = 0.4 into the formula and evaluate. t4 = 0.384

remember
1. The sum of the first n terms in a geometric sequence is:

a(1 – rn)
S n = ---------------------- with r ≠ 1
1–r
n
a(r – 1)
or S n = ---------------------- with r ≠ 1
r–1
2. When the magnitude of r is less than one, that is, −1 < r < 1, the sum of a
geometric sequence to infinity, S∞ is given by:
a
S ∞ = -----------
1–r
360 M a t h s Q u e s t M a t h s B Ye a r 1 1 f o r Q u e e n s l a n d

8B Geometric series
WORKED 1 Consider the following sequences and find the terms indicated.
Example
4 a tn: {1, 2, 4, . . .}. Find S5, S10, S20.
b tn: {1, 3, 9, . . .}. Find S5, S10, S20.
c tn = 3(−2)n − 1, n ∈ {1, 2, 3, . . .}. Find S5, S10, S20.
d tn = −4(1.2)n − 1, n ∈ {1, 2, 3, . . .}. Find S1, S10, S20.
eBook plus
e tn + 1 = 2tn, t1 = 3--- . Find S1, S5, S10.
2
Digital doc:
1
EXCEL Spreadsheet f tn + 1 = --- tn, t1 = − 2--- . Find S1, S5, S10.
2 3
Geometric sequences
and series g The first term is 3000 and the common ratio is 1.05. Find S4, S16, S64.
h The first term is 1400 and the common ratio is −1.1. Find S4, S9, S14.
i The first term is 20; every other term is obtained by multiplying the preceding
term by 5. Find S5, S10.
j The first term is −2; every other term is obtained by multiplying the preceding
term by − 1--- . Find S5, S10.
2
2 Consider the following geometric sequences and find the terms indicated.
a The first term is 440 and the 12th term is 880. Find S6.
b The 5th term is 1 and the 8th term is 8. Find S1, S10, S20.
3 Find the sum of the first 12 terms of a geometric sequence in which the second term
is 8--- and the fifth term is 9.
3
4 What minimum number of terms of the series 2 + 3 + 4 1--- + . . . must be taken to give
2
a sum in excess of 100?
5 The sum of the first four terms of a geometric sequence is 312, and the sum of the
next four terms is 625 times that number. Find the sum of the following four terms.
6 Find the sum of all powers of 2 between 500 and 50 000.
7 Find the sum of all powers of 4 between 500 and 50 000.
WORKED 8 Find the sum to infinity for the following geometric sequences.
Example
5a a tn: {1, 1--- , 1--- , . . .} b tn: {1, − 1--- , 1--- , − 1--- ,. . .}
2 4 2 4 8

c tn: {1, 1--- , 1--- , . . .} d tn: {1, 2--- , 4--- , . . .}


3 9 3 9

e tn: {1, − 2--- , 4--- , − 8--- , . . .}


3 9 9

9 For the infinite geometric sequence { 1--- , 1--- , 1--- , . . .}, find the sum to infinity. Consequently,
2 4 8
find what proportion each of the first three terms contributes to this sum as a percentage.

10 For the infinite geometric sequence {1, 1--- , 1


------ , . . .}, find the sum to infinity. Consequently,
4 16
find what proportion each of the first three terms contributes to this sum as a percentage.

11 For the infinite geometric sequence {1, 3--- , -----


9
- , . . .}, find the sum to infinity. Consequently,
4 16
find what proportion each of the first three terms contributes to this sum as a percentage.
Chapter 8 Applications of exponential and logarithmic functions in financial mathematics 361
12 A sequence of numbers is defined by tn = 3  1--- n − 1, n ∈ {1, 2, 3, . . .}.
2
a Find the sum of the first 20 terms.
b Find the sum of all the terms between and including t21 and t40.
c Find the sum to infinity, S∞.
13 A sequence of numbers is defined by tn: {9, −3, 1, . . .}.
a Find the sum of the first 9 terms.
b Find the sum of all the terms between and including t10 and t15.
c Find the sum to infinity, S∞.
14 The first term of the geometric sequence is 5 and the fourth term is 0.078 125. Find
the sum to infinity.
15 The sum of the first four terms of a geometric sequence is 30 and the sum to infinity
is 32. Find the first three terms of the sequence.
16 For the geometric sequence 5 + 3, 5 – 3 , . . ., find the common factor, r, and the
sum of the infinite series, S∞.
17 If 1 + 3x + 9x2 + . . . = --2- , find the value of x.
3
WORKED
Example 18 The first term in a geometric sequence is 4 and S∞ = 6. Find the common factor, r.
5b
19 If the common ratio for a geometric sequence is 0.99 and the sum to infinity is 100,
eBook plus
what is the value of the first and second terms in the sequence?
20 Show that x n − 1 always has a factor (x − 1) for n ∈ {1, 2, 3, . . .}.
Digital doc:
WorkSHEET 8.1

Crossing the road


A student stands at one side of a road
10 metres wide, and walks halfway
across. The student then walks half of
the remaining distance across the road,
then half the remaining distance again
and so on.

1 Will the student ever make it past the


other side of the road?

2 Does the width of the road affect


your answer?
362 M a t h s Q u e s t M a t h s B Ye a r 1 1 f o r Q u e e n s l a n d

Introduction to growth and decay


Certain quantities in nature and business may change in a uniform (constant) way
over time. A change may be an increase, as in the case of the value of an investment
such as a house, or it may be a decrease, like the fall in the population of an
endangered species.
Growth and decay can often be modelled by equations or graphs. These in turn can be
used to analyse the situations being modelled and to make predictions about them. For
instance, if we know an equation that relates the falling value of a car to time, the future
value of the car can be determined. Falling values of this type are called depreciation.
This chapter investigates the general principles of growth and decay and takes a
more detailed look at specific examples in the business world, such as compound
interest, depreciation and inflation.

Growth and decay functions


Straight line and exponential growth
If a quantity increases in size over a period of time, it is growing. This growth process
may be straight line growth or exponential growth.

Straight line growth


A quantity may increase by a fixed amount for each time unit; that is, a fixed amount is
added. Hence, the relationship is linear.
A stamp collection worth $5000, for instance, may increase in value by $200 each
year. This growth relationship can be written as the equation
V = 5000 + 200T
where V = value of collection
T = time in years.
The general form of the linear growth function is
y = a + bx
where y is the dependent variable
x is the independent variable (usually time)
a is the initial or starting value of y
b is the rate of growth.

A graph of the equation could be drawn to represent this situation, like the one shown.
By using the graph or the equation we can analyse the situation; for example, to find
the collection’s value at any future time.
V
6000
5800
Value ($)

5600
5400
5200
5000
0 1 2 3 4 5 T
Time (years)
Chapter 8 Applications of exponential and logarithmic functions in financial mathematics 363
WORKED Example 6
Write an equation which describes the relationship between the number, P, of bees and
time, T, if the population of 500 bees is known to be increasing by 20 each month. Find:
a the size of the population in a year’s time
b when the population will have doubled.
THINK WRITE/DISPLAY
Let population be P and time in months be T. Let P = population and T = time in months.
Write an equation to describe the relationship. P = 500 + 20T

Using the equation


a 1 Substitute T = 12 a When T = 12,
(12 months = 1 year). P = 500 + 20 × 12

2 Write your answer. P = 740 bees in one year’s time

b 1 Substitute P = 1000 and solve. b When P = 1000,


1000 = 500 + 20T
T = 25 months

2 Write your answer. The population will double after 2 years


and 1 month.

Using the graph


a
a 1 Draw a graph of population against T 0 10 20 30
time.
i(i) Set out the data in a table. P 500 700 900 1100

(ii) Plot the points and join them with a line P


to produce the graph of P = 500 + 20T. 1000
Number of bees

800

600

400
0 10 20 30 T
Time (months)

2 Find T = 12 and read from the graph. When T = 12, P = 740.

3 Write your answer. The population in one year’s time will


be 740 bees.

b 1 Find P = 1000 and read from the graph. b When P = 1000, T = 25.
2 Write your answer. The population will double after 2 years
and 1 month.

Continued over page


364 M a t h s Q u e s t M a t h s B Ye a r 1 1 f o r Q u e e n s l a n d

THINK WRITE/DISPLAY

Using a graphics calculator


For the Casio fx-9860G AU
1. Press MENU and then select GRAPH.
Enter the equation in terms of Y (in place
of P) and X (in place of T). Press EXE .

2. Press SHIFT F3 (V-WIN) and enter 0 for


Xmin, 0.25 for dot, 400 for Ymin and 1200
for Ymax. Note that changing the dot
setting automatically adjusts the Xmax
setting. (The dot setting has been changed
so that we can obtain whole numbers for
the X and Y coordinates later.)
3. Press EXE until you return to the Graph
function screen. Press F6 (DRAW) to
display the graph and then SHIFT F1
(TRCE) to trace along the line.

a 1 Use the arrow keys to move along the a


line until X = 12 is displayed. Read the
value for Y. This shows that P = 740
bees.

2 Write your answer. The population in one year’s time will be


740 bees.
b 1 Use the arrow keys to move along the b
line until Y = 1000 is displayed. Read
the value for X. This shows that T = 25
months.

2 Write your answer. The population will double after 2 years and
1 month.

For the TI-Nspire CAS


1. Open a new Graphs & Geometry
document. Enter the equation in terms of
f1(x) (in place of P) and x (in place of T ).
Chapter 8 Applications of exponential and logarithmic functions in financial mathematics 365
THINK WRITE/DISPLAY

2. Press ·. Press b and then select 4: Window


followed by 1: Window Settings. Enter the
settings shown.

3. Press e until OK is highlighted and then press


· to display the graph with these settings.
Press b and then select 5: Trace followed by
1: Graph Trace. Use the arrow keys on the
NavPad to trace along the line. The coordinates
of a given point on the line will be displayed.

4. Since we are dealing with whole number values,


press b and then select 5: Trace followed by
2: Trace Settings. Enter 1 for the Trace Step.
This will display points with a whole number
value for the x-coordinate as you move along
the line.

a 1 To find the coordinates of the point where


x = 12, move along the line until you reach
the required point. Alternatively, enter 12 by
pressing 12 and then press ·. Read the
value for the y-coordinate of the point. This
shows that P = 740 bees.

2 Write your answer. The population in one year’s time


will be 740 bees.

b 1 Use the NavPad to move along the line until


the y-coordinate is 1000. Read the value for
the x-coordinate. This shows that T = 25
months.

2 Write your answer. The population will double after


2 years and 1 month.
366 M a t h s Q u e s t M a t h s B Ye a r 1 1 f o r Q u e e n s l a n d

Exponential growth N
16
Growth is exponential when the quantity present is

Number of bacteria ('000)


multiplied by a constant for each unit time interval.
This constant is called the growth or compounding 12
factor and is greater than one.
Consider the situation represented in the graph at right 8
where the changes to a certain population of bacteria
over a period of time are displayed. 4
It can be seen that the population is increasing but also
that the rate of growth is increasing; that is, the graph is 0
getting steeper. 0 1 2 3 4 5 T
Time (hours)
The readings taken from the graph are:

Time, T 0 1 2 3 4 5
Number, N 500 1000 2000 4000 8000 16 000

The population is doubling every hour, so the growth or compounding factor is 2.


The relationship could be written as N = 500 × 2T. This means that as T increases
by 1, the number of bacteria increases by a factor of 2.
In general, the exponential growth function has an equation of the form:
y = ka x, where k and a are constants
a > 1 is the growth or compounding factor
k is the initial value of y (when x = 0).

WORKED Example 7 Number of bacteria ('000)


N
The number of bacteria in a group was recorded over a 6-hour 16
period and a graph of the data is shown at right. 14
a Determine whether the population growth is exponential. 12
b If it is exponential, write an approximate equation for the 10
relationship. 8
6
THINK WRITE
4
a Find the initial population of bacteria, a From the graph, 2
1
N0 = 2000, 0
N0(T = 0), and population after 1 hour, 0 1 2 3 4 5 T
N1 = 3000 Time (hours)
N
N1(T = 1), and evaluate the ratio, ------1 . N1 3000
N0 ------ = ------------
N0 2000
= 1.5
2 Find N2. For exponential growth, 1.5 is N1 = 3000
the growth or compounding factor. That N2 = 3000 × 1.5
is, N2 = N1 × 1.5. N2 = 4500
3 Find N3. It should be that N3 = N2 × 1.5. N3 = 4500 × 1.5
N3 = 6750
4 Repeat the process. N4 = 6750 × 1.5
N3 = 10 125
N5 = 10 125 × 1.5
N3 = 15 150
Chapter 8 Applications of exponential and logarithmic functions in financial mathematics 367

THINK WRITE
5 By using 1.5 as the growth or compounding Growth or compounding factor = 1.5.
factor, the calculated values compare Growth is exponential.
favourably with values from the graph.

b In the equation the initial value is multiplied b NT = 2000(1.5)T


by the growth or compounding factor, 1.5,
raised to the power of unit time interval, that
is, y = ka x.

Once an equation has been determined for a relationship it can be used to analyse the
situation.

WORKED Example 8
The cost, C ($), of a deluxe puff pastry after time,
T (years), is given by the equation C = 0.8(1.6)T.
Use the equation to complete the table below and
plot a graph of cost against time.

Time, T 0 1 2 3 4 5
Cost, C

THINK WRITE
1 Write the equation and substitute T = 0. C = 0.8(1.6)T
When T = 0, C = 0.8(1.6)0
= 0.8 × 1
= 0.80
2 Substitute T = 1 and evaluate. When T = 1, C = 0.8(1.6)
= 1.28
3 Substitute T = 2 and evaluate. When T = 2, C = 0.8(1.6)2
= 2.05
4 Repeat for T = 3 to 5 and complete the table of
T 0 1 2 3 4 5
T and C values.
C 0.80 1.28 2.05 3.28 5.24 8.39

5 Draw the graph, joining the points with a C


smooth curve. 8
Cost ($)

6
4
2
0
0 1 2 3 4 5 T
Time (years)
Continued over page
368 M a t h s Q u e s t M a t h s B Ye a r 1 1 f o r Q u e e n s l a n d

Note: xy is the power function on a scientific calculator. It may be represented as y x or a x.


On a graphics calculator, use the power button ^ ; for example, 5 ^ 3 = 53 = 125.
The other alternative to that outlined above when finding a set of continuous points is
simply to multiply by the growth or compounding factor each time.

Alternative — Using a graphics calculator


THINK DISPLAY
For the Casio fx-9860G AU
1. Press MENU and then select TABLE. Enter the
equation in terms of Y (in place of C) and X (in
place of T). Press EXE .

2. Press F5 (SET) and fill in the appropriate


values for Start (0), End (5) and Step (1).

3. Press EXE until you return to the Table function


screen. Press F6 (TABL) to display the table. Use
the arrow keys to scroll through the table of values.
Note: To calculate more X values, press F4
(EDIT) and then enter the X value of your choice.
Press EXE to display the corresponding Y1 value
at the top of the table.

4. Press MENU and then select GRAPH. The


equation for Y1 will automatically be shown.

5. Press SHIFT F3 (V-WIN) and set the View


Window to appropriate values.

6. Press EXE until you return to the Graph


function screen. Press F6 (DRAW) to display
the graph. (If you receive an error message,
press F1 (SEL) and then F6 (DRAW).)
Press SHIFT F1 (TRCE) to investigate
values for C.
Chapter 8 Applications of exponential and logarithmic functions in financial mathematics 369
For the TI-Nspire CAS
1. Open a new Lists & Spreadsheet document. If
you wish, enter a title for column A and column
B. Enter the T values in column A, then move
the cursor to the formula box in column B.

2. Press = and then enter the formula 0.8(1.6)^.


Instead of entering T, we need to indicate that
the values from column A are to be used to
generate the corresponding values in column B.
Enter the title of the column time by pressing
TIME (or press A/( to enter a[] to
indicate column A). Then move the cursor to
the far right of the expression.

3. Press · to display the table values.

4. To display these points on a graph, you can use


the Quick Graph option. First, highlight the
two columns (hold down g to select more
than one column). Press b and then select
3: Data followed by 5: Quick Graph.

5. To display the full-screen graph, press c and


then select 5: Data and Statistics. Use the
NavPad to move to the horizontal axis and press
a. Select the required variable (time). Repeat
this for the vertical axis and select the required
variable (cost). To join the points with a line,
press b, select 2: Plot Properties and then
1: Connect Data Points.
370 M a t h s Q u e s t M a t h s B Ye a r 1 1 f o r Q u e e n s l a n d

Meaning of growth or compounding factor


The growth or compounding factor takes into account the quantity that we start with as
well as the amount of the increase for the unit time interval.
If the growth or compounding factor is 1.15 then the 1 accounts for the initial
quantity and the 0.15 accounts for the increase.
That is: 1.15 = 1 + 0.15
15
= 1 + --------
-
100
= 100% + 15%
= 115%
So a growth or compounding factor of 1.15 means an increase each unit time interval
of 15% of the previous value.
For example, if the cost of an article increases by 5% per annum (p.a.) then the
growth or compounding factor would be 1.05 (or 100% + 5% = 105%) each year.

WORKED Example 9
The cost, C ($), of a $6.50 cricket ball increases by 3% each year. Write the equation for
the relationship between the cost and time, T (years), and use it to find the cost of the ball
after 8 years.
THINK WRITE
1 Find the growth or compounding factor Growth factor = original amount + increase
per year. = 100% + 3%
= 103%
= 1.03
2 Write the equation in the form y = ka x, C = 6.5(1.03)T
where k is the initial value.
3 Substitute T = 8 and evaluate. When T = 8,
C = 6.5(1.03)8
C = $8.23
4 Write a summary statement. Cost of ball after 8 years is $8.23.

Decay
If a quantity decreases in size over a period of time, it is decaying. This decay process
may also be linear in nature or exponential, as was the case with growth.
Straight line decay
In this situation, a quantity decreases by a fixed amount for each time unit interval; that
is, a fixed amount is subtracted. Since the quantity is decreasing over time the slope of
the straight line is negative.
Suppose the number of starfish on a reef is 8000 at N
8000
Number of starfish

present but the population is decreasing by 250 each year.


The decay relationship can be written as the equation:
N = 8000 − 250T where N = number of starfish 7000
T = time in years.
A graph of the equation like the one shown could be 6000
drawn to represent this situation.
The graph or the equation can be used in the same way 50000
0 2 4 6 8 10 T
they were used for growth situations (see Worked example 6). Time (years)
Chapter 8 Applications of exponential and logarithmic functions in financial mathematics 371
Exponential decay
Decay can be exponential in a similar way to growth. That is, the quantity present can
be multiplied by the growth or compounding factor for each unit time interval. It may
seem to be a contradiction to say that there is a growth or compounding factor associ-
ated with decay; however, for decay the growth or compounding factor, a, is less than
one, whereas for exponential growth the growth or compounding factor is greater than
one. In comparison,
the general equation for exponential growth is:
y = ka x where k, a are constants, a >1
the general equation for exponential decay is:
y = ka x where k, a are constants, a <1
The procedure that was adopted to analyse situations of exponential growth can be
applied in the same way to exponential decay.

WORKED Example 10 v
The graph shows how the value of a car decreases 20 000
over a 5-year period. 18 000
16 000
a Determine whether the decay is exponential. 14 000
b If it is exponential, write an approximate Value ($) 12 000
equation for the relationship between value, v, 10 000
8 000
and time, T. 6 000
4 000
2 000
0
0 1 2 3 4 5 T
Time (years)
THINK WRITE
v
a 1 Find v0 and v1. Evaluate ----1- . a From the graph, v0 = 20 000,
v0
v1 = 12 000
v 1 12 000
----- = ----------------
v 0 20 000
= 0.6
2 Find v2. If exponential decay then 0.6 is growth v1 = 12 000
or compounding factor. v2 = 12 000 × 0.6
So, v1 × 0.6 = v2. = 7200
3 Find v3. It should be that v2 × 0.6 = v3. v3 = 7200 × 0.6
= 4320
4 Find v4. It should be that v3 × 0.6 = v4. v4 = 4320 × 0.6
= 2592
5 By using 0.6 as the growth or compounding Decay factor is 0.6.
factor, the calculated values compare favourably Decay is exponential.
with the values from the graph.
b The decay equation is of the form y = ka x where b v = 20 000(0.6)T
a < 1. That is, the initial value, v0, is multiplied by
the growth or compounding factor, 0.6, once for
each unit time interval.
372 M a t h s Q u e s t M a t h s B Ye a r 1 1 f o r Q u e e n s l a n d

Meaning of growth or compounding factor


As previously mentioned, the growth or compounding factor takes into account the
initial quantity and, in the case of decay, also takes into account the decrease for the
unit time interval.
Consider a growth or compounding factor of 0.85.
0.85 = 1 − 0.15
15
= 1 − --------
-
100
= 100% − 15%
= 85%
That is, 1 represents the initial quantity and 0.15 represents the actual decrease per
unit time interval.
So a growth or compounding factor of 0.85 means a decrease each unit time interval
of 15% of the previous value.

WORKED Example 11
The number, N, of tigers in a certain population is decreasing by 6% each year from an
initial population of 425. Write an equation for the relationship between the number and
time, T (years), and use it to find how many tigers there will be after 8 years.
THINK WRITE
1 Find the compounding factor per year. Compounding factor
= original amount − decrease
= 100% − 6%
= 0.94

2 Write the equation in the form y = ka x, N = 425(0.94)T


where k is the initial value.

3 Substitute T = 8 and evaluate. When T = 8,


N = 425(0.94)8
N = 259

4 Write a summary statement. After 8 years there will be 259 tigers.

Radioactive decay
Radioactive decay is an example of exponential decay. Nuclear radiation is emitted
from many different chemical elements. Uranium and plutonium are probably the best
known radioactive elements.
The original element decays to a different element over a period of time, which
means that the amount of the radioactive element decreases. For example, uranium-238
decays to thorium-234 and an alpha particle is emitted.
For the purposes of this work on growth and decay, all we need to remember is
that, as with other cases of exponential decay, the rate of decrease (in mass or
amount of the element in this case) is determined by the growth or compounding
factor, which, of course, is less than one. The compounding factor is unique to each
radioactive element.
Chapter 8 Applications of exponential and logarithmic functions in financial mathematics 373
remember
1. Growth and decay can be linear or exponential.
2. Linear growth and decay can be represented by the equation
y = a + bx where y is the dependent variable
x is the independent variable (usually time)
a is the initial or starting value of y
b is the rate of growth or decay.
3. Exponential growth and decay means an initial value multiplied by a growth or
compounding factor for each unit time interval.
4. Exponential growth and decay can be represented by the equation
y = kax where a = growth or compounding factor
a > 1 for growth, a < 1 for decay
k = the initial or starting value of y.

8C Growth and decay functions

WORKED 1 Write an equation that describes the relationship between the variables in each case,
Example
6
then solve each of the following problems.
a The value of a Beatles record, currently worth $50, will increase by $10 every
year. Assuming this relationship will continue indefinitely:
i what will it be worth in 15 years’ time?
ii when will its value have doubled?
b From an initial population of 600 ants in a nest, the number grows by 30 each
eBook plus
month indefinitely.
Digital docs: i What will be the population in 2 years’ time?
SkillSHEET 8.1 ii When will the population reach 900?
Solving indicial
equations
c If $1200 is invested for 10 years and earns simple interest of $120 each year:
EXCEL Spreadsheet
Function grapher
i what will be the amount altogether after 8 years?
ii when will the total amount ($1200 + interest) be $1800?
d A coin collection, currently valued at $1560, will increase in value by 5% of the
current value each year. Assuming the trend will continue:
i what will its value be in 8 1--- years?
2
ii when will its value reach $2000?

2 Verify your answers to question 1 by graphing each relationship.

WORKED 3 Determine whether the situations described below represent exponential growth.
Example
7a a The price of a certain food item over a 5-year period is detailed below.
Year 1 2 3 4 5
Price ($) 0.85 1.02 1.43 2.00 2.40
374 M a t h s Q u e s t M a t h s B Ye a r 1 1 f o r Q u e e n s l a n d

b The value of a coin collection over a period of 5 years is shown below.


Year 1 2 3 4 5
Value ($) 500 550 605 665.50 732.05

c The number of rabbits in a population over a 3-month period is shown in the graph.
N
800

Number of rabbits
600

400

200

0
0 1 2 3 T
Time (months)

d The amount in an investment account after each of 6 years is shown below.


Amount in account, A ($)

800 759.38

600 506.25
400 337.50
225
200 150
100
0
2003 2004 2005 2006 2007 2008
Year

WORKED 4 Using T to represent time in hours, write an equation to describe the increase in
Example
7b number, N, of a population of bacteria if initially there are:
a 1000 bacteria and the number increases by a factor of 2 each hour
b 2000 bacteria and the number increases by a factor of 1.4 each hour
c 860 bacteria and the number increases by a factor of 1.25 each hour
d 1250 bacteria and the number increases by 150% each hour
e 2300 bacteria and the number increases by 200% each hour.
5 Using T to represent time in years, write an equation to describe the increase in value,
V ($), of a painting if it was bought for:
a $700 and its value increased by a factor of 1.1 each year
b $1100 and its value increased by a factor of 1.05 each year
c $5000 and its value increased by a factor of 1.16 each year
d $2750 and its value increased to 120% each year
e $380 and its value increased to 108% each year.
6 Using T to represent time in years, write an equation to describe the increase in cost,
C ($), of buying:
a a $25 000 new car if the cost of purchase grew by a factor of 1.07 each year
b a $1.50 loaf of bread if its cost grew by a factor of 1.03 each year
c a $250 bike if its cost grew by a factor of 1.05 each year
d a new $29.95 DVD if its cost grew to 106% each year
e a $7.20 glossy magazine if its cost grew to 110% each year.
Chapter 8 Applications of exponential and logarithmic functions in financial mathematics 375
7 Using T to represent time in years, write an equation to describe the increase in the
amount, A ($), in an investment account if it was initially:
a $2000 and it increased by a factor of 1.16 each year
b $850 and it increased by a factor of 1.12 each year
c $1900 and it increased by a factor of 1.06 each year
d $25 000 and it increased to 109% each year
e $12 600 and it increased to 115% each year.

WORKED 8 a The amount, A ($), in an investment account after time, T (years), is given by the
Example
8 equation, A = 1500(1.08)T. Using this equation, copy and complete the table below
and plot a graph of amount against time.
T 0 1 2 3 4 5 6 7
eBook plus
A
Digital doc:
EXCEL Spreadsheet
b The value, V ($), of an antique chair over time, T (years), is given by the equation
Plotting relations V = 850(1.06)T. Use the equation to complete the table below and plot a graph of
value against time.
T 0 1 2 3 4 5 6 7
V
c The number, N, of possums in a national park over time, T (months), is given by
the equation, N = 400(1.02)T. Use the equation to find the values of N for T values
from 0–7 and plot a graph of number against time.
d If the cost, C ($), of a new car is given by C = 17 000(1.1)T, where T is the time in
years, plot a graph of cost against time for 0–5 years.

9 Given the exponential growth equations below, solve the problems provided.
eBook plus
a If N = 650(1.59)T, find N if T = 5.
Digital doc: b If C = 210(1.15)T, find C if T = 9.
SkillSHEET 8.2
Transposition of
c If A = 3600(1.09)T, find A ($) if T = 7 years.
formulae
d If V = 1050(1.02)T, find V ($) if T = 20 years.
e If N = 2500(1.85)T, find N (number of bacteria) if T = 12 hours.

WORKED 10 The amount, A ($), in an investment account is initially $2000 and increases by
Example
9
8% p.a.
a Write the equation for the relationship between the amount and time, T (years).
b Use the equation to find the amount in the account after 6 years.

11 The value, V ($), of a piece of art bought for $12 000 increases by 4% each year.
a Write an equation for the relationship between the value and time, T (years).
b Use the equation to find how much the art is worth after 10 years.

12 multiple choice
The number, N, of bacteria in a colony increases by 10% per hour from an initial
colony of 1200. After 15 hours the number of bacteria present would be closest to:
A 250 B 1320 C 1350 D 3000 E 5010
376 M a t h s Q u e s t M a t h s B Ye a r 1 1 f o r Q u e e n s l a n d

13 multiple choice
The cost, C ($), of a can of soft drink, which currently is $1.15, increases by 5.5% p.a.
The cost of the can in 20 years’ time will lie between:
A 30 and 50 cents B $2.10 and $2.30 C $2.30 and $2.50
D $3.30 and $3.50 E $3.50 and $3.70

14 multiple choice
The equation representing an increase of 9% p.a. in the number of animals, N, in a
certain population which initially numbered 3600 is:
A N = 3600T + 9 B N = 3600T − 9 C N = 3600(1.09)T
T T
D N = 3600(0.91) E N = 9(3600)

15 By using the equation that exists between the variables in each case below, solve the
given problems.
a The number of elephants in a game reserve was initially 500, but the population is
decreasing by 35 per year. If this trend continues:
i what will be the population in 12 years’ time?
ii when will the population have halved?
b The value of a car, currently worth $12 000, is decreasing by $800 per year. If this
continues:
i what will be its value in 6 years’ time?
ii when will its value be $8800?
c A cylindrical tank, containing water to a height of 3 m, has just been punctured
and water is leaking out so that the water height is falling by 5 cm every minute. If
this trend continues:
i what will be the water height after 15 minutes? (Assume the puncture is below
this height.)
ii when will the height reach 2.65 m?
d The number of wombats on an island is decreasing by a fixed number each year
from an initial population of 366. If this trend continues and the population
reaches 281 after 5 years:
i what is the decay rate per year?
ii what will be the population after 8 years?
iii when will the population reach 162?
e The value of a computer bought for $2500 decreases by a fixed amount each year.
If the value after 4 years is $1740 and this trend continues:
i what is the decay rate?
ii what will be the computer’s value after 7 years?
iii when will its value be $1930?

16 Verify your answers to question 15 by graphing each relationship.

WORKED 17 Determine whether the situations described below represent exponential decay.
Example
10a
a The value of a car over a 5-year period is detailed below.
Year 1 2 3 4 5
Value ($) 30 000 24 000 19 200 15 360 12 288
Chapter 8 Applications of exponential and logarithmic functions in financial mathematics 377
b The number in a colony of frogs over a 4-year period is described below.
Year 1 2 3 4
Number 2000 1800 1620 1458
c The value of a computer over a 6-year period is graphed below.
V
8000
8000
7000
6000
Value ($'000)

6000
5000 4800
4000 3375
3000 2531.25
2000 1898.44
1000
0
2003 2004 2005 2006 2007 2008 T
Year

d The mass of a radioactive element present over a m

Mass of radioactive element (g)


4 time-unit interval is given at right. Does this 800
situation represent exponential decay? 700
600
500
400
300
200
100
0
0 1 2 3 4 5 t
Time units
WORKED 18 In each case below, write an equation to represent the decrease in the given variable
Example
10b
over time:
a Amount, A, decreases from 300 by a compounding factor of --1- every day.
2
b Value, V, decreases from $5000 by a compounding factor of 0.75 every year.
c Number, N, decreases from 2500 by a compounding factor of 0.95 each month.
d Mass, m, decreases from 900 g to 80% each minute.
e Value, V, decreases from $850 to 92% each year.
f Number, N, decreases from 15 000 by a compounding factor of 75% each year.
19 multiple choice
Which one of the equations below represents a decrease of 17% p.a. in the value,
BV ($), of a boat after time, T (years), if it was bought for $17 800?
A BV = 17 800(1.17)T B BV = 17 800(0.83)T C BV = 17 800T 0.83
D BV = 17 800 − 17T E BV = 17 800 + 17T
20 Given the exponential decay equations below, solve the given problems.
a If N = 700(0.85)T, find N if T = 4. b If A = 10 000(0.92)T, find A if T = 10.
c If V = 1200(0.75)T, find V if T = 7. d If N = 160(0.96)T, find N if T = 15.
e If A = 185.5(0.52)T, find A if T = 6.
378 M a t h s Q u e s t M a t h s B Ye a r 1 1 f o r Q u e e n s l a n d

21 a The mass, m (g), of a radioactive element after time, T (days), is decreasing and is
given by the equation, m = 840(0.76)T. Use this equation to complete the table
below and plot a graph of mass against time.
T 0 1 2 3 4 5 6 7 8
m
b The value, V ($), of a computer after time, T (years), is given by the equation,
V = 2600(0.80)T. Use this equation to complete the table below and plot a graph of
value against time.
T 0 1 2 3 4 5 6 7 8
V
c The number, N, in a population of seahorses after time, T (months), is given by the
equation, N = 290(0.91)T. Use the equation to find the values of N for T values
from 0–8 and plot a graph of number against time.
d The amount, A (grams), of a radioactive element after time, T (minutes), is
decreasing and is given by the equation, A = 1350(0.70)T. Use the equation to find
the values of A for T values from 0–8 and plot a graph of amount against time.

WORKED 22 The number, N, of sulfur-crested cockatoos in a certain population is decreasing by


Example
11
2% each month from an initial number of 1400.
a Write an equation for the relationship between the number and time, T (years).
b Use the equation to find the number of cockatoos after 1 year.

23 The value, V ($), of a washing machine bought for $899 decreases by 30% p.a.
a Write an equation for the relationship between the value and time, T (years).
b Use the equation to find how much the machine is worth after 5 years.

24 multiple choice
The value, V ($), of a car bought for $27 500 decreases by 20% each year. The value
of the car after 6 years would be closest to:
A $27 300 B $24 400 C $7200 D $5000 E $300

25 multiple choice
The mass, m (g), of a radioactive sample of sodium decreases from an initial mass of
660 g by 5% each hour. The mass of the sodium left after 24 hours would lie within
the range:
A 610–630 g B 530–550 g C 510–530 g
D 180–200 g E 50–70 g

26 A 10-year research program is being carried out on two penguin rookery populations.
Rookery A had 2000 penguins at the start of the study and the population has been
decreasing by 5% p.a. Rookery B had 3000 penguins initially and this population has
been decreasing by 10% p.a.
a On the same set of axes draw population against time graphs for the rookeries over
the 10 years.
b From your graphs, estimate when the two populations will be the same.
c When the populations are the same, what is the population?
Chapter 8 Applications of exponential and logarithmic functions in financial mathematics 379
Compound interest formula
PrT
You will recall that simple interest can be calculated using the formula I = ---------- , where
eBook plus 100
I = interest ($)
Digital docs: P = principal invested at the start ($)
SkillSHEET 8.3 r = rate of interest per period
Substitution into the
simple interest
T = time or number of periods.
formula The amount present at the start does not change throughout the life of the invest-
SkillSHEET 8.4 ment. Interest is added at the end.
Simple interest and
arithmetic In contrast, in the previous section graphical and algebraic methods were used to
progressions
illustrate the concept of exponential growth whereby an amount increases at regular
intervals over a period of time. This increasing factor was called the growth or com-
pounding factor.
Another example of exponential growth is compound interest.
For investments, interest is added to the initial amount (principal) at the end of an
interest-bearing period. Both the interest and the principal then earn further interest
during the next period, which in turn is added to the balance. This process continues for
the life of the investment. The interest is said to be compounded.
The result is that the balance of the account increases at regular intervals and so too
does the interest earned.
Compound interest is illustrated in the next example.
Consider $1000 invested for 4 years at an interest rate of 12% p.a. with interest com-
pounded annually (added on each year). What will be the final balance of this account?
Time Starting principal, Interest Balance
period P ($) ($) ($)
1 1000 12% of 1000 = 120 1120
2 1120 12% of 1120 = 134.40 1254.40
3 1254.40 12% of 1254.40 = 150.53 1404.93
4 1404.93 12% of 1404.93 = 168.59 1573.52
So the balance after 4 years is $1573.52.
During the total period of an investment, interest may be compounded many times,
so a formula has been derived to make calculations easier.
In the above example the principal is increased by 12% each year. That is, the end of
year balance = 112% or 1.12 of the start of the year balance.
Now let us look at how this growth or compounding factor of 1.12 is applied in the
example.
Time
period Balance ($)
1 1120 = 1000 × 1.12 = 1000(1.12)1
2 1254.40 = 1120 × 1.12 = 1000 × 1.12 × 1.12 = 1000(1.12)2
3 1404.93 = 1254.40 × 1.12 = 1000 × 1.12 × 1.12 × 1.12 = 1000(1.12)3
4 1573.52 = 1404.93 × 1.12 = 1000 × 1.12 × 1.12 × 1.12 × 1.12 = 1000(1.12)4
If this investment continued for n years the final balance would be:
1000(1.12)n = 1000(1 + 0.12)n = 1000 1 + ( )
12 n
---------
100
380 M a t h s Q u e s t M a t h s B Ye a r 1 1 f o r Q u e e n s l a n d

The answer now is only in terms of information that was known at the start of the
investment. From this pattern we are able to write a general formula that can be used to
calculate compound interest.
A = PRn where A = final or total amount ($)
P = principal ($)
r
R = growth or compounding factor = 1 + ---------
r = interest rate per period 100
n = number of interest bearing periods
Note that, unlike the simple interest formula, the compound interest formula gives
the total amount in an account, not just the interest earned.

To find the total interest compounded, I:


I = A − P where A = final or total amount ($)
P = principal ($)
Now let us consider how the formula is used.

WORKED Example 12
Find the amount in the account (balance) and the interest earned after $5000 is invested
for 4 years at 6.5% p.a., interest compounded annually.
THINK WRITE
1 What is n? n =4
2 What is r? r = 6.5
3 What is P? P = 5000
r
4 Write the compound interest (CI) formula. A = P(1 + --------- )n
100
6.5
5 Substitute known values into the formula. = 5000(1 + --------- )4
100
6 Simplify. = 5000(1.065)4
7 Evaluate (to 2 decimal places). A = $6432.33
8 Subtract the principal from the balance. I =A−P
= 6432.33 − 5000
= $1432.33
9 Write a summary statement. The amount of interest earned is $1432.33
and the balance is $6432.33.

In the last example interest was compounded annually. However, in many cases the
interest is compounded more often than once a year, for example semi-annually (twice
each year), quarterly (every 3 months), or weekly. In these situations n and r still have
their usual meanings and we calculate them as follows.
Number of interest periods, n
= number of years × number of interest periods per year
nominal interest rate per annum
Interest rate per period, r = -------------------------------------------------------------------------------------------
number of interest periods per year
Note: Nominal interest rate per annum is simply the annual interest rate advertised by a
financial institution.
Chapter 8 Applications of exponential and logarithmic functions in financial mathematics 381
WORKED Example 13
If $3200 is invested for 5 years at 6% p.a., interest compounded quarterly, find:
a the number of interest bearing periods, n
b the interest rate per period, r
c the balance of the account after 5 years.

THINK WRITE
a Calculate n. a n = 5 (years) × 4 (quarters)
= 20

6% p.a.
b Convert % p.a. to % per quarter to match b r% = ------------------
time over which interest is calculated. 4
r% = 1.5% per quarter
Divide r% p.a. by the number of
compounding periods per year, namely 4. %r = 1.5
Write as a decimal.

c 1 What is P? c P = $3200
r
2 Write the CI formula. A = P(1 + --------- )n
100
1.5
Substitute known values. = 3200(1 + --------- )20
3 100
4 Simplify. = 3200(1.015)20
5 Evaluate to 2 decimal places. A = $4309.94
6 Write a summary statement. Balance of account after 5 years is $4309.94.

On occasions when interest is compounded monthly or more often, the value of r is


a recurring decimal. Now the accuracy of the calculation should be maintained, so the
value that has been determined for r is not to be approximated or truncated. Therefore,
the order of the calculation on a calculator is changed slightly.

WORKED Example 14
Find the amount that accrues in an account which pays compound interest (compounded
weekly) at a nominal rate of 5.6% p.a. if $2450 is invested for 3 1--- years.
2

THINK WRITE
1 Calculate n. n = 3.5 × 52
= 182
5.6
2 Calculate r and retain on the calculator for r = -------
step 5. 52
= 0.107 69
3 What is P? P = $2450

Continued over page


382 M a t h s Q u e s t M a t h s B Ye a r 1 1 f o r Q u e e n s l a n d

THINK WRITE
r
4 Write the CI formula and substitute. A = P(1 + --------- )n
100
182
= 2450(1 + 0.10769
--------------------- )
100
5 Evaluate to 2 decimal places. A = $2980.18
6 Subtract the principal from the balance. I =A−P
= 2980.18 − 2450
= $530.18
7 Write a summary statement. Amount accrued after 3 --1- years is $530.18.
2

Note: The order of calculation used here can be applied in all previous cases when
finding A; see Worked examples 12 and 13.
The situation often arises where we require a certain amount of money by a future
date. It may be to pay for a holiday or to finance the purchase of a car. It is then
necessary to know what principal should be invested now so that it will increase in
value to the desired final balance within the time available.

WORKED Example 15
Find the principal that will grow to $4000 in 6 years, if interest is added quarterly at
6.5% p.a.
THINK WRITE
1 Calculate n. n = 6 × 4 = 24
Calculate r. 6.5
2 r = -------
4
= 1.625
3 What is A? A = $4000
r
4 Write the CI formula, substitute and A = P(1 + --------- )n
simplify. 100
1.625
4000 = P(1 + ------------- )24
100
4000 = P(1.01625)24
Transpose to isolate P. 4000
5 P = ----------------------------
( 1.01625 ) 24
6 Evaluate to 2 decimal places. P = $2716.73
7 Write a summary statement. $2716.73 would need to be invested.

Sometimes we know how much we can afford to invest as well as the amount we
want to have at a future date. Using the compound interest formula we can calculate the
interest rate that is needed to increase the value of our investment to the amount we
desire. This allows us to ‘shop around’ various financial institutions for an account
which provides the interest rate we want.
Chapter 8 Applications of exponential and logarithmic functions in financial mathematics 383
We must first find the interest rate per period, r, and convert this to the corresponding
nominal rate per annum.

WORKED Example 16
Find the interest rate per annum (to 2 decimal places) that would enable an investment of
$3000 to grow to $4000 over 2 years if interest is compounded quarterly.
THINK WRITE
1 What are A, P and n? For this example n A = $4000
needs to represent quarters of a year and P = $3000
therefore r will be evaluated in % per n =2×4
quarter. =8
2 Write the CI formula and substitute. A = PRn
4000 = 3000R8
4000
3 Divide A by P. ------------ = R 8
3000
1 1
--- ---
4 Obtain R to the power of 1, that is, raise  4--- 8 = ( R 8 ) 8 = R
 3
both sides to the power of 1--- .
8 1
---
 4--- 8 = 1 + --------
r
-
 3 100
1
---
r 4 8
5 Isolate r and evaluate. --------- =  --- − 1
100  3
= 0.0366
r = 3.66
r% = 3.66% per quarter
6 Multiply r by the number of interest Annual rate = r% per quarter × 4
periods per year to get the annual rate = 3.66% per quarter × 4
(to 2 decimal places). = 14.65% per annum
7 Write a summary statement. Interest rate of 14.65% p.a. is required.

WORKED Example 17
Calculate the number of interest bearing periods, n, required and hence the time it will
take $3600 to amount to $5100 at a rate of 7% p.a., with interest compounded quarterly.

THINK WRITE
1 What are A, P and r? A = 5100
P = 3600
r = 7---
4
= 1.75
Continued over page
384 M a t h s Q u e s t M a t h s B Ye a r 1 1 f o r Q u e e n s l a n d

THINK WRITE
n
r
2 Substitute into the formula and simplify. A = P  1 + ---------
 100
Retain 1.416 66 on your calculator display
5100 = 3600(1.0175)n
for step 6.
1.0175n = 1.416 66
3 Take the log of both sides. log 1.0175n = log 1.416 66
4 Apply the log rule and isolate n. n log 1.0175 = log 1.416 66
log 1.416 66
n = -------------------------------
log 1.0175
5 Evaluate n. n = 20.08 quarters
6 As n represents quarters, raise n to the As the interest is compounded quarterly,
next integer. n = 21 quarters.
7 Write the time in more meaningful terms. Time = ------ years = 5 1--- years.
21
4 4
8 Write a summary statement. After 5 1--- years, $3600 will amount to $5100.
4

remember
1. Compound interest calculations can be made using the formula
A = PRn where A = final amount ($)
P = principal ($)
r
R = growth or compounding factor = 1 + ---------
100
r = interest rate per period
n = number of interest bearing periods.
2. For compound interest, I = A − P.

8D Compound interest formula

WORKED 1 Use the compound interest formula to find the amount, A, when:
Example
12
a P = $500, n = 2, r = 8 b P = $1000, n = 4, r = 13
c P = $3600, n = 3, r = 7.5 d P = $2915, n = 5, r = 5.25
e P = $850.20, n = 10, r = 1 f P = $1215, n = 24, r = 0.5

eBook plus 2 Find: i the balance, and ii the interest earned (interest compounded annually) after:
a $2000 is invested for 3 years at 8% p.a.
Digital doc: b $7000 is invested for 4 years at 6% p.a.
EXCEL Spreadsheet
Compound interest
c $6000 is invested for 2 years at 5% p.a.
d $1900 is invested for 5 years at 10% p.a.
WORKED 3 Find the number of interest bearing periods, n, if interest is compounded:
Example
13a a annually for 5 years b quarterly for 5 years
c semi-annually for 4 years d monthly for 6 years
1
e 6-monthly for 4 --- years.
2
Chapter 8 Applications of exponential and logarithmic functions in financial mathematics 385
WORKED 4 Find the interest rate per period, r, if the annual rate is:
Example
a 6% and interest is compounded quarterly
13b
b 4% and interest is compounded half-yearly
c 11% and interest is compounded 6-monthly
d 18% and interest is compounded monthly
e 7% and interest is compounded quarterly.

WORKED 5 Find the balance of the account after:


Example
13c a 2 years if $3000 is invested at 8% p.a., interest compounded quarterly
b 5 years if $2000 is invested at 6% p.a., interest compounded 6-monthly
c 4 years if $5000 is invested at 12% p.a., interest compounded monthly
d 7 years if $1500 is invested at 12% p.a., interest compounded quarterly
e 3 years if $2500 is invested at 7% p.a., interest compounded half-yearly.

WORKED 6 Find the amount of interest that accrues in an account which pays compound interest
Example
14
at a nominal rate of:
a 7% p.a. if $2600 is invested for 3 years (compounded monthly)
b 10% p.a. if $4100 is invested for 2 years (compounded monthly)
c 8% p.a. if $3500 is invested for 4 years (compounded monthly)
d 5% p.a. if $1850 is invested for 3 1--- years (compounded fortnightly)
2
e 11% p.a. if $960 is invested for 5 1--- years (compounded fortnightly).
2

7 multiple choice
The greatest return is likely to be made if interest is compounded:
A annually B semi-annually C quarterly
D monthly E fortnightly

8 multiple choice
If $12 000 is invested for 4 1--- years at 6.75% p.a., compounded fortnightly, the amount
2
of interest that would accrue would be closest to:
A $3600 B $4200 C $5000 D $12 100 E $16 300

9 multiple choice
Which account would provide the best investment opportunity if $2500 was invested
for one year?
A simple interest at 6% p.a.
B interest compounded annually at 6% p.a.
C interest compounded 6-monthly at 6.2% p.a.
D interest compounded quarterly at 6.1% p.a.
E interest compounded monthly at 6% p.a.

10 Peta wishes to invest $3200 for 5 years. By comparing the interest earned, which of
the following would be Peta’s best investment option?
a 11% p.a. simple interest
b compound interest at 10.5% p.a., compounded annually
c compound interest at 10% p.a., compounded monthly
386 M a t h s Q u e s t M a t h s B Ye a r 1 1 f o r Q u e e n s l a n d

11 Cyril has just inherited $10 000 and after spending $910 on a holiday he would like to
invest the balance for 4 1--2- years. He is offered the investment opportunities detailed
below. Which option should Cyril choose?
a simple interest at 9.5% p.a.
b compound interest at 9.4% p.a., adjusted semi-annually
c compound interest at 9.3% p.a., adjusted quarterly

12 Agnes invests $2050 for 4 years. Interest is added quarterly. For the first 2 years the
rate is 7% p.a. and for the remaining 2 years the rate rises to 9% p.a. What interest
would accrue during this time?

13 Alex invests $6185 for 3 years. Interest is added half-yearly. The rate starts at 8.5%
p.a. for the first half of the investment period before it rises to 9.6% p.a. for the
remaining time. What interest would Alex earn from this account?

WORKED 14 Use the compound interest formula to find the principal, P, when:
Example
15 a A = $5000, r = 9, n = 4 b A = $2600, r = 8.2, n = 3
c A = $3550, r = 1.5, n = 12 d A = $6661.15, r = 0.8, n = 36
e A = $5495.74, r = 1.2, n = 48.
15 Find the principal that will grow to:
a $3000 in 4 years, if interest is compounded 6-monthly at 9.5% p.a.
b $2000 in 3 years, if interest is compounded quarterly at 9% p.a.
c $2900 in 3 1--- years, if interest is compounded quarterly at 10.6% p.a.
2
d $5600 in 5 1--- years, if interest is compounded quarterly at 8.7% p.a.
4
e $10 000 in 4 1--- years, if interest is compounded monthly at 15% p.a.
4

16 Find the interest accrued in each case in question 15.

17 multiple choice
Lillian wishes to have $24 000 in a bank account after 6 years so that she can buy a
new car. The account pays interest at 15.5% p.a. compounded quarterly. The amount
(to the nearest dollar) that Lillian should deposit in the account now, if she is to reach
her target, is:
A $3720 B $9637 C $10 109 D $12 117 E $22 320

18 multiple choice
Peter has his heart set on a holiday in 2 years’ time and it will cost him $1700. His
bank account pays interest at a rate of 10.25% p.a., compounded 6-monthly. The
amount that Peter will need to deposit now into this account will lie between:
A $1000 and $1050 B $1150 and $1200 C $1310 and $1360
D $1360 and $1410 E $1500 and $1550

19 Sarah needs $1560 for a new stereo system which she is planning to buy in 1 1--- years’
2
time. Her bank offers a rate of 9.6% p.a. with interest compounded monthly. How
much should she deposit now?
Chapter 8 Applications of exponential and logarithmic functions in financial mathematics 387
20 Glen’s credit union offers an account which pays a rate of 8.4% p.a. with interest com-
pounded monthly. His house extension will cost him $15 000 in 4 1--- years’ time. How
2
much should Glen invest in this account to be able to pay for his extension?
21 Calculate the interest rate per year (to 2 decimal places), given that the interest rate
per period, r, is:
a 2% and interest is compounded quarterly
b 1.5% and interest is compounded quarterly
c 1% and interest is compounded monthly
d 3.5% and interest is compounded semi-annually
e 0.65% and interest is compounded monthly.
WORKED 22 Find the interest rates per annum (to 2 decimal places) that would enable investments of:
Example
16 a $2000 to grow to $3000 over 3 years if interest is compounded 6-monthly
b $8000 to grow to $9000 over 2 years (interest compounded quarterly)
c $12 000 to grow to $15 000 over 4 years (interest compounded quarterly)
d $5000 to grow to $7000 over 5 years (compounded semi-annually)
e $2650 to grow to $3750 over 3 1--- years (compounded quarterly).
2
23 If it takes 2 years for $1460 to grow to $2100, find the annual interest rate (com-
pounded semi-annually).
24 After 3 1--- years $950 has accumulated to $1300. Find the annual interest rate
2
(compounded quarterly).

25 multiple choice
What is the minimum interest rate per annum (compounded quarterly) needed for
$2300 to grow to $3200 in 4 years’ time?
A 6% p.a. B 7% p.a. C 8% p.a. D 9% p.a. E 10% p.a.

26 multiple choice
What will be the minimum annual rate of interest (compounded monthly) needed to
enable $18 500 to accrue $4000 interest in 18 months’ time?
A 12.25% p.a. B 13.25% p.a. C 14.25% p.a. D 15.25% p.a. E 16.25% p.a.
27 Sophie has been told that if you invest $500 at 8% p.a. compounded annually then its
value will double in 9 years. Is this true?
28 At an annual rate of 18.1% (compounded half-yearly) $1000 will double in value in
4 years. Is this true?
29 a ii If you invested $1000 now at 11% p.a. (compounded quarterly), how much
would you have in 10 years’ time?
ii At that time, what annual interest rate would give you $10 000 in a further
10 years (interest compounded quarterly)?
b How would your answers to part a vary if interest was compounded fortnightly?
WORKED 30 Calculate the number of interest bearing periods, n, and hence the time in more
Example
17
meaningful terms when:
a A = $2100, P = $1200, r = 3% per half-year
b A = $4000, P = $3100, r = 4% per 6-month
c A = $13 500, P = $8300, r = 2.5% per quarter
388 M a t h s Q u e s t M a t h s B Ye a r 1 1 f o r Q u e e n s l a n d

d A = $8200, P = $4850, r = 2.25% per quarter


e A = $16 900, P = $9600, r = 1% per month.
31 Calculate the number of interest bearing periods, n, and hence how long it will take
for:
a $7800 to amount to $10 000 at a rate of 8% p.a. (compounded quarterly)
b $2500 to amount to $4600 at a rate of 9% p.a. (compounded half-yearly)
c $800 to amount to $1900 at a rate of 11% p.a. (compounded quarterly)
d $2650 to amount to $4800 at a rate of 10% p.a. (compounded 6-monthly)
e $20 200 to amount to $28 600 at 12% p.a. (credited monthly).
32 Wanda has invested $1600 in an account at a rate of 10.4% p.a., interest compounded
quarterly. How long will it take to reach $2200?
33 Baden has invested $5680 at a rate of 9.4% p.a. with interest credited semi-annually.
How long will this investment take to accumulate to $7000?
34 Stefan has invested $4400 in an account which pays interest at a rate of 12.6% p.a.,
interest credited monthly. What time period is needed to enable this investment to
grow to $7600?
35 Don would like to have $3800 at some time in the near future. He has $2200 that he
can invest now and his bank offers him an account which pays 8.6% p.a., interest
compounded quarterly. How long will it take him to achieve his goal?

36 multiple choice
What will be the least number of interest periods, n, required for $6470 to grow to at
least $9000 in an account with interest paid at 6.5% p.a. and compounded half-yearly?
A 10 B 11 C 12 D 20 E 22

37 multiple choice
What will be the minimum investment period required for $12 750 to result from an
investment of $8000 at a rate of 6.9% p.a., interest credited monthly?
A 6 months B 7 months C 6 years 9 months
D 6 years 10 months E 7 years

38 multiple choice
Rhiannon currently has $2200. She wants to take a holiday to Thailand costing $3860.
Her credit union offers an account which pays compound interest at 9.5% p.a. (cred-
ited semi-annually). If Rhiannon invests her $2200 in this account, the first occasion
that she can afford the holiday is after:
A 3 years B 2 1--- years C 6 years
2
1
D 6 --- years E 13 years
2
39 Jennifer and Dawn both want to save $15 000 for a car. Jennifer has $11 000 to invest
eBook plus
in an account with her bank which pays 8% p.a., interest compounded quarterly.
Digital doc: Dawn’s credit union has offered her 11% p.a., interest compounded quarterly.
WorkSHEET 8.2 a How long will it take Jennifer to reach her target?
b How much will Dawn need to invest in order to reach her target at the same time
as Jennifer? Assume their accounts were opened at the same time.
Chapter 8 Applications of exponential and logarithmic functions in financial mathematics 389
Reducing balance loans

When we invest money with a financial institution the institution pays us interest
because it is using our money to lend to others. Conversely, when we borrow
money from an institution we are using the institution’s money and so it charges
us interest.
In reducing balance loans, interest is usually charged every month by the finan-
cial institution and repayments are made by the borrower also on a regular basis.
These repayments nearly always amount to more than the interest for the same
period of time and so the amount still owing is reduced. Since the amount still
owing is continually decreasing and interest is calculated on a daily balance but
debited monthly, the amount of interest charged decreases as well throughout the
life of the loan.
This means that less of the amount borrowed is paid off
Amount owing

p
in the early stages of the loan compared to the end.
If we graphed the amount owing against time for a loan
it would look like the graph at right. That is, the rate at
which the loan is paid off increases as the loan progresses.
Time
The terms below are often used when talking about reducing balance loans:
Principal, P = amount borrowed ($)
Balance, A = amount still owing ($)
Term = life of the loan (years)
To discharge a loan = to pay off a loan (that is A = $0)

It is possible to have an ‘interest only’ loan account whereby the repayments


equal the interest added and so the balance doesn’t reduce. This option is available
to a borrower who wants to make the smallest repayment possible.
Though the focus of this chapter is reducing balance loans, note that the theory
behind reducing balance loans can also be applied to other situations such as
superannuation payouts, for people during retirement, and bursaries. In each of
these situations a lump sum is realised at the start of a period of time and regular
payments are made during that time. Regular payments are called annuities. So
these situations are often called annuities in arrears because the annuity follows
the realisation of the lump sum.
390 M a t h s Q u e s t M a t h s B Ye a r 1 1 f o r Q u e e n s l a n d

Loan schedules
The first amount of interest is added to the balance of a loan account one month after
the funds are provided to the customer. The first repayment is usually made on the
same day.
Consider a loan of $800 that is repaid in 5 monthly instalments of $165.81 at an
interest rate of 1.2% per month, interest debited each month. A loan schedule can be
drawn for this information, showing all interest debits and repayments.
From the schedule the amount owing after each month is shown and the total interest
charged can be calculated.
For any period of the loan:
Total repayments = Interest paid + Principal repaid

Interest
(1.2% of
Balance at monthly Total owing Balance
start of starting at end of after
month balance) month Repayment repayment
Month ($) ($) ($) ($) ($)

1 800.00 9.60 809.60 165.81 643.79

2 643.79 7.73 651.52 165.81 485.71

3 485.71 5.83 491.54 165.81 325.73

4 325.73 3.91 329.64 165.81 163.83

5 163.83 1.97 165.80 165.80 0.00

Each month, interest of 1.2% of the monthly starting balance is added to that balance
and then the repayment value is subtracted, leaving the starting balance for the next
month. This process continues until the loan is paid off after the 5 months.
Note that the amount of interest charged falls each month and so the amount of
principal paid each month increases as outlined earlier.
Another method can be used to analyse this account, but it doesn’t display interest amounts.
Since the interest rate is 1.2% per month the balance increases by this rate each
month. Recalling the work covered previously about the growth factor, we can write:
r
Growth factor, R = 1 + --------- where 1 represents the original amount and
100
1.2
=1+ --------- r represents the increase per period
100
= 1.012
So:
Balance at start of second month = balance at start of first month × R − repayment
A2 = Α1 × R – Q
where Q is the regular repayment.
Chapter 8 Applications of exponential and logarithmic functions in financial mathematics 391
WORKED Example 18
An $800 loan is repaid in 5 monthly instalments of $165.81 at an interest rate of 1.2% per
month, interest debited each month. Calculate:
a the amount still owing after the 4th month
b the total interest charged during the 5 months.
THINK WRITE

r
a 1 Calculate the growth factor. a R = 1 + ---------
100
r 1.2
R = 1 + --------- = 1 + ---------
100 100
= 1.012
2 Find the balance, A1, at the start of the A1 = A0 × R − Q
2nd month. = 800(1.012) − 165.81
A0 = starting principal A1 = $643.79
A0 = $800
3 Find the balance, A2, at start of the 3rd A2 = A1 × R − Q
month. = 643.79(1.012) − 165.81
A2 = $485.71
4 Continue this process to find A3, A4 A3 = A2 × R − Q
and A5. = 485.71(1.012) − 165.81
A3 = $325.73
A4 = A3 × R − Q
= 325.73(1.012) − 165.81
A4 = $163.83.

5 The amount still owing at the end of the The amount still owing at the end of the
4th month is A4. 4th month is $163.83.
b Total interest = Total repayments − b Total interest = 165.81 × 5 − 800
Principal repaid = 829.05 − 800
= $29.05

As mentioned earlier, institutions usually debit a loan account with interest each
month. In this chapter we also consider situations in which interest is debited fortnightly
and quarterly. The frequency with which a customer can make repayments may be
weekly, fortnightly or monthly. We also consider quarterly repayments.
In all cases in this chapter the frequency of debiting interest will be the same as the
frequency of making repayments, although this is not necessary in practice. It simply
makes calculations easier.
The calculations outlined for monthly repayments would follow exactly the same
pattern for other repayment frequencies.
In Worked example 18, the loan was paid off with only a few repayments. In prac-
tice, the repayment of most loans takes considerably longer than this. The process out-
lined in the example continues throughout any part of the term of the loan.
392 M a t h s Q u e s t M a t h s B Ye a r 1 1 f o r Q u e e n s l a n d

WORKED Example 19
A loan of $16 000 is repaid by monthly instalments of $430.83 over 4 years at an interest
rate of 1.1% per month, interest debited monthly. Calculate:
a the amount still owing after the 5th repayment
b the decrease in the principal during the first 5 repayments
c the interest charged during this time.

THINK WRITE

a r
1 Calculate the growth factor, R. a R = 1 + ---------
100
1.1
= 1 + ---------
100
= 1.011

2 Find the balance, A1, at the end of the A1 = A0 × R − Q


1st month (or the start of the 2nd = 16 000(1.011) − 430.83
month). A1 = $15 745.17
A0 = 16 000, Q = 430.83.

3 i(i) Find A2 from A1. A2 = A1 × R − Q


(ii) Repeat until A5 is found. = 15 745.17(1.011) − 430.83
(A5 is the balance at the end of the A2 = $15 487.54
5th month.) A3 = 15 487.54(1.011) − 430.83
= $15 227.07
A4 = 15 227.07(1.011) − 430.83
= $14 963.74
A5 = 14 963.74(1.011) − 430.83
= $14 697.51

4 Write a statement. The amount owing after 5 months is


$14 697.51.
b 1 The decrease in the principal is the b Decrease in principal
difference between the amount owing = A0 − A5
initially, A0, and after the 5th month, A5. = 16 000 − 14 697.51
= $1302.49

2 Write a statement. The principal has decreased by


$1302.49 in the first 5 months of
the loan.

c 1 Interest charged = Total repayments − c Interest charged = 430.83 × 5 − 1302.49


Interest charged = Principal repaid = $851.66

2 Write a statement. The interest charged during the first


5 months is $851.66.
Chapter 8 Applications of exponential and logarithmic functions in financial mathematics 393
More often than not a financial institution provides the nominal interest rate per year
rather than the interest rate per period. As outlined previously in the compound interest
formula section, the rate per period can be obtained from the nominal annual rate as
follows:
Nominal interest rate per annum
Interest rate per period, r = -------------------------------------------------------------------------------------
Number of interest periods per year
It is important to note that while a loan can be drawn at a certain interest rate, that
rate will generally not remain the same for the life of the loan. This means that when
we consider borrowing, we should be aware that the amount of the repayments may
increase (due to an increase in the interest rate) during the term of the loan. We should
be confident that repayments can be met even if the rate rises.
It has been said that if a potential borrower can maintain repayments for a rate of
11% p.a. over the term of the loan then the borrower can withstand rate changes that
may range from perhaps 5% p.a. to 17% p.a.
Let us now look at how quickly the principal decreases at the end of a loan compared
with the earlier stages.

WORKED Example 20
a A family takes out a loan of $40 000 to extend their home. The loan is made at a rate of
interest of 10% p.a. (debited monthly) and is repaid over 10 years by monthly instal-
ments of $528.60. For the 3rd repayment find:
i the amount of principal repaid
ii the amount of interest paid.
b After 8 years the amount still owing is $11 455.71. Assuming the same conditions apply
as in part a, for the 97th repayment find:
i the principal repaid ii the interest paid.
THINK WRITE

a i 10
1 Calculate the monthly interest rate, r. a i r = ------
12
= 0.833 33% per month
r
2 Calculate the growth factor, R. R = 1 + ---------
100

0.833 33
= 1 + --------------------
100
= 1.008 333 3
3 Calculate the amount owing after A1 = A0 × R − Q
each of the first 3 months — A1, A2 = 40 000(1.008 333 3) − 528.60
and A3. A1 = $39 804.73
A2 = 39 804.73(1.008 333 3) − 528.60
= $39 607.84
A3 = 39 607.84(1.008 333 3) − 528.60
= $39 409.31
Continued over page
394 M a t h s Q u e s t M a t h s B Ye a r 1 1 f o r Q u e e n s l a n d

THINK WRITE

4 Principal repaid = A2 − A3 Principal repaid = 39 607.84 − 39 409.31


(3rd repayment) = $198.53
ii Interest paid = Total repayments − ii Interest = 528.60 × 1 − 198.53
Principal repaid = $330.07
b i 1 Monthly repayment = b i A97 = A96 × R − Q
8 years × 12 payments/year = 96. = 11 455.71(1.008 333 3) − 528.60
So, A96 = $11 455.71. Find A97. = $11 022.57
2 Principal repaid = A96 − A97 Principal repaid = 11 455.71 − 11 022.57
(97th repayment) = $433.14
ii Interest paid = Repayments ii Interest = 528.60 − 433.14
Interest paid = − Principal repaid = $95.46

As mentioned in the introduction, a greater percentage of each repayment made in


the early part of a loan is interest, compared with the repayments toward the end. This
is confirmed by the calculations made in the last example.

In summary, with each of 120 repayments being $528.60;

for the 3rd repayment: interest = $330.07, principal repaid = $198.53

for the 97th repayment: interest = $95.46, principal repaid = $433.14.

That is, the principal decreases faster towards the end of the loan.

remember
1. In a loan schedule:
(a) the interest charged each period increases the amount owed
(b) the repayment each period decreases the amount owed.

r
2. Growth factor, R = 1 + --------- where 1 represents the original amount and
100
r represents the increase per period in %.

3. Balance at the end of the month = balance at start of the month × R − Q


An + 1 = An × R − Q where Q = repayment

4. Total repayments = Interest paid + Principal repaid

Nominal interest rate per annum


5. Interest rate per period, r = -------------------------------------------------------------------------------------
Number of interest periods per year
Chapter 8 Applications of exponential and logarithmic functions in financial mathematics 395

8E Loan schedules

WORKED 1 A loan of $1000 is repaid in five monthly instalments of $206.04 at a rate of


Example
18 1% per month, interest debited monthly. Calculate:
a the amount still owing after the 4th repayment
b the total interest charged during the 5 months.
2 Dimitri takes out a loan of $1500 and repays it in five monthly instalments of
eBook plus
$309.97 at a rate of 1.1% per month, interest debited monthly. Calculate:
Digital doc: a the amount still owing after the 4th repayment
EXCEL Spreadsheet b the total interest charged during the 5 months.
Reducing balance
loans
3 A loan of $2000 is repaid in four quarterly instalments of $525.25 at a rate of
2% per quarter, interest debited quarterly. Calculate:
a the amount still owing after the 3rd repayment
b the total interest charged during the 4 quarters.
4 Gaetana borrows $900 which she repays in five quarterly instalments of $193.72 at
a rate of 2.5% per quarter, interest debited quarterly. Calculate:
a the amount still owing after the 4th repayment
b the total interest charged during the 5 quarters.
5 Josh’s loan of $3000 is repaid in four half-yearly instalments of $807.08 at a rate of
3% per half-year, interest debited half-yearly. Calculate:
a the amount still owing after the 3rd repayment
b total interest charged during the 4 repayments.
6 Rebecca takes out a loan of
$2500 to purchase a new
computer. The loan is repaid in
four 6-monthly instalments of
$696.86 at a rate of 4.5% per
6-months, interest debited
6-monthly. Calculate:
a the amount still owing after
the 3rd repayment
b the total interest charged
during the 4 repayments.
WORKED 7 a A loan of $20 000 is repaid by monthly instalments of $444.89 over 5 years at
Example
19 an interest rate of 1% per month, interest debited monthly. Calculate:
i the amount still owing after the 5th repayment
ii the decrease in the principal during the first 5 repayments
iii the interest charged during this time.
b A loan of $20 000 is repaid by quarterly instalments of $1344.31 over 5 years at
an interest rate of 3% per quarter, interest debited quarterly. Calculate:
i the amount still owing after the 5th repayment
ii the decrease in the principal during the first 5 repayments
iii the interest charged during this time.
396 M a t h s Q u e s t M a t h s B Ye a r 1 1 f o r Q u e e n s l a n d

8 a Jose borrows $30 000 which he repays in fortnightly instalments of $206.45 over
10 years at an interest rate of 0.5% per fortnight, interest debited fortnightly.
Calculate:
i the amount still owing after the 5th repayment
ii the decrease in the principal during the first 5 repayments
iii the interest charged during this time.
b A loan of $30 000 is repaid by quarterly instalments of $1350.84 over 10 years at
an interest rate of 3.25% per quarter, interest debited quarterly. Calculate:
i the amount still owing after the 5th repayment
ii the decrease in the principal during the first 5 repayments
iii the interest charged during this time.
9 a Angela takes out a loan of $20 000 to set up a
catering business. The loan is repaid by monthly
instalments of $664.29 over 3 years at an interest
rate of 1% per month, interest debited monthly.
Calculate:
i the amount still owing after the 5th repayment
ii the decrease in the principal during the first
5 repayments
iii the interest charged during this time.
b Emad borrows $20 000 to establish a pet-minding business. The loan is repaid by
monthly instalments of $325.06 over 8 years at an interest rate of 1% per month,
interest debited monthly. Calculate:
i the amount still owing after the 5th repayment
ii the decrease in the principal during the first 5 repayments
iii the interest charged during this time.
c Hank takes out a loan of $20 000 which he repays in monthly instalments of
$286.94 over 10 years at an interest rate of 1% per month, interest debited
monthly. Calculate:
i the amount still owing after the 5th repayment
ii the decrease in the principal during the first 5 repayments
iii the interest charged during this time.
d In parts a–c above the three loan accounts are the same except for the term. As the
term of the loan increases how does this affect:
i the repayment?
ii the amount still owing after the 5th repayment?
iii the amount of interest paid during the 5 repayments?
10 a Jaques borrows $30 000 which he repays in quarterly instalments of $1373.05
over 8 years at an interest rate of 2.5% per quarter, interest debited quarterly.
Calculate:
i the amount still owing after the 5th repayment
ii the decrease in the principal during the first 5 repayments
iii the interest charged during this time.
b Isabel borrows $30 000 and repays it by quarterly instalments of $1195.09 over
10 years at an interest rate of 2.5% per quarter, interest debited quarterly. Calculate:
i the amount still owing after the 5th repayment
ii the decrease in the principal during the first 5 repayments
iii the interest charged during this time.
Chapter 8 Applications of exponential and logarithmic functions in financial mathematics 397
c George takes out a loan of $30 000 which he repays in quarterly instalments of
$1080.18 over 12 years at an interest rate of 2.5% per quarter, interest debited
quarterly. Calculate:
i the amount still owing after the 5th repayment
ii the decrease in the principal during the first 5 repayments
iii the interest charged during this time.
d In parts a–c above the 3 loan accounts are the same except for the term. As the
term of the loan increases how does this affect:
i the repayment?
ii the amount still owing after the 5th repayment?
iii the amount of interest paid during the 5 repayments?
In questions 11–13 find:
i the amount still owing after the 4th repayment
ii the decrease in the principal during the first 4 repayments
iii the total interest paid during this time.
11 A loan of $50 000 is to be paid by monthly instalments of:
a $525.13 over 15 years at 0.8% per month (interest debited monthly)
b $487.13 over 18 years at 0.8% per month (interest debited monthly)
c $440.33 over 25 years at 0.8% per month (debited monthly)
d $639.22 over 15 years at 1.1% per month (debited monthly)
e $607.15 over 18 years at 1.1% per month (debited monthly)
f $571.46 over 25 years at 1.1% per month (debited monthly).
12 A loan of $60 000 is to be repaid by monthly instalments of:
a $429.86 over 20 years at 0.5% per month (interest debited monthly)
b $472.41 over 20 years at 0.6% per month (interest debited monthly)
c $516.90 over 20 years at 0.7% per month (interest debited monthly)
d $563.20 over 20 years at 0.8% per month (interest debited monthly)
e $635.73 over 20 years at 0.95% per month (interest debited monthly)
f $685.92 over 20 years at 1.05% per month (interest debited monthly).
13 A loan of $60 000 is to be repaid by quarterly instalments of:
a $1292.90 over 20 years at 1.5% per quarter (debited quarterly)
b $1421.02 over 20 years at 1.8% per quarter (debited quarterly)
c $1554.87 over 20 years at 2.1% per quarter (debited quarterly)
d $1694.06 over 20 years at 2.4% per quarter (debited quarterly)
e $1911.89 over 20 years at 2.85% per quarter (debited quarterly)
f $2062.53 over 20 years at 3.15% per quarter (debited quarterly).
14 The loan accounts outlined in question 12 are the same except for the interest rate.
The same applies to question 13. In these cases, as the interest rate increases, what
happens to:
a the repayment?
b the amount still owing after the 4th repayment?
c the amount of interest paid during the 4 repayments?
WORKED 15 a Madako’s loan of $50 000 has interest charged at a rate of 9% p.a. (debited
Example
20
monthly) and it is repaid over 10 years by monthly instalments of $633.38. For the
3rd repayment find:
i the principal repaid
ii the interest paid.
398 M a t h s Q u e s t M a t h s B Ye a r 1 1 f o r Q u e e n s l a n d

b After 8 years the amount still owing is $13 863.96. Assuming the same conditions
apply as in part a, for the 97th repayment find:
i the principal repaid ii the interest paid.
16 a Pina’s loan of $60 000 has interest charged at a rate of 8% p.a. (debited monthly)
and it is repaid over 20 years by monthly instalments of $501.86. For the
3rd repayment find:
i the principal repaid ii the interest paid.
b After 18 years the amount still owing is $11 098.43. Assuming the same conditions
apply as in part a, for the 217th repayment find:
i the principal repaid ii the interest paid.
17 a Katharine’s loan of $80 000 has interest charged at a rate of 12% p.a. (debited
quarterly) and it is repaid over 20 years by quarterly instalments of $2648.94. For
the 3rd repayment find:
i the principal repaid ii the interest paid.
b After 18 years the amount still owing is $18 594.66. Assuming the same conditions
apply as in part a, for the 73rd repayment find:
i the principal repaid ii the interest paid.
18 a Tony and Marietta take out a loan of
$90 000 as part payment on their new
house. The loan is to be repaid over
25 years at 13% p.a. (debited fortnightly)
and with fortnightly instalments of $468.31.
For the 3rd repayment find:
i the principal repaid
ii the interest paid.
b If the principal is reduced to $80 268.49 after 10 years
(use the same conditions as in part a), for the 261st repayment find:
i the principal repaid
ii the interest paid.
c If the principal is reduced to $44 676.17 after 20 years (use the same conditions as
in part a), for the 521st repayment find:
i the principal repaid
ii the interest paid.
19 multiple choice
If the quarterly instalments for a $15 000 loan, which is to be repaid over 4 years, are
$1148.98 and interest is debited quarterly at 2.5% per quarter, the decrease in the
principal in the first year would be (to the nearest dollar):
A $11 786 B $3214 C $1382 D $774 E $375
20 multiple choice
John’s $23 000 loan has interest charged at 9% p.a., debited fortnightly, and is repaid
over 8 years by fortnightly instalments of $155.30. For the 3rd repayment the amount
of interest paid is:
A $13.98 B $75.95 C $76.21 D $79.09 E $155.30
21 multiple choice
The term of a loan is 120 monthly instalments. Which of the following repayments
will reduce the principal by the greatest amount?
A 10th B 20th C 30th D 100th E 110th
Chapter 8 Applications of exponential and logarithmic functions in financial mathematics 399
22 multiple choice
Which of the following loan terms would have the greatest amount of interest debited?
(Assume other conditions are the same.)
A 20 years B 22 years C 14 years D 12 years E 10 years

23 Voula’s loan of $55 000 starts with quarterly repayments of $1396.64 and is due to
run for 15 years at 6% p.a., interest debited quarterly. However, after 1 year the
interest rate rises to 7% p.a. and consequently the quarterly repayments rise to
$1482.84 to maintain the 15 year term.
a What amount is still owing after 2 years?
b What amount would have still been owing after 2 years if the rate had remained at
6% p.a.?
c What would be the difference in interest charged between the two scenarios?

24 Cynthia takes out a loan of $85 000 to set up an outdoors adventure business. She starts
with quarterly repayments of $2300.42 and the loan is due to run for 20 years at 9% p.a.,
interest debited quarterly. However, after 1 year the interest rate falls to 8% p.a. and
consequently the quarterly repayments fall to $2143.88 to maintain the 20 year term.
a What amount is still owing after 2 years?
b What amount would have still been owing after 2 years if the rate had remained at
9% p.a.?
c What would be the difference in interest charged between the two scenarios?
400 M a t h s Q u e s t M a t h s B Ye a r 1 1 f o r Q u e e n s l a n d

Loan schedules using spreadsheets


The most effective way of constructing loan schedules is to use a spreadsheet.

eBook plus You can find this spreadsheet by logging into www.jacplus.com.au and accessing
the weblinks for this chapter but you are encouraged to construct it for yourself.
Digital doc: Then you will have the flexibility to adjust it to your needs. Use the spreadsheet to
EXCEL Spreadsheet answer the following questions.
Loan repayments

1 What balance will be owing on a loan of $80 000 at 8% p.a. with monthly
repayments of $800, after 5 years?

2 Adjust the spreadsheet to answer the following question. What balance will be
owing on a loan of $100 000 at 6.5% p.a. with monthly repayments of $950,
after 5 years?

3 Consider the following loan: $90 000 at 7% p.a. with monthly payments of
$900.
a How many months does it take to reduce the amount owing to 0?
b How much is repaid in total?
c Suppose you are advising people who are about to take out such a loan.
They are not sure whether they should repay $900 each month or $950 each
month. Explain to them, giving details, the consequences of these two
courses of action.
Chapter 8 Applications of exponential and logarithmic functions in financial mathematics 401
Spreadsheets and investing
for the future
With a small change, the spreadsheet used in the previous investigation can also be
used to calculate the future value of an annuity. That is, the value in the future of a
series of regular payments into a fund that pays interest on deposits.
Superannuation is one example of such a situation. Each pay period, an employee
sets money aside to be invested so that when he or she retires they will have
accumulated sufficient wealth to provide for their needs when they no longer
receive a regular income.

This spreadsheet can be found by logging into www.jacplus.com.au and locating


eBook plus the weblinks for this chapter, however, you are encouraged to construct it for
Digital doc:
yourself.
EXCEL Spreadsheet
Annuity
Use this spreadsheet to answer the following questions.
1 A person invests $400 per month into an account paying 8% p.a. interest. How
much will have accumulated after 10 years?
Adjust the spreadsheet to answer the following questions.
2 A person invests $250 per month into an account paying 7% p.a. interest. How
much will have accumulated after 20 years?
3 Jan is 28 years of age and decides to pay a regular amount into an account
paying 9% p.a.. If she wants to retire when she is 55, how much need she invest
per month if she wants to have $1 000 000 when she retires?
402 M a t h s Q u e s t M a t h s B Ye a r 1 1 f o r Q u e e n s l a n d

The annuities formula


In the previous section, step-by-step calculations were made to determine the amount
still owing. The process was restrictive in that the previous balance was needed to
calculate subsequent balances. A method is needed to enable calculation of the amount
still owing at any point in time during the term of the loan.
An annuities formula can be used to enable such calculations to be made. An annuity
is a regular payment. When a consumer borrows money from a financial institution that
person contracts to make regular payments or annuities in order to repay the sum
borrowed over time.
Let us now use, in general terms, the process adopted in the previous section to
develop this annuities formula.
Let P = amount borrowed (principal)
R = growth factor for amount borrowed
r
= 1 + --------- (r = interest rate period)
100
n = number of repayments
Q = amount of regular repayments made per period
An = amount owing after n repayments
Assuming interest is debited to the account before a repayment is credited, then:

A0 = P
A1 = A0R − Q = PR − Q
A2 = A1R − Q = (PR − Q)R − Q

= PR2 − QR − Q = PR2 − Q(R + 1)


A3 = A2R − Q = (PR2 − QR − Q)R − Q
= PR3 − QR2 − QR − Q = PR3 − Q(R2 + R + 1)
A4 = A3R − Q = (PR3 − QR2 − QR − Q)R − Q
= PR4 − QR3 − QR2 − QR − Q = PR4 − Q(R3 + R2 + R + 1)

In general,
An = PRn − Q(Rn − 1 + . . . + R2 + R + 1)
The term in the bracket (Rn − 1 + . . . + R2 + R + 1) is the sum of n terms of a geometric
sequence (refer to the start of this chapter).
First term, a = 1
Common ratio, r = R
Now, the sum of n terms of a geometric sequence is:
a(rn – 1)
Sn = ----------------------
r–1
Hence, in this case,
1( Rn – 1 )
1 + R + R2 + . . . + Rn − 1 = -----------------------
R–1
Q( Rn – 1 )
An = PR n – ------------------------
R–1
Chapter 8 Applications of exponential and logarithmic functions in financial mathematics 403
So, in general, the amount owing in a loan account for n repayments is given by the
annuities formula:

Number of repayments made


Amount still owing Repayment value
Interest rate per period

Q( Rn – 1 ) r
A = PR n – ------------------------ where R = 1 + ---------
R–1 100

Amount borrowed Growth factor

WORKED Example 21
A loan of $50 000 is taken out over 20 years at a rate of 6% p.a. (interest debited monthly)
and is to be repaid with monthly instalments of $358.22. Find the amount still owing after
10 years.

THINK WRITE

1 State the loan amount, P, and regular P = 50 000


repayment, Q. Q = 358.22

2 Find the number of payments, n, n = 10 × 12


interest rate per month, r, and = 120
growth factor, R. 6
r = ------
12
= 0.5
r
R = 1 + ---------
100
0.5
= 1 + ---------
100
= 1.005

Q( Rn – 1 )
3 Substitute into the annuities formula. A = PRn − ------------------------
R–1
358.22 ( 1.005 120 – 1 )
= 50 000(1.005)120 − ---------------------------------------------------
1.005 – 1

4 Evaluate A. A = $32 264.98

5 Write a statement. The amount still owing after 10 years will be


$32 264.98.

Note: If R is a recurring decimal, place the value in the calculator memory and bracket R if
needed when evaluating A.
404 M a t h s Q u e s t M a t h s B Ye a r 1 1 f o r Q u e e n s l a n d

Note that, even though 10 years is the halfway point of the term of the loan, more
than half of the original $50 000 is still owing.
When we consider borrowing money we usually know how much is needed and we
choose a term which requires a repayment that we can afford.
To find the repayment value, Q, the annuities formula is used where A is zero, that is,
the loan is fully repaid. Q is then isolated.
Q( Rn – 1 )
A = PRn − ------------------------
R–1
Q( Rn – 1 )
When A = 0, 0 = PRn − ------------------------
R–1
Q( Rn – 1 )
------------------------ = PRn
R–1
PR n ( R – 1 )
Q = ---------------------------
-
Rn – 1

WORKED Example 22
Rob wants to borrow $2800 for a new hi-fi system from a building society at 7.5% p.a.,
interest adjusted monthly.
a What would be Rob’s monthly repayment if the loan is fully repaid in 1 1--2- years?
b What would be the total interest charged?
THINK WRITE

a 1 (a) Find P, n, r and R. a P = 2800


(b) Store in your calculator memory the n = 18
growth factor, R. 7.5
r = -------
12
= 0.625
0.625
R = 1 + -------------
100
= 1.006 25
2 Substitute into the annuities formula to PR n ( R – 1 )
find the regular monthly repayment, Q. Q = ---------------------------
-
Rn – 1
2800 ( 1.00625 ) 18 ( 1.00625 – 1 )
Q = --------------------------------------------------------------------------
-
1.00625 18 – 1
3 Evaluate Q. Keep the value of Q on Q = $164.9547
your calculator for part b.
4 Write a statement. The monthly regular payment is $164.95
over 18 months.
b 1 Total interest = Total repayments − b Total interest = 164.9547 × 18 − 2800
Amount borrowed = $169.18
2 Write a statement. The total interest on a $2800 loan over
18 months is $169.18.
Chapter 8 Applications of exponential and logarithmic functions in financial mathematics 405

Graphics Calculator tip! Financial


using TVM
calculations

Graphics calculators can perform financial calculations using a function called TVM.
This allows quick analysis of reducing balance loans using the annuities formula. The
table below lists the matching variables used in the annuities formula and the TVM
function of each graphics calculator.

Casio TI-Nspire Annuities


fx-9860G AU CAS formula Definition
n N n Number of payments
I% I or I(%) Annual interest rate
r Interest rate per period
PV PV P Present value = amount borrowed
(or amount invested)
PMT Pmt Q Regular payment amount
FV FV An Future value = amount owing on
loan (or amount saved if it is an
investment) after n payments
P/Y PpY Number of payments per year
C/Y CpY Number of compounds per year

Note: When entering the amount for PV on a graphics calculator, consider whether it is
a loan or an investment. For a loan, the amount borrowed is entered as a negative
number. For an investment, the amount is entered as a positive number.
Steps for solving Worked examples 21 and 22 are shown below.
For the Casio fx-9860G AU
1. Press MENU and then select TVM. A number of
financial options are available to you. (Press
F6 ( ) to view the second page of options.)

2. To access the Compound Interest screen, press


F2 (CMPD). The default values or previous
values are shown.

3. For Worked example 21, enter the appropriate


values as shown, pressing EXE after each entry.
Note that PV is a negative amount as it is a loan.
Skip over FV as this is the amount to be
calculated.
406 M a t h s Q u e s t M a t h s B Ye a r 1 1 f o r Q u e e n s l a n d

4. Press F5 (FV) to display the future value. The


amount still owing is $32 264.98.
(Note: Press F1 (REPT) to return to the previous
screen to make adjustments or perform another
calculation.)

5. For Worked example 22, you need to access the


Compound Interest screen for part a. Enter the
values as shown. Note that FV is set to 0 as the
loan is fully repaid.

6. Press F4 (PMT) to display the payment amount.


The monthly payment is $164.95.

7. For part b of Worked example 22, press


F4 (AMT) to move to the amortization screen.
We need to consider the total interest paid from
the first to last payment, so set PM1 to 1 and PM2
to 18, leaving the other values.

8. Press F4 (ΣINT) to find the sum of the interest


paid. The total interest charged is $169.18.

For the TI-Nspire CAS


1. Open a new Calculator document. Press b and
select 3: Algebra.

2. Select C: Finance Solver. The default values or


previous values are shown. Note that the help
information at the bottom of the screen describes
each item that is highlighted.
Chapter 8 Applications of exponential and logarithmic functions in financial mathematics 407
3. For Worked example 21, enter the appropriate
values as shown. Press e to move to the next
line or to cycle through the items. Note that PV is
a negative amount as it is a loan. Skip over FV for
now as this is the amount to be calculated.
Remember to show PpY as 12 and CpY as 12.
Keep PmtAt at its default setting of END (as
interest is charged at the end of the period). Once
all the values have been entered, press e until
you cycle back to FV.
4. Press · to display the future value. The amount
still owing is $32 264.98.

5. For part a of Worked example 22, enter the values


as shown. This time, skip Pmt for now as this is
the value to be calculated. Note that FV is set to 0
as the loan is fully repaid. Once all values have
been entered, press e to cycle back to Pmt.

6. Press · to display the payment amount. The


monthly payment is $164.95.

7. For part b of Worked example 22, we need to


access the amortization screen. First, press d to
return to the Calculator screen. Press k to
access the catalog and then select 2:
∫ Σ . Scroll
down to Finance and press ·, then scroll down
to Amortization and press ·. Scroll down to
highlight Interest Paid. Notice the order of the
items shown in the help information at the bottom
of the screen.
8. Press · to display SInt in the Calculator screen.
Enter the values in the order: NPm+1 (first
payment period), NPm+2 (final payment period),
N, I, PV, Pmt, FV, PpY, CpY. (Note that there has
been no entry here for PmtAT as the default
setting is END.) Press ) to close the set of
brackets and then press · to display the sum of
the interest over 18 months. The total interest
charged is $169.19.
408 M a t h s Q u e s t M a t h s B Ye a r 1 1 f o r Q u e e n s l a n d

9. You will have noticed that in the Finance section


of the catalog, there is an option titled TVM
Functions (just above Amortization). Highlight
TVM Functions and then press · to show the
items that can be calculated. This provides an
alternate way to calculate N (Number of Periods),
I (Interest Rate per Year), PV (Present Value),
Pmt (Payment Amount) or FV (Future Value).
Follow the help information shown at the bottom
of the screen to enter the data.
10. The screen at right illustrates the TVM Functions
method for Worked example 21 and part a of
Worked example 22. To see the full answer for
the future value you can press /v and ·.

WORKED Example 23
Josh borrows $12 000 for some home office equipment. He agrees to repay the loan over
4 years with monthly instalments at 7.8% p.a. (adjusted monthly). Find:
a the instalment value
b the principal repaid and interest paid during the:
i 10th repayment ii 40th repayment.
THINK WRITE/DISPLAY
a 1 (a) Find P, n, r and R. a P = 12 000
n = 4 × 12
= 48
7.8
r = -------
12
= 0.65
0.65
R = 1 + ----------
100
(b) Store R in your calculator memory. = 1.0065
2 Substitute into the annuities formula to PR n ( R – 1 )
find the monthly repayment, Q. Q = ---------------------------
-
Rn – 1
12 000 ( 1.0065 ) 48 ( 1.0065 – 1 )
= -------------------------------------------------------------------------
-
1.0065 48 – 1
3 Evaluate Q. Q = $291.83
If using the TVM function on a graphics calculator:
For the Casio fx-9860G AU
1. Press MENU and then select TVM. Press
F2 (CMPD) to access the Compound Interest
screen. Enter the appropriate values.
Chapter 8 Applications of exponential and logarithmic functions in financial mathematics 409

THINK WRITE/DISPLAY
2. Press F4 (PMT) to display the payment
amount. (Keep this screen for part b.)

For the TI-Nspire CAS


1. Open a new Calculator document, press k


to access the catalog and then select 2: Σ .
Scroll down to Finance and press ·,
then scroll down to TVM Functions and
press ·. Scroll down to highlight
Payment Amount. Notice the order of the
items shown in the help information at the
bottom of the screen.
2. Press · to display the function tvm.Pmt(
on the Calculator screen. Enter the values
for N, I, PV, FV, PpY and CpY, each
separated by a comma. (Note that there is
no entry here for PmtAT as the default
setting is END.) Press ) to close the set
of brackets and then press · to display
the payment amount. (Keep this screen for
part b.)
4 Write a statement. The monthly repayment over a 4-year
period is $291.83.
b i 1 Find the amount owing after b i
9 months.
(a) State P, n, R. P = 12 000, n = 9, R = 1.0065
Q( Rn – 1 )
(b) Substitute into the annuities A = PRn − ------------------------
R–1
formula.
291.83 ( 1.0065 9 – 1 )
= 12 000(1.0065)9 − -------------------------------------------------
1.0065 – 1
2 Evaluate A9. A
9 9 = $10 024.73

3 Find the amount owing after A10 = 12 000(1.0065)10 −


10 months. Substitute (change n = 9 291.83 ( 1.0065 10 – 1 )
A10 = ---------------------------------------------------
to n = 10) and evaluate. 1.0065 – 1
A10 = $9798.06
4 Principal repaid = A9 − A10 Principal repaid = 10 024.73 − 9798.06
= $226.67
5 Interest paid = Total repayments − Total interest = $291.83 − 226.67
Principal repaid = $65.16
Continued over page
410 M a t h s Q u e s t M a t h s B Ye a r 1 1 f o r Q u e e n s l a n d

THINK WRITE/DISPLAY

If using a graphics calculator:


For the Casio fx-9860G AU
1. Press F4 (AMT) to access the Amortization
screen. As we are interested in payment 10
only, set PM1 as 10 and PM2 as 10.

2. Press F3 (PRN) to display the principal


repaid in the 10th payment.

3. Press F1 (REPT) to return to the


previous screen. Press F2 (INT) to
display the interest paid in the 10th
payment.

For the TI-Nspire CAS


1. From the Amortization option in the
Finance section of the catalog, select
Principal Paid. Enter the data as shown.
Since we are interested in payment 10
only, enter 10 for NPm+1 and 10 for
NPm+2. Press · to display the
principal repaid in the 10th payment.
2. From the Amortization option in the
Finance section of the catalog, select
Interest Paid. Enter the data as shown.
Again, since we are interested in
payment 10 only, enter 10 for NPm+1
and 10 for NPm+2. Press · to display
the interest paid in the 10th payment.
6 Write a statement. In the 10th repayment, $226.67 principal is
repaid and $65.16 interest is paid.
b ii 1 Repeat steps 1–6 for A39 and A40. b ii A39 = 12 000(1.0065)39 −
291.83 ( 1.0065 39 – 1 )
A39 = ---------------------------------------------------
1.0065 – 1
A39 = $2543.10
A40 = $2267.80
Principal repaid = A39 − A40
= 2543.10 − 2267.80
= $275.30
Interest = 291.83 − 275.30
= $16.53
2 Write a statement. In the 40th repayment, $275.30 principal is
repaid and $16.53 interest is paid.
Chapter 8 Applications of exponential and logarithmic functions in financial mathematics 411
remember
1. To calculate the amount in a loan account use the formula:
Q( Rn – 1 )
A = PR n – ------------------------
R–1
2. To calculate the repayment value use the formula:
PR n ( R – 1 )
Q = ----------------------------
Rn – 1
where P = amount borrowed (principal) ($)
R = growth factor for amount borrowed
r
= 1 + --------- (r = interest rate per period)
100
n = number of repayments
Q = amount of regular repayments made per period ($)
An = amount owing after n repayments ($)

8F The annuities formula

1 Use the annuities formula to find A, given:


eBook plus
a P = $50 000, n = 100, Q = $550, r = 1
Digital doc: b P = $50 000, n = 200, Q = $550, r = 1
EXCEL Spreadsheet c P = $60 000, n = 100, Q = $650, r = 1
Reducing balance
loans d P = $60 000, n = 200, Q = $650, r = 1
e P = $20 000, n = 50, Q = $300, r = 0.5
f P = $40 000, n = 100, Q = $400, r = 0.8
g P = $80 000, n = 150, Q = $700, r = 0.75
h P = $100 000, n = 200, Q = $720, r = 0.65.

WORKED 2 A loan of $65 000 is taken out over 20 years at a rate of 12% p.a. (interest debited
Example
21
monthly) and is to be repaid with monthly instalments of $715.71. Find the amount
still owing after:
a 5 years b 10 years
c 15 years d 18 years.

3 Matthew takes out a reducing balance loan of $75 000 over 25 years at a rate of 10%
p.a. (interest debited quarterly) and is to be repaid with quarterly instalments of
$2048.39. Find the amount still owing after:
a 5 years b 10 years
c 15 years d 20 years.

4 A loan of $52 000 is taken out over 15 years at a rate of 13% p.a. (interest debited
fortnightly) and is to be repaid with fortnightly instalments of $303.37. Find the
amount still owing after:
a 4 years b 8 years
c 12 years d 14 years.
412 M a t h s Q u e s t M a t h s B Ye a r 1 1 f o r Q u e e n s l a n d

5 Link borrows $48 000, taken out over 10 years and to be repaid in monthly instal-
ments. (Note: As the interest rate increases, the monthly repayment increases if the
loan period is to remain the same.) Find the amount still owing after 5 years if interest
is debited monthly at a rate of:
a 6% p.a. and the repayment is $532.90
b 9% p.a. and the repayment is $608.04
c 12% p.a. and the repayment is $688.66
d 15% p.a. and the repayment is $774.41.

6 A loan of $20 000 has interest charged monthly at a rate of 9% p.a. What will be the
amount still owing after 3 years if the term of the loan is:
a 4 years and monthly repayments of $497.70 are made?
b 5 years and monthly repayments of $415.17 are made?
c 6 years and monthly repayments of $360.51 are made?
d 7 years and monthly repayments of $321.78 are made?
e 8 years and monthly repayments of $293 are made?

7 Pablo’s loan of $30 000 has interest charged quarterly at a rate of 10% p.a. What will
be the amount still owing after 5 years if the term of the loan is:
a 6 years and quarterly repayments of $1677.38 are made?
b 7 years and quarterly repayments of $1502.64 are made?
c 8 years and quarterly repayments of $1373.05 are made?
d 9 years and quarterly repayments of $1273.55 are made?
e 10 years and quarterly repayments of $1195.09 are made?

8 multiple choice
Peter wants to borrow $8000 for a second-
hand car. His bank offers him a personal loan
for that amount at an interest rate of 13% p.a.,
interest debited fortnightly, with fortnightly
repayments of $124.11 over 3 years. After
2 years he wants to calculate how much he
still owes by using the annuities formula.
a Which of the following equations should
he use?
124.11 ( 1.005 78 – 1 )
A A = 8000 ( 1.005 ) 78 – ------------------------------------------------
1.005 – 1
124.11 ( 1.05 52 – 1 )
B A = 8000 ( 1.05 ) 52 – ---------------------------------------------
1.05 – 1
124.11 ( 1.005 52 – 1 )
C A = 8000 ( 1.005 ) 52 – ------------------------------------------------
1.005 – 1
124.11 ( 1.05 78 – 1 )
D A = 8000 ( 1.05 ) 78 – ---------------------------------------------
1.05 – 1
124.11 ( 0.005 52 – 1 )
E A = 8000 ( 0.005 ) 52 – ------------------------------------------------
0.005 – 1
b The actual amount that Peter still owes after 2 years is closest to:
A $2500 B $3000 C $3500 D $4000 E $4500
Chapter 8 Applications of exponential and logarithmic functions in financial mathematics 413
9 multiple choice
Gwendoline has borrowed $14 000 for renovations to her house.
The terms of this loan are monthly instalments of $297.46 over
5 years with interest debited monthly at 10% p.a. of the outstanding
balance.
a The amount still owing after 3 years is given by:
297.46 ( 1.008 333 3 36 – 1 )
A A = 14 000 ( 1.008 333 3 ) 36 – ---------------------------------------------------------------
1.008 333 3 – 1
297.46 ( 1.008 333 3 60 – 1 )
B A = 14 000 ( 1.008 333 3 ) 60 – ---------------------------------------------------------------
1.008 333 3 – 1
297.46 ( 1.1 60 – 1 )
C A = 14 000 ( 1.1 ) 60 – ------------------------------------------
1.1 – 1
297.46 ( 0.083 333 36 – 1 )
D A = 14 000 ( 1.083 333 ) 36 – -----------------------------------------------------------
0.083 333 – 1
297.46 ( 1.083 333 36 – 1 )
E A = 14 000 ( 0.083 333 ) 36 – -----------------------------------------------------------
1.083 333 – 1
b The actual amount that Gwendoline still owes after 3 years is
closest to:
A $5000 B $5500 C $6000
D $6500 E $7000

10 multiple choice
Ben takes out a loan for $20 000 to buy a new
car. The contract requires that he repay the
loan over 5 years with monthly instalments of
$421.02. After 2 1--- years Ben uses the annu-
2
ities formula to obtain the expression below to
calculate the amount he still owes.
421.02 ( 1.008 30 – 1 )
A = 20 000 ( 1.008 ) 30 – ------------------------------------------------
0.008
The interest rate per annum charged by the bank for this reducing balance loan is:
A 1.008% B 0.008% C 0.096% D 9.6% E 12.096%

11 Use the annuities formula to find the repayment value, Q, given:


a P = $5000, r = 1, n = 12 b P = $3000, r = 2, n = 8
c P = $1500, r = 3, n = 4 d P = $9000, r = 0.5, n = 30
e P = $14 000, r = 0.8, n = 24 f P = $120 000, r = 0.6, n = 240
g P = $95 000, r = 2.5, n = 100 h P = $64 000, r = 0.5, n = 520.

WORKED 12 Sergio’s reducing balance loan of $12 000 has interest charged at 9% p.a., interest
Example
22
adjusted monthly. Find:
i the monthly repayment
ii the total interest charged if the loan is fully repaid in:
a 2 years b 3 years c 4 years d 4 1--- years e 5 1--- years.
2 2
414 M a t h s Q u e s t M a t h s B Ye a r 1 1 f o r Q u e e n s l a n d

13 Conchita’s loan of $85 000 is charged interest at 7% p.a., interest adjusted monthly.
Find:
i the monthly repayment ii the total interest charged
if the loan is fully repaid in:
a 10 years b 12 years c 15 years d 18 years e 20 years f 25 years.
14 In each of questions 12 and 13 the only quantity which varied was the term of the
loan. As the term of the loan increases, what happens to:
a the repayment value?
b the amount of interest paid?
15 Declan borrows $32 000 and contracts to repay the loan over 10 years. Find:
i the repayment value ii the total interest charged
if the loan is repaid quarterly at:
a 6% p.a., interest charged quarterly
b 8% p.a., interest charged quarterly
c 10% p.a., interest charged quarterly
d 10.5% p.a., interest charged quarterly
e 11% p.a., interest debited quarterly
f 12.5% p.a., interest debited quarterly.
16 Felice borrows $46 500 and contracts to repay the loan over 15 years. Find:
i the repayment value
ii the total interest charged
if the loan is repaid fortnightly, with interest adjusted fortnightly at:
a 6% p.a. b 8% p.a. c 10% p.a.
d 10.5% p.a. e 11% p.a. f 12.5% p.a.
17 A loan of $94 000 is to be repaid over 20 years. Find:
i the repayment value ii the total interest charged
if the loan is repaid:
a weekly at 13% p.a., interest adjusted weekly
b fortnightly at 13% p.a., interest adjusted fortnightly
c monthly at 13% p.a., interest adjusted monthly
d quarterly at 13% p.a., interest adjusted quarterly
e weekly at 6.5% p.a., interest adjusted weekly
f fortnightly at 6.5% p.a., interest adjusted fortnightly.

18 Based on your answers to question 17 a–d when the frequency of repayments (and
interest charged) decreases, how does this affect:
a the repayment value?
b the total interest paid?

19 multiple choice
Which of the following would decrease the total amount of interest paid during the
life of a loan? (There may be more than one answer.)
A A fall in the interest rate
B A decrease in the frequency of repayment (repay less often)
C A greater amount borrowed
D A decrease in the term of the loan
E A rise in the interest rate
Chapter 8 Applications of exponential and logarithmic functions in financial mathematics 415
20 multiple choice
Which of the equations below would enable the quarterly repayment value, Q, to be
determined for a loan of $16 000 to be repaid over 5 years at 7.8% p.a., interest
debited quarterly?
Q ( 0.0195 20 – 1 )
A 0 = 16 000 ( 0.0195 ) 20 – ---------------------------------------
0.0195 – 1
Q ( 1.078 5 – 1 )
B 0 = 16 000 ( 1.078 ) 5 – ----------------------------------
1.078 – 1
Q ( 1.0195 5 – 1 )
C 0 = 16 000 ( 1.0195 ) 5 – -------------------------------------
1.0195 – 1
Q ( 1.0195 20 – 1 )
D 0 = 16 000 ( 1.0195 ) 20 – ---------------------------------------
1.0195 – 1
Q ( 1.078 20 – 1 )
E 0 = 16 000 ( 1.078 ) 20 – ------------------------------------
1.078 – 1

WORKED 21 Grace has borrowed $18 000 to buy a car. She agrees to repay the reducing balance
Example
23
loan over 5 years with monthly instalments at 8.1% p.a. (adjusted monthly). Find:
a the instalment value
b the principal repaid and the interest paid during:
i the 10th repayment ii the 50th repayment.
416 M a t h s Q u e s t M a t h s B Ye a r 1 1 f o r Q u e e n s l a n d

22 Tim has borrowed $45 000 to renovate a house. He agrees to repay the reducing bal-
ance loan over 15 years with monthly instalments at 9.3% p.a. (adjusted monthly).
Find:
a the instalment value
b the principal repaid and the interest paid during:
i the 20th repayment
ii the 150th repayment.

23 Gail has agreed to repay a $74 000 reducing balance loan with fortnightly instalments
over 20 years at 9.75% p.a. (adjusted fortnightly). Find:
a the instalment value
b the principal repaid and the interest paid during:
i the 1st repayment
ii the 500th repayment.

24 Terry is repaying a $52 000 loan over 15 years with quarterly instalments at 6.25% p.a.
(adjusted quarterly). Currently, 5 1--- years have passed since the loan was drawn down
2
(money borrowed). How much does Terry still owe?
Chapter 8 Applications of exponential and logarithmic functions in financial mathematics 417
25 Stefanie borrowed $18 000 exactly 3 1--- years ago.
2
The reducing balance loan was for a term of 5 years
and was to be repaid in monthly instalments of
10.2% p.a. (adjusted monthly). How much does
Stefanie still owe?

Questions 26 and 27 refer to the following information.


The interest charged to a housing loan account during a
financial year (1 July–30 June) is a tax deduction
against income if the house is rented to tenants.
26 Bruce borrowed $320 000 to finance the purchase of a rental property and he is
repaying the loan over 20 years by quarterly instalments at 8.6% p.a. (adjusted quar-
terly). By 1 July last year he had made 24 repayments. Find:
a the amount Bruce owed after 24 repayments
b the amount he owes by 30 June this year
c the total interest that Bruce can claim as a tax deduction for this particular
financial year.
27 Lyn is repaying a $220 000 housing loan over
15 years by monthly instalments at 9.9% p.a.
(adjusted monthly). By 1 July last year she had
made 42 repayments. If Lyn rented the house to
tenants, find:
a the amount she owed after 42 repayments
b the amount she owes by 30 June this year
c the total interest that Lyn can claim as a tax
deduction for this particular financial year.

Buying a home
Let us return to the problem of Harry and
Maude’s home loan. For a loan of $100 000 at
8% p.a., what are the benefits of making
fortnightly repayments of $400 instead of
monthly repayments of $800?
We now have three ways of examining this
problem — graphics calculator, the annuities
formula or the spreadsheet.
Use one of these ways to investigate this problem and write a report on your
findings. Include all necessary calculations to support your conclusion.
418 M a t h s Q u e s t M a t h s B Ye a r 1 1 f o r Q u e e n s l a n d

summary
Geometric sequences
• A geometric sequence is one where each successive term is obtained by multiplying
the preceding term by a constant number. This number is called the common ratio
tn + 1
and is given the symbol r. Thus ---------- = r for all values of n. The first term in the
tn
sequence is given the symbol a.
• If x, y, z are successive terms in an arithmetic sequence then y is called a geometric
mean and is given by y2 = xz.
• A geometric sequence can be written as a, ar, ar 2, . . . and so the nth term, tn, is
tn = ar n − 1 using the function notation, or
tn + 1 = rtn, t1 = a using the iterative notation.

Geometric series
The sum of the first n terms in a geometric sequence is given by
a(1 – rn)
S n = ---------------------- with r ≠ 1
1–r
a(rn – 1)
S n = ---------------------- with r ≠ 1
or
r–1
• When the magnitude of r is less than one, that is, −1 < r < 1, the sum of a geometric
a
sequence to infinity S∞ is given by S ∞ = ----------- .
1–r

Applications of arithmetic and geometric sequences and series


• To solve problems, use the following guidelines.
1. Identify the type of sequence by checking whether there is a common
difference, or a common ratio.
2. Translate given information into mathematical statements, using appropriate
notation.
3. Define what you have to find and write appropriate formula(e).
4. Use algebra to find what is required.

Growth and decay functions


• Growth and decay can be linear or exponential.
• Linear growth and decay can be represented by the equation
y = a + bx where y is the dependent variable
x is the independent variable (usually time)
a is the initial or starting value of y
and b is the rate of growth or decay.
Chapter 8 Applications of exponential and logarithmic functions in financial mathematics 419
• Exponential growth and decay means an initial value multiplied by a growth or
compounding factor for each unit time interval.
• Exponential growth and decay can be represented by the equation
y = kax where a = growth or compounding factor
a > 1 for growth, a < 1 for decay
and k = the initial or starting value of y.

Compound interest formula


• Compound interest is an example of exponential growth and is calculated using the
formula:
A = PRn where A = final amount ($)
P = principal ($)
r
R = growth or compounding factor = 1 + ---------
100
r = interest rate per period
n = number of interest bearing periods
• For compound interest, I = A − P

Loan schedules
• For each period in a loan schedule, the amount owed increases interest charged and
decreases repayments made.
• Total repayments = Interest paid + Principal repaid

The annuities formula


• The annuities formula for reducing balance loans:
To find the amount still owing, A
Q( Rn – 1 )
A = PR n – ------------------------ where A = amount owing after n repayments
R–1
P = amount borrowed
Q = repayment value
n = number of repayments made
r
R = 1 + --------- , r = interest rate per period
100
To find the repayment value, Q
PR n ( R – 1 )
Q = ----------------------------
Rn – 1
420 M a t h s Q u e s t M a t h s B Ye a r 1 1 f o r Q u e e n s l a n d

CHAPTER
review
1 multiple choice
8A
For a geometric sequence, the 4th term is 5 and the 7th term is −625. The second term in the
sequence is:
A −2.5 B −1.25 C 0.25 D −0.25 E 0.20

2 Consider the geometric sequence gn = 1.4 × 1.5n − 1; n = 1, 2, 3, . . .


8A,B a Find the first and 5th terms of the sequence.
b Find the sum of the first 10 terms.
c Find the percentage contribution that the 10th term makes to this sum.

3 At Bugas Heights a radiation leak in a waste disposal tank potentially exposes staff to a
8A,B 1000 milli-rem h dose on the first day of the accident, a 800 milli-rem h dose on the second
day after the accident and a 640 milli-rem h dose on the third day since the accident.
a Assuming a geometric sequence, find the amount of potential exposure dose by the 10th
day.
b Find the total potential exposure dose in the first 5 days.

4 multiple choice
8B
The sum of an infinite geometric sequence is 5.6 with the common ratio equal to 0.20. The
sum of the first four terms of the geometric sequence is closest to:
A 5.0 B 5.2 C 5.4 D 5.6 E 5.8

5 multiple choice
8C
The sum of the first 10 terms of a geometric sequence is 400. The next term in the sequence
is 3 times the previous term. The first term in the sequence is:
17 400 100 200 10
A --------- B ------------ C ------------ D --------- E ---------
731 1473 7381 781 387

6 The infinite sum of a geometric sequence is 99 and the first term is 10. Find the common
8B ratio for the sequence.

7 Find the sum of the following expressions:


8B a 1+ 1
--- + 1
------ + 1
------ ...
4 16 64
2 4 8
b 1− --- + --- − ------ ...
3 9 27

8 multiple choice
8C
What type of growth or decay does the graph at right display?
A Linear growth B Linear decay
C Exponential growth D Exponential decay
E Steady/no change
Chapter 8 Applications of exponential and logarithmic functions in financial mathematics 421
9 multiple choice
8C
Which of the following graphs displays an exponential decay?
A B C

D E

10 If the average inflation rate during a 5-year period had been 3.5% p.a., what would be the
cost of a jar of peanut butter at the end of the period if it cost $2.60 at the start? 8C
The following information refers to questions 11 and 12. An investment of $4500 earns
compound interest at a rate of 6.4% p.a. and is made for 5 years.

11 multiple choice
8D
The balance in the account at the end of the investment period, if interest is compounded
quarterly, is:
A $6181.40 B $4871.71 C $6136.50 D $15 561.27 E $8592.20

12 multiple choice
8D
The principal plus interest accrued during the investment, if interest is credited weekly, is:
A $6136.50 B $15 561.27 C $7985.47 D $6195.86 E $4527.76

13 multiple choice
8D
After 4 1--- years $1200 has grown to $1750 in an account where interest is compounded
2
monthly. The annual interest rate is:
A 7.0% B 0.7% C 8.4% D 3.2% E 38%

14 multiple choice
8D
A sum of $850 is invested at 8% p.a. compound interest, credited fortnightly. For the
balance to grow to $1200 the investment should be left for a minimum of:
A 112 years B 113 years C 4 years 8 fortnights
D 4 years 9 fortnights E 5 years

15 If $5400 is to be invested for 5 years, which of the options below would be the most
productive to use? 8D
a 12% p.a. simple interest
b compound interest at 11.8% p.a., credited quarterly
c compound interest at 11.7% p.a., credited monthly.
422 M a t h s Q u e s t M a t h s B Ye a r 1 1 f o r Q u e e n s l a n d

16 What amount must be invested at 9.25% p.a., interest compounded 6-monthly, if it is to


8D grow to $5000 over 4 years?

17 How much interest would $950 earn if it was invested for 3 years at 12% p.a., interest
8D credited daily?

18 How long would it take for $2000 to amount to $3450 by earning interest at 6.8% p.a.,
8D compounded monthly?

19 multiple choice
8E
A loan of $100 000 is taken over 25 years with monthly repayments of $1150.00. Interest is
charged at 12% p.a. calculated monthly. The amount of the loan repaid after the first
payment is:
A $0 B $150.00 C $750.00 D $850.00 E $900.00

20 A home loan of $150 000 is negotiated with monthly repayments of $1600 and interest
8E calculated monthly at 12% p.a.
a What is the balance of the loan after:
i the 1st payment? ii the 2nd payment? iii the 3rd payment?
b What is the interest charged for the first 3 months of the loan?

21 multiple choice
8E,F
The interest charged on a reducing balance loan of $9500 is at the rate of 6.6% p.a. (debited
monthly). Monthly repayments of $291.60 are made. The amount still owing after 6 months,
to the nearest dollar, is:
A $8044 B $8104 C $8145 D $7750 E $7785

22 multiple choice
8F
A loan of $14 000 is taken out over 4 years at 9.75% p.a. (debited fortnightly) on the
outstanding balance. The fortnightly repayment needed to repay the loan in full, to the
nearest dollar, is:
A $135 B $145 C $163 D $170 E $319

23 multiple choice
8F
Rachel repaid a reducing balance loan of $22 000 in 5 years by quarterly repayments and
with interest charged quarterly at 8.2% p.a. on the outstanding balance. The total amount of
interest that she paid was closest to:
A $27 000 B $5000 C $5100 D $9000 E $10 000

Questions 24–25 refer to the following information. Helmut and Su-Li want to buy a boat, so
they borrow $70 000 at 7% p.a. (adjusted monthly) on the balance outstanding and agree to
repay the loan over 20 years with instalments of $542.71 per month.

24 Find the total amount of interest to be paid.


8F
8F 25 Determine the amount still owing after 10 years.
Chapter 8 Applications of exponential and logarithmic functions in financial mathematics 423
Modelling and problem solving
Questions 1 to 3 refer to the following information.
Rhiannon is investigating a loan of $12 000 to finance the purchase of a car. The finance market
is very competitive and Rhiannon chooses to investigate several options.
Option 1: A flat rate loan at 7.2% p.a. with 36 monthly repayments
Option 2: A personal loan over 3 years at 12.75% p.a. calculated fortnightly on reducing balance
Option 3: A personal loan over 3 years at 3.0% per quarter on reducing balance
Option 4: Loan consolidated with her current home loan at 7.92% p.a. over a 15-year term
calculated monthly with monthly repayments.
1 a ii Complete the interest rate per month for each of the options (to 1 decimal place).
Option 1 2 3 4
Interest rate per
month (%)
ii Calculate the effective interest rate for the option 1 flat rate loan.
b Calculate the interest charged for the option 1 flat rate loan.
c Calculate the monthly repayment for the flat rate loan (to the nearest dollar).
2 a Calculate the fortnightly repayments and total interest charged for the option 2 loan (to the
nearest dollar).
b Calculate the quarterly repayments and total interest charged for the option 3 loan.
3 a Calculate the increase in home loan monthly repayments if the $12 000 is consolidated
with option 4.
b Calculate the total interest charged using option 4 over the 15 years.
4 On an island in the Pacific Ocean the population
of a species of insect (species A) is increasing
geometrically with a population of 10 000 in
1990 and an annual growth rate of 12.0%.
Another species of insect (species B) is also
increasing its population, but arithmetically with
numbers 15 000 in 1990 and an annual
increment of 1000 per annum.
a Using a spreadsheet or other method,
determine the difference in the number of the
two species during the last decade of the
twentieth century (that is, up to 1999).
b In what year will the first species be greater
in number than the second species assuming
the growth rates remain fixed?
A scientist has a mathematical model
where the species can cohabit provided that
they have equal numbers in the year 2000.
c If the growth rate in species A is to remain
unchanged, what would the annual increment
in species B need to be to achieve this?
d If the annual increment in species B is to
remain unchanged, what would the growth
rate in species A need to be to achieve this?
424 M a t h s Q u e s t M a t h s B Ye a r 1 1 f o r Q u e e n s l a n d

5 A small number of rabbits have migrated to Mr Smith’s farm. Mr Smith has brought in
experts to make some investigations.
a The small number of rabbits is estimated to be 80. From experience, the experts know that
the rabbit population increases by 20% on each previous month’s population.
ii Copy and complete the table of the expected population of the rabbits for the first three
months.

Month 0 1 2 3

Rabbit population 80

ii A population of 1000 or more is considered to be harmful to the ecosystem. How soon


(in months) will this occur on Mr Smith’s farm?
b The area of land used (and devastated) by a rabbit population is given in the table below.

Number of rabbits 100 200 300 400 500 600

Area of land used (hectares) 1.25 2.5 3.75 5.0 6.25 7.5

ii State whether it is an exponential or straight line growth.


ii Give evidence to justify your response in part i.
c The cost (C in dollars) to the farmer for every hectare of land (H) lost is given by the
equation C = 1000 × 1.4H.
ii What is the cost to the farmer if there are 400 rabbits on the farm?
ii Mr Smith can only sustain about $2000 loss. How much land is this equivalent to (to
the nearest hectare)?
d At the completion of the investigation, the experts suggest Mr Smith budget $12 400 for a
rabbit eradication program. Mr Smith needs to borrow this amount and he has 2 loan
options:
• 10% p.a. simple interest with full payment in 2 years
• 7% p.a. compound interest (adjusted monthly) with full payment in 3 years’ time.
iii Calculate the total payment to be made at the end of the term for each of the loan
options.
iii Which is the best option? Explain why.
Another option is to use the cash set aside for a tractor and get the tractor on hire-
purchase.
iii On hire-purchase he can get the $13 000 tractor with a deposit of $600 and monthly
instalments at 6% p.a. over 3 years. Find the monthly instalments.

6 A housing loan package offers 8.4% p.a. interest. Fees and charges associated with this product
of $795 are added at the end of each year, after the interest is calculated for that month. Using
this package, what monthly payment is needed if $310 000 is borrowed over 25 years?
eBook plus
7 A housing loan package offers 8.4% p.a. interest. Fees and charges associated with this product
Digital doc: of 0.25% of the outstanding amount are added at the end of each year, after the interest is
Test Yourself
Chapter 8 calculated for that month. Using this package, what monthly payment is needed if $310 000
is borrowed over 25 years?
Presentation
of data
9
syllabus reference
Topic:
• Applied statistical analysis

In this chapter
9A Types of variables and data
9B Collection of data
9C Bias
9D Stem plots
9E Frequency histograms and
bar charts
9F Describing the shape of
stem plots and histograms
9G Cumulative data
426 M a t h s Q u e s t M a t h s B Ye a r 1 1 f o r Q u e e n s l a n d

Introduction
Karen is a real estate agent. At the end of each year it is part of her job to perform a
statistical analysis of house prices in the local area. In her real estate agency, there are
currently 60 houses for sale. Karen has summarised their prices in the table below.

Price range Number of houses


$275 000–$300 000 1
$300 000–$325 000 5
$325 000–$350 000 7
$350 000–$375 000 6
$375 000–$400 000 11
$400 000–$425 000 14
$425 000–$450 000 9
$450 000–$475 000 4
$475 000–$500 000 0
$500 000–$525 000 1
$525 000–$550 000 0
$550 000–$575 000 0
$575 000–$600 000 2
Karen must make a presentation on her findings to the Real Estate Institute. What are
the different ways in which she can present this information?
In this chapter we will look at different types of data and consider statistics in terms
of their presentation.
Defining statistics
People have always been interested in the collection of information about themselves
and their environment. The collection of such information in a systematic fashion is
called statistics.

Types of variables and data


A statistical investigation usually involves looking at a characteristic of a population.
Because this characteristic varies for different members of the population, this charac-
teristic is called the variable.
Once a known piece of information is assigned to a variable, it becomes a piece of
data. For example, if we are studying the maximum daily temperature, we refer to the
temperature as the variable as it can be different every day. If we say that it was 25°C
on Thursday, a value has been assigned to the variable; this is now a piece of data. The
variable that is being studied in an investigation, or a piece of data, can be described as
either categorical or quantitative.
Chapter 9 Presentation of data 427
Categorical
Categorical data cannot be measured; they can only be put into categories.
An example of categorical data is makes of cars. The categories for the data would
be all possible makes of cars such as Ford, Holden, Toyota, Mazda etc. Other questions
that would lead to categorical data would be things such as:
• What is your hair colour?
• Who is your favourite musical performer?
• What method of transport do you use to get to school?

Quantitative
Quantitative data can be measured. They are data to which we can assign a numerical
value.
Data concerning quantitative variables are collected by measurement or by counting.
For example, the data collected by measuring the heights of students are quantitative
in nature. The data collected by counting the ages of students in years are also
quantitative.

WORKED Example 1
State whether the following variables are categorical or quantitative in nature.
a The value of sales recorded at each branch of a fast-food outlet
b The breeds of dog that appear at a dog show
THINK WRITE
a The value of sales at each branch can be a The value of sales is quantitative.
measured.
b The breeds of dog at a show cannot be b The breeds of dog is categorical.
measured.

There are two types of categorical data and two types of quantitative data.

Data

Categorical Quantitative
Data which are placed in categories; that Data which are in numerical form; such
is, non-numerical form, such as hair as height, number of children in the
colour, type of vehicle, and so on. family, and so on.

Nominal Ordinal Discrete Continuous


Need sub-groups to Need a ranking to Counted in exact Measured in a
complete the order the values, such as continuous decimal
description, such as description, such as goals scored in a scale, such as mass,
hair colour: blond, achievement levels: football match, temperature, length
brown and so on. very high, high, shoe size and so on. and so on.
satisfactory and Values are often,
so on. but not always,
whole numbers.
428 M a t h s Q u e s t M a t h s B Ye a r 1 1 f o r Q u e e n s l a n d

WORKED Example 2
Classify each of the following data using two selections from the following descriptive
words: categorical, quantitative, nominal, ordinal, discrete and continuous.
a the number of students absent from school
b the types of vehicle using a certain road
c the various pizza sizes available at a local takeaway
d the room temperature at various times during a particular day
THINK WRITE
a 1 Determine whether the data are a The data are quantitative as absences are
categorical or quantitative. represented by a number.
2 Determine whether the data are The data are discrete as the number of absences
discrete or continuous. can be counted and is an exact value.
b 1 Determine whether the data are b The data are categorical as the types of vehicle
categorical or quantitative. need to be placed in non-numerical categories.
2 Determine whether the data are The data are nominal as there is no ranking or
nominal or ordinal. order involved.
c 1 Determine whether the data are c The data are categorical as the pizza sizes need to
categorical or quantitative. be ranked in order ranging from small to family.
2 Determine whether the data are The data are ordinal as pizzas are ranked in order
nominal or ordinal. of size.
d 1 Determine whether the data are d The data are quantitative as room temperature is
categorical or quantitative. represented by a number.
2 Determine whether the data are The data are continuous as temperature can
discrete or continuous. assume any value and measurement is involved.

Types of data
Consider Karen’s summary of house prices.
1 Are the data that Karen has collected categorical or quantitative?
2 Are house prices an example of discrete or continuous data?

remember
1. Data can be classified as either:
(a) categorical — the data are in categories, or
(b) quantitative — the data can be either measured or counted.
2. Categorical data can be either:
(a) nominal — where the order of the categories is not important, or
(b) ordinal — the categories have a logical order.
3. Quantitative data can be either:
(a) discrete — the data can take only certain values, usually whole numbers, or
(b) continuous — the data can take any value depending on the degree of
accuracy.
Chapter 9 Presentation of data 429

9A Types of variables and data


WORKED 1 State whether the data collected in each of the following situations would be
Example
1
categorical or quantitative.
a The number of matches in each box is counted for a large sample of boxes.
b The sex of respondents to a questionnaire is recorded as either M or F.
c A fisheries inspector records the lengths of 40 cod.
d The occurrence of hot, warm, mild and cool weather for each day in January is
recorded.
e The actual temperature for each day in January is recorded.
f Cinema critics are asked to judge a film by awarding it a rating from one to five
stars.
2 State whether the categorical data formed by each of the following situations are
nominal or ordinal.
a On a school report students are ranked as Unsatisfactory, Satisfactory, Above
average or Outstanding.
b The day of the week that a business has the most customers is recorded.
c Visitors to a museum are recorded as being either male or female.
d The colour of each traffic light on a journey is recorded.
e The make of each television in an electronics store is recorded.
3 State whether the quantitative data formed by each of the following situations are
discrete or continuous.
a The heights of 60 tomato plants at a plant nursery
b The number of jelly beans in each of 50 packets
c The time taken for each student in a class of six-year-olds to tie their shoelaces
d The petrol consumption rate of a large sample of cars
e The IQ (intelligence quotient) of each student in a class
WORKED 4 Classify each of the following data using two words selected from the following
Example
2
descriptive words: categorical, quantitative, nominal, ordinal, discrete and continuous.
a The population of your town or city
b The types of motorbike in a parking lot
c The heights of people in an identification line-up
d The masses of babies in a group
e The languages spoken at home by students in your class
f The time spent watching TV
g The number of children in the families in your suburb
h The air pressure in your car’s tyres
i The number of puppies in a litter
j The types of radio program listened to
by teenagers
k The times for swimming 50 metres
l The quantity of fish caught in a net
m The number of CDs you own
n The types of shops in a shopping centre
o The football competition ladder at the
end of each round
430 M a t h s Q u e s t M a t h s B Ye a r 1 1 f o r Q u e e n s l a n d

p The lifetime of torch batteries


q The number of people attending a rock concert
r Exam grades
s The types of magazine sold at a newsagency
t Hotel accommodation rating
5 For each of the following, state if the data are categorical or quantitative. If
quantitative, state if the data are discrete or continuous.
a The number of students in each class at your school
b The teams people support at a football match
c The brands of peanut butter sold at a supermarket
d The heights of people in your class
e The interest rate charged by each bank
f A person’s pulse rate
6 An opinion poll was conducted. A thousand people were given the statement
‘Euthanasia should be legalised’. Each person was offered five responses: strongly
agree, agree, unsure, disagree and strongly disagree. Describe the data type in this
example.
7 A teacher marks her students’ work with a grade A, B, C, D, or E. Describe the data
type used.
8 A teacher marks his students’ work using a mark out of 100. Describe the data type
used.

9 multiple choice
The number of people who are using a particular bus service are counted over a two
week period. The data formed by this survey would be an example of:
A categorical and ordinal data B categorical and nominal data
C quantitative and discrete data D quantitative and continuous data
E insufficient information
10 The following graph shows the
number of days of each weather
type for the Gold Coast in January.
Number of days in January

14
12
10
8
6
4
2
0
ot m ild ol
H ar M Co
W
Weather 180
Height (cm)

Describe the data in this example. 160

140
11 The graph at right shows a girl’s height each year
for 10 years. 120

Describe the data in this example. 100


5 6 7 8 9 10 11 12 13 14 15
Age
Chapter 9 Presentation of data 431
Collection of data
A common method of collecting data is through a poll. A poll is the recording of
responses to a set of questions known as a questionnaire.

Gallup poll
The most famous poll is named after its founder, the American statistician, George
Gallup, who was born in 1901.
Find out about Gallup and his work and how Gallup polls are used today.

The first step in gathering the relevant data for a statistical investigation is to target
the population to be investigated. This means identifying the sections of the population
for whom the statistical investigation will have relevance.
For example, if investigating the medical needs of a community, we would not conduct
our survey at the local fitness club. For such a survey we would choose doctors and other
medical personnel, as well as a selection of patients who use the existing facilities.

Identifying the target population


For each of the following statistical investigations, identify the population that you
would target for a survey.
1 The school ‘End of Year’ Committee wants to find out the preferred venue,
band and meals for the Year 12 farewell.
2 The local council wants to know what sporting facilities are needed in the local area.
3 A newspaper wants a survey to predict the winner of a forthcoming election.
4 A group of people planning to build a preschool would like to know what
facilities attract people to a particular preschool.
5 A recording label wants to estimate the potential success of a ‘grunge’ band.

When starting an investigation, we must determine the quantity of data needed for
the database. Consider the case of a company calculating TV ratings. Does the com-
pany need to find out what every household is watching? Obviously they do not;
instead they ask a selection of homes to record their TV viewing.
Now consider the case of selecting a band to play at the Year 12 farewell. In this case
it is reasonable to ask every Year 12 student their opinion.
Data can be collected in one of two ways:
1. Census. In a census an entire population is counted. Australians complete ‘The
Census’ every five years. This is a survey conducted by the Bureau of Statistics of
every household in the nation. For the purposes of most statistical investigations, a
census is where everyone in the target population is surveyed, such as the Year 12
example above.
2. Sample. A sample is a more practical method for conducting most surveys. Only a
selection of the target population is surveyed with the results taken to be represen-
tative of the whole group. The TV ratings example is one where a sample is used.
432 M a t h s Q u e s t M a t h s B Ye a r 1 1 f o r Q u e e n s l a n d

WORKED Example 3
In each of the following, state if the information was obtained by census or sample.
a A school uses the roll to count the number of students absent each day.
b The television ratings, in which 2000 families complete a survey on what they watch
over a one week period.
c A light globe manufacturer tests every hundredth light globe off the production line.
d A teacher records the examination marks of her class.

THINK WRITE
a Every student is counted at roll call each a Census
morning.
b Not every family is asked to complete a b Sample
ratings survey.
c Not every light globe is tested. c Sample
d The marks of every student are recorded. d Census

Sampling methods
To ensure that the results of your sample are representative of the whole population, the
method of sampling is important. There are three main methods of choosing a sample:
random sample, stratified sample and systematic sample.
Method 1. Random sample
Number of
In a random sample, those to be surveyed are selected Year students
by chance. When a random sample is conducted, every
Year 8 180
person in the target population should have an equal
chance of being selected. For example, the names of the Year 9 190
people to complete your survey may be drawn from a Year 10 185
hat. If this method is used, you should get a good
mixture of people in your survey. Year 11 135
Suppose that we are going to survey students in a Year 12 110
school. We want a mixture of students and could choose
a fixed number of students from each year. Suppose we Total 800
decide to survey 60 students. We could select 12 from
each year, but if we did this the survey would not have the correct proportion of stu-
dents from each year. For example, 22.5% of the students at this school are in Year 8,
but only 20% of the survey participants are in Year 8.

Graphics Calculator tip! Choosing a


random sample
Suppose that we are to choose a random sample of 20 students from the population of
800. To choose a random sample each student would be allocated a number between
1 and 800 and the graphics calculator could then be used to make a random choice
using a random number function.
Chapter 9 Presentation of data 433
For the Casio fx-9860G AU
To choose a random sample using the Casio fx-9860G AU graphics calculator we need
to use the random number function.
We are going to write a small program that will generate a random integer between
0 and 800. To do this, we generate a random decimal number, multiply it by 800, add
1 to the result and then take the integer part.

1. From the MAIN MENU, select RUN-MAT. Press


OPTN and then F6 ( ) to see more options.
Press F4 (NUM) followed by F2 (INT) to
obtain the integer function.

2. Press ( and then enter 800 followed by × .

3. Press EXIT , then F3 (PROB) followed by


F4 (Ran#) to generate a random decimal
number. Press + and 1 , then close the set
of brackets and press EXE . A random integer
will then be generated. You will probably obtain a
different number to that seen in the display shown.

4. Press EXE again to generate a new random


integer. Repeat until enough random integers have
been selected.

For the TI-Nspire CAS


To choose a random sample using the TI-Nspire graphics calculator we need to use the
random integer function.
1. Open a new Calculator document (press /N
and then select 1: Add Calculator). Press k to
access the catalog and then press 1 to select the
function menu. Scroll down to randInt(. (The
required function can be found more quickly by
first pressing R.)

2. Press · to display the function on the calculator


screen. Enter the lower limit of 1 and the upper
limit of 800, separated by a comma. Press ) to
close the set of brackets and then press · to
obtain a random number. You will probably obtain
a different number to that seen in the display
shown.
434 M a t h s Q u e s t M a t h s B Ye a r 1 1 f o r Q u e e n s l a n d

3. Press · again to generate a new random integer.


Repeat until enough random integers have been
selected.
<TI04.01>

A scientific calculator will generate random numbers. Your calculator may generate a
random integer as does the graphics calculator, or may generate a random decimal
between 0 and 1. To generate random integers from this decimal we multiply the
decimal by the number the sample is being chosen from (in the example above, 800)
and round the result up to the next whole number (rather than to the nearest whole
number).

WORKED Example 4
Three students from a school are to be selected to participate in a statewide survey of
school students. There are 750 students at the school. To choose the participants, a random
decimal generator is used with the results 0.983, 0.911, and 0.421. What are the roll
numbers of the students who should be selected?

THINK WRITE

1 Multiply the results of the random 0.983 × 750 = 737.25


number generator by the size of the 0.911 × 750 = 683.25
population. 0.421 × 750 = 315.75

2 Round up to whole numbers. The 738th, 684th and the 316th people on the
roll would be surveyed.

Any other method may not give a truly representative sample. For example, if you
survey people in the playground you may:
• have a tendency to ask people you know
• choose an area where a lot of students from a particular year tend to sit
• choose more of one gender than another.

Method 2. Stratified sample


In this type of sample you deliberately choose people to complete your survey who are
representative of the whole population. In the school survey you would need to select
five strata that had the correct proportion of students from each year. For example, if
20% of the school population are in Year 8 then 20% of your sample should be from
Year 8.
Chapter 9 Presentation of data 435
WORKED Example 5 Number of
Adrian is conducting a survey of school students. The Year students
table at right shows the number of students in each year Year 8 150
of school. Year 9 180
If Adrian decides to survey 50 students, how many should Year 10 140
be chosen from each year? Year 11 90
Year 12 80
Total 640
THINK WRITE
150 180
1 Calculate the percentage of Year 8 = --------- × 100% Year 9 = --------- × 100%
640 640
the school population in each = 23.4% = 28.1%
year. 140 90
Year 10 = --------- × 100% Year 11 = --------- × 100%
640 640

= 21.8% = 14.1%
80
Year 12 = --------- × 100%
640
= 12.5%
2 Calculate the number of Year 8 = 23.4% of 50 Year 9 = 28.1% of 50
students that should be chosen = 11.7 = 14.05
from each year to do the = 12 = 14
survey, rounding to the
nearest whole number. Year 10 = 21.8% of 50 Year 11 = 14.1% of 50
= 10.9 = 7.05
= 11 =7
Year 12 = 12.5% of 50
= 6.25
=6
3 Give a written answer. Adrian should choose twelve Year 8 students, fourteen
Year 9 students, eleven Year 10 students, seven Year 11
students and six Year 12 students.

Method 3. Systematic sample


Systematic sampling is where
those chosen for the sample are
chosen in a systematic or organ-
ised way. This method is mostly
used in quality control situations.
For example, suppose that the
quality and strength of sports
shoes is being tested. The quality
control department may test every
20th pair of shoes that comes off
the production line. In doing a
survey, every 20th person on the
school roll may be surveyed.
436 M a t h s Q u e s t M a t h s B Ye a r 1 1 f o r Q u e e n s l a n d

remember
1. Before beginning a statistical investigation it is important to identify the target
population.
2. The survey can be conducted either by:
(a) Census — the entire target population is surveyed, or
(b) Sample — a selection is surveyed such that those selected are
representative of the entire target population.
3. There are three methods for selecting a sample.
Method 1. Random sample — chance is the only factor in deciding who is
surveyed. This is best done using a random number generator.
Method 2. Stratified sample — those sampled are chosen in proportion to the
entire population.
Method 3. Systematic sample — a system is used to choose those who are to
be in the sample.

9B Collection of data

WORKED 1 A school conducts an election for a new school captain. Every teacher and student in
Example
3
the school votes. Is this an example of a census or a sample? Explain your answer.
2 A survey is conducted by a council to see what sporting facilities the community
needs. If 500 people who live in the community are surveyed, is this an example of a
census or a sample?
3 For each of the following surveys, state whether a census or a sample has been used.
a Two hundred people in a shopping centre are asked to nominate the supermarket
where they do most of their grocery shopping.
b To find the most popular new car on the road, 500 new car buyers are asked what
make and model car they purchased.
c To find the most popular new car on the road, the make and model of every new
car registered are recorded.
d To find the average mark in the mathematics half-yearly exam, every student’s
mark is recorded.
e To test the quality of tyres on a production line, every 100th tyre is road tested.
4 For each of the following, recommend whether you would use a census or a sample to
obtain the results.
a To find the most watched television program on Monday night at 7:30 pm
b To find the number of cars sold during a period of one year
c To find the number of cars that pass through the tollgates on the Gateway Bridge
each day
d To find the percentage of computers produced by a company that are defective
5 An opinion poll is conducted to try to predict the outcome of an election. Two
thousand people are telephoned and asked about their voting intention. Is this an
example of a census or a sample?
Chapter 9 Presentation of data 437
WORKED 6 A factory has 500 employees. Each employee has an employee number between 1 and
Example
500. Five employees are selected to participate in an Occupational Health and Safety
4
survey. To choose the participants, a random number generator is used. The results are
0.326, 0.352, 0.762, 0.989 and 0.018. What are the employee numbers of those to
participate in the survey?
7 A school has 837 students. A survey of 10 students in the school is to be conducted. A
random number generator is used to select the participants. If the random numbers
chosen are:
0.988 0.251 0.498 0.661 0.247 0.031 0.967 0.932 0.229 0.443
what are the roll numbers of the students who should be selected?
8 A survey is to be conducted of 20 out of 50 000 people in a country town. Those
selected are to be chosen using a random number generator.
a Use your calculator to generate 20 random numbers.
b Calculate the electoral roll numbers of the people who should be chosen for the
survey.
9 For each of the following, state whether the sample used is an example of random,
stratified or systematic sampling.
a Every 10th tyre coming off a production line is tested for quality.
b A company employs 300 men and 450 women. The sample of employees chosen
for a survey contains 20 men and 30 women.
c The police breathalyse the driver of every red car.
d The names of the participants in a survey are drawn from a hat.
e Fans at a football match fill
in a questionnaire. The
ground contains 8000
grandstand seats and
20 000 general admission
seats. The questionnaire
is then given to 40 people
in the grandstand and
100 people who paid for
a general admission seat.

10 multiple choice
Which of the following is an
example of a systematic sample?
A The first 20 students who arrive at school each day participate in the survey.
B Twenty students to participate in the survey are chosen by a random number
generator.
C Twenty students to participate in the survey are selected in proportion to the
number of students in each school year.
D Twenty students to participate in the survey are selected in proportion to the
number of boys in each school year.
E Ten boys and 10 girls are chosen to participate in the survey.
438 M a t h s Q u e s t M a t h s B Ye a r 1 1 f o r Q u e e n s l a n d

11 multiple choice
Which of the following statistical investigations would be practical to complete by
census?
A A newspaper wants to know public opinion on a political issue.
B A local council wants to know if a skateboard ramp would be popular with young
people in the area.
C An author wants a cricket player’s statistics for a book being written.
D An advertising agency wants to know the most watched program on television.
E A pollster wants to know how many Australians believe the Prime Minister is
doing a good job.
WORKED12 The table at right shows the number of students
Example Number of
5
in each year at a school.
Year students
If a survey is to be given to 40 students at the
school, how many from each Year should be 8 110
eBook plus chosen if a stratified sample is used? 9 90
Digital doc: 10 80
13 A company employs 300 men and 200 women.
SkillSHEET 9.1
If a survey of 60 employees using a stratified 11 70
Percentage skills
sample is completed, how many people of each 12 50
gender participated? Total 400
14 The table below shows the age and
sex of the staff of a corporation.
Age Male Female
20–29 61 44
30–39 40 50
40–49 74 16
50–59 5 10
A survey of 50 employees is to
be done. Using a stratified survey,
suggest the breakdown of people to
participate in terms of age and
gender.

Census or sample?
For each of the following statistical investigations, state whether you would gather
data using a census or sample. For those for which you would use a sample, state
the best method for selecting the sample.
1 A company wants to test the life of its batteries.
2 A sporting club wants to elect a new club president.
3 A market research company wants to determine the most popular brand of
toothpaste.
4 A theme park wants to know from which state and suburb its visitors come.
5 A Gallup poll is conducted to determine the preferred prime minister.
Chapter 9 Presentation of data 439
Bias
No doubt you have heard the comment, ‘There are lies, damned lies and statistics’. This
implies that we should be wary of statistical figures quoted. Indeed, we should always
make informed decisions of our own and not simply accept the mass of statistics that
bombards us through the media.
Bias can be introduced into statistics by:
1. questionnaire design
2. sample selection
3. the interpretation of results.

Bias in questionnaire design


Consider a survey designed to collect data on
opinions relating to culling kangaroo numbers in
Australia.
The questions may be designed to be emotive in
nature. Respondents in these situations feel obliged
to show compassion. Posing a question in the form,
‘The kangaroo is identified as a native Australian
animal, not found anywhere else in the world.
Would you be in favour of culling kangaroos in Aus-
tralia?’, would almost certainly encourage a nega-
tive response.
Using a leading question (one which leads the
respondent to answer in a particular way) can cause
bias to creep into responses. Rephrasing the ques-
tion in the form, ‘As you know, kangaroos cause
massive damage on many farming properties. You’d agree that their numbers need
culling, wouldn’t you?’, would encourage a positive response.
Using terminology that is unfamiliar to a large proportion of those being surveyed
would certainly produce unreliable responses. ‘Do you think we need to cull herbiv-
orous marsupial mammals in Australia?’, would cause most respondents to answer
according to their understanding of the terms used. If the survey was conducted by an
interviewer, the term could be explained. In the case of a self-administered survey,
there would be no indication of whether the question was understood or not.

Sampling bias
As discussed previously, an ideal sample should reflect the characteristics of the popu-
lation. Statistical calculations performed on the sample would then be a reliable indi-
cation of the population’s features.
Selecting a sample using a non-random method, as discussed earlier, generally tends
to introduce an element of bias.
Particular responses can be selected from all those received. In collecting infor-
mation on a local issue, an interviewer on a street corner may record responses from
many passers-by. From all the data collected, a sample could be chosen to support the
issue, or alternatively another sample could be chosen to refute the same issue.
A sample may be selected under abnormal conditions. Consider a survey to deter-
mine which lemonade was more popular — Kirks or Schweppes. Collecting data one
week when one of the brands was on special at half price would certainly produce
misleading results.
440 M a t h s Q u e s t M a t h s B Ye a r 1 1 f o r Q u e e n s l a n d

Data are often collected by radio and television stations via telephone polls. A ‘Yes’
response is recorded on a given phone-in number, while the ‘No’ respondents are asked to
ring a different phone-in number. This type of sampling does not produce a representative
sample of the population. Only those who are highly motivated tend to ring and there is
no monitoring of the number of times a person might call, recording multiple votes.
When data are collected from mailing surveys, bias results if the non-response rate is
high (even if the selected sample was a random one). The responses received often rep-
resent only those with strong views on the subject, while those with more moderate
views tend to lack representation in their correct proportion.
Statistical interpretation bias
Once the data have been collected, collated and subjected to statistical calculations,
bias may still occur in the interpretation of the results.
Misleading graphs can be drawn leading to a biased interpretation of the data.
Graphical representations of a set of data can give a visual impression of ‘little change’
or ‘major change’ depending on the scales used on the axes.
The use of terms such as ‘majority’, ‘almost all’ and ‘most’ are open to interpret-
ation. When we consider that 50.1% ‘for’ and 49.9% ‘against’ represents a ‘majority
for’ an issue, the true figures have been hidden behind words with very broad mean-
ings. Although we would probably not learn the real facts, we should be wary of
statistical issues quoted in such terms.

Bias in statistics
The aim of this investigation is to study statistical data that you suspect to be
biased.
Conduct a search of newspapers, magazines or any printed material to collect
instances of quoted statistics that you believe to be biased. There are occasions
when television advertisements quote statistical figures as a result of questionable
sampling techniques. For each example, discuss:
1 the purpose of the survey
2 how the data might have been collected
3 the question(s) that may have been asked (try to pose the question(s) in a
variety of ways to influence different outcomes)
4 ways in which bias might be introduced
5 variations in interpretation of the data.

Biased sampling
Discuss the problems that would be caused by each of the following biased
samples.
1 A survey is to be conducted to decide the most popular sport in a local
community. A sample of 100 people was questioned at a local football match.
2 A music store situated in a shopping centre wants to know the type of music that
it should stock. A sample of 100 people was surveyed. The sample was taken
from people who passed by the store between 10 and 11 am on a Tuesday.
3 A newspaper conducting a Gallup poll on an election took a sample of
1000 people from Brisbane.
Chapter 9 Presentation of data 441
Spreadsheets creating
misleading graphs
Let us practise producing misleading graphs. Consider the data in this table.

Year 1990 1995 2000 2005


Wages ($m) 6 9 13 20
% increase in wages 25 50 44 54
Profits ($m) 1 1.5 2.5 5
% increase in profits 20 50 66 100 Graph 2

We shall use a spreadsheet to produce misleading graphs based on these data.


Graph 1

Graph 3

1 Enter the data as indicated in the spreadsheet.


2 Graph the data using the Chart Wizard. You should obtain a graph similar to
Graph 1.
3 Copy and paste the graph twice within the spreadsheet.
4 Graph 2 gives the impression that the wages are a great deal higher than the
profits. This effect was obtained by reducing the horizontal axis. Experiment with
shortening the horizontal length and lengthening the vertical axis.
5 In Graph 3 we get the impression that the wages and profits are not very
different. This effect was obtained by lengthening the horizontal axis and
shortening the vertical axis. Experiment with various combinations.
6 Print out your three graphs and examine their differences.
Note that all three graphs have been drawn from the same data using valid scales.
A cursory glance leaves us with three different impressions. Clearly, it is important
to look carefully at the scales on the axes of graphs.
Another method which could be used to change the shape of a graph is to
change the scale of the axes.
442 M a t h s Q u e s t M a t h s B Ye a r 1 1 f o r Q u e e n s l a n d

7 Right click on the axis value, enter the Format axis option, click on the Scale
tab, then experiment with changing the scale values on both axes.
Techniques such as these are used to create different visual impressions of the
same data.
8 Use the data in the table to create a spreadsheet, then produce two graphs
depicting the percentage increase in both wages and profits over the years
giving the impression that:
a the profits of the company have not grown at the expense of wage increases
(the percentage increase in wages is similar to the percentage increase in profits)
b the company appears to be exploiting its employees (the percentage
increase in profits is greater than that for wages).

WORKED Example 6
Discuss why the following selected samples could provide bias in the statistics collected.
a In order to determine the extent of unemployment in a community, a committee phoned two
households (randomly selected) from each page of the local telephone book during the day.
b A newspaper ran a feature article on the use of animals to test cosmetics. A form
beneath the article invited responses to the article.
THINK WRITE
a 1 Consider phone book selection. a Phoning two randomly selected households
per page of the telephone directory is
possibly a representative sample.
2 Consider those with no phone However, those without a home phone and
contact. those with unlisted numbers could not form
part of the sample.
3 Consider the hours of contact. An unanswered call during the day would
not necessarily imply that the resident was at
work.
b 1 Consider the newspaper circulation. b Selecting a sample from a circulated
newspaper excludes those who do not have
access to the paper.
2 Consider the urge to respond. In emotive issues such as these, only those
with strong views will bother to respond, so
the sample will represent extreme points of
view.

Cost of a house
Remember Karen at the real estate agency? She collected information on the prices
of houses for sale through the real estate agency where she works.
1 Are the data collected an example of a census or a sample? If they are a sample,
describe the type of sample that has been taken.
2 Are Karen’s data subject to any bias? Explain.
Chapter 9 Presentation of data 443
remember
Bias can be introduced at each of the following stages:
1. questionnaire design
2. sample selection
3. interpretation of results.

9C Bias

1 Rewrite the following questions, removing any elements or words that might contribute
to bias in responses.
a The poor homeless people, through no fault of their own, experience great hardship
during the freezing winter months. Would you contribute to a fund to build a shelter
to house our homeless?
b Most people think that, since we’ve developed as a nation in our own right and
broken many ties with Great Britain, we should adopt our own national flag. You’d
agree with this, wouldn’t you?
c You’d know that our Australian 50 cent coin is in the shape of a dodecagon,
wouldn’t you?
d Many in the workforce toil long hours for low wages. By comparison, politicians
seem to get life pretty easy when you take into account that they only work for part
of the year and they receive all those perks and allowances. You’d agree, wouldn’t
you?
2 Rewrite parts a to d in question 1 so that the expected response is reversed.
WORKED 3 What forms of sampling bias can you identify in the following samples?
Example
6
a Choosing a sample from students on a bus travelling to a sporting venue to answer a
questionnaire regarding sporting facilities at their school
b Sampling using ‘phone-in’ responses to an issue viewed on a television program
c Promoting the results of a mail-response survey when fewer than half the selected
sample replied
d Comparing the popularity of particular chocolate brands when one brand has a ‘two
for the price of one’ special offer
e Choosing a Year 7 class and a Year 12
class to gather data relating to the use of
the athletics oval after school Value of A$ compared with US $1
91c
Australian currency

4 Why does this graph produce a biased


visual impression? 90c

89c

9 May 11 May 12 May


Date
5 Comment on the following statement:
‘University tests have demonstrated that Double-White toothpaste is consistently used
by the majority of teenagers and is more effective than most other toothpastes.’
444 M a t h s Q u e s t M a t h s B Ye a r 1 1 f o r Q u e e n s l a n d

6 Surveys are conducted on samples to determine the characteristics of the population.


Discuss whether the samples selected would provide a reliable indication of the popu-
lation’s characteristics.

Sample Population
a Year 11 students Student drivers
b Year 12 students Students with part-time jobs
c Residents attending a Residents of a suburb
neighbourhood watch meeting
d Students in the school choir Music students in the school
e Cars in a shopping centre car park Models of Holden cars on the road
f Males at a football match Popular TV programs
g Users of the local library Popular teenage magazines

Bias
It is important that a sample is chosen randomly to avoid bias.
Consider the following situation.
The government wants to improve sporting facilities in Brisbane. They decide to
survey 1000 people about what facilities they would like to see improved. To do
this, they choose the first 1000 people through the gate at a football match at the
Gabba.
In this situation it is likely that the results will be biased towards improving
facilities for football. It is also unlikely that the survey will be representative of
the whole population in terms of equality between men and women, age of the
participants and ethnic backgrounds.
Questions can also create bias. Consider asking the question, ‘Is football your
favourite sport?’ The question invites the response that football is the favourite
sport rather than allowing a free choice from a variety of sports by the
respondent.
Consider each of the following surveys and discuss:
a any advantages, disadvantages and possible causes of bias
b a way in which a truly representative sample could be obtained.
1 Surveying food product choices by interviewing customers of a large
supermarket chain as they emerge from the store between 9.00 am and 2.00 pm
on a Wednesday.
2 Researching the popularity of a government decision by stopping people at
random in a central city mall.
3 Using a telephone survey of 500 people
selected at random from the phone book
to find if all Australian States should
have Daylight Saving Time in summer.
4 A bookseller uses a public library
database to survey for the most popular
novels over the last three months.
5 An interview survey about violence in
sport taken at a rugby league football
venue as spectators leave.
Chapter 9 Presentation of data 445
Displaying data
Once a data set has been collected it can be displayed in tabular and graphical form, for
various purposes. The type of display chosen depends on the type of data that are being
represented.

Stem plots
A stem-and-leaf plot, or stem plot for short, is a way of displaying a set of data. It is
best suited to data which contain up to about 50 observations (or records).
The following stem plot shows the ages of people Stem Leaf
attending an advanced computer class. 1 6
The ages of the members of the class are 2 2 2 3
16, 22, 22, 23, 30, 32, 34, 36, 42, 43, 46, 47, 53, 57 and 61. 3 0 2 4 6
A stem plot is constructed by breaking the numerals of a 4 2 3 6 7
record into two parts — the stem, which in this case is the 5 3 7
first digit, and the leaf, which is always the last digit. 6 1

WORKED Example 7
The number of cars sold in a week at a large car
dealership over a 20-week period is given below.
16 12 8 7 26 32 15 51 29 45
19 11 6 15 32 18 43 31 23 23
Construct a stem plot to display the number of
cars sold in a week at the dealership.

THINK WRITE
1 In this example the observations are one- or two-
digit numbers and so the stems will be the digits
referring to the ‘tens’, and the leaf part will be the
digits referring to the units. Lowest number = 6
Work out the lowest and highest numbers in the Highest number = 51
data in order to determine what the stems will be. Use stems from 0–5.
2 Before we construct an ordered stem plot, Stem Leaf
construct an unordered stem plot by listing the leaf 0 8 7 6
digits in the order they appear in the data. 1 6 2 5 9 1 5 8
2 6 9 3 3
3 2 2 1
4 5 3
5 1
3 Now rearrange the leaf digits in numerical order to Stem Leaf
create an ordered stem plot. Include a key so that 0 6 7 8
the data can be understood by anyone viewing the 1 1 2 5 5 6 8 9
stem plot. 2 3 3 6 9
3 1 2 2
4 3 5
5 1
Key: 2|3 = 23 cars
446 M a t h s Q u e s t M a t h s B Ye a r 1 1 f o r Q u e e n s l a n d

WORKED Example 8
The masses (in kilograms) of the members of an Under-17 football squad are given below.
70.3 65.1 72.9 66.9 68.6 69.6 70.8
72.4 74.1 75.3 75.6 69.7 66.2 71.2
68.3 69.7 71.3 68.3 70.5 72.4 71.8
Display the data in a stem plot.
THINK WRITE
1 In this case the observations contain 3 digits. The
last digit always becomes the leaf and so in this case
the digit referring to the tenths becomes the leaf and
the two preceding digits become the stem. Lowest number = 65.1
Work out the lowest and highest numbers in the data Highest number = 75.6
in order to determine what the stems will be. Use stems from 65–75.

2 Construct an unordered stem plot. Note that the Stem Leaf


decimal points are omitted since we are aiming to 65 1
present a quick visual summary of data. 66 9 2
67
68 6 3 3
69 6 7 7
70 3 8 5
71 2 3 8
72 9 4 4
73
74 1
75 3 6

3 Construct an ordered stem plot. Provide a key. Stem Leaf


65 1
66 2 9
67
68 3 3 6
69 6 7 7
70 3 5 8
71 2 3 8
72 4 4 9
73
74 1
75 3 6
Key: 74|1 = 74.1 kg

Sometimes data which are very bunched make it difficult to get a clear idea about the
data variation. To overcome the problem, we can split the stems. Stems can be split into
halves or fifths.
Chapter 9 Presentation of data 447
WORKED Example 9
A set of golf scores for a group
of professional golfers trialling a
new 18-hole golf course is shown
on the following stem plot.
Stem Leaf
6 1 6 6 7 8 9 9 9
7 0 1 1 2 2 3 7

Key: 6 |1 = 61
Produce another stem plot
for these data by splitting
the stems into:
a halves b fifths.

THINK WRITE
a By splitting the stem 6 into halves, any leaf a Stem Leaf
digits in the range 0–4 appear next to the 6 1
first 6, and any leaf digits in the range 5–9 6 6 6 7 8 9 9 9
appear next to the second 6. Likewise for 7 0 1 1 2 2 3
the stem 7. 7 7
Key: 6 |1 = 61

b Alternatively, to split the stems into fifths, b Stem Leaf


each stem would appear 5 times. 6 1
Any 0s or 1s are recorded next to the first 6. 6
Any 2s or 3s are recorded next to the 6
second 6. Any 4s or 5s are recorded next to 6 6 6 7
the third 6. Any 6s or 7s are recorded next 6 8 9 9 9
to the fourth 6 and finally any 8s or 9s are 7 0 1 1
recorded next to the fifth 6. 7 2 2 3
This process would be repeated for those 7
observations with a stem of 7. 7 7
7
Key: 6 |1 = 61

Two sets of data can be displayed on the same stem-and-leaf plot. This is done by
having the stem in the centre of the plot, with both sets of data back to back.
448 M a t h s Q u e s t M a t h s B Ye a r 1 1 f o r Q u e e n s l a n d

WORKED Example 10
The data shown below display the marks of 15 students in both English and Maths.
English: 45 67 81 59 66 61 78 71 74 91 60 49 58 62 70
Maths: 85 71 49 66 64 68 75 71 69 60 63 80 87 54 59
Display the data in a back-to-back stem-and-leaf plot.

THINK WRITE
1 Write a key at the top of the Key: 4 | 5 = 45
stem-and-leaf plot. English Maths
2 Draw the stem showing categories of 95 4 9
10 in the centre of the page. 98 5 49
76210 6 034689
3 Display the information for English on
8410 7 115
the left of the stem.
1 8 057
4 Display the information for Maths on 1 9
the right of the stem.

This stem-and-leaf plot allows for both distributions to be easily seen, and for a
judgement on the skewness of the distribution to be made. (Skewness of a distribution
will be covered later in the chapter.)

remember
1. A stem-and-leaf plot is a useful way of displaying data containing up to about
50 observations (or records).
2. A stem plot is constructed by breaking the numerals of a record into two parts,
a ‘stem’ and a ‘leaf’. The last digit is always the leaf and any preceding digits
form the stem.
3. When asked to represent data using a stem-and-leaf plot, you should always
assume that the plot will be drawn with the data ordered.
4. If data are bunched then it may be useful to break the stems into halves or even
fifths.

9D Stem plots

1 In each of the following, write down all the pieces of data shown on the stem plot.
The key used for each stem plot is 3 | 2 = 32.
a Stem Leaf b Stem Leaf c Stem Leaf
eBook plus
0 1 2 1 0 1 10 1 2
Digital doc:
0 5 8 2 3 3 11 5 8
SkillSHEET 9.2
1 2 3 3 3 0 5 9 12 2 3 3
Presenting data 1 6 6 7 4 1 2 7 13 6 6 7
as a stem plot 2 1 3 4 5 5 14 1 3 4
2 5 5 6 7 6 2 15 5 5 6 7
3 0 2
Chapter 9 Presentation of data 449
d Stem Leaf e Stem Leaf
5 0 1 0 1 4
5 3 3 0 5 8
5 4 5 5 1 0 2
5 6 6 7 1 6 9 9
5 9 2 1 1
2 5 9
WORKED 2 The money (to the nearest dollar) earned each week by a busker over an 18-week
Example
7
period is shown below. Construct a stem plot for the busker’s weekly earnings.
5 19 11 27 23 35 18 42 29
31 52 43 37 41 39 45 32 36
3 The ages of those attending an embroidery class are given below. Construct a stem
plot for these data.
39 68 51 57 63 51 37 42
63 49 52 61 58 59 49 53
4 The number of dogs brought into a dog refuge each week over a 20-week period is
given below. Construct a stem plot for these data.
28 18 9 16 8 30 26 41 43 54
32 26 29 30 21 35 45 23 19 27
5 multiple choice
The observations shown on the stem plot at right are: Stem Leaf
A 4 10 27 28 29 31 34 36 41 0 4
B 14 10 27 28 29 29 31 34 36 41 41 1
C 4 22 27 28 29 29 30 31 34 36 41 41 2 2 7 8 9 9
D 14 22 27 28 29 30 30 31 34 36 41 41 3 0 1 4 6
E 4 2 27 28 29 29 30 31 34 36 41 4 1 1
Key: 2|5 = 25
6 The ages of the mothers of a class of children attending an inner city kindergarten are
given below. Construct a stem plot for these data.
32 37 29 23 29 32 25 38 39 32
28 30 33 34 28 35 35 29 33 30
7 The number of people attending a Neighbourhood Watch committee meeting each
fortnight for a year is given below. Construct a stem plot to display these data.
14 17 19 21 18 23 16 11 25 23 20
13 15 19 21 23 29 22 18 21 19 18
19 22 20
8 The number of hit outs made by each of the principal ruckmen in each of the AFL
teams for Round 11 is recorded below. Construct a stem plot to display these data.
Number of Number of
Team hit outs Team hit outs
Collingwood 20 Adelaide 32
Bulldogs 34 St Kilda 34
Kangaroos 29 Essendon 31
Port Adelaide 24 Carlton 26
Geelong 21 West Coast 29
Sydney 31 Fremantle 22
Melbourne 29 Hawthorn 33
Brisbane 25 Richmond 28
450 M a t h s Q u e s t M a t h s B Ye a r 1 1 f o r Q u e e n s l a n d

WORKED 9 The heights of members of a squad of 1.96 1.85 2.03 2.21 2.17 1.89
Example basketballers are given at right in metres. 1.99 1.87 1.95 2.03 2.09
8
Construct a stem plot for these data. 2.05 2.01 1.96 1.97 1.91
10 The March 2008 median house price of a number of Brisbane suburbs is given below.
Construct a stem plot for these data.
Price Price
Suburb (× $1000) Suburb (× $1000)
Auchenflower 830 Indooroopilly 680
Bulimba 1150 Milton 850
Balmoral 800 Spring Hill 1060
Cannon Hill 520 Mt Gravatt 500
Carrara 500 Nudgee 420
Coorparoo 630 Paddington 690
Brisbane City 500 Sandgate 490
Fairfield 570 Sth Brisbane 830
Holland Park 490 Woolloongabba 580

WORKED 11 The data at right give the head 48 49 47 52 51 50 49 48


Example
circumference (to the nearest cm) 50 50 53 52 43 47 49 50
9
of 16 four-year-old girls.
Construct a stem plot for head circumference, using:
a the stems 4 and 5 b the stems 4 and 5 split into halves
c the stems 4 and 5 split into fifths.

12 A random sample of 20 screws is 23 15 18 17 17 19 22


taken and the length of each is recorded 19 20 16 20 21 19 23
to the nearest millimetre (at right). 17 19 21 23 20 21
Construct a stem plot for screw length using:
a the stems 1 and 2 b the stems 1 and 2 split into halves
c the stems 1 and 2 split into fifths.

13 The number of seconds for which 12 Grade 2 children can hold their breath under
water is given below.
8.2 9.2 8.1 8.5 9.3 8.9 8.9 9.5 8.9 9.0 9.1 9.7
Construct a stem plot for holding breath using:
a the stems 8 and 9 b the stems 8 and 9 split into halves
c the stems 8 and 9 split into fifths.

WORKED 14 In a class of 30 students, there are 15 boys and 15 girls. Their heights are measured in
Example
10
metres and listed below.
Boys: 1.65, 1.71, 1.59, 1.74, 1.66, 1.69, 1.72, 1.66, 1.65, 1.64, 1.68, 1.74,
1.57, 1.59, 1.60
Girls: 1.66, 1.69, 1.58, 1.55, 1.51, 1.56, 1.64, 1.69, 1.70, 1.57, 1.52, 1.58,
1.64, 1.68, 1.67
Display this information in a back-to-back stem-and-leaf plot.

15 The number of points scored in each match by two rugby union teams are shown below.
Team 1: 34, 32, 24, 25, 8, 18, 17, 23, 29, 40, 19, 42
Team 2: 23, 20, 35, 21, 46, 7, 9, 24, 27, 38, 41, 30
Display these data in a back-to-back stem-and-leaf plot.
Chapter 9 Presentation of data 451
Frequency histograms and bar charts
Frequency histograms and bar charts display data in graphical form.

Frequency histograms
A histogram is a useful way of displaying large data sets (say, over 50 observations).
The vertical axis on the histogram displays the frequency and the horizontal axis
displays class intervals of the variable (for example height, income etc.).
When data are given in raw form — that is, just as a list of figures in no particular
order — it is helpful to first construct a frequency table.

WORKED Example 11
The data below show the distribution of masses (in kilograms) of 60 students in Year 7 at
Northwood State High School. Construct a frequency histogram to display the data more
clearly.
45.7 34.2 56.3 38.7 52.4 45.7 48.2 52.1 58.7 62.3
45.8 52.4 60.2 48.5 54.3 39.8 36.2 54.3 39.7 46.3
45.9 52.3 44.2 49.6 48.6 42.5 47.2 51.3 43.1 52.4
48.2 51.8 53.8 56.9 53.7 42.9 46.7 51.9 56.2 61.2
48.3 45.7 43.5 43.8 58.7 59.2 58.7 54.6 43.0 48.2
48.4 56.8 57.2 58.3 57.6 53.2 53.1 58.7 56.3 58.3
THINK WRITE
1 First construct a frequency table.
The lowest data value is 34.2 and the highest is Class
62.3. Divide the data into class intervals. If we interval Frequency
started the first class interval at, say, 30 kg and 30–34.9 1
ended the last class interval at 65 kg, we would
35–39.9 4
have a range of 35. If each interval was 5 kg, we
would then have 7 intervals which is a 40–44.9 7
reasonable number of class intervals. 45–49.9 16
While there are no set rules about how many 50–54.9 15
intervals there should be, somewhere between 55–59.9 14
about 5 and 15 class intervals is usual. So, in this
60–64.9 3
example, we would have class intervals of
30–34.9 kg, 35–39.9 kg, 40–44.9 kg and so on. Total 60
Count how many observations fall into each of
the intervals and record these in a table.

16
2 Check that the frequency column totals 60. 14
The data are in a much clearer form now.
Frequency

12
10
8
6
4
3 A histogram can be constructed. 2
0
30 35 40 45 50 55 60 65
Mass (kg)
452 M a t h s Q u e s t M a t h s B Ye a r 1 1 f o r Q u e e n s l a n d

WORKED Example 12
The marks out of 20 received by 30 students for a book-review assignment are given in the
frequency table below.
Mark 12 13 14 15 16 17 18 19 20
Frequency 2 7 6 5 4 2 3 0 1
Display these data on a histogram.
THINK WRITE
In this case we are dealing with integer
7
values. Since the horizontal axis should show 6

Frequency
a class interval, we extend the base of each of 5
4
the columns on the histogram halfway below 3
each score and halfway above it. 2
1
0
12 13 14 15 16 17 18 19 20
Mark out of 20

Graphics Calculator tip! Constructing a histogram


To construct a histogram of the data shown in
Worked example 12, follow these steps.
For the Casio fx-9860G AU
1. From the MAIN MENU, select STAT. Enter the
data for Mark in List 1. You may name the list if
you wish.

2. Enter the data for frequency in List 2. You may


wish to name this list as well.

3. Press F1 (GRPH).

4. Press F6 (SET). Then, using the arrow keys,


scroll down to Graph Type and press F1 (Hist).
(You may first need to press F6 ( ) for more
options). Enter List 1 for XList and List 2 for
Frequency.
Chapter 9 Presentation of data 453
5. Press EXIT to return to the previous screen.
Press F1 (GPH1). Use the arrow keys to
highlight Width, enter a value of 1 and press
EXE to accept the new setting.

6. Press EXE to draw the graph. If necessary, you


can use the V-Window to adjust the x- and y-axes
or use the Trace function to display values.

For the TI-Nspire CAS


With Operating System 1.4, this calculator cannot
display data that is grouped in a frequency table as a
histogram. However, we can produce a graph which
displays the heights of each column. This provides a
visual representation of the data.
1. Open a new Lists & Spreadsheet document
(press /N and then select 3: Add Lists &
Spreadsheet). Enter the data for Mark in
column A (press · after each entry) and provide
a title for this column.

2. Enter the data for frequency in column B and


provide a title for this column.

3. Press /I and then select 5: Add Data &


Statistics to insert a new Data & Statistics
document. Use the NavPad to move to the
horizontal axis and then press the Click button
(x). There are two options shown for the
variable.

4. Use the NavPad to highlight mark and then press


·. Similarly, use the NavPad to move to the
vertical axis. Press x and then, with freq
highlighted, press ·. The dots shown indicate
the height of each column in the matching
histogram.
454 M a t h s Q u e s t M a t h s B Ye a r 1 1 f o r Q u e e n s l a n d

Bar charts
A bar chart is similar to a histogram. However, it consists of bars of equal width
separated by small, equal spaces and may be arranged either horizontally or vertically.

Student pet preferences


25

Number of families
Dog
Cat 20
Rabbit 15
Snake
Bird 10
Goldfish 5
2 4 6 8 10 12
Number of students 0 1 2 3 4 5
Number of children in family

In bar charts the frequency is graphed against a variable as shown in both figures above.
The variable may or may not be numerical. However, in this chapter we consider
only numerical variables. The numerical variable should take discrete values; that is, it
should take only certain values (such as whole hours or number of people) rather than
being continuous (such as the height of
people) which could take any value 20
19

Mark out of 20
within a range. This is because the scale 18
is broken by the gaps between the bars. 17
The numerical values are generally close 16
15
together and have little spread, like con- 14
secutive years. 13
12
The bar chart at right represents the
data presented in Worked example 12. 1 2 3 4 5 6 7
Frequency or number of students
Of course, it could have been drawn with
vertical bars (columns).

eBook plus
Segmented bar charts
A segmented (divided) bar chart is a single bar which is used to represent all the
Digital doc: data being studied. It is divided into segments, each segment representing a
EXCEL Spreadsheet particular group of the data. Generally, the information is presented as percentages
Segmented
bar charts
and so the total bar length represents 100% of the data.
Consider the following table, showing fatal road accidents in Australia.

ROAD TRAFFIC ACCIDENTS INVOLVING FATALITIES


Accidents involving fatalities
Year NSW Vic. Qld SA WA Tas. NT ACT Aust.
2001 486 404 296 137 151 52 43 15 1584
2002 501 361 283 138 159 34 40 8 1524
2003 483 294 284 136 155 39 44 10 1445
2004 458 312 289 128 162 52 34 9 1444
2005 459 314 296 127 151 49 51 25 1472
2006 453 309 314 104 183 42 39 12 1456
Chapter 9 Presentation of data 455
ROAD TRAFFIC ACCIDENTS INVOLVING FATALITIES
Persons killed
Year NSW Vic. Qld SA WA Tas. NT ACT Aust.
2001 524 444 324 153 165 61 50 16 1737
2002 561 397 322 154 179 37 55 10 1715
2003 539 330 310 157 180 41 53 11 1621
2004 510 343 311 139 178 58 35 9 1583
2005 508 346 330 148 163 51 55 26 1627
2006 500 337 336 117 202 54 42 13 1601
Source: ABS Yearbook, 2006, 2007, 2008.

It is appropriate to represent the number of accidents involving fatalities in all states


and territories during 2001 as a segmented bar chart.
Firstly, using the data on page 454, we convert each state’s proportion of accidents
out of the total to a percentage.

State Number of accidents Percentage


NSW 486 486 ÷ 1584 × 100% = 30.7%
Vic. 404 404 ÷ 1584 × 100% = 25.5%
Qld 296 296 ÷ 1584 × 100% = 18.7%
SA 137 137 ÷ 1584 × 100% = 8.6%
WA 151 151 ÷ 1584 × 100% = 9.5%
Tas. 52 52 ÷ 1584 × 100% = 3.3%
NT 43 43 ÷ 1584 × 100% = 2.7%
ACT 15 15 ÷ 1584 × 100% = 1.0%
The segmented bar chart is drawn to scale. An appropriate scale would be
constructed by drawing the total bar 10 cm long, so that 1 mm represents 1%. That is,
NSW’s accidents would be represented by a segment of 30.7 mm, Victoria’s by a
segment of 25.5 mm and so on.
Each segment is then labelled directly, or a key may be used.

NSW 30.7% WA 9.5% Qld 18.7% NT 2.7%


Vic. 25.5% Tas. 3.3% SA 8.6% ACT 1.0%

remember
1. On a frequency histogram the vertical axis displays the frequency and the
horizontal axis displays the class intervals.
2. Data given in raw form should be summarised first in a frequency table.
3. A bar chart consists of bars of equal width separated by small, equal spaces.
The numerical variable should take discrete values only.
4. A segmented bar chart is a single bar, drawn to scale, which is used to
represent all the data being studied. Each segment represents a particular group
of the data.
456 M a t h s Q u e s t M a t h s B Ye a r 1 1 f o r Q u e e n s l a n d

Frequency histograms and


9E bar charts
WORKED 1 Construct a frequency table for each of the following sets of data.
Example
a 3 4 4 5 5 6 7 7 7 8 8 9 9 10 10 12 eBook plus
11
b 4.3 4.5 4.7 4.9 5.1 5.3 5.5 5.6 5.2 3.6 2.5 4.3 2.5 3.7 4.5 6.3 1.3
Digital docs:
c 11 13 15 15 16 18 20 21 22 21 18 19 20 16 18 20 16 10 23 SkillSHEET 9.3
24 25 27 28 30 35 28 27 26 29 30 31 24 28 29 20 30 32 33 Presenting data in a
frequency table
29 30 31 33 34
EXCEL Spreadsheet
d 0.4 0.5 0.7 0.8 0.8 0.9 1.0 1.1 1.2 1.0 1.3 0.4 0.3 0.9 0.6 Frequency
histograms
WORKED 2 Using the frequency tables from question 1, construct a histogram for
Example
11, 12
each set of data.
3 Using a graphics calculator, construct a histogram for each of the sets of data given in
question 1. Compare this histogram with the one drawn for question 2.
4 Using the frequency table from question 1a, construct a bar chart for the data.
5 The data at right represent the number 2 5 2 0 8 7 8 5 1 0
of hours each week that 40 teenagers 2 1 8 0 4 2 2 9 8 5
spent on household chores. Represent 7 5 4 2 1 2 9 8 1 2
these data by a bar chart. 8 5 8 10 0 3 4 5 2 8
6 A class of 30 students sat for a Mathematics test. Their results out of 100 are shown
below.
68 72 58 45 69 92 38 51 70 65 69 73 52 76 48
69 73 41 42 73 80 50 60 49 65 94 88 85 53 60
a Use these results to copy and complete the frequency table below.

Score Class centre Tally Frequency


30–39
40–49
50–59
60–69
70–79
80–89
90–99

b Construct a histogram to display these data.


7 A farmer measures the heights of his tomato plants. The results, in metres, are shown
below.
0.93 1.21 2.03 1.40 1.17 1.53 1.82 1.77 1.65 0.63 1.24 1.99
0.80 2.14 1.53 2.07 1.96 1.05 0.94 1.23 1.72 1.34 0.75 1.17
1.50 1.41 1.74 1.86 1.55 1.42 1.52 1.39 1.76 1.67 1.28 1.43 2.13
Chapter 9 Presentation of data 457
a Use the class groupings 0.6–<0.8, 0.8–<1.0, 1.0–<1.2, . . . etc. to complete a fre-
quency distribution table for these data.
b Construct a histogram to display these data.
8 The following data give the times (in
seconds) taken for athletes to complete
a 100 m sprint.
12.2 12.0 11.9 12.0 12.6 11.7
11.4 11.0 10.9 11.7 11.2 11.8
12.2 12.0 12.7 12.9 11.3 11.2
12.8 12.4 11.7 10.8 13.3 11.7
11.6 11.7 12.2 12.7 13.0 12.2
a Construct a frequency distribution
table for the data. Use a class size
of 0.5 seconds.
b Construct a histogram to display
these data.
9 Using the data provided on pages 454–5 for Road traffic accidents involving fatalities,
represent the number of people killed in all states and territories in Australia during
2006 as a segmented bar chart.

Segmented bar chart


Compare the proportion of fatal accidents in the states and territories during the
period 2001 to 2006 by drawing a segmented bar chart similar to that on page 455
for each of the 5 years since 2001.
1 What conclusions can be made from your charts?
2 Would you say that this presentation is misleading in any way?
3 Give reasons for your answers.
458 M a t h s Q u e s t M a t h s B Ye a r 1 1 f o r Q u e e n s l a n d

Looking at cost
Let’s return to house prices. Look at the table of data that Karen has collected.

Price range Number of houses

$275 000–$300 000 1

$300 000–$325 000 5

$325 000–$350 000 7

$350 000–$375 000 6

$375 000–$420 000 11

$400 000–$425 000 14

$425 000–$450 000 9

$450 000–$475 000 4

$475 000–$500 000 0

$500 000–$525 000 1

$525 000–$550 000 0

$550 000–$575 000 0

$575 000–$600 000 2

Present these data in an appropriate type of graph.

Using a database
If you have access to a computer database, collect the following information on
each property displayed in the real estate agents window in your local area.
Category: __________________________ (e.g. house, unit, vacant land, business)
Area: ____________________________
Number of bedrooms: _______________
Number of bathrooms: ______________
Special features: ___________________
Price: ____________________________
Enter your data into your database, and experiment to determine the different ways
that you can sort, select and display the data.
Chapter 9 Presentation of data 459
Describing the shape of stem plots and
histograms
Symmetric distributions
The data shown in the histogram at right can be described

Frequency
as symmetric.
There is a single peak and the data trail off on both
sides of this peak in roughly the same fashion.

Similarly in the stem plot at right, the distribution of the Stem Leaf
data could be described as symmetric. 0 7
The single peak for these data occur at the stem 3. On 1 2 3
either side of the peak, the number of observations reduces in 2 2 4 5 7 9
approximately matching fashion. 3 0 2 3 6 8 8
4 4 7 8 9 9
5 2 7 8
6 1 3

Skewed distributions
Each of the histograms below show examples of skewed distributions.
The figure below left shows data which are negatively skewed. The data in this case
peak to the right and trail off to the left.
The figure below right shows positively skewed data. The data in this case peak to the
left and trail off to the right.

Negatively skewed distribution Positively skewed distribution

Outliers
When one observation lies well away from other observations in a set, we call it an
outlier. Sometimes an outlier occurs because data have been incorrectly obtained or
misread. For example, below we see a histogram showing the weights of a group of
5-year-old boys.
25
Frequency

20
15
10
5
0
16 17 18 19 20 21 22 23 24 25 26 27 28 29 30 31 32 33
Weight (kg)
The outlier, 33, may have occurred because a weight was incorrectly recorded as 33
rather than 23, or perhaps there was a boy in this group who, for some medical reason,
weighed a lot more than his counterparts. When an outlier occurs, the reasons for its
existence should be checked.
460 M a t h s Q u e s t M a t h s B Ye a r 1 1 f o r Q u e e n s l a n d

WORKED Example 13
The ages of a group of
people who were taking
out their first home loan
are shown below.

Stem Leaf
1 9 9
2 1 2 4 6 7 8 8 9
3 0 1 1 2 3 4 7
4 1 3 5 6
5 2 3
6
7 7
Key: 1|9 = 19 years

Describe the shape of the


distribution of these data
and comment on the
existence of any outliers.

THINK WRITE
1 Check whether the distribution is The data are positively skewed.
symmetric or skewed. The peak of the
data occurs at the stem 2. The data trail
off as the stems increase in value. This
seems reasonable since most people
would take out a home loan early in life
to give themselves time to pay it off.
2 Check whether there is an outlier. The observation 77 is an outlier.
Note: It is unusual to have a 77-year-old person taking out a first home loan. Maybe this
observation was incorrectly recorded or maybe exceptional circumstances apply in this
case.

remember
1. When data are displayed in a histogram or a stem plot, we say that their
distribution is:
(a) symmetric if there is a single peak and the data trail off on either side of
this peak in roughly the same fashion
(b) negatively skewed if the data peak to the right and trail off to the left.
(c) positively skewed if the data peak to the left and trail off to the right.
2. An outlier is an observation that lies well away from the rest of the data.
Chapter 9 Presentation of data 461
Describing the shape of stem
9F plots and histograms
WORKED 1 For each of the following stem plots, describe the shape of the distribution of the data
Example
13 and comment on the existence of any outliers.

a Stem Leaf b Stem Leaf


0 1 3 1 3
1 2 4 7 2 6
2 3 4 4 7 8 3 3 8
3 2 5 7 9 9 9 9 4 2 6 8 8 9
4 1 3 6 7 5 4 7 7 7 8 9 9
5 0 4 6 0 2 2 4 5
6 4 7
7 1
Key: 1|2 = 12 Key: 2|6 = 2.6

c Stem Leaf d Stem Leaf


2 3 5 5 6 7 8 9 9 1
3 0 2 2 3 4 6 6 7 8 8 1 5
4 2 2 4 5 6 6 6 7 9 2 1 4
5 0 3 3 5 6 2 5 7 8 8 9
6 2 4 3 1 2 2 3 3 3 4 4
7 5 9 3 5 5 5 6
8 2 4 3 4
9 7 4
10
11
12
13 5
Key: 10|4 = 104 Key: 2|7 = 27

e Stem Leaf f Stem Leaf


3 1 60 2 5 8
3 61 1 3 3 6 7 8 9
3 62 0 1 2 4 6 7 8 8 9
3 63 2 2 4 5 7 8
3 8 9 64 3 6 7
4 0 0 1 1 1 65 4 5 8
4 2 3 3 3 3 3 66 3 5
4 4 5 5 5 67 4
4 6 7
4 8
Key: 4|3 = 0.43 Key: 62|3 = 623
462 M a t h s Q u e s t M a t h s B Ye a r 1 1 f o r Q u e e n s l a n d

2 For each of the following histograms, describe the shape of the distribution of the data
and comment on the existence of any outliers.
a b c

Frequency

Frequency
Frequency

d e f
Frequency

Frequency

Frequency
3 multiple choice
The distribution of the data shown in this stem plot Stem Leaf
could be described as: 0 1
0 2
A negatively skewed
0 4 4 5
B negatively skewed with one outlier 0 6 6 6 7
0 8 8 8 8 9 9
C positively skewed
1 0 0 0 1 1 1 1
D positively skewed with one outlier 1 2 2 2 3 3 3
E symmetric. 1 4 4 5 5
1 6 7 7
1 8 9

4 multiple choice
The distribution of the data shown in this histogram could be described as:
Frequency

A negatively skewed
B negatively skewed with one outlier
C positively skewed
D positively skewed with one outlier
E symmetric.

5 The average number of product enquiries per 8


day received by a group of small businesses who 7
Frequency

6
advertised in the Yellow Pages telephone direc- 5
tory is given at right. Describe the shape of the 4
3
distribution of these data and comment on the 2
1
existence of any outliers. 0
0 1 2 3 4 5 6 7 8 9 10 111213 1415
Number of enquiries
Chapter 9 Presentation of data 463
6 The number of nights per month spent interstate Stem Leaf
by a group of flight attendants is shown on the 0 0 0 1 1
stem plot at right. Describe the shape of the dis- 0 2 2 3 3 3 3 3 3 3 3
tribution of these data and explain what this tells 0 4 4 5 5 5 5 5
us about the number of nights per month spent
0 6 6 6 6 7
interstate by this group of flight attendants.
0 8 8 8 9
1 0 0 1
1 4 4
1 5 5
1 7
1
Key: 1|4 = 14 nights

7 The mass (to the nearest kilogram) of each dog at a dog Stem Leaf
obedience school is shown on the stem plot at right. 0 4
a Describe the shape of the distribution of these data 0 5 7 9
and comment on the existence of any outliers. 1 1 2 4 4
b What does this information tell us about this group of
1 5 6 6 7 8 9
dogs?
2 1 2 2 3
2 6 7
Key: 0|4 = 4 kg

8 The amount of pocket money (to the nearest 50 cents) received each week by students
in a Grade 6 class is illustrated in this histogram.
8
7
Frequency

6
5
4
3
2
1
0
2 2.5 3 3.5 4 4.5 5 5.5 6 6.5 7 7.5 8 8.5 9 9.51010.5
Pocket money ($)
a Describe the shape of the distribution of these data and comment on the existence of
any outliers.
b What conclusions can you reach about the amount of pocket money received
weekly by this group of students?

9 The number of hours of exercise completed each week Stem Leaf


by a group of employees at a company is shown on the 0 0 0 0 0 1 1
stem plot at right. 0 2 2 2 2 3 3 3
a Describe the shape of the distribution of these data 0 4 4 5
and comment on the existence of any outliers. 0 6 7
b What does this tell us about the number of hours of 0 8
exercise completed weekly by the employees in this 1
company? 1
eBook plus 1 4
1
Digital doc: 1 9
WorkSHEET 9.1
Key: 0|1 = 1 hour
464 M a t h s Q u e s t M a t h s B Ye a r 1 1 f o r Q u e e n s l a n d

Cumulative data
Cumulative frequency
It is often useful to consider the number of data points that are less than or equal to a
particular score. In such cases it is helpful to include a cumulative frequency column on
the frequency distribution table.
The cumulative frequency is the number of records equal to and less than a
particular score. The cumulative frequency of a particular score is obtained by
adding the frequency of that score to the sum of the frequencies of all preceding
scores.
In other words, if all the data were sorted in order of size, the cumulative frequency
would give a ‘running total’ of the number of observations up to each score. Consider
the following data that show the heights (in cm) of 40 girls who are competing in trials
to form a basketball squad.
181 191 185 174 192 186 188 182 179 172
186 188 193 198 182 175 176 188 180 191
184 186 183 180 179 175 180 188 190 193
172 179 188 183 186 184 182 193 194 181
A frequency distribution table including a cumulative column could be drawn as
follows:
Height Frequency Cumulative frequency
170–174 3 3
175–179 6 9
180–184 12 21
185–189 10 31
190–194 8 39
195–199 1 40

The cumulative frequency column in this case records the number of girls who had a
height as indicated by the particular group or those preceeding it. For example the
figure 31 in the cumulative frequency column can be interpreted as: ‘There were
31 girls who had a height of 189 cm or less’. The figure 31 in this example was found
by totalling 3 + 6 + 12 + 10. Note that the final number in the cumulative frequency
column should always equal the total number of scores.

Ogives
An ogive (also called a cumulative frequency polygon) is a line graph of the
cumulative frequency results.
An ogive is appropriate only for displaying grouped data. The graph is started on the
horizontal axis at a point corresponding to the lowest possible score in the smallest
group. In the case of the basketball squad data the graph will start at 170. The ogive is
then drawn by plotting the value of the cumulative frequency of each group against
each group end point. For the basketball squad data the points which form the rest of
the ogive will be (175, 3), (180, 9), (185, 21), (190, 31), (195, 39) and (200, 40).
The ‘S’ shape of this ogive is typical of most sets of data.
Chapter 9 Presentation of data 465
A percentage axis was added on the

Cumulative frequency (%)


40 Ogive of basketball 100%

Cumulative frequency
right-hand side of the ogive. A percentage 35 squad heights
axis can be added by ruling a vertical line 30
25
from the end point of the ogive to the hori- 20 50%
zontal axis. The end point is labelled 100% 15
and then the axis is scaled from 0–100% 10
5
appropriately. A percentage axis is not an
essential feature of an ogive but it will help 170 175 180 185 190 195 200
to answer a lot of questions like some of Height (cm)
the following: Figure A
1. How many girls had a height of less
than 182 cm? (Find 182 on the height axis, rule a vertical line to the ogive then hori-
zontally to the frequency axis. See line (a) in figure B.) Answer: About 15 girls
would have a height of less than 182 cm.
2. What percentage of girls had a height of less than 180 cm? (Find 180 on the height
axis then rule a vertical line to the ogive then horizontally to the percentage axis. See
line (b) in figure B.) Answer: About 22% of girls had a height of less than 180 cm.
3. What percentage of girls had a height

Cumulative frequency (%)


more than 180 cm? Answer: About 22% 40 Ogive of basketball 100%

Cumulative frequency
had a height of less than 180 cm so there 35 squad heights
30
must be 78% with a height of more than 25
180 cm. 20 (a) 50%
15
Note that when you are interpreting ogives, 10 (b)
less than (<) and less than or equal to (≤) 5
make no difference to the approach that we 170 175 180 185 190 195 200
take to solving a question or its answer. Height (cm)
Example 1 above would have had exactly Figure B
the same answer if the question had been:
‘How many girls had a height of less than or equal to 182 cm?’
Percentiles
A percentile is the score below which a particular percentage of
the distribution of data lies.
For example the 90th percentile is the score below which 90% of
the data lies. In the case of the basketball squad data the 90th per-
centile could be found by finding 90% on the percentage axis, going
horizontally to the ogive, then vertically down to the ‘height axis’.
The 90th percentile would be about 193 cm. This could be
interpreted as: ‘90% of the girls would have a height of 193 cm or
less’.
Cumulative frequency (%)

Ogive of basketball
Cumulative frequency

40 100%
35 squad heights
30
25
20 50%
15
10
5

170 175 180 185 190 195 200


Height (cm)
466 M a t h s Q u e s t M a t h s B Ye a r 1 1 f o r Q u e e n s l a n d

WORKED Example 14
Forty sample pieces of rope are tested in an effort to determine their breaking strain.
The maximum load that could be attached to each was recorded.

Breaking strain (kg) Frequency


40–<45 2
45–<50 6
50–<55 8
55–<60 10
60–<65 9
65–<70 4
70–<75 1

a Add a cumulative frequency column to the table.


b Represent the data using an ogive.
c What number of sample pieces broke under a strain of less than 52 kg?
d Find the 75th percentile and write a sentence to explain what it means.
e The manufacturer of the rope wishes to label the rope with an appropriate breaking
strain. What should the rope be rated at if the manufacturer wants 90% of all ropes to
be at least as strong as the labelled rate?

THINK WRITE
a The cumulative frequency column is a a
Breaking Cumulative
‘running total’ of the amounts in the
strain (kg) Frequency frequency
frequency column.
The final entry in the cumulative 40–<45 2 2
frequency column should match the 45–<50 6 8
number of observations. 50–<55 8 16
55–<60 10 26
60–<65 9 35
65–<70 4 39
70–<75 1 40

b The maximum height of the ogive will be b Ogive of rope strength


Cumulative frequency (%)

40 100%
Cumulative frequency

40 because there were 40 observations.


35
The ogive will start at 40 kg on the 30
horizontal axis. The next point will 25
represent the end of the first group. By 20 50%
15
the end of the first group (that is, 45 kg) 10
there had been 2 observations entered. 5

40 45 50 55 60 65 70 75
Breaking strain (kg)
Chapter 9 Presentation of data 467
THINK WRITE
So (45, 2) is the next point on the ogive. The
last point is (75, 40). Join the points to
complete the ogive. Draw a vertical line from
the end point of the ogive to the horizontal axis.
Label the top end point as 100%, then scale the
axis appropriately.
c Find 52 kg on the horizontal axis, go up to the c About 11 pieces of rope broke under a
ogive, then along to the frequency axis. Arrive at 11. strain of less than 52 kg.

Cumulative frequency (%)


Cumulative frequency

40 100%

30

20 50%
11
10

40 5052 60 70
d Find 75% on the percentage axis, go across to d The 75th percentile is 63 kg. 75% of the
the ogive, then down to the horizontal axis. sample pieces broke under a strain of
63 kg or less.
Cumulative frequency (%)
Cumulative frequency

40 100%

30 75

20 50%

10

40 50 6063 70
e If 90% of the ropes are to withstand the strain e 10% of the ropes will break under a
then 10% of the ropes will break with this strain of less than 46 kg. So if the rope is
strain or less. So find the 10th percentile. marketed as 46 kg breaking strain then
90% of the ropes will withstand the
Cumulative frequency (%)
Cumulative frequency

40 100% strain.
30

20 50%

10
10

40 4650 60 70
Find 10% on the percentage axis, go across to
the ogive, then down to the horizontal axis.

A different display
Display Karen’s real estate data in an ogive.
468 M a t h s Q u e s t M a t h s B Ye a r 1 1 f o r Q u e e n s l a n d

remember
1. The cumulative frequency is the number of times that a score plus all lower
scores occur in the set of data. It is obtained by adding together all the
preceding data in the frequency column.
2. An ogive is a line graph of the cumulative frequency results.
3. A percentile is the score below which a particular percentage of the data lies.

9G Cumulative data
WORKED 1 The frequency table below shows the lengths of 77 flathead caught in a fishing competition.
Example
14a, b
a Copy the table and add a cumulative frequency column to it.
b Prepare an ogive of the data.
Length of fish (mm) Frequency Length of fish (mm) Frequency
eBook plus
300–<310 9 340–<350 8
Digital doc:
EXCEL Spreadsheet
310–<320 15 350–<360 7
One-variable statistics
320–<330 20 360–<370 4
330–<340 12 370–<380 2
2 The following frequency table shows the times taken (in seconds) for 60 people
involved in a psychology experiment to complete a simple manipulative puzzle.
Time taken (s) Frequency Time taken (s) Frequency
6–<8 1 14–<16 12
8–<10 4 16–<18 8
10–<12 15 18–<20 2
12–<14 18
a Copy the table and add a cumulative frequency column to it.
b Prepare an ogive of the data.
Chapter 9 Presentation of data 469
WORKED 3 The salaries of the 40 employees of a small manufacturing company are represented
Example
by the accompanying frequency table.
14
Salary (× $1000) Frequency
15–<20 6
20–<25 12
25–<30 8
30–<35 7
35–<40 5
40–<45 1
45–<50 1

a Copy the table and add a cumulative frequency column to it.


b Prepare an ogive of the data.
c How many employees are earning less than $22 000?
d How many employees are earning less than $31 000?
e Find the 75th percentile of the data and write a sentence explaining what it means.
f Find the 50th percentile of the data and write a sentence explaining what it means.
g Find the 25th percentile of the data and write a sentence explaining what it means.
h The management decides to award pay rises to its highest earning employees. The
top 10% of employees will all get a pay rise. How much salary would an
employee need before qualifying for a pay rise?
4 A manufacturer of surf clothing needs to know how many clothes of different sizes to
produce. The manager organises a survey of young people which provides the
following data:
Waist size (cm) Frequency
70–<75 13
75–<80 28
80–<85 46
85–<90 30
90–<95 17
95–<100 8
100–<105 7
105–<110 1
a Copy the table and add a cumulative frequency column to it.
b Prepare an ogive of the data.
c How many young people had a waist size of less than 82 cm?
d How many young people had a waist size of greater than 94 cm?
e Find the 90th percentile of the data and write a sentence explaining what it means.
f Find the 50th percentile of the data and write a sentence explaining what it means.
g The manager decides that production costs can be minimised by only making
garments fitting sizes between 78 cm and 100 cm. What percentage of the
population will not be catered for by this manufacturer?
470 M a t h s Q u e s t M a t h s B Ye a r 1 1 f o r Q u e e n s l a n d

5 A biologist who counts the number of seeds in

Cumulative frequency (%)


60 100%

Cumulative frequency
each of 60 pumpkins presents his findings on
50
the ogive at right.
40
a How many pumpkins contained 30 or
30 50%
fewer seeds?
20
b How many pumpkins contained more than
50 seeds? 10

c What percentage of pumpkins had fewer 10 20 30 40 50 60 70


than 45 seeds? Number of seeds
d What percentage of pumpkins had fewer
than 20 seeds?
e Find the 90th percentile of the data and write
a sentence explaining
what it means.
f Find the 75th percentile of the data and
write a sentence explaining
what it means.
g Find the 50th percentile of the data
and write a sentence explaining
what it means.
h The worst 20% of pumpkins
(those with the fewest seeds)
are to be kept aside for further investigation.
Find the maximum number of seeds for any
pumpkin in this group.

6 A time trial is a race in which each


competitor rides separately, racing ‘against
the clock’.
The following are the times (in seconds) of
20 competitors in a 1 km cycling time trial.
75 72 68 78 75
68 77 80 85 82
73 75 82 90 92
75 73 72 70 83

a Copy and complete the fre- Cumulative


quency distribution table at right. Time Tally Frequency frequency
b Prepare an ogive of the data.
c How many riders finished with a 65–<70
time of 82 seconds or better? 70–<75
d Find the 90th percentile and
write a sentence which explains 75–<80
what it means.
80–<85
e The top 20% of riders are to be
selected for a special training 85–<90
squad. What time would be
needed to qualify for the squad? 90–<95
Chapter 9 Presentation of data 471
7 The following data, collected from a maternity hospital, give the birth weights (in kg)
of 30 babies.

3.7 3.2 3.8 4.1 2.9 3.3 3.6 3.1 3.6 3.9
4.2 2.5 2.7 3.9 3.6 3.2 3.0 2.9 3.4 3.0
3.1 2.8 2.9 3.2 3.1 3.8 3.9 3.3 4.4 3.4

a Copy and complete the following Cumulative


frequency distribution table. Weight Tally Frequency frequency
b Prepare an ogive of the data.
c What percentage of babies had a 2.4–2.7
birth weight of 3.8 kg or less? 2.8–3.1
d Find the 50th percentile and write
a sentence that explains what it 3.2–3.5
means. 3.6–3.9
e Babies of weight 2.6 kg or less
are given special attention by 4.0–4.3
medical staff. What percentage of
babies will be given special atten- 4.4–4.7
tion?
8 (You might like to attempt this question using a graphics calculator.) Thirty samples
of paint are exposed to high temperature until they begin to blister. The following data
show the temperature (°C) at which each began to blister.
75 88 76 92 77 88 82 81 93 96
81 88 75 79 80 81 77 88 92 79
81 83 88 82 85 79 93 84 84 77
a Represent the data on a frequency distribution table using a class size of 5°C.
Include a cumulative frequency column.
b Prepare a histogram of the data.
c Prepare an ogive of the data.
d How many paint samples blistered at a temperature of 85° or less?
e What percentage of the samples blistered at a temperature of 85° or less?
f What percentage of the samples could withstand a temperature of 88°?
g Find the 90th percentile and write a sentence explaining what it means.
h The manufacturer wishes to guarantee the paint at high temperatures. What is the
highest temperature at which the paint could be rated if the management wants at
most 20% of the paint returned because it failed to meet its rating?
472 M a t h s Q u e s t M a t h s B Ye a r 1 1 f o r Q u e e n s l a n d

9 multiple choice
Which of the following is untrue of ogives?
A Ogives display cumulative data.
B Ogives are a type of line graph.
C Ogives tell the number of scores that are of a particular value or less.
D An ogive cannot be interpreted without having the original experimental data at hand.
E An ogive can also be called a cumulative frequency polygon.

10 multiple choice
The 90th percentile of some data is 25. This means that:
A 10% of the data are less than or equal to 25
B 10% of the data are greater than or equal to 25
C 90% of the data are less than or equal to 25
D 90% of the data are greater than or equal to 25
E 25% of the data are less than or equal to 90

11 (You might like to attempt this question using a 85 103 98 72 88


graphics calculator.) The number of people who 86 92 108 112 84
are using a particular plane flight is recorded on 95 99 84 79 88
a weekly basis for 20 weeks. The results are 93 87 79 81 80
shown at right:
a Summarise the data using a frequency distri-
bution table. Use a class size of 10. Include a cumulative frequency column.
b Prepare an ogive of the data.
c How many of the flights had fewer than 95 passengers?
d Find the 75th percentile and write a sentence which explains what it means.
e Find the 50th percentile and write a sentence which explains what it means.
f Find the 25th percentile and write a sentence which explains what it means.
g The management of
the airline is consid-
ering using a smaller
aircraft for the flight.
One of the manage-
ment team calculates
that it would be more
profitable to use a
smaller plane if 20%
of current flights
have 80 or fewer pas-
sengers. Should a
smaller aircraft be
used?

eBook plus

Digital doc:
WorkSHEET 9.2
Chapter 9 Presentation of data 473

summary
Classification of variables and data
• Variables and data can be classified as being categorical or quantitative.
• Categorical data are non-numerical. For example, a survey of car types is not
numerical.
• Categorical data can be either nominal or ordinal.
• Nominal data can be placed in categories where the order is not important.
• Ordinal data can be placed in categories with a logical order.
• Quantitative data can be either counted or measured. For example, a survey of the
daily temperature is quantitative.
• Quantitative data and variables can be either discrete or continuous.
• Discrete data can take only certain values, generally whole numbers, but not
always.
• Continuous data can take any value within a certain range.

Data collection
• A statistical investigation can be done using either a census or a sample.
• A census is when an entire population is used in the investigation.
• A sample is when a small group is used in the investigation and the results are taken
to be representative of the whole group. There are three types of sample.
1. Random sample — chance is the only factor in deciding who participates.
2. Stratified sample — the sample taken is chosen so that it has the same
characteristics as the whole population.
3. Systematic sample — there is a method for deciding who participates in the
sample.

Bias
• If the sample is poorly chosen the results of the investigation will be biased. This
means the results will be skewed towards one section of the population.
• Bias can be introduced at each of the following stages: questionnaire design,
sample selection, interpretation of results.

Stem plots
• A stem-and-leaf plot is a useful way of displaying data up to about 50 observations.
• A stem plot is constructed by breaking the numerals of a record into two parts: a
stem and a leaf. The last digit is always the leaf and any preceding digits the stem.
• When asked to represent data using a stem-and-leaf plot, it is always assumed the
stem-and-leaf plot will be ordered.
• If data are bunched then it may be useful to break the stems into halves or even
fifths.
474 M a t h s Q u e s t M a t h s B Ye a r 1 1 f o r Q u e e n s l a n d

Frequency histograms and bar charts


• On a frequency histogram the vertical axis displays the frequency and the
horizontal axis displays the class intervals.
• Data given in raw form should be summarised first in a frequency table.
• A bar chart consists of bars of equal width separated by small, equal spaces. The
numerical variable should take discrete values only.
• A segmented bar chart is a single bar, drawn to scale, which is used to represent all
the data being studied. Each segment represents a particular group of the data.

Describing the shape of stem plots and histograms


• When data are displayed in a histogram or stem plot, we say that the distribution of
those data is:
1. symmetric, if there is a single peak and the data trail off on either side of the peak
in roughly the same fashion
2. negatively skewed, if the data peak to the right and trail off to the left
3. positively skewed, if the data peak to the left and trail off to the right.
• An outlier is an observation that lies well away from the rest of the data.

Cumulative data
• The cumulative frequency of a score is the number of times that score plus all lower
scores occur in the set of data.
• An ogive is a line graph of the cumulative frequency results.
• A percentile is the score below which a particular percentage of the data lies.
Chapter 9 Presentation of data 475

CHAPTER
review
1 State whether each of the following data types are categorical or quantitative.
a The television program that people watch at 7:00 pm 9A
b The number of pets in each household
c The amount of water consumed by athletes in a marathon run
d The average distance that students live from school
e The mode of transport used between home and school
2 For each of the quantitative data types below, determine if the data are discrete or continuous.
a The dress sizes of Year 11 girls 9A
b The volume of backyard swimming pools
c The amount of water used in households
d The number of viewers of a particular television program
e The amount of time Year 11 students spent studying
3 For each of the following statistical investigations, state whether a census or a sample has
been used. 9B
a The average price of petrol in Brisbane was estimated by averaging the price at 40 petrol
stations.
b The Australian Bureau of Statistics has every household in Australia complete an
information form once every five years.
c The performance of a cricketer is measured by looking at his performance in every
match he has played.
d Public opinion on an issue is sought by a telephone poll of 2000 homes.
4 Name and describe three different methods for selecting a sample.
9B
5 Which method of sampling has been used for each of the following?
a The quality-control department of a tyre manufacturing company road tests every 50th 9B
tyre that comes off the production line.
b To select the students to participate in a survey, a spreadsheet random number generator
selects the roll numbers of 50 students.
c An equal number of men and women are chosen to participate in a survey on fashion.
6 Use your random number generator to select 10 numbers between 1 and 1000.
9B
7 The table at right shows the number of students
in each year of school.
Number of 9B
Year students
In a survey of the school population, how many
students from each year should be chosen, if a 8 200
sample of 60 is selected using a stratified 9 189
sample?
10 175
8 Bias can be introduced into statistics through:
a questionnaire design
11 133 9C
b sample selection 12 124
c interpretation of statistical results
Discuss how bias could be a result of techniques in the above three areas.
476 M a t h s Q u e s t M a t h s B Ye a r 1 1 f o r Q u e e n s l a n d

9 The money (rounded to the nearest whole dollar) raised by Year 12 students is shown below.
9D 78 84 61 73 71 83 87 65 60 67 71 82 84 79 78
Construct a stem plot for the amount raised using:
a the stems 6, 7 and 8
b the stems 6, 7 and 8 split into halves
c the stems 6, 7 and 8 split into fifths.
10 The frequency table below shows the speeds of cars recorded by police. The cars were
9E travelling through a 60 km/h zone. Construct a histogram to display the data.

Class interval Frequency Class interval Frequency


50–51.9 3 64–65.9 10
52–53.9 5 66–67.9 8
54–55.9 6 68–69.9 5
56–57.9 7 70–71.9 3
58–59.9 9 72–73.9 4
60–61.9 10 74–75.9 2
62–63.9 9

11 The money raised (to the nearest whole dollar) by


9F each student in a Grade 3 class on the school
readathon is shown in the stem plot below.
Describe the shape of the distribution of these data
and comment on the existence of any outliers.
Key: 0|8 = $8
Stem Leaf
0 8 9
1 2 3 4 7
2 1 2 2 3 5 7 9
3 0 1 4 5 8
4 3 5 6 7
5 1 3 5
6 4 6
7
8 6
12 A researcher investigates the amount of money spent
9G by 50 people in a local Coles supermarket on Friday 60
Cumulative frequency

night between 6 pm and 8 pm. His findings are presented 50


in the ogive shown at right.
a How many people spent $50 or less? 40
b How many people spent more than $100 on 30
shopping?
20
c What percentage of people spent less than $80?
d Find the 80th percentile of the data and write a short 10
sentence explaining what it means.
e It Nathan was in the lowest 10% of buyers, what is 10 30 50 70 90 110 130
the maximum amount of money he could have spent? Dollars
Chapter 9 Presentation of data 477
Modelling and problem solving
1 A paint company says that 1 litre of paint
can cover an area of 4 m2. For quality
control, every 100th can is tested to check
it will paint at least 4 m2.
a Are the data categorical or
quantitative? Are the data ordinal,
continuous, nominal or discrete?
b What type of sample has been used?
c Five machines each mix a can of paint
in turn; therefore, every can tested
comes from the same machine. A
quality control officer then says the
data are biased. What is meant by the
term bias?
d Explain what could be done to prevent
the data from being biased.

2 The money raised (to the nearest whole


dollar) by each student in a class on the
school walkathon is listed below.
13, 25, 46, 8, 30, 66, 22, 17, 35, 51, 64, 9,
14, 27, 45, 12, 76, 55, 43, 29, 53, 31, 22,
47, 34, 21, 23, 38
a Present the data using an appropriate graphical display.
b Describe the shape of the distribution.
c Describe how this distribution would need to change for it to become a symmetric one.

3 In preparation for an upcoming debate, Rachel, a debating team captain, decides to conduct a
small survey. She asks a number of students whether they agree with the following statement:
‘LOTE should be a compulsory subject in years 7 to 10’. Students are offered one of the
following options:
• strongly disagree •• •
• disagree
• • •
• not sure •• ••• ••• ••
• agree •• •• •• •• •
• strongly agree.
• • • • •
Strongly

Disagree
Not sure
Agree
Strongly
agree
disagree

Rachel records the responses using a dot plot shown at right.


a How many people participated in the survey?
b State whether the data obtained in the survey are categorical or
numerical.
c Classify the data further by selecting an appropriate word from this list: continuous,
nominal, discrete, ordinal.
d Represent the information by using a segmented bar chart.
e What percentage of students agreed with the proposed statement?
f What percentage of students were not sure about their response?
g A particular response option was selected by 20% of all students. What was that option?
h What conclusions might Rachel draw from this data?
478 M a t h s Q u e s t M a t h s B Ye a r 1 1 f o r Q u e e n s l a n d

4 The following data give the marks of 30 students in a physics test.


90 61 58 88 58 69 72 58 62 45
59 64 32 72 52 63 93 58 72 95
57 65 21 44 64 34 72 69 56 63
a Prepare a histogram of the data. (Use a class size of 10.)
b Prepare an ogive of the data.
c Find the 25th, 50th and 75th percentiles of the data from the ogive. Explain what each one
means.
d The top 10% of students are to be given a certificate of excellence. What mark would be
needed to get a certificate?
e What observations can you make about the data?

5 Nathan owns two shops: Designer Tiles and Nathan’s Tiles. Each shop is stocked identically.
Nathan uses his sales records to construct the histograms as shown below.

Designer tiles Nathan’s tiles


12 13
11 12
10 11
9 10
9
Frequency

8
Frequency

7 8
6 7
5 6
4 5
3 4
2 3
1 2
1
20 30 40 50 60 70 80 90 100 20 30 40 50 60 70 80 90 100
Price per m2 ($) Price per m2 ($)

a For each shop describe the distribution of the data, shown by the histograms.
b Use the histograms to arrange the data for each shop into frequency distribution tables.
c Compare the buying trends in the two shops.
d Nathan feels that he needs to rearrange the stock in his shops. What recommendations
would you make?

eBook plus

Digital doc:
Test Yourself
Chapter 9
Summary
statistics
10
syllabus reference
Topic:
• Applied statistical
analysis

In this chapter
10A Measures of central
tendency
10B Range and interquartile
range
10C The standard deviation
10D Boxplots
10E Back-to-back stem plots
10F Parallel boxplots
480 M a t h s Q u e s t M a t h s B Ye a r 1 1 f o r Q u e e n s l a n d

Introduction
Yvonne works in the quality control department of a soft drink bottling company. Soft
drink is bottled by two machines, each of which is set to pour one litre of soft drink
into every bottle. It is part of Yvonne’s job to take 20 bottles of soft drink from each
machine and measure the contents in millilitres. The results she obtained from one such
check are shown below.

Machine A 1009 992 990 1018 1017 985 984 1008 1020 1005
992 983 1020 988 996 984 989 1014 995 1004
Machine B 1002 991 990 980 1004 1018 1008 997 992 999
1010 1004 1001 1003 1009 1004 1006 1001 997 994

Yvonne must use these results to assess if the machines are sufficiently accurate in
dispensing soft drink into the bottles.
One of the main tasks of a statistician is to summarise large volumes of data. It is
useful to find one score that is typical of a whole set of data, or a few figures which can
describe its distribution.

Everybody is different.
How do we measure what is typical?

Finding the mean, median and mode are three different methods of arriving at a
score that is typical or central to the data set. Mean, median and mode are often called
measures of central tendency.
Chapter 10 Summary statistics 481
Measures of central tendency
The mean
The mean of a set of data is what is referred to in everyday language as the average.
For the set of data 4 7 9 12 18:
4 + 7 + 9 + 12 + 18
mean = ----------------------------------------------
5
= 10
The symbol we use to represent the mean is x , that is, a lower-case x with a bar on
top. So, in this case, x = 10.
The formal definition of the mean is:
Σx
x = ------
n
where Σx represents the sum of all of the observations in the data set and n represents
the number of observations in the data set.
Note that the symbol, Σ, is the Greek letter, sigma, which represents ‘the sum of’.
The mean is also referred to as a summary statistic and is a measure of the centre of
a distribution. The mean is the point about which the distribution ‘balances’.
Consider the masses of 7 pieces of
sushi, given in grams, below.
100 120 130 145 160 170 190
The mean is 145 g. The observations
130 and 160 ‘balance’ each other since
they are each 15 g from the mean. Sim-
ilarly, the observations 120 and 170
‘balance’ each other since they are each
25 g from the mean, as do the observ-
ations 100 and 190. Note that the
median is also 145 g. That is, for this
set of data the mean and the median
give the same value for the centre. This
is because the distribution is symmetric.
Now consider two cases in which the
distribution of data is not symmetric.

Case 1
Consider the masses of a different set of 7 pieces of sushi, given in grams below.
100 105 110 115 120 160 200
The median of this distribution is 115 g and the mean is 130 g. There are 5 observations
that are less than the mean and only 2 that are more. In other words, the mean does not
give us a good indication of the centre of the distribution. However, there is still a ‘bal-
ance’ between observations below the mean and those above, in terms of the spread of
all the observations from the mean. Therefore, the mean is still useful to give a measure
of the central tendency of the distribution but in cases where the distribution is skewed,
the median gives a better indication of the centre. For a positively skewed distribution,
as in the case above, the mean will be greater than the median. For a negatively skewed
distribution the mean will be less than the median.
482 M a t h s Q u e s t M a t h s B Ye a r 1 1 f o r Q u e e n s l a n d

Case 2
Consider the data below, showing the
weekly income (to the nearest $10) of
10 families living in a suburban street.
$300 $670 $680 $690 $700
$710 $710 $720 $730 $750
6660
In this case, x = ------------ = $666, and
10
the median is $705.
One of the values in this set, $300,
is clearly an outlier. As a result, the
value of the mean is below the weekly
income of the other 9 households. In
such a case the mean is not very useful
in establishing the centre; however,
the ‘balance’ still remains for this negatively skewed distribution.
The mean is calculated by using the values of the observations and because of this it
becomes a less reliable measure of the centre of the distribution when the distribution is
skewed or contains an outlier. Because the median is based on the order of the observ-
ations rather than their value, it is a better measure of the centre of such distributions.

WORKED Example 1
Calculate the mean of the set of data below.
10, 12, 15, 16, 18, 19, 22, 25, 27, 29
THINK WRITE
1 Find the total of all the scores. Σx = 10 + 12 + 15 + 16 + 18 + 19 + 22 + 25 + 27 + 29
= 193
2 Divide the total by the number 193
of scores in the data set. x = ---------
10
= 19.3

Graphics Calculator tip! Calculating the mean


of data in a list
Consider the data set in Worked example 1, which is presented as a single list of
figures.
For the Casio fx-9860G AU
1. Press MENU and then select STAT. Clear any data
stored in List 1. Enter the data from the example
in List 1.

2. Press F2 (CALC) and then F6 (SET). Ensure


that 1Var XList is set as List 1 and 1Var Freq as 1.
Chapter 10 Summary statistics 483
3. Press EXIT (or EXE ) to return to the previous
screen and then press F1 (1VAR) to display all
summary statistics. Read the value shown for x .
<Casio04.01>

For the TI-Nspire CAS


1. Open a new Lists & Spreadsheet document
(press /N and then select 3: Add Lists &
Spreadsheet). Enter the data into the first column <TI04.01>
(column A).

2. Press b and then select 4: Statistics.

<TI04.01>

3. Select 1: Stat Calculations and then 1: One-


Variable Statistics. Ensure that Num of Lists
shows 1.
<TI04.01>

4. Press e to highlight OK and then press ·.


Ensure that a[] is shown for X1 List (as the scores
are in column A) and 1 is shown for Frequency
List. The 1st Result Column should show b[] (as <TI04.01>
we need the results to be displayed beginning in
column B). Use the e key to move to the next
line. Press e to highlight OK.

5. Press · to display the summary statistics. Read


the value shown for x .

<TI04.01>

When data are presented in a frequency table with class intervals and we don’t know
what the raw data are, we employ another method to find the mean of these grouped
data. This other method is shown in the example that follows and uses the midpoints of
the class intervals to represent the raw data.
484 M a t h s Q u e s t M a t h s B Ye a r 1 1 f o r Q u e e n s l a n d

WORKED Example 2
The ages of a group of 30 people attending a superannuation seminar are recorded in the
frequency table below.

Age Frequency Age Frequency


(class intervals) f (class intervals) f
20–29 1 50–59 6
30–39 6 60–69 3
40–49 13 70–79 1

Calculate the mean age of those attending the seminar.

THINK WRITE

1 Since we don’t have individual raw ages,


but rather a class interval, we need to
decide on one particular age to represent
each interval. We use the midpoint, m, of
the class interval. Add an extra column Mid-
to the table to display these. point of
The midpoint of the first interval is Age class
(class Frequency interval
20 + 29 f×m
------------------ = 24.5 , the midpoint of the intervals) f m
2
second interval is 34.5 and so on. 20–29 1 24.5 24.5
2 Multiply each of the midpoints by the 30–39 6 34.5 207
frequency and display these values in 40–49 13 44.5 578.5
another column headed f × m. For the 50–59 6 54.5 327
first interval we have 24.5 × 1 = 24.5. 60–69 3 64.5 193.5
For the second interval we have 70–79 1 74.5 74.5
34.5 × 6 = 207 and so on.
Σ f = 30 Σ( f × m)
3 Sum the product of the midpoints and = 1405
the frequencies in the f × m column.
24.5 + 207 + 578.5 + 327 + 193.5 +
74.5 = 1405 1405
So, x = ------------
4 Divide this sum by the total number 30
of people attending the seminar. ≈ 46.8 (correct to 1 decimal place)

Remember that the Greek letter sigma, Σ, represents ‘the sum of’. So, Σ f means the
sum of the frequencies and is the total of all the numbers in the frequency column.
To find the mean for grouped data,
Σ( f × m)
x = ------------------------
Σf
where f represents the frequency of the data and m represents the midpoint of the class
interval of the grouped data.
Chapter 10 Summary statistics 485

Graphics Calculator tip! Calculating the mean of data in a


frequency table
Consider the data set in Worked example 2 which is presented in a frequency table.
For the Casio fx-9860G AU
1. Press MENU and then select STAT. Clear any data
in List 1 and List 2. Enter the midpoint of the
class intervals in List 1 and the frequencies in
List 2. You can label the lists if you wish.
2. Press F2 (CALC) and then F6 (SET). Set 1Var
XList as List 1 and 1Var Freq as List 2.

3. Press EXIT to return to the previous screen and


then F1 (1VAR) to display all summary
statistics. Read the value shown for x .

For the TI-Nspire CAS


1. Open a new Lists & Spreadsheet document.
Enter the midpoint of the class intervals in
column A and the frequencies in column B.

2. Press b and then select 4: Statistics. Select


1: Stat Calculations followed by 1: One-Variable
Statistics. Ensure that Num of Lists shows 1.

3. Press e to highlight OK and then press ·. Enter


a[] for X1 List (as the scores are in column A),
b[] for Frequency List (as the frequencies are in
column B) and c[] for 1st Result Column (as we
need the results to be displayed beginning in
column C). Press e to highlight OK.

4. Press · to display the summary statistics. Read


the value shown for x .
486 M a t h s Q u e s t M a t h s B Ye a r 1 1 f o r Q u e e n s l a n d

The median
The median is the midpoint of a set of data. Half the data are less than or equal to the
median.
Consider the set of data: 2 5 6 8 11 12 15. These data are in ordered form (that is,
from lowest to highest). There are 7 observations. The median in this case is the middle
or fourth score; that is, 8.
Consider the set of data: 1 3 5 6 7 8 8 9 10 12. These data are in ordered
form also; however, in this case there is an even number of scores; that is, there are
10 scores. The median in this case lies halfway between the 5th score (7) and the
7+8
6th score (8). So the median is 7.5. (Alternatively, median = ------------ = 7.5.)
2
When there are n records in a set of ordered data, the median can be located at the
+ 1
 n-----------
- th position.
 2 
Checking this against our previous example, we have n = 10; that is, there were
10 + 1
10 observations in the set. The median was located at the  --------------- = 5.5th position;
 2 
that is, halfway between the 5th and the 6th terms.
A stem plot provides a quick way of locating a median since the data in a stem plot
are already ordered.

WORKED Example 3
Consider the stem plot below which contains 22 observations. What is the median?
Stem Leaf
2 3 3
2 5 7 9
3 1 3 3 4 4
3 5 8 9 9
4 0 2 2
4 6 8 8 8 9
Key: 3|4 = 34

THINK WRITE

n+1
1 Find the median position, where n = 22. Median =  ------------ th position
 2 
22 + 1
=  --------------- th position
 2 
= 11.5th position
2 Find the 11th and 12th terms. 11th term = 35
12th term = 38
3 The median is halfway between the 11th and Median = 36.5
12th terms.
Chapter 10 Summary statistics 487
Mode
There are many examples where neither the mean nor the median is the appropriate
measure of the typical score in a data set.
Consider the case of a clothing store. It needs to re-order a supply of dresses. To
know what sizes to order it looks at past sales of this particular style and gathers the
following data:
8 12 14 12 16 10 12 14 16 18
14 12 14 12 12 8 18 16 12 14
For this data set the mean dress size is 13.2. Dresses are not sold in size 13.2, so this
has very little meaning. The median is 13, which also has little meaning as dresses are
sold only in even-numbered sizes.
What is most important to the clothing store is the dress size that sells the most. In
this case size 12 occurs most frequently. The score that has the highest frequency is
called the mode.
When two scores occur most often an equal number of times, both scores are given
as the mode. In this situation the scores are bimodal. If all scores occur most often an
equal number of times, then the distribution has no mode.
To find the mode from a frequency distribution table, we simply give the score that
has the highest frequency.

WORKED Example 4
For the frequency distribution at right, state
Score Frequency
the mode.
14 3

15 6

16 11

17 14

18 10

19 7

THINK WRITE
The highest frequency is 14 which belongs Mode = 17
to the score 17 and so 17 is the mode.

When a table is presented using grouped data, we do not have a single mode. In
these cases, the class with the highest frequency is called the modal class.
Just as with ungrouped data, if grouped data has more than one modal class then
all modal classes should be stated in your answer.
488 M a t h s Q u e s t M a t h s B Ye a r 1 1 f o r Q u e e n s l a n d

Graphics Calculator tip! Calculating


and the mode
the median

When you enter data into your graphics calculator as shown earlier, a list of summary
statistics is given and can be accessed using the scroll function.
The Casio fx-9860G AU will display the median (shown as Med) and the mode or
modes (shown as Mod) as well as the frequency of any modes.
The TI-Nspire CAS will display the median (shown as MedianX) but not the mode
or modes.

remember
1. The mean is given by x = Σ ------x- where Σx represents the sum of all the
n
observations in the data set and n represents the number of observations in the
data set.
2. The mean is calculated by using the values of the observations and because of
this it becomes a less reliable measure of the centre of the distribution when the
distribution is skewed or contains an outlier.

Σ ( f × m)
3. To find the mean for grouped data, x = -------------------------- where f represents the
Σ f
frequency of the data and m represents the midpoint of the class interval of the
grouped data.
4. The median is the midpoint of a set of data. Half the data are less than or equal
to the median.
5. When there are n observations in a set of ordered data, the median can be
n+1
located at the  ------------ th position.
 2 
6. The mode is the score with the highest frequency.

10A Measures of central tendency

A graphics calculator may be used for this exercise.


WORKED 1 Find the mean of each of the following sets of data.
Example
a 5 6 8 8 9
1
b 3 4 4 5 5 6 7 7 7 8 8 9 9 10 10 12
c 4.3 4.5 4.7 4.9 5.1 5.3 5.5 5.6
d 11 13 15 15 16 18 20 21 22
e 0.4 0.5 0.7 0.8 0.8 0.9 1.0 1.1 1.2 1.0 1.3
Chapter 10 Summary statistics 489
2 Calculate the mean of each of the following and explain whether or not it gives us a
eBook plus good indication of the centre of the data.
Digital doc:
a 0.7 0.8 0.85 0.9 0.92 2.3
EXCEL Spreadsheet b 14 16 16 17 17 17 19 20
One-variable
statistics c 23 24 28 29 33 34 37 39
d 2 15 17 18 18 19 20

3 The number of people attending painting


classes at the local TAFE college for each week
during the first semester is given below.
15 12 15 11 14 8 14 15 11 10
7 11 12 14 15 14 15 9 10 11
What is the mean number of people attending
each week? (Express your answer to the nearest
whole number.)

4 multiple choice
The ages of a group of junior pilots joining an
international airline are indicated on the stem
plot below.
The mean age of this group of pilots is: Stem Leaf
A 20 2 1
B 28 2 2
C 29 2 4 5
D 29.15 2 6 6 7
E 29.5 2 8 8 8 9
3 0 1 1
3 2 3
3 4 4
3 6
Key: 2|1 = 21 yrs 3 8

5 multiple choice
The number of people present each week at a 15-week horticul- Stem Leaf
tural course is given by the stem plot at right. 0 4
The mean number of people attending each week was closest to: 0 7
A 17.7 B 18 1 2 4
C 19.5 D 20 1 5 5 6 7 8
E 21.2 2 1 2 4
Key: 2|4 = 24 people 2 7 7 7

6 For each of the following, write down whether the mean or the median would provide
a better indication of the centre of the distribution.
a A positively skewed distribution
b A symmetric distribution
c A distribution with an outlier
d A negatively skewed distribution
490 M a t h s Q u e s t M a t h s B Ye a r 1 1 f o r Q u e e n s l a n d

WORKED 7 Find the mean of each set of data given below.


Example
2 a Class interval Frequency, f b Class interval Frequency, f
0–9 1 0–4 2
eBook plus 10–19 3 5–9 5
Digital doc: 20–29 6 10–14 7
EXCEL Spreadsheet 30–39 17 15–19 13
One-variable statistics
with intervals 40–49 12 20–24 8
50–59 5 25–29 6

c Class interval Frequency, f d Class interval Frequency, f

0–49 2 1–6 14
50–99 7 7–12 19
100–149 8 13–18 23
150–199 14 19–24 22
200–249 12 25–30 20
250–299 5 31–36 14

8 The ages of people attending a beginner’s course in karate are indicated in the
frequency table below.

Frequency,
Age f

10–14 5
15–19 5
20-24 7
25–29 4
30–34 3
35–39 2
40–44 2
45–49 1

What is the mean age of those


attending the course?
(Express your answer correct
to the nearest whole number.)
Chapter 10 Summary statistics 491
WORKED 9 Write down the median of the sets of data shown in the following stem plots. The key
Example
for each stem plot is 3 | 4 = 34.
3 a Stem Leaf b Stem Leaf
0 7 0 0 0 1 1
1 2 3 0 2 2 3 3
2 2 4 5 7 9 0 4 4 5 5 5 5 5 5 5 5
3 0 2 3 6 8 8 0 6 6 6 6 7
4 4 7 8 9 9 0 8 8 8 9
5 2 7 8 1 0 0 1
6 1 3 1 3 3
1 5 5
1 7
1
c Stem Leaf d Stem Leaf
0 1 3 1
0 2 3
0 4 4 5 3
0 6 6 6 7 3 6
0 8 8 8 8 9 9 3 8 9
1 0 0 0 1 1 1 1 4 0 0 1 1 1
1 2 2 2 3 3 3 4 2 2 3 3 3 3
1 4 4 5 5 4 4 5 5 5
1 6 7 7 4 6 7
1 8 9 4 9
e Stem Leaf
60 2 5 8
61 1 3 3 6 7 8 9
62 0 1 2 4 6 7 8 8 9
63 2 2 4 5 7 8
64 3 6 7
65 4 5 8
66 3 5
67 4
10 For each of the following sets of data, write down the median.
eBook plus a 2 4 6 7 9
Digital doc:
b 12 15 17 19 21
EXCEL Spreadsheet c 3 4 5 6 7 8 9
One-variable
statistics
d 3 5 7 8 12 13 15 16
e 12 13 15 16 18 19 21 23 24 26
f 3 8 4 2 1 6 5
g 16 21 14 28 23 15 11 19 25
h 7 4 3 4 9 5 10 4 2 11
i 29 23 22 33 26 18 37 22 16
11 Find the mode for each of the following. (Hint: Some are bimodal and others have no
mode.)
a 16, 17, 19, 15, 17, 19, 14, 16, 17
b 147, 151, 148, 150, 148, 152, 151
c 2, 3, 1, 9, 7, 6, 8
d 68, 72, 73, 72, 72, 71, 72, 68, 71, 68
e 2.6, 2.5, 2.9, 2.6, 2.4, 2.4, 2.3, 2.5, 2.6
492 M a t h s Q u e s t M a t h s B Ye a r 1 1 f o r Q u e e n s l a n d

WORKED 12 Use the tables below to state the mode of the distribution.
Example
4 a Score Frequency b Score Frequency c Score Frequency
1 2 5 1 39 4
2 4 6 3 40 1
3 5 7 5 41 5
4 6 8 8 42 6
5 3 9 5 43 3
10 3 44 6
45 2

13 For each of the following grouped distributions, state the modal class.

a Class Frequency b Class Frequency


1–4 6 1–7 3
5–8 12 8–14 8
9–12 30 15–21 9
13–16 23 22–28 25
17–20 46 29–35 12
21–24 27 36–42 11
25–28 9 43–49 2

14 The following data give the


age of 25 patients admitted
to the emergency ward of a
hospital.
18 16 6 75 24
23 82 74 25 21
43 19 84 72 31
74 24 20 63 79
80 20 23 17 19
a Represent the data in a
frequency distribution
table. (Use classes 0–14,
15–29, 30–44, etc.)
b Find the mean age of
patients admitted.
c Find the median class of
age of patients admitted.
d Find the modal class for age of patients admitted.
e Do any of your statistics (mean, median or mode) give a clear representation of the
typical age of an emergency ward patient?
f Give some reasons that could explain the pattern of the distribution of data in this
question.
Chapter 10 Summary statistics 493
15 The batting scores for two cricket
players over six innings are as
follows:
Player A 31, 34, 42, 28, 30, 41
Player B 0, 0, 1, 0, 250, 0
a Find the mean score for each
player.
b Which player appears to be better
if the mean result is used?
c Find the median score for each
player.
d Which player appears to be better
when the decision is based on the
median result?
e Which player do you think would
eBook plus be more useful to have in a
cricket team and why? How can
Digital doc:
WorkSHEET 10.1
the mean result sometimes lead to
a misleading conclusion?

Mean and median amount of soft drink


Remember Yvonne’s quality control check? We are going to calculate the mean
and median amount of soft drink dispensed by each machine into each bottle.
1 Use your graphics calculator to store the data for Machine A as a list. Name the
list MA. The data for Machine A were

1009 992 990 1018 1017 985 984 1008 1020 1005
992 983 1020 988 996 984 989 1014 995 1004

2 Use your graphics calculator to store the data for Machine B as a list. Name the
list MB. The data for machine B were

1002 991 990 980 1004 1018 1008 997 992 999
1010 1004 1001 1003 1009 1004 1006 1001 997 994

3 Use the statistics function of the graphics calculator to find the mean and
median amount of soft drink dispensed into bottles by each machine.
4 At this stage can you say which machine most accurately dispenses soft drink
into bottles?

Notes:
1. If you do not have a graphics calculator, you can still calculate the mean and
median of each data set.
2. If you are using a TI-Nspire CAS calculator, save this problem for use later in
the chapter.
494 M a t h s Q u e s t M a t h s B Ye a r 1 1 f o r Q u e e n s l a n d

Range and interquartile range


Measures of central tendency may not always provide an accurate comparison;
although two sets of scores may have the same mean, median and mode, they may still
be very different data sets. Consider the results obtained by two groups of 10 students
on the same mathematics test.
Group A: 45, 46, 47, 48, 50, 50, 52, 53, 54, 55
Group B: 10, 20, 30, 40, 50, 50, 60, 70, 80, 90
Although the mean, median and mode mark in both groups is 50, the data sets are
very different. We can see that Group A has a bunched group of scores but Group B’s
scores are spread out.
The range and interquartile range are examples of a measure of spread. These
measures of spread help us analyse the spread of various data sets.
The range
The range is the easiest of this group of summary statistics to calculate. The range of a
set of data is the difference between the highest and lowest values in that set.
It is usually not too difficult to locate the highest and lowest values in a set of data.
Only when there is a very large number of observations might the job be made more
difficult. In the example above that compared the results on a mathematics test by two
groups, the range for Group A is found by subtracting the lowest score (45) from the
highest score (55). Similarly, we can say that the range for Group B is 90 − 10 = 80.
Statistically, we can write the lowest score as minX and the highest score as maxX; hence
the range is found using the formula:
Range = maxX − minX.
The values of maxX and minX can be found using a graphics calculator as you will
see later in the chapter.
While the range gives us some idea about the spread of the data, it is not terribly
informative since it gives us no idea of how the data are distributed between the highest
and lowest values.

The interquartile range


We have seen that the median divides a set of data in half. Similarly, quartiles divide a
set of data in quarters. The symbols used to refer to these quartiles are Q1, Q2 and Q3.
The middle quartile, Q2, is the median.
The interquartile range IQR = Q3 − Q1
The interquartile range gives us the range of the middle 50% of values in our set of data.
There are four steps to locating Q1 and Q3.
Step 1. Write down the data in ordered form from lowest to highest.
Step 2. Locate the median; that is, locate Q2.
Step 3. Now consider just the lower half of the set of data. Find the middle score. This
score is Q1.
Step 4. Now consider just the upper half of the set of data. Find the middle score. This
score is Q3.
The four cases given below illustrate this method.
Case 1
Consider data containing the 6 observations: 3 6 10 12 15 21.
The data are already ordered. The median is 11.
Chapter 10 Summary statistics 495
Consider the lower half of the set, which is 3 6 10. The middle score is 6, so Q1 = 6.
Consider the upper half of the set, which is 12 15 21. The middle score is 15, so
Q3 = 15.
Case 2
Consider a set of data containing the 7 observations: 4 9 11 13 17 23 30.
The data are already ordered. The median is 13.
Consider the lower half of the set, which is 4 9 11. The middle score is 9, so Q1 = 9.
Consider the upper half of the set, which is 17 23 30. The middle score is 23,
so Q3 = 23.
Case 3
Consider a set of data containing the 8 observations: 1 3 9 10 15 17 21 26.
The data are already ordered. The median is 12.5.
Consider the lower half of the set, which is 1 3 9 10. The middle score is 6,
so Q1 = 6.
Consider the upper half of the set, which is 15 17 21 26. The middle score is 19,
so Q3 = 19.
Case 4
Consider a set of data containing the 9 observations: 2 7 13 14 17 19 21 25 29.
The data are already ordered. The median is 17.
Consider the lower half of the set, which is 2 7 13 14. The middle score is 10,
so Q1 = 10.
Consider the upper half of the set, which is 19 21 25 29. The middle score is 23,
so Q3 = 23.

WORKED Example 5
The ages of the patients who attended the casualty department of an inner suburban
hospital on one particular afternoon are shown below.
14 3 27 42 19 17 73 60 62 21 23 2 5 58 33 19 81 59 25 17 69
Find the interquartile range of these data.

THINK WRITE

1 Order the data. 2 3 5 14 17 17 19 19 21 23


25 27 33 42 58 59 60 62 69 73 81
2 Find the median. The median is 25 since ten scores lie below it and ten
lie above it.
3 Find the middle score of the lower For the scores 2 3 5 14 17 17 19
half of the data. 19 21 23, the middle score is 17.
So, Q1 = 17.
4 Find the middle score of the upper For the scores 27 33 42 58 59 60 62
half of the data. 69 73 81, the middle score is 59.5.
So, Q3 = 59.5.
5 Calculate the interquartile range. IQR = Q3 − Q1 = 59.5 − 17 = 42.5
496 M a t h s Q u e s t M a t h s B Ye a r 1 1 f o r Q u e e n s l a n d

WORKED Example 6
Parents are often shocked by the amount of
money their children spend on junk food.
The data below give the amount spent (to
the nearest whole dollar) on junk food in a
particular week by each student in a Maths
B class.
15 12 17 23 21 19 16 11 17 18 23
24 25 21 20 37 17 25 22 21 19
Calculate the interquartile range for these
data.

THINK WRITE

1 Count the number of scores. n = 21


2 List the scores in ascending order and 11 12 15 16 17 17 17 18 19 19 20 21
divide the data into two groups of 10, 21 21 22 23 23 24 25 25 37
omitting the middle score of 20 from
either group.
3 Find the median of the lower half of the Q1 = 17
data to find Q1.
4 Find the median of the upper half of the Q3 = 23
data to find Q3.
5 The interquartile range is found IQR = 23 – 17
subtracting the lower quartile from the =6
upper quartile.

A graphics calculator provides possibly the fastest way of locating quartiles and
hence finding the value of the interquartile range.

Graphics Calculator tip! Finding summary statistics


Earlier in the chapter we looked at how to enter a set of scores into the graphics calcu-
lator to find the mean. The summary statistics screen shows all the information needed
to find the median, range and interquartile range. The Casio fx-9860G AU graphics
calculator can also find the mode. Consider the data set in Worked example 6.
For the Casio fx-9860G AU
1. Press MENU and then select STAT. Enter the data
in List 1.
Chapter 10 Summary statistics 497
2. Press F2 (CALC) and then Number of scores
F6 (SET) . Ensure that 1Var XList Lowest score
Lower quartile
is set as List 1 and 1Var Freq as 1. Median
Press EXIT to return to the <Casio04.01> Upper quartile
Highest score
previous screen and then press
F1 (1VAR) to display all
summary statistics. The mean is the
first piece of data displayed. We
need to scroll down to find the
information needed for the median,
range and interquartile range.
First mode
Second mode
3. Scroll further to see information <Casio04.01> Number of modes
Frequency of modes
relating to the mode.

For the TI-Nspire CAS


1. Open a new Lists & Spreadsheet document.
Enter the data into column A.

2. Press b and then select 4: Statistics. Select


1: Stat Calculations followed by 1: One-Variable
Statistics. Ensure that Num of Lists shows 1.
Press e to highlight OK and then press ·.
Check that X1 List shows a[] (scores are in
column A), Frequency List shows 1 and 1st
Result Column shows b[] (results to be displayed
beginning in column B).
3. Press · to accept the setting and
display the summary statistics. The
mean is the first piece of data Lowest score
displayed. Scroll down to find the <TI04.01> Lower quartile
information needed for the median, Median
Upper quartile
range and interquartile range. Highest score

In most cases we are asked to find the interquartile range of a grouped distribution.
This requires us to draw a cumulative frequency polygon and find the 25th and 75th
percentile.
A percentile is a measure of where in a set of scores an individual score lies. For
example, the 25th percentile has 25% of scores below it and 75% above it.
To find the interquartile range, draw a second vertical axis that shows the 25th, 50th,
and 75th percentile. A line is drawn from the 25th, 50th and 75th percentile to the ogive
and then down to the horizontal axis. The value for the quartiles can then be calculated.
The median is the score that is found at the 50th percentile.
498 M a t h s Q u e s t M a t h s B Ye a r 1 1 f o r Q u e e n s l a n d

WORKED Example 7

Cumulative frequency
50
The cumulative frequency histogram and polygon at right
40
shows the number of customers who order different
volumes of concrete from a ready-mix concrete company 30
during a day. 20
Find the: 10
a median 0
b interquartile range for this distribution. 5 5 5 5 5 5
0.2 0.7 1.2 1.7 2.2 2.7
Number of customers
THINK WRITE
a 1 Draw a vertical axis showing the a
50 100%
percentiles.
40
75%
30
50%
20
10 25%

0 0%
5 5 5 5 5 5
0.2 0.7 1.2 1.7 2.2 2.7
Number of customers

2 Draw a line for the 50th percentile to Median = 0.9


the ogive and estimate the median.

b 1 Draw a line for the 25th and 75th b Lower quartile = 0.4
percentiles and estimate these Upper quartile = 1.6
values.
2 Calculate the interquartile range by Interquartile range = 1.6 − 0.4
subtracting the lower quartile from = 1.2
the upper quartile.

remember
1. The range of a set of data is the difference between the highest and lowest
values in that set.
2. The interquartile range IQR = Q3 − Q1.
3. The interquartile range gives us the range of the middle 50% of values in our
set of data.
4. There are four steps to locating Q1 and Q3.
Step 1. Write down the set of data in ordered form from lowest to highest.
Step 2. Locate the median, that is, locate Q2.
Step 3. Now consider just the lower half of the set of data. Find the middle
score. This score is Q1.
Step 4. Now consider just the upper half of the set of data. Find the middle
score. This score is Q3.
5. The interquartile range of a grouped distribution is estimated from an ogive.
Chapter 10 Summary statistics 499
Range and interquartile
10B range
1 Write down the range of the sets of data shown in the following stem plots. The key for
each stem plot is 3 | 4 = 34.
a Stem Leaf b Stem Leaf
0 7 0 0 0 1 1
1 2 3 0 2 2 3 3
2 2 4 5 7 9 0 4 4 5 5 5 5 5 5 5 5
3 0 2 3 6 8 8 0 6 6 6 6 7
4 4 7 8 9 9 0 8 8 8 9
5 2 7 8 1 0 0 1
6 1 3 1 3 3
1 5 5
1 7
1

c Stem Leaf d Stem Leaf


0 1 3 1
0 2 3
0 4 4 5 3
0 6 6 6 7 3 6
0 8 8 8 8 9 9 3 8 9
1 0 0 0 1 1 1 1 4 0 0 1 1 1
1 2 2 2 3 3 3 4 2 2 3 3 3 3
1 4 4 5 5 4 4 5 5 5
1 6 7 7 4 6 7
1 8 9 4 9

e Stem Leaf
60 2 5 8
61 1 3 3 6 7 8 9
62 0 1 2 4 6 7 8 8 9
63 2 2 4 5 7 8
64 3 6 7
65 4 5 8
66 3 5
67 4
2 For each of the following sets of data, write down the range.
eBook plus a 2 4 6 7 9
Digital doc: b 12 15 17 19 21
EXCEL Spreadsheet c 3 4 5 6 7 8 9
One-variable
statistics d 3 5 7 8 12 13 15 16
e 12 13 15 16 18 19 21 23 24 26
f 3 8 4 2 1 6 5
g 16 21 14 28 23 15 11 19 25
h 7 4 3 4 9 5 10 4 2 11
i 29 23 22 33 26 18 37 22 16
500 M a t h s Q u e s t M a t h s B Ye a r 1 1 f o r Q u e e n s l a n d

WORKED 3 a The number of cars that used the drive-in at a McBurger restaurant during each hour,
Example
from 7.00 am until 10.00 pm on a particular day, is shown below.
5
14 18 8 9 12 24 25 15 18 25 24 21 25 24 14
eBook plus Find the interquartile range of this set of data.
b On the same day, the number of cars stopping during each hour at the nearby
Digital docs:
SkillSHEET 10.1
Kenny’s Fried Chicken restaurant was recorded and is shown below.
Finding the median 7 9 13 16 19 12 11 18 20 19 21 20 18 10
EXCEL Spreadsheet Find the interquartile range of these data.
Interquartile range

4 Write down a set of data for which n = 5, the median is 6 and the range is 7.

5 Write down a set of data for which n = 8, the median is 7.5 and the range is 10.

6 multiple choice
The quartiles for a set of data are calculated and found to be Q1 = 13, Q2 = 18, and
Q3 = 25. Which of the following statements is true?
A The interquartile range of the data is 5.
B The interquartile range of the data is 7.
C The interquartile range of the data is 12.
D The median is 12.
E The median is 19.
It is recommended that a graphics calculator be used for questions 7 and 8.

WORKED 7 For each of the following sets of data find the median, the interquartile range and the
Example
6 range.
a 16 12 8 7 26 32 15 51 29 45
19 11 6 15 32 18 43 31 23 23
b 22 25 27 36 31 32 39 29 20 30
23 25 21 19 29 28 31 27 22 29
c 1.2 2.3 4.1 2.4 1.5 3.7 6.1 2.4 3.6 1.2
6.1 3.7 5.4 3.7 5.2 3.8 6.3 7.1 4.9

8 For each set of data shown on the stem plots, find the median, the interquartile range
and the range.
a Stem Leaf b Stem Leaf
2 3 5 5 6 7 8 9 9 1 4
3 0 2 2 3 4 6 6 7 8 8 1
4 2 2 4 5 6 6 6 7 9 2 1 4
5 0 3 3 5 6 2 5 7 8 8 9
6 2 4 3 1 2 2 2 4 4 4 4
7 5 9 3 5 5 5 6
8 2 4 3 4
9 7 4
10
11 4 Key: 4|2 = 42 Key: 2|5 = 25
Chapter 10 Summary statistics 501
9 The frequency histogram and polygon at right

Cumulative frequency
WORKED 50
Example
displays the results of a survey of 50 drivers 45
7 40
who were asked about the number of speeding 35
fines they had received. 30
a Use the ogive to find the median of the 25
20
distribution. 15
10
b Find the lower quartile. 5
c Find the upper quartile. 0
0 1 2 3 4 5
d Calculate the interquartile range. Number of speeding fines
received by drivers
10 The frequency distribution table
below shows the result of a
survey of 90 households who
were asked about the number of
times they had been the victim of
crime.

Score Frequency

0 26

1 31

2 22

3 8

4 3

a Add a column for cumulative


frequency to the table.
b Draw a cumulative frequency
histogram and polygon.
c Use your graph to find the
median of the distribution.
d Calculate the interquartile
range.

Range of soft drink amounts


Take another look at Yvonne’s quality control check. We have previously found two
measures of central tendency for the two soft drink dispensing machines, but these
alone are not enough to state which machine dispenses soft drink most accurately.
1 Find the range of amounts dispensed by each machine.
2 Find the interquartile range of amounts dispensed by each machine.
3 From these results, which machine appears to dispense soft drink most
accurately?
502 M a t h s Q u e s t M a t h s B Ye a r 1 1 f o r Q u e e n s l a n d

The standard deviation


The standard deviation is the most sophisticated and also the most useful measure of
spread.
The standard deviation σ can be calculated by using the following formula:
∑ f ( xi – x )2
s= ------------------------------
n
To calculate the standard deviation by hand, use the following steps:
Step 1. Find the mean.
Step 2. Find the difference between each piece of data and the mean.
Step 3. Square the differences.
Step 4. Add the squared differences.
Step 5. Divide by the number of scores.
Step 6. Take the square root.
This algorithm is used to find the standard deviation in the following worked example.

WORKED Example 8
The following data give the number of lollies in each of 8 packets. Find the standard
deviation of the data.
14, 14, 13, 15, 16, 13, 14, 17
THINK WRITE

14 + 14 + 13 + 15 + 16 + 13 + 14 + 17
Find the mean. x = --------------------------------------------------------------------------------------------
1 8
= 14.5
2 Find the difference between each Differences from mean:
score and the mean. −0.5, −0.5, −1.5, 0.5, 1.5, −1.5, −0.5, 2.5
3 Square each difference. Squared differences:
0.25, 0.25, 2.25, 0.25, 2.25, 2.25, 0.25, 6.25
4 Add the squared differences. 0.25 + 0.25 + 2.25 + 0.25 + 2.25 + 2.25 + 0.25 + 6.25
= 14
5 Divide by the number of scores. 14 ÷ 8 = 1.75
6 Take the square root and round to 1.75 = 1.3229
4 decimal places. The standard deviation σ = 1.3229.

Fortunately, you will not always have to go through this series of steps each time you
wish to calculate a standard deviation. Your calculator should have a built-in program
for the computation of standard deviations.

Predicting the mean and standard deviation of a


population from a sample
It is not always practical to measure a particular statistic for a whole population so,
usually, a sample of the population is taken. It is found that the mean of a sample is a
reliable estimate of the mean of a population but the standard deviation of a population
is slightly less than the standard deviation of any sample drawn from it. In other words,
the complete population shows slightly less variability than any sample drawn from it.
Chapter 10 Summary statistics 503
A formula used to predict the standard deviation, s, of a complete population from
∑ f ( xi – x )2
a sample of scores is: s = ------------------------------
n–1
Notice that the only difference between the formulas is that the divisor has changed
from n to n − 1. Your calculator is also equipped with a built-in program for the calcu-
lation of this formula. It is worth checking that you can obtain both results from your
calculator and can distinguish between them. Try reworking the data from Worked
example 8. You should find that the standard deviation among the 8 packets of lollies
was σ = 1.3229. If you were to use this sample to predict the standard deviation of all
boxes of lollies then the standard deviation would be s = 1.4142.
A final point worth noting about the standard deviation is that, despite its sophisti-
cation, it is still influenced to a high degree by extreme values. Care should be taken
when using this statistic with data that include such values.

WORKED Example 9
The following frequency distribution gives the prices paid by a car wrecking yard for
40 car wrecks.
Price ($) Frequency Price $ Frequency
0–500 2 2000–2500 7
500–1000 4 2500–3000 6
1000–1500 8 3000–3500 3
1500–2000 10
a Find the mean and standard deviation in the price paid for these wrecks.
b Estimate the mean and standard deviation in the price paid for wrecks by this yard in
general.
THINK WRITE/DISPLAY
Use a graphics calculator to find the mean and standard
deviation.
For the Casio fx-9860G AU
1. Press MENU and then select STAT. Clear any data in
List 1 and List 2. Enter the class centres (the midpoints
of the class intervals) in List 1 and the frequencies in
List 2. You can label the lists if you wish.
2. Press F2 (CALC) and then F6 (SET). Set 1Var
XList as List 1 and 1Var Freq as List 2.

3. Press EXIT to return to the previous screen and then


F1 (1VAR) to display all summary statistics. The
mean ( x ), population standard deviation (xσn) and
sample standard deviation (xσn–1) are shown.

Continued over page


504 M a t h s Q u e s t M a t h s B Ye a r 1 1 f o r Q u e e n s l a n d

THINK WRITE/DISPLAY
For the TI-Nspire CAS
1. Open a new Lists & Spreadsheet document. Enter
the class centres (the midpoints of the class intervals)
in column A and the frequencies in column B. You
may wish to name both columns.

2. Press b and then select 4: Statistics. Select 1: Stat


Calculations followed by 1: One-Variable Statistics.
Ensure that Num of Lists shows 1. Press e to
highlight OK and then press ·. Enter a[] for X1 List <TI04.01>
(as the scores are in column A), b[] for Frequency List
(as the frequencies are in column B) and c[] for 1st
Result Column (as we need the results to be displayed
beginning in column C). Press e to highlight OK.
3. Press · to display the summary statistics. The mean
( x ) is the first piece of data displayed. Use the
NavPad to find the population standard deviation (σx)
and sample standard deviation (sx). <TI04.01>

a List the mean and population standard deviation of a Mean = $1825


the data given. Standard deviation = $787
b To estimate these figures in general we give the b In general:
mean and sample standard deviation. Mean = $1825
Standard deviation = $797

remember
1. The standard deviation of a group of scores can be found using the formula:
∑ f ( xi – x )2
σ = ----------------------------
-
n
2. The standard deviation of a population can be predicted from a sample of
scores by using the formula:
∑ f ( xi – x )2
s = ----------------------------
-
n–1
3. The lower the standard deviation the closer together the scores.
4. A graphics calculator can be used to find the standard deviation. Using the
Casio fx-9860G AU, the population standard deviation is displayed as xσn and
the sample standard deviation is shown as xσn–1. Using the TI-Nspire CAS,
the population standard deviation is displayed as σx and the sample standard
deviation is shown as sx.
Chapter 10 Summary statistics 505

10C The standard deviation

WORKED 1 Use the algorithm (series of steps) to find the standard deviation of the following data
Example
8
without using your calculator’s in-built program.
a 3, 5, 8, 2, 7, 1, 6, 5 b 11, 8, 7, 12, 10, 11, 14
c 25, 15, 78, 35, 56, 41, 17, 24 d 5.2, 4.7, 5.1, 12.6, 4.8

2 Now use the calculator’s in-built program to check each of the standard deviations
that you calculated in the previous question.
You may use your calculator’s in-built program for finding the standard deviation
and mean to answer the rest of the questions.
3 Consider the following two groups of people.
Group A Group B
160 170 170 170 170 170 180 160 170 170 110 230 170 180
Height (cm)

a Calculate the mean height, median height and mode height for each group. What
do you notice?
b Are the groups really the same?
c Which group would you expect to show the greatest range in heights?
d Which group would you expect to show the greatest interquartile range in heights?
e Which group would you expect to show the greatest standard deviation in heights?
f Calculate these statistics to confirm your predictions.
4 The following frequency distribution table shows the number of visitors that came to
a city museum during the course of a month.
Visitor
number 80–<90 90–<100 100–<110 110–<120 120–<130 130–<140
Frequency 1 4 11 9 4 2
a Find the range of the data.
b Find the mean of the data.
c Find the standard deviation of the data.
5 multiple choice
Calculate the standard deviation of the following data to 3 decimal places.
Score 10–<20 20–<30 30–<40 40–<50 50–<60
Frequency 1 6 9 4 1
A 3.027 B 9.437 C 9.209 D 34.048 E None of the above
506 M a t h s Q u e s t M a t h s B Ye a r 1 1 f o r Q u e e n s l a n d

WORKED 6 The following frequency distribution table shows the life expectancy of 175 household
Example
light globes.
9
Life (h) Frequency Life (h) Frequency
200–<250 2 450–<500 38
250–<300 5 500–<550 26
300–<350 12 550–<600 15
350–<400 25 600–<650 7
400–<450 42 650–<700 3

a Find the range of the data.


b Find the mean and standard deviation in the lifetimes of this sample of light globes.
c Estimate the mean and standard deviation in the lifetimes of all light globes of this
brand.

7 The following frequency distribution table shows the distribution of daily maximum
temperatures during the course of a full year.
Maximum Number of Maximum Number of
temperature (°C) days temperature (°C) days
0–<5 4 20–<25 94
5–<10 22 25–<30 19
10–<15 95 30–<35 5
15–<20 124 35–<40 2

a Add a cumulative frequency column to the table.


b Draw an ogive of the data.
c Find the upper and lower quartiles of the data and calculate the interquartile range.
d Use the ogive to find the median (50th percentile of the data).
e Find the mean of the data.
f Find the standard deviation of the data.
g Find the range of the data.

8 The following data give the number of fruit that have formed on each of 30 trees in an
orchard.
45 48 52 36 38 72 36 74 56 46
81 73 46 48 44 39 52 58 57 65
60 53 54 58 41 44 47 76 68 55
a Complete a frequency distribution table for the data.
b Draw an ogive of the data.
c Use the ogive to find the median, lower quartile and upper quartile of the data.
d Find the interquartile range of the data.
e Find the mean of the data.
f Find the standard deviation of the data.
g Estimate the standard deviation in the number of fruit for the whole orchard.
h Find the range of the data.
Chapter 10 Summary statistics 507
9 The polygons drawn below show the lifetimes of two samples of different brands of
toaster elements when subject to continued use.

Hot Wire

Electric Mate

150 160 170 180 190 200 210 220 230


Hours

a Which brand has the longest mean life?


b Estimate the mean life of each brand.
c Which brand has the greatest standard deviation in its performance?
d What does this say about the consistency of this element?
e Which brand is best? Give a case in support of Electric Mate. Give a case in
support of Hot Wire.
10 Crunch and Crinkle are two brands of potato crisps. Each is sold in packets nominally
eBook plus
of the same size and for the same price. Upon investigation of a sample of packets of
Digital doc:
each, it is found that Crunch and Crinkle have the same mean weight (25 g). The stan-
WorkSHEET 10.2 dard deviation of the weights of Crunch is, however, 5 g and the standard deviation of
the weights of Crinkle is 2 g. Which brand do you think would represent the best
value for money under these circumstances? Why?

Standard deviation of soft


drink amounts
Returning to Yvonne’s measurements, we are now ready to find the standard
deviation of the amounts of soft drink dispensed by each machine.
1 Use the data lists MA and MB stored on your graphics calculator to find the
standard deviation for each machine.
2 Explain your choice of σ or s.
3 Interpret the results in terms of assessing which machine dispenses the soft
drink most accurately.

Boxplots
Five number summary
A five number summary is a list consisting of the lowest score, lower quartile, median,
upper quartile and greatest score of a set of data.
A five number summary gives information about the spread of a set of data. The con-
vention is not to detail the numbers with labels but to present them in order; so, for
example, the five number summary:
4 15 21 23 28
would be interpreted as lowest score 4, lower quartile 15, median 21, upper quartile 23
and greatest score 28.
508 M a t h s Q u e s t M a t h s B Ye a r 1 1 f o r Q u e e n s l a n d

WORKED Example 10
From the following five number summary find:
a the median b the interquartile range c the range.
29 37 39 44 48
THINK WRITE
The figures are presented in the order of minX = 29, Q1 = 37, median = 39, Q3 = 44,
lowest score, lower quartile, median, upper maxX = 48
quartile, greatest score.
a The median is 39. a Median = 39
b The interquartile range is the difference b IQR = Q3 − Q1
between the upper and lower quartiles. = 44 − 37
=7
c The range is the difference between the c Range = maxX − minX
greatest score and the lowest score. = 48 − 29
= 19

Boxplots
A boxplot (or box-and-whisker plot) is a graph of the five number summary. It is a
powerful way to show the spread of data. Boxplots consist of a central divided box with
attached ‘whiskers’. The box spans the interquartile range. The median is marked by a
vertical line inside the box. The whiskers indicate the range of scores:

Indicates the Indicates the Indicates the Indicates the Indicates the
lowest score lower quartile median upper quartile greatest score

Boxplots are always drawn to scale. They 4 15 21 23 28


are presented either with the five number
summary figures attached as labels (diagram
at right) or with a scale presented alongside the boxplot like the diagram below.

0 5 10 15 20 25 30 Scale

Interpreting a boxplot
The boxplot neatly divides the data into four sections. One-quarter of the scores lie
between the lowest score and the lower quartile, one-quarter between the lower quartile
and the median, one-quarter between the median and the upper quartile, and one-quarter
between the upper quartile and the greatest score. The reader can easily see where clus-
tering of the data occurs. For example, a small box with relatively long whiskers would
indicate that half of the data (from Q1 to Q3 ) would be confined to a small range and the
data could be described as clustered. A wide box with relatively short whiskers would
indicate that half of the data (from Q1 to Q3 ) would be spread over a wide range and the
data could be described as spread.
Chapter 10 Summary statistics 509
Consider the boxplots below with their matching histograms.
f f

Size Size
Normally distributed data Clustered data
f f

Size Size
Spread data Positively skewed data
f

Size
Negatively skewed data

Identification of extreme values


Extreme values often make the whiskers appear longer than they should and hence give
the appearance that the data are spread over a much greater range than they really are.
If an extreme value occurs in a set of data it can be denoted by a small cross on the
boxplot. The whisker is then shortened to the next largest (or smallest) figure.
The boxplot below shows that the lowest score was 5. This was an extreme value as
the rest of the scores were located within the range 15 to 42.
0 5 10 15 20 25 30 35 40 45 Scale

WORKED Example 11
The stem-and-leaf plot at right gives the speed of 25 cars Key:8|2 = 82 km/h
caught by a roadside speed camera. 8*|6 = 86 km/h
a Prepare a five number summary of the data. Stem Leaf
b Draw a boxplot of the data. (Identify any extreme 8 2 2 4 4 4 4
values.) 8* 5 5 6 6 7 9 9 9
c Describe the distribution of the data. 9 0 1 1 2 4
9* 5 6 9
10 0 2
10*
11 4
Continued over page
510 M a t h s Q u e s t M a t h s B Ye a r 1 1 f o r Q u e e n s l a n d

THINK WRITE
25 + 1
1 First identify the positions of the median The median is the --------------- th score — that is,
and upper and lower quartiles. There are the 13th score. 2
25 pieces of data. The median is the 12 + 1
The Q1 is the --------------- th score in the lower
n+1 2
------------ th score. The lower quartile is the half — that is, the 6.5th score. That is,
2
median of the lower half of the data. The halfway between the 6th and 7th scores.
upper quartile is the median of the upper The Q3 is halfway between the 6th and 7th
half of the data (each half contains scores in the upper half of the data.
12 scores).
2 Mark the position of the median and upper Key: 8|2 = 82 km/h
and lower quartiles on the stem plot. 8*|6 = 86 km/h
Stem Leaf Q1
8 2 2 4 4 4 4 Median
8* 5 5 6 6 7 9 9 9
9 0 1 1 2 4
9* 5 6 9
10 0 2 Q3
10*
11 4

a Write the five number summary: a Five number summary:


The lowest score is 82. 82, 84.5, 89, 94.5, 114
The lower quartile is between 84 and 85 —
that is, 84.5.
The median is 89.
The upper quartile is between 94 and 95 —
that is, 94.5.
The greatest score is 114.
b Start by ruling a suitable scale. Remember b 80 90 100 110 km/h
to include the units of measurement. The
box represents the interquartile range so it ×
runs from 84.5 to 94.5. The median is a
vertical line in the box at 89. The
whiskers should extend to the lowest
score (82) and the highest score (114).
But the score 114 is a great deal higher
than any of the others in the set and might
be regarded as an extreme value. It should
be indicated by a cross and the whisker
will extend only as far as 102 (the second
largest number in the set).
c Even when the extreme value is excluded c The data are skewed (positively) and
the data appear to be skewed with high include one extremely high value.
values being spread over a much greater
range.
Chapter 10 Summary statistics 511

Graphics Calculator tip! Creating a boxplot from a


frequency table
A boxplot can be drawn on the graphics calculator. Consider the data used in Worked
example 11.
For the Casio fx-9860G AU
1. Press MENU and then select STAT. Enter the data
from Worked example 11 in List 1.

2. Press F1 (GRPH) and then F6 (SET). Set


Graph Type to a boxplot by pressing F6 ( ) for
more options and then F2 (Box). Set XList as
List 1, the Frequency as 1 and Outliers on. (Note
that outliers are switched on but can be switched
off if required.)
3. Press EXIT to return to the previous screen and
then F1 (GPH1) to draw the boxplot.

4. The Trace function can be used to display the key


values in the boxplot. Press SHIFT F1 (TRCE)
and use the left and right arrow keys.
To draw a boxplot using data from a frequency table,
follow the steps above with the following changes:
In step 1, enter the scores as List 1 and the
frequencies as List 2.
In step 2, set Frequency as List 2.

For the TI-Nspire CAS


1. Open a new Lists & Spreadsheet document.
Enter the data from the example into column A,
giving the column a title.

2. Press b and then select 3: Data followed by


5: Quick Graph. A dot plot results.

3. Press b and then select 1: Plot Type followed by


2: Box Plot. The arrow keys on the NavPad can
then be used to display the five number summary.
512 M a t h s Q u e s t M a t h s B Ye a r 1 1 f o r Q u e e n s l a n d

remember
1. A five number summary is a list consisting of the lowest score, lower quartile,
median, upper quartile and greatest score of a set of data.
2. A boxplot is a graphical representation of a five number summary and is a
powerful tool to show the spread of data.
3. The box spans the interquartile range; the median is marked by a vertical line
inside the box and the whiskers extend to the lowest and greatest scores.
4. Boxplots are always drawn to scale.
5. If an extreme value occurs in a set of data, it can be denoted by a small cross;
the whisker is then shortened to the next largest (or smallest) value.

10D Boxplots

WORKED 1 From the following five number summary find:


Example
10 a the median
b the interquartile range
c the range.
6, 11, 13, 16, 32

2 From the following five number summary find:


a the median
b the interquartile range
c the range.
101, 119, 122, 125, 128

3 From the following five number summary find:


a the median
b the interquartile range
c the range.
39.2, 46.5, 49.0, 52.3, 57.8

4 The boxplot below shows the distribution of final points scored by a football team
over a season’s roster.
50 70 90 110 130 150 Points

a What was the team’s greatest points score?


b What was the team’s least points score?
c What was the team’s median points score?
d What was the range of points scored?
e What was the interquartile range of points scored?
Chapter 10 Summary statistics 513
5 The boxplot below shows the distribution of data formed by counting the number of
honey bears in each of a large sample of packs.
30 35 40 45 50 55 60 Scale

a What was the largest number of honey bears in any pack?


b What was the smallest number of honey bears in any pack?
c What was the median number of honey bears in any pack?
d What was the range of numbers of honey bears per pack?
e What was the interquartile range of honey bears per pack?

Questions 6 to 8 refer to the following boxplot.

5 10 15 20 25 30 Scale

×
6 multiple choice
The median of the data is:
A 5 B 20 C 23 D 25 E 31

7 multiple choice
The interquartile range of the data:
A is 23 B is 26 C is 5 D is 20 to 25
E cannot be determined because of extreme values.

8 multiple choice
Which of the following is not true of the data represented by the boxplot?
A One-quarter of the scores are between 5 and 20.
B Half of the scores are between 20 and 25.
C The lowest quarter of the data is spread over a wide range.
D Most of the data are contained between the scores of 5 and 20.
E The data are skewed left.

9 The number of sales made each day by a salesperson is recorded over a 2-week
eBook plus
period:
Digital doc: 25, 31, 28, 43, 37, 43, 22, 45, 48, 33
EXCEL Spreadsheet a Prepare a five number summary of the data. (There is no need to draw a stem-and-
Boxplots
leaf plot of the data. Just arrange them in order of size.)
b Draw a boxplot of the data.

10 The data below show monthly rainfall in millimetres.

J F M A M J J A S O N D
10 12 21 23 39 22 15 11 22 37 45 30

a Prepare a five number summary of the data.


b Draw a boxplot of the data.
514 M a t h s Q u e s t M a t h s B Ye a r 1 1 f o r Q u e e n s l a n d

WORKED 11 The stem plot at right details the age of Key:1|8 = 18 years
Example
25 offenders who were caught during random Stem Leaf
11
breath testing. 1 8 8 9 9 9
a Prepare a five number summary of the data. 2 0 0 0 1 1 3 4 6 9
b Draw a boxplot of the data. 3 0 1 2 7
c Describe the distribution of the data. 4 2 5
5 3 6 8
6 6
7 4

12 The stem-and-leaf plot at right details the Key:32|4 = $324 000


price at which 30 apartments in a particular Stem Leaf
suburb sold for. 32 4 7 9
a Prepare a five number summary of the data. 33 0 0 2 5 5
b Draw a boxplot of the data. 34 0 0 2 3 5 5 7 9 9
(You might like to use a graphics calculator 35 0 0 2 3 7 7 8
for this question.) 36 0 2 2 5 8
37 5

13 The following data detail the number of


hamburgers sold by a fast food outlet every
day over a 4-week period.

M T W T F S S
125 144 132 148 187 172 181
134 157 152 126 155 183 188
131 121 165 129 143 182 181
152 163 150 148 152 179 181

a Prepare a stem-and-leaf plot of the data.


(Use a class size of 10.)
b Draw a boxplot of the data.
(You might like to use a graphics calculator
for this question.)

14 The following data show the ages of 30 mothers upon the birth of their first baby.
22 18 17 22 24
25 32 19 23 28
31 19 23 25 23
21 33 23 24 20
29 18 22 24 20
22 17 48 18 20
a Prepare a stem-and-leaf plot of the data. (Use a class size of 5.)
b Draw a boxplot of the data. Indicate any extreme values appropriately.
c Describe the distribution in words. What does the distribution say about the age
that mothers have their first baby?
(You might like to use a graphics calculator for this question.)
Chapter 10 Summary statistics 515
15 multiple choice
Match the boxplot with its most likely histogram.
A f B f

Size Size
Cf Df E f

Size Size Size

Back-to-back stem plots


In chapter 9, we saw how to construct a stem plot for a set of univariate data. We can
also extend a stem plot so that it displays bivariate data. Specifically, we shall create a
stem plot that displays the relationship between a numerical variable and a categorical
variable. We shall limit ourselves in this section to categorical variables with just two
categories, for example, gender. The two categories are used to provide two, back-to-
back leaves of a stem plot.
A back-to-back stem plot is used to display bivariate data involving a numerical
variable and a categorical variable with 2 categories.

WORKED Example 12
The girls and boys in Grade 4 at Kingston
Primary School submitted projects on the
Olympic Games. The marks they obtained
out of 20 are given below.

Girls’ marks 16 17 19 15 12 16 17 19 19 16
Boys’ marks 14 15 16 13 12 13 14 13 15 14

Display the data on a back-to-back stem plot.


Continued over page
516 M a t h s Q u e s t M a t h s B Ye a r 1 1 f o r Q u e e n s l a n d

THINK WRITE
1 Identify the highest and lowest scores Highest score = 19
in order to decide on the stems. Lowest score = 12
Use a stem of 1, divide into fifths.
2 Create an unordered stem plot first. Put Key: 1|2 = 12
the boys’ scores on the left, and the Leaf Stem Leaf
girls’ scores on the right. Boys Girls
1
3 2 3 3 1 2
4 5 4 5 4 1 5
6 1 6 7 6 7 6
1 9 9 9
3 Now order the stem plot. The scores on Key: 1|2 = 12
the left should increase in value from Leaf Stem Leaf
right to left, while the scores on the Boys Girls
right should increase in value from left 3 3 3 2 1 2
to right. 5 5 4 4 4 1 5
6 1 6 6 6 7 7
1 9 9 9

The back-to-back stem plot allows us to make some visual comparisons of the two
distributions. In the above example, the centre of the distribution for the girls is higher
than the centre of the distribution for the boys. The spread of each of the distributions
seems to be about the same. For the boys, the marks are grouped around the 12–15
marks; for the girls, they are grouped around the 16–19 marks. On the whole, we can
conclude that the girls obtained better marks than the boys did.
To get a more precise picture of the centre and spread of each of the distributions we
can use the summary statistics discussed earlier in this chapter. Specifically, we are
interested in:
1. the mean and the median (to measure the centre of the distributions), and
2. the interquartile range and the standard deviation (to measure the spread of the
distributions).
We saw that the calculation of these summary statistics is very straightforward and
rapid using a graphics calculator.

WORKED Example 13
The number of ‘how to vote’ cards handed out by various Australian Labor Party and
Liberal Party volunteers during the course of a polling day is shown below.
Labor 180 233 246 252 263 270 229 238 226 211
193 202 210 222 257 247 234 226 214 204
Liberal 204 215 226 253 263 272 285 245 267 275
287 273 266 233 244 250 261 272 280 279
Display the data using a back-to-back stem plot and use this, together with summary
statistics, to compare the distributions of the number of cards handed out by the Labor
and Liberal volunteers.
Chapter 10 Summary statistics 517
THINK WRITE
1 Construct the stem plot. Key: 18|0 = 180
Leaf Stem Leaf
Labor Liberal
0 18
3 19
4 2 20 4
4 1 0 21 5
9 6 6 2 22 6
8 4 3 23 3
7 6 24 4 5
7 2 25 0 3
3 26 1 3 6 7
0 27 2 2 3 5 9
28 0 5 7
2 Use a graphics calculator to calculate For the Labor volunteers:
the summary statistics: the mean, the Mean = 227.9
median, the standard deviation and the Median = 227.5
interquartile range. Enter each set of Interquartile range = 36
data as a separate list. (See the Standard deviation = 23.9
Graphics calculator tip on page 496 on For the Liberal volunteers:
how to calculate these values.) Mean = 257.5
Median = 264.5
Interquartile range = 29.5
Standard deviation = 23.4
3 Comment on the relationship. From the stem plot we see that the Labor distribution
is symmetric and therefore the mean and the median
are very close, whereas the Liberal distribution is
negatively skewed.
Since the distribution is skewed, the median is a
better indicator of the centre of the distribution than
is the mean.
Therefore, comparing the medians, we have the
median number of cards handed out for Labor at 228
and for Liberal at 265, which is a big difference.
The standard deviations were similar, as were the
interquartile ranges. There was not a lot of difference
in the spread of the data.
In essence, the Liberal Party volunteers handed out
a lot more ‘how to vote’ cards than the Labor Party
volunteers did.

remember
1. A back-to-back stem plot displays bivariate data involving a numerical variable
and a categorical variable with two categories.
2. In the ordered stem plot, the scores on the left side of the stem increase in value
from right to left.
3. Together with summary statistics, back-to-back stem plots can be used for
comparing two distributions.
518 M a t h s Q u e s t M a t h s B Ye a r 1 1 f o r Q u e e n s l a n d

10E Back-to-back stem plots

WORKED 1 The marks (out of 50), obtained for the end-of-term test by the students in German and
Example
12 French classes are given below. Display the data on a back-to-back stem plot.

German 20 38 45 21 30 39 41 22 27 33 30 21 25 32 37 42 26 31 25 37
French 23 25 36 46 44 39 38 24 25 42 38 34 28 31 44 30 35 48 43 34

2 The birth masses of 10 boys and 10 girls (in kilograms, to the nearest 100 grams) are
recorded in the table below. Display the data on a back-to-back stem plot.
Boys 3.4 5.0 4.2 3.7 4.9 3.4 3.8 4.8 3.6 4.3
Girls 3.0 2.7 3.7 3.3 4.0 3.1 2.6 3.2 3.6 3.1

WORKED 3 The number of delivery trucks making deliveries to a supermarket each day over a
Example
13 2-week period was recorded for two neighbouring supermarkets — supermarket A and
supermarket B. The data are shown below.

A 11 15 20 25 12 16 21 27 16 17 17 22 23 24
B 10 15 20 25 30 35 16 31 32 21 23 26 28 29

a Display the data on a back-to-back stem plot.


b Use the stem plot, together with some summary statistics, to compare the distri-
butions of the number of trucks delivering to supermarkets A and B.

4 The marks out of 20 for males and females on a science test for a Year 11 class are
given below.
Females 12 13 14 14 15 15 16 17
Males 10 12 13 14 14 15 17 19

a Display the data on a back-to-back stem plot.


b Use the stem plot, together with some summary statistics, to compare the distri-
butions of the marks of the males and the females.

5 The end-of-year English marks for 10 students in an English class were compared over
2 years. The marks for the first year and for the same students in the second year are
shown below.
First year 30 31 35 37 39 41 41 42 43 46
Second year 22 26 27 28 30 31 31 33 34 36

a Display the data on a back-to-back stem plot.


b Use the stem plot, together with some summary statistics, to compare the distri-
butions of the marks obtained by the students over the 2 years.
Chapter 10 Summary statistics 519
6 The age and gender of a group of people attending a fitness class are recorded below.
Female 23 24 25 26 27 28 30 31
Male 22 25 30 31 36 37 42 46
a Display the data on a back-to-back stem plot.
b Use the stem plot, together with some summary statistics, to compare the distri-
butions of the ages of the female to male members of the fitness class.

7 The scores on a board game are recorded for a group of kindergarten children and for a
group of children in a preparatory school.
Kindergarten 3 13 14 25 28 32 36 41 47 50
Prep. School 5 12 17 25 27 32 35 44 46 52
a Display the data on a back-to-back stem plot.
b Use the stem plot, together with some summary statistics, to compare the distributions
of the scores of the kindergarten children compared to the preparatory school children.
8 multiple choice
The pair of variables that could be displayed on a back-to-back stem plot is:
A the height of a student and the number of people in the student’s household
B the time put into completing an assignment and a pass or fail score on the assignment
C the weight of a businessman and his age
D the religion of an adult and the person’s head circumference
E the income bracket of an employee and the time the employee has worked for the
company
9 multiple choice
A back-to-back stem plot is a useful way of displaying the relationship between:
A the proximity to markets (km) and the cost of fresh foods on average per kilogram
B height and head circumference
C age and attitude to gambling (for or against)
D weight and age
E the money spent during a day of shopping and the number of shops visited on that day
520 M a t h s Q u e s t M a t h s B Ye a r 1 1 f o r Q u e e n s l a n d

Parallel boxplots
We saw in the previous section that we could display relationships between a numerical
variable and a categorical variable with just two categories by using a back-to-back stem
plot.
When we want to display a relationship between a numerical variable and a
categorical variable with more than two categories, a parallel boxplot can be used.
A parallel boxplot is obtained by constructing individual boxplots for each
distribution, using the common scale.
Construction of individual boxplots was discussed in detail earlier in this chapter
(see page 507). In this section we concentrate on comparing distributions represented
by a number of boxplots (that is, on the interpretation of parallel boxplots).

WORKED Example 14
The four Year 7 classes at Western Secondary College complete the same end-of-year
maths test. The marks, expressed as percentages for each of the students in the four
classes, are given below.

7A 7B 7C 7D 7A 7B 7C 7D
40 60 50 40 69 78 70 69
43 62 51 42 63 82 72 73
45 63 53 43 63 85 73 74
47 64 55 45 68 87 74 75
50 70 57 50 70 89 76 80
52 73 60 53 75 90 80 81
53 74 63 55 80 92 82 82
54 76 65 59 85 95 82 83
57 77 67 60 89 97 85 84
60 77 69 61 90 97 89 90
Display the data using a parallel boxplot and use this to describe any similarities or
differences in the distributions of the marks between the four classes.
THINK WRITE/DISPLAY
1 Create the first boxplot (for class 7A) on a graphics
calculator.
For the Casio fx-9860G AU
1. Press MENU and then select STAT. Enter the data for
Class 7A in List 1. Press F1 (GRPH) and then
F6 (SET). Set Graph Type to a boxplot with the
XList as List 1, the Frequency as 1 and Outliers off.
2. Press EXIT to return to the previous screen and then
F1 (GPH1) to draw the boxplot. Press SHIFT
F1 (TRCE) and use the left and right arrow keys to
display the key values.
Chapter 10 Summary statistics 521
THINK WRITE
For the TI-Nspire CAS
1. Open a new Lists & Spreadsheet
document. Enter the data for Class 7A
in the first column, giving the column
a title. Press b and then select
3: Data followed by 5: Quick
Graph. A dot plot results. Press b
and then select 1: Plot Type followed
by 2: Box Plot.
2. The NavPad can be used to display
the five number summary.
2 Sketch the first boxplot using pen 7D
and paper, leaving room for three
7C
additional plots.
7B
3 Repeat step 1 for the other three
classes. Show all four boxplots 7A
sharing a common scale.
30 40 50 60 70 80 90 100
Maths mark (%)
4 Describe the similarities and Class 7B had the highest median mark and the range
differences between the four of the distribution was only 37. The lowest mark in
distributions. 7B was 60.
We notice that the median of 7A’s marks is
approximately 60. So, 50% of students in 7A
received less than 60. This means that half of 7A had
scores that were less than the lowest score in 7B.
The range of marks in 7A was about the same as
that of 7D with the highest scores in each about
equal, and the lowest scores in each about equal.
However, the median mark in 7D was higher than the
median mark in 7A so, despite a similar range, more
students in 7D received a higher mark than in 7A.
While 7D had a top score that was higher than that
of 7C, the median score in 7C was higher than that of
7D and the bottom 25% of scores in 7D were less
than the lowest score in 7C. In summary, 7B did best,
followed by 7C, then 7D and finally 7A.
522 M a t h s Q u e s t M a t h s B Ye a r 1 1 f o r Q u e e n s l a n d

remember
1. A relationship between a numerical variable and a categorical variable with
more than two categories can be displayed using a parallel boxplot.
2. A parallel boxplot is obtained by constructing individual boxplots for each
distribution, using a common scale.

10F Parallel boxplots


WORKED 1 The heights (in cm) of students in 9A, 10A and 11A were recorded and
Example
14
are shown in the table below.
9A 10A 11A 9A 10A 11A 9A 10A 11A
eBook plus 120 140 151 146 153 164 158 168 175
Digital doc: 126 143 153 147 156 166 160 170 180
EXCEL Spreadsheet
Parallel boxplots 131 146 154 150 162 167 162 173 187
138 147 158 156 164 169 164 175 189
140 149 160 157 165 169 165 176 193
143 151 163 158 167 172 170 180 199
a Construct a parallel boxplot to show the data.
b Use the boxplot to compare the distributions of height for the 3 classes.
2 The amounts of money contributed annually to superannuation schemes by people in
3 different age groups are shown below.
20–29 30–39 40–49 20–29 30–39 40–49
2000 4000 10 000 6500 7000 13 700
3100 5200 11 200 6700 8000 13 900
5000 6000 12 000 7000 9000 14 000
5500 6300 13 300 9200 10 300 14 300
6200 6800 13 500 10 000 12 000 15 000
a Construct a parallel boxplot to show the data.
b Use the boxplot to comment on the distributions.
Chapter 10 Summary statistics 523
3 The numbers of jars of vitamin A, B, C and multi-vitamins sold per week by a local
chemist are shown below.

Vitamin
5 6 7 7 8 8 9 11 13 14
A
Vitamin
10 10 11 12 14 15 15 15 17 19
B
Vitamin
8 8 9 9 9 10 11 12 12 13
C
Multi-
12 13 13 15 16 16 17 19 19 20
vitamins

a Construct a parallel boxplot to display the data.


b Use the boxplot to compare the distributions of sales for the 4 types of vitamin.

4 multiple choice
The ages of the employees at 5 different companies of the same size are compared
using the parallel boxplots shown below.
Company A
Company B
Company C
Company D
Company E

20 25 30 35 40 45 50 55 60
For each of the following, select from:
A company A B company B
C company C D company D
E company E
a Which company has the greatest
range of ages?
b Which company has the greatest
interquartile range of ages?
c Which company has the lowest
median age?
d Which company has the greatest
range of ages among their oldest
25% of employees?
524 M a t h s Q u e s t M a t h s B Ye a r 1 1 f o r Q u e e n s l a n d

summary
Measures of central tendency
x
• The mean is given by x = ∑
-------- where ∑ x represents the sum of all observations in
n
the data set and n represents the number of observations in the data set.
• The mean is calculated by using the values of the observations, and because of this
it becomes a less reliable measure of the distribution when the distribution is
skewed or contains an outlier.
x = ∑ ( f × m)
• To find the mean for grouped data, ---------------------------------
- where f represents the frequency
∑f
of the data and m represents the midpoint of the class interval of the grouped data.
• The median is the midpoint of a set of data. Half the data are less than or equal to
the median. Where there are no observations in a set of ordered data, the median is
n+1
located at the  ------------ th position.
 2 
• The mode is the score in the data set with the highest frequency.

Range and interquartile range


• The range of a data set is the difference between the highest and lowest values in
that data set.
• The interquartile range IQR = Q3 − Q1.
• There are four steps to locating Q1 and Q3.
Step 1. Write down the data set in order from lowest to highest.
Step 2. Locate the median; that is, locate Q2.
Step 3. Now consider the lower half of the data set. Find the middle score. This
score is Q1.
Step 4. Now consider the upper half of the data set. Find the middle score. This
score is Q3.
• The values of the median, as well as Q1 and Q3, can be estimated by using an ogive.

Standard deviation
• The standard deviation of a group of scores can be found using the formula:
2
∑ f ( xi – x )
σ= ----------------------------
- or by using a calculator.
n
• The standard deviation of a population can be predicted from a sample of scores by
using the formula:
2
∑ f ( xi – x )
s = ----------------------------
- or by using a calculator.
n–1
Chapter 10 Summary statistics 525
Boxplots
• A five number summary is a list, consisting of the lowest score, lower quartile,
median, upper quartile and the greatest score (in that order) of the data.
• A boxplot is a graph of the five number summary.
• The boxplot is a powerful tool to show the spread of the data.
• Boxplots are always drawn to scale.
• The box spans the interquartile range; the median is marked by the vertical line
inside the box; the whiskers extend to the lowest and greatest scores.

Indicates the Indicates the Indicates the Indicates the Indicates the
lowest score lower quartile median upper quartile greatest score
• The extreme values can be denoted by a small cross; the whiskers are then
shortened to the next largest (or smallest) value.

Comparing sets of data


• Back-to-back stem plots
1. are useful to compare the distribution of two similar sets of data
2. share the same stem
3. contain a key, which usually relates to data on the right
4. have the data on the left arranged outwards from the plot as it increases.
• Parallel boxplots
1. are useful for quantitative comparisons
2. share a common scale
3. compare two or more sets of data.
526 M a t h s Q u e s t M a t h s B Ye a r 1 1 f o r Q u e e n s l a n d

CHAPTER
review
1 Calculate the mean of each of the following sets of scores.
10A
a 4, 9, 5, 3, 5, 6, 2, 7, 1, 10
b 65, 67, 87, 45, 90, 92, 50, 23
c 7.2, 7.9, 7.0, 8.1, 7.5, 7.5, 8.7
d 5, 114, 23, 12, 25
2 Complete the frequency distribution table below and use it to estimate the mean of the
10A distribution.

Class Class centre (x) Frequency (f )


21–24 3
25–28 9
29–32 17
33–36 31
37–40 29
41–44 25
45–48 19
49–52 10
Σf =

3 Use the statistics function on your calculator to find the mean of each of the following sets
10A of scores.
a 2, 18, 26, 121, 96, 32, 14, 2, 0, 0
b 2, 2, 12, 12, 12, 32, 32, 47, 58
c 0.2, 0.3, 0.6, 0.4, 0.3, 0.7, 0.8, 0.6, 0.5, 0.4, 0.1
4 Use the statistics function on your calculator to find the mean of the following distributions.
10A Where necessary, give your answers correct to 1 decimal place.
a b
Score Frequency Score Frequency
10 23 24 45
20 47 25 89
30 68 26 124
40 56 27 102
50 17 28 78
29 46
Chapter 10 Summary statistics 527
c Class Class centre Frequency
10–12 11 18
13–15 14 32
16–18 17 34
19–21 20 40
22–24 23 28
25–27 26 14
28–30 29 6

5 For each of the following sets of scores, find the median.


10A
a 25, 26, 26, 27, 27, 28, 30, 32, 35
b 4, 5, 8, 5, 8, 6, 7, 10, 4, 8, 4
c 3.2, 3.1, 3.0, 3.5, 3.2, 3.2, 3.2, 3.6
d 2, 3, 7, 4, 4, 8, 5, 7, 7, 6
e 121, 135, 111, 154, 147, 165, 101, 108

6 Copy and complete each of the following frequency tables and then use them to find the
median. 10A
a Cumulative
Score Frequency frequency
0 2
1 6
2 11
3 7
4 6
5 3

b Cumulative
Score Frequency frequency
54 2
55 5
56 14
57 11
58 6
59 1
60 1
528 M a t h s Q u e s t M a t h s B Ye a r 1 1 f o r Q u e e n s l a n d

c
Cumulative
Score Frequency frequency

66 8

67 10

68 12

69 14

70 7

71 5

72 4

7 Use the frequency table below to state the modal class.


10A
Class Class centre Frequency

30–33 31.5 12

34–37 35.5 26

38–41 39.5 34

42–45 43.5 45

46–49 47.5 52

50–53 51.5 23

8 Below are the number of goals scored by a netball team in ten matches in a tournament.
10A 25 26 19 24 28 67 21 22 28 18
a Calculate the mean.
b Calculate the median.
c Calculate the mode.
d Which of the above is the best summary statistic? Explain your answer.

9 Give an example of a statistical analysis where the best summary statistic is:
10A a the mean b the median c the mode.

10 Find the range of each of the following sets of scores.


10B a 28 24 26 24 25 29 22 27 25
b 118 2 56 45 72 43 69 84 159 0
c 1.9 0.7 0.5 0.8 1.1 1.5 1.4

11 For each of the data sets in question 10 calculate:


10B i the median iii the lower quartile
iii the upper quartile iv the interquartile range.
Chapter 10 Summary statistics 529
12 The cumulative frequency histogram and polygon at right
10B

Cumulative frequency
45
shows the number of goals scored by a soccer team in each 40
match over a season. 35
30
Use the graph to calculate: 25
20
a the median 15
b the lower and upper quartiles 10
5
c the interquartile range. 0
0 1 2 3 4
Number of goals scored
in a soccer match

13 The cumulative frequency histogram and polygon


below show the number of apples on each tree in 10B
an orchard.

300
Cumulative

250
frequency

200
150
100
50
0
.5 .5 .5 .5 .5 .5
34 44 54 64 74 84
Number of apples on a tree

Use the graph to find:


a the median
b the lower and upper quartiles
c the interquartile range.

14 The following scores were recorded:


2, 6, 9, 1, 3, 5, 6, 7, 8, 2. 10B,C
For these data find:
a the mean
b the median
c the mode
d the range
e the interquartile range
f the standard deviation.

15
Score Frequency Score Frequency 10B,C
30–39 3 60–69 15

40–49 6 70–79 18

50–59 12 80–89 10

For the above set of data find:


a the mean b the median group c the modal group
d the range e the standard deviation
530 M a t h s Q u e s t M a t h s B Ye a r 1 1 f o r Q u e e n s l a n d

16 For the box-and-whisker plot drawn below:


10D
0 5 10 15 20 25 30 35 40 45 50 55 60

a state the median


b calculate the range
c calculate the interquartile range.
17 The number of babies born each day
10D at a hospital over a year is tabulated
and the five number summary is given
below.
Lower extreme = 1
Upper quartile = 16
Lower quartile = 8
Upper extreme = 18
Median = 14
Show this information in a box-and-
whisker plot.
18 The number of hours of counselling
10E received by a group of 9 full-time
firefighters and 9 volunteer firefighters
after a serious bushfire is given below.

Full-time 2 4 3 5 2 4 6 1 3
Volunteer 8 10 11 11 12 13 13 14 15

a Construct a back-to-back stem plot to display the data.


b Comment on the distributions of the number of hours of counselling of the full-time
firefighters and the volunteers.
19 The IQ of 8 players in 3 different football teams were recorded and are shown below.
10F
Team A 120 105 140 116 98 105 130 102
Team B 110 104 120 109 106 95 102 100
Team C 121 115 145 130 120 114 116 123

Display the data in parallel boxplots.


Chapter 10 Summary statistics 531
Modelling and problem solving
1 A group of office workers and a group of sports instructors were asked to complete 5 minutes
of exercise as part of a study of heart rates. Following the exercise, participants rested for
2 minutes before their pulse rates were measured. The results are set out below in the stem plots.
Pulse rates for office
workers (beats/min)
Stem Leaf
7 6
8
9 5
10 6 7
11 0 2
12 0 1 2 4 6 7 9
13 0 0 4
Pulse rates for sports
instructors (beats/min)
Stem Leaf
6 2 4 8 8 9
7 2 2 3 5 7 9
8 2 8
9 6
10 8
Key: 12|4 = 124 beats/min
a Describe the shape of each
distribution.
b Calculate the median, the
interquartile range, the mode and the range for both.
c Represent each set of data using a boxplot.
d Calculate the mean and the standard deviation for both sets of data.
e Using the summary statistics that you have calculated for the office workers, comment on
their pulse rates. Do likewise for the sports instructors. Comment on any differences
between the two groups.

2 To compare two textbooks, a teacher recommends one book to one of his classes and the other
book to another class. At the end of the year the classes are each tested. The results are
detailed below:
Text A (25 students) Text B (28 students)
44 52 95 76 13 94 83 65 72 48 63 68 59 68
72 55 81 22 25 64 72 62 75 79 81 72 64 53
35 48 56 59 84 98 84 58 59 64 66 68 42 37
21 35 69 28 39 55 58 52 82 79 55
a Prepare a back-to-back stem-and-leaf plot of the data.
b Prepare a five number summary for each group. (Note that the groups are different sizes.)
c Prepare parallel boxplots of the data.
d Compare the performance of each of the classes.
e Which textbook do you think would be best? Why?
f What other things would you need to take into account before drawing final conclusions?
532 M a t h s Q u e s t M a t h s B Ye a r 1 1 f o r Q u e e n s l a n d

3 For marketing purposes, the administration of the Arts Centre needs to compare the ages of
people attending two different concerts: a Symphony Orchestra concert and a jazz concert.
Twenty people are randomly selected from each audience and their ages recorded as shown.

Event Ages of people attending the event


Symphony Orchestra concert 20, 23, 30, 35, 39, 42, 45, 45, 47, 48,
48, 49, 49, 50, 53, 54, 56, 58, 58, 60
Jazz concert 16, 18, 19, 19, 20, 23, 24, 27, 29, 30,
33, 34, 38, 39, 40, 42, 43, 45, 46, 62
a Display the data on a back-to-back stem plot.
b For each category calculate the following statistics:
i minX
ii Q1
iii median
iv Q3
v maxX
vi mean
vii interquartile range (IQR)
viii standard deviation.
c Use the stem plot together with some summary statistics to compare the distributions of
the ages of patrons attending the two concerts.

One month later, at the beginning of the opera season, twenty people are again selected
(this time from the opera audience) and their ages recorded as shown.

Event Ages of people attending the event


Opera 12, 18, 29, 30, 33, 35, 38, 39, 42, 46,
49, 50, 54, 56, 56, 57, 59, 63, 65, 68
The administration of the Arts Centre now wishes to compare all three distributions of the
eBook plus ages.
d Explain why it is not possible to use a back-to-back stem plot for this task.
Digital doc: e Calculate the eight summary statistics for the ages of the opera-goers (as in part b above).
Test Yourself
Chapter 10 f Display the data for the three events using parallel boxplots.
g Use the boxplots and some summary statistics to compare the three distributions.
Introduction
to probability
11
syllabus reference
Topic:
• Applied statistical
analysis

In this chapter
11A Informal description of
chance
11B Single event probability
11C Relative frequency
11D Modelling probability
11E Long-run proportion
534 M a t h s Q u e s t M a t h s B Ye a r 1 1 f o r Q u e e n s l a n d

Introduction
A major outdoor rock concert is to be staged in
your local area. The date that has been suggested
for the concert is the last weekend in February.
Your job is to report back to the organisers on the
suitability of this date.
You have been told that the organisers will
lose a lot of money if the concert is rained out.
You have also been told that if the temperature
rises to above 35°C, there will be a strain on
medical services as many in the crowd will col-
lapse from heat exhaustion.
You must comment on the likelihood of either
of these events occurring. If either is likely, you
will need to suggest a more suitable date for the
concert.

Informal description of chance


You have booked a ski holiday to Thredbo for the middle of July. What is the chance
there will be enough snow on the ground for you to ski? There is no exact answer to
this question, but by looking at the amount of snow in Thredbo during July over past
years, we know that there is a very good chance there will be enough snow to ski again
this year.
You can then say that it is very likely you will be able to ski during July at
Thredbo. Terms such as ‘very likely’, ‘almost certain’, ‘unlikely’ and ‘fifty-fifty’ are
used in everyday language to describe the chance of an event occurring. For the
purposes of probability, an event is the outcome of an experiment a statistician is
interested in. They can describe an outcome as a possible result to the probability
experiment.
Imagine you are playing a board game and it is your turn to roll the die. To win the
game you need to roll a number less than 7. If you roll one die you must get a number
less than 7. The chance of this event occurring is described as certain.

When an event is certain to occur, the probability of that event occurring is 1.

Now consider an impossible situation. certain 1


almost certain
In a board game you have one last throw of the die. To win you
must roll a 7. We know that this cannot be done. We would say that probable
this is impossible.
fifty-fifty 1
When an event is impossible, the probability of the event is 0. 2

The chance of any event occurring will often be somewhere unlikely


between being certain and impossible. We use a variety of
very unlikely
terms to describe where the chance lies in this range as shown in impossible 0
the figure at right.
We use these terms based on our general knowledge of the world, the total possible
outcomes and how often an event occurs.
Chapter 11 Introduction to probability 535
WORKED Example 1
Describe the chance of each of the following events occurring.
a Tossing a coin and it landing Heads.
b Rolling a 6 with one die.
c Winning the lottery.
d Selecting a spot card (ace, 2, 3 . . ., 10) from a standard deck.
THINK WRITE
a There is an equal chance of the coin a The chance of tossing a head is fifty-fifty.
landing Heads and Tails.
b There is only one chance in six of b It is unlikely that you will roll a 6.
rolling a 6.
c There is only a very small chance of c It is very unlikely that you will win the
winning the lottery. lottery.
d There are more spot cards than picture d It is probable that you will select a spot card.
cards in a deck.

You will need to use these terms to describe events that are more likely to occur than
others.

WORKED Example 2
Mrs Graham is expecting her baby to be born between July 20 and 26. Is it more likely
that her baby will be born on a weekday or a weekend?
THINK WRITE
There are 5 chances that the baby will be It is more likely that Mrs Graham’s baby will
born on a weekday and 2 chances that it be born on a weekday.
will be born on a weekend.

The term frequency refers to how often an event occurs. We use our knowledge about
possible outcomes to order outcomes from the most frequent to the least frequent.

WORKED Example 3
A card is chosen from a standard deck. List the following outcomes in order from least
likely to most likely.
selecting a picture card selecting an ace
selecting a diamond selecting a black card

8 7 6 5 4 3 2 8 7 6 5 4 3 2 8 7 6 5 4 3 2 8 7 6 5 4 3 2
09 09 09 09
J1 J1 J1 J1
Q
Q
Q

Q
K
K
K

K
A
A
A

2 2 2 2

Continued over page


536 M a t h s Q u e s t M a t h s B Ye a r 1 1 f o r Q u e e n s l a n d

THINK WRITE
1 There are 12 picture cards in the deck.
2 There are 4 aces in the deck.
3 There are 13 diamonds in the deck.
4 There are 26 black cards in the deck. The order of events in ascending order of likeli-
hood:
selecting an ace
selecting a picture card
selecting a diamond
selecting a black card.

In the above examples, we have been able to calculate which event is more likely by
counting the number of ways an event may occur. This is not always possible. In some
cases we need to use general knowledge to describe the chance of an event occurring.
Consider the following probability problems.

Problem A
‘The letters of the alphabet are written on cards and one card is selected at random.
Which letter has the greater chance of being chosen, E or Q?’
Each letter has an equal chance of being chosen because there is one chance that E
will be chosen and one chance that Q will be chosen.

Problem B
‘Stacey sticks a pin into a page of a book and then writes down the letter nearest to the
pin. Which letter has the greater chance of being chosen, E or Q?’

Problem B is more difficult to answer because each letter does not occur with equal
frequency. However, we know from our experience with the English language that Q
will occur much less often than most other letters. We can therefore say that E will
occur more often than Q.
This is an example of using knowledge of the world to make predictions about which
event is more likely to occur. In this way, we make predictions about everyday things
such as the weather and which football team will win on the weekend.

WORKED Example 4
During the 2006 NRL season, the Brisbane Broncos won 9 of their first 12 games. In
Round 13 they played South Sydney who had won 0 of their first 12 games. Which team
would be more likely to win?
THINK WRITE
The Brisbane Broncos have a better record The Brisbane Broncos would be more likely to
than South Sydney. win, based on their previous results.

(Footynote: South Sydney won the game 34–14. Brisbane was more likely to win the game but nothing
in football is certain.)
Chapter 11 Introduction to probability 537
This is one example of past results being used to predict future happenings. There
are many other such examples.

WORKED Example 5
Weather records show that it has rained on Christmas Day 12 times in the last 80 years.
Describe the chance of it raining on Christmas Day this year.
THINK WRITE
It has rained only 12 times on the last 80 It is unlikely that it will rain on Christmas Day
Christmas Days. This is much less than half this year.
of all Christmas Days.

remember
1. The chance of an event occurring ranges from being certain to impossible.
2. (a) An event that is certain has a probability of 1.
(b) An event that is impossible has a probability of 0.
3. There are many terms that we use to describe the chance of an event occurring,
such as improbable, unlikely, fifty-fifty, likely and probable.
4. Sometimes we can describe the chance of an event occurring by counting the
possible outcomes, while other times we need to rely on our general knowledge
to make such a description.

Informal description of
11A chance
WORKED 1 Describe the chance of each of the following events occurring, using an appropriate
Example
1
probability term.
a Selecting a ball with a double-digit number from a bag with balls numbered 1 to
40
b Selecting a female student from a class with 23 boys and 7 girls
eBook plus
c Selecting a green marble from a barrel with 40 blue marbles and 30 red marbles
Digital docs: d Choosing an odd number from the numbers 1 to 100.
SkillSHEET 11.1
Understanding 2 For each of the events below, describe the chance of it occurring as impossible,
‘chance’ words unlikely, even chance (fifty-fifty), probable or certain.
SkillSHEET 11.2 a Rolling a die and getting a negative number
Understanding a
deck of cards b Rolling a die and getting a positive number
c Rolling a die and getting an even number
d Selecting a card from a standard deck and getting a red card
e Selecting a card from a standard deck and getting a spot (numbered) card
f Selecting a card from a standard deck and getting an ace
g Reaching into a moneybox and selecting a 30c piece
h Selecting a blue marble from a bag containing 3 red, 3 green and 6 blue marbles.
538 M a t h s Q u e s t M a t h s B Ye a r 1 1 f o r Q u e e n s l a n d

3 Give an example of an event which has a probability that could be described as:
a certain b probable c even chance
d unlikely e impossible.
WORKED 4 Is it more likely that a person’s birthday will occur during a school term or during the
Example
2
school holidays?
5 For each event on the left, state whether it is more likely, less likely or equally likely
to occur than the event on the right.
a Fine weather Christmas Day Wet weather Christmas Day
b A coin landing Heads A coin landing Tails
c Rolling a total of 3 with two dice Rolling a total of 7 with two dice
d Winning a raffle made up of 50 tickets Winning a raffle made up of 200 tickets
e Winning a prize in the Lotto draw Not winning a prize in the Lotto draw.
WORKED 6 A die is thrown and the number rolled is noted. List the following events in order
Example
3
from least likely to most likely.
Rolling an even number
Rolling a number less than 3
Rolling a 6
Rolling a number greater than 2
7 Write the following events in order from least likely to most likely.
Winning a raffle with 5 tickets out of 30
Rolling a die and getting a number less than 3
Drawing a green marble from a bag containing 4 red, 5 green and 7 blue marbles
Selecting a court card (king, queen, jack) from a standard deck
Tossing a coin and having it land Heads
WORKED 8 Before meeting in the cricket World Cup in 1999, Australia had beaten Zimbabwe in
Example
4
10 of the last 11 matches. Who would be more likely to win on this occasion?
9 Which of the following two runners would be expected to win the final of the 100 metres
at the Olympic Games?
Carl Bailey — best time 9.92 s and won his semi-final
Ben Christie — best time 10.06 s and 3rd in his semi-final
Give an explanation for your answer.

10 multiple choice
A stack of 26 cards has the letters of the alphabet written on them. Vesna draws a card
from that stack. The probability of selecting a card that has a vowel written on it could
best be described as:
A impossible B unlikely C even chance D probable E almost certain

11 multiple choice
Which of the following events is the most likely to occur?
A Selecting the first number drawn from a barrel containing 20 numbered marbles
B Selecting a diamond from a standard deck of cards
C Winning a raffle with one ticket out of 150 000
D Drawing the inside lane in the Olympic 100-metre final with eight runners
E Correctly selecting the first drawn ball in Gold Lotto
Chapter 11 Introduction to probability 539
12 multiple choice
The ski season opens on the first weekend of June. At a particular ski resort there has
been sufficient snow for skiing on that weekend on 32 of the last 40 years. Which of
the following statements is true?
A Sufficient snow on the opening day of the ski season is impossible.
B It is unlikely there will be sufficient snow at the opening of the ski season this year.
C There is a fifty-fifty chance there will be sufficient snow at the opening of the ski
season this year.
D It is probable there will be sufficient snow at the opening of the ski season this year.
E It is certain there will be sufficient snow at the opening of the ski season this year.
WORKED 13 On a production line, light globes are tested to see how long they will last. After
Example
5
testing 1000 light globes it is found that 960 will burn for more than 1500 hours.
Wendy purchases a light globe. Describe the chance of the light globe burning for
more than 1500 hours.
14 Of 12 000 new cars sold last year, 1500 had a major mechanical problem during the
first year. Edwin purchases a new car. Describe the chance of Edwin having a major
mechanical problem in the first year.
15 During an election campaign, 2000 people were asked for their voting preferences.
One thousand said that they would vote for the government. If one person is chosen at
random, describe the chance that they would vote for the government.

What will the weather be?


As you work through this chapter your task is to analyse the suitability of the last
weekend in February to stage a major outdoor rock concert. Based only on your
own knowledge of the weather pattern in your local area, describe in words:
• the chance of the concert being rained out
• the chance that the temperature will rise to 35°C on this particular weekend.

Single event probability


So far in this chapter we have discussed the chances of certain events occurring. In
doing so, we used informal terms such as probable and unlikely. Although these terms
give us an idea of whether something is likely to occur or not, they do not tell us how
likely it is. To do this, an accurate way of stating the probability is needed.
We stated earlier that the chance of any event occurring was somewhere between
impossible and certain. We also said that:
• if an event is impossible the probability was 0
• if an event is certain the probability was 1.
It therefore follows that the probability of any event must lie between 0 and 1 inclusive.
A probability of an event is a number which describes the chance of that event occur-
ring. All probabilities are calculated as fractions but can be written as fractions,
decimals or percentages. Probability is calculated using the formula:
number of favourable outcomes
P ( event ) = ----------------------------------------------------------------------------
total number of outcomes
540 M a t h s Q u e s t M a t h s B Ye a r 1 1 f o r Q u e e n s l a n d

The total number of favourable outcomes is the number of different ways the event
can occur, while the total number of outcomes is the number of elements in the sample
space. The sample space, S, is a list of all the possible outcomes enclosed in curled
brackets.

WORKED Example 6
Zoran is rolling a die. To win a game, he must roll a number greater than 2. List the
sample space and state the number of favourable outcomes.
THINK WRITE
1 There are 6 possible outcomes. S = {1, 2, 3, 4, 5, 6}
2 The favourable outcomes are to roll a 3, There are 4 favourable outcomes.
4, 5 or 6.

Consider the case of tossing a coin. If we are calculating the probability that it will
land Heads, there is 1 favourable outcome out of a total of 2 possible outcomes. Hence
we can then write P(Heads) = 1--- . This method is used to calculate the probability of any
2
single event.

WORKED Example 7
Andrea selects a card from a standard deck. Find the probability that she selects an ace.
THINK WRITE
1 There are 52 cards in the deck (total
number of outcomes).
2 There are 4 aces (number of favourable
outcomes).
4
3 Write the probability. P(ace) = ------
52

4 1
In the above example the fraction ------ could be simplified to ------ .
52 13

WORKED Example 8
In a barrel there are 6 red marbles, 2 green marbles and 4 yellow marbles. One marble is
drawn at random from the barrel. Calculate the probability that the marble drawn is red.
THINK WRITE
1 There are 12 marbles in the barrel (total
number of outcomes).
2 There are 6 red marbles in the barrel
(number of favourable outcomes).
6
3 Write the probability. P(red) = ------
12

Some questions have more than one favourable outcome. In these cases, we need to
add together each of these outcomes to calculate the number of which are favourable.
Chapter 11 Introduction to probability 541
WORKED Example 9
On a bookshelf there are 6 history books, 7 novels, 2 dictionaries and 5 sporting books. If
Theo selects one at random, what is the probability that the book chosen is not a novel?
THINK WRITE
1 There are 20 books on the shelf (total
number of outcomes).
2 Seven of these books are novels,
meaning that 13 of them are not novels
(number of favourable outcomes).
13
3 Write the probability. P(not a novel) = ------
20

Some questions do not require us to calculate the entire sample space, only the
sample space for a small part of the experiment. We can often do this in number ques-
tions such as Worked example 10. We may need to concern ourselves with only one
digit rather than the whole number, meaning we do not need to write out the whole
sample space.

WORKED Example 10
The digits 1, 3, 4 and 5 are written on cards. These cards are then used to form a four-digit
number. Calculate the probability that the number formed is:
a even
b greater than 3000.
THINK WRITE
a 1 If the number is even the last digit a
must be even.
2 There are four cards that could go in
the final place (total number of
outcomes).
3 Only one of these cards (the 4) is
even (number of favourable
outcomes).
1
4 Write the probability. P(even) = ---
4
b 1 If the number is greater than 3000, b
then the first digit must be a 3 or
greater.

2 There are four cards that could go in


the first place.
3 Three of these cards are a 3 or
greater.
3
4 Write the probability. P(greater than 3000) = ---
4
542 M a t h s Q u e s t M a t h s B Ye a r 1 1 f o r Q u e e n s l a n d

remember
1. The sample space is the list of all possible outcomes in a probability
experiment.
2. The event space is a list of all favourable outcomes to a probability experiment.
3. The probability of an event is calculated using the formula:
number of favourable outcomes
P(event) = ----------------------------------------------------------------------------
total number of outcomes

11B Single event probability


WORKED 1 A coin is tossed at the start of a cricket match. Manuel calls Heads. List the sample
Example
6
space and the number of favourable outcomes.

2 For each of the following


probability experiments, state the
number of favourable outcomes.
a Rolling a die and needing a 6
b Rolling two dice and needing
a total greater than 9
c Choosing a letter of the
alphabet and it being a vowel
d The chance a baby will be
born on the weekend
e The chance that a person’s
birthday will fall in summer

3 For each of the following


probability experiments, state the
number of favourable outcomes
and the total number of outcomes.
a Choosing a red card from a
standard deck
b Selecting the winner of a 15-
horse race
c Selecting the first ball drawn
in a Lotto draw (The balls are
numbered 1 to 44.)
d Winning a raffle with 5 tickets
out of 1500
e Selecting a yellow ball from a
bag containing 3 yellow, 4 red and 4 blue balls

WORKED
Example
4 A coin is tossed. Find the probability that the coin will show Tails.
7
Chapter 11 Introduction to probability 543
5 A regular die is cast. Calculate
the probability that the uppermost
face is:
a 6
b 1
c an even number
d a prime number
e less than 5
f at least 5.

6 A barrel contains marbles with


the numbers 1 to 45 written on
them. One marble is drawn at
random from the bag. Find the
probability that the marble
drawn is:
a 23 b 7 c an even number
d an odd number e a multiple of 5 f a multiple of 3
g a number less than 20 h a number greater than 35 i a square number.
7 Many probability questions are asked about decks of cards. You should know the
cards making up a standard deck.

8 7 6 5 4 3 2 8 7 6 5 4 3 2 8 7 6 5 4 3 2 8 7 6 5 4 3 2
09 09 09 09
J1 J1 J1 J1
Q
Q
Q

Q
K
K
K

K
A
A
A

A
2 2 2 2

A card is chosen from a standard deck. Find the probability that the card chosen is:
a the ace of diamonds b a king c a club
d red e a picture card f a court card.
WORKED 8 A bag contains 12 counters: 7 are orange, 4 are red and 1 is yellow. One counter is
Example
8
selected at random from the bag. Find the probability that the counter chosen is:
a yellow b red c orange.
9 The digits 2, 3, 5 and 9 are written on cards. One card is then chosen at random. Find
the probability that the card chosen is:
a the number 2 b the number 5 c even
d odd e divisible by 3 f a prime number.
WORKED 10 In a bag of fruit there are 4 apples, 6 oranges and 2 pears. Larry chooses a piece of
Example
9
fruit from the bag at random but he does not like pears. Find the probability that Larry
does not select a pear.
WORKED 11 The digits 2, 3, 5 and 9 are written on cards. They are then used to form a four-digit
Example
10
number. Find the probability that the number formed is:
a even b odd c divisible by 5
d less than 3000 e greater than 5000.

12 multiple choice
A die is rolled. The probability that the number on the uppermost face is less than 4 is:
1 1 1 2 5
A --- B --- C --- D --- E ---
6 3 2 3 6
544 M a t h s Q u e s t M a t h s B Ye a r 1 1 f o r Q u e e n s l a n d

13 multiple choice
When a die is rolled, which of the following outcomes does not have a probability
equal to 1--- ?
2
A The number on the uppermost face is greater than 3.
B The number on the uppermost face is even.
C The number on the uppermost face is odd.
D The number on the uppermost face is at least a 3.
E The number on the uppermost face is a prime number.

14 multiple choice
A card is chosen from a standard deck. The probability that the card chosen is a court
card is:
1 1 3 4 1
A ------ B ------ C ------ D ------ E ---
52 13 13 13 4

15 multiple choice
When a card is chosen from a standard deck, which of the following events is most
likely to occur?
A choosing a seven B choosing a club
C choosing a picture card D choosing a black card
E choosing a spot card

16 One thousand tickets are sold in a raffle. Craig buys five tickets.
a One ticket is drawn at random. The holder of that ticket wins first prize. Find the
probability of Craig winning first prize.
b After the first prize has been drawn, a second prize is drawn. If Craig won first
prize, what is the probability that he now also wins second prize?

17 A lottery has 160 000 tickets. Janice buys one ticket. There are 3384 cash prizes in the
lottery.
a What is the probability of Janice winning a cash prize?
b If there are 6768 consolation prizes of a free ticket for being one number off a
cash prize, what is the probability that Janice wins a consolation prize?
c What is the probability that Janice wins either a cash prize or a consolation prize?

18 A number is formed using all five of the digits 1, 3, 5, 7 and 8. What is the probability
that the number formed:
a begins with the digit 3? b is even? c is odd?
d is divisible by 5? e is greater than 30 000? f is less than 20 000?

19 Write down an example of an event which has a probability of:


1 1 2
a --- b --- c --- .
2 4 5

20 A three-digit number is formed using the digits 2, 4 and 7.


a Explain why it is more likely that an even number will be formed than an odd
number.
b Which is more likely to be formed: a number less than 400 or a number greater
than 400?
Chapter 11 Introduction to probability 545
Comparing theoretical probabilities
with experimental results
In this activity, we compare the probability of certain events to practical results.
You may be able to do a simulation of these activities on a spreadsheet.
1 Tossing a coin
a If we toss a coin P(Heads) = 1--- . Therefore, if you toss a coin, how many
2
Heads would you expect in:
i 4 tosses? ii 10 tosses? iii 50 tosses? iv 100 tosses?
Now toss a coin 100 times and record the number of Heads after:
i 4 tosses? ii 10 tosses? iii 50 tosses? iv 100 tosses.
Combine your results with the rest of the class. How close to 50% is the
total number of Heads thrown by the class?
2 Rolling a die
When you roll a die, what is the probability of rolling a 1? The probability for
each number on the die is the same.
Roll a die 120 times and record each result in the table below.
Number Occurrences Percentage of throws
1
2
3
4
5
6

How close are the results to the results that were expected?
3 Rolling two dice
Roll two dice and record the total on the faces of the two dice. Repeat this
100 times and complete the table below.
Number Occurrences Percentage of throws
2
3
4
5
6
7
8
9
10
11
12

Do you notice anything different about the results of this activity, compared to
the others?
546 M a t h s Q u e s t M a t h s B Ye a r 1 1 f o r Q u e e n s l a n d

Experimental or theoretical?
We have just been calculating the probability of certain events occurring. Now
consider the following statements made about the weather conditions for the
proposed rock concert.
• The weather on the day of the concert will either be wet or dry. Therefore, the
probability of rain on the day of the concert is 1--- . There is a 50% chance that the
2
organisers will lose their money.
• The coldest possible temperature in your area is 10 °C and the hottest possible
temperature is 40 °C. Therefore the probability that the temperature will rise to
5
35 °C is -----
-.
30
Are these statements correct? Why or why not?

Relative frequency
You are planning to go skiing on the first weekend in July. The trip is costing you a lot
of money and you don’t want your money wasted on a weekend without snow. So what
is the chance of it snowing on that weekend? We can use past records only to estimate
that chance.
If we know that it has snowed on the first weekend of July for 54 of the last 60 years,
we could say that the chance of snow this year is very high. To measure that chance, we
calculate the relative frequency of snow on that weekend. We do this by dividing the
number of times it has snowed by the number of years we have examined. In this case,
we can say the relative frequency of snow on the first weekend in July is 54 ÷ 60 = 0.9.
The relative frequency is usually expressed as a decimal or percentage and is calculated
using the formula:
number of times an event has occurred
Relative frequency = ---------------------------------------------------------------------------------------------
number of trials
In this formula, a trial is the number of times the probability experiment has been
conducted.
The formula for relative frequency is similar to that for probability.
The term relative frequency refers to actual data obtained, but the term probability
generally refers to theoretical data unless experimental probability is specifically stated.

WORKED Example 11
The weather has been fine on Christmas Day in Brisbane for 32 of the past 40 Christmas
Days. Calculate the relative frequency of fine weather on Christmas Day.
THINK WRITE
1 Write the formula. Relative frequency =
number of times an event has occurred
---------------------------------------------------------------------------------------------
number of trials
2 Substitute the number of fine Christmas Days Relative frequency = 32 ------
40
(32) and the number of trials (40).
3 Calculate the relative frequency as a decimal. = 0.8
Chapter 11 Introduction to probability 547
The relative frequency is used to assess the quality of products. This is done by
finding the relative frequency of defective products.

WORKED Example 12
A tyre company tests its tyres and finds that 144 out of a batch of 150 tyres will withstand
20 000 km of normal wear. Find the relative frequency of tyres that will last 20 000 km.
Give the answer as a percentage.
THINK WRITE
1 Write the formula. Relative frequency =
number of times an event has occurred
---------------------------------------------------------------------------------------------
number of trials
2 Substitute 144 (the number of times the event Relative frequency = 144 ---------
150
occurred) and 150 (number of trials).
3 Calculate the relative frequency. = 0.96
4 Convert the relative frequency to a percentage. = 96%

Relative frequencies can be used to solve many practical problems.

WORKED Example 13
A batch of 200 light globes was tested. The batch is considered unsatisfactory if more than
15% of globes burn for less than 1000 hours. The results of the test are in the table below.

Number of hours Number of globes


less than 500 4
500–750 12
750–1000 15
1000–1250 102
1250–1500 32
more than 1500 35

Determine if the batch is unsatisfactory.


THINK WRITE
1 Count the number of light globes that 31 light globes burn for less than 1000 hours.
burn for less than 1000 hours.
2 Write the formula. Relative frequency =
number of times an event has occurred
---------------------------------------------------------------------------------------------
number of trials
Continued over page
548 M a t h s Q u e s t M a t h s B Ye a r 1 1 f o r Q u e e n s l a n d

THINK WRITE
31
3 Substitute 31 (number of times the Relative frequency = ---------
200
event occurs) and 200 (number of
trials).
4 Calculate the relative frequency. = 0.155
5 Convert the relative frequency to a = 15.5%
percentage.
6 Make a conclusion about the quality of More than 15% of the light globes burn for
the batch of light globes. less than 1000 hours and so the batch is
unsatisfactory.

remember
1. The relative frequency is used to estimate the probability of an event.
2. The relative frequency, usually expressed as a decimal or percentage, is a figure
that represents how often an event has occurred.
3. The relative frequency is calculated using the formula:
number of times an event has occurred
Relative frequency = --------------------------------------------------------------------------------------------- .
number of trials

11C Relative frequency


WORKED 1 At the opening of the ski season, there has been sufficient snow for skiing for 37 out
Example
11
of the past 50 years. Calculate the relative frequency of sufficient snow at the begin-
ning of the ski season.

2 A biased coin has been tossed 100 times with the result of 79 Heads. Calculate the
eBook plus
relative frequency of the coin landing Heads.
Digital doc:
SkillSHEET 11.3 3 Of eight Maths tests done by a class during a year, Peter has topped the class three
Converting a fraction or a times. Calculate the relative frequency of Peter topping the class.
decimal to a percentage

4 Farmer Jones has planted a wheat crop. For the wheat crop to be successful, Farmer
Jones needs 500 mm of rain to fall over the spring months. Past weather records show
that this has occurred on 27 of the past 60 years. Find the relative frequency of:
a sufficient rainfall
b insufficient rainfall.

WORKED 5 Of 300 cars coming off an assembly line, 12 are found to have defective brakes.
Example
12
Calculate the relative frequency of a car having defective brakes. Give the answer as a
percentage.
Chapter 11 Introduction to probability 549
6 A survey of 25 000 new car buyers found that 750 had a major mechanical problem in
the first year of operation. Calculate the relative frequency of:
a having mechanical problems in the first year
b not having mechanical problems in the first year.

7 On a production line, light globes are tested to see how long they will last. After testing
1000 light globes, it is found that 960 will burn for more than 1500 hours. Wendy
purchases a light globe. What is the relative frequency that the light globe will:
a burn for more than 1500 hours?
b burn for less than 1500 hours?

8 multiple choice
A study of cricket players found that of 150 players, 36 batted left handed. What is
the relative frequency of left-handed batsmen?
A 0.24 B 0.36 C 0.54 D 0.64 E 0.76

9 multiple choice
Five surveys were conducted and the following results were obtained. Which result
has the highest relative frequency?
A Of 1500 P-plate drivers, 75 had been involved in an accident.
B Of 1200 patients examined by a doctor, 48 had to be hospitalised.
C Of 20 000 people at a football match, 950 were attending their first match.
D Of 50 trucks inspected, 2 were found to be unroadworthy.
E Of 300 drivers breath tested, 170 were found to be over the legal limit.
10 During an election campaign 2000 people were asked for their voting preferences.
One thousand and fifty said that they would vote for the government, 875 said they
would vote for the opposition and the remainder were undecided. What is the relative
frequency of:
a government voters?
b opposition voters?
c undecided voters?

11 Research over the past 25 years shows that each November there is an average of two
wet days on Sunnybank Island. Travelaround Tours offer one-day tours to Sunnybank
Island at a cost of $150 each, with a money back guarantee against rain.
a What is the relative frequency of wet November days as a percentage?
b If Travelaround Tours take 1200 bookings for tours in November, how many
refunds would they expect to give?

12 An average of 200 robberies takes place each year in the town of Amiak. There are
10 000 homes in this town.
a What is the relative frequency of robberies in Amiak?
b Each robbery results in an average insurance claim of $20 000. What would be the
minimum premium per home the insurance company would need to charge to
cover these claims?
550 M a t h s Q u e s t M a t h s B Ye a r 1 1 f o r Q u e e n s l a n d

WORKED 13 A car maker recorded the first time that its cars came in for mechanical repairs. The
Example
results are in the table below.
13

Time taken Number of cars

0–<3 months 5

3–<6 months 12

6–<12 months 37

1–<2 years 49

2–<3 years 62

3 years or more 35

The assembly line will need to be upgraded if the relative frequency of cars needing
mechanical repair in the first year is greater than 25%. Determine if this will be
necessary.

14 For the table in question 13 determine, as a percentage, the relative frequency of:
a a car needing mechanical repair in the first 3 months
b a car needing mechanical repair in the first 2 years
c a car not needing mechanical repair in the first 3 years.

15 A manufacturer of shock absorbers measures the distance that its shock absorbers can
travel before they must be replaced. The results are in the table below.

Number of shock
Number of kilometres absorbers

0–<20 000 1

20 000–<40 000 2

40 000–<60 000 46

60 000–<80 000 61

80 000–<100 000 90

What is the maximum distance the manufacturer will guarantee so that the relative
frequency of the shock absorbers lasting is greater than 0.985?

16 A soccer team plays 40 matches over a season and the results (wins, losses and draws)
are shown below.
W W W D L L L D W L W D L D
eBook plus W W L L L D W W D L L W W
W L D L D D L W W W D D L
Digital doc: a Put this information into a table showing the number of wins, losses and draws.
WorkSHEET 11.1
b Calculate the relative frequency of each result over a season.
Chapter 11 Introduction to probability 551
Researching relative frequencies
Choose one of the topics below (or another of your
choice) and calculate the relative frequency of the
event. Most of the information needed can be found
from books or the internet.
1 Examine weather records and find out the
relative frequency of rain on New Year’s Eve in
Brisbane.
2 Choose your favourite sporting team. Find the
relative frequency of them winning over the past
three seasons.
3 Find the relative frequency of the stock market
rising for three consecutive days.
4 Check the NRL or AFL competitions and find
the relative frequencies of win, loss and draw
for each team.

Applying relative frequency


You are now in a position to look at the actual likelihood of either of the given
conditions applying on the proposed date of the rock concert.
1 Use the internet or other sources to find the weather conditions in your area on
the last weekend of February for the past 20 years.
2 What is the relative frequency of rain on this weekend?
3 What is the relative frequency of the temperature reaching 35°C on this weekend?
4 Based on these results would you recommend the concert proceed on the
weekend suggested? Explain why or why not.
5 Can you find the weekend during the year where these two conditions are least
likely to occur?

Modelling probability
We have now seen examples where probability can be calculated using our knowledge
of possible outcomes and can be estimated using past results. We are able to compare
the theoretical probability and actual results using a simulation.
While it may be possible to count the number of outcomes in the event space and the
number of successful outcomes, often it is preferable to calculate probabilities
experimentally, and to compare this result with the theoretical result. For example, we
could toss a coin 100 times and, perhaps, find that 56 of the tosses resulted in a Head.
56 1
Therefore the experimental probability = --------
- , although the theoretical probability = --- .
100 2
It can be shown that the more times the experiment is repeated the closer the
552 M a t h s Q u e s t M a t h s B Ye a r 1 1 f o r Q u e e n s l a n d

experimental result should be to the theoretical one. Therefore, if a coin is tossed 1000
times instead of a 100 times, the result should be closer to 1--- . As with most things in
2
probability, there are no guarantees, but generally the larger the number of repetitions
of an experiment (or of a trial), the closer the experimental probability should be to the
theoretical one.
It is often difficult to complete an actual probability experiment. For example, it may
take too long to roll a die 1000 times and record the results. To make this quicker and
easier, we may perform a simulation.
A simulation is when random numbers are used to obtain results that will follow the
same pattern as a real life experiment. For example, a computer or graphics calculator
could randomly select the number 1 or 2 to simulate the toss of a coin, where 1 represents
a Head and 2 represents a Tail.
Consider the following result when rolling a die 12 times:
5, 3, 1, 4, 4, 3, 6, 1, 6, 4, 1, 3.
This sequence of numbers can be treated as a set of random numbers, whose possible
values are 1, 2, 3, 4, 5, 6.
A deck of playing cards could be numbered from 1 to 52; by drawing a card, then
replacing it in the deck, shuffling the deck and drawing another card, a sequence of
random numbers between 1 and 52 would be generated. There are several other ways
of generating random numbers; can you think of any?
There are three general rules that a set of random numbers must follow.
Rule 1. The set must be within a defined range (for example, whole numbers between
1 and 10, or decimals between 0 and 1). The range need not have a definite starting and
finishing number, but in most cases it does. For example, when simulating rolling a die
we must set a lower limit of 1 and an upper limit of 6.
Rule 2. All numbers within the defined range must be possible outcomes (for example
using a die but not counting 4s is not a proper sequence of random numbers between 1
and 6).
Rule 3. No random number in a sequence can depend on any of the previous (or
future) numbers in the sequence. (This is difficult to prove but is assumed with dice,
spinners and so on.)
These rules can make it extremely difficult to obtain a proper set of random numbers
in practice. Furthermore, to generate many random numbers, say 1000, by rolling dice
could take a long time! Fortunately, computers can be used to generate random
numbers for us. (Technically, they are known as pseudo-random numbers, because
rules 2 and 3 above cannot be rigorously met.)

Graphics Calculator tip! Random number generation


There are a number of ways of generating a random number using a graphics calcu-
lator. One method has already been shown in Chapter 9 on page 432. To generate a par-
ticular number of random numbers and store them in a list, follow the steps shown
below. We will consider generating 100 random numbers ranging from 1 to 8 as an
example.
For the Casio fx-9860G AU
The random number generator on your calculator produces a number between 0 and 1
(not including 0 or 1). If you want numbers with a different range, you need to use the
Chapter 11 Introduction to probability 553
integer function to use just the integer component and multiplication and addition to
produce numbers in a suitable range. It is normal to multiply by the range and then add.
To produce a number from 1 to 8, you will need to multiply the random number by 8
(giving a number between 0 and 8) and then add 1 (giving a number between 1 and 9)
so that when you use just the integer value it will be one of {1, 2, 3, 4, 5, 6, 7, 8}. We
will also use the sequence function so that the numbers can be stored in a list.
1. Press MENU and then select RUN-MAT.
Press OPTN and then F1 (LIST) followed by
F5 (Seq).

2. To obtain the integer function, press OPTN and


then F6 ( ) for more options, followed by
F4 (NUM) and F2 (Int).

3. To enter the rule (8 × Ran# + 1), press ( and


then enter 8 followed by × . Press EXIT
followed by F3 (PROB) and F4 (Ran#) to
generate a random decimal number. Press +
and then 1 , then press ) to close the set of
brackets and , to enter a comma.
4. Now enter the variable (N), the start value of N
(1), the end value of N (100) and the increment
(1), each separated by a comma. Press ) to
close the set of brackets. (To enter the variable N,
press ALPHA [N].)
To store this set of numbers, press → and
then OPTN F1 (LIST) followed by F1 (List)
and then 1 . Press EXE to store the numbers
in List 1.
5. Press MENU and then select STAT. The numbers
generated will be displayed in List 1. Scroll down
to view the numbers. Note that your calculator
will display a different set of numbers as the
values will differ each time this sequence is
repeated.
For the TI-Nspire CAS
1. Open a new Lists & Spreadsheet document
(press /N and select 3: Add Lists &
Spreadsheet). Place the cursor in the formula bar
and then press =. Type in RANDINT( using the
letter keys or press k to access the catalog and
then press 1 to select the function menu. Scroll
down to randInt(. (The required function can be
found quicker by first pressing R.) Note that the
type of entries to follow are shown at the bottom
of the display.
554 M a t h s Q u e s t M a t h s B Ye a r 1 1 f o r Q u e e n s l a n d

2. Press · to insert the function into the formula


bar. Enter the lower bound (1), the upper bound
(8) and the number of trials (100), each separated
by a comma. Press ) to close the set of brackets. <TI04.01>

3. Press · to generate the numbers in the first


column. Scroll down to view the numbers. Note
that your calculator will display a different set of
numbers as the values will differ each time this <TI04.01>
list is repeated. (Press /R to generate a new set
of numbers.)

WORKED Example 14
Use a sequence of 20 random numbers to simulate rolling a die 20 times. Record your
results in a frequency table.
THINK WRITE/DISPLAY
1 Generate 20 random numbers in the range 1 to 6.
For the Casio fx-9860G AU
1. Use the sequence and integer functions as shown
in the previous Graphics calculator tip. Enter the
rule Int (6 × Ran# + 1) with a start value of 1, an <Casio04.01>
end value of 20 and an increment of 1 for the
variable N. Store the numbers in List 1.
2. Press MENU and then select STAT to display
the numbers.

<Casio04.01>

3. To make it easier to record the results in a


frequency table, we can sort the list of numbers
into ascending order. Press F6 ( ) for more
options and then F1 (TOOL) followed by <Casio04.01>
F1 (SRT-A). Follow the prompts to enter the
number of lists and to nominate the list required.
Press EXE after each prompt. Scroll down the
list to count the frequency of each number.
For the TI-Nspire CAS
1. Use the randInt( function as shown in the
previous Graphics calculator tip . Enter 1 for the
lower bound, 6 for the upper bound and 20 for
the number of trials.
Chapter 11 Introduction to probability 555
THINK WRITE/DISPLAY
2. Press · to generate the numbers in the first
column.

3. To make it easier to record the results in a frequency


table, we can sort the list of numbers into ascending
order. Press the up arrow on the NavPad until column
A is highlighted. Press b and then select 1:Actions
followed by 6: Sort. Press · to select OK at the
prompt screen. Ensure that the Sort screen shows a[]
for the range and Ascending. Press the e key until
OK is highlighted.
4. Press · to obtain the sorted list. Scroll down the
list to count the frequency of each number.

2 Enter this information in a table as shown. (Your


Die value 1 2 3 4 5 6
table will show different values.)
Frequency 2 4 6 2 5 1

WORKED Example 15
Generate 50 random numbers from 5 to 10 and sketch a histogram of the results.
THINK DISPLAY
Use a graphics calculator to generate 50 random
numbers in the range 5 to 10. Then use the list of
numbers to produce a histogram of the results.
For the Casio fx-9860G AU
1. Use the sequence and integer functions as shown in
the Graphics calculator tip on page 552. Enter the
rule Int (6 × Ran# + 5) with a start value of 1, an end <Casio04.01>
value of 50 and an increment of 1 for the variable N.
Store the numbers in List 1. Note that we multiply
by 6 because there are 6 possible numbers in the
desired set (that is, 5, 6, 7, 8, 9 and 10). We add 5 as
the numbers generated must start at 5.
2. Press MENU and then select STAT. The numbers
generated will be stored in List 1.
Continued over page
556 M a t h s Q u e s t M a t h s B Ye a r 1 1 f o r Q u e e n s l a n d

THINK DISPLAY
3. Press F1 (GRPH). To specify the graph is to be a
histogram, press F6 (SET) and then scroll down
to Graph Type. Press F6 ( ) for more options
and then press F1 (Hist).

4. Press EXE . Press F1 (GPH1), scroll down to


set the width to 1 and then press EXE to accept
this setting.

5. Press EXE to display the histogram. (Your


histogram will look different to that shown here.)
Press SHIFT F1 (TRCE) to explore the graph.

For the TI-Nspire CAS


1. Since we wish to plot the values later, enter a name
for column A. Use the randInt( function as shown
in the Graphics calculator tip on page 553. Enter 5
for the lower bound, 10 for the upper bound and
50 for the number of trials.

2. Press · to generate the numbers in the first


column.

3. Highlight column A and press b, then select


3: Data followed by 5: Quick Graph. A dot plot of
the data will be displayed.

4. Press b and then select 1: Plot Type followed by


3: Histogram. The histogram of the results will be
displayed. (Your histogram will look different to
that shown here.)
Chapter 11 Introduction to probability 557

11D Modelling probability

WORKED 1 Use a sequence of 6 random numbers to simulate rolling a die 9 times. Record your
Example
14
results in a frequency table.

WORKED
Example 2 Generate 100 numbers between 10 and 20. Produce a histogram of your results.
15
3 Produce a histogram showing the results of 1000 simulated rolls of a die.
4 a Suggest how a graphics calculator could be used to simulate the tossing of a coin.
eBook plus
b Generate 10 ‘coin tosses’ using this method.
Digital doc: c Sketch a frequency histogram for your results.
EXCEL Spreadsheet
Random numbers 5 Repeat question 4 for 100 coin tosses. What do you notice about the histogram?
6 A mini-lottery game may be simulated as follows. Each game consists of choosing two
numbers from the whole numbers 1 to 6. The cost to play one game is $1. A particular
player always chooses the numbers 1 and 2. A prize of $10 is paid for both numbers
correct. No other prizes are awarded.
a Simulate a game by generating 2 random numbers between 1 and 6. Do this
20 times (that is, ‘play’ 20 games of lotto). How many times does the player win?
b What is the player’s profit/loss based on the simulation?
7 Simulate 20 tosses of two dice (die 1 and die 2). How many times did die 1 produce a
lower number than die 2? (Hint: Generate two lists of 20 values between 1 and 6.)

8 A board game contains two spinners pictured below.

1 2 1 2
5 3
3 4
4

a Simulate 10 spins of each spinner.


b How many times does the total (that is, both spinners’ numbers added) equal 5?
c How often is there an even number on both spinners?
d How often does the highest possible total occur?
558 M a t h s Q u e s t M a t h s B Ye a r 1 1 f o r Q u e e n s l a n d

Random choice
1 Use your graphics calculator to generate 50 random numbers between 1 and
100.
2 Display the results in the table below.

Random number Frequency


1–10
11–20
21–30
31–40
41–50
51–60
61–70
71–80
81–90
91–100

3 Display the results in a frequency histogram.


4 Now ask 50 people chosen at random to select a number between 1 and 100.
Place the results in a similar table and display the results in a histogram.
Which set of results appears to be more random in nature? Explain your answer
with reference to the shape of the histograms you have drawn.

Long-run proportion
Unless we happen to ‘know’ the true probability of an event, a simulation as outlined
previously provides us with a method of finding the ‘experimental’ probability. In fact,
how do we know that the probability of getting a Tail when tossing a fair coin is really
eBook plus 1
--- ? The answer to this lies in the belief that if we tossed a coin enough times, the pro-
2
portion of Tails would be close to, if not exactly, --1- . However, there are no guarantees;
Digital docs: 2
EXCEL Spreadsheet
different simulation experiments will yield different results, but it is the sum total of all
Coin tossing coin-toss simulations that would be likely to yield a result close to 0.5.
EXCEL Spreadsheet With software or calculator simulations it is possible to ‘toss’ a coin hundreds or
Die rolling
thousands of times to see if this occurs. By recording the ratio of Tails to ‘coins tossed’
we can compute the experimental probability or long-run proportion. Note that in this
chapter, the term proportion is similar to probability.
Log into www.jacplus.com.au and locate the weblinks for this chapter to access files
that simulate coin tossing and die rolling. The screen on the following page shows the
frequency of each number rolled after simulating 10, 100 or 1000 throws of a die.
Chapter 11 Introduction to probability 559

Finding the long-run proportion


Similarly, the screen below shows the number of Heads and Tails obtained after simu-
lating the toss of a coin 100 times. The number of tosses can be changed and a new
simulation can be produced by pressing F9.

eBook plus

Digital doc:
EXCEL Spreadsheet
Simulating
coin tosses

By starting with a sequence of Heads/Tails, we can determine the long-run pro-


portion for coin tosses. This is best shown by an example.
560 M a t h s Q u e s t M a t h s B Ye a r 1 1 f o r Q u e e n s l a n d

WORKED Example 16
Given the following sequence of 20 coin tosses, determine its long-run proportion for Heads.
HTHHTTTHTHTTHHTHHTHH
THINK WRITE
1 Create 2 columns. Column 1 Column 2 Column 3
Column 1 counts coin tosses. (coin toss) (Heads) (proportion)
Column 2 counts Heads. 1 1 1 ÷ 1 = 1.000
That is, after 1 toss, there was 1 Head.
2 1 1 ÷ 2 = 0.500
After 2 tosses there was still 1 Head.
After 3 tosses there were 2 Heads. 3 2 2 ÷ 3 = 0.667
After 4 tosses there were 3 Heads . . . 4 3 3 ÷ 4 = 0.750
and so on. 5 3 3 ÷ 5 = 0.600
2 Divide column 2 by column 1. 6 3 3 ÷ 6 = 0.500
The result is shown in column 3,
7 3 3 ÷ 7 = 0.429
rounded to 3 decimal places.
3 (a) The long-run proportion for
8 4 4 ÷ 8 = 0.500
20 tosses is given by the last 9 4 4 ÷ 9 = 0.444
number in column 3, namely 0.550. 10 5 5 ÷ 10 = 0.500
(b) Observe how the proportion varies 11 5 5 ÷ 11 = 0.455
wildly after the first few tosses, but
12 5 5 ÷ 12 = 0.417
begins to ‘stabilise’ as the number
increases. 13 6 6 ÷ 13 = 0.462
(c) Note that 20 tosses are not enough 14 7 7 ÷ 14 = 0.500
to give a long-run proportion very 15 7 7 ÷ 15 = 0.467
close to the ‘true’ probability of 0.5. 16 8 8 ÷ 16 = 0.500
17 9 9 ÷ 17 = 0.529
18 9 9 ÷ 18 = 0.500
19 10 10 ÷ 19 = 0.526
20 11 11 ÷ 20 = 0.550

Another application of long-run proportion is to use experimental data to estimate


the true probability of an event. Consider an application in the game of cricket.

WORKED Example 17
A possible measure of a batsman’s effectiveness in test cricket is the number of times he
makes a run-scoring stroke as a proportion of balls faced. Consider this set of data from
10 innings over 5 test matches for well-known cricketer Nalla Redrob. Calculate the final
long-run proportion of scoring strokes to balls faced.
Innings 1 2 3 4 5 6 7 8 9 10
Run-scoring 34 1 67 38 12 15 47 69 43 18
strokes
Balls faced 62 6 107 87 29 19 75 119 67 31
Chapter 11 Introduction to probability 561
THINK WRITE
1 Create 4 columns.
Column 2
(a) Put the innings number Column 1 (run-scoring Column 3 Column 4
in column 1. (innings) strokes) (balls faced) (proportion)
(b) Put the cumulative
1 34 62 34 ÷ 62 = 0.548
(total) number of run-
scoring strokes in 2 34 + 1 = 350 62 + 6 = 680 35 ÷ 68 = 0.515
column 2.
(c) Put the cumulative 3 35 + 67 = 102 68 + 107 = 175 102 ÷ 175 = 0.583
(total) number of balls 4 102 + 38 = 140 175 + 87 = 262 140 ÷ 262 = 0.534
faced in column 3.
Complete column 4. 5 140 + 12 = 152 262 + 29 = 291 152 ÷ 291 = 0.522
2
(a) Divide the number in 6 152 + 15 = 167 291 + 19 = 310 167 ÷ 310 = 0.539
column 2 by the
number in column 3. 7 167 + 47 = 214 310 + 75 = 385 214 ÷ 385 = 0.556
(b) Enter this result in 8 214 + 69 = 283 385 + 119 = 504 283 ÷ 504 = 0.562
column 4. This is the
long-run proportion for 9 283 + 43 = 326 504 + 67 = 571 326 ÷ 571 = 0.571
run-scoring strokes to 10 326 + 18 = 344 571 + 31 = 602 344 ÷ 602 = 0.571
balls faced.
(c) Round the answer to
3 decimal places.

This implies the batsman has a probability of making a run-scoring stroke of about
0.57. It is significant to note that this proportion didn’t change much after the first 6 or
7 innings. Although this might not be the best way to measure a batsman’s
‘effectiveness’ in cricket, it is similar to the method used in baseball to measure
‘batting average’, a method which has been used for over 100 years.
It is not necessary to calculate long-run proportion at all stages of the experiment.
Often it is sufficient merely to take the data at the end of the experiment and perform
the appropriate division.

WORKED Example 18
A die which is suspected of being biased (unfair) is tossed 800 times and it is observed that
a 6 appeared 205 times. Calculate the long-run proportion and comment on the result.
THINK WRITE
205
1 Calculate the long-run proportion. In Long-run proportion = ---------
this case divide the number of 6s by the 800
Long-run proportion = 0.256
total.
2 Compare with the ‘theoretical’ result. We expect about 1--- of the results to be 6s, or
6
about 133 out of 800. This proportion is 0.167.
The experimental result indicates that the die is
likely to be unfair with 6s appearing too often.
562 M a t h s Q u e s t M a t h s B Ye a r 1 1 f o r Q u e e n s l a n d

It is only after a very large number of trials, say 100 000, that we could be confident
the experimental probability was close to the theoretical probability.
The screen view of a spreadsheet showing 1000 simulated coin tosses is shown
below.

eBook plus

Digital doc:
EXCEL Spreadsheet
Long-run proportion

remember
1. In the long run, the experimental probability obtained using random numbers
becomes closer to the true probability. This is known as long-run proportion.
2. The greater the number of trials, the closer the experimental probability
approaches the theoretical probability.

11E Long-run proportion

WORKED 1 Given the following sequence of 20 rolls of a die, determine its long-run proportion
Example
16
for sixes.
2, 5, 6, 3, 3, 2, 4, 4, 5, 5, 3, 6, 4, 3, 3, 5, 3, 3, 2, 1
WORKED 2 A possible measure of a batsman’s effectiveness in test cricket is the number of times
Example
17
he makes a run-scoring stroke as a proportion of balls faced. Consider the data from
12 innings over 6 test matches for cricketer Ian Toppig. Calculate the final long-run
proportion of scoring shots to balls faced.
Innings 1 2 3 4 5 6 7 8 9 10 11 12
Scoring shots 23 26 45 9 23 34 56 37 18 23 31 46
Balls faced 57 57 89 19 46 72 100 68 31 50 66 89
Chapter 11 Introduction to probability 563
3 a A travelling salesperson records her daily success rate at selling vacuum cleaners.
On Day 1 she visited 28 houses and sold 7 vacuum cleaners. Calculate the long-
run proportion of sales to houses visited for Day 1.
b Now calculate the long-run proportion of sales to houses visited for each day and
comment on whether she is improving her ability as a salesperson.
Day 1 2 3 4 5 6 7 8 9 10
Sales 7 4 6 8 16 12 7 12 12 13
Houses visited 28 19 21 25 42 29 22 31 27 26

4 multiple choice
A vacuum cleaner salesman on his first 5 days sold the following number of cleaners: 5,
2, 4, 5, 3. He restricted himself to 20 visits per day. His long-run proportion of sales is:
1 5 19 19 20
A --- B ------ C ------ D --------- E ---------
2 20 20 100 100

5 multiple choice
Two statisticians perform a simulation of tossing a single coin. Statistician A does a
simulation of 100 tosses, while Statistician B simulates 1000 tosses. Which statistician
will get a result closest to 0.5 for the proportion of Heads?
A Definitely Statistician A
B Definitely Statistician B
C They will get the same result.
D More likely to be Statistician B
E None of the above
WORKED 6 A coin suspected of being biased is tossed 600 times and it is observed that a Head
Example
18
appeared 425 times. Calculate the long-run proportion and comment on the result.
7 A student answers 200 questions from a mathematics textbook and answers 156 of
them correctly. Estimate the probability that the student will answer the next question
correctly.
8 Another student answers only 145 questions from the same textbook as in question 7
and answers 119 of them correctly. Is this student more effective at answering
mathematics questions?
9 A die is rolled to determine whether it is a fair one and the results are recorded.
Calculate the proportion of each value and comment on the results.
Value 1 2 3 4 5 6
Frequency 457 590 189 623 234 444
10 Three different bread shops recorded the number of loaves of bread sold and the
number ordered for 1 month. Which of the three was the most effective? Support your
statement with mathematical evidence.
Bread shop 1 2 3
Loaves sold 67 167 267
Loaves ordered 89 199 309
564 M a t h s Q u e s t M a t h s B Ye a r 1 1 f o r Q u e e n s l a n d

11 Use a spreadsheet or other means to calculate the long-run proportion for:


eBook plus a 1000 coin tosses
b 1000 die rolls.
Digital docs:
EXCEL Spreadsheet
Die rolling 12 A cricketer wishes to measure her effectiveness against left- and right-handed bowlers
EXCEL Spreadsheet and records the following data over 2 years.
Coin tossing

Innings Innings caught Innings bowled Innings not out


Left-handers 6 4 4
Right-handers 24 26 6

a Is she more likely to be caught by left- or right-handers?


b Is she more likely to be caught or bowled?
c Is she more likely to be not out against left- or right-handers?
d Use the results from a to c to comment on the cricketer’s measure of effectiveness
against left- and right-handed bowlers.

13 A method of estimating the population of trout in a lake works as follows. Take a


number of foreign trout, tag them and put them in the lake. A week or so later, catch
the same number of trout from the lake. Calculate the proportion of tagged fish caught
eBook plus to tagged fish originally thrown in. The method uses this same proportion for the
number of native trout to native trout caught in the lake. Suppose 100 fish are tagged
Digital doc:
WorkSHEET 11.2
and thrown in. A week later 100 fish are caught and 7 of them are tagged. Estimate the
population of trout in the lake (excluding the tagged ones).

Footy season
A football league has 8 teams. Each team
plays all the other teams once. Thus there are
28 games played in all.
1 Simulate a full season’s play, assuming
that each team has a 50:50 chance of
winning each game.
2 Modify the probabilities so that they are
unequal (hint: sum of probabilities = 4)
and simulate a full season’s play. Did the
better teams reach the top of the ladder?
Discuss your results with other students.
(Hint: If Team 1 has a probability of winning
of 0.7 and Team 2 has a probability of
winning of 0.6, then when they play against
each other, the probability of Team 1
0.7 .)
winning is --------------------
-
0.7 + 0.6
Chapter 11 Introduction to probability 565

summary
Informal description of chance
• The chance of an event occurring can be described as being from certain (a
probability of 1) to impossible (a probability of 0).
• Terms used to describe the chance of an event occurring include improbable,
unlikely, fifty-fifty, likely and probable.
• The chance of an event occurring can be described by counting the possible
outcomes and sometimes by relying on our general knowledge.

Single event probability


• The probability of an event can be found using the formula
number of favourable outcomes
P (event) = ----------------------------------------------------------------------------
total number of outcomes
• Probabilities are often written as fractions, but can also be expressed as decimals or
percentages.

Relative frequency
• Relative frequency describes how often an event has occurred.
• It is found by dividing the number of times an event has occurred by the total
number of trials.

Random numbers and simulations


• Computers and calculators can generate random decimals between 0 and 1.
• To generate 10 random numbers between 1 and 6 inclusive use the
Int (6 × Ran# + 1), N, 1, 10, 1 function on the Casio fx-9860G AU graphics
calculator, or the randInt(1, 6, 10) function on the TI-Nspire CAS calculator,
or randbetween(1, 6) on an Excel spreadsheet.

Long-run proportion
• Long-run proportion is when the experimental probability obtained using random
numbers gets closer to the true probability.
• The probability of an event, A, is symbolised by P(A).
566 M a t h s Q u e s t M a t h s B Ye a r 1 1 f o r Q u e e n s l a n d

CHAPTER
review
1 Graham and Marcia are playing a game. To see who starts they each take a card from a
11A standard deck. The player with the highest card starts. Graham takes a five. Describe
Marcia’s chance of taking a higher card.

2 Describe each of the following events as being either certain, probable, even chance (fifty-
11A fifty), unlikely or impossible.
a Rolling a die and getting a number less than 6
b Choosing the eleven of diamonds from a standard deck of cards
c Tossing a coin and it landing Tails
d Rolling two dice and getting a total of 12
e Winning the lottery with one ticket

3 Give an example of an event which is:


11A a certain b impossible.

4 The Chen family are going on holidays to Alice Springs during January. Are they more
11A likely to experience hot weather or cold weather?

5 List each of the events below in order from most likely to least likely.
11A Winning a lottery with 1 ticket out of 100 000 tickets sold
Rolling a die and getting a number greater than 1
Selecting a blue marble out of a bag containing 14 blue, 15 red and 21 green marbles
Selecting a picture card from a standard deck

6 Mark and Lleyton are tennis players who have played eight previous matches. Mark has
11A won six of these matches. When they play their ninth match, who is more likely to win?
Explain your answer.
Chapter 11 Introduction to probability 567
7 The numbers 1 to 5 are written on the front of 5 cards that are turned face down. Michelle
then chooses one card at random. She wants to choose a number greater than 2. List the 11B
sample space and all favourable outcomes.

8 A barrel contains 25 balls numbered from 1 to 25. One ball is drawn from the barrel. Find
the probability that the ball drawn is: 11B
a 13 b 7 c an odd number
d a square number e a prime number f a double-digit number.

9 A card is to be chosen from a standard deck. Find the probability that the card chosen is:
a the 2 of clubs b any 2 c any club 11B
d a black card e a court card f a spot card.

10 A DVD collection has 12 dramas, 14 comedies, 4 horror and 10 romance movies. If Pablo
chooses a movie at random from the collection, find the probability that the movie chosen is: 11B
a a comedy b a horror c not romance.

11 The digits 5, 7, 8 and 9 are written on cards. They are then arranged to form a four-digit
number. Find the probability that the number formed is: 11B
a 7895 b odd c divisible by 5
d greater than 7000 e less than 8000.

12 A raffle has 2000 tickets sold and has two prizes. Jacqui buys five tickets.
a Find the probability that Jacqui wins 1st prize. 11B
b If Jacqui wins 1st prize, what is the probability that she also wins 2nd prize?

13 From every 100 televisions on a production line, two are found to be defective. If a
television is chosen at random, find the relative frequency of defective televisions. 11C
14 It is found that 150 of every thousand 17-year-old drivers will be involved in an accident
within one year of having their driver’s licence. 11C

a What is the relative frequency of a 17-year-old driver having an accident?


b If the average cost to an insurance company of each accident is $5000, what would be
the minimum premium that an insurance company should charge a 17-year-old driver?
568 M a t h s Q u e s t M a t h s B Ye a r 1 1 f o r Q u e e n s l a n d

15 Simulate 30 rolls of two dice (die 1 and die 2). How many times does the total shown on
11D both dice equal 6?

16 Given the following sequence of 20 rolls of a die, determine its long-run proportion for
11E twos.
3, 1, 5, 6, 2, 4, 3, 3, 2, 6, 2, 5, 4, 6, 1, 2, 4, 3, 1, 4

17 A netball team wins 15 of the past 20 matches played. Estimate the probability that the team
11E will win the next match.

Modelling and problem solving


1 John lives in Mackay. Last year he flew to New York to visit his sister. He was about to
collect his luggage at the JFK terminal when he bumped into an old friend from school. He
thought, ‘This is incredible. I probably only know about 100 people and I meet one on the
other side of the world. The probability of this must be about 100 out of 7 000 000 000.’
Assuming there are about 7 billion people in the world, comment on the likelihood of John’s
accidental meeting.

2 A dice game is played whereby a die is


rolled. It costs $1 to play. If the player
rolls a 6 they receive a return of $6 (they
actually win $5 as it cost $1 to play). If
eBook plus the player rolls a 1 they get the $1 cost of
the game back. Any other number means
Digital doc: they lose $1. Use a long-run proportion
Test Yourself
Chapter 11 to calculate whether the player can
expect to win or lose money.
Rates of
change
12
syllabus reference
Topic:
• Rates of change

In this chapter
12A Constant rates
12B Variable rates
12C Average rates of change
12D Instantaneous rates
12E Motion graphs
12F Relating the gradient
function to the original
function
12G Relating velocity–time
graphs to position–time
graphs
12H Rates of change of
polynomials
570 M a t h s Q u e s t M a t h s B Ye a r 1 1 f o r Q u e e n s l a n d

Introduction
A heavy spanner falls from a tower, 45 metres high, and crashes into the ground. How
can we predict the damage the spanner causes when it hits the ground? (Let’s hope it
hits the ground!)
We will begin by attempting to develop a mathematical model for the speed of the
spanner as it falls.
Speed is a rate — a measure of distance travelled per unit time. Our modelling thus
begins with a review of rate and the concept of a constant rate.

Constant rates
When the rate of change of one quantity with respect to another does not alter, the rate
is constant.
For example, if petrol is $1.70 cents per litre then every litre of petrol purchased at
this rate always costs $1.70. This means 10 litres of petrol would cost $17.00 and
100 litres of petrol would cost $170.00. Calculating the gradient from the graph:
rise
Gradient = --------
run 170
$(170 – 17)
= ---------------------------------------
( 100 – 10 ) litres
Price ($)

$153
= -------------------
90 litres
$1.70
= -------------- 17
1 litre
= $1.70/litre 10 100
= rate Number of litres (L)
That is, the gradient of this graph represents a rate.
A straight line graph shows that the rate of change between
two quantities is constant.

WORKED Example 1 1100


The graph shown represents the height, H, in metres (m)
Height (m)

of a hot air balloon and time, T, in hours (h) after it is


launched from a mountain top. Use the graph to find: 500
a the gradient b the rate of change
c the rule for height as a function of time.

THINK WRITE 5
Time (h)
rise 1100 – 500
a Use gradient of straight line, m = -------- . a Gradient = ---------------------------
run 5–0
600
= ---------
5
= 120
b Rate of change is the same as the b Rate of change = 120 m/h
gradient. Include appropriate units.
c As the graph is a straight line, express c The rule is of the form H = mT + c where
the rule in the form y = mx + c. m = 120 and c = 500.
H = 120T + 500
Chapter 12 Rates of change 571
WORKED Example 2
The table below shows the number of people, N, that are left inside an office building
T minutes after the alarm is triggered during a fire drill.

a Show that the relationship is linear. T 0 1 2 3 4


b Find the rate of change.
c Express N as a function of T. N 140 105 70 35 0

THINK WRITE

a 1 Plot the graph of the points given. a N

140
105
70
35

1 2 3 4 T

2 Check whether the points are A straight line graph was produced, so the
collinear, that is, whether they lie on relationship is linear.
a straight line.

0 – 140
b 1 Calculate the gradient, m. b m = ------------------
4–0
– 140
= ------------
4
= −35
2 Use the gradient to state the rate of Rate of change = −35 people/min
change. Include the appropriate The number of people left inside the building
units. is decreasing at a constant rate.

c Express the rule in the form y = mx + c c The rule is N = −35T + 140.


where m = −35 and c = 140. Use N and T
instead of y and x.

remember
1. The gradient of a graph represents the rate of change.
2. A positive gradient shows a positive or increasing rate of change.
3. A negative gradient shows a negative or decreasing rate of change.
4. A linear or straight line graph shows that the rate of change is constant.
572 M a t h s Q u e s t M a t h s B Ye a r 1 1 f o r Q u e e n s l a n d

12A Constant rates

1 Which of the following rates are con-


stant?
a A person’s pulse rate when run-
ning 3 km
b The rate of growth of Australia’s
population
c A person’s pulse rate when lying
down
d The daily hire rate of a certain car
e The rate of growth of a baby
f The rate of temperature change
during the day
g The commission rate of pay of a
salesperson
h The rate at which the Earth spins
on its axis
i The rate at which students arrive at school in the morning
j The rate at which water runs into a bath when the tap is left on
k The number of hours of daylight per day
2 Which of the following graphs show a constant rate of change?
a b c d e
d d d d d

0 t 0 t 0 t 0 t 0 t
f g h i j
d d d d d

0 t 0 t 0 t 0 t 0 t

3 For the constant rates of change found in question 2, which of these rates are:
i positive? ii negative? iii zero?

4 multiple choice
Distance (metres)

The distance travelled by a car is shown in this graph.


The graph shows that the car is:
A slowing down B speeding up
C travelling uphill D travelling at constant speed
E stationary 0 Time (hours)
Chapter 12 Rates of change 573
5 multiple choice v (litres)
18
The volume of water, v litres, in a container is shown
as a function of time, t minutes, in the graph.
The rate of change of water in the container is: 10
A 2 L/min B 8 L/min C −2 L/min
D −8 L/min E 0 L/min
0 t (min)
4
WORKED 6 For each of the functions graphed below state:
Example
1
i the gradient ii the rate of change iii the rule.
a b c
d (m) v (litres) w (kg)
260 100
200

60

50
0 t (h) 0
4 0 t (min) 40 t (weeks)
30
d e f
h (m) w (g) w (g)
75 16 30

10

5
0 0 0
10 t (years) 8 t (min) 20 120 v (litres)

7 multiple choice
Water is being pumped out of a swimming pool holding 30 000 litres at a constant
rate. It takes 25 minutes to remove 1000 litres of water.
a The rate of change of the volume of water in the pool is:
A 400 L/min B 1200 L/min C −1200 L/min
D −40 L/min E 40 L/min
b The time taken to empty the pool is:
A 12 hours and 5 minutes B 12 hours and 30 minutes
C 6 hours and 30 minutes D 2 days E 18 hours
8 The number of crates of fruit picked by a fruit-picker
over the course of a day is shown in the graph. 13
Number of crates

If the fruit-picker is paid $12 per crate, answer the


following:
a What is the rate of pay per hour in the first 3 hours? 7
6
b Explain what probably happened between 12.00 pm
and 1.00 pm.
c What is the rate of pay per hour in the last 4 hours? 0
d Suggest two possible reasons why the line is not as
8 am

11 am
12 pm
1 pm

5 pm

steep in the afternoon.


Time
e How much is earned for the day?
574 M a t h s Q u e s t M a t h s B Ye a r 1 1 f o r Q u e e n s l a n d

9 An athlete runs at a constant speed of 6 m/s for the first 420 m of a race. The next 400 m
are run at a constant speed of 5 m/s and the final 180 m at a constant speed of 6 m/s.
a How far is the race?
b How long does it take to run the first 420 m?
c How long does it take to run the next 400 m?
d How long does it take to run the final 180 m?
e Sketch the graph of distance, D metres, versus time, T seconds.

WORKED 10 A spring is stretched a distance x cm beyond its natural length when a weight of W kg
Example
2
is attached to one end as shown in the following table.

x 0 5 10 15 20 25
eBook plus
W 0 1 2 3 4 5
Digital doc:
EXCEL Spreadsheet
Plotting relations a Show that there is a linear relationship between the variables.
b What is the rate of change of W with respect to x?
c Express W as a function of x.

11 A bucket contains 40 litres of water when it starts leaking through a hole in the
bottom, such that the volume of water, V litres, in the bucket at any time, t hours, is
given by the rule: V = 40 − 6t.
a At what constant rate, in litres/hour, is the water leaking?
b How long will it take the bucket to be empty?

12 Water is poured at a constant rate into vessels a, b, c and d shown below.


a b c d

Match the appropriate graph below, representing height of water, h, at any time, t,
with each vessel above.
i ii iii iv
h h h h

0 t 0 t 0 t 0 t

13 Devise a multiple choice question in the style of the previous question, but using
vessels of your own design.
Chapter 12 Rates of change 575
Variable rates 50
A useful technique for modelling the 40

d (metres)
speed of the spanner as it falls to the
ground is to draw a graph of distance 30

travelled, d, versus time, t. 20


From the graph we can see that the
spanner travels 45 metres in 3 seconds 10

and then stops moving. In other words, 0


0.5 1.0 1.5 2.0 2.5 3.0 3.5 4.0 4.5 5.0 5.5
it falls 45 metres and then hits the t (seconds)
ground. We also notice that the speed
of the spanner is not constant but is variable.
If a rate is not constant but is changing then it must be a variable rate.
For example, the amount of electricity used per hour by a household is most likely to
be a variable rate as the need for electricity will change throughout the day.
Graphically, a variable rate will not be linear, because the gradient (or rate of
change) is not constant.
The distance–time graph (at right) shows a variable rate d
between t = 0 and t = 3 because the gradient is changing
over this interval. However, it shows a constant rate
between: t = 0 and t = 1; t = 1 and t = 2; and t = 2 and t = 3
because the gradient is not changing over these intervals.
Relationships which give curves when graphed show
that the rate of change is continually changing, that is, a
variable rate exists. 0 t
1 2 3

WORKED Example 3
B C
Consider the graph shown. For each interval, state whether:
a the rate of change is constant or variable
b the rate of change is positive, negative or zero.
THINK WRITE A D

a Consider whether the graph is linear for a Interval AB: variable rate of change
each interval. The rate of change is Interval BC: constant rate of change
constant if the graph is linear, otherwise Interval CD: constant rate of change
it is variable.
b Since the gradient of the graph gives a b Interval AB: positive rate of change
measure of the rate of change, consider Interval BC: zero rate of change
whether the gradient is positive, negative Interval CD: negative rate of change
or zero in each interval.
576 M a t h s Q u e s t M a t h s B Ye a r 1 1 f o r Q u e e n s l a n d

remember
1. A variable rate exists when the rate of change is not constant.
2. On a graph, a variable rate of change can be seen when the points do not lie on
a straight line.

12B Variable rates

1 List which of the following rates are variable.


a The pulse rate of a person watching a horror movie
b The cost per kg of tomatoes at the local greengrocer today
c The speed of a car travelling through the city
d The cost per kg of tomatoes at the local greengrocer during the year
e The number of hours of sunlight per day
f The hourly rate of a receptionist
g The hourly rate of a private taxi driver
h The speed of a marble rolling down an incline

2 Which of the following graphs show variable rates of change between points A and B?
a b c
B
B B
A A

d e f
B

B B
A A

WORKED 3 The graph at right shows how the height of a ‘big


Example Q R
3
dipper’ ride varies with the horizontal distance.
P
a For each interval, state whether i the rate of T
Height

change is constant or variable; and ii the rate of U


change is positive, negative or zero.
S V
W
O Distance
Chapter 12 Rates of change 577
Assume the big dipper ‘free wheels’ from point R to point W to answer parts b to f.

multiple choice
b Between R and U, the point at which the big dipper will be fastest is:
A T B S C U D R E W

c Between points S and V, the point at which the big dipper will be slowest is:
A S B U C V D T E W

d Travelling from R to S, the speed is:


A increasing B constant C decreasing
D increasing then decreasing E decreasing then increasing

e Between S and T the speed is:


A increasing B constant C decreasing
D increasing then decreasing E decreasing then increasing

f From U to W the speed is:


A increasing B constant C decreasing
D increasing then decreasing E decreasing then increasing

4 The following table shows weight, W kg, varying with time, t min.

t (min) 0 1 2 3 4

W (kg) 0 3 6 9 12

a Plot a graph of W versus t.


b What do you notice about the plotted points?
c Is the rate of change variable or constant?
d What is the rate of change in kg/min?

5 The following table also shows weight, W kg, varying with time, t min.

t (min) 0 1 2 3 4

W (kg) 2 4 7 11 18

a Plot these points on a graph.


b Do these points form a straight line?
c Is the rate of change variable or constant?
d What is the ‘average rate of change’ during:
i the first minute?
ii the second minute?
iii the fourth minute?
578 M a t h s Q u e s t M a t h s B Ye a r 1 1 f o r Q u e e n s l a n d

6 The following tables of values show distance travelled, d km, at various times, t hours.
eBook plus Decide whether the rate of change of distance with respect to time appears ‘constant’ or
‘variable’. Justify your answer.
Digital doc:
EXCEL Spreadsheet a b
Plotting relations
t 0 1 2 3 4 t 0 1 2 3 4
d 0 5 10 15 20 d 0 15 20 45 80

c d
t 0 1 2 3 4 t 0 1 2 3 4
d 0 6 12 18 24 d 0 4 12 24 40

e f
t 0 1 2 3 4 t 1 2 3 4 5
d 0 1.5 4 8.5 11 d 6 9 13 16 20

Average rates of change


If a rate is variable it is sometimes useful to know the average rate of change over a
specified interval. For example, a tree grew from 6.2 m this time last year to 6.75 m now.
change in height
Its average rate of growth = ---------------------------------------
change in time
( 6.75 – 6.2 ) m
= ----------------------------------
1 year
= 0.55 m/year.
This means that the tree grew by 0.55 metres over the past year but not necessarily
constantly at that rate during the year.

WORKED Example 4
Q
Calculate the average rate of change between points P and Q 48
Distance (m)

marked on the graph. P


30

0 2 7
THINK WRITE Time (s)

48 – 30 18
1 Find the gradient of the chord PQ. Gradient of PQ = ------------------ = ------ = 3.6
7–2 5
2 Use the gradient to state the average rate of Average rate of change = 3.6 m/s
change. Include the appropriate units.
Chapter 12 Rates of change 579
WORKED Example 5
Find the average rate of change of height between t = 1 and t = 3 from the table below.

t (min) 0 1 2 3 4 5
d (m) 20 60 90 130 140 145

THINK WRITE
Calculate the average rate of change of height Average rate of change of height
with respect to time by considering the change in change in height
= ---------------------------------------
each quantity. When the time changes from t = 1 change in time
min to t = 3 min, the height changes from 60 m ( 130 – 60 ) m
to 130 m. = --------------------------------
( 3 – 1 ) min
70 m
= --------------
2 min
= 35 m/min

WORKED Example 6
Over a period of 6 hours, the temperature of a room is described by the function
T(h) = h2 − 4h + 22 where T is the temperature in degrees Celsius after h hours.
a What is the initial temperature of the room?
b Sketch the graph of the function over the given time interval.
c Draw a chord between the points where h = 1 and h = 5.
d What is the gradient of this chord?
e What is the average rate of change of temperature between h = 1 and h = 5?

THINK WRITE
a Initial temperature is the temperature at a When h = 0, T(0) = 0 − 0 + 22
the start of the time period. Substitute = 22
h = 0 into the function to find T(0). Initial temperature is 22°C.
b Use a graphics calculator or rewrite the b T(h) = h2 − 4h + 22
function in turning point form. = (h2 − 4h + 4) − 4 + 22
= (h − 2)2 + 18
Turning point of parabola is (2, 18).
c 1 Find the required points: c T (°C)
T(1) = 12 − 4(1) + 22 = 19 27
T(5) = 52 − 4(5) + 22 = 27.
22
2 Indicate the points (1, 19) and 19
(5, 27) on the graph and join with a 18
straight line. H (hours)
1 2 3 4 5

Continued over page


580 M a t h s Q u e s t M a t h s B Ye a r 1 1 f o r Q u e e n s l a n d

THINK WRITE
rise
d Use gradient = -------- and the points d Gradient = 27 – 19
------------------
run 5–1
(1, 19) and (5, 27). 8
= ---
4
= 2

e Use the gradient to state the average rate e Average rate of change is 2°C/h.
of change. Include appropriate units.

remember
1. Over a specified interval:
change in A
Average rate of change of A with respect to B = ----------------------------- .
change in B
Average rate of change of a function f(t) over the time interval t = a to t = b is:
change in f ( t ) f (b) – f (a)
----------------------------------- = ----------------------------- .
change in t b–a
rise
2. When using a graph, find the gradient  -------- of the chord formed between the
 run 
end points of the interval.
3. When using a table of values or rule, find the change in each quantity over the
specified interval.

12C Average rates of change

WORKED 1 Calculate the average rate of change between points P and Q marked on each graph.
Example
4
a y b y P
10
eBook plus
8
Q
Digital doc: P
SkillSHEET 12.1 6
Gradient of a
straight line

0 x
2 5
0 x –2 Q
5
Chapter 12 Rates of change 581
2 Which of the following graphs have a straight line drawn that can aid in finding the
average rate of change over a specified interval?
a y b y c y d y

0 x 0 x 0 x 0 x

3 For each of the distance–time graphs below:


i draw a chord to the graph for the interval t = 1 to t = 3
ii find the gradient of this chord
iii hence, find the average speed from t = 1 to t = 3.
a d (km) b d c d
70 70 70
50
40

10
0 0 t 0 t
1 3 t (h) 1 3 1 3
d d e d f d
70 70 90
60
70

0 t 0 t 0 t
1 3 1 3 1 3

4 Use this temperature–time graph to answer the T (∞C)


following questions. 90
a Find the gradient of the chord to the graph for the
interval t = 0 to t = 20. 50
b What is the average rate of change of temperature
with respect to time from t = 0 to t = 20?
c What is the average rate of change of temperature
with respect to time from t = 5 to t = 10? Explain 0 t (min)
10 20
your answer.

5 The graph at right represents the weight of a non- W (kg)


uniform beam, W (kg), versus its length, L (m). 32.5
30
a Find the gradient of the chord to the graph for the
interval L = 5 to L = 20.
b Find the gradient of the chord from L = 15 to L = 20. 10
c What is the average rate of change between:
i L = 5 and L = 20? ii L = 15 and L = 20? 0
5 10 15 20 L (m)
d Are these rates the same? Explain your answer.
582 M a t h s Q u e s t M a t h s B Ye a r 1 1 f o r Q u e e n s l a n d

6 multiple choice
Consider the graph at right. 4 5
3
The chord with the highest gradient is the 2
one from 0 to point number: 1
A 4 B 3 C 2
D 1 E 5
0

7 multiple choice H

S
Using the graph shown, the average rate of change between R
T = 10 and T = 30 is equal to the gradient of the chord:
T
A PT B QT C QS P
Q
D RS E PQ

0 10 20 30 40 T

8 The total number of people at the zoo at various times of the day is shown in the table below.

am pm
T (time of day) 10.00 11.00 12.00 1.00 2.00 3.00 4.00 5.00
N (number of 0 200 360 510 540 550 550 550
people at the zoo)

a Plot the graph of N versus T.


b Draw chords to the graph for the interval:
i 10.00 am to 1.00 pm ii 1.00 pm to 3.00 pm iii 3.00 pm to 5.00 pm.
c Find the gradient of each of these chords.
d What is the average rate of change from:
i 10.00 am to 1.00 pm?
ii 1.00 pm to 3.00 pm?
iii 3.00 pm to 5.00 pm?
e Briefly describe what these rates suggest about the number of people attending the
zoo during the course of the day.

9 The total distance, d km, travelled by Sally during a hike after various times, t hours,
is displayed in the table below.

t (hours) 0 0.5 1 1.5 2 2.5 3 3.5


d (km) 0 2.5 4 5 6.5 7 7 8

a Plot the graph of d versus t.


b Find the average rate of change of distance from:
i t = 0 to t = 0.5 ii t = 0.5 to t = 1 iii t = 2.5 to t = 3.
c What is likely to have happened between t = 2.5 and t = 3?
d In which half-hour time interval is she travelling fastest?
e In which half-hour time interval(s) is she averaging 2 km/h?
Chapter 12 Rates of change 583
WORKED 10 The height, h metres, reached by a balloon released from ground level after t minutes,
Example
is shown in the table below.
5
t (mins) 0 2 4 6 8 10
h (m) 0 220 360 450 480 490
a Without drawing the graph, find the average rate of change of height with respect
to time between:
vi t = 0 and t = 2 ii t = 2 and t = 4 iii t = 4 and t = 6
iv t = 6 and t = 8 v t = 8 and t = 10.
b Is the average rate of change for each 2-minute interval increasing or decreasing?
WORKED 11 The temperature of an iron rod placed in a furnace is described by the function
Example
6
T(t) = t2 + 20 between t = 0 and t = 10.
T represents the temperature of the rod in degrees Celsius and t is the time in seconds.
a What is the initial temperature of the iron rod?
eBook plus b Sketch the graph of the function over the given time interval.
c Draw a chord between the points where t = 2 and t = 8.
Digital doc:
d What is the gradient of this chord?
EXCEL Spreadsheet
Gradient of a chord e What is the average rate of change of temperature between t = 2 and t = 8?
12 The weight of a rabbit in its first 8 weeks is described by the function
W(t) = t3 − 3t2 + 200, where W represents the weight of the rabbit in grams and t is the
time in weeks.
a What is the weight of the rabbit at birth?
b What is the average rate of change of weight of the rabbit between:
i weeks 1 and 3?
ii weeks 1 and 2?
iii weeks 2 and 4?
vi weeks 2 and 6?
13 Jasmine and Jesse drive from Abingdon to Boulia at a speed of 90 km/h, and from
Boulia to Clarenvale at an average speed of 100 km/h. If the total time taken for the
journey from Abingdon to Clarenvale was 2 hours, and the average speed for the
entire journey was 96 km/h, how far is it from Abingdon to Boulia?

eBook plus
14 Tom and Jerry have a mowing business. It takes Tom 30 minutes to mow a particular
Digital doc: yard and it takes Jerry 40 minutes to mow the same yard. If they work together, how
WorkSHEET 12.1 long will it take them to mow this yard?
584 M a t h s Q u e s t M a t h s B Ye a r 1 1 f o r Q u e e n s l a n d

Instantaneous rates
If a rate is variable, it is often useful to know the rate of change at any given time or
point; that is, the instantaneous rate.
For example, a police radar gun is designed to give an instantaneous reading of a
vehicle’s speed. This enables the police to make an immediate decision as to whether a
car is breaking the speed limit or not.
y
Instantaneous rates can be found from a curved graph by:
1. drawing a tangent to the curve at the point in question, and
2. calculating the gradient of the tangent over an appro-
priate interval (that is, between two points whose
coordinates are easily identified).
Note: The gradient of the curve at a point, P, is defined as P Tangent
the gradient of the tangent to that point.
0 x

WORKED Example 7
a Use the following graph to find the gradient of the tangent at the point where L = 10.
b Hence, find the rate of change of weight, W, with respect to length, L, when L = 10.
W (kg)

10 L (m)

THINK WRITE
a 1 Draw in the required tangent. a W (kg)
2 To find the gradient of the tangent, 12
choose a convenient interval
8
(between the points where L = 5 and
L = 15). 4

0 5 10 15 20 L (m)

rise 4–8
3 Use gradient = -------- . Gradient = --------------
-
run 15 – 5
–4
= ------
10
= – 0.4

b Use the gradient to state the rate of b Rate = −0.4 kg/m


change. Include appropriate units. The weight is decreasing with respect to
length at a rate of 0.4 kg/m.
Chapter 12 Rates of change 585
WORKED Example 8
By drawing the graph of y = x2 − 2 find the instantaneous rate of change where x = 1.
THINK WRITE/DISPLAY
1 Use a graphics calculator to draw the graph of
y = x2 − 2 and display a tangent to the curve at the
point where x = 1. Find the gradient of the tangent.
For the Casio fx-9860G AU
1. Press MENU and then select GRAPH . Press
SHIFT F3 (V-WIN) to adjust the View Window
settings so that Xmin is −5, Xmax is 5, Ymin is −10
and Ymax is 20.

2. Press EXIT to return to the Graph Function


screen and then enter x2 − 2 for Y1. Press EXE .

3. Press F6 (Draw) to display the graph.

4. Press SHIFT [SET UP] and ensure that the setting


for Derivative shows ON. This will allow us to find
the gradient.

5. Press EXIT to return to the graph screen. To find


the tangent at x = 1, press SHIFT F4 (SKTCH)
and then F2 (Tang). Press 1 and a text box
will appear showing X: 1.

6. Press EXE to show the point on the graph at


(1, −1). The gradient at the point is also displayed
as dY/dX = 2.

7. Press EXE again to display the graph with the


tangent drawn at x = 1. The equation of the tangent
is y = 2x − 3.

Continued over page


586 M a t h s Q u e s t M a t h s B Ye a r 1 1 f o r Q u e e n s l a n d

THINK WRITE/DISPLAY

For the TI-Nspire CAS


1. Open a new Graphs & Geometry document.
Adjust the Window Settings (press b and then
select 4: Window followed by 1: Window Settings)
so that XMin is −5, XMax is 5, YMin is −10 and
YMax is 20.

2. Enter x2 − 2 for f1(x) and then press · to display


the graph. Move the label for the rule if you wish.

3. To place a point on the curve, press b and then


select 6: Points & Lines followed by 2: Point On.
Move the pencil (✎) to the line and when any point
is revealed, press · and then d.

4. To change the x-coordinate of the point to 1, move


the pointer (å) until it hovers over the x-value of the
point. When it changes into an open hand (÷), press
the Click button (x) twice. A text box appears.
Delete the existing x-value and type in 1. Press ·.
The point (1, −1) will be displayed in the correct
position.
5. To draw the tangent to this point, press b and
then select 6: Points & Lines followed by 7:
Tangent. Move the pointer (å) to the point and then
press ·.

6. To find the gradient of the tangent, press b and


then select 7: Measurement followed by 3: Slope.
Move the pointer to the line (it will start to flash)
and then press ·. The gradient is 2.
Chapter 12 Rates of change 587

THINK WRITE/DISPLAY

7. If you wish to obtain the equation of the tangent,


press b and then select 1: Actions followed by
7: Coordinates and Equations. Move the pointer to
the line (it will start to flash) and then press ·.
Press the Click button (x) to anchor the equation in
position. The equation of the tangent is y = 2x − 3.

2 State the gradient of the tangent either from the The gradient of the tangent is 2.
derivative or slope value found, or from the
equation of the tangent.

3 Use the gradient to state the rate of change. Hence, the instantaneous rate of
change at x = 1 is 2.

remember
1. The instantaneous rate of change is the rate of change at a particular instant
rather than over an interval.
2. Instantaneous rate of change can be found by calculating the gradient of the
tangent drawn to the curve at the required point.

12D Instantaneous rates

1 Which of the following graphs have a tangent drawn at the point where x = 4?
a y b y c y

0 x 0 x 0 x
4 4 4

d y e y f y

0 x 0 x 0 x
4 4 4
588 M a t h s Q u e s t M a t h s B Ye a r 1 1 f o r Q u e e n s l a n d

g y h y

0 x 0 x
4 4

WORKED 2 In each of the following graphs find an approximate value for the gradient of the
Example
7a
tangent at the point indicated.
ay b y cy d y
x=1 20 x=2 x = 1.7 x=4
3 5 22
20
2
10
1 2

0 x 0 x 0 x 0 x
1 2 2 4 1 1.7 4 4 8

WORKED 3 Find the approximate value for the instantaneous rate of change of weight with respect
Example
7b
to time, at the time indicated on the graphs below.
a b c d
w (g) w (g) w (g) w (g)
130 t = 20 160 t = 10 140 t=4 t=1
110

70
80 50
40

0 0 0
10 20 30 t (min) 5 10 15 t (min) 2 4 6 t (min) 0 1 5 t (min)

4 a Plot the graph of y = x2 from x = 0 to x = 4.


b Draw tangents at the points where x = 1, x = 2 and x = 3.

c multiple choice
The gradient of the tangent at x = 1 is closest to:
A 0 B –1 C 2 D 3 E 4

d multiple choice
The gradient of the tangent at x = 2 is closest to:
A 8 B 6 C 4 D 2 E 0

e multiple choice
The gradient of the tangent at x = 3 is closest to:
A 6 B 9 C 3 D 7 E −3

WORKED 5 a Draw the graph of y = 9 − x2 from x = −4 to x = 4.


Example
b Draw tangents at the points x = −2, x = 0 and x = 2.
8
c Hence, estimate the instantaneous rate of change at the points:
i x = −2 ii x = 0 iii x = 2.
Chapter 12 Rates of change 589
6 Describe each of the rates below as either an
instantaneous rate (I) or an average rate (A).
a Bill walked 12 kilometres in 2 hours.
b An aircraft leaves the runway at 270 km/h.
c A household used 560 litres of water in one
day.
d The pulse rate of a runner as he crosses the
finish line
e A gas heater raises the temperature of a
room by 10°C in half an hour.
f A baby put on 300 g in one week.
g A road drops 20 m over a distance of 100 m.
h Halfway along a flying fox, Jill is travelling
at 40 km/h.
i The maximum speed of a power drill is 320
revolutions per minute.
j Water flows through a fire hose at 60 litres
per minute.
7 A tank is being filled at a variable rate. The
eBook plus depth of the water, H cm, at any time,
t minutes, is described by the rule H = t2 + 2t.
Digital doc:
EXCEL Spreadsheet At what rate is the depth of water changing after 2 minutes?
Instantaneous rates
8 The weight of a person over a 40-week 90
period is illustrated in the graph at right. 80
a Estimate the maximum weight and the
Weight (kg)

70
time at which it occurs.
60
b Estimate the average rate of change of
weight between week 10 and week 20. 50
c Estimate the rate of change of weight 40
after 30 weeks. 0 10 20 30 40
Time (weeks)
9 The change of temperature in a restaurant
kitchen over a 12-hour period is illustrated
by the graph below.

35
30
Temperature (°C)

25
20
15
10
5
0
0 1 2 3 4 5 6 7 8 9 10111213
Time (hours)
a When is the rate of temperature rise greatest?
b Estimate the rate of temperature rise after 6 hours.
590 M a t h s Q u e s t M a t h s B Ye a r 1 1 f o r Q u e e n s l a n d

Spanner falling
Returning to the problem of the spanner falling from a high tower — we are inter-
ested in the speed of the spanner as it travels, particularly when it hits the ground.
Earlier we saw how a graph can be used to plot distance travelled versus time.
Now we are in a position to calculate the instantaneous speed of the spanner at any
time.

50

40

d (metres)
30

20

10

0
0.5 1.0 1.5 2.0 2.5 3.0 3.5 4.0 4.5 5.0 5.5
t (seconds)
Using the techniques developed in Exercise 12D, we can calculate the instantaneous
speed of the spanner.
The tangent drawn at t = 2 seconds passes through (2, 20) and (3, 40).
( 40 – 20 )
The gradient of the tangent is ---------------------- = 20 m/s.
(3 – 2)
After 2 seconds the spanner is falling at 20 m/s.

50

40
d (metres)

30

20

10

0
0.5 1.0 1.5 2.0 2.5 3.0 3.5 4.0 4.5 5.0 5.5
t (seconds)
Using the tangent drawn at t = 3 seconds, calculate the speed of the spanner as it hits
the ground.

Motion graphs
In this section we will consider only objects moving in straight lines, either right and
left or up and down.
Definitions
1. Position, x, describes where an object is or was.
2. Distance, d, is how far an object has travelled.
3. Displacement, s, describes the change in an object’s position; that is,
displacement = change in position = final position − initial position
or s = xfinal − xinitial.
Chapter 12 Rates of change 591
distance d
4. Speed = ------------------------- or speed = --- .
time taken t
5. Velocity is the rate of change of position with respect to time, so
change in position displacement s
velocity = -------------------------------------------- = ------------------------------- ; that is, v = - .
change in time time taken t
Consider an object that begins at the –3-metre mark on a number line, moves
5 metres to the right, then 5 metres to the left, taking 2 seconds to do so.
Then, for this example we have:
1. Position (initially and finally) = −3 m –4 –3–2–1 0 1 2 3 4 x
2. Distance = 10 m
3. Displacement = 0 m
10 m
4. Speed = ------------ = 5 m/s
2s
0m
5. Velocity = --------- = 0 m/s
2s
Distance and speed are (technically) always positive. Displacement and velocity can
be either positive or negative, depending on the direction of motion.
(Note: Some texts use displacement and position interchangeably, perhaps assuming
displacement from a fixed origin.)

WORKED Example 9 t = 14 s
Top
t = 20 s
Consider a lift starting from the ground floor moving Positive
up to the top floor, stopping, then coming down to the
5th floor at the times shown on the diagram. 100 m

Find: 150 m
a the total distance travelled by the lift
b the displacement of the lift after 25 s t = 25 s
(5th floor)
c the average speed of the lift
d the average velocity of the lift.
t=0 Ground floor

THINK WRITE
a Add the distance travelled up (150 m) to the a Total distance = 150 m + 100 m
distance travelled down (100 m). = 250 m

b At t = 0 s, position of the lift is 0 m. b Displacement = change in position


At t = 25 s, position is +50 m. Displacement = +50 – 0
Displacement = +50 m

total distance travelled 250 m


c Average speed = ------------------------------------------------------ c Average speed = ---------------
time taken 25 s
= 10 m/s

change in position ( +50 – 0 ) m


d Average velocity = -------------------------------------------- d Average velocity = -----------------------------
change in time 25 s
= +2 m/s
592 M a t h s Q u e s t M a t h s B Ye a r 1 1 f o r Q u e e n s l a n d

WORKED Example 10
Use this position–time graph, which shows the position of a x (m)
particle, x, travelling in a horizontal straight line at any
20
time, t, to find:
a where the journey started 10
b in which direction the particle moved initially 5
c when and where the particle changed direction
d when and where the particle finished its journey. 0
2 4 t (s)

THINK WRITE

a Find s when t = 0. a When t = 0, s = 10 m. The journey started 10


metres from the origin or reference point.
b Compare the position at t = 0 to the b As the position changes from 10 m at t = 0 s,
position a short time after this. Notice to 5 m at t = 2 s, the particle is moving to the
that the graph has a negative gradient left initially.
during this time.
c Find x and t where the graph turns or c The particle changes direction at t = 2. When
changes direction. Notice that the t = 2, x = 5.
instantaneous gradient is zero at this After 2 s at a position of 5 m, the particle
point. changed direction.
d Find x and t where the curve stops. d When t = 4, x = 20. The particle finished its
journey at a position of 20 m after 4 s.

WORKED Example 11
Using the graph in Worked example 10, consider the interval between t = 2 and t = 4 to
decide whether:
ii the velocity of the particle is positive or negative
ii the particle is speeding up or slowing down.
THINK WRITE
rise
ii 1For this graph, -------- (or gradient) is the ii
run
change in position
same as -------------------------------------------- (or velocity).
change in time
So the gradient of a position–time graph
gives the velocity.
2 Consider the gradient between t = 2 From t = 2 to t = 4, the gradient is positive so
and t = 4. the velocity of the particle is positive.
ii Consider how the gradient between t = 2 ii The gradient is increasing so the velocity is
and t = 4 is changing. increasing. This means the particle is
speeding up.
Chapter 12 Rates of change 593
Similarly, for the interval between t = 0 and t = 2 in Worked example 10:
1. the velocity is negative because the gradient is negative
2. the velocity is increasing because the gradient is increasing (from a ‘larger’ negative
value to a ‘smaller’ negative value)
3. the speed is decreasing between t = 0 and t = 2 because the magnitude of the
gradient is decreasing.
Positive gradient
x getting steeper
Negative gradient
20 getting less steep

10
5

0 t
2 4

remember
1. Position, x, describes where an object is or was.
2. Distance, d, is how far an object has travelled.
3. Displacement, s, describes the change in an object’s position; that is, s = xf – xi.
distance d
4. Speed = ------------------------- or speed = --- .
time taken t
displacement s
5. Velocity = ------------------------------- or v = - .
time taken t
The gradient of a position–time graph for an object gives the velocity of that
object.

12E Motion graphs

1 Match the correct description (A, B, C or D) to each of the quantities (a, b, c or d) below.

Quantity Description
a Distance A Rate of change of position with respect to time
b Displacement B Change in position
c Speed C Length travelled
d Velocity D Distance travelled with respect to time

2 State whether each of the following is TRUE or FALSE.


a Speed can be positive or negative.
b Displacement can be positive or negative.
c Distance travelled is positive only.
d Velocity is positive only.
594 M a t h s Q u e s t M a t h s B Ye a r 1 1 f o r Q u e e n s l a n d

WORKED 3 A parachute ride takes people in a t = 55 s t = 60 s


Example
basket vertically up in the air from a
9
platform 2 metres above the ground,
then ‘drops’ them back to the ground.
Use the illustration showing the pos-
ition of the parachute basket at various eBook plus
times to find:
a the total distance travelled by the 19 m
Digital doc:
SkillSHEET 12.2
parachute basket during a ride Displacement
b the displacement of the parachute and distance

basket after 80 s
c the average speed of the parachute
basket during the ride
d the average velocity of the para-
chute basket during the ride.
t=0
2 m Platform
t = 80 s Ground

4 Consider the position and direction, t = 5 t=2


at various times, of a particle t=0
x
travelling in a straight line as –3 –2 –1 0 1 2 3 4 5 6 7
indicated at right.
a Where does the particle start?
b Where does the particle finish?
c In which direction does the particle move initially?
d When does the particle change direction?
multiple choice
e Which position–time graph below most accurately represents the particle’s motion?

A x (m) B x (m) C x (m)


6 6 6
5 5 5
4 4 4
3 3 3
2 2 2
1 1 1
0 0 0
–1 2 5 t (s) –1 2 5 t (s) 2 5 t (s)
–1
–2 –2 –2
–3 –3 –3

D x (m) E x (m)
6 6
5 5
4 4
3 3
2 2
1 1
0 0
2 5 t (s) 2 5 t (s)
Chapter 12 Rates of change 595
f The total distance travelled in the first 5 seconds is:
A 4m B 13 m C 9m D 14 m E 11 m

g The displacement of the particle after 5 seconds is:


A −3 m B 14 m C 4m D −4 m E 0m

h The average speed in the first 2 seconds is:


A 3 m/s B −2.5 m/s C 6 m/s D 2.5 m/s E 1 m/s

i The average velocity between t = 2 and t = 5 is:


A 3 m/s B −2 m/s C −3 m/s D 2 m/s E 1 m/s

j The instantaneous speed when t = 2 is:


A 2.5 m/s B 0 m/s C 3 m/s D 2.8 m/s E 1 m/s

WORKED 5 The following position–time graphs show the journey of a particle travelling in a
Example
10
straight line. For each graph find the following:
i where the journey started
ii in which direction the particle moved initially
iii when and where the particle changed direction
iv when and where the particle finished its journey.

a x (m) b x (m) c x (m)


12 12
8
10 10

4
0 3
1 2 3 4 5 t (s)
–3 0 0
1 2 3 4 5 6 t (s) 1 2 3 4 5 6 7 8 t (s)

d x (m) e x (m) f x (m)


18
5 5
4
2
0 0
1 2 3 4 5 t (s) 1 2 3 4 5 6 t (s)
–3

0 –5
1 2 3 t (s)

–5
596 M a t h s Q u e s t M a t h s B Ye a r 1 1 f o r Q u e e n s l a n d

6 For each position function of a particle given below, sketch the position–time graph.
In each case explain:
i where the particle started its journey
ii in which direction it moved initially
iii whether the particle changed its direction and, if so, when and where that happened
iv where the particle finished its journey.
a x(t) = 2t, t ∈ [0, 5]
eBook plus b x(t) = 3t − 2, t ∈ [0, 6]
Digital doc:
c x(t) = t2 − 2t, t ∈ [0, 5]
SkillSHEET 12.3 d x(t) = 2t − t2, t ∈ [0, 4]
Interval notation
e x(t) = t2 − 4t + 4, t ∈ [0, 5]
f x(t) = t2 + t − 12, t ∈ [0, 5]

7 a Plot the position–time graph for x(t) = 4t − t2.


b Find the gradient at:
vi t = 0 ii t = 1 iii t = 2
iv t = 3 v t = 4.
c Hence, give the instantaneous rate of change of position with respect to time (that
is, velocity) at:
vi t = 0 ii t = 1 iii t = 2
iv t = 3 v t = 4.
d Sketch the velocity–time graph from t = 0 to t = 5.

WORKED 8 For each of the following position–time graphs determine:


Example
11 i whether the velocity is positive or negative
ii whether the particle is speeding up or slowing down.

a x b x c x

0 t 0 0
t t

d x e x

0 t 0 t
Chapter 12 Rates of change 597
9 Match the following position–time graphs with the corresponding velocity–time graphs
below.
Position–time graphs
a x (m) b x (m) c x (m)

3 3 3
2 2 2
1 1 1
0 0 0
1 2 3 4 t (s) 1 2 3 4 t (s) 1 2 3 4 t (s)
–1 –1 –1
–2 –2 –2
–3 –3 –3
d x (m) e x (m) f x (m)
4
3 3 3
2 2 2
1 1 1

0 0 0
1 2 3 4 t (s) 1 2 3 4 t (s) –1 1 2 3 4 t (s)
–1 –1
–2
–2 –2
–3
–3 –3
–4

Velocity–time graphs
A v (m/s) B v (m/s) C v (m/s)

0 0 0
1 2 3 4 t (s) 1 2 3 4 t (s) 1 2 3 4 t (s)

D v (m/s) E v (m/s) F v (m/s)

0 0 0
1 2 3 4 t (s) 1 2 3 4 t (s) 1 2 3 4 t (s)
–1

10 A ball is projected vertically upwards from the top of a building 25 m high. Its
position relative to the ground is given by the equation x = 25 + 20t − 5t2 where t is
the time in seconds.
Sketch a position–time graph for the ball and hence find:
a the greatest height reached
b when the ball reaches the ground
c when the velocity of the ball is zero
d an estimate for the velocity at which the ball is initially projected.
598 M a t h s Q u e s t M a t h s B Ye a r 1 1 f o r Q u e e n s l a n d

11 The diagram shows the distance–time graph of


two swimmers in a 200-metre race.
a Who won the race? 200

Distance (metres)
b Who was in front after 150 metres? 150
Alan
c What was the approximate winning margin 100
(in metres)?
50 Bill
d What was the average speed of the winner?
e For approximately how many seconds did 0 20 40 60 80 100 120
Bill lead the race?
Time (seconds)
f How many times did the lead change?
eBook plus

Digital doc:
WorkSHEET 12.2

Relating the gradient function to the


original function y
The gradient of the function at right has been 6 y = x3 – 2x2 + 2
calculated at several points as shown, and a table of
4
gradient values has been constructed.
Gradient = 7 2
x −1.0 −0.5 0 0.5 1.0 1.5 2.0
–2 –1 0 1 2x
–2
Gradient 7 2.75 0 −1.25 −1 0.75 4

A graphics calculator or graphing software may be used as an y


alternative to hand sketching tangents and calculating gradients. 8
You may also like to log into www.jacplus.com.au and locate 6
4
the weblinks for this chapter to use the files provided. 2
The table of values above may be used to sketch a gradient –2 –1 –2 0 1 2x
function such as that at right.
Chapter 12 Rates of change 599

Graphics Calculator tip! Relating the gradient function


to the original function
Consider the function y = x3 – 9x. We can use a graphics calculator to calculate the
gradient of the function at a number of points, create a table and then draw both func-
tions on the one set of axes.
For the Casio fx-9860G AU
1. Press MENU and then select GRAPH. Press
SHIFT F3 (V-WIN) to adjust the View
Window settings so that Xmin is −5, Xmax is 5,
Ymin is −15 and Ymax is 15.

2. Press EXIT to return to the Graph Function


screen and then enter x3 − 9x for Y1. Press
EXE .

3. Press F6 (Draw) to display the graph.

4. To create a table of values from the equation,


press MENU and then select TABLE. The
equation for Y1 will be shown there. To choose
the x-values, press F5 (SET) and change the
table settings to show Start as −5, End as 5 and
Step as 1.

5. To include the derivative (or gradient) in the


table, press SHIFT [SET UP] and ensure that the
setting for Derivative shows ON. Press EXIT
to return to the Table Function screen. Press
F1 (SEL) and then press F6 (TABL) to
display the table of values. The third column
labelled Y’1 lists the values of the gradient.

6. With the cursor in the first column, press OPTN


and then F1 (LMEM). Press 1 to store the
column values in List 1 and then press EXE .
Now move the cursor to the third column and
press F1 (LMEM). Press 2 to store the third
column values in List 2.
600 M a t h s Q u e s t M a t h s B Ye a r 1 1 f o r Q u e e n s l a n d

7. Press EXE followed by MENU and then select


STAT. The screen will display the x-values in
List 1 and the gradient values in List 2.

8. To plot the points, press F1 (GRPH) and then


F6 (SET). Ensure the Graph Type is set as
xyLine. Press EXIT and then F1 (GPH1) to
display the graph of the gradient function.

9. To save the picture, press OPTN and then F1


(PICT) followed by F1 (STO). To store this as
picture 1, press 1 .

10. Press EXE to complete the storage procedure.


To see the gradient function with the original
function, press MENU and then select GRAPH.
The original equation will still be Y1. Press F6
(Draw) to display the original curve. Press
OPTN and then F1 (PICT) followed by F2
(RCL). Press 1 for the picture number. Press
EXE to draw the saved picture on the same
axes.

11. Press SHIFT F1 (TRCE) and then use the


arrow keys to move along the graph of the
function. The x- and y-coordinates, as well as the
gradient (dY/dX), will be displayed at points
along the curve.

For the TI-Nspire CAS


1. Open a new Graphs & Geometry document.
Adjust the Window Settings (press b and then
select 4: Window followed by 1: Window
Settings) so that XMin is −5, XMax is 5, YMin is
−15 and YMax is 15. Enter x3 − 9x for f1(x) and
then press · to display the graph.
Chapter 12 Rates of change 601
2. As before, place a point on the line. Press b
and then select 6: Points & Lines followed by
2: Point On. Move the pencil (✎) to the line.
When any point is revealed, press · and then
d.

3. To draw the tangent to this point, press b and


then select 6: Points & Lines followed by
7: Tangent. Move the pointer (å) to the point
and then press ·.

4. Press b and then select 7: Measurement


followed by 3: Slope. Move the pointer to the
line (it will start to flash) and then press ·.
Press the Click button (x) to anchor the
gradient value in position.

5. Press d. Move the pointer to the coordinates of


the point and click (press x) on the x-coordinate.
Pressing /h allows you to store this as a
variable.

6. Enter xval in place of var and then press ·.


Now move the pointer to the gradient value and
click to highlight it. Press /h and enter
slope in place of var. Then press ·.

7. Press /I and then insert a new Lists &


Spreadsheet document. With the cursor in any
cell of column A, press b. Select 3: Data then
2: Data Capture followed by 2: Manual Data
Capture.
602 M a t h s Q u e s t M a t h s B Ye a r 1 1 f o r Q u e e n s l a n d

8. Press h and select xval. Move the cursor to the


far right of the expression and then press ·.
Now repeat this process for column B. Select
slope to replace var. You may like to resize the
column width.

9. Return to the Graph screen (press / and then


the left arrow on the NavPad) and move the
pointer until the open hand (÷) is shown over the
point.

10. Press /x to close the hand over the point.


Now move the point a small way along the
curve. Press /^ to record a data point in the
spreadsheet. Repeat this procedure for small
steps along the curve to generate a set of
coordinates (xval, slope). Note that these two
variables are shown in bold type.

11. You can return to the spreadsheet window to


examine the data (press / and then the right
arrow on the NavPad). Label column A with
xvals and column B with yvals.

12. Return to the Graph screen. With the cursor in


the entry line for f2(x), press b and then select
3: Graph Type followed by 4: Scatter Plot.
Press x and then select xvals for x. Press e to
move to the entry box for y. Press x and then
select yvals for y. The scatterplot for the gradient
function will be displayed with the original
function on the same set of axes.
Chapter 12 Rates of change 603
Relating the gradient function
12F to the original function
1 Select several points (at least 6) on each of the graphs below, and construct a table of
eBook plus gradient values. Use each table to plot the gradient function in each case.
You may wish to copy, paste and enlarge graphs from this page using the electronic
Digital docs:
EXCEL Spreadsheet
version, accessed by logging into www.jacplus.com.au and locating the weblinks for this
Gradient at a point chapter. You can then plot gradient functions on the same axes as the original functions.
EXCEL Spreadsheet
Plotting relations a y y = x2 – 4 b y
4 4
y = 2x + 3
2 2

0 x –3–2 –1 0 1 2 3 x
–2–1 1 2
–2 –2

–4 –4

c y d y
6 6

4 4

2 2 y = 2x

–3–2–1 0 1 2 x –3 –2 –1 0 1 2 3
x
–2
y = x3 + 3x2 + 2

e y f y
2 πx 40
y = sin —–
4 32
1 24
16 y = x4
0 1 2 3 4 5 6 7 8x 8
–1 –2–1 0 1 2 x

2 What do you notice about the gradient function of the quadratic function in question 1 a?
3 Investigate the gradient functions of other cubics besides that shown in question 1 c.
What type of functions are the gradient functions of cubics?
4 Which functions have a gradient function similar to the original function?
5 What do you notice about the original functions at x-values where the gradient function
intersects the x-axis?
6 Consider the so-called ‘square wave’ signal produced by electronic devices:

2 4 6 8 t (s)
–1

If this were the gradient function for another function, f(x), sketch the graph of f(x).
604 M a t h s Q u e s t M a t h s B Ye a r 1 1 f o r Q u e e n s l a n d

Relating velocity–time graphs to


position–time graphs
The gradient of a position–time graph gives the velocity because velocity is the rate of
change of position with respect to time.
Therefore, by measuring the gradient of a position–time graph at various points, a
velocity–time graph can be derived.

WORKED Example 12
The position–time graph for a particle moving in a straight line is shown below.

x
3
2
1 Gradient = 2
Gradient = –2
0 t
1 2 3 4 5
–1
Gradient = 1
–2
Gradient = 0
Gradient = –1

The gradient of the curve at various times is indicated on the graph.


Use this information to draw a velocity–time graph for the particle.

THINK WRITE
1 Set up a table of corresponding velocity t 0 1 2 3 4
and time values from the graph.
v −2 −1 0 1 2

2 Use the table of values to plot the v


velocity–time graph. 3
2
1

t
–3 –2 –1 0 1 2 3 4 5
–1
–2

Similarly, a position–time graph can be derived from a velocity–time graph.


The value of the velocity at a given time is equal to the gradient of the position–time
graph at that time. By obtaining several of these values, and knowing the initial position
of a particle, the general shape of the position–time graph can be produced.
The following example demonstrates the steps involved.
Chapter 12 Rates of change 605
WORKED Example 13
The velocity–time graph for a particle moving in a v
straight line and starting at the origin is shown in 4
the diagram. Sketch the corresponding position–time 3
graph. 2
1
0 1 2 3 4 5 t
–1
–2
–3

THINK WRITE
1 Set up a table of corresponding velocity
and time values. (These velocity values t 0 1 2 3 4
represent the gradient of the position–time
v 4 2 0 −2 −4
curve at the given times.)
2 Set up the axes for the position–time x
graph.
3 Draw in a curve starting at (0, 0) with a
gradient of 4 decreasing to a gradient of
0 at t = 2 (turning point). From t = 2 to
t = 4, the gradient changes from 0 to −4.
This means the curve will become steeper
but with a negative slope. 0 t
1 2 3 4 5

remember
1. The gradient of a position–time graph gives the velocity of an object at any
given time.
2. To draw a velocity–time graph (or gradient function) from a position–time
graph (or original function):
(a) calculate the gradient of the graph at given time values (for an x–t graph
this gives us the velocity)
(b) construct a table of values showing the velocity (or gradient) at given time
values (or x-values)
(c) use this table to plot the velocity–time graph (or gradient function).
3. To draw a position–time graph (or original function) from a velocity–time
graph (or gradient function):
(a) set up a table of corresponding velocity (gradient) and time values
(x-values) from the graph
(b) on a set of axes, plot a point for the initial position. Starting from this point,
use the values in the table to estimate the appropriate relative slope at each
given time (or x-value) to produce a curve.
606 M a t h s Q u e s t M a t h s B Ye a r 1 1 f o r Q u e e n s l a n d

Relating velocity–time graphs


12G to position–time graphs
WORKED 1 For each position–time graph for a particle moving in a straight line, draw a velocity–
Example
12
time graph. The gradient of the curve at various times is indicated on each of the
graphs.
a x b x

Gradient = 0

Gradient = –3 Gradient = 3
Gradient = –2
0 t
1 2 3 4 5 6
Gradient = 2 0 t
1 2
Gradient = –2 Gradient = 1 Gradient = 2
Gradient = –1 Gradient = 0

2 multiple choice m=0


x
The gradients are shown at various points on a
position–time graph. m = –6
m=6

m = 12 m = –12

0 t
1 2 3 4

The velocity–time graph obtained from this position–time graph is:


A v B v C v
12 12 12

0 0 0
t t 1 2 3 4 t
1 2 3 4 1 2 3 4

–12 –12

D v E v
12 12

0
1 2 3 4 t
0 t
1 2 3 4

–12
Chapter 12 Rates of change 607
3 Sketch the velocity–time graph for each of the following position–time graphs.

a x b x c x d x
10 8
1

0 t
1 2
5 –1

0 t
1 2 3 4
0 t
1 2 3 4

0 t
1 2 3

–6

–5 –8
–8
WORKED 4 For each velocity–time graph shown below, sketch a position–time graph, given that
Example
13
the particle starts at the origin.
a v b v c v
3 –3

0 t 0 t 0 t
4 4 –1 1 4
–2

d v e v f v
8 3
3

0 t 0 t 0 t
–1 3 4 2 4 1 2 3 4

–8 –3

5 A particle starting at the origin moves with velocity v(t) = 2t − 4.


a Complete the table below.

t 0 1 2 3 4
v −4 −2 0 2 4
Gradient of x–t graph

b Sketch the position–time graph.


608 M a t h s Q u e s t M a t h s B Ye a r 1 1 f o r Q u e e n s l a n d

6 The velocities for a particle starting at the origin are given as a function of time below.
Sketch a position–time graph for each using t ∈ [0, 4].
av=t+2 b v=2−t c v = 3t d v = −t
v

3
7 multiple choice
A position–time graph is obtained from the
velocity–time graph shown at right. For a particle 0 t
1 2 3 4
starting at the origin, this position–time graph will
look like:
–3

x A x B x C x D x E

2
0 t 0 t 0 t 0 t 0 t
2 4 2 4 2 4 2 4 2 4
–1

8 Briefly describe the motion of a car during the first 30 seconds for each of the
velocity–time graphs below.
v (m/s) a v (m/s) b v (m/s) c
25
20
18

0 0 0
30 t (s) 15 30 t (s) 10 25 30 t (s)

9 Sketch a velocity–time graph for each car described below for its first 20 seconds of
motion.
a A car, initially at rest, increases its speed at a constant rate until it reaches a speed
of 35 m/s after 10 s. It then maintains this speed.
b A car is travelling at a constant speed of 30 m/s. After 15 seconds the brakes are
applied, decreasing its speed at a constant rate until it stops after another 5 seconds.
10 A parachutist jumps from an aircraft and free-
falls for 6 seconds.
a If the parachutist falls y metres in t seconds
where y = 5t 2, find the average speed of the
parachutist between:
ii t = 0 and t = 3
ii t = 3 and t = 6.
b What is the speed of the parachutist after
6 seconds of freefall?
c When the parachute is released (6 seconds
after freefall), the speed of the parachutist is
reduced by 2 m/s every second until a speed
of 4 m/s is reached. How long after jumping
from the aircraft does it take the parachutist
to reach a speed of 4 m/s?
Chapter 12 Rates of change 609
Rates of change of polynomials
We have seen that instantaneous rates of change can be found from a graph by finding
the gradient of the tangent drawn through the point in question. However, this method
is not likely to be very accurate if the tangent is drawn by hand. A more reliable method
of finding instantaneous rates of change is outlined in the following worked examples.

WORKED Example 14
a Plot the graph of y = 2x2, x ∈ [0, 4].
b Draw a line through the following points on the curve and find the gradient of each line.
i x = 2 and x = 3
ii x = 2 and x = 2.1
iii x = 2 and x = 2.01
iv x = 2 and x = 2.001
c Hence, predict the gradient of the tangent at the point on the curve where x = 2.

THINK WRITE

a 1 Use a table of values or graphics a


calculator. x 0 1 2 3 4
y 0 2 8 18 32

2 Plot the points using graph paper. y


32
i
ii
18 iii
iv

0 x
1 2 3 4

b i 1 Mark the points on the curve b i Line i:


where x = 2 and x = 3 and rule a
line through them.

18 – 8
2 Select suitable values to calculate Gradient = ---------------
3–2
rise 10
the gradient using -------- . = ------
run 1
When x = 2, y = 8; = 10
when x = 3, y = 18.
Continued over page
610 M a t h s Q u e s t M a t h s B Ye a r 1 1 f o r Q u e e n s l a n d

THINK WRITE/DISPLAY

ii 1 Again, mark the points and rule a ii Line ii


line through them.
2 Select suitable points to calculate the
gradient. On a graphics calculator,
make Xmin = –4.7, Xmax = 4.7, and
TRACE to x = 2 and x = 2.1.
Alternatively, calculate the
y-value for x = 2.1 using the rule
y = 2x2. When x = 2.1, y = 2(2.1)2 = 8.82
8.82 – 8
Gradient = -------------------
2.1 – 2
0.82
= ----------
0.1
= 8.2
iii 1 Again, mark the points and rule a iii Line iii
line through them.
2 Select suitable points to calculate the When x = 2.01, y = 2(2.01)2 = 8.0802
gradient. This may be difficult so Gradient = 8.0802 –8
-------------------------
select an x-value and calculate the 2.01 – 2
y-value using the rule y = 2x2. 0.0802
= ----------------
0.01
= 8.02
iv 1 Again, mark the points and rule a iv Line iv
line through them.
2 Select suitable points to calculate the When x = 2.001, y = 2(2.001)2
gradient. Calculate the y-value for = 8.008 002
x = 2.001 using the rule y = 2x2. 8.008 002 – 8
Gradient = --------------------------------
2.001 – 2
0.008 002
= -----------------------
0.001
= 8.002

c Continue to bring the second x-value closer c The gradient of the tangent at the point on
and closer to 2. As the second x-value the curve where x = 2 appears to be 8.
becomes closer to 2, the gradient of the line
is becoming closer to 8. Notice that the
slope of each line has progressively
become closer to the slope of the tangent at
x = 2.
Chapter 12 Rates of change 611
Investigating rates of change
1 Repeat Worked example 14 using a graphics calculator. Plot the graph of
y = 2x2 and draw the lines through the pairs of points indicated. Investigate how
the gradient of each line moves closer to the value of 8 as the x-value of the
second point moves closer to 2.
2 Use this method to predict the gradient of the tangent to the curve y = x3 + 2 at
the point where x = 1.
3 Explain how this method allows you to predict the gradient of the tangent to a
curve at a given point and hence the rate of change.
4 Could this method of calculating gradients of lines joining two points be used
without drawing a graph? Explain.

The previous example demonstrates that the gradient of a tangent to a function at a


given point, x = a, can be approximated by finding the gradient of a line joining the
point x = a and another point as close as possible to x = a. If the coordinates of the two
points can be found using the rule, the gradient can be calculated without using a graph.

WORKED Example 15
If d(t) = t2 − 2t, find the rate of change of d when t = 4.

THINK WRITE

1 The rate of change is the gradient. For the line joining t = 4 and t = 4.01:
Consider the gradient of a line joining
the point t = 4 and another point close
to t = 4 on the curve.
2 Evaluate d(4) and d(4.01). d(4) = 8 and d(4.1) = 8.0601

3 Find the gradient of the line joining Gradient = 8.0601 –8


-------------------------
t = 4 and t = 4.01. 4.01 – 4
0.0601
= ----------------
0.01
= 6.01
4 Pick a value closer still to t = 4, say For the line joining t = 4 and t = 4.001:
t = 4.001 and evaluate d(4.001). d(4.001) = 8.006 001
5 Find the gradient of the line joining
t = 4 and t = 4.001. Gradient = 8.006 001 – 8
--------------------------------
4.001 – 4
0.006 001
= -----------------------
0.001
= 6.001
6 Estimate the gradient of the tangent at When t = 4, the gradient appears to be 6 so the
t = 4. (Further gradient calculations can rate of change of d is 6.
be made using a second value of t that is
still closer to 4 to improve the estimate.)
612 M a t h s Q u e s t M a t h s B Ye a r 1 1 f o r Q u e e n s l a n d

remember
1. The gradient of a tangent to a function at a given point, x = a, can be
approximated by finding the gradient of a line joining the point x = a and a
point as close as possible to x = a. The closer the two points are, the better the
estimate of the gradient.
2. The gradient of a function gives us the rate of change of the function.

Rates of change of
12H polynomials
WORKED 1 a Plot the graph of y = x2, x ∈ [0, 4].
Example
b Draw a line joining the following points on the curve and find the gradient of
14
each line.
iii x = 2 and x = 4 iii x = 2 and x = 3
iii x = 2 and x = 2.5 iv x = 2 and x = 2.1
c Hence, predict the gradient of the tangent at the point on the curve where x = 2.
eBook plus
2 a Plot the graph of y = x3, x ∈ [0, 2].
Digital doc: b Draw a line joining the following points on the curve and find the gradient of
EXCEL Spreadsheet
Rates of change each line.
of polynomials iii x = 1 and x = 2 iii x = 1 and x = 1.5
iii x = 1 and x = 1.1 iv x = 1 and x = 1.01
c Hence, predict the gradient of the tangent to the curve where x = 1.
3 a If y = x2 − x + 1, copy and complete the following table of values.

x 3 2 1.5 1.1 1.01 1


y

b Hence, find the gradient of a line joining the points on the curve between x = 1
and:
iii x = 3 ii x = 2 iii x = 1.5
iv x = 1.1 v x = 1.01.
c Predict the gradient of the tangent at the point on the curve where x = 1.

4 multiple choice
The gradient of the tangent at point P on the curve at right y T Q
is best approximated by the gradient of the line joining S R
P to point: P

A O B R C S D T E Q
0 x
5 multiple choice
The gradient of a tangent to a curve at the point where x = 3 is best approximated by
the gradient of the line joining the points on the curve where:
A x = 3 and x = 4 B x = 2 and x = 4 C x = 3 and x = 3.2
D x = 3 and x = 3.01 E x = 3 and x = 3.001
Chapter 12 Rates of change 613
6 multiple choice
The rate of change at a point, P, on a curve is best approximated by finding the
gradient of a line joining the point, P, and another point which is:
A furthest from it B closest to it C anywhere on the curve
D the origin E on the x-axis
WORKED 7 Without sketching a graph, predict the gradient of the following functions at the point
Example
15
indicated.
a f ( x ) = x2 at x = 1
b f ( x ) = 3x 2 at x = 2
c f ( x) = x2 + 2x at x = 2
d f ( x) = x3 +x at x = 1
e f ( x ) = 2 – 3x – x2 at x = 2
f f ( x) = x3 – 2x + 3 at x = 1.5
g f ( x ) = x 3 + x 2 – 2x at x = 3
h f ( x ) = 1--2- x 2 – 3x + 5 at x = 0.5

8 Hailstones like those in the photo at right will fall


x metres in t seconds where x = 4.9t 2.
Estimate the speed of the hailstone after:
a 2 seconds b 3 seconds.

9 The temperature of a meteor, T °C, t minutes after reaching the Earth’s atmosphere is
T = 2t3 + 5t2 + 200 where t ∈ [0, 10].
Estimate the rate of change of the temperature of the meteor after:
a 1 minute b 10 minutes.

10 The height, h metres, of a roller-coaster is


1 3
given by h = – ------ d 3 + --- d 2 where d is the
80 8
horizontal distance from 0, in metres.
h (m) 50 m

50

0 20 d (m)
20 m
Estimate the gradient of the curve
describing the path of the roller-coaster,
accurate to 2 decimal places, at the point
where d is:
a 10 metres
b 15 metres
c 20 metres
d 0 metres.
614 M a t h s Q u e s t M a t h s B Ye a r 1 1 f o r Q u e e n s l a n d

More on the falling spanner


The problem which stimulated much of the discussion in this chapter relates to a
spanner falling from a high tower. We have been interested in determining how fast
the spanner is travelling when it hits the ground.
In this chapter we learned that the speed of the spanner is not constant but vari-
able, and this change was depicted using a graph of distance travelled versus time.
We also learned how to calculate the instantaneous speed of the spanner at any
time by referring to this graph.
In the next chapter we will examine how a formula relating distance and time can
be used to determine the speed of the spanner at any instant in time.

Career profile
R O G E R FA R R E R — D a t a M a n a g e r
A knowledge of mathematics is vital when
using spreadsheets on a computer. I calculate:
• average growth rates to see which group in
the population is growing the fastest
• fractions and percentages, to find out which
hospital has the largest share of a particular
service (such as heart surgery)
• the average number of times chemotherapy
patients attend radiology sessions (by adding
the number of sessions and dividing by the
number of patients).
Almost all jobs involve mathematics or
interpretation of data or graphs. Without
mathematics, we can be easily fooled or
Qualifications:
misunderstand the actual situation. These
BSc (Statistics), BComm (Economics &
mistakes can be costly or embarrassing. I
Econometrics)
once had to convince someone that the
Employer: population of Melbourne was 3 million, not
Acute Health, Department of Human Services 300 million, because the scale on their graph
was wrong!
I entered this field because I wanted to apply
my statistical and analytical skills to the health Questions
industry. A typical day for me at the moment 1. Why would Roger need to estimate the
includes under-taking projections of patient average growth rates?
numbers for a proposed metropolitan hospital, 2. What is the difference between economics and
estimating the site required for the hospital and econometrics?
considering the services that should be 3. Investigate examples of incorrect graphs in
provided (such as maternity, intensive care, newspapers and explain why each is misleading.
number of beds and so on).
Chapter 12 Rates of change 615

summary
Identifying rates
• A rate describes how much one quantity changes with respect to another.
• For two quantities A and B:
change in A
Rate of change of A with respect to B = amount of A per B = ----------------------------- .
change in B

Constant rates
• The gradient of a graph represents the rate of change. v

• A positive gradient shows a positive or increasing rate of change.


• A negative gradient shows a negative or decreasing rate of change.
• A linear or straight line graph shows that the rate of change is
t
constant.

Variable rates
• A variable rate exists when the rate of change is not constant.
• On a graph, a variable rate of change can be seen when the points do not lie on a
straight line.
d

0 1 2 3 t

Average rates of change


• Over a specified interval:
change in A
1. Average rate of change of A with respect to B = ----------------------------- .
change in B
2. Average rate of change of a function f(t) over the time interval t = a to t = b is:
change in f ( t ) f (b) – f (a)
----------------------------------- = -----------------------------
change in t b–a
y
rise
• When using a graph, find the gradient  -------- of the chord
 run  Rise
formed between the end points of the interval. Run
• When using a table of values or rule, find the change in each x
0
quantity over the specified interval.

Instantaneous rates
• The instantaneous rate of change is the rate of change at a y
particular instant rather than over an interval.
• Instantaneous rate of change can be found by calculating the P Tangent
gradient of the tangent drawn to the curve at the required
point. 0 x
616 M a t h s Q u e s t M a t h s B Ye a r 1 1 f o r Q u e e n s l a n d

Motion graphs
• Position, x, describes where an object is or was.
• Distance, d, is how far an object has travelled.
• Displacement, s, describes the change in an object’s position.
distance d
• Speed = ------------------------- or speed = ---
time taken t
x
displacement s
• Velocity = ------------------------------- or v = -
time taken t
• The gradient of a position–time graph for an object gives
the velocity of that object. 0 t

Relating velocity–time graphs (the gradient function) to position–time


graphs (the original function)
• To draw a velocity–time graph (or gradient function) from a position–time graph
(or original function):
1. Calculate the gradient of the graph at given time values. For an x–t graph, this
gives us the velocity.
2. Construct a table of values showing the velocity (or gradient) at given time
values (or x-values).
3. Use this table to plot the velocity–time graph (or gradient function).
• To draw a position–time graph (or original function) from a velocity–time graph (or
gradient function):
1. Set up a table of corresponding velocity (gradient) and time values (x-values)
from the graph.
2. On a set of axes, plot a point for the initial position. Starting from this point, use
the values in the table to estimate the appropriate relative slope at each given
time (or x-value) to produce a curve.

Rates of change of polynomials


• The gradient of a tangent to a function at a given point, x = a, can be approximated
by finding the gradient of a line joining the point x = a and a point as close as
possible to x = a. The closer the two points are, the better the estimate of the gradient.
Chapter 12 Rates of change 617

CHAPTER
review
1 multiple choice
Which one of the following is a constant rate?
12A
A The number of people entering the zoo per hour
B The number of days it rains in Brisbane per year
C The hourly rate of pay of a tutor
D The number of crates of fruit picked per hour by a fruit-picker
E The number of patients visiting a doctor per day

2 multiple choice w (kg)


70
12A
The graph shows the weight, w kg, of a tank of water
t minutes after water is poured into it.
The rate of change of w in kg/min is:
20
A 1.75 B 0.8 C 2.25
D 1.25 E 2 0
40 t (min)

3 a Determine the rate at which water is leaking from a tank w (kg)


using the graph at right. 12A
840
b How long does it take the tank to empty?
600
4 multiple choice
Which one of the following rates is not variable?
12B
0 8 t (hours)
A The number of pets per family in Melbourne
B The cost per kilogram of apples at your greengrocer today
C The amount of rainfall in Perth per year
D The length of time students need for homework per night
E The number of letters delivered per hour by the mail deliverer

5 multiple choice
y
12B
The graph shown on the right has a negative rate of change
in the interval: P S

A OP B PQ C QR R

D RS E OQ Q
6 a Use the table below to plot the graph of M versus t. x
0
12B
t 0 1 2 3 4
M −9 −7 −1 9 23

b Is the rate of change of M constant or variable? Explain your answer.


c Estimate the gradient when t = 2.
618 M a t h s Q u e s t M a t h s B Ye a r 1 1 f o r Q u e e n s l a n d

7 multiple choice y
12C The gradient of the chord PQ at right is: 21 Q

1
A 3 B 12 C ---
3 P
D 6 E 2 9

0 x
8 multiple choice 5 9
12C From the table below, the average rate of change of H between t = 2 and t = 5 is:

t (h) 0 1 2 3 4 5 6
H (m) 0 20 40 70 120 190 280

A 20 m/h B 38 m/h C 30 m/h D 190 m/h E 50 m/h


9 The amount of substance, A kg, in a container at any time, t hours, is
12C A = t 2 – 3t + 4, t ∈ [ 0, 5 ] .
a Find the average rate of change during the first 4 hours.
b At what rate is the amount changing after 4 hours?

10 multiple choice
12D The instantaneous rate of change of v at t = 4 on v (litres)
the graph at right is: 80

A 13.75 L/min B 12.5 L/min 55


C 6.25 L/min D 55 L/min
E 8 L/min 30

0
4 8 t (min)
11 multiple choice
12D Which one of the following describes an instantaneous rate?
A The temperature dropped 15°C in 2 hours.
B Harry was caught doing 85 km/h.
C Jill ran 5 km in 28 minutes.
D Jenny earnt $80 in 7 1--- hours.
2
E A balloon rose 600 metres in 90 minutes.

12 multiple choice
12E The following diagram shows the position of a particle (x metres) travelling in a straight line
at various times (t seconds).

t=5 t=2
t=0
x
–3 –2 –1 0 1 2 3 4 5

a The displacement after 5 seconds is:


A −2 m B 11 m C 7m D −3 m E 5m
b The average speed in the first 5 seconds is:
11 7 11 7
A – ------ m/s B --- m/s C ------ m/s D – --- m/s E 1 m/s
5 5 5 5
Chapter 12 Rates of change 619
c The average velocity in the first 5 seconds is:
11
A ------ m/s
5
3
B – --- m/s
5
3
C --- m/s
5
11
D – ------ m/s
5
E 1 m/s
13 A particle is moving in a straight line according to the position–time function
x(t) = t2 − 2t + 5 where t ∈ [0, 5]. 12E
a Sketch the position–time graph.
b Where did the journey start?
c When and where did the particle change direction?
d What was the distance travelled in the first 4 seconds?
14 From the position–time graph at right, find: x (m)
a the initial position 200
12E
b how long the object was not moving
c the velocity during the first 5 seconds
d the velocity during the last 4 seconds
30
e the total distance travelled
f the average speed for the journey. 0 5 8 12 t (s)

15 multiple choice
The graph of a gradient function is shown below.
12F
y

2
Gradient
function
0 x

The graph of the original function could be:


A y B y Cy
4

0 x
4
0 x –2 0 x
2

D y Ey

2
1

0 x 0 x
1 1
620 M a t h s Q u e s t M a t h s B Ye a r 1 1 f o r Q u e e n s l a n d

16 multiple choice
12G A position–time graph is shown below.
x

0 t
2
Which one of the following diagrams represents its velocity–time graph?
A v B v C v

0 t 0 t
2 0 t 2
2

D v E v

0 t 0 t
2 2

17 multiple choice
12G 3
v
The velocity–time graph of a particle travelling in a straight line
and starting at the origin is shown at right.

0 t
3 4
–1
Which one of the following represents its position–time graph?
A x B x C x

0 t
3 4
0 t 0 t
3 4 3 4

D x E x

0 t
3 4
0 4 t
Chapter 12 Rates of change 621
18 For the velocity–time graph shown below, sketch a position–time graph, given that the
object starts at the origin. 12G
v
2

0 3 6 t

–2

19 multiple choice
The rate of change of a polynomial f(x) when x = 3 is closest to:
12H
f ( 3.1 ) – f ( 3 ) f ( 3.001 ) – f ( 3 ) f ( 3.5 ) – f ( 3 )
A --------------------------------- B --------------------------------------- C ---------------------------------
3.1 – 3 3.001 – 3 3.5 – 3
f (3) – f (0) f ( 3.000 01 ) – f ( 3 )
D ----------------------------- E ----------------------------------------------
3–0 3.000 01 – 3

20 multiple choice
The rate of change of y = x2 + x when x = 2 is:
12H
A 2 B 3
C 4 D 5
E 6

21 The height, h metres, of a golf ball above the ground at any time, t seconds after it is hit
down the fairway, is given by the function h(t) = 5 + 12t − t2. 12H
a Find the rate of change of height when t equals:
i 4 seconds
ii 6 seconds
iii 10 seconds.
b Briefly describe the change of height of the golf ball within the first 12 seconds.
622 M a t h s Q u e s t M a t h s B Ye a r 1 1 f o r Q u e e n s l a n d

Modelling and problem solving


1 A vending machine at a railway station dispenses
cans of soft drink. It has a capacity of 600 cans
when full.
On a particular day:
• the machine is half full at 6.00 am.
• in the next four hours drinks are dispensed at a
constant rate of 15 per hour.
• at 10.00 am the machine is filled.
• in the next four hours the machine dispenses an
average of 60 cans per hour at a steadily
increasing rate.
• finally, from 2.00 pm to 6.00 pm an average of
30 cans per hour are dispensed at a steadily
decreasing rate.
a Sketch a graph showing the number of cans in the machine over the period from 6.00 am
to 6.00 pm. Hence:
b find the number of cans in the machine at 6.00 pm
c estimate at what time in the afternoon the machine is half full
d estimate the rate at which cans are dispensed at 1.00 pm.
2 The concentration (x mg/L) of a tranquiliser in a patient’s bloodstream at any time t hours
after it is administered is given by the following rule:
2t
x = ------------
2
-, t ≥ 0
t +1
The tranquiliser is only effective if the concentration is at least 0.5 mg/L.
a Determine the concentration at:
i t=0
ii t = 1
iii t = 2
iv t = 4.
b With the aid of a graphics calculator,
sketch the graph for the concentration
function.
c Find the exact length of time that the
tranquiliser is effective.
d Determine the average rate at which the
tranquiliser is absorbed into the
bloodstream from t = 1 to t = 3 hours.
Explain why the rate is negative.
e Determine the instantaneous rate at which
the tranquiliser is absorbed into the
bloodstream at t = 0.5 and t = 2 hours.
(Give answers to 2 decimal places.)
eBook plus f Verify that the rate at which the
tranquiliser is absorbed into the
Digital doc: bloodstream is 0 at t = 1. Hence, give the
Test Yourself
Chapter 12 time and concentration when the
concentration is at a maximum.
Differentiation
and
applications 13
syllabus reference
Topic:
• Rates of change
• Optimisation using
derivatives

In this chapter
13A Introduction to limits
13B Limits of discontinuous,
rational and hybrid
functions
13C Differentiation using first
principles
13D Finding derivatives by rule
13E Rates of change
13F Solving maximum and
minimum problems
624 M a t h s Q u e s t M a t h s B Ye a r 1 1 f o r Q u e e n s l a n d

Introduction
In the previous chapter we considered the problem of determining the speed of a
spanner as it falls from a high tower. Such a calculation is not straightforward because
the speed of the spanner is constantly changing. We modelled the situation by drawing
a graph of the distance, d, the spanner had travelled versus time, t.
We noted that the instantaneous speed of the spanner at t = 3 seconds was the
gradient of the tangent to the curve at t = 3 seconds.
In this case, the gradient of the tangent is:

45 – 0
---------------- = 30 m/s
3 – 1.5

That is, the instantaneous speed of the spanner at t = 3 seconds is 30 m/s.

50

40
d (metres)

30

20

10

0
0.5 1.0 1.5 2.0 2.5 3.0 3.5 4.0 4.5 5.0 5.5
t (seconds)

In this chapter we will extend this concept of finding the instantaneous rate of
change. Suppose we know that the equation relating the distance, d, the spanner falls
with time, t, is:

d = 5t2

Taking the techniques developed previously we can now give an expression for the
gradient of the tangent at any point on the curve between t = 0 and t = 3.
What is the gradient at the point, P; that is, at t seconds?
Consider the chord joining P(t, 5t2) and Q(t + 0.01, 5(t + 0.01)2).

50

40
d (metres)

30

20
Q (t + 0.01, 5(t + 0.01)2)
10
P (t, 5t 2)
0
0.5 1.0 1.5 2.0 2.5 3.0 3.5 4.0 4.5 5.0 5.5
t (seconds)
Chapter 13 Differentiation and applications 625
The gradient of this chord is:

Gradient = 5(t + 0.01)2 − 5t2


= 5t2 + 0.1t + 0.0001 − 5t2
= 10t + 0.01
We can see that as Q approaches P the speed tends to 10t exactly.
The function 10t is called the gradient function of 5t2.

The limit

Look carefully at the words used in the previous solution: ‘As Q approaches P the
speed tends to 10t exactly’.
The point Q never quite reaches P and the speed never quite reaches 10t. However,
they do get arbitrarily close — as close as you can get.
This is the limit concept in informal terms.
In mathematics it is important to understand the concept of a limit. This concept is
especially important in the study of calculus. In everyday life we use the term limit to
describe a restriction put on a quantity. For example, the legal blood alcohol concen-
tration limit for a driver is normally 0.05 g/100 mL. As the number of standard alco-
holic drinks consumed in 1 hour approaches 2, the average adult male’s blood alcohol
concentration approaches 0.05. Likewise, some time after a celebration, a person who
has been drinking heavily at an earlier time may have a blood alcohol concentration
which is approaching the legal limit of 0.05 from a higher level as the number of drinks
not yet metabolised by their body approaches 2. We could say that as the number of
standard drinks remaining in the body approaches 2 the blood alcohol concentration
approaches 0.05. In essence the blood alcohol concentration is a function, say f (x), of
the number of drinks, x, remaining in the body.
626 M a t h s Q u e s t M a t h s B Ye a r 1 1 f o r Q u e e n s l a n d

WORKED Example 1
Add the following series of numbers and state what value it is approaching.
1 1 1 1 1
---
2
+ ---
4
+ ---
8
+ ------
16
+ ------
32
+ ...

THINK WRITE
3
1 Add the first 2 terms. Sum of first 2 terms is --- (= 0.750).
4
7
2 Add the first 3 terms. Sum of first 3 terms is --- (= 0.875).
8
15
3 Add the first 4 terms. Sum of first 4 terms is ------ (≈ 0.938).
16
31
4 Add the first 5 terms. Sum of first 5 terms is ------ (≈ 0.969).
32
63
5 Add the first 6 terms. Sum of first 6 terms is ------ (≈ 0.984).
64
6 Give the upper limit. The sum is approaching 1.

Expressing limits in mathematical language, we say that a limit can be used to


describe the behaviour of a function, f (x), as the independent variable, x, approaches a
certain value, say a. In some cases the function will not be defined at a. Using the
correct notation for the earlier example on the gradient of the chord PQ, we write:
lim f ( x ) = 10t
x→t
This is read as ‘the limit of f (x) as x approaches t is equal to 10t’.

WORKED Example 2
By investigating the behaviour of the function f (x) = x + 3 in the vicinity of x = 2, show
that lim f ( x ) = 5 .
xÆ2
THINK WRITE
1 Create a table of values for x and
f (x) in the vicinity of x = 2. x 1.95 1.99 1.995 2 2.005 2.01 2.05
f (x) 4.95 4.99 4.995 5 5.005 5.01 5.05

2 Consider the values taken by f (x) As x approaches 2 from the y


as x approaches 2. left and the right, f (x)
approaches a value of 5. 5 f (x)

So lim f ( x ) = 5. 3
x→2

–3 0 2 x

Limits of continuous functions


A continuous function has a graph that forms a continuous line; that is, it has no breaks.
If a function is continuous at the point where a limit is being found then the limit
always exists and can easily be found by direct substitution.
Chapter 13 Differentiation and applications 627
WORKED Example 3
Find lim ( x 2 + 2 ) .
xÆ2
THINK WRITE
1 To consider whether the function is y
continuous, sketch the graph of f (x)
6
y = x 2 + 2 in the vicinity of x = 2.

2
0 2 x

2 As the graph is continuous, substitute The function is continuous at x = 2.


x = 2 into x 2 + 2 to evaluate the limit. lim ( x 2 + 2 ) = 22 + 2
x→2
=6

In general if a function, f (x), is continuous when x = a, then lim f ( x ) = f ( a ) .


xÆa

Theorems on limits
Theorem 1
For the constant function f where f (x) = c, then lim f ( x ) = c .
x→a

If lim f ( x ) = A and lim g ( x ) = B , then


x→a x→a

Theorem 2
lim [ f ( x ) ± g ( x ) ] = lim f ( x ) ± lim g ( x ) = A ± B
x→a x→a x→a
For example: lim ( x 2 + 2x ) = lim x 2 + lim 2x
x→1 x→1 x→1
=1+2
=3
Theorem 3
lim [ f ( x ) × g ( x ) ] = lim f ( x ) × lim g ( x ) = A × B
x→a x→a x→a

For example: lim [ x 2 ( 3x ) ] = lim ( x 2 ) × lim ( 3x )


x → –1 x → –1 x → –1
= 1 × −3
= −3
Theorem 4
lim f ( x )
f ( x) x→a A
lim ----------- = ---------------------
- = --- if B ≠ 0
x→a g ( x ) lim g ( x ) B
x→a
lim ( x 2 + 4 )
x 2 + 4 -----------------------------
x→5
For example: lim -------------- = -
x→5 x + 1 lim ( x + 1 )
x→5
29
= ------
6
628 M a t h s Q u e s t M a t h s B Ye a r 1 1 f o r Q u e e n s l a n d

These theorems on limits can be expressed in word form:


1. The limit of a sum = the sum of the limits.
2. The limit of a difference = the difference of the limits.
3. The limit of a product = the product of the limits.
4. The limit of a quotient = the quotient of the limits.

remember
1. If a function, f (x), is continuous when x = a, then lim f ( x ) = f ( a ) .
x→a
2. For the constant function f where f (x) = c, lim f ( x ) = c .
x→a
3. lim [ f ( x ) ± g ( x ) ] = lim f ( x ) ± lim g ( x )
x→a x→a x→a
4. lim [ f ( x ) × g ( x ) ] = lim f ( x ) × lim g ( x )
x→a x→a x→a
lim f ( x )
f ( x) x→a
5. lim ----------- = ---------------------
-
x → a g( x) lim g ( x )
x→a

13A Introduction to limits

WORKED 1 Add the following series of numbers and state what value it is approaching.
Example
1 4 + 2 + 1 + 1--- + 1--- + . . .
2 4

2 The diagram at right shows regular polygons with 3, 4


and 5 sides. As the number of sides gets very large
(→ ∞), what shape emerges?
n=3 n=4 n=5
3 multiple choice
If n represents the number of sides of a regular polygon, then which of the following
correctly describes the situation given in question 2 above?
A n B nlim
→∞
C nlim
→0
D n→0 E n→5

1
4 a Find the value of --- as n gets infinitely large.
n
b Write this using limit notation.
5 a If S = 1 + 1--- + 1--- + 1--- + -----
1
- + . . . and n represents the number of terms to be
2 4 8 16
summed in the series, copy and complete the following table:
n 1 2 3 4 5 6 10
S 1 1 −12

b multiple choice
Which of the following is equal to lim S ?
n→∞
A 1.75 B 1.95 C 2 D 1 E 0
Chapter 13 Differentiation and applications 629
WORKED 6 By investigating the behaviour of the function f (x) = x + 5 in the vicinity of x = 3
Example
show that lim f ( x ) = 8 .
2 x→3

7 multiple choice y
eBook plus f (x)
From the graph at right it can be seen that lim f ( x ) is 3
x→2
Digital doc: equal to: 2
SkillSHEET 13.1
Substituting into 1
a function
0 x
A 2 B –1 C 3 D 1 E 0 –1 2

8 For each of the functions graphed below find lim f ( x ) .


x→4
a y b y c
30
f (x) f (x) y
0 4 x
2
14
–3 f (x) x
0 4
0 4 x
d y
e f y
y f (x) f (x)
21
4 15
10
f (x)
5 1
0 x x 0 x
4 0 4 4

WORKED 9 Evaluate the limits below.


Example
3 a lim ( 2x + 5 ) b lim ( 10 – 2k ) c lim ( 7 p – 4 )
x→2 k→4 p → –1
d lim ( 49a + 1 ) e lim ( x 2 + 2 ) f lim ( x 2 – 7 )
a→1 x→0 x→2
g lim ( h 2 + 3h ) h lim ( 8 – 5x 2 ) i lim ( h 2 + h – 4 )
h→5 x→2 h→1
j lim ( 3 p 2 – 2 p + 3 ) k lim ( x 3 + 2x ) l lim ( t 3 + 5t – 9 )
p → –2 x→0 t → –2

10 multiple choice
If f (x) = 3x − 2 then the value of lim f ( x ) is equal to:
x→0
A 0 B 3 C 2 D −2 E 1

11 multiple choice

The lim ( x 2 – 5 ) is equal to:


x → –3
A 4 B −5 C 9 D –4 E 0
12 Find the value of the following limits.
x2 + 3 2x 2 – 3x c
6x – 2 x 2 + 3x – 2
a lim -------------- b lim -------------------- lim --------------- d lim --------------------------
x→2 x x→0 x + 1 x → –2 x x→3 x–2
630 M a t h s Q u e s t M a t h s B Ye a r 1 1 f o r Q u e e n s l a n d

Sneaking up on a limit
x 2 + x – 2- .
1 Consider the expression ----------------------
x–1
a Complete a table of values by evaluating the expression for x = 1.5, 1.2, 1.1,
1.01, 1.001, 1.0001 and 1.
b Copy and complete:
x2 + x – 2
As x gets closer to 1 from above, ----------------------- gets closer to , or
x2 + x – 2 = . x–1
lim -----------------------
x → 1+ x–1
c Complete a table of values by evaluating the expression for x = 0.5, 0.6, 0.9,
0.95, 0.99, 0.999 and 1.
d Copy and complete:
x2 + x – 2
As x gets closer to 1 from below, ----------------------- gets closer to , or
x–1
x2 + x – 2 = .
lim -----------------------
x → 1– x–1
e Explain why the limit can be evaluated, but not the actual value of the
expression above when x = 1.
x 2 + 2x – 15 as x approaches 3.
2 Use a similar technique to evaluate the limit of -----------------------------
x 2 – 7x x–3
3 Find lim ----------------- .
x→7 x – 7

Limits of discontinuous, rational and


hybrid functions
Limits of discontinuous functions
If a function is discontinuous at the point where the limit is being investigated then the
limit will exist only if the function is approaching the same value from the left as from
the right. Consider the discontinuous functions graphed below.
1. From the left, lim − f ( x ) = 1 . y
x→1
(The symbol x → 1− indicates that we are letting
x approach 1 from the left side.) 1
From the right, lim + f ( x ) = 1 .
x→1
0 x
Left limit = right limit. 1
Therefore, lim f ( x ) = 1.
x→1
2. From the left, lim − f ( x ) = 2 . y f (x)
x→1
From the right, lim + f ( x ) = 4 . 4
x→1
Left limit ≠ right limit.
Therefore, lim f ( x ) does not exist. 2
x→1

0 1 x
2
Chapter 13 Differentiation and applications 631
Limits of rational functions
Finding the limit of a rational function involves simplifying the function before direct
substitution can take place.

WORKED Example 4
x2 – x
a Find lim --------------- .
xÆ1 x – 1
x2 – x
b Sketch the graph of f ( x ) = --------------- , stating the value of x for which it is
x–1
discontinuous.

THINK WRITE
x2 – x x( x – 1)
a 1 We cannot substitute x = 1 directly as a lim -------------- = lim -------------------
x→1 x – 1 x→1 x – 1
we will get 0--- which is undefined, so we
0
factorise the numerator.

2 Cancel out the common factor. = lim x , x ≠ 1


x→1

3 Now substitute x = 1 and evaluate. =1

x2 – x b
b The graph of f ( x ) = -------------- is the same as y
x–1 f (x)
the graph of f (x) = x, except where the
point (1, 1) does not exist. 1

0 x
1
The function is discontinuous at x = 1.

Graphics Calculator tip! Finding the limit of


a rational function
Some graphics calculators can find the limit of a rational function. The instructions
x2 – x
below show how to find lim -------------- from Worked example 4.
x→1 x – 1

For the TI-Nspire CAS


1. Open a new Calculator document. Press b and
then select 4: Calculus. A number of Calculus
options are displayed.
632 M a t h s Q u e s t M a t h s B Ye a r 1 1 f o r Q u e e n s l a n d

2. Press 3 to select 3: Limit. The limit template is


now displayed with fields for the relevant
information.

3. Enter the information in the fields provided. Press


e (or use the arrows on the NavPad) to move
between fields. Move the cursor to the far right of
the expression.

4. Press · to display the required limit. The limit is 1.

5. It is also possible to calculate the limit as x


x2 – x
approaches from the left by typing lim --------------
x→1 x – 1

x2 – x
and from the right by typing lim -------------- .
x→1 x – 1
+

6. The process shown in the previous step is useful


1
in a case like lim --- . We can calculate the limit
x→1 x
from the left and the right as shown in the screen
opposite. However, if the + or − is omitted, the
limit at x = 0 is calculated. As the actual limit for
this function at x = 0 does not exist (since the left
and right limits are not the same), the calculator
displays an answer of undefined.
Note: The limit template can also be obtained
from the catalog. Press k to access the catalog
and then press 5 to select the template palette.
Highlight the limit template and then press ·.
Chapter 13 Differentiation and applications 633
WORKED Example 5
x2 + 5 x + 6
a By first factorising the numerator, simplify the rational function f ( x ) = --------------------------- ,
x+3
stating the value for which the function does not exist (that is, is discontinuous).
b Find lim f ( x ) , where a is the value at which f (x) is discontinuous.
xÆa

THINK WRITE
x 2 + 5x + 6
a 1 Factorise the numerator. a f ( x ) = ---------------------------
x+3
( x + 3)( x + 2)
= ----------------------------------
( x + 3)
2 Cancel out the common factor. f ( x ) = x + 2 , x ≠ −3
so f(x) is discontinuous
at x = –3
b Write an expression for lim f ( x ) and b lim ( x + 2 ) = −3 + 2
x→a x → –3
evaluate by substituting x = −3. = −1

Limits of hybrid functions


Hybrid functions are functions that have different rules for different parts of the domain.

WORKED Example 6
2
{
a Sketch the graph of the hybrid function f (x) = x , x ∈ (−∞, 2]
x − 1, x ∈ (2, ∞)
b Find i lim − f ( x ) ii lim + f ( x ) iii lim f ( x ) if it exists.
xÆ2 xÆ2 xÆ2

THINK WRITE
a 1 Sketch f(x) = x 2 over the domain a y
(−∞, 2]. 4 f (x)
2 Sketch (on the same axes)
f (x) = x − 1 over the domain (2, ∞).
1
0 x
2
b i Substitute x = 2 into f (x) = x 2. b i lim f ( x ) = lim −x 2
x → 2− x→2
=4
ii Substitute x = 2 into f (x) = x − 1. ii lim + f ( x ) = lim +( x – 1 )
x→2 x→2
=1
iii Are these limits equal? iii lim f ( x ) does not exist (as left limit
x→2
≠ right limit).
634 M a t h s Q u e s t M a t h s B Ye a r 1 1 f o r Q u e e n s l a n d

Dirichlet’s function
The French mathematician Dirichlet devised the following hybrid function.
 0 if x is a rational number
D(x) = 
 1 if x is not a rational number
So D(0.5) = 0 but D( 2 ) = 1
Can you decide the value of lim D ( x ) ?
x→2

remember
1. If a function is discontinuous at the point where the limit is being investigated
then the limit will exist only if the function is approaching the same value from
the left as from the right.
2. Finding the limit of a rational function involves simplifying the function before
direct substitution can take place.
3. Hybrid functions are functions that have different rules for different parts of the
domain.

Limits of discontinuous,
13B rational and hybrid functions
1 Which of the following graphs are discontinuous?
a y b y c y

3
0 x

0 x
3 0 x
4

d y e y f y
4
0 x
–1

0 3 x
0 x
2

2 For each discontinuous function above state the value of x for which it is discontinuous.

x 2 – 4x
WORKED 3 a Find lim ----------------- .
Example x→4 x – 4
4
x 2 – 4x
b Sketch the graph of f ( x ) = ----------------- , stating the value of x for which it is
x–4
discontinuous.
Chapter 13 Differentiation and applications 635
x2 + x
4 a Evaluate f ( x) when x = 0 if f ( x ) = -------------- . Comment on this result.
x
b For what value of x is f ( x) discontinuous?
c Factorise the numerator of f ( x).
d Now simplify f ( x).
e Sketch the graph of f ( x).
f Evaluate lim f ( x ) if it exists.
x→0
WORKED 5 By first factorising the numerator, simplify the following rational functions, stating
Example
5a
the value for which the function does not exist (is discontinuous).
x 2 + 3x 6x – 18
b f ( x ) = ------------------
a f ( x ) = -----------------
x x–3
2
x – 5x 2
x + 5x + 4
c f ( x ) = ----------------- d f ( x ) = ---------------------------
x x+4
2
x – 7x + 6 3
x +8
e f ( x ) = -------------------------- f f ( x ) = --------------
x–6 x+2
2
x + 3x – 4 x 3 – 27
g f ( x ) = -------------------------- h f ( x ) = -----------------
x–1 x–3
WORKED 6 For each rational function in question 5 above, find lim f ( x ) , where a is the value at
Example x→a
5b
which f (x) is discontinuous.
WORKED 7 Sketch the graphs of the following hybrid functions.
Example
6a  x + 3 , x ∈ ( – ∞, 2 )  4 – x, x ∈ ( – ∞, – 1]
a f ( x) =  b g( x) = 
 4, x ∈ [2, ∞)  3x + 1, x ∈ ( – 1, ∞ )

 2x, x ∈ ( – ∞, 0 )  2
c h( x) =  2 d p ( x ) =  x + 1, x ∈ ( – ∞, 1]
 x , x ∈ [0, ∞)  x + 2, x ∈ (1, ∞)
WORKED 8 For each of the corresponding functions in question 7 above evaluate the following.
Example
6b a i lim − f ( x ) ii lim + f ( x ) iii lim f ( x )
x→2 x→2 x→2

b i lim g ( x ) ii lim g ( x ) iii lim g ( x )


x → –1 − x → –1 + x → –1

c i lim −h ( x ) ii lim +h ( x ) iii lim h ( x )


x→0 x→0 x→0

d i lim − p ( x ) ii lim + p ( x ) iii lim p ( x )


x→1 x→1 x→1

9 Investigate whether the following limits exist. For those that do exist state the limit.
 x + 2 , x ∈ ( – ∞, 2 )  x – 4, x ∈ ( – ∞, 0]  5 – x , x ∈ ( – ∞, 3 )
a lim  b lim  c lim 
x → 2  3, x ∈ [2, ∞) x → 2  x – 4, x ∈ ( 0, ∞ ) x → 3  2x + 1, x ∈ [3, ∞)

 2x – 3, x ∈ ( – ∞, 1]  2  4 – x 2, x ∈ ( – ∞, – 2 )
d lim  e lim  x , x ∈ ( – ∞, 2] f lim 
x → 1  2x + 1, x ∈ ( 1, ∞ ) x → 1  x + 2, x ∈ ( 2, ∞ ) x → – 2  x + 2, x ∈ [ – 2, ∞)

10 Evaluate the following.


14x + 7
a lim ( x 2 + 3x + 2 ) b lim ------------------
x→2 1 2x + 1
x → – ---
2
636 M a t h s Q u e s t M a t h s B Ye a r 1 1 f o r Q u e e n s l a n d

lim 3x + 1, x ∈ ( – ∞, 1 ) x 2 – 9x – 10
c x→1 x 2 + 3, x ∈ [1, ∞ ) d lim -----------------------------
 x → –1 x+1

 2
e lim ( 12 – 3x – 2x 2 )
x→2
f lim  x – 2x + 1, x ∈ ( – ∞, 1]
x → 0  x + 3, x ∈ ( 0, ∞ )

x 2 + 3x + 2 h lim ( x 3 + x 2 – 5x )
g lim ---------------------------
x → –1 x+2 x→1

( x + 3)( x – 3) ( x + 2 ) ( 2x – 3 )
i lim ---------------------------------- j lim -------------------------------------
x→3 x–3 x → –2 x+2
x 2 + 7x + 6 x 2 – 6x + 8
k lim --------------------------- l lim --------------------------
x → –1 x+1 x→4 x–4

Differentiation using first principles


The gradient function is the rule for the instan-
taneous rate of change of a given function at any y y = f (x)
point. The gradient at any point (x, y) can be found Q
f (x + h)
by substitution into the gradient function.
Consider the secant PQ drawn to the curve f (x)
Tangent at P
at right. f (x) P

The coordinates of P are [x, f (x)] and the coordi-


nates of Q are [x + h, f (x + h)]. So the gradient of 0 x x+h x
rise
the secant PQ = --------
run
f ( x + h) – f ( x)
= --------------------------------------
x+h–x
f ( x + h) – f ( x)
= --------------------------------------
h
As h approaches zero, that is, as Q draws as close as possible to P along the curve,
PQ effectively becomes a tangent to the curve at P.
y
Q
Q moves closer to
Q P as h approaches 0

P Q

0 h x

We can therefore say that the gradient of the tangent at P is


f ( x + h) – f ( x)
lim --------------------------------------- or
h→0 h
f ( x + h) – f ( x)
f ′( x ) = lim -------------------------------------- , h ≠ 0,
h→0 h
where f ′(x) denotes the gradient of a tangent at any point, x, on the graph of f (x).
That is, f ′(x) is the gradient function of f (x).
Chapter 13 Differentiation and applications 637
f ( x + h) – f ( x)
The process of finding the gradient function lim -------------------------------------- is called
h→0 h
differentiation from first principles, or finding the derivative from first principles.
Two different forms of notation are commonly used to represent a function and its
derivative.
1. The European notation of Leibniz is: 2. The alternative notation is:
(a) y for the function (a) f (x) for the function
dy d
(b) ------ for the derivative. (b) f ′(x) or ------ [ f ( x ) ] for the derivative.
dx dx

Secants and tangents


A spreadsheet may be used to study the effect h has on the gradient of a secant by
f ( x + h) – f( x)
calculating f(x + h), f(x) and h, as well as the gradient ------------------------------------- .
h
Use a spreadsheet (you may wish to use the Maths Quest spreadsheet ‘Gradient
of a secant’ by logging into www.jacplus.com.au and accessing the weblinks for
eBook plus this chapter) to answer the following.
1 For f(x) = x2 + 1, find the gradient of the secant between:
Digital doc:
EXCEL Spreadsheet
a x = 1 and x = 2
Gradient of a secant b x = 1 and x = 1.5
c x = 1 and x = 1.1
d x = 1 and x = 1.01
e x = 1 and x = 1.001.
2 Use your answers to question 1 to predict the gradient of the tangent to
f(x) = x2 + 1 at x = 1.
3 For f(x) = –x2 + 4x (enter this as –(x2) + 4*x if using the Maths Quest
spreadsheet), find the gradient of the secant between:
a x = 0 and x = 2
b x = 0 and x = 1
c x = 0 and x = 0.5
d x = 0 and x = 0.1
e x = 0 and x = 0.001.
4 What is the gradient of the tangent to f(x) = –x2 + 4x at x = 0?

WORKED Example 7
Find the derivative of x2 − 2x using first principles.

THINK WRITE
1 Define f (x). f (x) = x 2 − 2x
f ( x + h) – f ( x)
2 The derivative is equal to: f ′(x) = lim --------------------------------------
h→0 h
f ( x + h) – f ( x)
lim -------------------------------------- .
h→0 h
Continued over page
638 M a t h s Q u e s t M a t h s B Ye a r 1 1 f o r Q u e e n s l a n d

THINK WRITE

3 Simplify the numerator f (x + h) − f (x). f (x + h) − f (x)


= (x + h)2 − 2(x + h) − (x 2 − 2x)
= x2 + 2xh + h2 − 2x − 2h − x 2 + 2x
= 2xh + h2 − 2h
4 Factorise the numerator f (x + h) − f (x). = h(2x + h − 2)
f ( x + h) – f ( x) f ( x + h) – f ( x)
5 Simplify lim -------------------------------------- by lim --------------------------------------
h→0 h h→0 h
cancelling the common factor of h. h ( 2x + h – 2 )
= lim --------------------------------
h→0 h
= lim ( 2x + h – 2 ), h ≠ 0
h→0

6 Evaluate the limit by substituting h = 0. = 2x − 2

Graphics Calculator tip! Finding the derivative


using first principles
The limit template can also be used to find the derivative of a function using the first
principles method. Consider the function f(x) = x2 − 2x from Worked example 7.
For the TI-Nspire CAS
1. Open a new Calculator document. Press b and
then select 1: Actions followed by 1: Define.
Enter the function and then press ·.

2. Press b and then select 4: Calculus followed by


3: Limit to obtain the limit template. Enter the
information in the appropriate fields.

3. Press · to display the required limit. The


derivative is 2(x − 1) or 2x − 2.
Chapter 13 Differentiation and applications 639
WORKED Example
2
8
If g(x) = 2x + 5x − 2, find:
a g′(x) using first principles b the value(s) of x where the gradient equals 0.

THINK WRITE
a 1 Let g(x) = 2x 2 + 5x − 2. a g(x) = 2x 2 + 5x − 2
g( x + h) – g( x)
2 The derivative is equal to: g′(x) = lim --------------------------------------
h→0 h
g( x + h) – g( x)
lim --------------------------------------
h→0 h
3 Simplify the numerator g(x + h) − g(x)
g(x + h) − g(x). = 2(x + h) 2 + 5(x + h) − 2 − (2x 2 + 5x − 2)
= 2(x 2 + 2xh + h2) + 5x + 5h − 2 − 2x 2 − 5x + 2
= 2x 2 + 4xh + 2h2 + 5x + 5h − 2 − 2x 2 − 5x + 2
= 4xh + 2h2 + 5h
4 Factorise the numerator = h(4x + 2h + 5)
g(x + h) − g(x).
g( x + h) – g( x) g( x + h) – g( x)
5 Simplify lim -------------------------------------- by lim -------------------------------------
h→0 h h→0 h
cancelling the common factor of h. h ( 4x + 2h + 5 )
= lim ------------------------------------
h→0 h
= lim ( 4x + 2h + 5 ), h ≠ 0
h→0
6 Evaluate the limit by substituting = 4x + 5
h = 0. So g′(x) = 4x + 5.

b Solve g′(x) = 0. b g′(x) = 0


4x + 5 = 0
4x = −5
x = − 5---
4

So the gradient equals 0 when x = − 5--- .


4

Note: For all polynomial functions, f (x), when the expression f (x + h) − f (x) is simpli-
fied all of its terms have h as a factor.

remember
f ( x + h) – f ( x)
1. The process of finding the gradient function lim -------------------------------------- is called
h→0 h
differentiation from first principles.
dy
2. Differentiating y gives ------ .
dx
3. Differentiating f (x) gives f ′(x).
640 M a t h s Q u e s t M a t h s B Ye a r 1 1 f o r Q u e e n s l a n d

Differentiation using first


13C principles
WORKED 1 Find the derivative of the following using first principles.
Example
7 a 5x − 7 b x 2 + 10x c x 2 − 8x d x 3 + 2x

eBook plus dy
2 Use first principles to find ------ .
dx
Digital doc: a y = x2 + 3 b y = x 2 − 3x + 1 c y = 4x 2
SkillSHEET 13.2
2
Differentiating from d y=9−x e y = 6x − 2x 2 f y = x 3 + 5x − 4
first principles

WORKED 3 If g(x) = x 2 − 6x find:


Example
8
a g′(x) using first principles
b the value(s) of x where the gradient equals 0.

4 a If f (x) = x3 − 8, find f ′(x) using first principles.


b Hence, determine the value(s) of x where the gradient function is equal to 12.

5 By first deriving the gradient function f ′(x), evaluate f ′(3) when f (x) is equal to:
a 7x + 5 b x 2 + 4x c x 2 − 3x + 2 d x 3 − 5.

6 multiple choice
Which of the following do not denote the gradient at any point on a graph? (One or
more answers.)
f ( x + h) – f ( x) f ( x + h) – f ( x)
A f ′(x) B lim -------------------------------------- C lim --------------------------------------
h→0 h h→∞ h
dy f ( x + h) – f ( x)
D ------ E --------------------------------------
dx h

7 multiple choice
The most accurate method for finding the gradient when x = 3 for the function
f (x) = x 2 + 2x is by:
A sketching the graph and drawing a tangent at x = 3 to find the gradient
B finding the gradient of the secant to the curve joining the points where x = 3 and
x = 3.1
C finding f ′(x) using first principles and evaluating f ′(3)
D guessing
E finding the gradient of the line from the origin to the point (3, 15)

8 multiple choice
Given that f ′(x) = 4x if f (x) = 2x2 and g′(x) = 3x 2 + 1 if g(x) = x 3 + x, then the deriv-
ative of x3 + 2x 2 + x must be equal to:
eBook plus
A 3x 2 + 4x + 1 B 12x 3 + 4x C 3x 2 + 4x
Digital doc:
WorkSHEET 13.1 D 2x 5 + 2x 3 E 5x 2 + 1
Chapter 13 Differentiation and applications 641
Finding derivatives by rule
Fortunately the tedious process of finding derivatives from first principles need not be
applied once rules are established. For polynomial functions the following rules apply.
Rule 1. If f (x) = x n, then f ′(x) = nx n − 1.
Rule 2. If f (x) = ax n, then f ′(x) = nax n − 1.
Rule 3. If f (x) = c, then f ′(x) = 0 (where c is constant).
Rule 4. If f (x) = g(x) + h(x), then f ′(x) = g′(x) + h′(x).

WORKED Example 9
Differentiate each of the following:
a y = x8 b y = 3x 2 c y = 7x + 3 d y = 2 x 5 + 3--5- x 2 – 6 x .
THINK WRITE
a Write the expression for y. Apply rule 1 a y = x8
to find the derivative. dy
------ = 8x 8 – 1
dx
= 8x 7
b Apply rule 2. b y = 3x2
dy
------ = 2 ( 3x 2 – 1 )
dx
= 6x
c Apply rules 2, 3 and 4. c y = 7x + 3
Remember that x0 = 1. dy
------ = 7x 1 – 1 + 0
dx
= 7x 0
= 7
d Differentiate the 3 terms separately (that d y = 2x5 + 3--- x2 – 6x
5
is, apply rules 2 and 4).
dy
------ = 5 ( 2x 5 – 1 ) + 2 ( --35- x 2 – 1 ) – 1 ( 6x 1 – 1 )
dx
= 10x 4 + 6--5- x – 6x 0

= 10x 4 + 6--5- x – 6

WORKED Example 10
Find f ′(x) if f (x) = 3x(x − 2).
THINK WRITE
1 Write down f(x). f (x) = 3x(x − 2)
2 Expand the brackets. f (x) = 3x 2 − 6x
3 Differentiate by rule. f ′(x) = 6x − 6
642 M a t h s Q u e s t M a t h s B Ye a r 1 1 f o r Q u e e n s l a n d

WORKED Example 11
4 x3 + 3 x2
If g ( x ) = ------------------------ , find g′(x).
x
THINK WRITE
Factorise the numerator because at this stage we can only 4x 3 + 3x 2
1 g ( x ) = -----------------------
x
differentiate a constant denominator.
x 2 ( 4x + 3 )
= --------------------------
x
2 Simplify g(x). = x(4x + 3), x ≠ 0
3 Expand the brackets. = 4x2 + 3x
4 Differentiate g(x) by rule. g′(x) = 8x + 3

WORKED Example 12
Differentiate each of the following:
1
1 --- 4
a f ( x ) = x –3 b f ( x ) = ----7- c f ( x) = x3 d f ( x ) = ------- .
x x
THINK WRITE
a 1 Write down f(x). a f(x) = x–3
2 Differentiate by rule 1. f ′(x) = −3x−3 − 1
= −3x−4
1-
b f (x) = ----
b 1 Write down f(x).
x7
2 Bring the x term to the numerator using the index laws, = 1x−7
as we can only differentiate a constant denominator.
3 Differentiate by rule 1. f ′(x) = −7(1x−7 − 1)
= −7x−8
4 Express answer with a positive index to follow style of f (x). = – ----7-
x8
1
c 1 Write down f(x). c f (x) = x --3- 1
--- – 1
2 Differentiate by rule 1. f ′(x) = 1--- (x 3 )
3
– 2---
x -3
= ------
3
1-
= -------
3 Express answer with a positive index. 2
---
3x 3
d Write down f(x). d f ( x ) = ------4-
1
x
4-
f (x) = ----
2 Convert x to index form. 1
---
x2
1
– ---
3 Bring the x term to the numerator using the index laws. f (x) = 4x 2

1
4 Differentiate by rule 2. f ′(x) = − 1--- 4x
2
( – --- – 1
2 )
– --3-
= – 2x 2

2-
= – ----
5 Express with a positive index. 3
---
x2
2-
= – --------
6 Express the power of x back in surd (square root) form.
x3
Chapter 13 Differentiation and applications 643

Graphics Calculator tip! Finding the derivative


Some graphics calculators can display the derivative of a function directly. The instruc-
tions below show how to find the derivative of f(x) = 2x3 – 4x2 + 6 and also the value of
the derivative at x = 5.
For the TI-Nspire CAS
1. Open a new Calculator document. Press b and
then select 4: Calculus followed by 1: Derivative.
This displays the derivative template.

2. Enter the variable x for dx and the expression


2x3 − 4x2 + 6 in the appropriate fields and then
press ·.

3. Alternatively, we can define the function and then


find the derivative. To define the function, press
b and then select 1: Actions followed by
1: Define. Enter the function and then press ·.
To find the derivative, press b and then select
4: Calculus followed by 1: Derivative. Enter the
variable x and the function as f1(x). Move the
cursor to the far right of the expression. Press ·
to obtain the expression for the derivative.

4. To find the value of the derivative at x = 5,


complete the entry line as shown. (Repeat step 3
without pressing ·, then press * and follow
with X=5.) Now press ·. The gradient of the
function at x = 5 is 110.

Note: The derivative template can also be obtained


from the catalog. Press k to access the catalog
and then press 5 to select the template palette.
Highlight the derivative template and then press
·.
644 M a t h s Q u e s t M a t h s B Ye a r 1 1 f o r Q u e e n s l a n d

Tangents and normals


As mentioned earlier in this chapter the derivative
f ′(x) is actually the gradient function. This means that
y
the value of the gradient at any particular point on a
curve is equal to the numerical value of the derivative
at that point. Normal
Tangent
Recall that if the gradient of a tangent to a curve at
point P is m T, then the normal, mN, is a straight line P
perpendicular (at right angles) to the tangent such that
1 and passing through the point P as shown at
mN = − ------
m T- 0 x
right.
Also recall that the equation of a straight line is given by y − y1 = m(x − x1) where
(x1, y1) is the point P, above, and m is the gradient.

WORKED Example 13
a Find the equation of the tangent to the curve f (x) = x2 + 6x − 8 at the point where the
gradient has a value of 8.
b Hence, find the equation of the normal at this point.

THINK WRITE

a 1 Find the gradient function of the a f (x) = x2 + 6x – 8


curve, f ′(x). f ′(x) = 2x + 6
2 Find x1, the value of x where For gradient = 8
f ′(x) = 8; that is, solve 2x + 6 = 8. 2x + 6 = 8
2x = 2
x=1 So x1 = 1.
3 Find f (x1) to determine the value y1 = f (x1)
of y1. = f (1)
= (1)2 + 6(1) − 8
= −1
4 Simplify the equation The equation of the tangent at the point (1, −1)
y − y1 = m T (x − x1) to find the is y − −1 = 8(x − 1)
equation of the tangent. y + 1 = 8x − 8
y = 8x − 9
b 1 Find the gradient of the normal using b mN = − 1---
8
1
m N = – ------- .
mT
2 Simplify the equation The equation of the normal at the point (1, −1)
y − y1 = mN (x − x1) to find the is y − −1 = − 1--- (x − 1)
equation of the normal. 8
( x – 1)
y + 1 = − ----------------
8
8y + 8 = −x + 1
x + 8y + 7 = 0
Chapter 13 Differentiation and applications 645
remember
1. Rule 1: If f (x) = x n, then f ′(x) = nx n − 1.
2. Rule 2: If f (x) = ax n, then f ′(x) = nax n − 1.
3. Rule 3: If f (x) = c, then f ′(x) = 0 (where c is constant).
4. Rule 4: If f (x) = g(x) + h(x), then f ′(x) = g′(x) + h′(x).
5. Equation of tangent: y − y1 = m T (x − x1)
6. Equation of normal: y − y1 = mN (x − x1)
1
where mN = − -------
mT

13D Finding derivatives by rule

WORKED 1 Differentiate each of the following.


Example
9 a y = x6 b y = 7x 2 c y = 5x + 2 d y = 4x4 + 2--- x 2 − 5x
3
dy
2 Find ------ if y is:
dx
a 3x4 b 8x7 c 5x 5 d −4x6 e −2x3 f −7x
3 Match the correct derivative from the set A to G below to each of the following.
a x8 b x4 + x 2 c 2x3 − 4x + 7 d x 2 + 6x − 5
4 3 5 3 p
e x + 2x − 3 f x + 6x − 4x g x
A 5x4 + 18x 2 − 4 B 4x 3 + 2x C px p − 1 D 8x7
E 6x2 − 4 F 2x + 6 G 4x3 + 6x 2
4 Differentiate the following.
a y = x6 + 3x 2 − 4 b y = 5x4 − 7x3 + 6x
c y = x11 − 3x6 + 4x5 + 3x 2 d y = 10x5 − 3x4 + 2x3 − 8x
e y=6 f y = 3x4 + 5x4
5 Find f ′(x) if f (x) is:
2 4
a --- x b − 5--- x 2 c 6 7
--- x
3 8 7
3 3
d --- x + 1--- x 2 − 3x e 2 5
--- x + 3--- x4 + 1--- x3 f 4x3 − 4--- x 2 − 5--- x + 8
4 2 5 4 6 7 6

WORKED 6 Find f ′(x) for each of the following.


Example
10
a f (x) = x(x + 3) b f (x) = 3x(2x − 5) c f (x) = (x + 4)2
d f (x) = 9(8 − 3x)2 e f (x) = (x + 2)3 f f (x) = (2x − 5)3
WORKED 7 Find g ′(x) by first simplifying g (x).
Example
11 x 3 + 5x 8x 2 – 6x
a g ( x ) = ----------------- b g ( x ) = --------------------
x 2x
3x 3 + 2x 2 – 5x 5x 4 + x 3 + 7x 2
c g ( x ) = ----------------------------------- d g ( x ) = ----------------------------------
-
x x2
646 M a t h s Q u e s t M a t h s B Ye a r 1 1 f o r Q u e e n s l a n d

WORKED 8 Differentiate each of the following:


Example
a x−4 b x−7
12
c 3x−4 d 5x−8
e −4x−6 f −3x−5
1 1
g ----4- h -----
x x9
5 10
i ----- j ------
x3 x6
1 2
--- ---
k 2x 2 l x3
1 2
--- ---
m 4x 4 n 3x 5
1
o x p -------
x
q 4 x r 3 x
2
s -------
3 x

9 Evaluate i f ′(1) ii f ′(−2) and iii f ′(0) for each of the following.
eBook plus
a f (x) = 5x 2 + 3x − 1
Digital docs: b f (x) = 1--3- x3 + 2x 2 − 4
EXCEL Spreadsheet
Gradient at a point c f (x) = 3x 2 − 2x + 6
EXCEL Spreadsheet d f (x) = x3 + 7x − 8
Tangent and normal
10 a Find the x-intercepts of the parabola y = x 2 − 5x + 6.
b Find the gradient of the parabola at the points where it crosses the x-axis.
c Determine the value of x for which the gradient of the parabola is:
i 0
ii 7
iii −3.
11 a Find the x-intercepts of the curve y = 2x 2 + 5x − 3.
b Find the gradient of the curve at these points.
c Find the coordinates of the point where the gradient is 0.
12 Find the coordinate(s) of the points on the curve x 3 − 3x2 where the tangent:
a is parallel to the x-axis
b is parallel to the line y = −3x + 2.
WORKED 13 a Find the equation of the tangent at the point on the curve x 2 + 4x − 1 where the
Example
13
gradient is 6.
b Hence, find the equation of the normal at this point.
14 Find the equation of the normal to the curve y = 2x 2 − 2x + 5 at the point where the
curve crosses the y-axis.
15 Find the equation of the normal to the curve y = –x2 + 4x at:
a x=2
b x = 1.
16 Find the equation of the normal to the graph of y = x3 + 2x2 – 3x + 1 at x = –2.
Chapter 13 Differentiation and applications 647
Graphs of derivatives
Now that we have a quick way to find the rule for the y
dy- of a function f(x), we may graph f'(x) 40
derivative f ′(x) or -----
dx
the derivative using its rule, rather than by finding several 20
gradients and plotting points. f(x)

0 x
For example, if f(x) = x3 + 12x, then –5 5
f ′(x) = 3x2 – 12 = 3(x2 – 4) = 3(x + 2)(x – 2). –20
f(x) and f ′(x) may be plotted on the same axes as shown
–40
at right.

1 Use technology to produce graphs of the following functions and their


eBook plus
derivatives, with each pair on the same set of axes. Show all turning points of
Digital doc: the original function.
EXCEL Spreadsheet
Two function grapher a f(x) = x3 – 5x2 + 1
b f(x) = 2x2 + 4x + 5
c f(x) = x4 – 3x3 – 3x2 + 11x – 6
d f(x) = 6x – 3

2 Copy and complete: The of the original function (thick


graph) are above or below the x of the derivative graph.

Graphics Calculator tip! Graphing the derivative function

We can graph the function f(x) = (x − 1)(x − 2)(x + 1) and its derivative on the same set
of axes.
For the TI-Nspire CAS
1. Open a new Graphs & Geometry document.
Enter the function at f1(x) and press ·.

2. To change the Window Settings, press b and


then select 4: Window followed by 1: Window
Settings. Enter the values as shown, pressing e
to move to the next field.
648 M a t h s Q u e s t M a t h s B Ye a r 1 1 f o r Q u e e n s l a n d

3. Press e until OK is highlighted. Press · to


display the graph of the function with the new
settings.

4. To draw the graph of the derivative we need to


enter the derivative template. Press k to access
the catalog and then press 5 for the template
palette. Highlight the template shown.

5. Press ·. The derivative template will appear in


the function entry line for f2(x). Complete the
required fields with the variable x and the function
f1(x). Remember to move the cursor to the far
right of the expression.

6. Press · to display both graphs on the one set of


axes.

7. To see the rule of each graph more clearly, we can


move each rule to a different position. First, press
/G to hide the function entry line, then press
d. The pointer will appear on the screen. Use
the NavPad to move the pointer so that it hovers
over the rule. The pointer will then appear as an
open hand (÷). Press /x and the hand will
close ({) and the rule will flash. Use the NavPad
to move the rule to the desired location. Press d
to fix the rule into its new position.
Chapter 13 Differentiation and applications 649
Rates of change
The rate of change of a function refers to the rate at which its gradient is changing. For
linear functions the gradient is constant; however, the gradient for other functions such
as quadratic or cubic polynomials is continually changing.
Differentiation provides us with a tool to describe the gradient of a function and
hence determine its rate of change at any particular point. In essence, while average
rates of change can be determined from the original function, differentiation of this
function provides a new function that describes the instantaneous rate of change.

Note: The term instantaneous rate of change is often referred to as rate of change.


The rate of change
of position with respect
to time is velocity.

The rate of change of



velocity with respect to
time is acceleration.

WORKED Example 14
If f (x) = x2 − 2x + 4 find:
a the average rate of change between x = 2 and x = 4
b a new function that describes the rate of change
c the instantaneous rate of change when x = 4.
THINK WRITE
a 1 Write the function. a f(x) = x2 − 2x + 4
change in f ( x ) f (4) – f (2)
2 Average rate of change = ------------------------------------ . Average rate of change = -----------------------------
change in x 4–2
12 – 4
= ---------------
2
=4
b Differentiate f (x). b f ′(x) = 2x − 2
c Substitute x = 4 into f ′(x). c f ′(4) = 2(4) − 2
=6
So the rate of change when x = 4 is 6.
650 M a t h s Q u e s t M a t h s B Ye a r 1 1 f o r Q u e e n s l a n d

WORKED Example 15
A javelin is thrown so that its height, h metres above the
ground, is given by the rule:
h(t) = 20t − 5t2 + 2, where t represents time in seconds.

a Find the rate of change of the height at any time, t.


b Find the rate of change of the height when i t = 1 ii t = 2 iii t = 3.
c Briefly explain why the rate of change is initially positive,
then zero, and then negative over the first 3 seconds.
d Find the rate of change of the height when the javelin first
reaches a height of 17 metres.

THINK WRITE
a 1 Write the rule. a h(t) = 20t − 5t 2 + 2
2 Differentiate h(t). h′(t) = 20 − 10t

b 1 Evaluate h′(1). b h′(1) = 20 − 10(1)


= 10 m/s
2 Evaluate h′(2). h′(2) = 20 − 10(2)
= 0 m/s
3 Evaluate h′(3). h′(3) = 20 − 10(3)
= −10 m/s

c For rates of change: c The javelin travels upward during the


Positive means increasing. first 2 seconds.
Zero means neither increasing nor decreasing. When t = 2 seconds the javelin has
Negative means decreasing. reached its maximum height.
When t > 2 seconds the javelin is
travelling downward.

d 1 Find the time at which a height of 17 m d 20t − 5t2 + 2 = 17


occurs, by substituting h = 17 into h(t).
2 Make RHS = 0. −5t2 + 20t − 15 = 0
t 2 − 4t + 3 = 0
3 Solve for t. (t − 1)(t − 3) = 0
Note: the quadratic formula could also t = 1 or 3
be used to solve for t.
4 The first time it reaches 17 m is the It first reaches 17 m when t = 1 s.
smaller value of t.
5 Evaluate h′(1). h′(1) = 20 − 5(1)2
= 15 m/s
Rate of change of height is 15 m/s.

It is worth noting that there are two common ways of writing the derivative as a
function. For example, the derivative of the function P(x) = x2 + 5x − 7 may be written
dP
as P′(x) = 2x + 5 or as ------- = 2x + 5 .
dx
Chapter 13 Differentiation and applications 651
WORKED Example 16
The shockwave from a nuclear
blast spreads out at ground level
in a circular manner.
a Write down a relationship
between the area of ground,
A km2, over which the shock-
wave passes and its radius,
r km.
b Find the rate of change of A
with respect to r.
c Find the rate of change of A
when the radius is 2 km.
d What is the rate of change of
A when the area covered is r km
Area A km2
314 km2?

THINK WRITE

a State the formula for the area of a circle. a A(r) = π r 2

b Differentiate A(r). b A′(r) = 2π r

c Substitute r = 2 into A′(r). c A′(2) = 2(3.14)(2)


Note: The units for the rate of change of = 12.56
A (km2) with respect to r (km) are Rate of change of A when the radius is
km2 per km or km2/km. 2 km is 12.56 km2/km.

d 1 Substitute A = 314 into the area d A(r) = π r 2


function A(r) and solve for r. 314 = 3.14r 2
314
r 2 = ----------
3.14
= 100
r = 10 since r > 0
2 Find the rate of change when r = 10. A′(10) = 2π (10)
= 62.8
Rate of change of A when area is 314 km2
is 62.8 km2/km.

remember
Average rates of change are calculated using the original function, while
differentiation of this function is needed in order to calculate instantaneous rates
of change at specific points.
652 M a t h s Q u e s t M a t h s B Ye a r 1 1 f o r Q u e e n s l a n d

13E Rates of change


WORKED 1 If f (x) = x2 + 5x + 15 find:
Example
14
a the average rate of change between x = 3 and x = 5
b a new function that describes the rate of change
c the (instantaneous) rate of change when x = 5.
2 A balloon is inflated so that its volume, V cm3, at any time, t seconds later is:
V = − 8--- t 3 + 24t 2, t ∈ [0, 10]
5
eBook plus a What is the volume of the balloon when:
Digital docs:
i t = 0? ii t = 10?
SkillSHEET 13.3 b Hence, find the average rate of change between t = 0 and t = 10.
Average rate of change c Find the rate of change of volume when
EXCEL Spreadsheet i t=0 ii t = 5 iii t = 10.
Gradient between two
points on a graph
3 multiple choice
The average rate of change between x = 1 and x = 3 for the function y = x2 + 3x + 5 is:
A 1 B 9 C 5 D 3 E 7

4 multiple choice
The instantaneous rate of change of the function f (x) = x3 − 3x2 + 4x when x = −2 is:
A 2 B −2 C 28 D 3 E 12

5 multiple choice
dy
If the rate of change of a function is described by ------ = 2x2 − 7x, then the function
could be: dx

A y = 6x3 − 14x B y = 2--- x3 − 7x C y = 2--- x3 − 7--- x2 + 5


3 3 2
3 7 2
D y=x − --- x +2 2
E 2x − 7x + 5
2

WORKED 6 In a baseball game the ball is hit


Example
15
so that its height above the
ground, h metres, is
h(t) = 1 + 18t − 3t 2
t seconds after being struck.
a Find the rate of change, h′(t).
b Calculate the rate of change
of height after:
iii 2 seconds
iii 3 seconds
iii 4 seconds.
c What happens when
t = 3 seconds?
d Find the rate of change of
height when the ball first
reaches a height of 16 metres.
Chapter 13 Differentiation and applications 653
7 The position, x metres, of a lift (above ground level) at any time, t seconds,
eBook plus is given by: x(t) = −2t2 + 40t
Digital doc: a Find the rate of change of displacement (velocity) at any time, t.
SkillSHEET 13.4 b Find the rate of change when:
Instantaneous
rate of change
i t=5 ii t = 9 iii t = 11.
c What happened between t = 9 and t = 11?
d When and where is the rate of change zero?
8 The number of seats, N, occupied in a soccer
stadium t hours after the gates are opened is
given by:
N = 500t 2 + 3500t, t ∈ [0, 5]
a Find N when:
i t = 1 and ii t = 3.
b What is the average rate of change
between t = 1 and t = 3?
c Find the instantaneous rate when:
i t=0 ii t = 1 iii t = 3 iv t = 4.
d Why is the rate increasing in the first 4 hours?
9 The weight, W kg, of a calf at any time, t weeks, after birth is given by:
3 2
W = 80 + 12t − ------t where 0 < t < 20
10
a What is the weight of the calf at birth?
b Find an expression for the rate of change of weight at any time, t.
c Find the rate of change after:
i 5 weeks ii 10 weeks iii 15 weeks.
d Is the rate of change of the calf’s weight increasing or decreasing?
e When does the calf weigh 200 kg?
654 M a t h s Q u e s t M a t h s B Ye a r 1 1 f o r Q u e e n s l a n d

3
---
10 The weekly profit, P (hundreds of dollars), of a factory is given by P = 4.5n – n 2 ,
where n is the number of employees.
dP
a Find ------- .
dn
b Hence, find the rate of change of profit, in dollars per employee, if the number of
employees is:
i 4
ii 16
iii 25.
c Find n when the rate of change is zero.
11 Gas is escaping from a cylinder so that its volume, V cm3, t seconds after the leak
1 2
starts, is described by V = 2000 − 20t − --------
-t .
100
a Find the rate of change after:
i 10 seconds
ii 50 seconds
iii 100 seconds.
b Is the rate of change ever positive? Why?
WORKED 12 Assume an oil spill from an oil tanker is circular and remains that way.
Example
16
a Write down a relationship between the area of the spill, A m2, and the radius,
r metres.
b Find the rate of change of A with respect to the radius, r.
c Find the rate of change of A when the radius is:
i 10 m
ii 50 m
iii 100 m.
d Is the area increasing more rapidly as the radius increases? Why?
13 A spherical balloon is being inflated.
a Express the volume of the balloon, V m3, as a function of the radius, r metres.
b Find the rate of change of V with respect to r.
c Find the rate of change when the radius is:
i 0.1 m
ii 0.2 m
iii 0.3 m.

r
Chapter 13 Differentiation and applications 655
14 A rectangular fish tank has a square base with its height being equal to half the length
of its base.
a Express the length and width of the base in terms of its height, h.
b Hence, express the volume, V m3, in terms of the height, h, only.
c Find the rate of change of V when:
i h=1m
ii h = 2 m
iii h = 3 m.

15 For the triangular package shown find:


6
a x in terms of h
b the volume, V, as a function of h only
c the rate of change of V when 30°
i h = 0.5 m h
x
ii h = 1 m.

16 A new estate is to be established on the side of a hill.

80

200 x

Regulations will not allow houses to be built on slopes where the gradient is greater
than 0.45. If the equation of the cross-section of the hill is
y = −0.000 02x3 + 0.006x2
find:
dy
a the gradient of the slope ------
dx
b the gradient of the slope when x equals:
i 160
ii 100
iii 40
iv 20
c the values of x where the gradient is 0.45
d the range of heights for which houses cannot be built on the hill.

17 A bushfire burns out A hectares of land t hours after it started according to the rule
A = 90t 2 − 3t 3
a At what rate, in hectares per hour, is the fire spreading at any time, t?
b What is the rate when t equals:
i 0 ii 4 iii 8 iv 10 v 12 vi 16 vii 20?
c Briefly explain how the rate of burning changes during the first 20 hours.
d Why isn’t there a negative rate of change in the first 20 hours?
e What happens after 20 hours?
f After how long is the rate of change equal to 756 hectares per hour?
656 M a t h s Q u e s t M a t h s B Ye a r 1 1 f o r Q u e e n s l a n d

Solving maximum and minimum problems


There are many practical situations where it is necessary to determine the maximum or
minimum value of a function. For quadratic functions, differentiation makes this a rela-
tively simple task. The derivative of a function gives the gradient of a tangent to the
curve. When the derivative equals zero, the tangent is horizontal. The point, or points,
on the curve where this occurs are called stationary points. These points can be local
minimum or maximum turning points or points of inflection. A function f(x) has
stationary points when f ′(x) = 0.
When solving maximum or minimum problems (to obtain the value(s) of x) it should
be verified that it is in fact a maximum or minimum by checking the sign of the deriv-
ative to left and right of the turning point. y Local f (x)
In the case of cubic and higher order polynomials, maximum Absolute
the local maximum (or minimum) may or may not be maximum in
the highest (or lowest) value of the function in a given the interval
[a, b]
domain.
An example where the local maximum, found by 0 a b x
solving f ′(x) = 0, is not the largest value of f(x) in the
domain [a, b] is shown. Here, B is the point where f(x) is greatest in this domain, and
is called the absolute maximum for the interval.

Case 1. The function is known


WORKED Example 17
A cricket fielder throws a ball so that the equation of its path is:
y = 1.5 + x − 0.02x2
where x (metres) is the horizontal distance travelled by the ball and y (metres) is the
vertical height reached.
a Find the value of x for which the maximum height is reached (verify that it is a maximum).
b Find the maximum height reached.
THINK WRITE
a 1 Write the equation of the path. a y = 1.5 + x − 0.02x2
dy dy
2 Find the derivative ------ . ------ = 1 − 0.04x
dx dx
dy dy
3 Solve the equation ------ = 0 to find the For stationary points: ------ = 0
dx dx
value of x for which height is a 1 − 0.04x = 0
maximum. −0.04x = −1
x = 25
4 Determine the nature of this stationary When x = 24,
dy dy
point at x = 25 by evaluating ------ to the ------ = 1 − 0.04(24)
dx dx Zero gradient
Positive Negative
left and right, say, at x = 24 and at x = 26. = 0.04 gradient gradient
When x = 26,
dy
------ = 1 − 0.04(26)
dx
= −0.04
Chapter 13 Differentiation and applications 657

THINK WRITE

5 Since the gradient changes from positive to The stationary point is a local maximum.
negative as we move from left to right in the
vicinity of x = 25, the stationary point is a
local maximum.

b Substitute x = 25 into y = 1.5 + x − 0.02x2 to b When x = 25,


find the corresponding y-value (maximum y = 1.5 + 25 − 0.02(25)2
height). = 14
So the maximum height reached is
14 m.

Case 2. The rule for the function is not given


If the rule is not given directly then the following steps should be followed:
1. Draw a diagram if necessary and write an equation linking the given information.
2. Identify the quantity to be maximised or minimised.
3. Express this quantity as a function of one variable only (often this will be x).
4. Differentiate, set the derivative equal to zero, and solve.
5. Determine, in the case of more than one value, which one represents the maximum
or minimum value.
6. For some functions, a maximum or minimum may occur at the extreme points of the
domain so check these also.
7. Answer the question that is being asked.
8. Sketch a graph of the function if it helps to answer the question, noting any restrictions
on the domain.

WORKED Example 18
A farmer wishes to fence off a rectangular paddock on a straight stretch of river so that
only 3 sides of fencing are required. Find the largest possible area of the paddock if
240 metres of fencing is available.

THINK WRITE

1 Draw a diagram to represent the River Let w = width


situation, using labels to represent the l = length
variables for length and width and write P = perimeter
w Fence Fence w
an equation involving the given
information. Fence
l
P = l + 2w = 240 [1]
2 Write a rule for the area, A, of the paddock A = l × w [2]
in terms of length, l, and width, w.
Continued over page
658 M a t h s Q u e s t M a t h s B Ye a r 1 1 f o r Q u e e n s l a n d

THINK WRITE

3 Express the length, l, of the rectangle l + 2w = 240


in terms of the width, w, using l = 240 − 2w [3]
equation [1].

4 Express the quantity, A, as a function of Substituting [3] into [2]:


one variable, w, by substituting [3] A(w) = (240 − 2w)w
into [2]. = 240w − 2w2

5 Solve A ′(w) = 0. A′(w) = 240 − 4w


For stationary points: A′(w) = 0
240 − 4w = 0
240 = 4w
w = 60

6 Test to see if the stationary point at When w = 59,


w = 60 will produce a maximum or A′(59) = 240 − 4(59)
minimum value for the area by evaluating =4
A ′(w) to the left and right, say, at w = 59 When w = 61,
and at w = 61. A′(61) = 240 − 4(61)
= −4

7 Since the gradient changes from positive to Zero gradient


negative as we move from left to right in the
Positive
vicinity of w = 60, the stationary point is a gradient Negative
local maximum. gradient

The stationary point is a local maximum.


The area of the paddock is a maximum when
w = 60.
8 Find the maximum area of the paddock A(60) = (240 − 2 × 60) × 60
by substituting w = 60 into the function = 7200 m2
for area.

remember
Defining a function and setting its derivative equal to zero to form an equation
helps to tell us when a local maximum or minimum occurs. The solution(s) must
then be substituted into the original function to find the actual maximum or
minimum value(s).
Chapter 13 Differentiation and applications 659
Solving maximum and
13F minimum problems
WORKED 1 A golfer hits the ball so that the
Example
17
equation of its path is:
y = 1.2 + x − 0.025x2
where x (metres) is the horizontal
distance travelled by the ball and
y (metres) is the vertical height
reached.
a Find the value of x for which
the maximum height is reached
(and verify that it is a
maximum).
b Find the maximum height
reached.

2 If the volume of water, V litres, in a family’s hot water tank t minutes after the shower
eBook plus
is turned on is given by the rule V = 200 − 1.2t 2 + 0.08t 3, where 0 ≤ t ≤ 15:
Digital docs: a find the time when the volume is minimum (that is, the length of time the shower
EXCEL Spreadsheet is on)
Quadratic graphs
b verify that it is a minimum by checking the sign of the derivative
EXCEL Spreadsheet
Cubic graphs
c find the minimum volume
d find the value of t when the tank is full again.

3 A ball is thrown into the air so that its height, h metres, above the ground at time t
seconds after being thrown is given by the function:
h(t) = 1 + 15t − 5t 2
a Find the greatest height reached by the ball and the value of t for which it occurs.
b Verify that it is a maximum.

WORKED 4 A gardener wishes to fence off a rectangular vegetable patch against her back fence so
Example
18
that only 3 sides of new fencing are required. Find the largest possible area of the
vegetable patch if she has 16 m of fencing material available.

5 The sum of two numbers is 16.


a By letting one number be x, find an expression for the other number.
b Find an expression for the product of the two numbers, P.
c Hence, find the numbers if P is a maximum.
d Verify that it is a maximum.

6 The rectangle at right has a perimeter of 20 cm. Length


a If the width is x cm, find an expression for the
length.
b Write an expression for the area, A, in terms of x Width = x
only.
c Find the value of x required for maximum area.
d Find the dimensions of the rectangle for maximum area.
e Hence, find the maximum area.
660 M a t h s Q u e s t M a t h s B Ye a r 1 1 f o r Q u e e n s l a n d

7 A farmer wishes to create a rectangular pen to contain as much area as possible using
60 metres of fencing.
a Write expressions for the dimensions (length and width) of the pen.
b Hence, find the maximum area.

8 The cost of producing a particular toaster is $(250 + 1.2n2) where n is the number
produced each day. If the toasters are sold for $60 each:
a write an expression for the profit, P, dollars
b find how many toasters should be produced each day for maximum profit
c hence, find the maximum daily profit possible.

9 A company’s income each week is $(800 + 1000n − 20n2) where n is the number of
employees. The company spends $760 per employee for wages and materials.
a Write an expression for the company weekly profit, P dollars.
b Determine the number of employees required for maximum profit and hence
calculate the maximum weekly profit.

10 The sum of two numbers is 10. Find the numbers if the sum of their squares is to be a
minimum.

11 A square has four equal squares cut out of the corners as 12 cm


shown at right. It is then folded to form an open rectangular x
box.
a What is the range of possible values for x? 12 cm
b In terms of x find expressions for the
i height, ii length, and iii width of the box.
c Write an expression for the volume, V (in terms of x
only).
d Find the maximum possible volume of the box.

12 The base and sides of a shirt box are to be made from a rectangular sheet of cardboard
measuring 50 cm × 40 cm. Find:
a the dimensions of the box required for maximum volume
b the maximum volume.
(Give answers correct to 2 decimal places.)

13 The volume of the square-based box shown at right is 256 cm3.


a Find h in terms of l.
If the box has an open top find:
b the surface area, A, in terms of l only h
c the dimensions of the box if the surface area is to be l
a minimum l
d the minimum area. (Hint: 1 --- = l –1.)
l
14 A closed, square-based box of volume 1000 cm3 is to be constructed using the
minimum amount of metal sheet possible. Find its dimensions.
1 2
15 The cost of flying an aircraft on a 900 km journey is 1600 + --------
- v dollars per hour,
eBook plus 100
where v is the speed of the aircraft in km/h. Find:
Digital doc: a the cost, C dollars, of the journey if v = 300 km/h
WorkSHEET 13.2 b the cost, C dollars, of the journey in terms of v. (Hint: time = distance ÷ speed.)
c the most economical speed and minimum cost.
Chapter 13 Differentiation and applications 661
When is a maximum not a maximum?
Often finding a maximum (or minimum) of a function is achieved by setting the
derivative equal to zero and solving for the independent variable (such as x). This
value of x is then substituted into the original function to find the maximum
(or minimum) value of the function.
One danger of this method is that it assumes the local maximum or minimum is
the absolute maximum or minimum.
Consider finding the maximum value of the function f(x) = ---- x 3- − 2x2 + 3x + 2
between x = 0 and x = 6: 3
f ′(x) = x2 – 4x + 3
f ′(x) = (x − 1)(x − 3)
So x = 1 or x = 3 for a local maximum or minimum.
A computer generated graph of f(x) reveals that the absolute maximum of f(x) in
the interval [0, 6] is not at x = 1 or x = 3!
3
y f (x) = x–3 – 2x2 + 3x + 2
10
5

0 x
–2 2 4 6
–5
–10
The absolute maximum of f(x) obviously occurs at x = 6, and equals
63
f(6) = ----- − 2(32) + 3(3) + 2
3
= 20
1 Find the absolute maximum of f(x) = x3 + 5x2 − 8x − 12 between x = 0 and x = 3.
2 Find the minimum of f(x) = −x3 + 4x2 + 11x − 30 in the interval [1, 5].
3 The temperature T °C of a pottery classroom x minutes after the class has started
is described by the function T(x) = 0.000 08x(x + 2)2 + 21.
Sketch a graph of the temperature during a 50-minute class and determine when
the classroom is hottest, and what the temperature is then.
662 M a t h s Q u e s t M a t h s B Ye a r 1 1 f o r Q u e e n s l a n d

summary
Deriving the gradient function y y = f (x)
rise Q
• Gradient of a secant = -------- f (x + h)
run
f ( x + h) – f ( x) Tangent at P
= --------------------------------------
h f (x) P
• f ′(x) is the gradient function of f (x).
f ( x + h) – f ( x) 0 x x+h x
• f ′(x) = lim -------------------------------------- , h ≠ 0
h→0 h
Differentiation using first principles
f ( x + h) – f ( x)
• The process of finding the gradient function lim -------------------------------------- is called
differentiation from first principles. h→0 h
dy y
• Differentiating y gives ------ .
dx Q
• Differentiating f (x) gives f ′(x). Q moves closer to
Q P as h approaches 0.
Finding derivatives by rule
P Q
• If f ( x) = xn, then f ′( x) = nx n − 1.
n n−1
• If f ( x) = ax , then f ′( x) = nax .
• If f ( x) = c, then f ′( x) = 0 (where c is 0 h x
constant).
• If f ( x) = g ( x) + h( x), then f ′( x) = g ′( x) + h′( x).
• Equation of tangent: y − y1 = m T (x − x1)
1
• Equation of normal: y − y1 = m N (x − x1) where mN = − -------
mT
Rates of change
change in y
• Average rate of change = ---------------------------
change in x
dy
• The derivative of a function, f ′(x) (or ------ ), is needed in order to calculate the
dx
(instantaneous) rate of change at a particular point. The rate of change of a
function, f (x), at x = a is given by f ′(a).
Solving maximum and minimum problems y Local f (x)
maximum
• By solving the equation f ′(x) = 0, and substituting the Absolute
solutions into the original function, the maximum or maximum in
the interval
minimum value of a quantity may be found. When the [a, b]
function is not provided it is necessary to formulate a 0 a b x
rule in terms of one variable using the information given.
Drawing a diagram to represent the situation is often useful.
• Always test to determine whether a stationary point is a maximum or a minimum
by checking the sign of the gradient to the left and right of the point.
• Check whether or not the local maximum or minimum is the absolute maximum or
minimum. The absolute maximum or minimum may be the value of the function at
one end of a specified interval.
Chapter 13 Differentiation and applications 663

CHAPTER
review
1 multiple choice
13A
The lim ( 3x – 7 ) is:
x→5
A −7 B undefined C 15 D −2 E 8

2 multiple choice
13A
The lim ( x 2 – 2x + 5 ) is:
x → –3
A 20 B undefined C 5 D 8 E 0
3 Find lim ( 3x – 12 ) .
x→2 13A
4 If f (x) = x3 − 3x + 3 find lim f ( x ) .
x→0 13A
5 multiple choice
13B
x 2 + 3x – 10
The lim ----------------------------- is equal to:
x→2 x–2
A 1 B 7 C 0 D −3 E 5
Questions 6 to 8 refer to the function f (x) graphed at right. y f (x)

6 multiple choice 2
1
13B
The lim f ( x ) equals:
x→0
A 1 B ∞ C 4 D 0 E −1 –1 0 1 2 x

7 multiple choice
13B
The lim f ( x ) is:
x → –1
A 1 B 2 C undefined D 0 E −1

8 multiple choice
13B
The lim f ( x ) is:
x→1
A 1 B 2 C undefined D 0 E −1

x 2 + 7x + 12
9 a By first factorising the numerator, simplify the rational function f ( x ) = ------------------------------ ,
x+3 13B
stating the value for which the function does not exist (that is, is discontinuous).
b Find lim f ( x ) , where a is the value at which f (x) is discontinuous.
x→a
664 M a t h s Q u e s t M a t h s B Ye a r 1 1 f o r Q u e e n s l a n d

10 a Sketch the graph of the hybrid function f (x) = x + 1, x ∈ (−∞, 0)


13B {
2, x ∈ [0, ∞)
b Find xlim
→0
f (x).

11 multiple choice
13C
f ( x + h) – f ( x)
If f (x) = x2 − 2x then lim --------------------------------------- equals:
h→0 h
A 2x − 2 B 2x + h C 2x D −2 E x2

12 multiple choice
13C
If f (x) = x 2 + 7x, then f ′(2) is equal to:
f (2 + h) – f (2) f (2 + h) – f (2) f ( x + h) – f (2)
A -------------------------------------- B lim -------------------------------------- C --------------------------------------
h h→0 h h
f (2 + h) f (h)
D lim -------------------- E lim -----------
h→0 h h→0 h

13 multiple choice
13C
The gradient of the tangent to the curve f (x) at x = 5 is:
f ( x + h) – f (5) f (5) f (5 + h) – f ( x)
A lim -------------------------------------- B lim ----------- C lim --------------------------------------
x→0 h h→0 h h→0 h
f (5 + h) – f (5) f (h)
D lim -------------------------------------- E lim -----------
h→0 h h→0 h

f ( x + h) – f ( x)
14 Find lim --------------------------------------- ; that is, find f ′(x) for f (x) = 2x + 3.
13C h→0 h

15 Differentiate f (x) = 5 + 4x − 3x 2 using first principles.


13C
16 multiple choice
13D
If g(x) = 18 − 3x − 7x 2, then g′(x) equals:
A 15 − 4x B −3 − 7x C 3 − 14x D 18 E −3 − 14x

17 multiple choice
13D
dy
If y = x3 + 10x2 − 7x + 2, then ------ is equal to:
dx
1 2 10
A --- x + ------ x −7 B x 2 + 10x − 7 C 3x2 + 10x − 7
2 3
2 E 20x − 7
D 3x + 20x − 7

18 multiple choice
13D
The function f (x) = (x − 3)(x + 5), so f ′(x) equals:
A x+5 B 2x + 2 C x−3 D 1 E 2x + 5
Chapter 13 Differentiation and applications 665
19 multiple choice
dy
13D
If y = x3 − 10x 2 then ------ equals:
dx
3x 2 – 20x
A 3x2 − 20x B x − 10 C ----------------------- D 1 − 20x E x 2 − 10x
2x

20 multiple choice
The derivative of 2x−3 is:
13D
6
A 6x2 B – ----- C −6x−4 D −6x−2 E −6x
x2

21 multiple choice
When differentiated, 5 3 x is equal to:
13D
5 – 2--- – 1--- 1
---
2
---
2
---
A --- x 3 B 15x 2 C 15x 3 D x3 E 5x 3
3

22 multiple choice
Given that f (x) = 2x 2 + 7x − 1, f ′(−1) equals:
13D
A 7 B −11 C −1 D −4 E 3
23 multiple choice
13D
The gradient of y = x 2 − 7x − 5 equals 3 when x is:
A 8 B 5 C −1 D 0 E −5
24 If g(x) = 1--- x3 − 7--- x2 − 8x + 1:
3 2 13D
a find g′(x)
b evaluate i g′(3) ii g′(−2)
c find the coordinates when the gradient is 0.
25 Find the coordinates where the gradient of the tangent to y = 2x2 − 5x + 2 is:
a parallel to the x-axis 13D
b parallel to the line 1 − 3x
c equal to the gradient of the function g(x) = 1--- x3 − x2 for the same value(s) of x.
3

26 multiple choice
The rate of change of f (x) = 2x3 − 5x2 + 7 when x = 2 is:
13E
A −4 B 7 C −36 D 0 E 4
27 multiple choice
If V = −3t 2 + 7t + 50 then the average rate of change between t = 1 and t = 4 is:
13E
A −10 2--- B −10 C −6 D −8 E 0
3
28 If the position of a particle moving in a straight line is given by the rule
x(t) = −2t 2 + 8t + 3, where x is in centimetres and t is in seconds, find: 13E
a the initial position of the particle
b the rate of change of displacement (that is, the velocity) at any time, t
c the rate of change when t = 4
d when and where the velocity is zero
e whether the particle is moving to the left or to the right when t = 3
f the distance travelled in the first 3 seconds.
666 M a t h s Q u e s t M a t h s B Ye a r 1 1 f o r Q u e e n s l a n d

29 multiple choice
13F
A curve with a local maximum and a local minimum is:
A y = x3 + 2x2 − 7x + 1 B y = x2 − 3x + 1 C y = x3 + 7
3
D y = (x − 2) E y = x2 + 6x

30 If the volume of liquid in a vat, V litres, during a manufacturing process is given by


13F V = 6t − t 2, where t ∈ [0, 6], find:
a the rate of change 2 hours after the vat starts to fill up
b when the vat has a maximum volume.

31 If a piece of wire is 80 cm long:


13F a find the area of the largest rectangle that can be formed by the wire
b determine whether a circle would give a larger area.

Modelling and problem solving


1 Find the maximum possible volume of a fully enclosed cylindrical water tank given that the
total internal surface area of the tank is 600 π square units.

2 A mountain trail can be modelled by the curve with equation y = 1.8 + 0.16x − 0.005x4,
where x and y are, respectively, the horizontal and vertical distances measured in kilometres,
0 < x < 3.
a Find the gradient at the beginning and end of the trail.
b Calculate the point where the gradient is 0.
c Verify that this point represents the maximum gradient within the given domain by tracing
the function with a graphics calculator.
d Hence, state the maximum height of the path.
e Find the point where the path is lowest and, hence, state the minimum height.

3 A piece of wire of length 100 cm is to be cut so that one piece is used to form a square, while
the other is used to form a circle. What is the minimum area of the square and the circle
combined?
y
4 The graph of g′(x) is shown at right.
If g(0) = 6 and g(3) = 0, sketch the graph of g(x). 0 x
3

5 A giant park slide is made from a straight section


of metal sheet joined to a curved section of metal g'(x)
sheet such that its profile is modelled by the hybrid
function
 10 – 2x, x ∈ [0, 4]
f ( x ) = 1 2
 2 (x – 6) , x ∈ [4, 5]
where x is the horizontal distance in metres from the start of the slide.
a Find the height of the slide at the start.
eBook plus b Find the height of the slide at the end.
c Sketch the graph of f(x).
Digital doc: d Given that the two sections are joined at a horizontal distance of 4 metres from the start of
Test Yourself
Chapter 13 the slide, find the exact length of the straight section.
e Show that the slide is smoothly joined so that children will not experience a bump.
Appendix 667

Instructions for the TI-89 Titanium graphics calculator


Chapter 1 — Modelling using linear functions
Graphics Calculator tip: Solving equations (page 5) ................................................. 669
Isolating a pronumeral (page 10) ................................................................................ 670
Graphics Calculator tip: Substituting for pronumerals (page 11)............................... 671
Graphics Calculator tip: Graphing linear functions (page 24).................................... 672
Graphics Calculator tip: Finding x- and y-intercepts (page 32).................................. 673
Worked example 12 (page 34)..................................................................................... 674
Graphics Calculator tip: Solving simultaneous equations algebraically (page 38) .... 676
Graphics Calculator tip: Finding the equation of a line through two given points
(page 43) ................................................................................................................. 677
Chapter 2 — Relations and functions
Graphics Calculator tip: Plotting points (page 65) ..................................................... 679
Graphics Calculator tip: Graphing relations with a restricted domain (page 73)....... 680
Graphics Calculator tip: Defining and using functions (page 83) .............................. 681
Graphics Calculator tip: Graphing a hybrid function (page 86) ................................. 682
Graphics Calculator tip: Graphing a circle (page 94) ................................................. 683
Chapter 3 — Other graphs and modelling
Graphics Calculator tip: Graphing the absolute value function (page 144) ............... 684
Graphics Calculator tip: Addition of ordinates (page 150)......................................... 685
Modelling using a graphics calculator (page 161)...................................................... 687
Chapter 4 — Triangle trigonometry
Graphics Calculator tip: Degrees mode (page 177) .................................................... 689
Graphics Calculator tip: Calculating trigonometric ratios involving degrees
and minutes (page 179)........................................................................................... 690
Graphics Calculator tip: Displaying an answer in degrees, minutes and seconds
(page 183) ............................................................................................................... 691
Graphics Calculator tip: Using the equation solver (page 186).................................. 692
Graphics Calculator tip: Using the equation solver to find an angle (page 192) ....... 693
Graphics Calculator tip: Finding a solution in degrees and minutes (page 194) ....... 694
Graphics Calculator tip: Using equation solver to solve cosine rule problems
(sides) (page 216) .................................................................................................. 695
Chapter 5 — Graphing periodic functions
Graphics Calculator tip: The graph of y = sin x (page 256) ....................................... 696
Investigation: The effect of 2 (page 259) .................................................................... 698
Chapter 6 — Trigonometric equations
Graphics Calculator tip: Solving a simple trigonometric equation (page 280) .......... 699
668 Appendix

Chapter 7 — Exponential and logarithmic functions


Graphics Calculator tip: Simplifying expressions (page 304) .................................... 700
Solving indicial equations using a graphics calculator (page 313) ............................ 701
Worked example 13 (page 318) .................................................................................. 702
Investigation: A world population model (page 321) ................................................. 703
Investigation: Logarithmic graphs (page 332) ........................................................... 704
Chapter 8 — Applications of exponential and logarithmic functions in
financial mathematics
Graphics Calculator tip: Listing the terms of a geometric sequence (page 351) ...... 705
Graphics Calculator tip: Finding the sum of a geometric sequence (page 356) ........ 706
Worked example 6 (page 363) .................................................................................... 707
Worked example 8 (page 367) .................................................................................... 709
Graphics Calculator tip: Financial calculations using TVM (page 405) .................... 710
Worked example 23 (page 408) .................................................................................. 710
Chapter 9 — Presentation of data
Graphics Calculator tip: Choosing a random sample (page 432)............................... 710
Graphics Calculator tip: Constructing a histogram (page 452) .................................. 711
Chapter 10 — Summary statistics
Graphics Calculator tip: Calculating the mean of data in a list (page 482) ............... 712
Graphics Calculator tip: Calculating the mean of data in a frequency table
(page 485) ............................................................................................................... 713
Graphics Calculator tip: Calculating the median and the mode (page 488) .............. 713
Graphics Calculator tip: Finding summary statistics (page 496) ............................... 714
Worked example 9 (page 503) .................................................................................... 715
Graphics Calculator tip: Creating a boxplot from a frequency table (page 511) ....... 716
Worked example 14 (page 520) .................................................................................. 717
Chapter 11 — Introduction to probability
Graphics Calculator tip: Random number generation (page 552).............................. 718
Worked example 14 (page 554) .................................................................................. 719
Worked example 15 (page 555) .................................................................................. 720
Chapter 12 — Rates of change
Worked example 8 (page 585) .................................................................................... 721
Graphics Calculator tip: Relating the gradient function to the original function
(page 599) ............................................................................................................... 723
Chapter 13 — Differentiation and applications
Graphics Calculator tip: Finding the limit of a rational function (page 631) ............ 724
Graphics Calculator tip: Finding the derivative using first principles (page 638) ..... 725
Graphics Calculator tip: Finding the derivative (page 643)........................................ 726
Graphics Calculator tip: Graphing the derivative function (page 647) ...................... 727
Appendix 669
Chapter 1 page 5
Graphics Calculator tip! Solving equations

A graphics calculator can be used to solve an equation. To find the solution to the linear
x
equation 3(x − 2) + 4 = --- + 1, follow the steps below.
3
1. From the MENU (press APPS or 2ND [QUIT]),
highlight Home.

2. Press ENTER to select this application. Press


CATALOG and use the arrow keys to scroll down to
access solve(. You can find it more quickly by first
pressing ALPHA [S].

3. Press ENTER to select solve(. This will be


displayed in the function entry line of the Home
screen. Enter the equation and then press ,
followed by X to indicate that we are solving with
respect to the variable x. Press ) to close the set of
brackets.
4. Press ENTER to display the solution. This gives the
solution as a common fraction.

5. To obtain the solution in decimal notation, the


calculator will need to be put in APPROX mode. To
do this, press MODE followed by F2 (Page 2) for
the second page of options. Press the down arrow
key until the entry next to Exact/Approx is
highlighted and then press the right arrow to display
your options. Highlight 3: APPROXIMATE.
6. Press ENTER to accept this setting and then again
to return to the Home screen. Press ENTER to
recalculate the solution to the equation. The solution
9
is --- or 1.125.
8
670 Appendix

Chapter 1 page 10
Isolating a pronumeral

Some graphics calculators are able to isolate a pronumeral in an equation. For the
TI-89 Titanium graphics calculator, the solve function has been used to rearrange the
equation v2 = u2 + 2as to make s the isolated pronumeral. (When entering 2as on the
calculator, remember to enter a multiplication sign between each pronumeral.) Notice
that the pronumeral shown after the comma is s, the pronumeral being solved for.
To enter the solve function into the function entry
line of the Home screen, press CATALOG and then
select solve(. Alternatively, press F2 (Algebra) and
then select 1: solve(.
Appendix 671
Chapter 1 page 11
Graphics Calculator tip! Substituting for pronumerals

Consider the equation v2 = u2 + 2as given in Worked example 5 on page 10. In part c,
we were asked to find the distance travelled by an object which begins with a velocity
of 10 m/s, and has a final velocity of 4 m/s while accelerating at −1 m/s2. With a
graphics calculator, we can substitute directly into the given equation and find the value
for s.
1. From the MENU, select Home. (To select Home,
highlight Home and then press ENTER .) Enter the
equation v2 = u2 + 2as. Use the ALPHA key to enter
each variable. It is important to enter a multiplication
sign between variables.

2. To enter the known values of the pronumerals v, u


and a, we first press Ù . This is the ‘given’ or ‘with’
symbol and is located on a grey button beneath the
equals button ( = ). Enter u = 10. (Press ALPHA [U]
to enter the variable u.)

3. Follow each piece of known information with the


word ‘and’. To enter the word ‘and’, press
CATALOG and then scroll down to and.

4. Press ENTER to insert and into the function entry


line. Now enter v = 4 and a = −1 with and entered
between each. Press ENTER to display the equation
in terms of s.

5. To solve for s, first press CATALOG and then select


solve(. Press 2ND [ANS] to enter the answer from
the previous line, followed by a comma and the
variable s. (Press , followed by ALPHA [S].)
Close the set of brackets (press ) ) and press
ENTER to display the solution.
672 Appendix

Chapter 1 page 24
Graphics Calculator tip! Graphing linear functions

A graphics calculator can be used to draw the graph of a linear function. As an


example, let’s draw the graph of y = 2x − 7.
1. From the MENU, highlight Graph.

2. Press ENTER to select Graph. A set of axes for x


and y (with default settings for the axis scale) will be
displayed.

3. Press ♦ [Y=]. Enter 2x − 7 in the function entry line


for y1(x) and then press ENTER .

4. Press ♦ [GRAPH] to display the line.

5. If you wish to alter the scale on each axis, press


♦ [WINDOW]. Enter −5 for xmin, 5 for xmax,
−5 for ymin and 5 for ymax.

6. Press ♦ [GRAPH] to redraw the line with these


settings.
Appendix 673
7. Press F3 (Trace). A point on the line with its
coordinates will be displayed. The arrow keys can
then be used to navigate the graph and explore points
that lie on the graph.

Chapter 1 page 32
Graphics Calculator tip! Finding x- and y-intercepts

To find the x- and y-intercepts from a graph displayed on a graphics calculator, follow
the steps below. As an example, we will consider the graph of y = 2x − 7 that we
produced earlier.
1. Draw the graph of y = 2x − 7. (Refer to the previous
Graphics calculator tip on page 672.)

2. To find the y-intercept, press F3 (Trace). This will


automatically display a point on the line where x is 0.
The coordinates will also be shown. The y-intercept
is −7.

3. To find the x-intercept, press F5 (Math) and then


select 2: Zero. (To select 2: Zero, press 2 or
highlight 2: Zero and then press ENTER .)

4. We need to set limits between which to search for a


zero. Press ENTER at any point on the left of the
zero and then use the arrow keys to move to any
point to the right of the zero. Press ENTER and the
x-intercept will be displayed.
674 Appendix

Chapter 1 page 34
WORKED Example 12
Use a graphics calculator to solve the following simultaneous equations graphically and
sketch the screen from which the solution was obtained.
y = -3x + 5
4x - 7y + 8 = 0
THINK WRITE/DISPLAY
1 Convert the second equation into y = form so it 4x − 7y + 8 = 0
may be entered in a graphics calculator. 4x + 8 = 7y
7y = 4x + 8
4 8
y = --- x + ---
7 7
2 Enter the two equations in a graphics calculator
so that a graph can be produced showing both
lines on the same set of axes. Find the point of
intersection of the two lines.
1. From the MENU, select Graph. Enter
−3x + 5 for y1(x) in the function entry line
and then press ENTER . Press ♦ [GRAPH]
to display the graph of the first equation.

4 8
2. Enter --- x + --- for y2(x) in the function entry
7 7
line and then press ENTER . Press
♦ [GRAPH] to display this graph on the
same set of axes.

3. Adjust the Window settings if necessary


(press ♦ [WINDOW]). In this example,
xmin is −6, xmax is 6, ymin is −6 and ymax
is 6. Press ♦ [GRAPH] to display the
graph with the new settings.

4. To find the point of intersection, press


F5 (Math) and then select 5: Intersection.
Use the arrow keys to move the pointer to
one of the lines. A graph label will appear
and the line will begin flashing.
Appendix 675
THINK WRITE/DISPLAY
5. Press ENTER to click on the line, then move
the pointer and click on the second line. You
will then need to press ENTER at a point
either side of the intersection. The
coordinates of the point of intersection will
then appear. The coordinates are (1.08, 1.76).
y
3 Draw a sketch of the solution screen. 5
4x – 7y + 8 = 0

8
— (1.08, 1.76)
7

–2 0 5
x

3
y = –3x + 5

4 State the solution. The solution to the simultaneous


equations y = −3x + 5 and 4x − 7y + 8 = 0
is x = 1.08 and y = 1.76 or (1.08, 1.76).
676 Appendix

Chapter 1 page 38
Graphics Calculator tip! Solving simultaneous equations
algebraically
Consider the following simultaneous equations solved graphically in Worked example 12
on page 34:
y = −3x + 5 and 4x − 7y + 8 = 0
We can also solve these equations algebraically with a graphics calculator.
1. From the MENU, select Home. Press CATALOG
and then select solve(. Enter the first equation
y = −3x + 5.

2. Press CATALOG and then scroll down to and.

3. Press ENTER to display and in the function entry


line. Now enter the equation 4x − 7y + 8 = 0. Enter a
comma followed by the variable x. Close the set of
brackets and press ENTER to display the solution.
(To obtain the solution in decimal notation, the
calculator will need to be put in APPROX mode.
Refer to the instructions on page 669.) The solution
is (1.08, 1.76) or ( 27 44
------ , ------ ).
25 25
Appendix 677
Chapter 1 page 43
Graphics Calculator tip! Finding the equation of a line
through two given points
Consider the points (2, −5) and (−3, 1) given in Worked example 17 on page 42. We
can use a graphics calculator to find the equation of the straight line containing these
points.
1. From the MENU, highlight Stats/List Editor.

2. Press ENTER to select this application. Enter x for


the title of List 1 and y for the title of List 2 and then
enter the x- and y-values of the two points.

3. Press F2 (Plots) and then select 1: Plot Setup


followed by F1 (Define). Select a scatterplot for
Plot Type and complete the settings as shown.

4. Press ENTER to accept the settings. Press


F5 (ZoomData) to view the graph.

5. Press APPS and return to the Stats/List Editor.


Press F4 (Calc) and select 3: Regressions
followed by 2: LinReg (ax + b).

6. Enter x in X List and enter y in Y List. The equation


should be stored to y1(x).
678 Appendix

7. Press ENTER to display the coefficients a and b for


y = ax + b. The equation of the line is y = −1.2x − 2.6.

8. Press ENTER to accept and then press ♦ [GRAPH]


to see the graph of the linear equation. (To see the
equation listed as y1(x), press ♦ [Y=].)
Appendix 679
Chapter 2 page 65
Graphics Calculator tip! Plotting points

A graphics calculator can be used to plot points rather than draw a continuous graph
based on a known formula or rule. To plot the points (1, 3), (2, 5), (6, 6), (9, 6), follow
the instructions below.
1. From the MENU, select Data/Matrix Editor. Select
3: New and enter the settings shown.

2. Press ENTER to accept the settings and then enter


the x-values of the points in column 1 (c1) and the
corresponding y-values in column 2 (c2). Press
ENTER after each entry.

3. Press F2 (Plot Setup) and then F1 (Define) to


define the plot. Enter the settings shown.

4. Press ENTER to save the plot settings, then press


♦ [WINDOW] to adjust the axes settings so that
xmin is 0, xmax is 10, ymin is 0 and ymax is 10.

5. Press ♦ [GRAPH] to display the points.

6. To view the coordinates of the points, press


F3 (Trace) and use the arrow keys to move between
points.
680 Appendix

Chapter 2 page 73
Graphics Calculator tip! Graphing relations with a restricted
domain
We can specify the restricted domain of a relation when entering the rule on a graphics
calculator. Consider the graph of y = x2 – 4, x ∈ [0, 4].
1. From the MENU, select Graph. Press ♦ [Y=] and
then enter x2 – 4 for y1(x).

2. To enter a relation with a restricted domain, first


press Ù and enter x ≥ 0. To enter ≥, select the
symbol from the CATALOG menu or press
2ND ♦ [>].

3. Press CATALOG and select and, then enter x ≤ 4 and


press ENTER .

4. Press ♦ [WINDOW] and adjust the settings so that


xmin is −5, xmax is 5, ymin is −5 and ymax is 20.

5. Press ♦ [GRAPH] to display the function.

6. To view the coordinates of points along the curve


and to investigate the range of the relation, press
F3 (Trace) and then use the arrow keys to move
the point (or trace) along the curve.
Appendix 681
Chapter 2 page 83
Graphics Calculator tip! Defining and using functions

Consider the function f(x) = x2 − 3 shown in Worked example 9 on page 82. We can
define this function and then use it to evaluate the function for different x-values.
1. From the MENU, highlight Y= Editor.

2. Press ENTER to select this application. Enter


x2 − 3 for y1(x) and then press ENTER .

3. We can now evaluate the function for different values


of x. Press HOME to display a Home screen. To find
the function value when x is −2, enter y1(–2) in the
function entry line and press ENTER . Similarly, to
find the function value when x is 5, enter y1(5) and
press ENTER . To find the expression for the function
when x is 2a, enter y1(2a) and press ENTER .
682 Appendix

Chapter 2 page 86
Graphics Calculator tip! Graphing a hybrid function

To graph the hybrid function shown in Worked example 12 on page 85, follow the
instructions below.
1. From the MENU, select Graph. To enter a hybrid
function in the function entry line, we need to use
when( from the CATALOG menu. The when
function has syntax when(condition,
trueExpression, falseExpression) and is designed
for 2-piece functions.
x, x <x 0< 0
x,
For example, consider the function y = 5 − x, x ≥ 0
 5 – x, x ≥ 0
This would be entered as when(x < 0, x, 5 – x) and is
displayed as shown in the screen above right.

2. Since we have three expressions, we need to nest


another when function under the true part of the
expression. The following steps are required to enter
the 3-piece hybrid function for Worked example 12:
(a) Press CATALOG and select when(.
(b) Enter x < 0 followed by a comma (press , ) and
then enter x followed by a comma.
(c) Press CATALOG and again select when(.
(d) Enter x < 2 followed by a comma and then enter
x + 1 followed by a comma.
(e) Enter 5 – x. Press ) twice to close the two sets
of brackets and then press ENTER .

3. Press ♦ [WINDOW] and adjust the settings so that


xmin is −10, xmax is 10, ymin is −10 and ymax is
10.

4. Press ♦ [GRAPH] to display the function. To view


the coordinates of points along the curve and to
investigate the range of the relation, press
F3 (Trace) and then use the arrow keys to move
the point (or trace) along the function.
Appendix 683
Chapter 2 page 94
Graphics Calculator tip! Graphing a circle

To graph the circle with the equation (x + 3)2 + (y + 2)2 = 9 shown in part b of Worked
example 14 on page 93, we need to graph two separate functions.
To find the rules for the two functions, we can rearrange the equation to isolate y.
(x + 3)2 + (y + 2)2 = 9
(y + 2)2 = 9 − (x + 3)2
2
y+2 =± 9 – ( x + 3)
2
y = −2 ± 9 – ( x + 3)
2
So the rule for the upper semicircle is given by y = −2 + 9 – ( x + 3 ) and the rule
2
for the lower semicircle is given by y = −2 − 9 – ( x + 3 ) . To obtain the graph of the
circle we will graph both functions on the one set of axes.

1. From the MENU, select Graph. Press ♦ [Y=] and


2
enter −2 + 9 – ( x + 3 ) in the function entry line
for y1(x).

2. Press ♦ [GRAPH] to display the upper semicircle.

2
3 Press ♦ [Y=] and enter −2 − 9 – ( x + 3 ) in
the function entry line for y2(x). Press ♦ [GRAPH]
to display the lower semicircle on the same set of
axes.

4. To see the circle in true proportion, we can use the


zoom function. Press F2 (Zoom) and then select
5: ZoomSqr. (You can also select 2: ZoomIn to
enlarge the circle.)
684 Appendix

5. To view the coordinates of points along the curve


and to investigate the range of the relation, press
F3 (Trace) and then use the arrow keys to move
the point (or trace) along the curve.

Chapter 3 page 144


Graphics Calculator tip! Graphing the absolute value
function
To graph the function y = x – 2 + 1, follow the instructions below.
1. From the MENU, select Graph. Press ♦ [Y=] to
display the function entry screen. To obtain the
absolute value function, press CATALOG and then
scroll down to abs(.

2. Press ENTER to display abs( in the function entry


line. Complete the entry line as shown in the screen
at right.

3. Press ♦ [GRAPH] to display the graph.

4. If you wish to adjust the view of the graph, press


♦ [WINDOW] and then enter the desired settings.
(In this example, xmin is −3, xmax is 6, ymin is −5
and ymax is 5.) To explore points of interest on the
graph, press F3 (Trace) and then use the arrow keys
to move the point (or trace) along the graph.
Appendix 685
Chapter 3 page 150
Graphics Calculator tip! Addition of ordinates

To explore addition of ordinates on a graphics calculator, let’s consider the two


functions f(x) = x2 and g(x) = 3x − 4. We can graph both of these functions on the one
set of axes and then graph the resulting function f(x) + g(x). The steps are shown below.
You may like to then investigate the addition of ordinates for the graphs of f(x) = 0.25x3
and g(x) = (x − 1)2 to obtain the resulting function f(x) + g(x).
1. From the MENU, select Graph. Press ♦ [Y=] and
enter x2 for y1(x) and then press ENTER . Enter
3x − 4 for y2(x) and then press ENTER .

2. Press ♦ [GRAPH] to display the two graphs.

3. Press ♦ [Y=] and enter y1(x) + y2(x) in the function


entry line for y3(x).

4. Press ♦ [GRAPH] to display the graph of y3(x) on


the same axes.

5. To highlight the graph of y3(x), we can change the


appearance of the curve. Press ♦ [Y=] and highlight
the y3 function. Press 2ND [F6] and then select
4: Thick.
686 Appendix

6. Press ♦ [GRAPH] to redraw the graphs. To explore


points of interest on the graph, press F3 (Trace) and
then use the arrow keys to move the point (or trace)
along the curve.

Note: The 3 shown in the top right of the screen


indicates that the cursor is moving along y3(x).
Appendix 687
Chapter 3 page 161
Modelling using a graphics calculator

Refer to page 161 for the data to be used.


1. From the MENU, select Stats/List Editor. Enter the
data for the distance from the hoop post in List 1 and
the accuracy of the basketballers in List 2. Press
ENTER after each entry. Label List 1 as dist and
List 2 as acc.

2. Press F2 (Plots) and then select 1: Plot Setup.


Press F1 (Define) and select a scatterplot for
Plot Type. Complete the settings as shown in the
screen at right.

3. Press ENTER to accept the settings. Press


F5 (ZoomData) to view the graph.

4. Press APPS and return to the Stats/List Editor.


Press F4 (Calc) and select 3: Regressions.

5. To try a linear model for the data, select


2: LinReg (ax + b). This brings up a screen to define
the regression. For X List, enter dist and then press
ENTER . Similarly, enter acc for Y List and then
press ENTER . (Note that you can save the regression
equation as y1(x). This means that you can easily
graph the equation if you wish.)
6. Press ENTER to display the details of the linear
regression. This shows the model would have an
equation of y = ax + b where a = −1.14 and b = 7.69.
(Remember that y represents accuracy and x
represents distance.) However, the r2 value is 0.64
which indicates a poor fit to the data.
688 Appendix

7. Press ENTER to return to the Stats/List Editor


screen. As before, press F4 (Calc) and select
3: Regressions. To try a quadratic model for the
data, select 4: QuadReg. Again enter dist for X List
and acc for Y List but, this time, store the regression
equation to y2(x).
Details of the regression equation are displayed,
showing that the model would have an equation of
y = ax2 + bx + c where a = 0.64, b = −4.99 and
c = 12.40. The value of 0.91 for r2 indicates that the
model is a reasonable match for the data.
8. To see if a better match can be found, we will perform
a cubic regression on the original data. Repeat step 7
but select 5: Cubic Regression and store the equation
to y3(x). This shows that the model would have an
equation of y = ax3 + bx2 + cx + d where a = −0.32,
b = 3.50, c = −12.62 and d = 18.15. The value of 0.98
for r2 indicates that the model is a good fit for the data.
This model has the equation
y = −0.32x3 + 3.5x2 – 12.62x + 18.15
where y represents the accuracy and x represents
the distance.
9. To see how the models fit the data, we can draw the
data points and the curves for the different models on
the same axes. Press ♦ [GRAPH] to view the
graphs.
Appendix 689
Chapter 4 page 177
Graphics Calculator tip! Degrees mode

The following steps show how to set your calculator in degrees mode.
1. From the Home screen, press MODE .

2. Use the down arrow key to scroll down to the Angle


line. Press the right arrow to display the available
options. Highlight 2: DEGREE.

3. Press ENTER to accept this.

4. The calculator also needs to be set to approximate


values so that it will give decimal answers to
trigonometric expressions. Press F2 (Page 2) to
move to the second page of settings. Scroll down to
the Exact/Approx line and then press the right arrow
to display the available options. Highlight
3: APPROXIMATE.
5. Press ENTER to accept this setting and then press
ENTER again to save the settings and return to the
Home screen. Both DEG and APPROX can be seen
at the bottom of the screen.
690 Appendix

Chapter 4 page 179


Graphics Calculator tip! Calculating trigonometric ratios
involving degrees and minutes
The following steps show how to calculate the value of both tan 60° and tan 49°32.
Remember to check that your calculator is in degrees mode.
If the angle is in degrees only, the tangent of the angle is found in exactly the same
way as if using a scientific calculator. Check you can do this by finding the value of
tan 60° as shown in Worked example 1 on page 178. If you get a different answer it
may be because the calculator is not set to degrees or approximation. Check the
bottom of your screen to see if it is set correctly.
To calculate the tangent of an angle in degrees and minutes (for example,
tan 49°32′), follow these steps.
From the MENU, select Home. Press 2ND [TAN],
enter 49 and then press 2ND [°] (degree symbol).
Enter 32 and then press 2ND [¢] (minutes symbol).
Close the set of brackets and then press ENTER to
display the answer.
Appendix 691
Chapter 4 page 183
Graphics Calculator tip! Displaying an answer in degrees,
minutes and seconds
The following steps show how to convert an angle in decimal degrees to degrees,
minutes and seconds. Consider Worked example 5 on page 182. If the tan of an angle is
equal to 1.647, we can find the value of the angle to the nearest minute.
1. From the MENU, select Home. Press ♦ [TAN-1]
and enter 1.647. Close the set of brackets and then
press ENTER . This displays the angle in degrees as a
decimal.

2. To convert to degrees, minutes and seconds, first


press 2ND [ANS] and then press CATALOG . Scroll
down to DMS. (You can do this more quickly by

first pressing ALPHA [D].)

3. Press ENTER to display DMS in the function entry


line of the Home screen. Press ENTER again to


display the answer. The answer required (to the
nearest minute) is 58°44′. (Note that when rounding
to the nearest minute, any value equal to or greater
than 30 seconds will result in the angle being
rounded up to the next minute.)
692 Appendix

Chapter 4 page 186


Graphics Calculator tip! Using the equation solver

When finding the length of a side, you will be left with an equation to solve to obtain
your final answer. The solution steps can be cut out by using the solve function.
x
Consider Worked example 7. At step 3 of the solution we are left with sin 50° = ------ ,
which can be solved as shown below. 24

1. From the MENU, select Home. Press CATALOG and


scroll down to access solve(. You can do this more
quickly by first pressing ALPHA [S].

2. Press ENTER to display solve( in the function entry


line. (Alternatively, press F2 (Algebra) and select
x
1: solve(.) Enter the expression sin 50° = ------ . Notice
24
that ) was used to close the set of brackets around 50.

3. We then need to tell the calculator to solve the


equation for x. First press , to enter the comma
symbol and then press X to indicate the variable x.
Close the set of brackets and then press ENTER to
solve the equation.
Appendix 693
Chapter 4 page 192
Graphics Calculator tip! Using the equation solver to find an
angle
The solve function can be used to find an angle in the same way that it can be used to
find the length of a side. Consider Worked example 10.
1. From the MENU, select Home. Press CATALOG
and select solve(. (Alternatively, press F2 (Algebra)
and select 1: solve(.) Enter the equation
tan x = 4.3 ÷ 6.5. (Remember to include brackets
where appropriate.)

2. The equation must be solved with respect to x so,


after the equation, press , to insert a comma
followed by X for the variable. Press ) to close
the set of brackets.

3. On this calculator we must specify the domain over


which we are to solve the equation; in this case,
between 0° and 90°. To do this, first press to
enter the symbol and then enter 0 ≤ x ≤ 90. (Either
access the ≤ symbol from the CATALOG or enter it in
the function entry line by pressing 2ND ♦ [<].)
Finally, press ENTER to solve the equation.
694 Appendix

Chapter 4 page 194


Graphics Calculator tip! Finding a solution in degrees and
minutes
The solve function will display angles in degrees as a decimal. The DMS function


must be used to convert the answer into degrees, minutes and seconds. As shown
earlier, it is best to use the inverse sin function rather than trying to use the solve
function.
Consider Worked example 11. To obtain the solution in degrees and minutes, use the
following steps.
1. Ensure your calculator is in degrees mode. Enter the
expression and display the answer.

2. Press 2ND [ANS] for the previous answer and then


press CATALOG and scroll down to DMS.

3. Press ENTER to display DMS in the function entry


line and then press ENTER again to display the


answer in degrees, minutes and seconds. The
required answer (to the nearest minute) is 40°23′.
Appendix 695
Chapter 4 page 216
Graphics Calculator tip! Using equation solver to solve
cosine rule problems (sides)
Using the solve function for the cosine rule is very useful as many students forget the
final step of the solution, which is to take the square root of a2, b2 or c2.
In the same way as with earlier questions, we write the formula and then substitute
the appropriate values, leaving one unknown. Hence we have an equation, which can be
typed into the equation solver of a graphics calculator. Consider Worked example 23 on
page 215.

Enter solve( into the function entry line of the Home


screen. (Either press CATALOG and scroll down to
solve( or press F2 (Algebra) and then select
1: solve(.) Enter b2 = 62 + 102 – 2 × 6 × 10 × cos 76o.
Remember to solve the equation for b at the end of the
expression. Press ENTER to display the solution.
Notice that because the formula uses b2 there can be
two solutions; hence the calculator shows both solutions
–10.3426 and 10.3426. In this situation we should
ignore the negative solution.
696 Appendix

Chapter 5 page 256


Graphics Calculator tip! The graph of y = sin x

The following steps can be used to draw the graph of y = sin x and explore the key
points on the graph.
1. Before beginning you will need to check that the
calculator is in radian mode. To do this, press MODE
and then use the arrow keys to scroll down to the
Angle line. Press the right arrow to display the
available options. Highlight 1: RADIAN.

2. Press ENTER to accept the radian setting and then


ENTER again to save the settings. Press APPS and
highlight Graph.

3. Press ENTER to select Graph. Press ♦ [WINDOW]


and adjust the settings so that xmin is 0, xmax is 10,
ymin is −1.2 and ymax is 1.2.

4. Press ♦ [Y=], enter sin x for y1(x) and then press


ENTER .

5. Press ♦ [GRAPH] to display the graph of the


function.

6. The Trace tool can be used to identify key points on


the graph. Press F3 (Trace) and then use the arrow
keys to move the point (or trace) along the curve.
Appendix 697
7. To find the amplitude, press F5 (Math) and then
select 4: Maximum.

8. Use the arrow keys to move the cursor to a point


close to the maximum and press ENTER . Then
move the cursor to a point on the other side of the
maximum and press ENTER again. The coordinates
of the maximum point will be displayed. The y-value
gives the amplitude, which we can see is 1.

9. The period can be explored by looking at the


x-intercepts. Press F5 (Math) and then select
2: Zero.

10. Press ENTER on a point either side of the intercept


that completes the first cycle. The x-value of 6.28319
(or, more accurately, 2π) gives the period.
698 Appendix

Chapter 5 page 259


The effect of 2
1 On the same set of axes, sketch the graphs of y = sin x and y = 2 sin x.
1. From the MENU, select Graph. Press
♦ [WINDOW] and adjust the settings so that
xmin is −6.28 (or −2π), xmax is 6.28 (or 2π),
ymin is −2 and ymax is 2.

2. Press ♦ [Y=] and enter sin x for y1(x) and


2 sin x for y2(x).

3. Press ♦ [GRAPH] to display the graphs.

2 What has been the effect of the 2 on the graph of y = 2 sin x?


3 Now, on the same axes, draw the graphs of y = sin x and y = sin 2x.
4 What has been the effect of the 2 on the graph of y = sin 2x?
Appendix 699
Chapter 6 page 280
Graphics Calculator tip! Solving a simple trigonometric
equation
As we saw in chapter 4, you can solve a trigonometric equation using the solve
function. Consider part a from Worked example 1 on page 279.
1. From the MENU, select Home. Check that your
calculator is in degrees and approximation mode.
Press CATALOG and then scroll down to access
solve(. You can do this more quickly by first pressing
ALPHA [S].

2. Press ENTER to display solve( in the function entry


line of the Home screen. (Alternatively, press
F2 (Algebra) and then select 1: solve(.) Enter the
equation sin x = 0.412, remembering to include
brackets where appropriate. Include the variable and
the domain over which we are to solve the equation.
(The ≤ symbol can be entered by pressing
2ND ♦ [<].) Finally, press ENTER to solve the
equation.
700 Appendix

Chapter 7 page 304


Graphics Calculator tip! Simplifying expressions

Some graphics calculators have the facility to simplify expressions. Consider simplifying
4n n+1
3 × 18
the expression ----------------------------
3n – 2
- shown in Worked example 4 on page 303.
6
1. From the MENU, select Home. Ensure that the
calculator is set to AUTO mode. (Press MODE and
then F2 (Page 2) for the second page of options.
Scroll down to the Exact/Approx line and press the
right arrow to display the options. Select 1: AUTO
and then press ENTER to save the settings.)

2. Enter the expression, being sure to use brackets


around the entire top line of the expression and each
of the index expressions. Press ENTER to display
the simplified expression.

3. To check that this is the same as the answer


given in Worked example 4, enter the expression
33n + 4 × 23 − 2n and then press ENTER .
Appendix 701
Chapter 7 page 313
Method 3: Solving indicial equations using a
graphics calculator
Consider the equation 2x = 5 shown in Worked example 11. We can solve this equation
using the solve( function.
1. Enter solve( in the function entry line of the Home
screen. (Either press CATALOG and select solve( or
press F2 (Algebra) and select 1: solve(.) Enter the
equation to be solved and then the variable, separated
by a comma. Close the set of brackets and then press
ENTER . This displays the solution as an exact value.
The function ln(x) is a logarithmic function that uses
a base other than 10.
2. To display the approximate value of the solution,
switch the calculator into approximation mode. Press
MODE and then F2 (Page 2) for the second page
of options. Scroll down to the Exact/Approx line and
press the right arrow to display the options. Highlight
3: APPROXIMATE.

3. Press ENTER to accept the setting and then again to


save the settings and return to the Home screen.
Press ENTER to display the approximate solution.
702 Appendix

Chapter 7 page 318


WORKED Example 13
Use a graphics calculator to solve 2x = 15 using the intersection of two graphs. Give the
answer rounded to 2 decimal places.
THINK WRITE/DISPLAY
x
1 To solve 2 = 15, we can draw the graphs of Let f1(x) = 2x and f2(x) = 15.
two functions f1(x) and f2(x) and then find
where they intersect. Write the equations of the
two functions to be graphed.
2 Use a graphics calculator to draw the two
functions on the one set of axes and then locate
the coordinates of the intersection point.
1. From the MENU, select Graph. Press
♦ [Y=], enter 2x in the function entry line
for y1(x) and then press ENTER . Similarly,
enter 15 in the function entry line for y2(x)
and then press ENTER .

2. Press ♦ [WINDOW] and adjust the settings


so that xmin is −1, xmax is 10, ymin is −1
and ymax is 20 as shown.

3. Press ♦ [GRAPH] to display the two


graphs.

4. To display the coordinates of the


intersection point, press F5 (Math) and
then select 5: Intersection. Navigate the
cursor to each line and press ENTER , then
press ENTER at a point either side of the
intersection. The coordinates of the point of
intersection will be displayed.
3 Write the solution to 2 decimal points. Solution is x = 3.91.
Appendix 703
Chapter 7 page 321
A world population model
The statistics below describe P, the estimated world population (in billions) at
various times, t.

t 0 1000 1250 1500 1750 1800 1850 1900 1910


P 0.30 0.31 0.40 0.50 0.79 0.98 1.26 1.65 1.75

t 1920 1930 1940 1950 1960 1970 1980 1990 2000


P 1.86 2.07 2.30 2.52 3.02 3.70 4.45 5.30 6.23

1 Use a graphics calculator to plot the data and fit


an exponential curve.
1. From the MENU, select Stats/List Editor.
Enter the data for the year in List 1 and enter
the population in List 2. Press ENTER after
each entry. Label List 1 as yr and List 2 as pop.

2. Press F2 (Plots) and select 1: Plot Setup.


Press F1 (Define) and select a scatterplot for
Plot Type. Complete the settings as shown
with yr entered for x and pop entered for y.

3. Press ENTER to accept the settings and then


press F5 (ZoomData) to view the graph.

4. Press APPS and return to the Stats/List


Editor. Press F4 (Calc) and select
3: Regressions.
704 Appendix

5. For an exponential model of the data, select


8: ExpReg. This brings up a screen to define
the regression. For X List, enter yr and press
ENTER . Similarly, enter pop for Y List and
press ENTER . (Note that you can save the
regression equation as y1(x). This means that
you can easily graph the equation if you
wish.)
6. Press ENTER to display the exponential
regression statistics. The r2 value indicates
that the model does not fit the data very well.
(The closer the value of r2 is to 1, the better
the fit.)

7. To draw the regression function with the


original data, press ♦ [GRAPH].

8. Now consider modelling the data from 1750


onwards. Repeat the steps above to plot the
new set of data. You should see that the r2
value of 0.89 indicates a much better fit.

2 Write the equation of the exponential curve which best models the data.
3 Use the equation for the curve to predict the world population in 2050.
4 What limitations are there on the use of the equation to predict future
populations?

Note: Refer to pages 322 and 323 for information on the Excel spreadsheet
and the associated question 5.

Chapter 7 page 332


Logarithmic graphs
To ensure equal axis scales with a TI-89 Titanium graphics calculator, press
F2 (Zoom) and then select 5: ZoomSqr.
Appendix 705
Chapter 8 page 351
Graphics Calculator tip! Listing the terms of a geometric
sequence
If you know the rule for a geometric sequence, successive terms can be listed using a
graphics calculator. The steps for generating these terms are shown below. Consider
listing the first 5 terms of a sequence where the first term is 3 and the ratio is 2, so the
rule is tn = 3 × 2n − 1.
1. From the MENU, highlight Data/Matrix Editor.

2. Press ENTER to select this application and then


highlight 3: New.

3. Press ENTER and define the data using variable n.

4. Press ENTER to accept the settings and then use the


arrow keys to highlight the cell containing c1, the
heading for the first column.

5. Press CATALOG and then scroll down to seq(.

6. Press ENTER to display seq( in the function entry


line and then complete the entry line with the rule
(3 × 2n − 1), the variable (n), the value of n for the
starting term (1) and the value of n for the last term
(5). That is, enter 3 × 2^(n − 1), n, 1, 5. Close the set
of brackets and then press ENTER to display the
terms of the sequence.
706 Appendix

Chapter 8 page 356


Graphics Calculator tip! Finding the sum of a geometric
sequence
Consider the sequence tn = 3 × 2n − 1 used in the previous Graphics calculator tip on
page 705. Let’s find the sum of the first 5 terms of this sequence. There are a number of
ways to find the sum using a graphics calculator. Two methods are shown below.

Method 1: Using the SUM function


1. As before, produce the first 5 terms of the sequence.

2. Use the arrow keys to move to an empty cell, say


cell r1c2.

3. To calculate the sum of the first 5 terms, press


CATALOG and then select sum(. Press CATALOG
again and select seq(. Enter the information for the
sequence as shown previously and close both sets of
brackets. The function entry line should show
sum(seq(3*2^(n-1), n, 1, 5)). Press ENTER to
display the result.
Note: Step 1 above is optional as the sum could have been calculated without
displaying the sequence of terms.

Method 2: Finding the sum from the Home screen


From the MENU, select Home. Press CATALOG and
then select sum(. Press CATALOG again and select
seq(. Enter the information for the sequence as shown
previously and close both sets of brackets. The function
entry line should show sum(seq(3*2^(n-1), n, 1, 5)).
Press ENTER to display the result.
Appendix 707
Chapter 8 page 363
WORKED Example 6
Write an equation which describes the relationship between the number, P, of bees and
time, T, if the population of 500 bees is known to be increasing by 20 each month.
Find:
a the size of the population in a year’s time
b when the population will have doubled.
THINK WRITE/DISPLAY
Let population be P and time in months be T. Write Let P = population and T = time in
an equation to describe the relationship. months.
P = 500 + 20T

Using a graphics calculator


1. From the MENU, select Graph. Press
♦ [Y=], enter 500 + 20x for y1(x) and
then press ENTER . The equation is
entered in terms of y1 (in place of P) and x
(in place of T).

2. Press ♦ [WINDOW] and adjust the settings so


that xmin is 0, xmax is 30, xscl is 5, ymin is
400, ymax is 1200 and yscl is 100 as shown.

3. Press ♦ [GRAPH] to display the graph with


these settings. Press F3 (Trace) and use the
arrow keys to trace along the line. The
coordinates of a given point on the line will be
displayed.

a
a 1 To find the coordinates of the point
where x = 12, press F5 (Math) and
select 1:Value. Enter 12 and then press
ENTER . Read the value for the
y-coordinate of the point. This shows
that P = 740 bees.

2 Write your answer. The population in one year’s time will be


740 bees.
708 Appendix

THINK WRITE/DISPLAY
b
b 1 To find T when P = 1000, we can find
the point of intersection of two graphs
representing P = 500 + 20T and
P = 1000.
Press ♦ [Y=], enter 1000 for y2(x)
and then press ENTER . Press
♦ [GRAPH] to display both graphs on
the same axes. Press F5 (Math) and
select 5: Intersection. Navigate the
cursor to each line and press ENTER ,
then press ENTER at a point either side
of the intersection. The coordinates of
the point of intersection will be
displayed. This shows that T = 25
months.

2 Write your answer. The population will double after 2 years


and 1 month.
Appendix 709
Chapter 8 page 367
WORKED Example 8
The cost, C ($), of a deluxe puff pastry after time, T (years), is given by the equation
C = 0.8(1.6)T. Use the equation to complete the table below and plot a graph of cost against
time.

Time, T 0 1 2 3 4 5
Cost, C

THINK WRITE/DISPLAY
Using a graphics calculator
1 From the MENU, select Table. Press ♦ [Y=]
and enter 0.8(1.6)x for y1(x). Press ENTER .

2 Press ♦ [TABLE] to display the table values.

3 Before we draw the graph, press


♦ [WINDOW] and adjust the settings so that
xmin is −1, xmax is 10, ymin is −1 and ymax
is 10 as shown.

4 To display these points on a graph, press


♦ [GRAPH].
710 Appendix

Chapter 8 pages 405, 408


Graphics Calculator tip! Financial calculations using TVM,
Worked example 23
There is no TVM function on the TI-89 Titanium graphics calculator.

Chapter 9 page 432


Graphics Calculator tip! Choosing a random sample

Suppose that we are to choose a random sample of 20 students from the population of
800. To choose a random sample, each student would be allocated a number between
1 and 800 and the graphics calculator could then be used to make a random choice
using a random number function.
To choose a random sample using the TI-89 Titanium graphics calculator, we need
to use the random integer function.
1. From the MENU, select Home. Press CATALOG and
then scroll down to rand(. (This can be found more
quickly by first pressing ALPHA [R].)

2. Press ENTER to display rand( in the function entry


line of the Home screen. Enter the upper limit of 800
and press ) to close the set of brackets. Press
ENTER to obtain a random number. You will
probably obtain a different number to that seen in the
display shown.

3. Press ENTER again to generate a new random


integer. Repeat until enough random integers have
been generated.
Appendix 711
Chapter 9 page 452
Graphics Calculator tip! Constructing a histogram

To construct a histogram of the data shown in Worked example 12, follow these steps.
1. From the MENU, highlight Data/Matrix Editor.

2. Press ENTER to select this application and then


select 3: New. Enter the settings shown at right.

3. Enter the data for Mark in column 1 (c1).

4. Enter the data for Frequency in column 2 (c2).

5. Press F2 (Plot Setup) followed by F1 (Define).


For Plot Type, select 4: Histogram.

6. Enter the remaining settings as shown.


712 Appendix

7. Press ♦ [WINDOW] and adjust the settings so that


xmin is 12, xmax is 20, ymin is 0 and ymax is 10 as
shown.

8. Press ♦ [GRAPH] to display the histogram. You


may need to use the arrow keys to scroll right to see
the end of the graph. If you wish, you can press
F3 (Trace) to explore key points.

Chapter 10 page 482


Graphics Calculator tip! Calculating the mean of data in a list

Consider the data set in Worked example 1 which is presented as a single list of figures.
1. From the MENU, select Data/Matrix Editor. As
shown previously, select 3: New. Enter the data into
the first column (c1).

2. Press F5 (Calc) and adjust the settings so that


Calculation Type is OneVar and x (scores) is c1 as
shown.

3. Press ENTER to save the settings. The summary


statistics on this data set will be displayed. The first
piece of information shown is the mean. Read the
value shown for x .
Appendix 713
Chapter 10 page 485
Graphics Calculator tip! Calculating the mean of data in a
frequency table
Consider the data set in Worked example 2 on page 484 which is presented in a
frequency table.
1. Open a new Data/Matrix Editor screen. Enter the
midpoint of the class intervals in column 1 (c1) and
the frequencies in column 2 (c2).

2. Press F5 (Calc) and adjust the settings so that


Calculation Type is OneVar, x (scores) is c1,
Use Freq and Categories? is YES and Freq is c2
as shown.

3. Press ENTER to save the settings. The summary


statistics on this data set will be displayed. Read the
value shown for x .

Chapter 10 page 488


Graphics Calculator tip! Calculating the median and the
mode
When you enter data into your graphics calculator as shown earlier, a list of summary
statistics is given and can be accessed using the scroll function. The TI-89 Titanium
will display the median (shown as medStat) but not the mode or modes.
714 Appendix

Chapter 10 page 496


Graphics Calculator tip! Finding summary statistics

Earlier in the chapter we looked at how to enter a set of scores into the graphics calcu-
lator to find the mean. The summary statistics screen shows all the information needed
to find the median, range and interquartile range. Consider the data set in Worked
example 6.
1. Open a new Data/Matrix Editor screen. Enter the
data into the first column (c1).

2. Press F5 (Calc) and adjust the settings so that


Calculation Type is OneVar and x (scores) is c1 as
shown.

3. Press ENTER to save the settings.


The summary statistics on this data
set are shown. The first piece of
information shown is the mean. We
need to scroll down to find the
information needed for the median,
range and interquartile range.
Appendix 715
Chapter 10 page 503
WORKED Example 9
The following frequency distribution gives the prices paid by a car wrecking yard for
40 car wrecks.
Price ($) Frequency Price $ Frequency
0–500 2 2000–2500 7
500–1000 4 2500–3000 6
1000–1500 8 3000–3500 3
1500–2000 10
a Find the mean and standard deviation in the price paid for these wrecks.
b Estimate the mean and standard deviation in the price paid for wrecks by this yard
in general.

THINK WRITE/DISPLAY
Use a graphics calculator to find the mean and
standard deviation.

1 Open a new Data/Matrix Editor screen. Enter


the midpoint of the class intervals in column 1
(c1) and the frequencies in column 2 (c2).

2 Press F5 (Calc) and adjust the settings so that


Calculation Type is OneVar, x (scores) is c1,
Use Freq and Categories? is YES and Freq is
c2 as shown.

3 Press ENTER to save these settings. The


summary statistics on this data set are
displayed. The mean ( x ) is the first item
shown, the sample standard deviation (Sx) is
the fourth item shown with the population
standard deviation (σx) shown below that.
a List the mean and population standard a Mean = $1825
deviation of the data given. Standard deviation = $787
b To estimate these figures in general we b In general:
give the mean and sample standard Mean = $1825
deviation. Standard deviation = $797
716 Appendix

Chapter 10 page 511


Graphics Calculator tip! Creating a boxplot from a frequency
table
A boxplot can be drawn on the graphics calculator. Consider the data used in Worked
example 11 on page 509.

1. Open a new Data/Matrix Editor screen. Enter the


data into the first column (c1).

2. Press F2 (Plot Setup) followed by F1 (Define).


For Plot Type, select 3: Box Plot.

3. Enter the remaining settings as shown.

4. Press ♦ [WINDOW] and adjust the settings so that


xmin is 80, xmax is 120, xscl is 5, ymin is 0, ymax
is 10 and yscl is 1 as shown.

5. Press ♦ [GRAPH] to display the boxplot. Press


F3 (Trace) to explore key points.
Appendix 717
Chapter 10 page 520
WORKED Example 14
The four Year 7 classes at Western Secondary College complete the same end-of-year
maths test. The marks, expressed as percentages for each of the students in the four
classes, are given below.

7A 7B 7C 7D 7A 7B 7C 7D
40 60 50 40 69 78 70 69
43 62 51 42 63 82 72 73
45 63 53 43 63 85 73 74
47 64 55 45 68 87 74 75
50 70 57 50 70 89 76 80
52 73 60 53 75 90 80 81
53 74 63 55 80 92 82 82
54 76 65 59 85 95 82 83
57 77 67 60 89 97 85 84
60 77 69 61 90 97 89 90
Display the data using a parallel boxplot and use this to describe any similarities or
differences in the distributions of the marks between the four classes.
THINK WRITE/DISPLAY
1 Create the first boxplot (for class 7A) on a
graphics calculator.
1. Open a new Data/Matrix Editor screen.
Enter the data for Class 7A in the first
column and then press F2 (Plot Setup)
followed by F1 (Define). For Plot Type,
select 3: Box Plot. Press ♦ [WINDOW]
and adjust the settings so that xmin is 30,
xmax is 100, xscl is 10, ymin is 0, ymax is
10 and yscl is 1. Press ♦ [GRAPH] to
display the boxplot.
2. Press F3 (Trace) and use the arrow keys to
display the five-number summary.

Refer to page 521 for steps 2, 3 and 4.


718 Appendix

Chapter 11 page 552


Graphics Calculator tip! Random number generation

There are a number of ways of generating a random number using a graphics calcu-
lator. One method has already been shown on page 710. To generate a particular
number of random numbers and store them in a list, follow the steps shown below. We
will consider generating 100 random numbers ranging from 1 to 8 as an example.
1. Open a new Data/Matrix Editor screen. Highlight the
cell containing c1 (the heading for column 1). Press
CATALOG and then scroll down to seq(.

2. Press ENTER to display seq( in the function entry


line. Press CATALOG again and select rand(. Enter
8 to define the maximum random number. Close the
set of brackets and then enter a comma (press , ).
Complete the entry line with the variable (n), the
value of n for the starting term (1) and the value of n
for the last term (100). That is, enter n, 1, 100 as
shown. Press ) to close the set of brackets.
3. Press ENTER to display the set of random numbers.
Appendix 719
Chapter 11 page 554
WORKED Example 14
Use a sequence of 20 random numbers to simulate rolling a die 20 times. Record your
results in a frequency table.

THINK WRITE/DISPLAY
1 Generate 20 random numbers in the range
1 to 6.
1. Open a new Data/Matrix Editor screen and
generate 20 random integers between 1 and
6 as shown in the previous Graphics
calculator tip.

2. Scroll down the list to count the frequency


of each number.
Note that the Data/Matrix Editor
application locks the cells and this prevents
us from sorting the numbers in ascending
order.

2 Enter this information in a table as shown. Die value 1 2 3 4 5 6


(Your table will show different values.)
Frequency 2 4 6 2 5 1
720 Appendix

Chapter 11 page 555


WORKED Example 15
Generate 50 random numbers from 5 to 10 and sketch a histogram of the results.

THINK DISPLAY
Use a graphics calculator to generate 50 random
numbers in the range 5 to 10. Then use the list of
numbers to produce a histogram of the results.
1. Open a Data/Matrix Editor screen and
highlight the cell containing c1. Because we
want random numbers between 5 and 10, we
generate 50 random numbers between 1 and 6
as shown earlier, but add 4 to the result. Enter
seq(rand(6), m, 1, 50) + 4 in the function entry
line.
Note the use of the variable m. Any variable,
other than the one used when initially defining
the matrix, can be used.
2. Press ENTER to generate the numbers in the
first column. Note that your results will differ
from those shown here.

3. Press F2 (Plot Setup) followed by


F1 (Define). For Plot Type, select
4: Histogram. Enter the settings as shown.

4. Press ENTER to save the settings. Then press


♦ [WINDOW] and adjust the settings so that
xmin is 5, xmax is 10, ymin is 0 and ymax is
15 as shown.

5. Press ♦ [GRAPH] to display the histogram.


(Your histogram will look different to that
shown here.) If you wish, you can press
F3 (Trace) to explore key points.
Appendix 721
Chapter 12 page 585
WORKED Example
2
8
By drawing the graph of y = x − 2, find the instantaneous rate of change where x = 1.

THINK WRITE/DISPLAY
1 Use a graphics calculator to draw the graph of
y = x2 − 2 and display a tangent to the curve at
the point where x = 1. Find the gradient of the
tangent.

1. From the MENU, select Graph. Press


♦ [WINDOW] and adjust the settings so
that xmin is −5, xmax is 5, ymin is −10 and
ymax is 20 as shown.

2. Press ♦ [Y=] and enter x2 − 2 for y1(x).


Press ♦ [GRAPH] to display the graph.

3. To find the gradient of the tangent at the


point where x = 1, press F5 (Math) and
select 6: Derivatives followed by 1: dy/dx.
Press 1 to enter 1 for xc and then press
ENTER to display the gradient at that
point.
4. To draw the tangent to the curve at x = 1,
press F5 (Math) and select A: Tangent
(you will need to scroll to see this).

5. Press ENTER to select the Tangent option


and then press 1 to enter x = 1. Press
ENTER and the tangent will be drawn and
its equation displayed. The equation of the
tangent is y = 2x – 3.
722 Appendix

THINK WRITE/DISPLAY
2 State the gradient of the tangent either from the The gradient of the tangent is 2.
derivative or slope value found, or from the
equation of the tangent.

3 Use the gradient to state the rate of change. Hence, the instantaneous rate of change
at x = 1 is 2.
Appendix 723
Chapter 12 page 599
Graphics Calculator tip! Relating the gradient function to the
original function
Consider the function y = x3 – 9x. We can use a graphics calculator to draw this
function with its gradient function on the one set of axes.
1. From the MENU, select Graph. Press ♦ [WINDOW]
and adjust the settings so that xmin is −5, xmax is 5,
ymin is −10 and ymax is 20 as shown.

2. Press ♦ [Y=] and enter x3 – 9x for y1(x). Press


♦ [GRAPH] to display the graph.

3. Press ♦ [Y=] to enter a function for y2(x). Press


CATALOG and then scroll down to d( differentiate.

4. Press ENTER to display d( in the function entry line


for y2(x). (Alternatively, press F3 (Calc) and then
select 1: d( differentiate.) Enter y1(x) as the function
to differentiate (or find the gradient of) and then
enter a comma , followed by the variable x. Press
) to close the set of brackets.

5. Press ENTER and then press ♦ [GRAPH] to


display the graph of the original function with its
gradient function.
724 Appendix

Chapter 13 page 631


Graphics Calculator tip! Finding the limit of a rational
function
Some graphics calculators can find the limit of a rational function. The instructions
2
x –x
below show how to find lim -------------- from Worked example 4.
x→1 x – 1

1. From the MENU, select Home. Press CATALOG and


then scroll down to limit(.

2. Press ENTER to display limit( in the function entry


line. (Alternatively, press F3 (Calc) and then select
3: Limit(). Complete the entry line with the function
2
x –x
-------------- followed by the variable x and then the value
x–1
of x that the function is approaching, each separated
by a comma as shown. Press ) to close the set of
brackets.

3. Press ENTER to display the required limit. The limit


is 1.
Appendix 725
Chapter 13 page 638
Graphics Calculator tip! Finding the derivative using first
principles
The limit can also be used to find the derivative of a function using the first principles
method. Consider the function f(x) = x2 – 2x from Worked example 7.
1. From the MENU, select Home. To define the
function, first press CATALOG and then select
Define (or press F4 (Other) and select 1: Define).
Enter the required function as y1(x) = x2 – 2x and
then press ENTER .

2. Now enter limit( in the function entry line.


(Press CATALOG and select limit( or press
F3 (Calc) and select 3: Limit(). Enter the expression
(y1(x + h) – y1(x)) ÷ h) followed by the variable h
and then the value of h that the function is
approaching, each separated by a comma as shown.
Press ) to close the set of brackets.
3. Press ENTER to display the required limit. The
derivative is 2(x − 1) or 2x − 2.
726 Appendix

Chapter 13 page 643


Graphics Calculator tip! Finding the derivative

Some graphics calculators can display the derivative of a function directly. The instruc-
tions below show how to find the derivative of f(x) = 2x3 – 4x2 + 6 and also the value of
the derivative at x = 5.
1. From the MENU, select Home. Press CATALOG and
scroll down to d( differentiate.

2. Press ENTER to display d( in the function entry line.


(Alternatively, press F3 (Calc) and select
1: d( differentiate.) Enter the expression 2x3 − 4x2 + 6,
followed by a comma ( , ) and the variable x. Close
the brackets and then press ENTER .

3. Alternatively, we can define the function and then


find the derivative. To define the function, first
press CATALOG and then select Define (or press
F4 (Other) and select 1: Define). Enter the required
function as y1(x) = 2x3 − 4x2 + 6 and then press
ENTER .
To find the derivative, enter d( (press CATALOG
and scroll down to d( differentiate) in the function
entry line, then enter y1(x) followed by the variable
x, each separated by a comma as shown. Close the
set of brackets and press ENTER to obtain the
expression for the derivative.

4. To find the value of the derivative at x = 5, complete


the function entry line as shown. (Repeat step 3
without pressing ENTER , then press Ù and follow
with X = 5 for x = 5.) Now press ENTER . The
gradient of the function at x = 5 is 110.
Appendix 727
Chapter 13 page 647
Graphics Calculator tip! Graphing the derivative function

We can graph the function f(x) = (x − 1)(x − 2)(x + 1) and its derivative on the same set
of axes.
1. From the MENU, select Graph. Press ♦ [Y=] and
enter (x − 1)(x − 2)(x + 1) for y1(x). Press ENTER .

2. Press ♦ [WINDOW] and adjust the settings so that


xmin is −3, xmax is 4, ymin is −5 and ymax is 12 as
shown.

3. Press ♦ [Y=] again to enter a function for y2(x).


First enter d( in the function entry line (press
CATALOG and select d( differentiate or press
F3 (Calc) and select 1: d( differentiate). Then
enter y1(x) followed by the variable x, each separated
by a comma. Close the set of brackets and press
ENTER .

4. Press ♦ [GRAPH] to display both graphs on the


same set of axes.
729

answers
Answers

Answers
CHAPTER 1 Modelling using mg – F v–u
c v = ----------------- , 59.161 d a = ----------- , 4.167
linear functions k t
Exercise 1A — Solving linear equations S S – πr2
e h = ------ – r or h = ----------------- , 17.108
1 a 2 b −8 c −3 d 7 πr πr
e −8 f 6 g 2 h −11 T 2
i − 7--3- j 13 f l = g  ------ , 3.976 g d = l 2 – 4 fl , 10.247
------
6
 2 π
2 a 7 b −6 c 5 d −11
H–U ( 1 – α )K
e −1 f 18 g − 31
------ h − 7--2- h V = --------------- , 10.75 i c = ---------------------- , 2622
3 P α2
3 a 9 b −4 c −7 d 4
e −1 f −5 g 4 h 3 H 0v
j u = ---------
- , 4.706
4 a 10 b 6 c −9 d −12 Hi
e 14 f −18 P P – 2l
5 a 6 b −2 c 5 d −5 4 a 42 cm b w = --- – l or w = -------------- c 40 mm
2 2
e −13 f 4 g 11 h −3
F
6 a 12 b −5 c 7 d 7 5 a 10 N b a = ---- c 10.77 m/s2
76 2 m
e −9 f ------
59
6 a 240 m
7 a 3 b −4 c 5 d 8 2A 2 A – bh
b a = ------- – b or a = ------------------- c 18 cm
e −9 f −7 h h
7 5
8 a ---
3
b ---
2
c 9 7 a $1123.60
116 19
d 4 e f A A– D
b r = 100  ---- – 1 = 100  ---------------------- c 41.4%
--------- ------
47 21
 D   D 
Exercise 1B — Rearrangement and 8 a 153° b 17.17 cm
substitution 9 a 60.25 cm b 6 cm
A d uv fv
1 a P=A−L b l = ---- c t = --- 10 a f = ----------- b u = ------------ c 150 cm
w v u+v v– f
C 100 A 3V
d r = ------ e R = ------------- f r = 3 ------- Exercise 1C — Gradient of a straight
2π PT πh
I 4 R4 + I 2 R2 – I 1 R2 – E
line
g I 3 = ------------------------------------------------------- 1 a 2 b 5 c 1
--- d − 1--3-
R3 4
16
R e −4 f −1 g ------
5
h − 20
------
9
-----1- – 1
R2 R1 – R2 2 a 2 b 5 c −4 d 1
---
h α = --------------
- or α = -----------------
- 2
θ R2 θ 1 1
e −6 f ---
3
g ---
8
h 2
E – αθ Et
i - j r = kQq
β = ---------------- ---------- k φ = ----- i 0 j − 1--2- k −2 l 5
---
θ2 F n 2

V 1N2 pV 2 ( s – ut ) 3 a 1
--- b −1 c 1
--- d 8
---
l V 2 = ------------- m n = ------- n a = --------------------- 2 4 5
N1 RT t2 e 1 f −12 g 0 h Undefined
2Fd + mu 2 µI 2 4 a 3--4- b 2 c 1--2- d 7--6-
o v = --------------------------- p r = ----------
m 2πF 1
e ---
4
f − 9--2- g −2 h 0
Vf V f1 – V f2
q U = ---------1- – V or U = -------------------------
- 5 a 1 b 4
--- c 1
--- d − 11
------
f2 f2 3 5 5

v 2 S – 2lh S – 2πr2 e −6 f − 3--2- g 12 h 5


---
r γ = ------ s w = ------------------- t H = --------------------- 8
rT 2(l + h) 2πr 6 a 1.192 b 3.078 c 0.176 d −0.577
2 a 56 b 30 c 80 d 16.97 e −0.577 f 0 g 1 h 57.290
e 33.33 f 0.267 g 350 h 7 7 a 0.93 b 2.61 c −0.53 d −3.73
i 13 100 j 2.498 8 a D b C c A d B
3V 9 a B b E c D d D
3 a l = A , 7.746 b r = 3 ------- , 6.204
4π 10 e, b, a, c, d 1A

1C
answers 730 Answers

11 a y b y 17 a y = 4x + 2 b y = −3x − 5 c y = 6--5- x − 2
d y = − 5--6- x + 5 e y = 2x − 1 f y = −5x
x
x 18 a y = 10.7x b 84.7°
a c a c
19 a a, b b − --- , --- c − --- , − --- d 2k, −3h
b b b b
c y

Exercise 1E — Sketching linear graphs


x
using intercepts
1 a y b y

18 7 x
2 17 2
12 ------
17
13 – ---------
300
14 ------
25
–21
15 a 4 b 31 c −5 d 3 x
–3
16 a No b Yes c 224 cm
c y d y

12 x
– 3–2
Exercise 1D — Equations of the
– 12

form y = mx + c 5
x
–3
1
e y f y

10

1
1 x
2 The higher the number, the steeper the graph. 2 x
Positive values make the graph slope up when g y h y
moving (or tracing) to the right; negative values
make the graph slope down when moving to the 8
right. 30
3 4
–16

10
— x
3

5 The number is where the graph cuts the y-axis 2 a y b y


(hence the name ‘y-intercept’). 4
6 a 5 b 6 c −9 d 2
e −8 f 1 g −1 h 5 2
i 3 j 0 k 0 l 0 3 x
7 a 7 b −4 c 1 d −13 5 x
e −5 f 2 g −10 h 0 c y d y
i 0 j 17 k 2 l 0
5– 6
8 a y = 2x + 7 b y = −3x + 1 c y = 5x − 2 4

d y=3 e y=x f y = 1--2- x − 5 –2 x


–3 x
2 1
g y= x+ ---
3
---
3
h y= − 3--4- x− 1
---
2
i y = −2x + 12
e y f y
9 a 3, 9 b 7, −42 c −4, 12 d −5, −35
e 3, 10 f −6, 24 g −16, −15 h −9, −1 5 x 1–
i 1, −23 2
10 a 4, 5 b 4, −8 c 4, −6 d −3, 1 –4 x
e −2, 4 f 3, 11 g −7, −9 h 2, 5 –7

i − 1--2- , −3 11
2
j − ----- 8 2
- , −6 k − --- , ---
3 3
l − 3--4- , 13
------
4 g y h y
1 5
m ---
6
, ---
2
n − 5--2- , − 7--2- 11

6
2 x
11 C 12 E –2
13 y = −7x + • 14 y=•x−6 11 x

15 3y + 5x = • 16 3y + • x = 17 2
731

answers
Answers

3 a y b y i j
10
–7 x

–6
2 a (1, 4) b (−2, 6) c (−4, −15) d (3, −15)
–4 x
e (−7, −5) f (3, 3) g ( 5--9- , 17
------ )
9
h ( 12 32
------ , ------ )
5 5
c y d y
i ( 23 20
------ , − ------ ) j (13, −3)
14 7
4 2 x
3 a (7, 9) b (−6, 5) c (6, 5) d (10, 1)
e (1, −2) f ( 10 4
------ , --- ) g ( 1--2- , 19
------ )
9 3
- , − --- )
h (− -----
– 16

3 –6 3 3 2 10 5
59 21 84 99
x i ( -----
8
-, ------
8
) j ( -----
67
-, ------
67
)
e y 4 15 cents and 35 cents
5 22 and 19
9 6 16 emus, 41 sheep
7 Basketballs $9.45, cricket balls $3.05
8 Limousine $225 (sedan $75)
9 A
10 D
–2 x

4 a y b y
Exercise 1G — Formula for finding the
(1, 1) equation of a straight line
x 1 a i 3x − y − 1 = 0 ii y = 3x − 1
(1, –1) x b i 2x − y − 4 = 0 ii y = 2x − 4
c i 5x − y − 19 = 0 ii y = 5x − 19
d i 4x − y + 11 = 0 ii y = 4x + 11
c y
e i x+y−1=0 ii y = −x + 1
x
f i 3x + y + 5 = 0 ii y = −3x − 5
g i x − 2y + 7 = 0 ii y = 1--2- x + 7--2-

(1, –2) h i 4x + 3y − 36 = 0 ii y = − 4--3- x + 12


5 D 6 E 7 A i i 4x − 5y + 19 = 0 ii y = 4--5- x + 19
------
5
j i x + 6y − 60 = 0 ii y = − --16- x + 10
Exercise 1F — Simultaneous equations k i 8x − 7y + 60 = 0 ii y = --87- x + 60
------
7
1a b
3
l i 3x + 11y − 33 = 0 ii y = − -----
11
-x + 3

2 a i x − 2y − 1 = 0 ii y = --2- x − 1--2-
1

b i x−y=0 ii y=x
c i x + 2y − 12 = 0 ii y = − --12- x + 6
c d
d i 3x + 2y − 2 = 0 ii y = − 3--2- x + 1
e i 3x + y + 7 = 0 ii y = −3x − 7
f i x+y−4=0 ii y = −x + 4
g i 14x − 3y + 2 = 0 ii y = 14 2
------ x + ---
3 3
e f
h i 3x − 4y − 12 = 0 ii y = 3--4- x − 3
i i 4x + 7y + 42 = 0 ii y = – 4--7- x − 6
j i x+y−1=0 ii y = −x + 1
3 A 4 C 5 y=x−6 6 y = 3x − 23 7 C
g h
8 a y = 2--5- x − 43
------
5
b y = − 1--2- x + 3
c y = 2x − 3 d y = − 3--4- x − 9
---
2
9 y = − 5--2- x − 3
---
2
1D

1G
answers 732 Answers

10 a y = 6--7- x + 82
------
7
b y = −x + 8 c y d y
x
c y = − --- + 39
------
4 7 x
8
11 94 12 C = 22n + 280 13 H = 22 + 6t
x
Exercise 1H — Linear modelling –9
1 a C = 5 + 10t
b Cost ($) c $35
25 (1, –6)
20
15 25 E
10 26 E
5 27 a b (−5, −5)
1 2 Time (h)
2 a C = 60 + 8m
b Cost ($) c $100
76
68
60
28 ( 5--3- , −5)
29 ( –--------
63 – 33
10
- , --------- )
5
30 21 two dollar and 46 one dollar coins
31 B
1 2 Time (min) 32 C
3 a P = 32 + 0.1n 33 y = −x + 4
b Payment ($) c $197 34 y = –-----76- x + 34 ------
7
34 35 D
33 36 a C = 75 + 65t
32
b C ($) c $302.50
205

140
10 20 Number
of leaflets 75
4 $960 5 Yes ($410 compared to $450)
6 a Opus $24, Belecom $20 b After 14 minutes 1 2 t (h)
7 a PinkCabs $28.50, NoTop $26
b After 6.7 km (6 2--3- km) Modelling and problem solving
8 After 4 rides 1 No, the points are not co-linear. This may be shown
9 6 visits by calculating gradients or equations for lines joining
10 Savus would be cheaper for up to 9 days hire. different pairs of points.
2 (−3, −4), (−1, 8), (3, 4)
Chapter review 3 a C = 250 + 55j b 13 jumps
1 D 2 A 3 −2 4 6 5 D 4 a 4 cm
b, c, e
Force applied to spring (N)

4π2 R3 R 60
6 D 7 A 8 T = --------------- or 2 π R ---------
GM GM 50
9 C 10 D 11 B 12 B 40
3 7 5
13 a ---
4
b − -----
11
- c -----
11
- d − 7--8- 30
20 Student 1
14 − --73- 15 4.331 16 Undefined 17 B Student 2
10
18 A 19 D
–5 1 0 5 10 15 20 25 30
20 a 3, −7 b ------
3
, 10 c ---
2
, −2
Length of spring (cm)
21 y = --25- x − 3 22 B 23 E d Answers will vary. One possible answer is
y = 30 95
------ x − ------ .
13 13
24 a y b y
f Answers will vary. One possible answer is y = 2x − 10.
24 g The first spring is stiffer than the second, as the
gradient of its graph is greater than that of the
5
second spring.
–40 x h The graphs intersect at the point (−8.75, −27.5). It
is not possible for the springs to have a negative
8 x length, so this point is not achievable.
733

answers
Answers

CHAPTER 2 Relations and b i Approx. 110 km/h ii Approx. 320 km/h


functions 10 a n 15 16 17 18 19 20 21 22 23 24 25
Exercise 2A — Relations and graphs
C($) 140 146 152 158 164 170 176 182 188 194 200
1 B 2 A 3 E 4 C
5 a D b C c C b C ($) c The variables are
d D e D f C 200 discrete.
b 190
6 a 70 y
180
9
170
Cost (¢)

160
(D) 4 150
(D) 140
1
60 0 x 130
M T W T F S S 1 2 3 0 5 10 15 20 25 n
Day
c d y
y y=x–2
Exercise 2B — Domain and range
x
–2 –1 0 1 2 0 x 1 a [−2, ∞) b (−∞, 5)
–1 2
c (−3, 4] d (−8, 9)
(D) (C)
–2 e (−∞, −1] f (1, ∞)
–3 –2 g (−5, −2] ∪ [3, ∞) h (−3, 1) ∪ (2, 4]
–4 2 a b
e y f y –6 0 2 –9 –3 0
7 6
6 c d
5
4 4 0 2 0 5
3
2 (C) e f
(D) 2
1 01 10 0 2 7
x
–2 –1 0 1 2 g h
x
–2 –2 –1 0 1 2
–2 0 1 3 –8 0 2 6
7 a T (°C) b T (°C)
3 a [−4, 2) b (−3, 1]
80 80
70 70
c ( – 1, 3 ) d  – 1---, ------
1
-

60 60 2 2
50 50
40 40
e (3, ∞) f (−∞, −3]
30 30 g (−∞, ∞) h (−∞, 0) ∪ (0, ∞)
20 20
10 10 4 C 5 B
0 2 4 6 8 t (minutes) 0 2 4 6 8 10 t (minutes) 6 a i {3, 4, 5, 6, 7} ii {8, 10, 12, 14, 16}
c Because the variables are continuous b i {1.1, 1.3, 1.5, 1.7} ii {1.4, 1.6, 1.8, 2}
d Approx. 11 minutes c i {3, 4, 5, 6} ii {110, 130, 150, 170}
d i {M, T, W, Th, F} ii {25, 30, 35}
8 a
n 0 1 2 3 4 5 6 e i {3, 4, 5} ii {13, 18, 23}
f i R ii [−1, ∞)
P($) 300 340 380 420 460 500 540
7 a R, R b R, (0, ∞) c [−2, 2], [0, 2]
b P ($) d [1, ∞), R e R, (0, 4] f R, (−∞, −3]
550 g R\{0}, R\{0} h R, (−∞, 1] i R, R
500
8 a y
450
400 2
350
300 – 2 0 2 x
250
200
0 1 2 3 4 5 6 n
Domain = (−∞, ∞), Range = (−∞, 2]
c The variables are discrete.
9 a V (km/h) b y
9 y = x3 + 1
350
300 x ∈ [–2, 2]
250 1
200 –2 0 x
2
150
–7
100
50 Domain = [−2, 2], Range = [−7, 9] 1H

0 1 2 3 4 5 t (s)
2B
answers 734 Answers

c y Domain = (−∞, ∞), 4 a, c, d, f, g, h


Range = [− 1--4- , ∞) 5 i a, b, c, d, f, h, i, j, k, l ii c, h, i, k
y = x2 + 3x + 2 a B b C
2 6
7 a y f (x)
b (−∞, 0) ∪ [1, ∞)
2
–2 –1 0 x 1
01 2 x
d y Domain = [−2, 1], –2 –1
Range = [−4, 0]
–2 –1 0 1 x
8 a y
y = x2 – 4, x ∈ [–2, 1] g(x)
–3 3
–4 2
1
e y y = 2x – 5, x ∈ [–1, 4] Domain = [−1, 4),
Range = [−7, 3) x
3 –2 –1 0 1 2

x
b [1, ∞)
–1 0 1 2 3 4 c i 3 ii 1 iii 2
–5 9 a y
–7 5
4
f y y= 2x2 –x –6
Domain = (−∞, ∞), 3
Range = [−6 1--8- , ∞) 2
1

–2 –1 0 1 2 x –3 –2 –1 0 1 2 3 x

–4
–6 –5

9 a R b [0, ∞) c [−4, 4] b (−∞, 0] ∪ (4, ∞)


c i −5 ii 0 iii −3 iv 0 v 7
d R e R\{0} f R
 x + 2, x ≤ 0
Exercise 2C — Types of relations 10 f ( x ) = 
(including functions)  2x + 1, x > 0
1 a One-to-many b Many-to-one 11 f : [0, 1] → R, f (x) = 1 – x 2 with range [0, 1] or
c Many-to-one d One-to-one
e One-to-one f Many-to-one f : [0, 1] → R, f (x) = – 1 – x 2 with range [−1, 0].
g Many-to-many h Many-to-one
i One-to-one j Many-to-one Exercise 2E — Inverse relations and
k Many-to-many l Many-to-one functions
2 b, c, d, e, f, h, i, j, k, l 1 a {(7, 1), (5, 2), (3, 3)}
3 C
y
4 a E b D c B y=x
8
5 b {−3, −1, 0, 1, 2}, {−2, −1, 1, 3} 7
c {3, 4, 5, 6}, {−1} e R, {2} 6
g R, R j [−1, ∞), [0, ∞) 5
k R, R 4
3
Exercise 2D — Function notation and 2
1
special types of functions
1 a i 1 ii 7 iii –5 iv 16 0 1 2 3 4 5 6 7 8 x
b i 2 ii 1 iii 3 iv 0 b {(3, 1), (6, 2), (9, 3)}
c i 3 ii 2 iii 6 iv 9 y
d i 9 ii 1 iii 16 iv a2 + 6a + 9
9 y=x
e i 12 ii 6 iii −4 iv 2 8
2 a 3 b −3 or 3 c 1
--- 7
3
6
d 2 or 3 e −4 or 1 f −1 5
4
3 a 3 b 3 c 5--- – 2x 3
x 2
10 10 10 1
d ------ – x 2 e ------------ – x – 3 f ----------- – x + 1
x2 x+3 x–1 0 1 2 3 4 5 6 7 8 9 x
735

answers
Answers

c {(11, −2), (6, 0), (1, 2)} 3 a y y = 4x y=x


y y=x
10
8
6
4
x
2

0 2 4 6 8 10 x
b y
y=x
y = x2 + 3
2 a y by

0 x
3
c y
x x

c y dy
y = 1_2 x + 1

1
0 x
1
d y
x y = x3 + 4
x

e 4
y
0 4 x

Exercise 2F — Circles
1 a x2 + y2 = 9 b x2 + y2 = 1 c x2 + y2 = 25
x d x2 + y2 = 100 e x2 + y2 = 6 f x2 + y2 = 8
g y = 9 – x 2 h y = – 16 – x 2
2 a Both [−3, 3] b Both [−1, 1]
c Both [−5, 5] d Both [−10, 10]
f y e Both [− 6 , 6 ] f Both [−2 2 , 2 2 ]
g [−3, 3], [0, 3] h [−4, 4], [−4, 0]
3 a y b y
2 4

x –2 0 2 x –4 0 4 x

–2 –4
2C

2F
answers 736 Answers

c y d y e y f y
7 3
7 5

x – 7 0 7 x –8 –3 0 2 x 2
–7 0 7 –2
–1 0 x
3 6
–7 – 7
e y f y –7
2 3 1– [−8, 2] and [−7, 3] [0, 6] and [−1, 5]
2
g y h y
10

–2 3 0 2 3 x – 1–2 0 1–
2
x
–1 0 1– 2
x
2
– 1–2 – 3–2
–2 3 4

4 a y b y
9 0 1 x
–3
–11 –5
–2

[−11, 1] and [−2, 10] [−1, 2] and [−3, 0]


−9 0 9 x
–2 0 2
x 8 y = 36 – x 2 ; Domain [−6, 6] and range [0, 6] or
−9 y = – 36 – x 2 ; Domain [−6, 6] and range [−6, 0]
Not a function Function 9 y = 2 + 9 – x 2 ; Domain [−3, 3] and range [2, 5]
c y d y or
y = 2 – 9 – x 2 ; Domain [−3, 3] and range [−1, 2]
1–
3 10 a 2 cm, 13.8 cm b 3.9 cm/s
–1 0 1 x

0 x
Exercise 2G — Functions and modelling
– 1–3 1–
3
 40, 0<t≤1

Function Function  70, 1<t≤2
e y f y 1 a C(t) = 
 110, 2<t≤4
5  160, 4<t≤6

– 1–2 0 1– x
2 b C ($)
0 x 160
– 1–2 – 5 5
110
Function Function
g h 70
y y
40
3
10
0 1 2 3 4 5 6 t (hours)
– 10 0 10 x 0 x
– 3  0.70, 0 < t ≤ 30
 1.10, 30 < t ≤ 60
– 10 – 3  1.50, 60 < t ≤ 90
Not a function Not a function 
 1.90, 90 < t ≤ 120
5 a D b B 2.30, 120 < t ≤ 150
2 a C(t) = 
6 a C b E  2.70, 150 < t ≤ 180
7 a y b y  3.10, 180 < t ≤ 210
4  3.50, 210 < t ≤ 240
x

–1 –1 0 1  3.90, 240 < t ≤ 270
2  4.30, 270 < t ≤ 300
–2
x
b
–2 0 2 4.30
–3 3.90
[−1, 1] and [−3, −1] [−2, 2] and [0, 4] 3.50
c d 3.10
y y
2.70
Cost ($)

3 3 2.30
1.90
1.50
–2 –1 0 2 6 x 1.10
0 1 4 7 x 0.70

–3 –5 0 30 60 90 120 150 180 210 240 270 300


[1, 7] and [−3, 3] [−2, 6] and [–5, 3] Time (s)
737

answers
Answers

 60t, 0 ≤ t ≤ 1.5 4 a y
 1
3 a d ( t ) =  90, 1.5 ≤ t ≤ 2
 –3–2 –1 01 2 3 x
 80t – 70, 2≤t≤4
–3
b Domain [0, 4]; range [0, 250]
c i 60 km ii 170 km
n y = 1 – x2
4 a B = ------ –8
12 b Domain = [−3, 3]; range = [−8, 1]
b B (hours) c $90
10
5 E 6 C 7 B 8 E 9 A
10 D
11 D
5
12 E
13 B
0 60 120
n 14 C
15 a, b, e
5 a T = 0.34x − 3978 16 B
b T ($) 17 C
8942
18 a x + 2, x ≥ 0
b Domain = [0, ∞); range = [2, ∞)
19 A 20 D 21 E 22 A
3060
1
23 a f : R \ {0} → R, f (x) = ---
x
20 700 38 000 x ($)
b f : (–∞, 2] → R, f (x) = 2 – x
Domain [20 700, 38 000]; range [3060, 8942]
c $6902 24 y
6 a P = 4x + 6 5
4
b Domain (1, 6]; range (6, 30] 3
7 a A = x2 + 4x 2
1
b Domain (0, 8]; range (0, 96] x
–2 –1 0 1 2 3 4 5
8 a P = 100 000(1.02)t
b $121 899 25 E
9 a 47 26 D
b 21 27 a y
c 9 weeks
96
d No, as t increases ----------- approaches zero, so N x
approaches 15. t +3 –1 0 1

10 a T = 6000 + 100n − 50n2 –1


b T
6000
5000 Domain = [−1, 1]; range = [−1, 0]
4000 b y
3000
2000 2
1000
–1 0 5 x
0 1 2 3 4 5 6 7 8 9 101112 n
(2, –1)
c $11 –4

Domain = [−1, 5]; range = [−4, 2]


Chapter review 28 E 29 C 30 B
1 B 31 a y
2 C x2 + y2 = 100
10
3 a
500
Graph is not 0 10 x
No. of cars (n)

–10
400 continuous
as n ∈ N –10
300
200
b f 1 : [−10, 10] → R, f (x) = ( 100 – x 2 ) with
100
dom f = [−10, 10], ran f = [0, 10] and
0 1 2 3 4 5 t (hours) f 2 : [−10, 10] R, f (x) = – ( 100 – x 2 ) with
b The number of cars is a discrete variable. dom f = [−10, 10], ran f = [−10, 0]
c 120 32 E

2G
answers 738 Answers

33
100
CHAPTER 3 Other graphs and
Cost ($)
75
modelling
50 Exercise 3A — Transforming graphs
25 1 a III b II c IV dI
0 1 2
2 a IV bI c II d III
Number of truck loads 3 a y b y
Modelling and problem solving y = 2f(x)

1 a A = xy + 10y − x2 b P = 2x + 2y + 20 or y = f(x)
y = f(x)
P = 2(x + y + 10)
0 x 0 x
c A = 260 + 16x − 2x2 d (0, 13)
e A (m2) f 292 m2 y = f(x−2)
292

260
130
c y d y
y = f(x) y = f(x)

0 2 4 6 8 10 12 14 x (m)

2 a x ∈ (–∞, –2) ∪ (−2, 4) b [0, 18] 0 x 0 x


c y=4 d y = –2x
2 y = −f(0.5x)
e y = 2x f y = –18x + 72 y = −f(x) + 2

 4, x ∈ ( – ∞, – 2 ) 4 a y y = 3f(x) b y = f(x+3) y

 – 2x, x ∈ ( – 2, 0 ] y = f(x)
g f ( x) = 
 2x 2 , x ∈ [ 0, 3 ] y = f(x)
 x ∈ [ 3, 4 ] 0 x 0 x
 –18x, +72,
3 a 5, 0, −3, −4, 0 b a=5
c f (x)
12 (5, 12) c d
y y
y = f(x) y = f(x)

(–2, 5)

(–1, 0) 0 x 0 x
–2 0 1 5 x y = −f(x) y = f(x) − 5

(0, –3)
–4 (1, –4) 5 a b
y y
d ii Domain = [−2, 5]
y = 2f(x) + 1
ii Range = [−4, 12] y = f(x)
4 a 2x
0 x 0 x

x y = f(x)
y = f(x−3) + 4

b P(x) = 6x c y d y
c 4
d Domain = [0, 4]; Range = [0, 24] y = f(x) y = f(x)
e P:[0, 4] → R where P(x) = 6x
f A:[0, 4] → R where A(x) = 2x2 0 x 0 x
g Length = 3, Width = 6 y = f(x+1) −3
5 a A(x) = 0.2 + 0.3x y = −f(x+4) − 2
b B(x) = 0.5 + 0.2x
c ii 80 cents
ii 90 cents
d ii 16 min Exercise 3B — Sketching graphs using
ii 22.5 min transformations
e 3 min 1 a Dilation by the factor of 2 in the y direction
f Plan B b Dilation by the factor of 1--3- in the y direction
739

answers
Answers

c Dilation by the factor of 3 in the y direction, 6 a y b y


reflection in the x-axis
d Translation 6 units down 1
0.8
e Dilation by the factor of 3 in the y direction, x x
translation of 4 units up
f Dilation by the factor of 1--2- in the y direction,
reflection in the x-axis, translation of 1 unit up
g Translation of 2 units to the right c y d y
h Reflection in the x-axis, translation of 3 units to
x
the left 4
i Dilation by the factor of 2 in the y direction, 2.08
x
translation of 3 units to the right –6
j Translation of 2 units to the left, translation of
1 unit down –128

k Translation of 0.5 unit to the right, translation of


e y f y
2 units up
l Dilation by the factor of 2 in the y direction, 4
4
reflection in the x-axis, translation of 3 units to
the left, translation of 1 unit up
2 a II b V c I d IV e III
3 a y b y x x
2 1
–1 g y h y
4
3
x x
– 1– 1– 2 3
2 2
1
–0.3 x x
c y d y 1 –2
–5
–0.6
1
x i y
12
–1
x –1 0.4 x
2
–4
e y f y –6
1
11–2 2 2 1–2 x
20
7 a Dilation in the y direction by the factor of 5,
2 reflection in the x-axis
x b Translation by 4 units down
–3
–15 c Translation by 1 unit to the right
d Dilation in the y direction by the factor of 2,
translation by 3 units up
4 a Dilation in the y direction by the factor of 7 e Reflection in the x-axis, translation by 2 units to
b Dilation in the y direction by the factor of 2--3- , the right
reflection in the x-axis f Translation by 1 unit to the left, translation by
c Translation by 4 units up 3 units down
d Reflection in the x-axis, translation by 6 units up 8 a y b y
e Translation by 1 unit to the right
f Reflection in the x-axis, translation by 3 units to
the left
g Dilation in the y direction by the factor of 4, 0 x −2 0 2 x
reflection in the y-axis, translation by 2 units to
the right −16
h Translation by 2 units to the right, translation by c d
1 unit up y y
i Dilation in the y direction by the factor of 3, 81
translation by 3 units to the left, translation by 1
2 units down 0 x x
3 −1 0 1
5 a i, iv b iii, v c ii
d i, ii, iv e ii, v f iii, iv 3A

3B
answers 740 Answers

e y f y 3 a y b y
15
−1 0 x
(−1,−2) −2 –4 2–
5
x

−5 –2 4– x –8
−3 −2 −1 0 x 3
−1 (– 1– , – 25
—)
(−2, −1) 3 3 –8
(– 9– , – 121
—–)
5 5
Exercise 3C — Sketching graphs using
intercepts
1 a 1, 6; 6 b 3, −2; −6 c −5, −1; 5 c y d y
d 4, 5; 20 e −7; 49 f 0, 2; 0
g 3, 4; 12 h −5, 5; 25 i 0, −8; 0
– 5– 3 x 3
j −9; −81 k −6, 6; −72 l 2, −10; −60 3
2 a y b y
–15
1– 3– x
2 2
3 ( 2– , – 49
—) (1, –1)
3 3
1
1 3 x (–1, 0) x
(2, –1) e y f y
c y d y 3–
7
3 –3 x
8 –9
35

1– 3– x
2 4
( 5– , – 1– ) (– 9– , – 144
–— )
x 8 8 7 7
–4 –2
(–3, –1)

–7 –5 4 a y b y
x (–4, 49)
(–6, –1) (1, 4)
e y f y 3
33
–9 7 x –1 3 x
12

–11 3 x

2 6 x
–63
(4, –4) (–1, –64)
g y h y c y d y
(9, 0)
(–9, 36)
x
6
–15 –3 x
2
–45
–2 –1 x
2 3 x
(– 3– , – 1– ) –81
2 4 ( 5– , – 1– )
2 4
i y

e y f y
( 3– , 4) (– 3– , 49
—)
2 8 8

–1 12 x
5
–12 1– 5– x
2 2
–5
– 5– 1– x
(11 169 )
— , – —– 4 2
2 4
741

answers
Answers

5 a y b y e y f y
1 12 3–
–5 5 x 4 3 6 x
– 1– 4 x
2

–1 1 x

–54

–25 7 a y b y
30
c y d y

3 –8 – 3– 8 x
–3 3 x 2
–192
– 3 3 x
–5 –3 2 x

c y d y
–18
e y f y 150
12 –1 2 x

–1
–10 –5 x

–5 0 x

(– 5– , – 25
—)
e y f y
2 4

–2 2 x

g y h y
(–3, 36)
7
–8 x –7 1– x
6
0 2 x 8 D
9 C
10 B
(1, –5) –6 0 x
11 a y b y
i y
( 7– , 147
–—) 12
2 4 8

0 x
–2 –1 x
–2 –1 0
2 3
2
c y d y

0 7 x 0 1 x (3, 16)
–5 1
–5
6 a b 0 x
y y 1

x –6 –1 7 x
e y f y
1 2 3
–42

–6 4

0 x
3 x
0 1–
c y d y –3 2
–1 2

g h y
–8 –1 11
60
y
x 125
–88

1 –5 0 1 x
5–
2 0 1– x
–4 3 x 3

3C
answers 742 Answers

12 C 3 a i x = 4, y = 0 ii Domain: R\{4}
13 E iii Range: R\{0}
14 b y b i x = 0, y = 2 ii Domain: R\{0}
12 iii Range: R\{2}
c i x = 3, y = 2 ii Domain: R\{3}
iii Range: R\{2}
d i x = −1, y = −1 ii Domain: R\{−1}
– 2– iii Range: R\{−1}
3
–3 –2 x
e i x = m, y = n ii Domain: R\{m}
iii Range: R\{n}
15 b y
14 f i x = b, y = a ii Domain: R\{b}
iii Range: R\{a}
–7 –2 1 x
4 a y b y

–1
3
– –21
–2 –1 x
–3 x
–1
16 b y
840
c y d y

x
1 5 –5 x
– –43 – –52
–8 –5 0 3 7 x
–3 –43

17 b e y f y
y
30 1
7 –2
3 6
1
x 2
–1 x
−5 −3 −2 0 1 x –3 2 –21

g y h y
4 –2
5

Exercise 3D — The hyperbola 1


–2
5

1 a Dilation in the y direction by the factor of 2 2 x


–1
x –11 –1
b Dilation in the y direction by the factor of 3, 2 1
reflection in the x-axis
c Translation by 6 units to the right
i y
d Dilation in the y direction by the factor of 2,
translation by 4 units to the left 4 –31
e Translation by 7 units up 4
f Dilation in the y direction by the factor of 2,
x
translation by 5 units down –1 –85 – –23
g Translation by 4 units to the left, translation by
3 units down
h Dilation in the y direction by the factor of 2, 5 E
translation by 3 units to the right, translation by 6 a y b y
6 units up
i Dilation in the y direction by the factor of 4, 1
–1
reflection in the x-axis, translation by 1 unit to 2
–1
x
the right, translation by 4 units down –2 x
2 a v b iii c i
d v, iii e v, ii, iii f i, iii
g v, i, iv h ii, iv
743

answers
Answers

c y d y e y f y
3 3.7 –1
2
– –21 2
2
x
x –4 –3 –43
x
–2 1–21 x –3
1
–1 –21

g y h y
e 4.4
y
x 2
–3
2
x
–2
1 x
–1
i y

0.4
1 2 x
–1

7 E
Exercise 3E — The square root function
1 a Dilated in the y direction by the factor of 2 8 y y=3 4–x +3
b Dilated in the y direction by the factor of 1--3- , 9
reflected in the x-axis
c Dilated in the y direction by the factor of 3, (4, 3)
translated 1 unit to the right x
d Dilated in the y direction by the factor of 2,
reflected in the x-axis, translated 4 units to the 9 a i p=8 ii y = −4 x + 1 + 8
left iii x = 3 iv x ≥ −1
e Translated 1 unit down vy≤8
f Dilated in the y direction by the factor of 3, b y
reflected in the x-axis, translated 2 units up (–1, 8)

g Translated 4 units to the right, translated 3 units


4
up x
h Dilated in the y direction by the factor of 2, 3
reflected in the x-axis, translated 3 units to the
left, translated 6 units up
i Dilated in the y direction by the factor of --12- , Exercise 3F — The absolute value
reflected in the x-axis, reflected in the y-axis, function
translated 2 units to the right and --23- units up 1 a y b y
2 a (0, 0) b (0, 0) c (1, 0)
1
d (−4, 0) e (0, −1) f (0, 2)
g (4, 3) h (−3, 6) i (2, 2--3- ) x x
1
3 E
4 D c y d y
5 a Domain: x ≥ −1, range: y ≥ 0 3 6
b Domain: x ≥ 0, range: y ≥ −3
c Domain: x ≥ 0, range: y ≤ 5 x x
–1 – 6 6
d Domain: x ≥ −2, range: y ≥ −1 2

e Domain: x ≥ 4--3- , range: y ≤ 2


e y f y
f Domain: x ≤ 2, range: y ≥ 6
5
6 a y b y 4 4

1.4 x
3 –2 2 x
1 3 5
–2 x x g y h y
c y d y
7
2
x 1
4 (6, 1)
x x
–2 –1 x 3D

3F
answers 744 Answers

i y Exercise 3G — Addition of ordinates


1 a R\{0} b [0, ∞) c [0, ∞)
2 d [−2, ∞) e R f R\{3}
x
g R\{−1} h (−∞, 1] i R\{0}
1
j [−1, 3]
2 C 2 C
3 a Domain: R, range: y ≥ 0 3 a y b y
h(x)
b Domain: R, range: y ≥ 1
f(x)
c Domain: R, range: y ≤ 4
g(x)
d Domain: R, range: y ≥ −2
x x
e Domain: R\{−1}, range: y > 1 f(x)
f Domain: R\{0}, range: y ≥ 0 g(x)
h(x)
4 a y b y
c y d y
f(x)
x x
–11 –5 –1 1 g(x) h(x)
x
–6 g(x)
x

f(x)
c y d y
h(x)
7 2 e y f
h(x) y

(–1, 1) (1, 1) f(x)


1
3 x x
x
g(x)
e f f(x)
y y
x
2 g(x)
–2 –1 0.7
x 4 y h(x)
–2.7
– 2 x –1 x2 + 5x + 6
–2 2 2 8
–2 7
6
g y h y 5
4
3
– –34 –4 3 –31 f(x)
3 2
x 3 1
– –43 6 x
–6 –5 –4 –3 –2 –1 0 1 2 x
g(x)
i y j y
5 y y = x3 + x2 – 1
–1 1
x
– –41 –1 3
4 –1
x 2
–4 1
x
–3 –2 2
k y l y
5 f(x) –2
2
g(x)
3.6 –5 x
35 63 99
3 6 a y
y= x+ 2–x
(–1, –6)
2
–2 2 x
y= 2–x y= x
1
3
5 a y = --- x , −2 ≤ x ≤ 2
2
1 2x
3
b Yellow: y = 6 − --- x , −2 ≤ x ≤ 2; b y y = 2x
2
3
green: y = --- x − 6, −2 ≤ x ≤ 2;
2 y = 2x – x
–1 x
3 4
blue: y = − --- x , −2 ≤ x ≤ 2
2 y=– x
745

answers
Answers

c y y = x–3 9 a $
11
10
x 9
8
7

Price
6
5
4
3
y = x – 3 – x2 2
1
y = –x2
0 2 4 6 8 10 Month
d y
b p=2 m +4
2 5 y = x+5+ 5–x c 10.63, 10.93
y = 5–x
y = x+5

–5 5
x Exercise 3I — Modelling using a
graphics calculator
7 E
Check with your teacher.
Exercise 3H — Modelling
Chapter review
1 a y = ax3, a = 0.3 b y = ax2, a = −6
1 a y
a y = 2f(x)
c y = a x , a = 1.6 d y = --- , a = 5
x
e y = ax3, a = −1.5
−3 0 4 x
2 a iii b ii c i
d iv y = f(x)

3 D b y
4 a y b y
50 50 y = f(x − 3)
40 y = f(x)
40
30 30 −3 0 34 7x
20 20
10 10
0 5 10 15 20 25 x2
0 1 2 3 4 5 x –10 c y
–10
c a = 2, b = −3.2 y = −f(x)

3 y = f(x)
5 y = 1--4- x − 12
−3 0 x
4
6 a f b f
1000 1000
800 800
600 d y
600 y = f(x) + 4
400
400
200 4
200 y = f(x)
0 0.5 1 1.5 2 2.5 3 3.5 —
1
λ
−3 0 4 x
0 2 4 6 8 10 λ
340
c f = ---------
λ
2 a Translated 7 units up
7 a I b Translated 1 unit left and 3 units down
250 c Dilated in the y direction by the factor of 2,
200 reflected in the x-axis, translated 3 units right and
150 1 unit up
100 3 a (3, −4) b Domain: R, range: y ≥ −4
50 c y
0 14
1 2 3 4d
270
b I = --------
2
-
3
d x
1.6 4.4
8 y=3 x +4 –4 3G

3I
answers 746 Answers

4 a y e y

(−1,2)
2

−1 0 x −3 0 1 6 x
−2
−18

b y f y

0 x −8 0 1 x
5
−3

c y g y

11

−2 0 x −5 −2 0 1 3 x
−5

−30

5 A
h y
6 C
7 a y
84 −4 0 2 7x

0 6 14 x
(10, −16)
i y

b y

−4 −1 0 1
2
x
−10 0 8 x
−16

−80 8 E
(−1, −81)
9 B
c y
10 C 11 E
(−1 1–4 , 1–8 ) 12 a x = −2, y = −1

−3
2
−1 0 x b Domain: R\{−2}, range: R\{−1}
−3 –4
c y = ------------ − 1
x+2
d y d y

–6 –2
x
−11 0 2 x –1
–3
747

answers
Answers

13 a y b y
7

3
−2 0 2 x
0 2 x
–1

c y
b y 5

−5 0 5 x
–4
x
–1

19 B
20 a y (f + g) (x)
c y
g(x)
f(x)
2 x
1 1–2
0 x
3 4
b y
(f + g) (x)
14 a y
g(x)
6 f(x)

0 x

0 x

b 100
y 21 E 22 E 23 y = ---------
2
x

x
Modelling and problem solving
−3 0 1
−2 1 a i
(−3, −2) 3−2
y
f(x) D(4, 6)

c y
B C(2, 3)
A 2
–2
A' B' x
5 (1, 5) –2
C'(2, –3)
x
01 f(x)
26 D'(4, –6)

A(−2, 0), B(0, −2), C(2, −3), D(4, −6)


15 C 16 A 17 B ii y
18 a y f(–x) f(x)
D'(–4, 6) D(4, 6)

B
−6 01 8 x C'(–2, 3) C(2, 3)
A A'
–2 2 x
−6
−7
(1, −7)
A(2, 0), B(0, 2), C(−2, 3), D(−4, 6)
answers 748 Answers

iii y e h
f(x–2) f(x)
50 (0, 48.4)
D (4, 6)
D'(6, 6) (5, 48) (20, 42)
C(2, 3) C'(4, 3) 40
B2
A B'(2, 2) 30
–2 A' x
20

A(0, 0), B(2, 2), C(4, 3), D(6, 6) 10


(24.2, 0)
iv y 0 5 10 15 20 25 t
D'(4, 9) f a = −2.5, c = 10
C' (2, 6)  40 0 ≤ t ≤ 24
B' D(4, 6)  50 + ------------- ,
5 g h (t) =  t – 25
A'(–2, 3)  −2.5(t − 24)2 + 10, 24 ≤ t ≤ 26
B C(2, 3) 
2
f(x) + 3 A
–2 x CHAPTER 4 Triangle
f(x) trigonometry
A(−2, 3), B(0, 5), C(2, 6), D(4, 9) Exercise 4A — Calculating
trigonometric ratios
v 2f(x)
y 1 a 1.540 b 17.663 c 40.460
D'(4, 12) d 0.657
2 a 0.602 b 2.092 c 15.246
f(x) d 51.893
C'(2, 6)
D(4, 6) 3 a 0.707 b 0.247 c 6.568
B'4
C(2, 3) d 5.896
A 2B 4 a 0.5 b 0.9659 c 1
A' –2 x
d 548.6 e 64 f 1.301
g 5.306 h 1.374 i 15.77
A(−2, 0), B(0, 4), C(2, 6), D(4, 12) 5 a 0.42 b 1.56 c 0.09
d 5.10 e 2.87 f 0.38
vi y f(x) g 7.77 h 73.30 i 0.87
1 – f(x + 1) D(4, 6) 6 10°
7 a 44° b 80° c 57°
B C(2, 3) 8 86°40′
A'(–3, 1) 2
A x
9 a 42°57′ b 31°21′ c 16°5′
C'(1, –2)
B'(–1, –1) Exercise 4B — Finding an unknown
D'(3, –5) side
1 a hyp
A(−3, 1), B(−1, −1), C(1, −2), D(3, −5) opp
θ
b Add multiples of 2, for example, f(x) + 2, adj
f(x) + 4, f(x) + 6, f(x) − 2 etc. and keep the b
domain fixed at [−3, 7]. α
2 adj hyp
2 a y = a(x − h) + k
b h=9
opp
c Straight line (negative gradient)
d a = −0.55, k = 275 c
adj opp
e y = −0.55(x − 9)2 + 275 γ
f y = −0.56x2 + 9.96x + 230.39 hyp
g No, the prices started going down. 2 148.1 mm
h $266 000, $261 000 3 5.08 m
i About 4 months 4 30.0 cm
3 aa = 40 5 a 12.1 cm b 55.2 m c 9.43 km
b i 48 m 6 a 12.5 m b 89.3 mm c 10.1 m
ii 42 m 7 a 5.42 m b 1.35 km c 2.06 km
c 24.2 s d 18.4 mm e 3.20 cm f 66.5 m
d The speed is slow to start but increases rapidly as g 5.40 m h 5.39 km i 0.240 m
the eagle approaches the ground. j 41.6 km k 84.4 m l 13.2 cm
749

answers
Answers

8 D 9 A Exercise 4E — Using the sine rule to


10 6 m 11 4.2 m 12 20 km find side lengths
13 a b 30.3 cm a b c
13.5 cm 1 a ------------ = ------------ = -------------
sin A sin B sin C
24°

14 a b 1.6 m x y z
b ------------ = ------------ = ------------
sin X sin Y sin Z
1.4 m p q r
c ------------ = ------------- = ------------
sin P sin Q sin R
60°
2 a 14.8 cm b 1.98 km c 112 mm
15 9.65 m 3 a 10.0 m b 22.1 cm c 39.6 km
16 a b 58 m 4 9.8 cm
5 27.0 m
15°
60 m 6 37.8 m
7 a B b 43.2 m c 33 m

34°
c 15.5 m 49° M
N 20 m
8 43.62 m
Exercise 4C — Finding angles 9 a 6.97 m b 4m
1 a 30° b 75° c 81° 10 a 8.63 km b 6.48 km/h c 9.90 km
2 a 32°48′ b 45°3′ c 35°16′ 11 22.09 km from A and 27.46 km from B.
3 a 53°8′ b 55°35′ c 45°27′
12 D
4 a 50° b 32° c 33°
13 B
d 21° e 81° f 34°
5 a 39°48′ b 80°59′ c 13°30′ 14 Yes, she needs 43 m altogether.
d 79°6′ e 63°1′ f 19°28′
6 A 7 D 8 37°
9 75°31′ 10 8°38′ 11 14° 12 4°35′ Exercise 4F — Using the sine rule to
find angle sizes
1 a 43° b 34° c 27°
Exercise 4D — Applications of d 75° e 37° f 2°
right-angled triangles 2 B
1 571 m 2 30 m 3 91 m 4 43.18 m 3 B
4 38°
5 a 22.33 m b 13.27 m 5 20°
6 2°44′ 6 84°
7 a 57° b 63°
7 a b 1319.36 m 8 54°
Helicopter
9 a 13.11 km b N20°47′W
35°
48°
2500 m
Exercise 4G — Using the cosine rule to
S2 S1 find side lengths
8 22 m 1 a a2 = b2 + c2 − 2bc cos A
b r2 = p2 + q2 − 2pq cos R
50 3
9 50 – ------------- m c n2 = l2 + m2 − 2lm cos N
3 2 7.95
10 a 325° T b 227° T c 058° T d 163° T 3 a 8.05 m b 14.3 cm c 12.0 m
11 a S66°W b S73°E c N39°W d N74°E 4 a 185.1 cm b 14.4 m c 104.4 mm
5 55.22
12 a C b D 13 1691 m
6 23.08, 41°53′, 23°7′
14 a 5.39 km b N21°48′W 7 7 km
15 201°48′ T 8 40 m
16 a 4.36 km b 156°35′ T 9 2218 m
17 a 12.2 km b 348° T or N12°W 10 28.5 km
11 1.14 km/h
18 a 29.82 km b 38.08 km c 232° T 12 E
19 a 112.76 km b 5 hours 30 minutes 13 B
20 a 82.08 m b 136.03 m c 301°6′ T 14 55 cm 4A

4G
answers 750 Answers

Exercise 4H — Using the cosine rule to CHAPTER 5 Graphing periodic


find angle sizes functions
2 2 2
b +c –a
1 a cos A = ---------------------------- Exercise 5A — Period and amplitude of
2bc a periodic function
2 2 2 1 a 4 b 1
p +r –q
b cos Q = ---------------------------- 2 a T = 4π A = 2
2 pr
b T=π A=1
2 2 2
a +m – p c T = 3π A = 1.5
c cos P = ------------------------------
2am 3π
d T = ------ A = 4
2 a 85° b 83° c 45° 2
3 a 103° b 137° c 10° e T = 2π A = 2
4 88°15′ 4π
5 28°57′ f T = ------ A = 3
3
6 A = 61°15′, B = 40°, C = 78°45′
g T = π A = 2.5
7 82°, 54°, 44°
8 32° 2π
h T = ------ A = 0.5
9 a 12.57 km b S35°1′E 3
10 23° 11 89.12 m 1
3 --4-
12 a 130 km b S22°12′E 4 T=2 A=2
13 74.3 km 14 70°49′
5 a Approximately periodic
b 12 months
Chapter review c February
1 a 0.7193 b 4.2303 c 2.7400 d August
d 8.1955 e 21.9845 f 14.2998 e 18
2 a 54° b 51° c 53° 6 A
3 a 78°31′ b 26°34′ c 14°54′ 7 a Oestrogen
4 a 37.9 cm b 3.8 m c 13.6 cm b 28 days
d 11.7 cm e 14.7 cm f 14.6 m
g 1.5 m h 4.9 cm i 15.6 mm
j 7.5 m k 10.7 m l 5.3 km
Exercise 5B — Radian measure
5 8.5 m π π π
1 a --- b --- c ---
6 2.5 km 6 4 3
7 63.9 m 5π
π e ------ π
8 a 57° b 27° c 68° d --- f ---
9 18 2
9 a 23°4′ b 61°37′ c 59°35′
10 39° 11 24° 3π 5π
g ------ h 2π i ------
12 9.38 m 2 6
13 a 12.59 km b S36°10′E 5π 7π 5π
14 2783 m j – ------ k – ------ l ------
4 3 3
15 a 1.67 cm b 81.7 mm c 9.81 km
16 12.4 cm 4π 2π
m ------ n ------
17 a 52° b 21° c 68° 15 5
18 809 cm2 2 a 36° b 120° c 40° d 220°
19 a 8.64 m b 8.80 m c 11.8 cm e 648° f −30° g −45° h 67.5°
20 84.0 cm
21 985 m 3 E
22 a 60° b 112° c 139° 4 A
23 34° 5 a 0.855 b 1.361 c −2.182
24 29° d 3.334 e 4.084 f 5.707
g 2.967 h 3.787
6 a 20° b 84° c 180°
Modelling and problem solving d 55° e 894° f −155°
1 a 39° b 25° g 233° h 458°
2 a 15° b, c Check with your teacher. d 22.7 m
3 a i 60 mm ii 394 mm iii 10° Exercise 5C — Exact values
b i 312 mm ii 137 mm, 376 mm 3 b 1
c 1 1
1 a ------- ------- ------- d ---
4 Tower B, 11.6 km 2 2 3 2
5 14.8 km from the northern lighthouse, 22.7 km from e 1 3
--- f 1 g ------- h 3
the southern lighthouse 2 2
751

answers
Answers

1
2 a ------- b 1
--- c 3 d 1
--- 2 a i 2π ii 1 b i 2π ii 3
2 2 2

3 1 1 c iπ ii 2 d i ------ ii 4
e 1 f -------
2
g ------- h ------- 3
2 3
3 a P b P c P d P ii 1 π
e i 6π ---
2
f i --- ii 2
e N f N g N h N 2
i P j N 2π
g i 6π ii 0.4 h i ------ ii 3
4 a P b P c N d N 5
e N f P g P h P π
i N j N i i8 ii 2.5 j i --- ii 1
3
5 a P b P c N d N 1 1
k i2 ii ---
5
l i4 ii ---
4
e P f P g N h N
i N j P 3 a i π ii 1
6 a Quadrant 3 b Quadrant 1 y
7 a C b B c A 1
d B e C
x
8 a −1 b −1 c 0 d 0 − −
0
−2 −3 3 2
e 0 f 1 g Undefined h 0 2 2 2 2
−1
i −1 j 1
b y i 2π ii 2
2
Exercise 5D — Symmetry
1 a 0.63 b −0.63 c −0.63 d −0.63 − 0 x
−2 −3 3 2
2 2 2 2
2 a −0.25 b −0.25 c 0.25 d 0.25 −2
3 a −2.1 b −2.1 c 2.1 d −2.1 c y i 4π ii 3
4 a –0.3 b −0.7 c −0.9 d −0.3
3
e 0.3 f 0.7 g 0.7 h −0.9
i 0.9
0 x
−2 2
1 1 1
5 a --- b − ------
- c 3 d − ------
-
2 2 3 −3

e 3
− ------
- f 3
− ------
- g − 3 h 3
− ------
- d y i π ii 4
2 2 2
4
1 1
i ---
2
j − ------
- k 0 l −1
2

1
6 a − ------ 1 3 − − 0 x
- b ---
2
c −1 d ------- −2 −3 3 2
2 2
−7 2 −5 −3 2 − 2 3 5 2 7
1 4 4 4 4 4 4 4 4
e 1
--- f − ------
- 3
g − ------
-
3
h − ------
- −4
2 3 2 2
1
i − ------ j 1 2π ii 1
- ------- k −1 l −1 e i ------ ---
2
3 2 3
7 a –0.383 b −0.924 c 0.414 d 0.924 y
1
e 0.383 f −0.414 2

8 a 0.966 b −0.259 c −3.732 d −0.966 − − 0 x


−2 −5 −4 −2 2 4 5 2
e 0.259 f −3.732 3 3 3 3 1 3 3 3 3

2
9 a 0.644 b −0.765 c −0.842 d −0.644 i 4π ii 2
f y ---
3
2
3
Exercise 5E — Trigonometric graphs
− 0 x
1 a i 4π ii 2 b iπ ii 1 2

−2 3 2

c i 3π ii 1.5 d i ------ ii 4
2 g y i 6π ii 5
4π 5
e i 2π ii 2 f i ------ ii 3
3 −2
2π 0 x
2
g iπ ii 2.5 h i ------
3
ii 0.5
−5
4H

5E
answers 752 Answers

h y i 4π ii 4 9 a y b y
4 1.5 1.8
1.3

0 −2 2 −2 2
−2 − 2 x x
−3 −3 0 3 3 x −3 0 3
−4 2 4 4 2 2 −0.9 2
−1.3
−1.5
i i π
--- ii 2
2
y
c y
2 1.4
0.6 2
− − 0 x
−2 −3 3 2 0 x
2 2 2 2 −6 −4 −2 2 4 6
−2 −0.6
−2
4 a D b C c A −1.4

5 E d y
2x 3
6 a y = 1.5 sin ------ b y = 2 cos 2x
3 −4 4
3 3
x 2x 0 x
c y = 5 sin --- d y = 4 cos ------ −2 2
2 3 −2 2
3 3
3x −3
e y = −sin ------ f y = −3 cos 3x
2 8π 3x
10 a f: [0, ------ ] → R, f(x) = 3 sin ------
7 a y
6 2
2x
1 b f: [0, 5π] → R, f(x) = --52- cos ------
5
0 x
c f: [−1, 1] → R, f(x) = 2 sin π x
−360° −270° −180° −90° 90° 180° 270° 360°
πx
−1 d f: [−1, 3] → R, f(x) = 1.8 cos ------
2
b y 2πx
e f: [0, 3] → R, f(x) = −3 sin ---------
−180° 3 180° 3
1 3πx
f f: [− --3- , 1] → R, f(x) = −2.4 cos ---------
0 x 2
−360° 360° 11
−3 Vertical asymptotes
y
c y
(54 , 1) y = tan x
2
1 − ( 4 , 1)

x −2 −3 − −
−1
0 3 2 x
−360° −240° −120° 120° 240° 360° 2 2 4 2
(34 , −1) 2 ( 74 , −1)
−2

8 a y b y
1.5
3 1.3
−2 Exercise 5F — Applications
0 2 x 1 a i 1 kg ii 6 days
0 x − 1.3
−2 −3 − − 3 2 − 1.5 πt
4 2 2 4 b W = cos ----- + 3
−3 3
2 a 110 beats/min
c y b i 50 ii 60 min
4 πt
c H = 50 sin ------ + 110
30
3 a 1.6 m b i 1 m ii 0.7 m
−2 4 a 26°C at 2 pm
0 x
2 b i 18°C ii 22°C iii Approx. 11.1°C
−3 − − 3
2 2 2 2 1
5 a i 12 mm ii -----
10
- s

−4 b 10
753

answers
Answers

π
c −11.41 mm; if the displacement is positive to the 2 a 40 cm b 80 cm c 0.3 sin  ------ x
 16 
right then the string is 11.41 mm to the left (or
vice versa)
6 a 40 m b 3.9 s c 7.8 s
7 a 60 s b 50 m c 100 m CHAPTER 6 Trigonometric
d 314.16 m e i 50 m ii 75 m equations
8 a i 6 m ii 3 m b Yes, by approx. 24 minutes.
Exercise 6A — Simple trigonometric
Chapter review equations
1 T=6 A=2 1 a 37° and 143° b 104° and 256°
2 f = 1--6- c 80° and 260° d 238° and 302°
3 A periodic function repeats itself over time. e 79° and 281° f 140° and 320°
g 199° and 341° h 41° and 319°
π 5π 5π 13 π
4 a --- b ------ c ------ d --------- 2 a 60° and 120° b 45° and 315°
4 6 18 9 c 60° and 240° d 210° and 330°
5 a 30° b 216° c 300° d 172° e 120° and 240° f 150° and 330°
6 E 7 E 8 E 9 B 10 D g 225° and 315° h 30° and 330°
11 A 12 E 3 C
13 a 1--2- b 1--2- c 1 4 Third quadrant, as sin is negative and tan is positive
14 a 1
------- b 3
------- c 3 in this quadrant.
2 2
5 a The value of sin θ can never be greater than 1.
15 a 0.69 b 0.69 c −0.69 b There is no maximum value to tan θ.
16 D 17 C 18 E 19 E 6 a 90° b 90° and 270°
20 a y b y c 0°, 180° and 360° d 0°, 180° and 360°
4 1.5 e 180° f 270°
7 a 30° and 150° b 70° and 290°
c 135° and 315° d 240° and 300°
–2π –π 0 π 2π 3π 4π x e 150° and 210° f 54° and 234°
–4 –π – –π 0 –π π x Exercise 6B — Equations using radians
2 2
1 a 0.93 and 2.21 b 2.09 and 4.19
c 0.98 and 4.12 d 5.95 and 3.47
–1.5 e 0.79 and 5.50 f 2.77 and 5.91
21 a i 2800 ii 1200 iii 2000 π 2π 2π 4π π 5π
b i 12 months ii 0.8 iii 1.25 2 a --- and ------ b ------ and ------ c --- and ------
3 3 3 3 4 4
c P (thousands)
π 7π 2π 5π 7π 11 π
d --- and ------ e ------ and ------ f ------ and ---------
2.8
4 4 3 3 6 6
2 π 3π
3 a 0, π and 2π b 0, π and 2π c --- and ------
1.2 2 2
π
0 3 6 9 12 t (months) d --- e 0 and 2π f π
2
d 1600

g ------
Modelling and problem solving 2
1 a 25.4 °C, 20.6 °C π 5π π 11 π π 5π
4 a --- and ------ b --- and --------- c --- and ------
b i 23 °C ii 25.4 °C 6 6 6 6 4 4
c T (°C) 4π 5π 3π 5π 2π 5π
d ------ and ------ e ------ and ------ f ------ and ------
27 25.4 3 3 4 4 3 3
26 5 D
25 6 a 0.25 and 2.89 b 2.30 and 3.98
24 c 1.29 and 4.43 d 3π (or 4.71)
23 e 1.36 and 4.92 f 1.89 and 5.03
22
21
Exercise 6C — Further trigonometric
20 equations
19 20.6 1 a 2.29, 3.99, 8.57, 10.27
18 b 1.14, 2.00, 7.42, 8.28
c 1.07, 5.21, 7.35, 11.49
0 1 2 3 4 5 6 7 8 t d 3.52, 5.90, 9.80, 12.19
t (hours) e 0.53, 3.67, 6.81, 9.95
d 2 hours, 40 minutes f 2.02, 5.16, 8.30, 11.44 5F

6C
answers 754 Answers

2 a −6.01, −3.41, 0.27, 2.87 11 5 1 4


b −3.88, −2.41, 2.41, 3.88 e ---------- f --- g ------- h ---
6 6 5 5
c −3.59, −0.45, 2.69, 5.83
d −2.57, −0.57, 3.71, 5.71 5 2 3 11
e −4.90, −1.38, 1.38, 4.90 i --- j ------- k – --- l ----------
6 5 5 5
f −5.03, −1.89, 1.25, 4.39
3 a
11 π 7 π π 5 π
– --------- , – ------ , --- , ------ b
3π π π 3π
– ------ , – --- , --- , ------ Exercise 6E — Using the Pythagorean
6 6 6 6 2 2 2 2 identity
2π π 4π 5π 4π π 2π 5π π π 3π 5π
c – ------ , – --- , ------ , ------ d – ------ , – --- , ------ , ------ 1 --- , --- , ------ , ------
3 3 3 3 3 3 3 3 3 2 2 3
7π π π 7π 11 π 5 π π 7 π 2 0°, 60°, 180°, 240°, 360°
e – ------ , – --- , --- , ------ f – --------- , – ------ , --- , ------
4 4 4 4 6 6 6 6 π π 3π
3 a 0, --- , π, 2π b --- , π, ------
4 a −4.71, 1.57 b −6.28, 0, 6.28 2 2 2
c −5.15, −1.14, 1.14, 5.15 4π 5π π 5π
d −5.76, −3.67, 0.52, 2.62 c 0, ------ , ------ , π, 2π d --- , π, ------
3 3 3 3
e −5.50, −0.79, 0.79, 5.50
f −3.67, −2.62, 2.62, 3.67 π π 7 π 11 π
e --- f --- , ------ , ---------
5 a 3m b 3.99 seconds 2 2 6 6

Exercise 6D — Identities π 5π 3π
4 --- , ------ , ------
1 6 6 2
q 30o 81o 129o 193o 260o 350o -47o 5 45°, 135°, 225°, 315°
π 5π 3π π 5π 3π
sin2 θ 0.250 0.976 0.604 0.051 0.970 0.030 0.535 6 a --- , ------ , ------ b --- , ------ , ------
6 6 2 6 6 2
cos2 θ 0.750 0.024 0.396 0.949 0.030 0.970 0.465 π 3π 5π 7π π 5π
c --- , ------ , ------ , ------ d --- , π, ------
4 4 4 4 3 3
sin2 θ + cos2 θ 1.000 1.000 1.000 1.000 1.000 1.000 1.000
π 3π 3π
e --- , π, ------ f ------
2 a 0.6 b 1.333 2 2 2
3 a 0.954 b 3.18 7 a 0.79, 2.55, 3.93, 5.69
4 a ±0.917 b ±0.714 c ±0.971 d ±0.436 b 0.15, 2.99
5 2 c 1.23, 1.37, 4.91, 5.05
5 a 2 b ------- c ---
3 3 Chapter review
7 3 1 a 64°, 116° b 114°, 246° c 58°, 238°
6 a 6 b ------- c -------
4 7 2 a 240°, 300° b 60°, 300° c 45°, 225°
5 4 24 1 3 a 270° b 180° c 135° 315°
7 a – ------ b – --- c ------ d – ---
13 5 25 2 4 a −240°, −120°, 120°, 240°
b −246°, −66°, 114°, 294°
80 −288°, −252°, 72°, 108°
8 a ---------- b 80 c
9
1 5 a 0.78, 2.37 b 2.59, 3.69 c 0.20, 3.34
9 a − --- b− 3
2 5π 7π 2π 5π 5π 7π
11 5 6 a ------ , ------ b ------ , ------ c ------ , ------
10 a ---------- b − ---------- 6 6 3 3 4 4
4 11
5 π 2π 2π 4π 3π 7π
11 ------- 7 a --- , ------ b ------ , ------ c ------ , ------
2 3 3 3 3 4 4
12 a B b D c C d A 8 a 49°, 131° b 139°, 221° c 72°, 252°
13 a 70 b 32 c 51 d 8 9 a 2.05, 4.24, 8.33, 10.52
e 82 f 46 g 1 h 73 b 3.67, 5.75, 9.95, 12.04
14 c 2.44, 5.58, 8.73, 11.87
sin q 0.8 0.28 0.954 0.77 0.954 0.573 10 a −2.19, −0.95, 4.09, 5.33
b −5.15, −1.13, 1.13, 5.15
cos q 0.6 0.96 0.3 0.64 0.477 0.82 c −4.17, −1.03, 2.11, 5.25
tan q 1.33 0.29 3.18 1.2 2 0.7 11 a −4.10, −2.19, 2.19, 4.10
5π 2π π 4π
b −2π, −π, 0, π, 2π c – ------ , – ------ , --- , ------
4 4 1 3 3 3 3
15 a --- b --- c ------- d 2
5 3 5 119
12 a −0.87 b – -------------
12
755

answers
Answers

13 a 60 ° b 62 ° 2wt 6
e ----------- f x13 g −81x9y3 h 3m4p
π 5π 9
14 0, --- , ------ , π, 2π
6 6 u 11 v 4
i mp2 j -------------
9
π 3π π 125k 11 d 6
15 a 0.84, --- , ------ , 5.44
2 2
b ---
2 4 a 15
------ b ---------------------- c 16p17
4 24
π 5π 3π π 3 4
c --- , ------ , ------ d --- , 3.48, 5.94 d 27j3n2 e xy f 12
6 6 2 2 5 a A b E c B
6 a x3yn + 1z b x5y6m − 1
Modelling and problem solving 7 a 211 b 324 c 55 × 34
1 a Amplitude = 6, period = 2
b 74, 86 d 222 × 58 3 f 5
---
e ----- 2
c 1--2- s 24
8 a 8 b 59 049 c 16 d 1
d 1--6- s
1 1
e 5 f ---
5
g 16 h ---
3
e L (t)
86 9 a 22 × 33n + 4 b 53n − 6
−5n − 3 −6n
80 c 23x − 2 d 2 ×3 e 2n − 2 × 76n − 3
74 f 5−1 × 72 or 49
------ g 25n − 6
× 39n − 3 h 3
5
0 1 2 t
i − 2--3-
2 a i 19.9 cm ii 8.1 cm
10 E
b i 11.742 cm ii 8.1 cm
c, d H
Exercise 7B — Negative and rational
19.9 powers
1 1 52 45
1 a ----3- b ----- c ----- d -----
6 54 32 75
14
1
f -------- 1 2 12
e 92 g – --- h -------
64 6 3 3 16
8.1 1
b -------- p2
2 a –2 2 c -----
xy 2 m
t
1
0 1 2 d x 10 e 2 13 × 3 8 f --------------
4x 6 y 2
e 0.571 s f 52.5 times 3 a 3 b 3 c 5 d 2
2
5πt e 4 f 27 g 625 h ---
g H = 14 + 5.9 cos  -------- 5
 2  2 125 9 1
i ---
3
j ---------
64
k ------
16
l ---
4
1 9 11 6
m ------ n --- o ------ p ---
CHAPTER 7 Exponential and 27 4 4 5
4 a D b C c E
logarithmic functions 7 5 3
--- --- ---
5 a 3 3 b x 6 c x8
Exercise 7A — Index laws
1
1 a x10 b m5p5 c 518 d 8y11 7
--- y2
---
7
---
d x 6 e ----2 f 24
e x7y8 f 46x18 g 6m9p17 h 57x10y13 ---
x3
a9b5 x3 y9 1 1 i
1
------------ -
2 a a5b3 b ----------- c ---------- d p11q2 g -----7 h -----2 5
4 2 --- --- ---
24 33 2(33 )
8m 2 n 5 r 10 t j x k 64m 10 l x
e --------------- f a2b2 g --------- h 5a3b3
7 5 3
--- x–1
6x 2 m x2 n ( x + 1)2 o -----------
1
i -------- x2
---
5
3 3
2( x + 1) --- ---
4 p 10 m 8 9xy 4 11 5 9 2 p -------------------1- q ( y – 4)2 r ( p + 3)5
3 a ------------------
3
b -----------
2
c 18u v d 10e f
( x + 2)2
---
6D

7B
answers 756 Answers

Exercise 7C — Indicial equations i y j y


1 a 5 b 4 c 5 d 2
–1 –1 0 x
e −2 f −3 g 4 h 0 y = 0.5x
1
1 3 4
i 2 j ---
2
k ---
4
l ---
3 –3 1–
2

5 y = –3 –x 0 x
2 a ---
3
b − 1--2- c −1 d − 9--4- 1

11
e ------ f 9
--- k y l y
6 8 y = 2.7x
10
3 a −3 b ------
7
c − 18
------
5
d 3
---
4
2.7 y = ( 2–3 )x
1
2–
21
e ------
11
f 4
------ g − --52- h 25
------ 1 3
11 9
x 0 x
9 0 1
4 a ------
10
b 7 c − 5--7- d 19
------ 1
4
1
e −3 f − -----
11
- g 1 h 5 2 a b
y y = 2(3x) y y = 3(2x)
i 3 j −9
6 6
5 a 0 or 1 b 1 or 2 c 2 or 3 d 0 or 1
e 0 or 1 f 1 or 2 g 3 h 0 or 1 2
3
6 A
0 x 0 x
7 D 1 1
8 B y = 0, (0, 2) y = 0, (0, 3)
9 a 1.58 b 3.58 c 5.49 d 1.65 c d
y y = 0.5(4x) y y = 4(5x)
e 1.28 f 1.66
10 E 2 20

0.5 4
Exercise 7D — Graphs of exponential 0 1
x 0 1
x
functions y = 0, (0, 0.5) y = 0, (0, 4)
1 a y y = 3x
b y y = 5x
5 e y y = 1–4 (2x)
f y
3
1–
1 2 y = 4( 1–3 )x
1–
4
0 x 1 4–
1 4 3
0 x
1
0 x 0 x
1 1
c y y = 6x
d y y = 10 x y = 0, (0, 1--4- ) y = 0, (0, 4)
6 10

3 a y = 0, (0, 1--- ) b y = 0, (0, 9)


2
dom = R, ran = (0, ∞)
1 1 dom = R, ran = (0, ∞) y
y = 3x + 2
0 x 0 x y
1 1 y = 2x – 1

9
e y f y
1–
y = 2–x y = 4–x 2
4
2 0 x
0 x
1
1
x x
–1 0 –1 0 c y = 0, (0, 5) d y = 3, (0, 4)
dom = R, ran = (0, ∞) dom = R, ran = (3, ∞)
g y h y y = 51 – x y y

0 1 x y = 2x + 3
0 1 x 4
–1 –1
5 3
–3 –2

y = –3x y = –2x 0 x 0 x
757

answers
Answers

e y = −3, (0, −2) f y = −1, (0, 7) ( x + 3 )2


dom = R, ran = (–3, ∞) dom = R, ran = (–1, ∞) i log 3 64 j log 10 -------------------
x–2
y y
2
y = 3x – 3 9 a ---
3
b 2 c 2 d 3
---
7 2
y = 2x + 3 – 1 e 9
--- f 2 g 3 h 6
0 x 4
–2 0 i 3 j 6
x
–3 –1 10 E 11 B 12 D 13 C
14 a log 3 81 = 4 b log 4 1024 = 5 c log 5 -----8-
25
g y = 3, (0, 4) h y = 5, (0, 105)
dom = R, ran = (3, ∞) dom = R, ran = (5, ∞) d log 10 (100x3) e log 2 25
------
8
f log 3 4
y y
g log 6 9 h log10 ( 10x 6 )
y = 102 – x + 5
y = 6–x + 3 15
4 Exercise 7F — Solving logarithmic
3 equations
5 2 0 c 3 d 4
1 a b
0 x 0 1 x e −1 f −2 g 6 h 2
i 2 j 3
80
i y = −2, (0, −1 -----
-) j y = 1, (0, −3) 2 a 8 b 9 c 625 d 10
81
dom = R, ran = (–∞, 1) 1
dom = R, ran = (–2, ∞) y
e ---
8
f 1
------ g 64 h 10 000
27
y
1 6
y = 3x – 4– 2 –1
i 30 j 5 k 5 l -------
3
0 x
–1
m 24 n 10
(0, –180
—)
81
–3 3 a 6 b 5 c 4 d 10
0 x
y = –2x + 2 + 1
e 2 f 25 g 2 h 1
–2 4 a B b A c E d C
4 a A b E 5 a 3.459 b −0.737 c 2.727 d 0.483
e 1.292 f −3.080 g 2.255 h 0.262
5 a 3.32 b 4.39 c −0.51 i 0.661 j 0.431 k −0.423 l 2.138
d 1.30 e 0.90 f 1.65
6 B 7 C
g 1.37 h 2.26 i 2.44, −2.86
j 1.56, −3.99 Exercise 7G — Applications of
exponential and logarithmic functions
Exercise 7E — Logarithms 1 a P = 10 000(2t )
1 a log 2 8 = 3 b log 3 243 = 5 b i 80 000 ii 640 000 c 3.32 months
c log 5 1 = 0 2 a 15 000
d log 10 0.01 = −2
b i 15 529 ii 16 077
e log b a = n f 1
- = −4
log 2 ----- c 17 838 d 2032
16
2 a 42 = 16 b 106 = 1 000 000 3 a 3 kg
c 2−1 = 1 b i 3.77 kg ii 5.1 kg iii 5.86 kg
--- d 33 = 27
2 c W d 26 weeks
e 54 = 625 f 27 = 128
3 W = 3 log10(8t + 10)
g 3−2 = 1
---
9
h bx = a
3 D 4 C 5 A –5– 0 t
4
6 a 4 b 4 c 3
d −2 e 3 f −5 4 a A = P(1.05)n b $16 288.95 c 20 years
g −2 h −5 i 5
5 a $25 000 b $14 427 c 10 years
j −6 k Undefined l 5
6 a 90°C
7 a log 2 80 b log 3 105 c log 10 100 = 2 b i 76.3°C ii 64.7°C c 12 min 37 s
d log 6 56 e log 2 4 = 2 f log 3 3 = 1 7 a 120
g log 5 12.5 h log 2 3 i log 4 5 b i 145 ii 176 iii 213
j log 10 1--4- k log 3 4 l log 2 3 c 11 1--2- years d N

m log 3 20 n log 4 2 = 1--2- N = 120(1.1t)


120
8 a log 10 250 b log 2 1728 c log 3 4
d log 5 3 e log 10 1--4- f log 3 2
g log 2 1--2- = −1 h log 2 [x3(x − 4)] e Population will reach a limit at some stage.
0 t
7C

7G
answers 758 Answers

8 a 80 kg Modelling and problem solving


b i 72.1 kg ii 52.8 kg c 67 weeks 1 a i 20 ii 25
d i W b i L = 25, C = 28 ii L = 28, C = 30
c Lions by 1 year 1 month
80
d 31 after 1 year 11 months
W = 80(2–0.015t) 2 a 80 ºC
b i 72.2 ºC ii 30.6 ºC
0 t c T
ii No, the model suggests virtually no rubbish will 80 T(x) = 60 (4–0.05t) + 20
70
be disposed of in 10 years or so which is unlikely. 60
9 a i 108 hectares ii 120 hectares iii 148 hectares 50
40
b 15 h 30
10 a i 51.18 m ii 51.65 m iii 51.95 m iv 52.2 m 20
b d c 67 10
0 10 20 30 40 50 x
52 d 13.2 minutes e 20 ºC
51 d = 50 + log10(15n) 3 a 12 000
b i 16 970 ii 40 363
0 n c 11.3 days d 4 days
1 2 3 4
2–
3 e 4 more days
11 a 400 b i 1005 ii 6340 iii 16 643 iv 17 146
c 17.48 months
12 a h = 5(0.7n) b i 1.20 m ii 0.29 m CHAPTER 8 Applications of
c h
exponential and logarithmic
5 h = 5(0.7n), n ≥ 0, n ∈ J
functions in financial
4
3
mathematics
2
1 Exercise 8A — Geometric sequences
0 1 2 3 4 5
n 1 a Not geometric
13 a V = 5000(1.1t ) b $8857.81 c 8 years b Geometric, ratio = 3; t4 = 108; tn = 4 × 3n − 1
c Geometric, ratio = 2; t4 = 24; tn = 3 × 2n − 1
d Geometric, ratio = 3--2- ; t4 = 13 1--2- ; tn = 3n − 123 − n
Chapter review
4y 2 e Geometric, ratio = − --13- ; t4 = --19- ; tn = (−3)2 − n
1 B 2 C 3 -------- 4 A 5 B
6 E 3x 4
f Geometric, ratio = −3; t4 = −54; tn = 2 × (−3)n − 1
7 a x = 2.187 b x=5  3--- n − 1
g Geometric, ratio = 3--2- ; t4 = 27
------
28
; tn = 2
---
7
×  2
8 x = 0.63 or 1
9 C 10 A 11 D 12 E h Geometric, ratio = 2; t4 = 6; tn = 3 × 2n − 3
13 a (0, 10
------ ) b y=1 i Not geometric
9
j Geometric, ratio = −6; t4 = −54; tn = 1--4- × (−6)n − 1
c y y = 3x – 2+ 1
k Geometric, ratio = 2π; t4 = 16π 4; tn = (2π)n
(0,10
—)
9 2 a tn = 5 × 2n − 1, t6 = 160, t10 = 2560
1
(2, 2) b tn = 2 × 2.5n − 1, t6 = 195.31, t10 = 7629.39
0 x c tn = 1 × (−3)n − 1, t6 = −243, t10 = −19 683
d tn = 2 × (−2)n − 1, t6 = −64, t10 = −1024
d dom = R ran = (1, ∞) e tn = 2.3 × (1.5)n − 1, t6 = 17.47, t10 = 88.42
14 C 15 B f tn = 1--2- × 2n − 1, t6 = 16, t10 = 256
( x + 1 )2  1--- n − 1, t
16 a −3 b y = ------------------- g tn = 1--3- ×  4 6 = 1
------------ , t10 = 1
-------------------
125
x 3072 786 432
17 a log 4 --------
- b 12
36 h tn = 3--5- ×  – 1--- n − 1, t 1
= − -------- 1
- , t10 = − ----------------
 3 6 405 32 805
18 C 19 A 20 D 21 B 22 E
23 C i tn = x × (3x3)n − 1, t6 = 243x16, t10 = 19 683x28
14
24 a x = 216 b x=5 c x= ------ n−1
1 2 32 512
j tn = --- ×  ---
3
25 0.861 , t6 = ------ , t10 = ---------
26 x = 0.366 x  x x
6
x
10

27 a 1500 3 a There are two possible answers because the ratio


b i 2800 ii 5200 could be −3 or 3. The nth term is tn = 2 × 3n − 1 or
c 14.36 days tn = 2 × (−3)n − 1, t10 = ±39 366.
759

answers
Answers

b There are two possible answers because the ratio ( 3 + 5) (4 3 + 3 5)


could be −2 or 2. The nth term is tn = 2n − 1 or 16 4 – 15 , ------------------------- = --------------------------------
( 15 – 3 ) 3
tn = (−2)n − 1, t10 = ±512.
c The nth term is tn = 5 × 2n − 1, t10 = 2560. 17 − 1--6-
d The nth term is tn = −1 × (−2)n − 1, t10 = 512.
1
e There are two possible answers because the ratio 18 ---
3
1 1 5 − 3n
- or ------ . The nth term is tn = 3
could be – ----- or 19 1, 0.99
27 27
5 − 3n −25 20 Check with your teacher.
tn = (−3) , t10 = ±3 .
4 ± 3--4- Exercise 8C — Growth and decay
functions
(n – 1) 1 a V = 50 + 10T i $200 ii 5 years
----------------
5 3×2 2 b P = 600 + 30T i 1320 ii 10 months
6 m = 12, n = 48 c A = 1200 + 120T i 2160 ii 5 years
7 m = 36, n = 729
--------- d V = 1560 + 78T i 2223 ii 5.64 years
4
2 Individual graphs to verify.
8 a = 300, b = 0.75
3 a Not exponential growth
9 t1 = 25, r = ±2, tn = 25 × 2n − 1 or tn = 25 × (−2)n − 1 b, c, d Exponential growth
10 t1 = 1--3- , r = 3--2- , tn = 3n − 221 − n 4 a N = 1000(2)T b N = 2000(1.4)T
T
11 −6 c N = 860(1.25) d N = 1250(1.5)T
T
e N = 2300(2)
12 2, 1--2- , 1--8- , or −2, 1--2- , − 1--8-
5 a V = 700(1.1)T b V = 1100(1.05)T
3 24 c V = 5000(1.16) T
d V = 2750(1.2)T
13 a --- b ------
2 n T
2 e V = 380(1.08)
14 k = 6 6 a C = 25 000(1.07)T b C = 1.5(1.03)T
T
c C = 250(1.05) d C = 29.95(1.06)T
Exercise 8B — Geometric series e C = 7.2(1.1) T
1 a 31, 1023, 1 048 575
b 121, 29 524, 1.74 × 109 7 a A = 2000(1.16)T b A = 850(1.12)T
T
c 33, −1023, −1 048 575 c A = 1900(1.06) d A = 25 000(1.09)T
d −4, 103.8, −746.8 T
e A = 12 600(1.15)
3
e --- , 46.5, 1534.5
2 8 a b c d
f − 2--3- , − 31
------
24
, − 341
---------
256 T A ($) T V ($) T N T C ($)
g 12 930, 70 972, 1 302 280
0 1500 0 850 0 400 0 17 000
h −309, 2239, −1865
i 15 620, 48 828 120 1 1620 1 901 1 408 1 18 700
j −1.375, −1.332
2 a 3108 1 15 15
- , 63 ------ , 66 535 ------
b ----- 2 1749.60 2 955.06 2 416.16 2 20 570
16 16 16
3 458 4 9 5 121 875 000 3 1889.57 3 1012.36 3 424.48 3 22 627
6 65 024 7 21 504
4 2040.73 4 1073.11 4 432.97 4 24 889.70
8 a 2 b 2--3- c 3--2- d 3 e 3
---
5
5 2203.99 5 1137.49 5 441.63 5 27 378.67
9 1.0; 50%, 25%, 12.5%
10 4--3- ; 75%, 18.75%, 4.6875% 6 2380.31 6 1205.74 6 450.46
11 4; 25%, 18.75%, 14.06% 7 2570.74 7 1278.09 7 459.47
12 a 6[1−  1--2- 20] = 5.999 994 278
a b
−6 2600 1300
b 5.722 × 10 2500 1250
c 6 2400 1200
2300 1150
13 a 6 3--4- [1 − (− 1--3- )9] = 6.750 343 2200 1100
V ($)

2100 1050
2000 1000
A ($)

b −3.425 × 10−4 1900


1800
950
900
1700 850
c 6 3--4- 1600
1500 0
1 2 3 4 5 6 7
14 6 2--3- 0 T (years)
15 16, 8, 4
1 2 3 4 5 6 7
T (years)
8A

8C
answers 760 Answers

c d c d
28000 300 1400
460 27000 1300
455 280
26000 260 1200

Seahorses (N)
450 25000
445 240 1100
24000 220 1000

C ($)
N (possums)

440 23000 900


200

A (grams)
435 22000 800
430 180
21000 160 700
425 20000
420 140 600
19000 120 500
415 18000
410 400
17000 300
405 0
400 1 2 3 4 5 6 7 8 200
0 100
1 2 3 4 5 T (months)
0
0 T (years) 1 2 3 4 5 6 7 8
1 2 3 4 5 6 7
T (months) T (minutes)

9 a 6605.40 b 738.75 22 a N = 1400(0.98)12T b 1098 cockatoos


c $6580.94 d $1560.24 23 a V = 899(0.7)T b $151.09
e 4 017 915 bacteria 24 C 25 D
10 a A = 2000(1.08)T b $3173.75 26 a 3000 b 7.5 years c 1361

Number of penguins
2800
11 a V = 12 000(1.04)T b $17 762.93 2600 Rookery B
2400
12 E 13 D 14 C 2200
2000
15 a N = 500 − 35T i 80 ii 7.14 years 1800
1600
b V = 12 000 − 800T i $7200 ii 4 years 1400 Rookery A
1200
c h = −5T + 300 i 225 cm ii 7 minutes 1000
d i 17 wombats/year ii 230 iii 12 years 0
1 2 3 4 5 6 7 8 9 10
e i $190/year ii $1170 iii 3 years T (years)
16 Individual graphs to verify.
17 a, b, d Exponential growth Exercise 8D — Compound interest
c Not exponential growth formula
18 a A = 300( --12- )T b V = 5000(0.75)T 1 a $583.20 b $1630.47 c $4472.27
c N = 2500(0.95) T
d m = 900(0.8)T d $3764.86 e $939.15 f $1369.50
e V = 850(0.92) T
f N = 15 000(0.75)T 2 a i $2519.42 ii $519.42 b i $8837.34 ii $1837.34
c i $6615.42 ii $615 d i $3059.97 ii $1159.97
19 B
3 a 5 b 20 c 8 d 72
20 a 365.40 b 4343.88
e 9
c 160.18 d 86.73
e 3.67 4 a 1.5% b 2% c 5.5%
d 1.5% e 1.75%
21 a b c d
5 a $3514.98 b $2687.83 c $8061.13
T m T V T N T A d $3431.89 e $3073.14
6 a $605.61 b $903.60 c $1314.83
0 840 0 2600 0 290 0 1350
d $353.44 e $795.76
1 638.4 1 2080 1 263.9 1 945 7 E 8 B 9 C
10 a I = $1760
2 485.18 2 1664 2 240.15 2 661.5 b I = $2071.83; Best option
c I = $2064.99
3 368.74 3 1331.2 3 218.54 3 463.05
11 a A = $12 975.98
4 280.24 4 1064.96 4 198.87 4 324.14 b A = $13 743.08
c A = $13 747.87; Best option
5 212.98 5 851.97 5 180.97 5 226.89 12 $764.08 13 $1880.88
6 161.87 6 681.57 6 164.68 6 158.83 14 a $3542.13 b $2052.54 c $2969.18
d $5000 e $3100
7 123.02 7 545.26 7 149.86 7 111.18 15 a $2069.61 b $1531.34 c $2010.82
8 93.49 8 436.21 8 136.37 8 77.82 d $3564.10 e $5307.05
16 a $930.39 b $468.66 c $889.18
a b d $2035.90 e $4692.95
900 3000 17 B 18 D 19 $1351.56 20 $10 292
800 2700
700 2400 21 a 8% b 6% c 12% d 7%
600 2100
m (g)

500 1800 e 7.8%


V ($)

400 1500
300 1200 22 a 13.98% b 5.93% c 5.62% d 6.84%
200 900
100 600 e 10.04%
0 300
1 2 3 4 5 6 7 8 0 23 19.03% 24 9.06% 25 8.34%, i.e. D
T (days) 1 2 3 4 5 6 7 8
T (years) 26 13.12%, i.e. B 27 Yes 28 Yes
761

answers
Answers

29 a i $2959.87 ii 12.36% p.a. 16 a i $103.22 ii $398.64


b i $2997.20 ii 12.08% p.a. b i $427.87 ii $73.99
30 a 19, 9 1--2- years b 7, 3 1--2- years 17 a i $264.10 ii $2384.84
c 20, 5 years d 24, 6 years b i $2901.10 ii $557.84
e 57, 4 3--4- years 18 a i $18.49 ii $449.82
31 a 13, 3 1--4- years b 14, 7 years b i $66.97 ii $401.34
c 32, 8 years d 13, 6 1--2- years c i $244.93 ii $223.38
e 35, 2 years 11 months 19 B 20 D 21 E 22 B
32 13, 3 1--4- years 23 a $50 356.80 b $50 179.46 c $522.14
33 5, 2 --12- years 34 53, 4 years 5 months
24 a $81 433.37 b $81 640.94 c $833.73
35 26, 6 1--2- years 36 B 37 D 38 D
39 a 4 years b $9718.11 Exercise 8F — The annuities formula
1 a $41 475.93 b $18 419.91
c $51 475.93 d $28 419.91
Exercise 8E — Loan schedules
e $8670.97 f $27 815.32
1 a $204 b $30.20 g $52 435.82 h $71 419.61
2 a $306.61 b $49.85 2 a $59 633.49 b $49 884.16
3 a $514.94 b $101 c $32 172.59 d $15 200.98
4 a $189.01 b $68.60 3 a $70 570.81 b $63 313.07
5 a $783.58 b $228.32 c $51 420.42 d $31 932.92
6 a $666.85 b $287.44 4 a $46 102.98 b $36 196.88
7 a i $18 750.82 ii $1249.18 iii $975.27 c $19 556.12 d $7381.52
b i $16 048.36 ii $3951.64 iii $2769.91 5 a $27 564.36 b $29 291.80
8 a i $29 714.91 ii $285.09 iii $747.16 c $30 958.81 d $32 551.72
b i $27 994.62 ii $2005.38 iii $4748.82 6 a $5691.20 b $9087.53
9 a i $17 631.65 ii $2368.35 iii $953.10 c $11 350.93 d $12 930.79
b i $19 362.07 ii $637.93 iii $987.37 e $14 115.16
c i $19 556.52 ii $443.48 iii $991.22 7 a $6310.40 b $10 774.07
d i Decreases ii Increases iii Increases c $14 084.40 d $16 626.10
10 e $18 630.33
a i $26 725.05 ii $3274.95 iii $3590.30
8 a C b B
b i $27 660.46 ii $2339.54 iii $3635.91
9 a A b D
c i $28 264.47 ii $1735.53 iii $3665.37
10 D
d i Decreases ii Increases iii Increases
11 a $444.24 b $409.53
11 a i $49 493.44 ii $506.56 iii $1593.96 c $403.54 d $323.81
b i $49 647.28 ii $352.72 iii $1595.80 e $643.45 f $944.82
c i $49 836.73 ii $163.27 iii $1598.05 g $2594.63 h $345.85
d i $49 637.19 ii $362.81 iii $2194.07 12 a i $548.22 ii $1157.28
e i $49 767.60 ii $232.40 iii $2196.20 b i $381.60 ii $1737.60
f i $49 912.73 ii $87.27 iii $2198.57 c i $298.62 ii $2333.76
12 a i $59 476.65 ii $523.35 iii $1196.09 d i $271.07 ii $2637.78
b i $59 546.30 ii $453.70 iii $1435.94 e i $231.18 ii $3257.88
c i $59 608.31 ii $391.69 iii $1675.91 13 a i $986.92 ii $33 430.40
d i $59 663.19 ii $336.81 iii $1915.99 b i $874.12 ii $40 873.28
e i $59 733.31 ii $266.69 iii $2276.23 c i $764 ii $52 520
f i $59 772.77 ii $227.23 iii $2516.45 d i $693.18 ii $64 726.88
13 a i $58 392.68 ii $1607.32 iii $3564.28 e i $659 ii $73 160
b i $58 598.65 ii $1401.35 iii $4282.73 f i $600.76 ii $95 228
c i $58 782.84 ii $1217.16 iii $5002.32 14 a Decreases b Increases
d i $58 946.59 ii $1053.41 iii $5722.83 15 a i $1069.67 ii $10 786.80
e i $59 157.26 ii $842.74 iii $6804.82 b i $1169.78 ii $14 791.20
f i $59 276.58 ii $723.42 iii $7526.70 c i $1274.76 ii $18 990.40
14 a Increases b Increases c Increases d i $1301.74 ii $20 069.60
15 a i $262.27 ii $371.11 e i $1329.01 ii $21 160.40
b i $529.40 ii $103.98 f i $1412.51 ii $24 500.40
8D

8F
answers 762 Answers

16 a i $180.95 ii $24 070.50 4 a Pop A Pop B


b i $204.91 ii $33 414.90 Growth Annual
c i $230.40 ii $43 356 rate increment
Year n = 1.12 = 1000 Difference
d i $237.01 ii $45 933.90
1990 1 10 000 15 000 5000
e i $243.70 ii $48 543 1991 2 11 200 16 000 4800
f i $264.27 ii $56 565.30 1992 3 12544 17 000 4456
1993 4 14 049 18 000 3951
17 a i $253.92 ii $170 076.80 1994 5 15 735 19 000 3265
b i $507.97 ii $170 144.40 1995 6 17 623 20 000 2377
1996 7 19 738 21 000 1262
c i $1101.28 ii $170 307.20 1997 8 22 107 22 000 -107
d i $3311.33 ii $170 906.40 1998 9 24 760 23 000 -1760
1999 10 27 731 24 000 -3731
e i $161.57 ii $74 032.80 2000 11 31 058 25 000 -6058
f i $323.23 ii $74 079.60 2001 12 34 785 26 000 -8785

18 a Increases b Increases b During 1997


19 A, D c Annual increment of 1606 insects
20 D d Annual growth rate of 1.095 96
21 a $365.84 5 a ii 96, 115.2, 138.24 ii 14 months
b i $259.59, $106.25 ii $339.75, $26.09 b ii Straight line
ii Increase is a constant 1.25 hectares.
22 a $464.49 c ii $5378.24 ii 2 hectares
b i $134.03, $330.46 ii $365.64, $98.85 d i $14 880 — simple; $15 288.28 — compound
23 a $323.73 ii Simple. Less interest is charged compared to
b i $46.23, $277.50 ii $299.28, $24.45 compound interest calculated on a monthly
24 $38 231.10 increasing loan amount
25 $6387.82 iii $406.44
26 a $272 454.72 b $261 893.41 c $23 096.54 6 $2540.86
27 a $193 241.40 b $183 740.59 c $18 707.47 7 $2526.38

CHAPTER 9 Presentation of data


Chapter review
1 E Exercise 9A — Types of variables and
2 a t1 = 1.4; t5 = 7.0875 data
b S10 = 158.66 1 a Quantitative b Categorical c Quantitative
c 33.9% d Categorical e Quantitative f Categorical
3 a 134.2 milli-rem 2 a Ordinal b Ordinal c Nominal
b 3361.6 milli-rem d Nominal e Nominal
4 D 5 C 3 a Continuous b Discrete c Continuous
6 89/99 d Continuous e Discrete
7 a 4/3 b 3/5 4 a Quantitative, discrete b Categorical, nominal
8 B 9 D c Quantitative, continuous d Quantitative, continuous
10 $3.09 e Categorical, nominal f Quantitative, continuous
11 A 12 D 13 C 14 D g Quantitative, discrete h Quantitative, continuous
15 C i Quantitative, discrete j Categorical, nominal
16 $3482.46 k Quantitative, continuous l Quantitative, discrete
17 $411.58 m Quantitative, discrete n Categorical, nominal
18 8 years 1 month o Categorical, ordinal p Quantitative, continuous
19 B q Quantitative, discrete r Categorical, ordinal
20 a i $149 900 ii $149 799 iii $149 696.99 s Categorical, nominal t Categorical, ordinal
b $4496.99 5 a Quantitative and discrete
21 A 22 C 23 B b Categorical
24 $60 205.40 c Categorical
25 $46 741.44 d Quantitative and continuous
e Quantitative and continuous
f Quantitative and continuous
Modelling and problem solving 6 Categorical and ordinal
1 a i 0.6%, 1.1%, 1.0%, 0.7% ii 14% 7 Categorical and ordinal
b $2592 8 Quantitative and discrete
c $406 9 C
2 a $186, $2470 b $1206, $2467 10 Categorical and ordinal
3 a $144.12 b $8541.60 11 Quantitative and continuous
763

answers
Answers

Exercise 9B — Collection of data Exercise 9D — Stem plots


1 Census — every member of the population 1 a 1 2 5 8 12 13 13 16 16 17
participates. 21 23 24 25 25 26 27 30 32
2 Sample b 10 11 23 23 30 35 39 41 42 47
3 a Sample b Sample c Census 55 62
d Census e Sample c 101 102 115 118 122 123 123 136
4 a Sample b Census c Census 136 137 141 143 144 155 155 156
d Sample 157
5 Sample d 50 51 53 53 54 55 55 56 56 57
6 163, 176, 381, 495, 9 59
7 827, 211, 417, 554, 207, 26, 810, 781, 192, 371 e 1 4 5 8 10 12 16 19 19 21 21
8 Check with your teacher. 25 29
9 a Systematic b Stratified c Systematic 2 Stem Leaf
d Random e Stratified 0 5
10 A 1 1 8 9
11 C 2 3 7 9
12 Year 8 — 11, Year 9 — 9, Year 10 — 8, 3 1 2 5 6 7 9
Year 11 — 7, Year 12 — 5 4 1 2 3 5
13 36 men and 24 women 5 2 Key: 0|5 = $5
14 Age Male Female 3 Stem Leaf
3 7 9
20–29 10 7 4 2 9 9
30–39 7 8 5 1 1 2 3 7 8 9
6 1 3 3 8 Key: 3|7 = 37 yrs
40–49 12 3
4 Stem Leaf
50–59 1 2 0 8 9
1 6 8 9
2 1 3 6 6 7 8 9
Exercise 9C — Bias 3 0 0 2 5
1 Check with your teacher. 4 1 3 5
2 Check with your teacher. 5 4 Key: 0|8 = 8 dogs
3 a Sample does not represent characteristics of
5 C
population
b No control over responses 6 Stem Leaf
c Unrepresentative sample 2 3
d Abnormal conditions 2 5 8 8 9 9 9
e Only extreme groups in sample 3 0 0 2 2 2 3 3 4
4 The decrease in the value of the Australian dollar 3 5 5 7 8 9 Key: 2|3 = 23 yrs
compared with the American dollar is accentuated by 7 Stem Leaf
the large scale on the vertical axis. The decrease is 1 1
actually only 2 cents. The scale on the horizontal axis 1 3
is not uniform (9 May, 11 May, 12 May). 1 4 5
5 What type of university tests? What do the terms 1 6 7
‘consistently’, ‘majority’, ‘more effective’, ‘most 1 8 8 8 9 9 9 9
other’ mean? No hard evidence has been provided to 2 0 0 1 1 1
support the claim. 2 2 2 3 3 3
6 a There would be many more student drivers in 2 5
Year 12 than in Year 11 — perhaps also some in 2
Year 10. 2 9 Key: 1|1 = 11 people
b Students with part-time jobs are in lower year 8 Stem Leaf
levels as well. 2 0 1 2 4
c Residents not at the neighbourhood watch meeting 2 5 6 8 9 9 9
have been ignored. 3 1 1 2 3 4 4
d Other music students who play instruments and 3 Key: 2|0 = 20 hit outs
don’t belong to the choir have been excluded. 9 Stem Leaf
e The composition of cars in a shopping centre car 18 5 7 9
park is not representative of the cars on the road. 19 1 5 6 6 7 9
f Females have been excluded. 20 1 3 3 5 9
g Users of the local library would not reflect the 21 7
views of teenagers. 22 1 Key: 18|5 = 1.85 m 9A

9D
answers 764 Answers

10 Stem Leaf c Stem Leaf


4 2 9 9 8 1
5 0 0 0 2 7 8 8 2
6 3 8 9 8 5
7 8
8 0 3 3 5 8 9 9 9
9 9 0 1
10 6 9 2 3
11 5 Key: 5|2 = $520 000 9 5
9 7
11 a Stem Leaf 9
4 3 7 7 8 8 9 9 9 Key: 8|1 = 8.1 seconds
5 0 0 0 0 1 2 2 3 Key: 4|3 = 43 cm
14
b Stem Leaf Boys Girls
4 3 9 9 7 1.51 2 5 6 7 8 8
4 7 7 8 8 9 9 9 9 8 6 6 5 5 4 0 1.64 4 6 7 8 9 9
5 0 0 0 0 1 2 2 3 4 4 2 1 1.70
5 Key: 4|3 = 43 cm Key: 1.5|5 = 1.55
c Stem Leaf 15
4 Team 1 Team 2
4 3
8 0 79
4
987 1
4 7 7
9543 2 01347
4 8 8 9 9 9
42 3 058
5 0 0 0 0 1
20 4 16
5 2 2 3
Key: 1|8 = 18
5
5
5 Key: 4|3 = 43 cm
Exercise 9E — Frequency histograms
12 a Stem Leaf and bar charts
1 5 6 7 7 7 8 9 9 9 9 Score Frequency Score Frequency
1 a
2 0 0 0 1 1 1 2 3 3 3 3 1 8 2
Key: 1|5 = 15 mm 4 2 9 2
b Stem Leaf 5 2 10 2
1 6 1 11
7 3 12 1
1 5 6 7 7 7 8 9 9 9 9
2 0 0 0 1 1 1 2 3 3 3 Class Frequency Class Frequency
Key: 1|5 = 15 mm b
1–1.9 1 4–4.9 6
c Stem Leaf 2–2.9 2 5–5.9 5
1 3–3.9 2 6–6.9 1
1
1 5 Class Class
1 6 7 7 7 c
interval Frequency interval Frequency
1 8 9 9 9 9
10–14 3 25–29 10
2 0 0 0 1 1 1
15–19 9 30–34 10
2 2 3 3 3 20–24 10 35–39 1
2
2 Score Frequency Score Frequency
2 Key: 1|5 = 15 mm d
0.3 1 0.9 2
0.4 2 1.0 2
13 a Stem Leaf 0.5 1 1.1 1
8 1 2 5 9 9 9 0.6 1 1.2 1
9 0 1 2 3 5 7 0.7 1 1.3 1
Key: 8|1 = 8.1 seconds 0.8 2
b Stem Leaf 2 a
Frequency

8 1 2 3
8 5 9 9 9 2
9 0 1 2 3 1
9 5 7
3 4 5 6 7 8 9 10 11 12
Key: 8|1 = 8.1 seconds Score
765

answers
Answers

b c b
6 10
10

Frequency
5 8
Frequency

Frequency
4 6 8
3 4 6
2 2 4
1 2
10 15 20 25 30 35 40
1 2 3 4 5 6 7 Score

6
8
0
2
4
6
8
0
2
0.
0.
1.
1.
1.
1.
1.
2.
2.
Score Height (m)
d
Frequency

2 8 a
Class Class centre Frequency
1
10.5–10.9 10.7 2
0.2 0.4 0.6 0.8 1.0 1.2 1.4
Score
11.0–11.4 11.2 5
3 Check your histograms against those shown in
question 2 answers. 11.5–11.9 11.7 8
4 12.0–12.4 12.2 8
Frequency

3
2
1 12.5–12.9 12.7 5

3 4 5 6 7 8 9 10 11 12 13.0–13.4 13.2 2
Score
5 b
Number of students

8 10
Frequency

6 8
4 6
2 4
2
0 1 2 3 4 5 6 7 8 9 10
Number of hours
.5

11 0
12 5
12 0
13 5
13 0
.5
.
.
.
.
.
10
11

Time (s)
6 a Score Class centre Tally Frequency
9
30–39 34.5 | 1
40–49 44.5 |||| 5
NSW 31% Vic. 21% Qld 21% SA 7%
50–59 54.5 |||| 5
WA 13% Tas. 3% NT 3% ACT 1%
60–69 64.5 |||| ||| 8
70–79 74.5 |||| | 6
Exercise 9F — Describing the shape of
80–89 84.5 ||| 3
stem plots and histograms
90–99 84.5 || 2 1 a Symmetric, no outliers
b b Negatively skewed, no outliers
c Positively skewed, one outlier
10 d Symmetric, no outliers
Frequency

8 e Symmetric, one outlier


6 f Positively skewed, no outlier
4
2 a Symmetric, no outliers
2
b Symmetric, one outlier
30 40 50 60 70 80 90 100 c Symmetric, no outliers
Score d Negatively skewed, no outliers
e Negatively skewed, one outlier
7 a Class Class centre Tally Frequency f Positively skewed, no outlier
0.6–<0.8 0.7 || 2 3 E 4 D
5 Negatively skewed, no outliers
0.8–<1.0 0.9 ||| 3 6 Positively skewed. This tells us that most of the
1.0–<1.2 1.1 ||| 3 flight attendants in this group spend a similar
1.2–<1.4 1.3 |||| | 6 number of nights interstate per month. A few stay
away more than this and a very few stay away a lot
1.4–<1.6 1.5 |||| |||| 9 more.
1.6–<1.8 1.7 |||| | 6 7 a Symmetric, no outliers
1.8–<2.0 1.9 |||| 4
b This tells us that there are few low-weight dogs
and few heavy dogs but most dogs have a weight
2.0–<2.2 2.1 |||| 4 in the teens (in kg). 9E

9F
answers 766 Answers

8 a Symmetric, one outlier b Ogive of time c 14

Cumulative frequency (%)


Cumulative frequency
b Most students receive about $8 (give or take $2). 20 trial results 100% d 90% of the
One student, however, is far removed from the rest 15 riders finished
and receives only $2. with a time of
10 50%
9 a Positively skewed, 2 outliers. This indicates that 90 s or less.
most workers do up to 3 hours of exercise per 5 e To qualify you
week. Very few do more. 0 need a time
b The company has 2 fitness fanatics; one does 65 70 75 80 85 90 95 under 72 s.
14 hours exercise a week and one does 19 hours Time (s)
exercise a week.
7 a Cumulative frequency column: 2, 11, 18, 27,
29, 30
Exercise 9G — Cumulative data b Ogive of baby weights c 75%

Cumulative frequency (%)


Cumulative frequency
1 a Cumulative frequency b Cumulative frequency 30 100% d 3.45 kg
80 Ogive of lengths 25 e 3%
column: 9, 24, 44, 56, 70 of flathead
60
20
64, 71, 75, 77 15 50%
50
40 10
30
20 5
10
2.4 3.2 4.0 4.8
300 320 340 360 380 Weight (kg)
Length (mm)
2 a Cumulative frequency column: 1, 5, 20, 38, 50, 8 a Cumulative frequency column: 9, 19, 25, 29, 30
58, 60 b Temperature at c Ogive of temperature
b which paint blistered at which paint blistered

Cumulative frequency (%)


Ogive of task times

Cumulative frequency
10
Cumulative frequency

Frequency
60 30 100%

40 5 20
50%
10
20
80 90 100
Temperature (°C) 80 90 100
6 10 14 18 Temperature (°C)
Time (s) d 19 e 63% f 26% g 93° h 78°
3 a Cumulative frequency column: 6, 18, 26, 33, 38, 9 D 10 C
39, 40 11 a Cumulative frequency column: 3, 12, 17, 19, 20
b Ogive of salaries c 11
Cumulative frequency (%)

b c 14
Cumulative frequency

Ogive of number
40 100% d 26
Cumulative frequency (%)

35 of passengers d 97 passengers
e $33 000
Cumulative frequency

30 20 100%
25 f $26 500 e 89 passengers
20 50%
g $21 500
15 f 81 passengers
15
10 h $38 000 10 50% g No
5
5
20 30 40 50
Salary ($×1000) 70 90 110
Number of passengers
4 a Cumulative frequency column: 13, 41, 87, 117,
134, 142, 149, 150 Chapter review
b Ogive of waist size c 60 1 a Categorical b Quantitative c Quantitative
150 100% d 16 d Quantitative e Categorical
Cumulative frequency (%)

2 a Discrete b Continuous c Continuous


Cumulative frequency

125
e 96 cm
f 84 cm d Discrete e Continuous
100
g 25% 3 a Sample b Census c Census
75 50% d Sample
50 4 Random sample — where the participants are
25 chosen by luck
Stratified sample — where the participants are
70 80 90 100 110 chosen in proportion to the entire population
Size (cm) Systematic sample — where a system is used to
select the participants
5 a 12 b 11 c 68% d 8%
5 a Systematic b Random c Stratified
e 55 f 46 g 37 h 30 or less
6 Check with your teacher.
6 a Cumulative frequency column: 2, 7, 13, 17, 7 Year 8 — 15, Year 9 — 14, Year 10 — 13,
18, 20 Year 11 — 10, Year 12 — 9
767

answers
Answers

8 Check with your teacher. h Half the people surveyed were against LOTE
9 a Stem Leaf b Stem Leaf being a compulsory subject in years 7 to 10.
6 0 1 5 7 6 0 1 4 a Physics test results
7 1 1 3 8 8 9 6 5 7
8 2 3 4 4 7 8
7 1 1 3

Frequency
7 8 8 9 6
Key: 6|0 = $60
8 2 3 4 4 4
c Stem Leaf
8 7 2
6 0 1
6 Key: 6|0 = $60
10 30 50 70 90
6 5 Result
6 7 b

Cumulative frequency (%)


Ogive of Physics
6 test results

Cumulative frequency
7 1 1 100%
30
7 3 25
7 20
7 15 50%
10
7 8 8 9 5
8 20 40 60 80 100
8 2 3 Result
8 4 4 c The 25th percentile is 53. This means that 25% of
8 7 students received a mark of 53 or lower.
8 Key: 6|0 = $60 The 50th percentile is 62. This means that 50%
10 of students received a mark of 62 or lower.
10 The 75th percentile is 72. This means that 75%
Frequency

8 of students received a mark of 72 or lower.


6 d 90
4
2
e From the histogram, it can be seen that the most
common marks received ranged from 50 to 70.
50 52 54 56 58 60 62 64 66 68 70 72 74 76 From the ogive, the median mark is 62, with 50%
Speed (km/h) of students receiving a mark between 53 and 72.
11 Positively skewed, one outlier 5 a Designer Tiles: negatively skewed; Nathan’s
12 a 17 b 5 c 72% Tiles: positively skewed.
d $88; 80% of people spent $88 or less on shopping b Frequency column:
e $20 Designer Tiles: 2, 2, 5, 8, 10, 10, 12, 9
Nathan’s Tiles: 6, 11, 13, 12, 8, 6, 1, 1
Modelling and problem solving c Measures of central tendency are all higher for
1 a Quantitative and continuous Designer Tiles. Spread is almost the same with
b Systematic standard deviation for Designer Tiles slightly
c The data are influenced by factors that don’t make higher. As an overall trend, people tend to buy
them representative of the whole population. more expensive tiles in Designer Tiles.
d Take cans of paint from each of the five machines. d Increase the range of cheaper ceramic tiles in
2 a Stem Leaf Nathan’s Tiles and move most of the expensive
0 8 9 porcelain stock to Designer Tiles.
1 2 3 4 7
2 1 2 2 3 5 7 9
3 0 1 4 5 8 CHAPTER 10 Summary statistics
4 3 5 6 7
5 1 3 5 Exercise 10A — Measures of central
6 4 6 tendency
7 6 1 a 7.2 b 7.125 c 4.9875
Key: 0|8 = $8 .
d 16.7̇ e 0.8818
b Positively skewed
2 a 1.0783 No, because of the outlier.
c There would need to be a shift of some of the
b 17 Yes
amounts in the twenties to the thirties and forties.
c 30.875 Yes
3 a 30 b Categorical c Ordinal
d 15.57 No, because of the outlier.
d
3 12
4 D
Strongly disagree 20% Agree 17%
Disagree 30% Strongly agree 6% 5 A
Not sure 27% 6 a Median b Mean
e 16 --23- % f 26 --23- % g ‘Strongly disagree’ c Median d Median 9G

10A
answers 768 Answers

7 a 36.09 b 16.63 2
Range
c 168.25 d 18.55
8 24 a 7
b 9
9 Median c 6
d 13
a 37 e 14
b 5 f 7
c 11 g 17
d 42.5 h 9
e 628 i 21

10 3 a 10 b 8
Median 4 An example could be 2 3 6 8 9.
5 An example could be 2 5 6 7 8 10 11 12.
a 6
6 C
b 17
c 6 7 Interquartile
d 10 Median range Range
e 18.5
f 4 a 21 18 45
g 19 b 27.5 8 20
h 4.5 c 3.7 3 5.9
i 23
8 Interquartile
11 a 17 b 148, 151 c No mode
d 72 e 2.6 Median range Range
12 a 4 b 8 c 42, 44 a 42 21 91
13 a 17–20 b 22–28 b 32 7 30
14 a Class Cumulative 9 a 1.5 b 0.7 c 2.2
Class centre Frequency frequency d 1.5
0–14 7.5 1 1 10 a Cumulative
15–29 22.5 13 14 Score Frequency frequency
0 26 26
30–44 37.5 2 16
1 31 57
45–59 52.5 0 16 2 22 79
60–74 67.5 4 20 3 8 87
75–89 82.5 5 25 4 3 90
b
Cumulative frequency

90
b 42.3 c 15–29 d 15–29 80
70
e No 60
50
f Check with your teacher. 40
30
15 a Player A: 34.3 Player B: 41.8 20
10
b Player B 0
0 1 2 3 4
c Player A: 32.5 Player B: 0 Score
d Player A c 1.3 d 1.6
e Check with your teacher.
Exercise 10C — The standard deviation
1 a 2.288 b 2.195 c 20.17 d 3.066
Exercise 10B — Range and 2 As for question 1.
interquartile range 3 a Group A: mean = median = mode = 170 cm
Group B: mean = median = mode = 170 cm
1 Range b No c Group B d Group B e Group B
f Group A: range = 20, interquartile range = 0,
a 56 σ = 5.345
b 17 Group B: range = 120, interquartile range = 20,
c 18 σ = 32.51
d 18 4 a 60 b 110.48 c σ = 11.59
e 72 5 C
769

answers
Answers

6 a 500 hours b Mean = 455.29, σ = 88.62 14 a Key: 1* 7 = 17 years


c Mean = 455.29, s = 88.88 Stem Leaf
7 a Cumulative frequency column: 4, 26, 121, 245, 1* 7 7 8 8 8 9 9
339, 358, 363, 365 2 0 0 0 1 2 2 2 2 3 3 3 3 4 4 4
b Ogive of c Q3 = 21, Q1 = 13.5, 2* 5 5 8 9
3 1 2 3

Cumulative frequency (%)


temperatures interquartile range
3*
Cumulative frequency

400 = 7.5
100%
d 17 4
300 75% 4* 8
e 17.55
200 50% f 5.649 b 15 25 35 45 Age
100 25% g 40 ×
c Data are positively skewed with one high extreme
0 10 20 30 40 value. Most mothers have their first baby between
Temperature
the ages of 20 and 25.
8 a Frequency column: 4, 9, 9, 3, 4, 1
15 C
b Ogive of number c Median = 52,
Cumulative frequency (%)

of fruit on each tree Q3 = 62, Exercise 10E — Back-to-back stem plots


Cumulative frequency

100%
Q1 = 45
30 1 Key: 2 3 = 23
25 d 15
20 e 54 German   French
15 50%
f 13.25 2 1 1 0 2 3 4
10
g 13.48 7 6 5 5 2 5 5 8
5
h 45 3 2 1 0 0 3 0 1 4 4
30 50 70 90 9 8 7 7 3 5 6 8 8 9
Number of fruit
9 a Electric Mate 2 1 4 2 3 4 4
b Electric Mate = 197 hours, Hot Wire = 185 hours 5 4 6 8
c Electric Mate 2 Key: 2 7 = 2.7 (kg)
10 Check with your teacher. Boys   Girls
Exercise 10D — Boxplots 2 6 7
1 a 13 b 5 c 26 4 4 3 0 1 1 2 3
8 7 6 3 6 7
2 a 122 b 6 c 27
3 2 4 0
3 a 49.0 b 5.8 c 18.6 9 8 4
4 a 140 b 56 c 90 0 5
d 84 e 26
3 a Key: 2 3 trucks
5 a 58 b 31 c 43
A   B
d 27 e 7
2 1 1 0
6 C 7 C 8 D 7 7 6 6 5 1 5 6
9 a (22, 28, 35, 43, 48) b 20 30 40 50 Sales 4 3 2 1 0 2 0 1 3
7 5 2 5 6 8 9
10 a (10, 13.5, 22, 33.5, 45)
b 0 10 20 30 40 50 Rainfall (mm)
3 0 1 2
3 5
b For supermarket A the mean is 19, the median is
11 a (18, 20, 26, 43.5, 74) b 10 30 50 70 Age
18.5, the standard deviation is 4.9 and the
interquartile range is 7. The distribution is
c The data are positively skewed, and include one symmetric.
extremely high value. For supermarket B the mean is 24.4, the median is
12 a (324 000, 335 000, 348 000, 357 000, 375 000) 25.5, the standard deviation is 7.2 and the
b 320 340 360 380 ($×1000) interquartile range is 10. The distribution is
symmetric.
The centre and spread of the distribution of
13 a Key: 12 1 = 121 supermarket B is higher than that of supermarket A.
Stem Leaf
There is greater variation in the number of trucks
12 1 5 6 9
arriving at supermarket B.
13 1 2 4
14 3 4 8 8 4 a Key: 1 7 = 17 marks
15 0 2 2 2 5 7 Females   Males
16 3 5 1 0
17 2 9 3 2 1 2 3
18 1 1 1 2 3 7 8 5 5 4 4 1 4 4 5
b 120 140 160 180 Number sold 7 6 1 7
1 9
10B

10E
answers 770 Answers

b For the marks of the females, the mean is 14.5, the b For the distribution of scores of the kindergarten
median is 14.5, the standard deviation is 1.6 and the children, the mean is 28.9, the median is 30, the
interquartile range is 2. The distribution is symmetric. standard deviation is 15.4 and the interquartile
For the marks of the males, the mean is 14.25, the range is 27.
median is 14, the standard deviation is 2.8 and the For the distribution of scores for the prep.
interquartile range is 3.5. The distribution is children, the mean is 29.5, the median is 29.5,
symmetric. the standard deviation is 15.3 and the
The centre of each distribution is about the same. interquartile range is 27.
The spread of marks for the boys is greater, The distributions are very similar. There is not a
however. This means that there is a wider variation lot of difference between the way the
in the abilities of the boys compared to the abilities kindergarten children and the prep. children
of the girls. scored.
5 a Key: 2 6 = 26 marks 8 B
First   Second 9 C
year   year
2 2 Exercise 10F — Parallel boxplots
2 6 7 8 1 a 11A
1 0 3 0 1 1 3 4
10A
9 7 5 3 6
3 2 1 1 4 9A
6 4
120 130 140 150 160 170 180 190 200
b The distribution of marks for the first year and for
Height (cm)
the second year are each symmetric.
For the first year marks, the mean is 38.5, the median b Clearly, the median height increases from Year
is 40, the standard deviation is 5.2 and the 9 to Year 11. There is greater variation in 9A’s
interquartile range is 7. The distribution is distribution than in 10A’s. There is a wide
symmetric. range of heights in the lower 25% of the
For the second year marks, the mean is 29.8, the distribution of 9A’s distribution. There is a
median is 30.5, the standard deviation is 4.2 and the greater variation in 11A’s distribution than in
interquartile range is 6. 10A’s, with a wide range of heights in the top
The spread of each of the distributions is much the 25% of the 11A distribution.
same but the centre of each is quite different with the 2 a
centre of the second year distribution significantly 40-49 age group
lower. The work may have become a lot harder!
30-39 age group
6 a Key: 2 5 = 25 years
20-29 age group
Female   Male
4 3 2 2 0 5 10 15
8 7 6 5 2 5
Annual superannuation contribution (× $1000)
1 0 3 0 1
3 6 7 b Clearly, there is a great jump in contributions to
4 2 superannuation for people in their 40s. The
4 6 spread of contributions for that age group is
b For the distribution of the females, the mean is smaller than for people in their 20s or 30s,
26.75, the median is 26.5, the standard deviation is suggesting that a high proportion of people in
2.8 and the interquartile range is 4.5. their 40s are conscious of superannuation. For
For the distribution of the males, the mean is 33.6, people in their 20s and 30s, the range is greater,
the median is 33.5, the standard deviation is 8.2 and indicating a range of interest in contributing to
the interquartile range is 12. super.
The centre of the distributions is very different: it is 3 a A
much higher for the males. The spread of the ages
B
of the females who attend the fitness class is very
small but very large for males. C

7 a Key: 3 2 = 32 Multi-vitamin
Kindergarten   Prep. 5 10 15 20
3 0 5
4 3 1 2 7 Number of jars sold
8 5 2 5 7 b Overall, the biggest sales were of multi-vitamins,
6 2 3 2 5 followed by vitamin B, then C and finally
7 1 4 4 6 vitamin A.
0 5 2 4 a C b B c E d C
771

answers
Answers

Chapter review 19 Team C


1 a 5.2 b 64.875 c 7.7 d 35.8 Team B

2 Class Team A
Class centre Frequency
90 100 110 120 130 140 150
21–24 22.5 3 IQ
25–28 26.5 9 Modelling and problem solving
29–32 30.5 17
1 a Office workers: negatively skewed with outlier
33–36 34.5 31 Sports instructors: positively skewed
37–40 38.5 29 b
Sports
41–44 42.5 25 Office workers instructors
45–48 46.5 19
Median 121.5 beats/min 73 beats/min
49–52 50.5 10 IQR 19.5 beats/min 14 beats/min
Σ f = 143 Range 58 beats/min 46 beats/min
Mean = 38.2 Mode 130 beats/min 68, 72 beats/min
3 a 31.1 b 23.2 c 0.445 c Office workers
4 a 29.9 b 26.4 c 18.6
5 a 27 b 6 c 3.2
d 5.5 e 128 70 80 90 100 110 120 130 140
6 a 2 b 56 c 68.5 Beats per minute
7 46–49 Sports instructors
8 a 27.8 b 24.5 c 28
d Median
9 Check with your teacher. 60 70 80 90 100 110
10 a 7 b 159 c 1.4 Beats per minute
11 a i 25 ii 24 iii 27.5 iv 3.5
d
b i 62.5 ii 43 iii 84 iv 41 Office workers Sports instructors
c i 1.1 ii 0.7 iii 1.5 iv 0.8
12 a 1.6 b Lower = 1, upper = 2.5 x = 116.8 beats/min x = 76.9 beats/min
c 1.5 σ = 15.3 beats/min σ = 12.4 beats/min
13 a 61 b Lower = 54, upper = 72 e Office workers: Pulse rates are generally very high,
c 18 clustered around 120–130 beats/min. Also, there is one
14 a 4.9 b 5.5 c 2 and 6 person whose rate was much lower than the rest. This
d 8 e 5 f 2.625 outlier (76) produces a large range and makes the mean
15 a 65.281 b 60–69 c 70–79 slightly lower than the median. As a result the median
d 59 e 13.728 is a more appropriate measure of the centre of the data
16 a 43 b 43 c 14 rather than the mean.
17 Sports instructors: Pulse rates are generally low,
0 2 4 6 8 1012 14 16 18 20 clustered around 60–70 beats/min, although there are a
few people with rates much higher, which makes the
18 a Key: 1 3 = 13 hours mean slightly higher than the median and also
Full-time   Volunteer produces quite a large range. As a result of the skewed
1 0 distribution the median is the more appropriate
2 2 0 measure of the centre of the data rather than the mean,
4 4 3 3 0 although there is little difference between these values.
6 5 0 2 a Key: 1 3 = 13
0 8 Text B Text A
1 0 1 1 Leaf Stem Leaf
1 2 3 3 1 3
1 4 5 2 1 2 5 8
1 9 7 3 5 5
1 8 2 4 4 8
b Both distributions are symmetric with the same 9 9 8 8 5 5 3 2 5 2 5 6 9
spread. The centre of the volunteers’ distribution is 8 8 8 6 5 4 4 3 2 6 4 9
much higher than that of the full-time firefighters’ 9 9 5 2 2 7 2 2 6
distribution. Clearly, the volunteers needed more 2 1 8 1 3 4 4
counselling. 9 4 5 8

10F
answers 772 Answers

b Text A: (13, 35, 59, 82, 98); CHAPTER 11 Introduction to


Text B: (37, 55, 63.5, 70, 82) probability
c 0 20 40 60 80 100 Result Exercise 11A — Informal description of
Text A chance
Text B 1 a Probable b Unlikely c Impossible
d to f Check with your teacher. d Fifty-fifty
2 a Impossible b Certain c Even chance
3 a Key: 1 6 = 16 d Even chance e Probable f Unlikely
Leaf Stem Leaf g Impossible h Even chance
Jazz concert Symphony concert 3 Check with your teacher.
9986 1 4 More likely during school term
97430 2 03 5 a More likely b Equally likely c Less likely
98430 3 059 d More likely e Less likely
65320 4 25578899 6 Rolling a 6
Rolling a number less than 3
5 034688
Rolling an even number
2 6 0 Rolling a number greater than 2
b 7 Winning a raffle with 5 tickets out of 30
Symphony Jazz Selecting a court card from a standard deck
concert concert Drawing a green marble from a bag containing 4 red,
5 green and 7 blue marbles
i minX 20 16 Rolling a die and getting a number less than 3
Tossing a coin and having it land Heads
ii Q1 40.5 21.5 8 Australia
9 Carl Bailey because he has better past performances.
iii Median 48 31.5 10 B 11 B 12 D
13 Probable 14 Unlikely 15 Fifty-fifty
iv Q3 53.5 41

v maxX 60 62 Exercise 11B — Single event probability


1 S = {Heads, Tails}, 1
vi Mean 45.45 32.35 2 a S = {1, 2, 3, 4, 5, 6}, 1
b S = {2, 3, 4, 5, 6, 7, 8, 9, 10, 11, 12}, 3
vii IQR 13 19.5 c S = {a, b, c, d, e, . . ., y, z}, 5
d S = {Sun, Mon, Tue, Wed, Thu, Fri, Sat}, 2
viii Standard deviation 11.20 12.04 e S = {Jan, Feb, Mar, . . ., Nov, Dec}, 3
3 a 26, 52 b 1, 15 c 1, 44
c Overall, it appears that people who attended the d 5, 1500 e 3, 11
Symphony concert were older than those who
4 1--2-
attended the jazz concert. The spread of ages is
nearly the same (slightly higher for the jazz 5 a 1
--- b 1
--- c 1
---
6 6 2
audience).
1 2 1
d Back-to-back stem plots can be used only for data d ---
2
e ---
3
f ---
3
with two categories. Since there are three events, 1 1 22
parallel boxplots should be used. 6 a ------
45
b ------
45
c ------
45
e minX = 12, Q1 = 34, median = 47.5, Q3 = 56.5, d 23
------ e 1
--- f 1
---
45 5 3
maxX = 68, mean = 44.95, IQR = 22.5, standard
19 2 2
deviation = 15.55 g ------
45
g ---
9
i ------
15
f 1 1 1
Opera 7 a ------
52
b ------
13
c ---
4
Jazz 1 3 3
d ---
2
e ------
13
f ------
13
Symphony
1 4 7
8 a ------
12
b ------
12
c ------
12
15 20 25 30 35 40 45 50 55 60 65 70
1 1 1
Age 9 a ---
4
b ---
4
c ---
4
g Overall, the people who went to the Symphony d 3
--- e 1
--- f 3
---
concert and to the opera were of similar ages 4 2 4

and older than those who went to the jazz 10


10 ------
12
concert. The ages of people who went to the
1 3 1
opera are the most spread out, while the ages of 11 a ---
4
b ---
4
c ---
4
people who attended the Symphony concert are 1 2
d --- e ---
the least spread out. 4 4
773

answers
Answers

12 C 13 D 14 C 15 E 9 1 2 3 4 5 6
5 4
16 a ------------
1000
b ---------
999 0.180 0.233 0.074 0.246 0.092 0.175
3384 6768 10 152
17 a -------------------
160 000
b -------------------
160 000
c -------------------
160 000 There is a huge bias for 2s and 4s, against 3s and 5s,
1 1 4 1s and 6s OK.
18 a --- b --- c ---
5 5 5 10 Bread shop 3, proportions were 0.75, 0.84 and 0.86
d 1
--- e 4
--- f 1
--- respectively. However, Bread shop 1 wasted only
5 5 5
22 loaves, versus 32 for Bread shop 2 and 42 for
19 Check with your teacher. Bread shop 3. This could reduce Bread shop 3’s true
20 a Because there are two numbers which could go in effectiveness.
the last place. 11 a, b Answers will vary.
b A number greater than 400
12 a Left-handers caught 6 out of 14 (0.43),
Exercise 11C — Relative frequency right-handers caught 24 out of 56 (0.43)
1 0.74 2 0.79 3 0.375 — no difference.
4 a 0.45 b 0.55 b Caught 30 out of 70, bowled 30 out of 70 — no
5 4% difference.
6 a 0.03 b 0.97 c Left-handers — not out 4 out of 14 (0.29),
7 a 0.96 b 0.04 right-handers — not out 6 out of 56 (0.11) —
8 A 9 E more likely to be not out against left-handers.
10 a 0.525 b 0.4375 c 0.0375 d The relatively small number of left-hander
11 a 6.67% b 80 observations means comparisons are not very
12 a 0.02 b $400 accurate. However, there seems little
13 Yes, the relative frequency is 27%. difference between left- and right-hander
14 a 2.5% b 51.5% c 17.5% effectiveness.
15 40 000 km 13 1329
16 a Result Number
Win 15 Chapter review
Loss 14 1 Marcia will probably get a higher card.
Draw 11
2 a Probable b Impossible c Even chance
b Win = 0.375, Loss = 0.35, Draw = 0.275 d Unlikely e Unlikely
3 Check with your teacher.
Exercise 11D — Modelling probability 4 Hot weather
1 Answers will vary. 5 Rolling a die and getting a number greater than 1
2 Answers will vary. Selecting a picture card from a standard deck
3 Answers will vary. Selecting a blue marble from a bag containing
4 a One way is to use randInt(0,1,10) to generate 14 blue, 15 red and 21 green marbles
10 values that are either equal to 0 or 1, and let 0s Winning the lottery with 1 ticket out of 100 000
represent Heads, and 1s represent Tails. tickets sold
b Answers will vary. 6 Mark is most likely to win based on past
c Answers will vary. performances.
5 Generally, the histogram for 100 tosses will be more 7 S = {1, 2, 3, 4, 5}, E = {3, 4, 5}
even than that for 10 tosses. 1
8 a -----
-
1
b -----
-
13
c -----
-
25 25 25
6 a, b The player can expect to win about once every
1 9 16
15 games, spending $15 to win $10 (a loss of $5). d ---
5
e ------
25
f ------
25
7 Answers will vary. 1 1 1
8 Answers will vary. 9 a ------
52
b ------
13
c ---
4
1 3 10
d --- e ------ f ------
Exercise 11E — Long-run proportion 2 13 13
7 1 3
1 0.1 10 a ------
20
b ------
10
c ---
4
2 0.499 1 3 1
3 a, b Note the steady improvement from about 0.25 11 a ------
24
b ---
4
c ---
4
(25% success) to about 0.36 (36%) success. d 3
--- e 1
---
4 2
Sales 7 11 17 25 41 53 60 72 84 97
1 4
Houses 28 47 68 93 135 164 186 217 244 270 12 a ---------
400
b ------------
1999
Proportion 0.25 0.234 0.25 0.269 0.304 0.323 0.323 0.332 0.344 0.359 13 0.02
4 D 5 D 14 a 0.15 b $750
6 0.708 This result supports the suspicion that the coin 15 Answers will vary.
is biased because the expected result would be 0.5. 16 0.2
7 0.78 8 Yes 17 0.75 11A

11E
answers 774 Answers

Modelling and problem solving Exercise 12B — Variable rates


1 All elements in the sample space are not equally 1 a, c, d, e, g, h 2 b, d, e, f
likely. There are a number of factors to consider. 3 a Constant or Positive,
Check with your teacher. variable negative or
2 They can expect to win an average 50 c/game. Interval rate? zero rate?
OP Constant Positive

CHAPTER 12 Rates of change PQ Variable Positive

QR Constant Zero
Exercise 12A — Constant rates
1 c, d, g, h, j RS Variable Negative
2 b, d, e, g, h, j
3 i b, e, h ii g, j ST Variable Positive
iii d TU Variable Negative
4 D
5 C UV Constant Negative
6 a i 50 ii 50 m/h
VW Variable Negative
iii d = 50t
b i 7 ii 7 L/min b B c D d A e C f D
iii v = 7t + 50 4 a W (kg) b Form a straight line
c i −1 ii −1 kg/week 12
iii w = 100 − t 9
6
d i 0 ii 0 m/yr
3
iii h = 75
e i 0.75 ii 0.75 g/min 0 1 2 3 4 t (min)
iii w = 0.75t + 10 c A constant d 3 kg/min
f i −0.25 ii −0.25 g/L
5 a W (kg) b No
iii w = −0.25v + 35
7 a D 20
16
b B
12
8 a $24/h 8
b Rest 4
c $18/h
0 1 2 3 4 t (min)
d The picker is tiring a little or fruit is scarcer.
e $156 c Variable d i 2 kg/min
9 a 1000 m b 70 s ii 3 kg/min iii 7 kg/min
6 a Constant b Variable
c 80 s d 30 s
c Constant d Variable
e d (metres)
e Variable f Variable
1000

820
420
Exercise 12C — Average rates of
change
2
0 1 a --- b −4
70 150 180 t (s) 5

10 a w (kg) b 0.2 kg/cm 2 a, c


5 3 a ii 30 iii 30 km/h
4 b ii 10 iii 10 km/h
3 c ii 15 iii 15 km/h
2 d ii −5 iii −5 km/h
1 e ii 0 iii 0 km/h
0 5 10 15 20 25 x (cm) f ii −10 iii −10 km/h
c W = 0.2x 4 a 2 b 2°C/min
c 2 °C/min because the rate of change is constant.
11 a 6 L/h b 6 2--3- h 5 a 1.5 b 0.5
12 a iv b i c i 1.5 kg/m ii 0.5 kg/m
c ii d iii d No, because it is a variable rate of change.
13 Check with your teacher. 6 D 7 C
775

answers
Answers

8 a, b 5 a, b
N y
600 9
500
400
300
5
200
100
0
0 10 am 11 am 12 pm 1 pm 2 pm 3 pm 4 pm 5 pm T
c i 170 ii 20 iii 0 –4 –3 –2 –1 0 1 2 3 4 x
–2
d i 170 people/h ii 20 people/h
iii 0 people/h
e Most people arrive in the morning, few in the –7
middle of the day and nobody later in the
afternoon. c i 4 ii 0 iii −4
9 a d b i 5 km/h 6 a A b I
9 c A d I
8
7 e A f A
6
5 g A h I
4
3 i I j I
2
1 7 6 cm/min
0
0 1 2 3 4 t 8 a 85 kg after 35 weeks
ii 3 km/h iii 0 km/h b approx. −1 kg/week
c A rest or meal break d The first half hour c Approx. 4.4 kg/week
e 1 to 1.5 hours, 3 to 3.5 hours 9 a Approx. 6 hours
10 a iii 110 m/min iii 70 m/min b Approx. 2.8 °C/hour
iii 45 m/min iv 15 m/min
v 5 m/min b Decreasing
11 a 20 °C Exercise 12E — Motion graphs
b, c d 10 1 a C b B
c D d A
T (°C)
2 a False b True
120
c True d False
(8, 84) 3 a 40 m
b −2 m (or 2 m below the platform)
c 0.5 m/s
20 (2, 24)
d −0.025 m/s (or 0.025 m/s downwards)
0 2 4 6 8 10 t (s) e 10°C/s 4 a x=1 b x = −3
12 a 200 g b i 1 g/week c Right d t=2
ii −2 g/week iii 10 g/week e C f D
iv 28 g/week g D h D
120
13 72 km 14 --------- min
7 i C j B
Exercise 12D — Instantaneous rates 5 a i x=0 ii Right
1 a, c, d, h iii t = 2, x = 8 iv t = 5, x = −3
2 a 1 b 2.5
b i x=4 ii Right
c −1 d 0.25
iii t = 4, x = 12 iv t = 6, x = 10
3 a 2 g/min b 16 g/min
c i x=0 ii Right
c −25 g/min d 12 g/min
4 a, b c C iii t = 3, x = 12 and t = 6, x = 3
y iv t = 8, x = 10
d i x=0 ii Left
16
iii t = 1, x = −5 iv t = 3, x = 18
12
e i x = −3 ii Left
8
iii t = 1 1--2- , x = −6 iv t = 5, x = 5
4
f i x=2 ii Left
0 1 2 3 4 x
iii t = 3, x = −5 and t = 5, x = 5
d C e A iv t = 6, x = 4 12A

12E
answers 776 Answers

6 a x i x=0 f x i x = −12
10 18
8
6
4
2
0 1 2 3 4 5 t
0 1 2 3 4 5 t

ii Right –12
iii No
ii Right iii No
iv x = 10
iv x = 18
b x i x = −2 7 a x b i 4
16
4

0 1 2 3 4 5 t

–5
0 6 t
–2 ii 2 iii 0
ii Right iv −2 v −4
c i 4 m/s ii 2 m/s
iii No
iii 0 m/s iv −2 m/s
iv x = 16
v −4 m/s d x
c x i x=0
15 4
2
0 1 2 3 4 5 t
–2
–4
–6
8 a i Positive ii Speeding up
0 1 2 3 4 5 t b i Negative ii Slowing down
–1 c i Negative ii Speeding up
ii Left d i Positive ii Slowing down
iii Yes, t = 1, x = −1 e i Positive ii Neither
iv x = 15 9 a C b E
d i x=0 c B d F
x
e A f D
1
10 x (m) a 45 m
0 1 2 3 4 5 t 45
25

0 1 2 3 4 5 t (s)

–8 b t=5 c t=2 d 20 m/s


11 a Alan b Bill
ii Right
iii Yes, t = 1, x = 1 c 12 metres d 2 m/s
iv x = −8 e 40 s f Twice
e x i x=4
Exercise 12F — Relating the gradient
9 function to the original function
1 a y b y
4 12 12
8 8 Gradient
function
0 t 4 4
1 2 3 4 5
0 x 0 x
ii Left –4 –2 2 4 –4 –2 2 4
–4 –4
iii Yes, t = 2, x = 0
Gradient function
iv x = 9
777

answers
Answers

c y d y 4 a x
b x
12 8 12
8 Gradient 6 0 t
4
4 function 4
Gradient
2 function –8
0 x
–4 –2 2 4 t
–4 0 x 0 4
–2 –1 1 2
c x d x

e f y
5
4
3
Gradient
y
function 2 0 1 2 3 4 t
1
0 1 2 3 4 5 t
1

0 x –3 –2 –1–1 1 2 3x e f
2 4 6 8 x x
–1 –2
–3
–4
–5 t
0 t 0 1 2 3 4
2 It is a straight line. 1 2 3 4
3 Quadratic functions.
4 The gradient function of sin x is cos x, the gradient 5 a
function of 2x is 0.7 × 2x. t 0 1 2 3 4
5 There are stationary (turning) points there.
6 V −4 −2 0 2 4
Gradient of x–t graph −4 −2 0 2 4
2
b x
2 4 6 8 t (s)

0 1 2 3 4 t

Exercise 12G — Relating


velocity–time graphs to position–time
graphs 6 a x b x
1 a v b v
3 2

t t 0 2 4 t
0 0 4
0 3 6 t 1 2

–2
c x d x
–3
0 t
4
2 C

3 a v b v
5 8 t
0 4
4
7 A
0 1 2 3 4 t
–4 8 a It travels at a constant velocity of 20 m/s.
0 1 2 3 t b It starts at rest, increasing its velocity at a
–8
constant rate for 15 seconds when it reaches
c v d v 18 m/s. It then maintains this velocity.
2 12 c It starts from rest, increasing its velocity at a
constant rate for 10 seconds when it reaches
25 m/s. It maintains this velocity for a further
0 1 2 t
3 15 seconds then decreases its velocity at a
constant rate for 5 seconds bringing it back to
–2
0 1 2 3 4 t
rest. 12F

12G
answers 778 Answers

9 a v (m/s) b v (m/s) 8 a 19.6 m/s b 29.4 m/s


35 9 a 16 °C/min b 700 °C/min
30
10 a 3.75 b 2.81
0
c 0 d 0
10 20 t (s) 0 15 20 t (s)

10 a i 15 m/s ii 45 m/s Chapter review


b 60 m/s c 34 seconds 1 C 2 D
3
3 a 30 m /h
b 28 hours
Exercise 12H — Rates of change of 4 B 5 B
polynomials 6 a M b Variable, as the graph
1 a y is not a straight line.
i 23
16 (4, 16)
ii
iii
12 9 Tangent
iv at t = 2
(3, 9)
8 t
0 1 2 3 4 5
(2.5, 6.25)

4 (2, 4) (2.1, 4.41) –9


(1, 1) c Approx. 8
0 x
1 2 3 4 7 A
b i 6 ii 5 iii 4.5 iv 4.1 8 E
c 4
9 a 1 kg/h
2 a y i
8 (2, 8) b 5 kg/h
7 10 C 11 B
ii
6 12 a D b C c B
5 13 a x b x=5m
4 20
(1.5, 3.375)
3 iii 15
iv
2 (1.1, 1.331) 10
x (t)
1 (1, 1) (1.01, 1.030301) 5
4
0 1 2 x 0 1 2 3 4 5 t

b i 7 ii 4.75 iii 3.31 iv 3.0301 c t = 1 s and x = 4 m d 14 m


c 3 14 a x = 30 m b 3s
3 a c 34 m/s d −50 m/s
e 370 m f 30.83 m/s
x 3 2 1.5 1.1 1.01 1
15 D 16 A 17 C
y 7 3 1.75 1.11 1.0101 1 18 x

b i 3 ii 2
0 3 6 t
iii 1.5 iv 1.1
v 1.01 c 1
4 C 5 E 6 B 19 E 20 D
7a 2 b 12 21 a i 4 m/s ii 0 m/s
c 6 d 4 iii –8 m/s
e −7 f 4.75 b The height of the golf ball increases during first
g 31 h −2.5 6 seconds then decreases after that.
779

answers
Answers

Modelling and problem solving 10 D


1 a 11 A
600
12 a 3 1--2- b 0 c 7 d 16
540
480
420
Number of cans

360 Exercise 13B — Limits of discontinuous,


300 rational and hybrid functions
240 1 b, c, d, f
180 2 b 4 c 0 d −1 f 3
120 3 a 4 b y
60
0
8.00 am 10.00 am 12.00 pm 2.00 pm 4.00 pm 6.00 pm
f (x) = x discontinuous at 4
Time
0 4 x
b 240
c Approx. 3 pm 4 a Undefined because you cannot divide by 0.
d Approx. 90 cans/hour b x=0 c x(x + 1) d f (x) = x + 1
2 a i 0 mg/L ii 1 mg/L iii 0.8 mg/L v 0.471 mg/L
e y
f 1
b f (x)
1
0.8 1
x (mg/L)
0.6 0 x
–1
0.4

0 5 a f (x) = x + 3, x ≠ 0 b f (x) = 6, x ≠ 3
1 2 3 4 t (hours)
c f (x) = x − 5, x ≠ 0 d f (x) = x + 1, x ≠ −4
c 2 3 hours
d −0.2 mg/L; it is negative because the concentration e f (x) = x − 1, x ≠ 6 f f (x) = x 2 − 2x + 4, x ≠ −2
is decreasing. g f (x) = x + 4, x ≠ 1 h f (x) = x 2 + 3x + 9, x ≠ 3
e 0.96 mg/L at t = 0.5 hours, −0.24 mg/L at 6 a 3 b 6 c −5 d −3
t = 2 hours e 5 f 12 g 5 h 27
f The maximum concentration is 1 mg/L after
1 hour. 7 a y b y
5 g(x)
CHAPTER 13 Differentiation and 4
3 f (x)
5

applications
Exercise 13A — Introduction to limits 0 2 x
–3 0 x
1 8 –1 1
–2
2 Circle
c y d y
3 B h(x)
1 p(x)
4 a 0 b lim --- = 0 1
n→∞ n
3
0 x 2
5 a n 1 2 3 4 5 6 10 –1 1
0 x
S 1 1 --1- 1 --3- 1 --7- 15
1 -----
-
31
1 -----
-
511
1 --------
- –2 1
2 4 8 16 32 512

b C 5 4 Does not exist


8 a i ii iii
6 x 2.95 2.99 2.995 3 3.005 3.01 3.05 b i 5 ii −2 iii Does not exist
c i 0 ii 0 iii 0
f(x) 7.95 7.99 7.995 8 8.005 8.01 8.05 d i 2 ii 3 iii Does not exist
7 C 9 a Does not exist b −2
8 a 14 b −3 c 0 c Does not exist d Does not exist
d 5 e 4 f 15 e 1 f 0
9 a 9 b 2 c −11 10 a 12 b 7 c 4
d 50 e 2 f −3 d −11 e −2 f Does not exist
g 40 h −12 i −2 g 0 h −3 i 6
j 19 k 0 l −27 j −7 k 5 l 2 12H

13B
answers 780 Answers

Exercise 13C — Differentiation using c i x = 2 1--2- ii x = 6 iii x = 1


first principles 11 a x = −3, x = 1--2-
1 a 5 b 2x + 10 c 2x − 8 d 3x 2 + 2 1
b At x = −3 gradient = −7, at x = --- gradient = 7
2 a 2x b 2x − 3 c 8x 2
d −2x e 6 − 4x f 3x 2 + 5 c (−1 1--4- , −6 1--8- )
3 a 2x − 6 b x=3 12 a (0, 0) and (2, −4) b (1, −2)
4 a 3x2 b x = −2 or 2 13 a y = 6x − 2 b x + 6y = 25
5 a 7 b 10 c 3 d 27
14 y = --12- x + 5 or 2y = x + 10
6 C, E 7 C 8 A
15 a x = 2 b y = – 1--- x + 7
---
Exercise 13D — Finding derivatives 16 y = –x + 5
2 2

by rule
dy dy Exercise 13E — Rates of change
1 a ------ = 6x 5 b ------ = 14x 1 a 13 b f ′(x) = 2x + 5 c f ′(5) = 15
dx dx
2 a i V = 0 cm3 ii V = 800 cm3
dy b 80 cm3/s
c 5 d ------ = 16x 3 + 4--3- x – 5
dx c i 0 cm3/s ii 120 cm3/s iii 0 cm3/s
2 a 12x3 b 56x6 c 25x4 3 E 4 C 5 C
5 2
d −24x e −6x f −7 6 a h′(t) = 18 − 6t
3 a D b B c E d F b i 6 m/s ii 0 m/s iii –6 m/s
e G f A g C c The ball stops rising, that is, it reaches its highest
dy dy point.
4 a ------ = 6x 5 + 6x b ------ = 20x 3 – 21x 2 + 6 d 12 m/s
dx dx
dy dx
c ------ = 11x 10 – 18x 5 + 20x 4 + 6x 7 a ------ = –4t + 40 b i 20 m/s ii 4 m/s iii –4 m/s
dx dt
dy c The lift changed direction.
d ------ = 50x 4 – 12x 3 + 6x 2 – 8 d t = 10 s and x = 200 m
dx
dy dy 8 a i 4000 ii 15 000
e ------ = 0 f ------ = 32x 3 b 5500 people per hour
dx dx
5 a --8- x 3 c i 3500 people/hour ii 4500 people/hour
b − --54- x
3 iii 6500 people/hour iv 7500 people/hour
c 6x6 d 9
--- x2 + x − 3 d More people arrive closer to starting time.
4

e 2x4 + 3x 3 + 1--2- x 2 f 12x2 − 8--7- x − 5 dW


---
6 9 a 80 kg b -------- = 12 – 0.6t
dt
6 a 2x + 3 b 12x − 15
c i 9 kg/week ii 6 kg/week iii 3 kg/week
c 2x + 8 d –432 + 162x
d Decreasing e 20 weeks
e 3x 2 + 12x + 12 f 24x 2 − 120x + 150 1
dP ---
7 a 2x, x ≠ 0 b 4, x ≠ 0 10 a ------- = 4.5 – 1.5n 2
dn
c 6x + 2, x ≠ 0 d 10x + 1, x ≠ 0
b i $37.50 ii −$9.38 iii −$12.00 c n=9
8 a −4x−5 b −7x−8 c −12x−5 11 a i −20.2 cm3/s ii −21 cm3/s iii −22 cm3/s
d −40x−9 e 24x−7 f 15x−6 b No, because the volume is always decreasing.
9 15 dA
4
g – ----- h – ------- i – ------ 12 a A = π r 2 b ------- = 2 π r
x5 x 10 x4 dr
c i 20π m2/m ii 100π m2/m iii 200π m2/m
60 – 1--- – 1---
j – ------ k x 2 l 2
--- x 3 dA
x7 3 d Yes, because ------- is increasing.
dr
3 – 3--- 1
m x – --4- n 6
---
5
x 5 o ---------- 13 4
a V = --3- π r 3 dV
b ------- = 4 π r 2
2 x dr
1
p – ------------ 2 – --2-
c iii 0.04π m3/m or 0.13 m3/m
q ------- r 1
--- x 3
iii 0.16π m3/m or 0.50 m3/m
2 x3 x 3
iii 0.36π m3/m or 1.13 m3/m
– 4---
s − 2--3- x 3 14 a Length = 2h, width = 2h

9 a i 13 ii −17 iii 3 b V = 4h3


b i 5 ii −4 iii 0 c i 12 m3/m ii 48 m3/m iii 108 m3/m
c i 4 ii −14 iii −2
15 a x = 2h b V = 6 3h 2
d i 10 ii 19 iii 7
10 a x = 2, x = 3 dV dV
c i ------- = 6 3 ii ------- = 12 3
b At x = 2 gradient = −1, at x = 3 gradient = 1 dh dh
781

answers
Answers

dy 10 a f (x) b No limit exists.


16 a ------ = −0.000 06x2 + 0.012x 2
dx
b i 0.384 ii 0.6 iii 0.384 iv 0.216 1
c x = 50 and x = 150 d 12.5 < y < 67.5
0 x
dA –1
2
17 a ------- = 180t – 9t hectares/hour
dt 11 A 12 B 13 D 14 2 15 4 − 6x
b i 0 ii 576 iii 864 iv 900 v 864 vi 576 16 E 17 D 18 B 19 A 20 C
vii 0 (all hectares/hour) 21 A 22 E 23 B
c The fire spreads at an increasing rate in the first 24 a g′(x) = x2 − 7x − 8
10 hours, then at a decreasing rate in the next b i −20 ii 10
10 hours. c (−1, 5 --16- ) and (8, −116 --13- )
d The fire is spreading, the area burnt out by a fire
does not decrease. 25 a ( 5--4- , − 9--8- ) b ( 1--2- , 0) c (1, −1) and (5, 27)
e The fire stops spreading; that is, the fire is put out
or contained to the area already burnt. 26 E 27 D
f t = 6 and t = 14 hours. dx
28 a 3 cm b v = ------ = −4t + 8 cm/s
dt
dx
c v = ------ = −8 cm/s d t = 2 s and x = 11 cm
Exercise 13F — Solving maximum and dt
minimum problems e Left f 10 cm
1 a x = 20 m, y′(19) > 0 and y′(21) < 0 (a maximum) 29 A
b y = 11.2 m 30 a 2 litres/hour b 3 hours
2 a t = 10 min 31 a 400 cm2
b V ′(5) = −6 and V ′(15) = 18 (a minimum) b Yes, a circle of circumference 80 cm has an area
c V = 160 litres of approx. 509.3 cm2
d t = 15 min
3 a h = 12.25 m (when t = 1.5 s) Modelling and problem solving
b h′(1) = 5 and h′(2) = −5 (a maximum) 1 2000 π cubic units
4 32 m2 2 a Gradient is 0.16 at the beginning and −0.38 at the
5 a 16 − x b A = x(16 − x) end.
c Both numbers are 8. b (2, 2.04)
d P ′(7) = 2, P ′(9) = −2, (a maximum) d Maximum height is 2.04 km.
6 a 10 − x b A = x(10 − x) c x=5 e (0, 1.8), minimum height is 1.8 km.
d Length and width = 5 cm e 25 cm2 3 350 cm2
7 a Length and width = 15 m b 225 m 4 y
2
8 a P = 60n − 250 − 1.2n b 25 c 500
6
9 a P = 800 + 240n − 20n2 b n = 6, p = 1520
10 Both numbers are 5.
11 a x ∈(0, 6) or 0 < x < 6
0 x
b i x ii 12 − 2x iii 12 − 2x 3
c V = x(12 − 2x)(12 − 2x) d 128 cm3
g(x)
12 a 7.36 cm by 25.28 cm by 35.28 cm
b 6564.23 cm3 5 a 10 m b 0.5 m
c f (x)
13 a h = 256
--------- 1024
b A = l 2 + ------------
l2 l 10
c 8 cm × 8 cm × 4 cm d 192 cm2
14 10 cm × 10 cm × 10 cm
1 440 000
15 a 7500 b C = ----------------------- + 9v
v
c v = 400 km/h and C = $7200 2
(5, 0.5)
0.5
Chapter review 4 5 x
1 E 2 A 3 −6 4 3 5 B d 4 5 m
e For x < 4, f ´(x) = −2
6 A 7 B 8 C For x = 4, f ´(x) = x − 6
9 a f (x) = x + 4, x ≠ −3 b 1 For x = 4, f ´(4) = 4 − 6 = −2 13C

13F
Index 783

Index
absolute value function 142–6, 168 data
addition of ordinates 147–53, 168 categorical 427
amplitude 233–4 collection of 431–8, 473
angles cumulative 464–72, 474
depression, of 199–200 displaying 445–58
elevation, of 199–200 quantitative 427
finding 192–8 types of 426–7, 473
sizes, finding, using cosine rule 220–4 decay functions 362, 370–3, 418–19
sizes, finding, using sine rule 212–14 exponential decay 371
annuities growth factor 372
spreadsheets calculating future value of 401 radioactive decay 372
annuities formula 402–4, 419 straight line decay 370
degrees mode in graphics calculator 177–8
average rates of change 578–83, 615
dependent variable 62
depression, angles of 199–200
back-to-back stem plots 515–16, 525
derivatives, finding by rule 641–6
balance loans differentiation
reducing 389 derivatives by rule 641–6, 662
bar charts 454–5, 474 first principles, using 636–7, 662
segmented 454–5 maximum and minimum problems, solving 656–62
bearings 200–2 rates of change 649–55, 662
bias 439–44, 473 dilation 113
questionnaire design, in 439 Dirichlet’s function 634
sampling 439–40 discontinuous functions, limits 630
statistical interpretation 440 discrete variable 62
boxplots 507–15, 525 displaying data 445–58
interpreting 508–9 bar charts 454–5, 474
parallel 520–3, 525 frequency histograms 451–3, 474
stem plots 445–50, 473
career profiles 23, 614 distributions
categorical data 427 outliers 459
census 431 skewed 459
chance 534–9, 565 symmetric 459
circles 93–8, 103 division (indices) 300
general equation of 94 domain and range 69–76, 102
collecting data 431–8, 473
census 431 elevation, angles of 199–200
identifying target population 431 equations
method 431 formula for finding equation of straight line 41–6, 52
sample 431 general equation of circle 94
indicial 311–16, 341
sampling methods 432–6
linear see linear equations
compound interest formula 379–88, 419
logarithmic see logarithmic equations
compounding factor 366
simultaneous 33–41, 52
constant rates 570–4, 615 trigonometric see trigonometric equations
continuous functions, limits 626 y = mx + c 24, 52
continuous variable 62 exact values 241–8, 272
cosine function, application of 273 exponential decay 371
cosine ratio 181–3 exponential functions
cosine rule 225 applications 335–40
using to find angle sizes 220–4 graphs of 317–4, 341
using to find side lengths 215–19 horizontal translations 317
cubic function, graphing 119 reflections 317
sketching using intercepts 127–8 vertical translations 318
cumulative data 464–72, 474 exponential growth 366
cumulative frequency 464 extreme values, identification of 509
784 Index

finding angles 192–8 graphing derivative 647–8


finding unknown side 185–91 graphing hybrid function 86–7
five number summary 507 graphing linear functions 24–6
frequency histograms 451–3, 474 graphing relations with restricted domain 73–4
functions 79 listing terms of geometric sequence 351–2
complementary 289 modelling using graphics calculator 161–6
defining and using 83 motion 590–8, 616
Dirichlet’s 634 plotting points 65–6
evaluating 82 random number generation 552–4, 565
exponential see exponential functions relating gradient function to original
function notation 82–3 function 599–602
hybrid 85 simplifying expressions 304
inverse relations and functions 90–2, 103 solutions in degrees and minutes 194–5
mixed 85 solving linear equations 5–6, 52
modelling and 99–101, 103 solving simple trigonometric equation 280
notation 103 solving simultaneous equations 34–6
periodic see periodic functions substituting for pronumerals 11–12
piecewise defined 85 velocity–time graphs, relating to position–time
types of, special 84–9, 103 graphs 604–8, 616
x- and y-intercepts, finding 32
Gallup poll 431 graphs
geometric sequences 348–54, 418 derivatives, of 647
applications of 418 dilation 113
geometric series 354–5, 418 exponential functions, of see exponential functions
applications of 418 hyperbola see hyperbola
gradient function linear, using calculator 24–6
deriving 624–5 logarithmic 332–4, 342
relating to original function 598–603 misleading, creating 441–2
gradient of straight line 15–22, 52 reflection 113
angle made by line with x-axis 16–17 relations and 62–9, 102
graphics calculator tips sketching graphs using intercepts 123–33
addition of ordinates 150–1 sketching graphs using transformations 117
algebraic solution to simultaneous equations 38–40 sketching linear graphs using intercepts 29–31, 52
calculating mean of data in frequency table 485 transforming 113–17, 167–8
calculating mean of data in list 482–3 translation 114
calculating median and mode 488 trigonometric 254–67, 273
calculating trigonometric ratios 179–80 y = cos x 258–9
choosing random sample 432–4 y = sin x 255–7
constructing a histogram 452–3 growth functions 362–70, 418–19
cosine rule problems 216–17 exponential growth 366
creating boxplot from frequency table 511 growth factor 370
defining and using functions 83 straight line growth 362
degrees mode 177–8
derivative, finding 643 histograms, describing shape of 459, 474
displaying answer in degrees, minutes and hybrid functions 85
seconds 183 graphing 86–7
equation of line through two given points, limits 633–4
finding 43–5 hyperbola 133–7
equation solver, using 186–7
financial calculations using TVM 405–8 identities 288–94, 296 see also Pythagorean identity
finding angles using equation solver 192–5 independent variable 62
finding derivative using first principles 638 index laws 300–4, 341
finding limit of rational function 631–2 indicial equations 311–16, 341
finding sum of geometric sequence 356–8 instantaneous rates 584–90, 615
finding summary statistics 496–7 interest only loans 389
graph of y = sin x 256–7 interquartile range 494–501, 524
graphing absolute value function 144–5 interval notation 70
graphing circle 94–6 inverse relations and functions 90–2, 103
Index 785
limits 624–30 radian measure 238–41, 272
continuous functions, of 626 symmetry 249–54, 272–3
discontinuous functions, of 630 trigonometric 254–67
hybrid functions, of 633–4 piecewise defined function 85
rational functions, of 631–3 plotting points 65–6
theorems on 627–8 poll
linear equations 2–15 Gallup 431
graphics calculator, using 5–6 method of collecting data 431
pronumerals, more than one 8–15 polynomials, rates of change 609–14, 616
rearrangements and substitution 8–15, 52 probability 534
solving 3–7, 52 chance 534–9, 565
linear functions long-run proportion 558–64, 565
graphing 24–6 modelling 551–8
linear graphs 24–31 relative frequency 546–51, 565
linear modelling 47–52 single event 539–46, 565
loans products (indices) 301
reducing balance loans 389 Pythagorean identity 289
schedules 390–400, 419 using 294–5, 296
spreadsheets for constructing schedules 400
logarithmic equations 329–34, 342 quantitative data 427
logarithmic functions quartic function, graphing 120
applications of 335–40 sketching using intercepts 129–31
logarithmic graphs 332–4, 342 questionnaire 431
logarithms 324–9, 342 bias in design 439
base 10, to the 329–32 quotients (indices) 301
laws 325
long-run proportion 558–64, 565
radian measure 238, 272
equations using 283–6, 296
many-to-many relations 78
radioactive decay 372
circles 93–8, 103
raising to a power (indices) 300
many-to-one relations 77
zero, of 301
mean 480, 481–2
random sample 432–4
measures of central tendency 481–93, 524
range 494–501, 524
median 480, 486
rates of change 649–55
mode 480, 487
modelling 153–60, 169 average 578–83, 615
functions and 99–101, 103 constant rates 570–4, 615
graphics calculator, using 161–6 identifying 615
linear 47–52 instantaneous rates 584–90, 615
mathematical, use of 60–1, 153 investigating 611
probability 551–8 polynomials, of 609–14, 616
motion graphs 590–8, 616 variable rates 575–8, 615
multiplication (indices) 300 rational functions, limits 631–3
rational powers 308, 341
negative powers 307, 341 reflection 113
relations
ogives 464–5 circles 93–8, 103
one-to-many relations 77 definition 62
one-to-one relations 77, 84 domain and range 69–76, 102
ordinates, addition of 147–53, 168 graphing relations with restricted domain 73–4
outliers 459 graphs and 62–9, 102
inverse relations and functions 90–2, 103
parabola, graphing 118–19 types of 77–81, 103
intercepts, use of 123–6 relative frequency 546–51, 565
parallel boxplots 520–3, 525 applying 551
percentiles 465 researching 551
periodic functions 232 right-angled triangles
applications 268, 273 applications of 199–204, 225
definition 232, 272 steps to find angle of 225
exact values 241–8, 272 steps to find side of 225
period and amplitude of 232–8 trigonometry formulas for 225
786 Index

sample 431 tangent ratio 176–7


predicting mean and standard deviation of population tangents 637
from 502–3 normals and (to curves) 644
random 432 transformations 113–17
stratified 434–5 graphing cubic function using 119
systemic sample 435–6 graphing parabola using 118–19
sampling bias 439–40 graphing quartic function using 120
sampling methods 432 investigating 118–23
secants 637 sketching graphs using 117
segmented bar charts 454–5 translation 114
sets and set notation 63, 102 triangles
simultaneous equations 33–41, 52 angles, finding 192–8
algebraic solution 36–40 cosine rule 215–24
definition 34 finding unknown side 185–91
graphical solution 34–6 right-angled triangles, applications of 199–204, 225
sine function, application of 273 sine rule 205–14
sine ratio 180–1 trigonometric equations 278–83, 286–8, 296
sine rule 225 identities 288–94
derivation of 205–6 radians, using 283–6, 296
using to find angle sizes 212–14 trigonometric graphs 254–67, 273
using to find side lengths 205–11 trigonometric ratios 176
single event probability 539–46, 565 cosine ratio 181–3
skewed distributions 459 finding angles 192–8
spreadsheets finding unknown side 185–6
calculating future value of annuity 401 formulas for right-angled triangles 225
loan schedules using 400 sine ratio 180–1
misleading graphs, creating 441–2 tangent ratio 176–7
square root function 138–41
standard deviation 502–7, 524 unknown side, finding 185–91
statistical interpretation bias 440
statistics variable rates 575–8, 615
bias, introduction of 439–44 variables
definition 426 applications of trigonometric functions 268
stem-and-leaf plot 447–8 continuous 62
stem plots 445–50, 473 dependent 62
back-to-back 515–16, 525 discrete 62
describing shape of 459, 474 independent 62
straight line types of 426–7, 473
decay 370 velocity–time graphs, relating to position–time
formula for finding equation of 41–6, 52 graphs 604–8, 616
gradient of 15–22, 52 vertical line test 79
growth 362
stratified sample 434–5 winemaking, mathematics in 23
summary statistics 480
symmetric distributions 459 x- and y-intercepts 32
symmetry 249–54, 272–3
systemic sample 435–6 y = mx + c 24, 52

Das könnte Ihnen auch gefallen